You are on page 1of 2019

Introduction

NOTE: The intent of StatPearls review books is to identify knowledge deficits and assist you in the learning process. Review
books are not intended to be a source of the knowledge base of medicine. The authors or editors do not warrant the information is
complete or accurate. The reader is encouraged to verify each answer in several references.

All drug indications and dosages should be verified before administration.

Copyright 2022 StatPearls Publishing, LLC.

All rights reserved, including the right of reproduction, in whole or in part, in any form.

http://www.StatPearls.com

Congratulations! This StatPearls book will help improve your fund of knowledge. Our books are designed to help you identify
and resolve knowledge deficits. To achieve this goal the text is written in a multiple-choice format with answers and
explanations. Explanations contain additional information intended to reinforce your knowledge. Emphasis has been placed on
covering facts that are easily overlooked, quickly forgotten, and often show up in specialty reviews. For further information, all
explanations are electronically linked to http://www.StatPearls.com .

We suggest you note all questions answered incorrectly to allow you to rapidly review the content prior to your exam.

We appreciate your comments, suggestions, and criticisms. Some answers and explanations may be in variance with your own
knowledge. This is usually attributable to variability among sources; however, please make us aware of any potential errors you
find as we update our books often. We appreciate your input in regard to format, content, or presentation. Please contact us at
support@statpearls.com or better yet, click on the flag button after each question to give us specific input on any errors or
omissions. We look forward to hearing from you.

Good luck with learning your specialty!

StatPearls eBooks

StatPearls books were created exclusively for use on eBook readers. Because we don't publish our books in paper format, we
have the luxury of optimizing them for eBook use.

How are our books optimized for the viewing on an eBook Reader?

1. Efficient User Interface: Each multiple choice question is on a page by itself so you can see the question and four
multiple choice answers, but you can't see the correct answer until you flip the page. This would be a complete waste of
paper in an ordinary book, but it makes for an easy to use eBook. You don't have to cover up the correct answer with a
piece of paper. Most devices have a dedicated button for page turning so with the click of a button, you can see the correct
answer.
2. Research Links: After each explanation we include web links to search for more information on the key concepts for that
question. If you miss a question, this is when you probably want to do more research on the question concepts. This would
be onerous and require a lot of typing using a paper book, but is a single click on an eBook reader, PC, or Mac.
3. Flag a Question: Medical standards and drug standards change. Also, human errors can occur resulting in questions that
have issues. StatPearls strongly believes in Continuous Quality Improvement (CQI). Flagging a question gives you, the
medical professional, the ability to tell us if something is wrong with a question and we will correct it and get back to you
with an updated book (providing you give us a phone number or email address.) If you don't provide us with contact
information, you can check for a book update using the normal methods from the books store and you will soon find an
update with the question corrected by one of our health professionals.
4. Volume of Questions: When we make a book we publish every question we have on a topic (up to 2,000 per book). This
would be ridiculously expensive to do with traditional book publishing, but in the eBook world it costs no more to publish
2,000 questions than 1,000 questions. You will find that in most of our specialties, we dwarf the competition with breadth
of material. This makes our books "just what the doctor ordered" for those wishing to go the extra mile when studying.

STATPEARLS AUTHORS AND EDITORS

Over 6,000 physician, nursing, and allied health professionals authored a database of over 50,000 classified multiple-choice
questions and explanations. Each question has been peer reviewed by two health professionals and a pharmacist. All questions
are reviewed annually and updated through a peer review process.

WE APPRECIATE YOUR COMMENTS!

We appreciate comments, suggestions, corrections and additional contributing authors and editors. Please e-mail us at:
support@statpearls.com.
Critical Care Specialty Review and Self-
Assessment
StatPearls Book ID:
MD_CRITICAL_CARESPECIALTY_REVIEW_AND_SELF_ASSESSMENT-2021-12-20

Version: 12/20/2021
Editors in Chief
Alan Heffner, MD; Director of Critical Care; United States, Charlotte, NC, USA
Susan Murin, BA, MD, MSc, MBA; Professor; UC Davis School of Medicine; United
States, Sacramento, CA, USA
Christian Sandrock, MD, MPH; Professor; UC Davis School of Medicine; United States,
Sacramento, CA, USA
Grammar and Illustration Editor
Gerson Cordero Rubio
Table of Contents
Please Contribute

Learn More

Introduction

Editors in Chief

Contributing Editors

Authors

Section 1 ( Questions 1 - 100 )

Section 2 ( Questions 101 - 200 )

Section 3 ( Questions 201 - 300 )

Section 4 ( Questions 301 - 400 )

Section 5 ( Questions 401 - 500 )

Section 6 ( Questions 501 - 600 )

Section 7 ( Questions 601 - 700 )

Section 8 ( Questions 701 - 800 )

Section 9 ( Questions 801 - 900 )

Section 10 ( Questions 901 - 1000 )

About StatPearls
Section 1
Question 1: A patient presents with fever, chills, and extreme pain 48 hours after cutting his
hand. His hand is purplish in color with bullae, putrid discharge, and decreased sensation. In
addition to starting empiric antibiotics, what is the most appropriate management of this patient?

Choices:
1. CT scan of the hand
2. Serial examinations to assess for progression
3. Surgical exploration
4. Incision and drainage to send specimen for culture and Gram stain

Photo:Contributed by DermNetNZ
Answer: 3 - Surgical exploration
Explanations:
This is a case of necrotizing fasciitis. It is a rapidly spreading infection along deep fascial
planes with subcutaneous tissue necrosis. It typically is caused by Streptococcus pyogenes
or Staphylococcus aureus, but anaerobes and Clostridium species may also be present.
Urgent surgical exploration is indicated with aggressive debridement and should not be
delayed for culture or radiographic results.
A CT scan will not reveal anything new that can change surgical management.
The patient must be aggressively resuscitated and undergo aggressive debridement. A
second-look procedure may be required in 24 to 48 hours.

Go to the next page if you knew the correct answer, or click the link image(s) below to further
research the concepts in this question (if desired).

Research Concepts:
Necrotizing Fasciitis

We update eBooks quarterly and Apps daily based on user feedback. Please tap flag to
report any questions that need improvement.
Question 2: A 69-year-old heavy smoker presents to the emergency department with severe
knife-like chest pain and diaphoresis that woke him up from his sleep 2 hours ago. He denies
being short of breath but claims that the pain is severe, continuous, and radiates to his upper back
and shoulders. His past medical history reveals longstanding poorly controlled hypertension,
diabetes, and peripheral vascular disease. He is not compliant with his medications, smoked 1
pack of cigarettes daily for the past 30 years, and drinks alcohol regularly. By the time the
physical exam is done, the patient’s chest pain has subsided. His blood pressure is 180/90
mm/Hg in his right arm and 120/63 mm/Hg in his left arm, pulse is 110 bpm, respiratory rate is
18 breaths/min, and temperature is 98.8 F. He is alert. His ECG shows T-wave inversions in
leads V3 an V4. He undergoes a CT angiogram of the chest depicted in the image. What is the
next best step in the management of this patient?

Choices:
1. Aspirin
2. Esmolol
3. Sodium nitroprusside
4. Observation for 24 hours

Photo:Contributed by chestatlas.com (H. Shulman MD)


Answer: 2 - Esmolol
Explanations:
This patient likely has an aortic dissection. Aortic dissection is a relatively uncommon
illness that can be life-threatening. Early diagnosis and treatment remain the cornerstone of
the management of aortic dissection and are crucial for the survival of the patient.
The walls of blood vessels in the body are made up of 3 layers, from inside to outside,
intima, media, and adventitia. In aortic dissection, a tear in the aortic intima layer occurs.
This allows blood to pass from the aortic lumen, through the intimal tear into the space
between the intima and media layers. This creates a false lumen between the intima and
media layers of the aortic wall. 65% of aortic intimal tears occur at the sinotubular junction
of the ascending aorta and extends to involve the remaining thoracic aorta, creating an
ascending aortic dissection. Around 30% of intimal tears originate close to the area where
the left subclavian artery branches off of the aorta and extends to involve the remaining
descending aorta, creating a descending aortic aneurysm. The reason that aortic dissection
most commonly affects these 2 areas is thought to be due to shear forces being the highest
in these areas.
Aortic dissection usually presents with stabbing chest pain that radiates to the upper back
and shoulders and a greater than 20 mm Hg difference in the systolic blood pressure
between both arms in the setting of a history of hypertension or a connective tissue disorder
(e.g., Marfan syndrome). Depending upon the area and behavior of the aortic dissection,
other signs and symptoms may be present. Ascending aorta dissections can be complicated
by aortic valve regurgitation (diastolic aortic murmur), myocardial infarction (due to
obstruction of the coronary artery ostia), cardiac tamponade (rupture of the dissection into
the pericardium), stroke (carotid artery occlusion), horners syndrome (compression of
superior cervical ganglia), vocal cord paralysis (recurrent laryngeal nerve compression).
Descending aorta dissections can be complicated by hemothorax (rupture of dissection into
the thorax), renal injury (renal artery obstruction), mesenteric ischemia (obstruction if
superior mesenteric artery), lower limb paraplegia (spinal cord ischemia due to obstruction
of intercostal vessels).
The goal of management in both ascending and descending aortic dissection is pain control,
lowering of the systolic blood pressure and decreasing left ventricular contractility to
decrease aortic wall shear stress. IV beta-blocker (e.g., esmolol, labetalol) decrease the heart
rate, both decreasing the blood pressure and left ventricular contractility and is the preferred
medication used in the initial management of acute aortic dissection. In addition to the
above, emergent surgical repair is indicated in cases of ascending aortic dissection due to its
risk of life-threatening complications. The mortality rate of ascending aortic dissection is
1%-2% per hour after the onset of symptoms.

Go to the next page if you knew the correct answer, or click the link image(s) below to further
research the concepts in this question (if desired).

Research Concepts:
Aortic Dissection

We update eBooks quarterly and Apps daily based on user feedback. Please tap flag to
report any questions that need improvement.
Question 3: A 65-year old male with uncontrolled type 2 diabetes mellitus who is admitted
to the cardiac care unit after undergoing three-vessel coronary artery bypass graft after a recent
myocardial infarction. He is currently receiving intravenous insulin infusion for his
hyperglycemia. On a postoperative day 3, you note that he is off vasopressors and his blood
glucose is maintained between 130 to 160 mg/dL on a stable rate of insulin infusion. However,
his caloric intake is minimal. You plan to transition him to subcutaneous insulin based on the
average rate of infusion over the last 6 hours of 1.5 units/hour. What are the best process and
insulin doses needed to manage this transition?

Choices:
1. Two hours prior to discontinuing the intravenous insulin infusion, administer subcutaneous
insulin glargine 25 units
2. Simultaneously, discontinue the intravenous insulin drip and administer subcutaneous insulin
glargine 25 units
3. Two hours prior to discontinuing intravenous insulin, administer subcutaneous insulin glargine
18 units
4. Discontinue intravenous insulin drip and start correctional insulin coverage every 4 hours
Answer: 1 - Two hours prior to discontinuing the intravenous insulin infusion, administer
subcutaneous insulin glargine 25 units

Explanations:
Due to the extremely short half-life of intravenous (IV) insulin and delayed onset of action
of long-acting subcutaneous insulin, it is important to overlap the two types of insulin by 2-
3 hours. Premature discontinuation of IV insulin creates a hiatus in basal insulin supply,
which risks rebound hyperglycemia or metabolic decompensation.
When using the rate of infusion to calculate the basal insulin dose, the average rate of
infusion over the last 6 to 8 hours is extrapolated to 24 hours. Seventy to eighty percent of
this extrapolated dose represents the total daily dose (TDD) of insulin requirement. In a
patient with minimal or no caloric intake, 100% of TDD is the basal requirement. In
patients with optimal caloric intake, half of TDD should be administered as basal insulin
and the other half as nutritional insulin.
In the above case scenario, the average rate of infusion is 1.5 units/hour over the last 6
hours. On extrapolating this to the 24-hour period, the average amount of insulin is 36 units.
As a patient has minimal caloric intake, 70-80% of the extrapolated dose (25-28 units) is the
basal requirement and should be administered at least 2 hours before discontinuation of IV
insulin.
If the patient in the above case had an optimal caloric intake while on the insulin drip, then
option three would have been the right choice. In that case, the calculate TDD (i.e.70-80%
of the extrapolated dose) would represent both basal and nutritional requirements as
correctional insulin alone does not provide any basal coverage and risks wide fluctuations in
glycemic control.

Go to the next page if you knew the correct answer, or click the link image(s) below to further
research the concepts in this question (if desired).

Research Concepts:
Diabetic Perioperative Management

We update eBooks quarterly and Apps daily based on user feedback. Please tap flag to
report any questions that need improvement.
Question 4: A 42-year-old male presents to the emergency department with cough, dyspnea,
and hemoptysis with a history of idiopathic pulmonary hemosiderosis. While in the hospital, the
patient's hemoptysis worsens, and he is now progressing towards acute respiratory failure,
leading to intubation. The patient underwent bronchoscopy with serial washings and was
diagnosed with diffuse alveolar hemorrhage. The patient is stabilized on the ventilator in the
ICU. Which of the following is the most appropriate treatment for the patient's ongoing disease
process at this time?

Choices:
1. IV methylprednisolone 500 to 2000mg/day for 5 days
2. IV Azathioprine 2mg/kg/day for 6 weeks
3. Inhaled fluticasone 1760 mcg/day indefinitely
4. Stat consult to surgery for emergent splenectomy
Answer: 1 - IV methylprednisolone 500 to 2000mg/day for 5 days
Explanations:
There are currently no well-done studies to indicate that the current management is “gold
standard”. Most treatment and recommendations are based off observation and clinical
experience of the treating physician. Of the many approaches to idiopathic pulmonary
hemosiderosis (IPH) treatment, systemic steroids and immunosuppressant therapy have
shown the most favorable outcome either used independently or in combination.
Patients with acute IPH and respiratory failure secondary to alveolar hemorrhage may
require invasive ventilation support. For patients with this presentation, it is recommended
they be started on IV methylprednisolone in pulse dosing of 500 to 2000mg/day for 5 days.
In children, pulse dosing is typically 20mg/kg per day. Once the patient is stabilized this can
be transitioned to oral steroids with tapering down to a maintenance dose of 10 to
15mg/day.
Immunosuppressive agents including cyclophosphamide, hydroxychloroquine, and
azathioprine have been shown to help with severe disease. Dosing, duration, and optimal
indications for immunosuppressive agents are poorly defined.
Studies for treatment of IPH have shown splenectomy does not provide any morbidity or
mortality benefit. Treatment of possible underlying causes such as Goodpasture syndrome
(anti-GBM antibodies) or drug-induced injury (such as that caused by amiodarone) should
also be addressed.

Go to the next page if you knew the correct answer, or click the link image(s) below to further
research the concepts in this question (if desired).

Research Concepts:
Idiopathic Pulmonary Hemosiderosis

We update eBooks quarterly and Apps daily based on user feedback. Please tap flag to
report any questions that need improvement.
Question 5: A 17-year-old teenager with known sickle cell disease presents to the
emergency department with severe back and extremity pain for the past two days. The patient
has been taking oxycodone and ibuprofen at home with no significant relief. He denies any fever,
cough, or shortness of breath. Past medical history is significant for several hospitalizations for
painful crisis. He has not taken any medications for the past six hours and had one episode of
vomiting prior to his arrival. Physical exam reveals that he is in significant discomfort and is
writhing in pain. His pain score is 9/10. Vital signs reveal a pulse of 79/min, respiratory rate
22/min, temperature 98.5 F, blood pressure 124/85 mmHg, and oxygen saturation 97% in room
air. He is given 4 mg morphine subcutaneously and IV access obtained. His pain score 30
minutes later is recorded as 8/10. What is the next best step in the management of this patient?

Choices:
1. Escalate the opiate dose given intravenously by 25% every 15 minutes until the pain is
controlled
2. Immediate start dose of one gram ceftriaxone given intravenously
3. Give IV fluids at 2 times the maintenance dose
4. Withhold opiates as the patient may be drug seeking
Answer: 1 - Escalate the opiate dose given intravenously by 25% every 15 minutes until the
pain is controlled

Explanations:
Patients presenting with sickle cell crisis should be triaged to emergency severity index
(ESI) category 2 (high priority) for prompt and aggressive treatment of pain and should
receive their first analgesic dose within 30 minutes of arrival. Reassessment of pain is
recommended every 15-30 minutes, and for persistent pain, escalation of the analgesic dose
by 25% is recommended until the pain is controlled.
Addition of non-steroidal agents (ketorolac) may provide additional analgesia. Non-
pharmacological measures such as local heat, distraction, acupuncture, and hypnosis have
been variably shown to be beneficial.
Patient-controlled analgesia has shown to be more effective than intermittent dosing to
reduce the length of stay in the hospital and increased patient satisfaction. This can often be
initiated in the emergency department and then continued during the in-patient course.
Antibiotics are not generally indicated for painful crisis except if the patient is febrile or
shows clinical features suggestive of acute chest syndrome. Intravenous fluids should be
used judiciously to maintain hydration as excess fluid therapy has been implicated as a
factor in the occurrence of acute chest syndrome and normal saline bolus infusions have
been associated with poorer pain control.

Go to the next page if you knew the correct answer, or click the link image(s) below to further
research the concepts in this question (if desired).

Research Concepts:
Sickle Cell Crisis

We update eBooks quarterly and Apps daily based on user feedback. Please tap flag to
report any questions that need improvement.
Question 6: A patient is being treated for malaria arrives in the emergency department with
a chief complaint of ringing in the ears, blurred vision, nausea and vomiting, and confusion. On
physical exam, the patient has a temperature, is hypotensive, and a fingerstick glucose reads 50.
Which of the following would not be an expected?

Choices:
1. Massive hemolysis
2. Hemoglobinuria
3. Renal failure
4. Sweet taste in the mouth
Answer: 4 - Sweet taste in the mouth
Explanations:
Drugs like quinidine are associated with cinchonism, which is characterized by tinnitus,
blurred vision, gastrointestinal upset, and delirium.
Cinchonism or quinism is caused by an overdose of quinine or quinidine, or cinchona bark.
Quinine is used to treat malaria. In small amounts, quinine is an ingredient of tonic drinks
acting as a bittering agent. Cinchonism can occur from therapeutic doses of quinine, either
from one or several large doses. Quinidine can cause cinchonism symptoms to develop with
as little as a single dose.
Symptoms of mild cinchonism include flushed and sweaty skin, ringing in the ears, blurred
vision, impaired hearing, confusion, reversible high-frequency hearing loss, headache,
abdominal pain, rashes, lichenoid photosensitivity, vertigo, dizziness, dysphoria, nausea,
vomiting, and diarrhea. Large doses may result in severe reversible symptoms of
cinchonism which includes skin rashes, deafness, somnolence, diminished visual acuity or
blindness, anaphylactic shock, and disturbances in abnormal heart rhythm or conduction,
and potentially death from cardiotoxicity. Quinine may trigger a rare form of
hypersensitivity reaction in malaria patients termed blackwater fever with massive
hemolysis, hemoglobinemia, hemoglobinuria, and kidney failure. Patients treated with
quinine may develop hypoglycemia and hypotension. Most symptoms of cinchonism are
reversible and disappear once quinine is withdrawn.
Quinine inactivates enzymes in the lysosomes of cells, and has an anti-inflammatory effect.
As a result, it sometimes used to treat rheumatoid arthritis. Switching off these enzymes can
cause abnormal accumulation of glycogen and phospholipids in lysosomes, resulting in a
toxic myopathy. This may be the cause of cinchonism.

Go to the next page if you knew the correct answer, or click the link image(s) below to further
research the concepts in this question (if desired).

Research Concepts:
Cinchonism

We update eBooks quarterly and Apps daily based on user feedback. Please tap flag to
report any questions that need improvement.
Question 7: A 16-year-old boy who had been recovering from a flu-like illness presented to
the emergency department with acutely worsening dyspnea at rest, dizziness, and lethargy for
two days. He had a low-grade fever, and vitals at the time of presentation were as follows blood
pressure 74/35 mmHg, heart rate 118/min, respiratory rate 36/min, and oxygen saturation of
89%. EKG showed sinus tachycardia, and labs were significant for elevated troponin and
proBNP. He was started on inotropic support after a failed fluid challenge. An echocardiogram
revealed a globally hypokinetic myocardium with severely reduced left ventricular ejection
fraction (18%), normal ventricular dimensions, and slightly thickened left ventricular walls. An
endomyocardial biopsy revealed multiple foci of CD3+ cell-rich infiltrates with widespread
myocyte necrosis and interstitial edema. The patient underwent intubation and was placed on
mechanical ventilation as he became more lethargic, but he continues to be hypotensive despite
maximal inotropic support. What is the most appropriate next step?

Choices:
1. Implantable cardioverter-defibrillator (ICD)
2. Biventricular pacemaker/cardiac resynchronization therapy (CRT)
3. Left ventricle assist device (LVAD)
4. Cardiac transplantation
Answer: 3 - Left ventricle assist device (LVAD)
Explanations:
Fulminant myocarditis occurs in children and young adults and is characterized by acute
severe decompensation of cardiac function following a viral respiratory or gastrointestinal
illness.
It is seen on echocardiogram as a hypokinetic myocardium with normal left ventricular
dimensions but a slightly thickened ventricular wall because of edema.
It can resolve spontaneously but may require aggressive interventions like LVAD
(mechanical circulatory support) during its active phase, which can improve cardiac output.
Extracorporeal membranous oxygenation (ECMO) is a quicker and less expensive
alternative for LVAD, which can be used as a bridge to recovery. However, it may not
increase the cardiac output as much as an LVAD.

Go to the next page if you knew the correct answer, or click the link image(s) below to further
research the concepts in this question (if desired).

Research Concepts:
Nonviral Myocarditis

We update eBooks quarterly and Apps daily based on user feedback. Please tap flag to
report any questions that need improvement.
Question 8: A 56-year-old man with a history of alcohol use disorder and recurrent bouts of
pancreatitis presents with severe abdominal pain, hypotension, and tachycardia. Lipase is
elevated over 1000 U/L. White blood cell count is 14,000/microL, and hemoglobin is 6.5 g/dL.
He is cachectic appearing and pale. He has tenderness to palpation in the epigastrium. His last
admission for pancreatitis was two weeks ago, and at that time he had a CT scan without contrast
that was concerning for a small peripancreatic fluid collection which may represent a developing
pseudocyst. He denies any bloody emesis or hematochezia. What should be done next?

Choices:
1. Obtain an ultrasound of the abdomen
2. Obtain a CT angiogram of the abdomen and pelvis
3. Admit the patient for serial abdominal exams and serial hematocrit checks only
4. Emergent exploratory laparotomy
Answer: 2 - Obtain a CT angiogram of the abdomen and pelvis
Explanations:
While ultrasound can show filling of a pseudoaneurysm, the exam can be limited by bowel
gas or body habitus and a more definitive test which can show cross-sectional anatomy is
better
This patient may have a visceral artery pseudoaneurysm secondary to chronic pancreatitis.
A non-contrasted scan cannot reliably differentiate between pancreatic fluid and blood, and
a contrasted scan would help identify both aneurysmal and pseudoaneurysmal degeneration
of an artery exposed to pancreatic fluid. After the diagnosis is confirmed, an endovascular
or open repair can be attempted.
This patient shows signs of hemorrhagic shock. Observation alone is not appropriate.
An exploratory laparotomy without a definitive diagnosis is ill-advised as the patient could
instead have chronic anemia from malnutrition and chronic disease and a systemic
inflammatory response syndrome from active pancreatitis.

Go to the next page if you knew the correct answer, or click the link image(s) below to further
research the concepts in this question (if desired).

Research Concepts:
Pseudoaneurysm

We update eBooks quarterly and Apps daily based on user feedback. Please tap flag to
report any questions that need improvement.
Question 9: A 17-year-old male presents after ingesting 100 pills of 81 mg aspirin 55
minutes ago. His Glasgow Coma Scale score is 15, airway reflexes are intact, respiratory rate is
25, heart rate 105 beats/min, blood pressure 120/80 mmHg, and temperature 101.5 F. He weighs
55 kg. What is the appropriate dose of single-dose activated charcoal?

Choices:
1. None, he is outside of the 30-minute timeframe for administration of activated charcoal
2. 100 grams of activated charcoal
3. 55 grams of activated charcoal
4. 80 grams of activated charcoal
Answer: 4 - 80 grams of activated charcoal
Explanations:
Activated charcoal (AC) can absorb certain non-polar toxic materials that are in the liquid
phase within the gastrointestinal (GI) lumen.
The typical timeframe of AC administration is within 1 hour of ingestion. However,
substances with a large pill burden that may form bezoars, substances that undergo
enterohepatic or enteroenteric recirculation, or substances that decrease GI motility may
benefit from activated charcoal administration up to 4 hours after ingestion.
In cases of a known xenobiotic dose, AC should be given at a 10:1 ratio of AC to the
xenobiotic. In this case, approximately 8 grams of aspirin was ingested, so 80 grams of
activated charcoal is indicated.
When the xenobiotic dose is unknown, doses of AC should be given at 1 to 2 gram/kg of
body weight. If the amount of aspirin was unknown, it would be reasonable to administer 55
grams of AC to this patient.

Go to the next page if you knew the correct answer, or click the link image(s) below to further
research the concepts in this question (if desired).

Research Concepts:
Activated Charcoal

We update eBooks quarterly and Apps daily based on user feedback. Please tap flag to
report any questions that need improvement.
Question 10: A 24-year-old man presents to the office for regular blood transfusion for
beta-thalassemia major. His recent laboratory results reveal the development of alloantibodies
against red blood cells (RBCs) surface antigen. His current blood pressure is 116/78 mmHg, the
pulse is 74/min, and the temperature is 99 F. He appears in no apparent distress, and his last
checked hemoglobin was 7 g. Which type of blood may be appropriate for transfusion for this
patient at this visit?

Choices:
1. Leukoreduced RBCs
2. Washed packed RBCs
3. Frozen packed RBCs
4. Irradiated RBCs
Answer: 3 - Frozen packed RBCs
Explanations:
Patients with thalassemia are usually given packed RBCs with the removal of leukocytes for
regular transfusion. But patients who develop alloantibodies may require frozen packed
RBCs. The product of choice is packed red blood cells depleted of leucocytes. It should be
matched with the patient’s red antigen phenotype for at least D, C, c, E, e, and Kell.
Frozen (or cryopreserved) red cells is the component derived from whole blood in which
red cells are frozen, preferably within 7 days of collection. These are used to maintain a
supply of rare donor units for patients who have unusual red cell antibodies or who are
missing common red cell antigens.
Washed RBCs are given when a patient has a history of allergic reactions to blood
transfusion in the past.
Irradiated RBCs are usually used to prevent graft-versus-host disease. It is largely
unnecessary unless the patient is undergoing a bone marrow transplant or has an underlying
immunodeficiency.

Go to the next page if you knew the correct answer, or click the link image(s) below to further
research the concepts in this question (if desired).

Research Concepts:
Thalassemia

We update eBooks quarterly and Apps daily based on user feedback. Please tap flag to
report any questions that need improvement.
Question 11: A 65-year-old female is postoperative day one status post right hip
replacement. She is hypotensive, tachycardic, and hypoxic. Which of the following is the
appropriate therapy for this patient?

Choices:
1. Pulmonary embolectomy
2. Heparin
3. Lytic therapy
4. Hirudin
Answer: 3 - Lytic therapy
Explanations:
Acute pulmonary embolectomy sometimes is required in patients who are hemodynamically
unstable with a confirmed embolus. However, current guidelines indicate that the treatment
of choice is lytic therapy in unstable patients.
Lytic therapy works rapidly and the dose of tPA is 100 mg over 2 hours. Surgery is only
undertaken if there are contraindications to lytic therapy. Surgery in these patients carries a
very high mortality and is the last treatment of choice.
Anticoagulant therapy is the mainstay of treatment for patients who are hemodynamically
stable.
Placement of an inferior vena cava filter should be considered if anticoagulants are
contraindicated or a pulmonary embolism occurs in a patient who is being anticoagulated.

Go to the next page if you knew the correct answer, or click the link image(s) below to further
research the concepts in this question (if desired).

Research Concepts:
Acute Pulmonary Embolism

We update eBooks quarterly and Apps daily based on user feedback. Please tap flag to
report any questions that need improvement.
Question 12: A 60-year-old male with a past medical history of hypertension, myocardial
infarction, and congestive heart failure with reduced ejection fraction comes to the emergency
department complaining of worsening exertional dyspnea over the past 3 days. He states that his
lower extremity edema and orthopnea has also been progressively increasing. His current
medications include metoprolol, losartan, furosemide, and aspirin. His blood pressure is 124/85
mmHg, and pulse is 95 bpm, irregularly irregular. On chest auscultation, there are significant
bilateral crackles over the lower lung fields, a holosystolic murmur at the apex, and an S3 gallop
heard at the left sternal border. Laboratory evaluation shows hemoglobin 9.5 g/dL, leukocyte
count of 5,500/microliter, and platelet count of 250,000/microliter. Serum chemistries show that
sodium is 125 mEq/L, and brain natriuretic peptide (BNP) is 800 pg/mL. Electrocardiogram
shows atrial fibrillation. In cases of acute decompensation of heart failure, what have studies
shown to be associated with poor prognosis?

Choices:
1. Maintaining oxygen extraction of 27% or less
2. Central venous oxygen saturation (ScvO2) 70%
3. Central venous oxygen saturation (ScvO2) 60%
4. Central venous oxygen saturation (ScvO2) > 80 %
Answer: 3 - Central venous oxygen saturation (ScvO2) 60%
Explanations:
In certain studies, maintaining oxygen extraction of 27% or less was found to lead to
decreased incidences of organ failure and decreased hospital length of stay in patients in the
perioperative setting. Studies also demonstrated poor prognosis of heart failure with central
venous oxygen saturation of less than 60%.
Studies have shown that ScvO2 70% is associated with poor prognosis in sepsis.
Meanwhile, studies have shown that ScvO2 60% is associated with poor prognosis in heart
failure.
Studies also demonstrate that heart failure causes decreased oxygen delivery, thus
increasing tissue oxygen extraction. This is exhibited in cardiogenic shock.
Studies have found that ScvO2 60% within the first 24 hours is also a predictor of major
adverse cardiovascular events.

Go to the next page if you knew the correct answer, or click the link image(s) below to further
research the concepts in this question (if desired).

Research Concepts:
Anesthesia Monitoring Of Mixed Venous Saturation

We update eBooks quarterly and Apps daily based on user feedback. Please tap flag to
report any questions that need improvement.
Question 13: A 17-year-old male with no past medical history presents to the emergency
department severely intoxicated after reportedly ingesting a bottle of isopropyl alcohol. His labs
are remarkable for a creatinine of 1.8 mg/dL and a BUN of 10 mg/dL. Ketones are detectable in
urine. He vomited twice and urinated in his hospital bed. What is the most appropriate
management?

Choices:
1. Observe until clinically well and discharge with a responsible adult
2. Admit to the intensive care unit
3. Administer fomepizole and admit to the intensive care unit
4. Consult nephrology for dialysis
Answer: 1 - Observe until clinically well and discharge with a responsible adult
Explanations:
Isopropyl is known to cause severe intoxication, hemorrhagic gastritis, and falsely elevated
creatinine due to acetone (Jaffe reaction).
Most patients improve with supportive care, including hydration, and can be discharged if
ingestion was not a self-harm attempt.
Fomepizole is not required to treat isopropyl alcohol ingestion because its metabolite does
not cause acidosis or any specific end-organ toxicity.
Dialysis is not necessary as the creatinine is falsely elevated. This requires a repeat basic
metabolic panel when clinically improved to reassess creatinine, but the normal blood urea
nitrogen does not correlate with the elevated creatinine, raising the concern for a falsely
elevated creatinine.

Go to the next page if you knew the correct answer, or click the link image(s) below to further
research the concepts in this question (if desired).

Research Concepts:
Isopropanol Toxicity

We update eBooks quarterly and Apps daily based on user feedback. Please tap flag to
report any questions that need improvement.
Question 14: A 32-year-old female with placenta percreta is posted for an elective caesarian
section and obstetric hysterectomy. After delivery, there was massive blood loss, and blood
transfusion protocol was initiated as per institutional guidelines. Thromboelastogram report
showed increased K time and R time and normal maximum amplitude. What should be
administered to the patient to aid in hemostasis?

Choices:
1. Packed red blood cells and platelets
2. Plasma and cryoprecipitate
3. Plasma and platelets
4. Platelets only
Answer: 2 - Plasma and cryoprecipitate
Explanations:
R time (reaction time) is the time of latency from the start of the test to initial fibrin
formation. It is dependent on clotting factors. The normal value is 4-8 minutes.
K time is dependent on fibrinogen and signifies time taken to achieve certain clot strength
(amplitude of 20 mm). Cryoprecipitate and fibrinogen concentrate are good sources for
fibrinogen. The normal value is 1-4 minutes.
Alpha angle (degrees) measures the rate at which fibrin builds up, and cross-linking
assesses, in turn, the rate of clot formation. The function also depends on fibrinogen levels.
The normal value is 47-74 degrees.
Maximum amplitude (mm) represents the clot strength which is a result of the function of
platelets (80%) and fibrin (20%). It helps to identify whether the cause of bleeding is due to
coagulopathy or mechanical disruption. The normal value is 55-73 mm.

Go to the next page if you knew the correct answer, or click the link image(s) below to further
research the concepts in this question (if desired).

Research Concepts:
Fibrinogen

We update eBooks quarterly and Apps daily based on user feedback. Please tap flag to
report any questions that need improvement.
Question 15: A 20-year-old presents to the emergency in an active seizure. Friends report
that she had taken a large amount of a medication prescribed for stomatitis. On arrival, blood
pressure is 80/50 mmHg, pulse 130 bpm, and respiratory rate 8/minute. What medication should
be avoided in this patient?

Choices:
1. Benzodiazepines
2. Levetiracetam
3. 20% lipid emulsion
4. Phenytoin
Answer: 4 - Phenytoin
Explanations:
Benzodiazepines are the first line for treatment of seizures.
Levetiracetam can be given if seizures are refractory to benzodiazepines. Average adult
patients should be given a loading dose of 1g or 20 mg/kg of levetiracetam.
This patient presents with lidocaine toxicity after overdosing on lidocaine prescribed for
stomatitis. Lipid emulsion should be used in an attempt to reverse the toxicity of the
lidocaine.
Phenytoin should be avoided in lidocaine toxicity as it also has sodium channel blocking
effects. Use of phenytoin could potentially worsen the condition.

Go to the next page if you knew the correct answer, or click the link image(s) below to further
research the concepts in this question (if desired).

Research Concepts:
Sodium Channel Blocker Toxicity

We update eBooks quarterly and Apps daily based on user feedback. Please tap flag to
report any questions that need improvement.
Question 16: A 65-year-old female with past medical history of atrial fibrillation,
hypertension, chronic old left bundle branch block and diabetes mellitus is admitted to the
hospital with community-acquired pneumonia. She is on dofetilide for atrial fibrillation and
apixaban for anticoagulation. She is started on levofloxacin 750 mg IV for pneumonia. Twelve
hours after the admission, the patient becomes unconscious, and the telemetry shows she has
polymorphic ventricular tachycardia. Immediate defibrillatory shock was delivered, and patient
reverted to sinus rhythm. The patient was transferred to the cardiac intensive care unit. It is noted
that she is on levofloxacin which prolongs QTc and dofetilide also has QTc prolonging effect.
Both medications are discontinued immediately. Which of the following should be
recommended?

Choices:
1. Dual chamber pacemaker as she went into ventricular tachycardia
2. List dofetilide in her allergies medications and never prescribe it to her again
3. Stop levofloxacin and hold the dofetilide until the QTc normalizes
4. Hold apixaban
Answer: 3 - Stop levofloxacin and hold the dofetilide until the QTc normalizes
Explanations:
A pacemaker with implantable cardioverter defibrillator (ICD) is indicated for patients who
go into ventricular tachycardia (VT) without a reversible cause. It is a very common
indication for the secondary prevention of VT.
She may have had good control with dofetilide for her atrial fibrillation hence listing in her
allergies is not reasonable.
Reversible polymorphic ventricular tachycardia is not an indication for the ICD placement.
In this scenario, stop the offending QTc prolonging drugs like fluoroquinolones and
dofetilide.
Patients with atrial fibrillation who have a CHADSVasc score greater than or equal to 2
should be prescribed an anticoagulation agent.

Go to the next page if you knew the correct answer, or click the link image(s) below to further
research the concepts in this question (if desired).

Research Concepts:
Ventricular Tachycardia

We update eBooks quarterly and Apps daily based on user feedback. Please tap flag to
report any questions that need improvement.
Question 17: A 48-year-old male presents to the emergency department with two-and-a-half
months of progressively worsening left-sided chest pains and shortness of breath with associated
low-grade fevers, chills, occasional night sweats, productive cough of foul tasting sputum, and
15 lbs unintentional weight loss. He does not have a primary care provider, denies any medical
or surgical history, does not believe in "Big Pharma" and takes no medications. He denies any
sick contacts, refuses all vaccinations, does not drink any alcohol, use any tobacco products or
illicit drugs, but does report an extensive travel history in South Asia over the last two years as a
Christian missionary. He reports that he got home about six weeks ago from Vietnam, but his
symptoms started off milder probably 6 to 8 weeks before that. He states they seemed to coincide
a week or so after he had gotten over a really bad case of food allergies. He was not sure what he
ate that set it off but reported a two-week history of very high fevers, abdominal pains, and
cramping with nausea, vomiting, loss of appetite, and generalized itching that gradually
subsided. He did seek medical treatment at one point, but only got intravenous fluids, an
antihistamine, and antiemetic medication. They did not do any blood work or imaging. He
reports feeling better for about a week before the new symptoms started. He was busy preparing
for his trip back to America, so he did not pursue any medical care until now. He comes in
tonight because he can't take the fevers and night sweats any more which woke him up once
again from sleep. He does admit that he searched his symptoms on the internet last week and is
worried that he might have cancer. Vitals signs are temperature 38.4 C, heart rate 108 bpm,
respiratory rate 24/minute, blood pressure 112/67 mmHg, and oxygen saturation 86% on room
air. He appears acutely ill, no acute distress and is normocephalic and atraumatic. Sclera are
nonicteric and pupils equal round and reactive to light and accommodation. The neck nontender
with no jugular venous distention. He is mildly tachycardic with regular rhythm and no murmur.
Lungs are clear to auscultation bilaterally. The abdomen is soft, nontender with normal bowel
sounds. There is no costovertebral angle tenderness or bladder distension. He has normal speech
with no neurologic deficits. His chemistry panel shows normal electrolytes, normal renal
function, and normal hepatic function. His complete blood count shows mild leukocytosis with
WBC count of 11.8/microL and differential reveals 6.8% eosinophils. His chest x-ray shows a
there is a large cavitary lung lesion in the left upper lobe. CT scan of the chest shows there is a 4
cm x 6 cm ring-enhancing cavitary lung abscess in the left upper lobe with air-fluid level. There
are no malignant findings. The patient is diagnosed with sepsis secondary to a lung abscess. He
is admitted to the medical floor and started on broad-spectrum antibiotics with intravenous
vancomycin and intravenous ampicillin-sulbactam. Over the next three days, the patient
continues to fever and have persistent symptoms. He undergoes bronchoscopy and
bronchoalveolar lavage on hospital day four. Cytology of the fluid is negative for malignancy but
does note a high preponderance of eosinophils and trematode eggs that are yet to be
characterized. His blood and sputum cultures remain negative since admission. Based on the
above clinical scenario, what is the most likely causative organism?

Choices:
1. Taenia solium
2. Enterobius vermicularis
3. Diphyllobothrium latum
4. Fasciola giganticum
Answer: 4 - Fasciola giganticum
Explanations:
Ectopic fascioliasis is very rare, but the parasite may present in nearly any area of the body
and has been described in the literature. While Fasciola gigantica go almost anywhere,
Enterobius vermicularis and Diphyllobothrium latum stay primarily in the gastrointestinal
tract. Taenia solium has been known to cause primarily central nervous system lesions from
larval migration.
Peripheral eosinophilia in varying degrees is common amongst all filarial worm infections
and is nonspecific, but should clue one into a parasite as the culprit.
Fasciola giganticum infection is more commonly associated with travel to Asia, the Pacific
Islands, and some parts of Northern Africa than any of the other species listed. The patient
had spent the last two years in South Asia as a missionary and had just returned.
The patient had complained of a severe gastrointestinal illness lasting a week or two that
preceded his current illness. This likely signified the acute phase of the fasciola infection
when metacercariae migrate out of the intestines and through the liver causing an
overwhelming inflammatory and immune response.

Go to the next page if you knew the correct answer, or click the link image(s) below to further
research the concepts in this question (if desired).

Research Concepts:
Fascioliasis

We update eBooks quarterly and Apps daily based on user feedback. Please tap flag to
report any questions that need improvement.
Question 18: A 73-year-old man with a past medical history of congestive heart failure,
atrial fibrillation, and hypertension presents with jugular venous distention, exertional dyspnea,
and bilateral lower leg edema for the past 2 days. Patient's ejection fraction is shown to be 20%
by echocardiogram. In the evening of admission, the patient's blood pressure drops to 65/30
mmHg and is placed on continuous intravenous dobutamine. Hours later, his blood pressure
increases to 90/45 mmHg, and EKG shows atrial fibrillation with rapid ventricular response.
Which of the following medications would have been the best choice to prevent the abnormal
EKG findings?

Choices:
1. Metoprolol after dobutamine administration
2. Digoxin before dobutamine administration
3. Atropine along with dobutamine
4. Amiodarone at any point during the infusion
Answer: 2 - Digoxin before dobutamine administration
Explanations:
Digoxin regimen is given prior to dobutamine to reduce the risk of rapid ventricular
response in patients with a history of atrial fibrillation.
Dobutamine increases the risk of risk of rapid ventricular response in patients with
preexisting atrial fibrillation.
Dobutamine can be administered in heart failure patients with systolic dysfunction and
hypotension for the short-term.
Dobutamine cannot be combined with beta-blocker medications because of its
sympathomimetic effects.

Go to the next page if you knew the correct answer, or click the link image(s) below to further
research the concepts in this question (if desired).

Research Concepts:
Dobutamine

We update eBooks quarterly and Apps daily based on user feedback. Please tap flag to
report any questions that need improvement.
Question 19: A 65-year-old white female is admitted to the intensive care unit after
presenting with acute onset of severe respiratory distress, productive cough, fever, and
progressive obtundation. Initial vitals showed a temperature of 94 F with a heart rate of 110/min,
respiratory rate of 34/min, and a mean arterial pressure of 55 mmHg. Respiratory status declined
rapidly to require endotracheal intubation. She also became progressively hypotensive requiring
3 liters of normal saline bolus within 2 hours and is now on norepinephrine infusion. She has a
central line in the right subclavian and a radial arterial line. She is asynchronous on the vent and
breathing at 30/min with a set rate of 16/min. Tidal volume is at 6 ml/kbw. What is the most
reliable index to ascertain the need for additional fluid bolus?

Choices:
1. Central venous pressure less than 10 cmH2O
2. IVC variability greater than 13%
3. Pulse pressure variability greater than 13%
4. Pulse pressure variability greater than 10% with passive leg raise test
Answer: 4 - Pulse pressure variability greater than 10% with passive leg raise test
Explanations:
The decision to administer fluid in shock is increasingly being guided by dynamic measures
of volume responsiveness than static measures like central venous pressure (CVP).
Dynamic measures of volume responsiveness include IVC variability with respiration, pulse
pressure variability, stroke volume variability with respiration or with passive leg raising.
The technique for performing passive leg raising involves 1) sitting patient at 45 degrees
head-up semi-recumbent position, 2) lower patient’s upper body to horizontal and passively
raise legs at 45 degrees up, 3) maximal effect occurs at 30-90 seconds, 4) assess for a 10%
increase in stroke volume (cardiac output monitor) or pulse pressure variability (using an
arterial line).
The passive leg raise (PLR) has the advantage of being reversible; its effect is reversed once
the legs are returned to horizontal. The volume expansion from a PLR is 250-350 mL.
Pulse pressure variability or stroke volume variability with passive leg raising is the most
specific index of volume responsiveness at the bedside, with the exception of significant
tachyarrhythmias.

Go to the next page if you knew the correct answer, or click the link image(s) below to further
research the concepts in this question (if desired).

Research Concepts:
Systemic Inflammatory Response Syndrome

We update eBooks quarterly and Apps daily based on user feedback. Please tap flag to
report any questions that need improvement.
Question 20: A 30-year-old woman was brought to the emergency department by her
mother altered mentation. This patient his on medications for anxiety, depression with suicide
attempt and chronic pain. She has attempted suicide in the past. According to her mother, she has
been under tremendous amounts of stress with work and personal relationships, and she reports
that at home the patient was found with scattered pills on the floor. She is tachycardic at 135
bpm, hypertensive at 165/100 mmHg, not tachypneic and she is febrile at 38.4 C. She has dry
mucous membranes with flushed skin and mydriasis which is equal to both pupils. She is awake
but noted to be picking and swiping at the air as if something is in front of her. She is
neurologically intact otherwise. What is the most likely substance she ingested?

Choices:
1. Phenelzine
2. Pseudoephedrine
3. Diphenhydramine
4. Tramadol
Answer: 3 - Diphenhydramine
Explanations:
For the patient with drug intoxication an possible medication overdose, the clinician must
rely on a good history and physical examination for signs and symptoms. Her clinical
picture is consistent with diphenhydramine toxicity/overdose which is a histamine H1-
receptor antagonist with moderate to high anticholinergic and antiemetic properties.
Given the history of chronic pain and depression she likely has access to antidepressants
and pain medicines, but over the counter drugs must also be taken into consideration. Exam
findings of tachycardia, hypertension, fever, dry mucous membranes, flushing, dilated
pupils, and altered mental status. fit with anticholinergic toxidrome.
Most patients with anticholinergic poisoning do well with supportive therapy alone.
With severe agitation and delirium, patients may benefit from treatment with
physostigmine. Consultation with a toxicologist is recommended before giving
physostigmine.

Go to the next page if you knew the correct answer, or click the link image(s) below to further
research the concepts in this question (if desired).

Research Concepts:
Diphenhydramine Toxicity

We update eBooks quarterly and Apps daily based on user feedback. Please tap flag to
report any questions that need improvement.
Question 21: A 65-year-old male presents complaining that he cannot move his legs. He
denies any trauma, loss of consciousness, or visual or speech problems. On examination, there is
significant motor weakness, numbness from the buttocks to the soles of the feet, no sensation in
the perineal area, absent rectal tone, and a full, distended bladder. What is the best test for this
patient?

Choices:
1. Lumbar myelogram
2. MRI spine
3. Spinal tap
4. CT head
Answer: 2 - MRI spine
Explanations:
Classic cauda equina syndrome presents with leg weakness, bladder or bowel failure, and
hypesthesias from the buttocks to the feet.
MRI will give the best view of the cauda equina and lower spinal cord.
It will quickly determine if there is a lesion causing compression that requires surgical
intervention.
Cauda equina syndrome can be caused by trauma, neoplasms, and spinal abscesses.

Go to the next page if you knew the correct answer, or click the link image(s) below to further
research the concepts in this question (if desired).

Research Concepts:
Cauda Equina And Conus Medullaris Syndromes

We update eBooks quarterly and Apps daily based on user feedback. Please tap flag to
report any questions that need improvement.
Question 22: A 60-year-old Indonesian man is brought by ambulance with generalized
weakness, joint pain, and double vision. The family reported that the patient had fallen twice this
week due to the inability to ambulate. The wife states that the patient had been dragging his feet.
Further history revealed that they had gone for a 30th wedding anniversary cruise two weeks ago
and ate "just about everything." The patient returned home and had been feeling ill with fever,
productive cough, sinus congestion that resolved with rest and acetaminophen. The patient
received his annual influenza vaccination. On neurological exam, the patient had inaccuracies on
the visual field testing, blunted corneal reflexes, numbness in his mouth, and loss of taste for
sweets. Deep tendon reflexes showed trace in patellar deep tendon reflexes and +1 in upper
extremities. Relevant past medical history includes past polysubstance abuse, intravenous heroin,
and marijuana, with the last use being 25 years ago. A chest X-ray showed hilar adenopathy. A
lumbar puncture was performed, and cerebrospinal fluid studies showed elevated protein.
Serological studies were positive for anti-ganglioside antibodies. Pulse oximetry showed a
saturation of 84% on room air. An ABG was performed and showed a PaO2 of 50 mmHg and a
PaCO2 of 52 mmHg. Which of the following findings will predict poor prognosis in this patient?

Choices:
1. Presence of autoantibodies against ganglioside proteins
2. Hilar adenopathy on chest X-ray
3. Multiple focal cranial nerve deficits on exam
4. Evidence of hypoxemia on room air
Answer: 4 - Evidence of hypoxemia on room air
Explanations:
This clinical presentation is suspicious for Miller Fisher syndrome (MFS), a rare variant of
Guillain Barre syndrome (GBS). The classical triad of symptoms includes acute
ophthalmoplegia, areflexia, and ataxia. Distal lower extremity weakness, pain, and
paresthesias may be present. Other physical exam findings include diplopia, cranial nerve
palsies, and dampened corneal reflex. However, clinical symptoms by themselves do not
predict the prognosis of this condition.
The presence of elevated cerebrospinal fluid (CSF) protein with normal findings in the rest
of CSF studies should be suspicious for GBS. Anti-ganglioside antibody is a specific
finding for Miller Fisher syndrome, brainstem encephalitis, or other forms of GBS,
including a pharyngeal-cervical-brachial weakness. In the setting of hyporeflexia and lack
of oropharyngeal dysphagia, Miller Fisher is the most likely diagnosis. However, serology
is not used as a predictor of mortality and morbidity, rather than a specific finding to
confirm the diagnosis of MFS.
Multifocal deficits in MFS have been reported in the literature, not surprising given the
pathophysiology of this condition, including the targeting of the myelin sheaths of the
central and peripheral nervous system. Although this phenomenon is known in many cases
of MFS, it is not a predictive factor of mortality or morbidity in patients.
The most worrisome finding in a patient more than 50 years of age regarding prognosis is
the presence of hypoxemia on room air, as it is a risk factor for impending respiratory
failure in patients. Additionally, this finding is a major criterion for intensive care unit
admission in adults. Mechanical ventilation and ICU admission are recommended in
patients with at least one major criterion or 2 minor criteria. Major criteria include
hypercapnia with PaCO2 above 48 mm Hg, hypoxemia with PaO2 below 56 mm Hg while
the patient is breathing ambient air, vital capacity less than 15 mL/kg of body weight, and
negative inspiratory force less than -30 cm H2O. Minor criteria include an inefficient
cough, impaired swallowing, and atelectasis. Life-threatening complications are more likely
to occur in patients who meet the criteria for intensive care unit admission. These
complications include sepsis, pneumonia, pulmonary embolism, autonomic dysfunction,
and gastrointestinal bleeding. The risk of mortality or morbidity is higher in these patients.
Among severely affected patients, 20 to 33% may be unable to walk for more than six
months after symptom onset, especially if infected with Campylobacter jejuni. Patients may
also suffer from chronic psychiatric illness due to persistent pain and disability, among
other complications.

Go to the next page if you knew the correct answer, or click the link image(s) below to further
research the concepts in this question (if desired).

Research Concepts:
Miller Fisher Syndrome
We update eBooks quarterly and Apps daily based on user feedback. Please tap flag to
report any questions that need improvement.
Question 23: A 50-year-old male with a history of chronic obstructive pulmonary disease
(COPD) was admitted for acute hypercapnic and hypoxic respiratory failure secondary to COPD
exacerbation. He developed severe acidosis from hypercapnia and required immediate intubation
and admission to the intensive care unit. On the second day of hospitalization, he was improving,
and plans were made to start a weaning trial the next morning. Overnight the ventilator starts
sounding alarms. When the clinician arrives, he sees that the ventilator shows a peak pressure of
58 and a plateau pressure of 26. A physical exam reveals bilateral breath sounds and good air
movement. There is no engorgement of the jugular veins, and the blood pressure is within
normal limits. What are the most likely diagnosis and the next step in management?

Choices:
1. Acute mucous plugging or kink in the airway; suction the patient and assess airway
2. Worsening COPD; give a pulse dose of corticosteroids and start continuous nebulizer
treatment
3. Tension pneumothorax; place a needle in the second intercostal space followed by a chest tube
4. Air trapping secondary to auto-PEEP; adjust the I:E (inspiratory:expiratory) ratio
Answer: 1 - Acute mucous plugging or kink in the airway; suction the patient and assess
airway

Explanations:
An increase in the peak pressure without change in plateau pressures is indicative of
increased airway pressures, which is likely secondary to a mucous plug, the patient biting
down on the tube, or the tube being kinked. Initial management is to evaluate the airway
and suction the patient for possible mucous plug(s).
Worsening chronic obstructive pulmonary disease would likely present as decreased breath
sounds bilaterally and poor air movement. Also, a synchronous increase in plateau pressure
would be expected.
Tension pneumothorax would present with hypotension, hypoxia, and increased plateau
pressure. One would expect for the patient to have decreased breath sounds unilaterally and
to have jugular venous distention.
Auto-PEEP with air trapping causes an increase in both peak and plateau pressures. Patients
frequently seem to be dyssynchronous with the ventilator.

Go to the next page if you knew the correct answer, or click the link image(s) below to further
research the concepts in this question (if desired).

Research Concepts:
Ventilation Assist Control

We update eBooks quarterly and Apps daily based on user feedback. Please tap flag to
report any questions that need improvement.
Question 24: A 45-year-old female with a past medical history of factor V, hypertension,
hysterectomy, and a current smoker who presents to the emergency department with complaints
of diffuse abdominal pain that started five days ago. She states that she is supposed to be taking
apixaban due to her history of blood clots in the past but stopped taking it ten days ago. She
denies any associated fever, chills, vomiting or diarrhea. Physical examination reveals a non-
distended, soft abdomen with diffuse abdominal tenderness without guarding or rebound.
Abdominal ultrasound reveals a thrombus within the inferior vena cava. The patient is started on
IV heparin. What management is recommended at this time?

Choices:
1. No further management, continue heparin and admit
2. Call vascular surgery for catheter-directed thrombolysis or thrombectomy
3. Call vascular surgery for percutaneous transluminal angioplasty with stenting
4. Discontinue heparin and start warfarin
Answer: 2 - Call vascular surgery for catheter-directed thrombolysis or thrombectomy
Explanations:
Caval thrombi are at increased risk for lethal pulmonary emboli and can extend to the renal
veins. Therefore aggressive treatment is recommended for acute development. Catheter-
directed therapy has been shown to have an 85% early success rate. Acute clots respond
more favorably to catheter-directed thrombolysis and thrombectomy than chronic thrombus.
The type of thrombolysis chosen varies, but typically most common is tPA or urokinase.
Some cases where there is an extensive clot mechanical thrombectomy is used in
conjunction with thrombolysis, which has been shown to have greater success with removal
of the residual clot. There are multiple mechanical thrombectomy devices that can be used.
One commonly used is pharmacomechanical peripheral thrombectomy device which
consists of high velocity and pressure saline jet to dissolve and aspirate the clot.
Catheter-directed thrombolysis delivers the thrombolytic directly within the vessel at the
thrombus which may be a safe alternative compared to systemic thrombolysis. It is typically
performed from a femoral or popliteal approach, depending on the extent of the clot. There
is a catheter with multiple holes for infusion, it is carefully positioned within the thrombus,
and the thrombolytic medication is infused when the catheter is in the proper position.
Lytic therapies may require longer lysis times, meaning that normally following direct
thrombolysis the clot is reevaluated in 12 to 24 hours with venography and if needed the
direct thrombolysis can be performed again. Normally this can occur due to the size of the
thrombus originally. An extensive thrombus may require multiple treatments.

Go to the next page if you knew the correct answer, or click the link image(s) below to further
research the concepts in this question (if desired).

Research Concepts:
Inferior Vena Caval Thrombosis

We update eBooks quarterly and Apps daily based on user feedback. Please tap flag to
report any questions that need improvement.
Question 25: A 56-year-old man with congestive heart failure presents to the office for his
regular blood transfusion for beta-thalassemia intermedia. His vital signs are significant only for
a pulse of 96/min. He has had well-controlled diabetes mellitus type 2 for the last 12 years. He
weighs 180 lbs (82 kg) and appears in no apparent distress. Which of the following is the most
appropriate rate of transfusion for this patient?

Choices:
1. 2 mL/kg per hour
2. 7 mL/kg per hour
3. 10 mL/kg per hour
4. 15 mL/kg per hour
Answer: 1 - 2 mL/kg per hour
Explanations:
The recommended rate of transfusion for thalassemia patients is 5 mL/kg per hour, and
post-transfusion hemoglobin should not exceed 14 g/dL. But for patients with severe
anemia (Hb less than 5 g/dL) or cardiac compromise, the transfusion rate should be reduced
to 2 mL/kg per hour to avoid fluid overload.
Patients with cardiac comprise are prone to developing fluid overload states. In such
patients, care must be taken in transfusing blood or any fluids. Diuretics can be used to
control a fluid overload state.
Transfuisng at higher rates (more than 5 mL/kg per hour) in this patient is risky and not
recommended.
Transfusing at 10-15 mL/kg per hour will put this patient's heart under extreme stress due to
sudden volume overload.

Go to the next page if you knew the correct answer, or click the link image(s) below to further
research the concepts in this question (if desired).

Research Concepts:
Thalassemia

We update eBooks quarterly and Apps daily based on user feedback. Please tap flag to
report any questions that need improvement.
Question 26: A female was positioned in the dorsal lithotomy position and underwent a
hysterectomy which was marred by bleeding and adhesions. The duration of the surgery was 6
hours, and because of the patient's age and comorbidity, she was extubated and monitored in the
intensive care unit. Within 6 hours of surgery, she complained of her right leg pain. Her blood
pressure was 95/65 mmHg, pulse 110 bpm, respiratory rate 18, and temperature 98.7 F. The leg
did not show any ecchymosis but was slightly swollen. Blood work revealed a potassium of 5.9
mmol/L, creatinine of 2.2 mg/dL, WBC 14 10^9/L, hemoglobin 8 g/dL, sodium 133 mmol/L,
chloride 90 mmol/L, BUN 22 mg/dL, blood glucose 95 mg/dl, creatine kinase of 33,000 units/L
and myoglobinuria. What is the most important step in her management?

Choices:
1. Administer intravenous fluids and blood stat
2. Order a renal consult
3. Duplex ultrasound of the leg
4. Orthopedic surgery consult
Answer: 4 - Orthopedic surgery consult
Explanations:
Acute compartment syndrome is a real complication when patients are placed for prolonged
periods in the lithotomy position. It is stated that when procedures are more than 4 hours,
the risk of compartment syndrome greatly increases.
The diagnosis of acute compartment syndrome is chiefly clinical. No one can agree on
compartment pressures and how to measure them. It is known that the leg does become
tense, and the pressure can be increased, but the key is where to measure the pressure as it
may not be elevated at all locations.
This patient has myoglobinuria and elevated creatine kinase, which indicates muscle
breakdown. This, together with the pain and leg position, is suspicious for compartment
syndrome.
While one can rule out a deep vein thrombus, more important is to call an orthopedic
surgeon to assess the patient for a bedside fasciotomy. Any delay in management and
irreversible injury can occur to the leg, leading to an amputation. The patient should also be
treated with intravenous fluids, and a renal consult would be appropriate.

Go to the next page if you knew the correct answer, or click the link image(s) below to further
research the concepts in this question (if desired).

Research Concepts:
Acute Compartment Syndrome

We update eBooks quarterly and Apps daily based on user feedback. Please tap flag to
report any questions that need improvement.
Question 27: A 39-year-old female was admitted to the hospital after she sustained
extensive third-degree burn to her lower limbs and abdomen 4 days ago. She developed acute
kidney injury and worsening respiratory distress that required mechanical ventilation. Chest
radiograph showed bilateral pulmonary infiltrates. Respiratory rate is set at 14 breaths per
minute, but she is breathing at a rate of 28 breaths per minute. Initial arterial blood gas showed
ph 7.55, PaCO2 28 and PaO2 55 on 70% FiO2, positive end-expiratory pressure (PEEP) of 5 cm
water, and a tidal volume of 10 ml per kg of ideal body weight. What is the next step in the
management of this patient?

Choices:
1. Decrease respiratory rate to 10 and increase FiO2 to 100%
2. Start deep sedation and neuromuscular blockade, increase the PEEP to 10 and set the tidal
volume to 6 ml per kg
3. Increase FiO2 to 100% and repeat ABGs in one hour
4. Sedate the patient deeply and reassess after 30 minutes
Answer: 2 - Start deep sedation and neuromuscular blockade, increase the PEEP to 10 and
set the tidal volume to 6 ml per kg

Explanations:
The patient meets the criteria for acute respiratory distress syndrome (ARDS) as she has
developed acute respiratory symptoms within one week of clinical insult (4 days after
severe, extensive burn) with radiographic features of bilateral patchy infiltrates and
hypoxemia (PaO2/FiO2 ratio 78.5) that are not fully explained by cardiogenic pulmonary
edema.
The patient should be started on low tidal volume ventilation with a target plateau pressure
less than 30 mmHg, a ventilation strategy that is associated with decreased mortality and
improved clinical outcomes.
Deep sedation and neuromuscular blockade are used in the early stage of severe ARDS to
ensure safe, low-pressure ventilation and to avoid further lung injury especially in patients
who are hard to control and are not synchronizing with the mechanical ventilation.
Increasing FiO2 alone without increasing the PEEP is not enough in this case as higher
positive end-expiratory pressure (PEEP) is recommended to prevent alveolar collapse
(atelectrauma) and increase the tidal volume participating in the gas exchange. Furthermore,
decreasing respiratory volume or deeply sedating patients are not appropriate in this patient
with severe ARDS.

Go to the next page if you knew the correct answer, or click the link image(s) below to further
research the concepts in this question (if desired).

Research Concepts:
Respiratory Distress Syndrome

We update eBooks quarterly and Apps daily based on user feedback. Please tap flag to
report any questions that need improvement.
Question 28: A 65-year-old male with a history of alcoholic cirrhosis presents to the
hospital with abdominal pain. His physical exam is significant for ascites. He also reports a
decrease in his urine output for the past 4-5 days. His vitals reveal a blood pressure of 94/62
mmHg and pulse 89/min. Labs are significant for serum creatinine 2.7 mg/dL (from normal
baseline one week ago). Urine analysis is negative for casts or proteinuria. Renal ultrasound is
negative for hydronephrosis. Paracentesis is suggestive of spontaneous bacterial peritonitis, and
he is started on ceftriaxone. He receives 2 L of normal saline and over 200 g of albumin in the
next 48 hours without improvement of his serum creatinine or urine output. What is the most
likely etiology of his kidney injury?

Choices:
1. Dehydration
2. Type 1 hepatorenal syndrome
3. Type 2 hepatorenal syndrome
4. Ischemic acute tubular necrosis
Answer: 2 - Type 1 hepatorenal syndrome
Explanations:
This patient likely has type 1 hepatorenal syndrome which is characterized by a rapid and
progressive decline in renal function defined by doubling of the serum creatinine to at least
more than 2.5 mg/dL or a decrease in the creatinine clearance by half or more over two
weeks.
This is associated with a urine output of less than 500 mL/day. It usually has a bland urine
sediment with no or minimal proteinuria. Most patients with cirrhosis who present with an
acute kidney injury are dehydrated and require stopping the diuretics and hydration.
If the kidney function does not improve after at least 48 hours of volume resuscitation with
albumin (1g/kg per day) and stopping the diuretics; given other etiologies of kidney injury
are ruled out, the diagnosis of hepatorenal syndrome is made.
Type 2 typically involves less severe kidney injury, and patients ordinarily present with
diuretic-resistant ascites. This patient received fluid resuscitation without improvement
ruling out dehydration. His urine analysis was bland without any casts or proteinuria
making acute tubular necrosis less likely.

Go to the next page if you knew the correct answer, or click the link image(s) below to further
research the concepts in this question (if desired).

Research Concepts:
Hepatorenal Syndrome

We update eBooks quarterly and Apps daily based on user feedback. Please tap flag to
report any questions that need improvement.
Question 29: A 78-year-old thin female is undergoing an operation for insertion of a
tunneled central venous catheter with a subcutaneous port. Initial venous access is unsuccessful
after two attempts at left subclavian vein access. On the third attempt, the anesthesiologist states
that the patient's oxygen saturation has acutely dropped to 88%. Heart rate is 101 bpm, and blood
pressure is 101/83 mmHg. Which of the following is the next best step in the management of this
patient?

Choices:
1. Switch to left internal jugular vein access as the patient is not tolerating left subclavian
intervention
2. Abort the procedure and obtain a chest x-ray and place a thoracostomy tube if pneumothorax
is present
3. Position the patient in the left lateral decubitus position with Trendelenberg
4. Intubate, start chest compressions and obtain an automated external defibrillator
Answer: 2 - Abort the procedure and obtain a chest x-ray and place a thoracostomy tube if
pneumothorax is present

Explanations:
Pneumothorax is a complication of both subclavian and internal jugular venous access and
should be discussed as a risk while obtaining surgical consent.
If there is a concern for an air embolism, immediately position the patient in the left lateral
decubitus position and Trendelenberg to prevent the air bubble from traveling to the
pulmonary arteries. If venous access is present, apply negative pressure via syringe to try to
retrieve and suck the air bubble out. The subclavian vein in this patient was not successfully
cannulated, so air embolism is unlikely.
Although ensuring an airway is an essential first step, chest compressions are not indicated
in a patient with a pulse.
An internal jugular vein is an option for tunneled central venous access in case of
unsuccessful subclavian cannulation. However, if there is a concern for pneumothorax, the
procedure should be aborted, and an immediate chest x-ray should be done. Subclavian vein
cannulation has a higher risk of pneumothorax as compared to the internal jugular vein.

Go to the next page if you knew the correct answer, or click the link image(s) below to further
research the concepts in this question (if desired).

Research Concepts:
Vascular Tunneled Central Catheter Access

We update eBooks quarterly and Apps daily based on user feedback. Please tap flag to
report any questions that need improvement.
Question 30: A middle-aged obese male presents with sudden onset of excruciating chest
pain that is now radiating into his back. He has woken up from his sleep 3 hours ago. In the
emergency department, he is diaphoretic, anxious, and complaining of chest pain. He is
administered nitroglycerin by the triage nurse but this does not provide any relief. Blood pressure
on the right arm is 190/90 mmHg and on the left arm, it is 145/75 mmHg. His respiration rate is
28/minute and he is afebrile. His initial set of cardiac enzymes is negative and the ECG reveals
non-specific ST changes. In this patient what will most likely hear be heard during auscultation?

Choices:
1. Crescendo-decrescendo systolic murmur
2. Loud P2
3. Systolic ejection click
4. High pitched early diastolic murmur
Answer: 4 - High pitched early diastolic murmur
Explanations:
Aortic dissection is a serious disorder that can be rapidly fatal if the diagnosis is missed.
The patient may present with excruciating pain that is often unresponsive to nitroglycerin.
The physical may reveal unequal pulses, bounding pulses, diastolic murmur, abdominal
pain, anxiety, fever, or neurological deficit.
If the aortic dissection progresses retrograde it can tear into the aortic annulus and cause
aortic regurgitation.
While the initial chest x-ray is often normal, one may see signs of acute heart failure or
elevated jugular venous pressure in some patients.

Go to the next page if you knew the correct answer, or click the link image(s) below to further
research the concepts in this question (if desired).

Research Concepts:
Aortic Dissection

We update eBooks quarterly and Apps daily based on user feedback. Please tap flag to
report any questions that need improvement.
Question 31: A 45-year-old healthy female patient presents to the emergency department
after a snake bite from a coral snake 4 hours before arrival. The patient is currently
asymptomatic, vital signs are all within normal limits and her physical exam is normal. She is
requesting to be discharged home. What is the best recommendation?

Choices:
1. Explain to the patient that she is safe for discharge home at this time, any significant effects
should have been noticed by now
2. Suggest that she enjoy some cheese, crackers, and a soda while being observed in the
emergency department for an additional 2 hours before being discharged home
3. Proceed immediately to endotracheal intubation and treatment with North American Coral
Snake Antivenin (NACSA)
4. Recommend hospital admission as respiratory failure and neurotoxic effects may be delayed
up to 13 hours in coral snake bites
Answer: 4 - Recommend hospital admission as respiratory failure and neurotoxic effects
may be delayed up to 13 hours in coral snake bites

Explanations:
Coral snake bites may cause delayed systemic neurotoxicity up to 13 hours after the initial
bite.
Patients bitten by a coral snake should be admitted and observed in an inpatient setting for
any development of respiratory muscle weakness. Observation in the emergency department
only is not recommended for coral snake bites.
Pre-emptive intuition is not required for all patients with coral snake bites. Patients who
develop symptoms of respiratory weakness and or failure may require additional support via
intubation and mechanical ventilation, but not all patients will develop these symptoms.
Neurotoxic effects of coral snake envenomation may develop gradually, starting with mild
weakness or sensory deficit that eventually progresses to full paralysis.

Go to the next page if you knew the correct answer, or click the link image(s) below to further
research the concepts in this question (if desired).

Research Concepts:
Coral Snake Toxicity

We update eBooks quarterly and Apps daily based on user feedback. Please tap flag to
report any questions that need improvement.
Question 32: A 35-year-old previously healthy female has had headaches for a few days
and her family reports behavior changes. She has been more talkative, outgoing, and flirtatious
with strangers. After she was sexually inappropriate with a store clerk her family brought her to
her healthcare provider. The only finding was a low-grade fever. MRI was ordered urgently and
showed an inferomedial temporal lobe lesion. Lumbar puncture is done. What treatment is
indicated?

Choices:
1. Corticosteroids
2. Ceftriaxone and vancomycin
3. Acyclovir
4. Antipsychotic medication
Answer: 3 - Acyclovir
Explanations:
The patient has viral encephalitis most likely secondary to herpes simplex virus type 1.
The lumbar puncture polymerase chain reaction (PCR) will most likely be positive but
treatment with acyclovir should be started before the results are available.
The patient should be monitored for seizures and increased intracranial pressure.
Even with early treatment, a significant number of patients have residual neurologic deficits
that are significant.

Go to the next page if you knew the correct answer, or click the link image(s) below to further
research the concepts in this question (if desired).

Research Concepts:
Viral Encephalitis

We update eBooks quarterly and Apps daily based on user feedback. Please tap flag to
report any questions that need improvement.
Question 33: A 75-year-old male is brought to the emergency department with confusion.
Two months ago he presented with acute right-sided weakness and new-onset atrial fibrillation.
He was diagnosed with a cerebrovascular accident and started on warfarin. CT head obtained in
the emergency room today shows intracranial hemorrhage. INR is 4.0. Which of the following is
the next best step in management?

Choices:
1. Administer four-factor prothrombin complex concentrate and vitamin K
2. Administer fresh frozen plasma (FFP) and vitamin K
3. Administer vitamin K alone
4. Administer FFP alone
Answer: 2 - Administer fresh frozen plasma (FFP) and vitamin K
Explanations:
This patient with warfarin-associated intracranial hemorrhage needs the reversal of
anticoagulation.
PCC is the first choice in patients with warfarin-associated major bleed. However, there is
risk of prothrombotic complications with PCC. Patients with history of myocardial
infarction, stroke or transient ischemic attack, disseminated intravascular coagulation within
the last three months were excluded from clinical trials of PCC.
FFP with vitamin K is preferred since he had a stroke within the last three months.
Vitamin K should be coadministered to sustain the reversal of anticoagulation.

Go to the next page if you knew the correct answer, or click the link image(s) below to further
research the concepts in this question (if desired).

Research Concepts:
Warfarin Toxicity

We update eBooks quarterly and Apps daily based on user feedback. Please tap flag to
report any questions that need improvement.
Question 34: A 65-year-old male with COPD is in the emergency department with dyspnea,
productive cough, and fatigue. Vital signs show a heart rate of 124/min, blood pressure 158/94
mmHg, respiratory rate 32/min, and SpO2 90% on 6 L nasal cannula. Physical examination is
notable for diminished breath sounds bilaterally with scant end-expiratory wheezes. He is given
non-invasive ventilation via a full face for two hours. Bronchodilators, antibiotics, and steroids
are prescribed. Current ventilator settings are PSV mode, IPAP 20 cmH2O, EPAP 8 cmH2O,
FiO2 50%. Now vital signs are heart rate 130/min, blood pressure 160/96 mmHg, respiratory rate
30/min, and SpO2 94%. He is alert and answers questions but appears in moderate distress and
complains of dyspnea. Tidal volumes returned to the ventilator measure between 250 and 350
mL. ABG shows pH 7.25, pCO2 68 mmHg, and pO2 65 mmHg. What is the next best step in the
management of this patient?

Choices:
1. Increase EPAP to 10 cmH2O and FiO2 to 75%; keep IPAP at 20 cmH2O
2. Perform endotracheal intubation and set the ventilator to a volume control mode
3. Perform endotracheal intubation and set the ventilator to a PSV mode
4. Increase IPAP to 28 cmH20 and FiO2 to 75%; keep EPAP at 8 cmH2O
Answer: 2 - Perform endotracheal intubation and set the ventilator to a volume control mode
Explanations:
PSV is not used as an initial mode of ventilation for intubated patients due to respiratory
depression following sedation given during intubation.
High airway resistance in patients with obstructive lung disease limits peak flow and can
result in small tidal volumes.
Work of breathing and thus oxygen consumption is higher in PSV than in control modes of
ventilation.
Tidal volumes in non-invasive PSV ventilation can be increased by increasing drive
pressure, but this may be limited by high airway resistance, mask leak, or air-trapping.

Go to the next page if you knew the correct answer, or click the link image(s) below to further
research the concepts in this question (if desired).

Research Concepts:
Pressure Support Ventilation

We update eBooks quarterly and Apps daily based on user feedback. Please tap flag to
report any questions that need improvement.
Question 35: A 59-year-old obese male presents to the emergency department with a 4-hour
history of sudden onset of sharp radiating chest pain. The pain he says started in the chest and is
now radiating to his arm and lower back. He feels nauseated and feels as if he is going to die.
Quick exam reveals a distressed male with right arm blood pressure of 210/110 mmHg and a left
arm blood pressure of 140/82 mmHg. He is diaphoretic and extremely anxious. The ECG is
unremarkable and the initial set of cardiac enzymes is negative. His pain is not responding to
nitroglycerin. In patients with this pathology, what one feature will be seen on the chest x-ray?

Choices:
1. Enlarged aortic knob
2. Displacement of trachea to the right
3. Calcified aortic knob
4. Double aortic contour
Answer: 4 - Double aortic contour
Explanations:
The chest x-ray in patients with aortic dissection can be normal or show a number of
features of which the most common is a widened mediastinum.
Other features include a double or irregular aortic contour.
There can be an inward displacement of the atherosclerotic calcification.
If the patient is stable, a CT scan of the chest is the next study of choice.

Go to the next page if you knew the correct answer, or click the link image(s) below to further
research the concepts in this question (if desired).

Research Concepts:
Aortic Dissection

We update eBooks quarterly and Apps daily based on user feedback. Please tap flag to
report any questions that need improvement.
Question 36: A 45-year-old male comes to the emergency department in August
complaining of dizziness, shortness of breath and palpitations. He reports having developed a red
rash with central clearing and successive flu-like symptoms one month ago after having gone
hiking with a group of friends in Pennsylvania. He took over the counter acetaminophen and
symptoms subsided after five days with no other therapy. Physical examination is unremarkable
except for a heart rate of 35 beats per minute. An electrocardiogram showed dissociation
between P waves and QRS complex. What is the next step in the management of this patient?

Choices:
1. Admission, initiate treatment with oral doxycycline and telemetry monitoring
2. Admission, initiate treatment with intravenous ceftriaxone and telemetry monitoring
3. Admission, initiate treatment with intravenous ceftriaxone and temporary pacemaker
placement
4. Admission, initiate treatment with intravenous ceftriaxone and permanent pacemaker
placement
Answer: 3 - Admission, initiate treatment with intravenous ceftriaxone and temporary
pacemaker placement

Explanations:
Pacemaker placement is indicated in patients with Lyme carditis and high second degree or
third-degree atrioventricular (AV) block.
Intravenous antibiotics are preferred for moderate to severe disease.
Need for permanent pacemaker has rarely been reported in patients with Lyme carditis and
atrioventricular block. Most of the time atrioventricular block resolves after some days of
treatment with antibiotics.
It is safe to remove pacemaker once second or third degree AV block has resolved.

Go to the next page if you knew the correct answer, or click the link image(s) below to further
research the concepts in this question (if desired).

Research Concepts:
Lyme Carditis

We update eBooks quarterly and Apps daily based on user feedback. Please tap flag to
report any questions that need improvement.
Question 37: A 75-year male presents to the emergency department with left facial, arm,
and leg numbness and tingling. He has a past medical history significant for type 2 diabetes
mellitus, essential hypertension, prostate cancer, cataracts, and two prior myocardial infarctions 5
and 8 years ago. He has smoked two packs of cigarettes daily for the past 40 years. He takes
metformin, atorvastatin, lisinopril, tamsulosin and undergoes radiation therapy every two weeks.
He has been compliant with his medications and check-ups. His vital signs are 38 C, blood
pressure 140/85 mmHg, respiratory rate 12/minute, and 98% oxygen saturation on 2 L nasal
cannula oxygen. The patient is alert and oriented to person, place, and time. He can draw a clock
fully with the time. The neurologic exam is significant for decreased pinprick, fine touch, and
vibration sensation in his left face, arm, and leg. Visual fields are full bilaterally. Strength is 5/5
throughout, and reflexes are 1+ bilaterally in his upper and lower extremities. His cardiac exam
shows a mild 1/6 systolic murmur in the right upper sternal border, and his pulmonary exam
shows an increased inspiratory and expiratory effort with bilateral expiratory wheezes in all lung
fields. Head CT scan is negative. Chest x-ray shows a flattened diaphragm and expanded lung
fields bilaterally. Where is the location of his lacunar syndrome and what is the artery infarcted?

Choices:
1. Right parietal lobe and middle cerebral artery
2. Left internal capsule and lenticulostriate artery
3. Right pons and perforating pontine arteries
4. Right thalamus and the thalamoperforant arteries
Answer: 4 - Right thalamus and the thalamoperforant arteries
Explanations:
The anatomic distribution of lacunar syndromes and infarctions is most commonly the basal
ganglia, the pons, and the subcortical white matter structures.
These anatomical sites correspond to lesions at the lenticulostriate arteries, the anterior
choroidal artery, thalamoperforant arteries, paramedian branches of the basilar artery, and
the recurrent artery of Heubner from the anterior cerebral artery.
In a pure sensory stroke, the patient presents with unilateral numbness of the face, arm, and
leg without cortical signs or motor deficits. All sensory modalities will be impaired.
A lesion at the contralateral thalamus, supplied by the thalamoperforant arteries, is the most
common cause of a pure sensory stroke.

Go to the next page if you knew the correct answer, or click the link image(s) below to further
research the concepts in this question (if desired).

Research Concepts:
Lacunar Syndromes

We update eBooks quarterly and Apps daily based on user feedback. Please tap flag to
report any questions that need improvement.
Question 38: A 65-year-old man is scheduled to undergo a radical prostatectomy for
prostate adenocarcinoma. During surgery, there is significant bleeding that becomes difficult to
control. Immediate blood work reveals hemoglobin of 9 g/dL, platelets of 200,000/mm3,
prothrombin time of 11.2 seconds, activated partial thromboplastin time of 41 seconds, and
elevated levels of fibrinogen degradation products. The patient was started on intravenous
aminocaproic acid to control the bleeding. Which of the following best describes the mechanism
of action of aminocaproic acid?

Choices:
1. Accelerates the conversion of prothrombin to thrombin
2. Binds to antithrombin and inactivates thrombin and factor Xa
3. Blocks the conversion of plasminogen to plasmin
4. Accelerates the conversion of fibrinogen to fibrin
Answer: 3 - Blocks the conversion of plasminogen to plasmin
Explanations:
The formation of a blood clot, also known as coagulation, starts immediately after injury to
a blood vessel occurs.
Platelets are activated and form a platelet plug at the site of the blood vessel injury. The
platelet plug is further stabilized by a fibrin mesh, a process that is facilitated by proteins
called coagulation factors.
Stabilization of the platelet plug with a fibrin mesh seals the site of blood vessel injury and
stops the bleeding. Plasminogen is an enzyme in the blood that gets converted into plasmin
by tissue plasminogen activator. Plasmin then works to degrade the fibrin mesh stabilizing
the platelet plug, releasing fibrin degradation products into the blood. Degradation of the
fibrin mesh will disrupt the platelet plug and lead to bleeding.
Aminocaproic acid stops bleeding by inhibiting the conversion of plasminogen into
plasmin, thus preventing degradation of the fibrin mesh stabilizing the platelet plug. On the
contrary, thrombolytics, such as alteplase, works by activating tissue plasminogen activator
(tPA), which will convert plasminogen into plasmin, which will degrade the fibrin mesh and
lead to bleeding.

Go to the next page if you knew the correct answer, or click the link image(s) below to further
research the concepts in this question (if desired).

Research Concepts:
Aminocaproic Acid

We update eBooks quarterly and Apps daily based on user feedback. Please tap flag to
report any questions that need improvement.
Question 39: A 70-year-old male, with left community-acquired pneumonia and left pleural
effusion being treated with ceftriaxone and azithromycin, undergoes diagnostic thoracentesis
under ultrasound guidance for his left-sided pleural effusion. The pleural fluid is turbid, but no
frank pus is seen. His pleural fluid pH is less than 7.2, with pleural glucose concentration normal.
Which of the following interventions is most appropriate for this patient?

Choices:
1. Continue current antibiotic regimen
2. Broaden antibiotic regimen
3. Perform tube thoracostomy and switch azithromycin to metronidazole
4. Perform tube thoracostomy and inject intrapleural fibrinolytics
Answer: 3 - Perform tube thoracostomy and switch azithromycin to metronidazole
Explanations:
Parapneumonic effusions require diagnostic thoracocentesis to further evaluate its etiology
and to guide therapeutic strategies.
Aspiration of frank pus from the pleural space during thoracocentesis invariably needs
surgical drainage.
However, if there is uncertainty about whether a turbid fluid is infected, a pH of less than
7.2 measured via a blood gas analyzer warrants an invasive procedure for drainage.
Anaerobic coverage should be initiated in any patient with suspected empyema, and
appropriate antibiotics should not be delayed pending culture data. Failure to treat empyema
appropriately with urgent drainage and adequate antibiotic coverage can lead to poor patient
outcomes.

Go to the next page if you knew the correct answer, or click the link image(s) below to further
research the concepts in this question (if desired).

Research Concepts:
Thoracic Empyema

We update eBooks quarterly and Apps daily based on user feedback. Please tap flag to
report any questions that need improvement.
Question 40: A 75-year-old male is diagnosed with multilobar pneumonia and is intubated
in the emergency department. A ventilator is placed on assist control mode with respiratory rate
of 22, tidal volume of 6 ml/kg, FIO2 of 24%, and PEEP of 10 cmH2O. An arterial blood gas
shows pH=7.21, PCO2=34 mmHg, and PO2=56 mmHg. Which of the following is the most
likely cause of the continued hypoxemia?

Choices:
1. Shunting
2. Hypoventilation
3. Heart failure
4. Ventilation-perfusion mismatch
Answer: 1 - Shunting
Explanations:
The patient has shunting due to alveoli that receive perfusion but cannot exchange gas.
A ventilation-perfusion mismatch most commonly occurs with pulmonary embolism.
The low PCO2 indicates that there is not hypoventilation.
The patient is acidotic not from respiratory sources but rather metabolic.

Go to the next page if you knew the correct answer, or click the link image(s) below to further
research the concepts in this question (if desired).

Research Concepts:
Bacterial Pneumonia

We update eBooks quarterly and Apps daily based on user feedback. Please tap flag to
report any questions that need improvement.
Question 41: A 62-year-old patient has an acute anterior myocardial infarction and
develops a new holosystolic murmur and dyspnea. Vital signs are blood pressure 90/52 mmHg,
heart rate 120 beats/min, and respiratory rate 20. Exam shows jugular venous distention, rales at
the lung bases, and cool extremities with edema. What is the best initial test?

Choices:
1. Chest x-ray
2. Echocardiography
3. Left and right cardiac catheterization
4. CT of the heart
Answer: 2 - Echocardiography
Explanations:
Echocardiography will most likely detect either papillary muscle or ventricular septal
rupture.
This can be done urgently at the bedside.
A chest x-ray is not helpful in finding the exact cause of congestive heart failure in a post-
myocardial infarction patient who has developed a mechanical complication.
Cardiac catheterization with coronary angiography will aid in diagnosis and management
but is not the first test for making the diagnosis. Echocardiography remains the initial test of
choice.

Go to the next page if you knew the correct answer, or click the link image(s) below to further
research the concepts in this question (if desired).

Research Concepts:
Acute Myocardial Infarction

We update eBooks quarterly and Apps daily based on user feedback. Please tap flag to
report any questions that need improvement.
Question 42: A 33-year-old African-American female with a history of intravenous drug
use and alcohol use presented to the emergency department with eye discharge, painful eye
movement, eye swelling on the right side and double vision for two days. The patient reported
having a fall two days ago, but she was not able to remember the details. Vitals showed blood
pressure 160/95 mmHg, heart rate 105 bpm, and temperature 37.5 C. She is alert and oriented x3.
There is chemosis, periorbital edema, proptosis, mild restriction of all extraocular movements of
the right eye and mild drooping of the right eyelid. The visual acuity was 20/20 normal on both
eyes. A bruit was noticed over the right orbit. Pupillary reflexes were normal. Fundus exam
showed mildly dilated vessels but no papilledema. There was a small bruise on the right side of
the face on the maxillary area. The rest of the exam was unremarkable. Basic blood workup
came back unremarkable. CT head without contrast showed a small maxillary fracture, proptosis,
and orbital edema on the right side. What are the most likely diagnosis and the best evaluation
for the definitive diagnosis?

Choices:
1. Cavernous sinus tumor so perform a biopsy
2. Cavernous sinus thrombosis so there is no need for further tests. Start stat treatment with
broad-spectrum IV antibiotics including antifungal and anticoagulation
3. Carotid cavernous aneurysm so check intraocular pressure from right eye STAT
4. Carotid cavernous fistula so order conventional digital subtraction angiography
Answer: 4 - Carotid cavernous fistula so order conventional digital subtraction angiography
Explanations:
The most common symptoms on carotid-cavernous fistula (CCF) are ocular bruit
auscultated over the globe in high-flow CCFs, proptosis, chemosis, and conjunctival
injection, ocular and/or orbital pain, headache, diplopia, and blurry vision.
Vascular etiologies can be seen on CTA, MRI, MRA, and orbital or transcranial ultrasound
but conventional digital subtraction angiography is the gold-standard test for diagnosis of
CCF.
CCFs may be an indirect (low-flow), or a direct (high-flow) CCFs based on a connection
between the cavernous sinus and the intracavernous carotid artery versus the internal or
external carotid artery, respectively.
The management of CCFs depends on the classification of CCFs, the onset of symptoms
and the risk of long-term neurological impairment. Twenty to sixty percent of indirect CCFs
will close spontaneously. Direct CCFs should be closed by transarterial/transvenous
embolization or other treatment options if they are symptomatic and at risk of progression
with attendant morbidity.

Go to the next page if you knew the correct answer, or click the link image(s) below to further
research the concepts in this question (if desired).

Research Concepts:
Cavernous Sinus Syndromes

We update eBooks quarterly and Apps daily based on user feedback. Please tap flag to
report any questions that need improvement.
Question 43: A 52-year-old man with a history of migraines, cocaine abuse, schizophrenia,
and hypertension is transferred to a tertiary care stroke center from a community hospital with a
cerebellar stroke. He received tissue plasminogen activator at the community hospital with
improvement in his symptoms while en route. On the third day of admission, he complains of a
worsening headache associated with nausea and vomiting. He locates the aching, severe pain in
the middle of his head. It is improved when can keep the lights in his hospital room off. He asks
for you to give him diphenhydramine and metoclopramide, which he says always relieve his
migraines at home. His blood pressure is 151/87 mmHg, and his heart rate is 87 bpm. An
admission EKG had a QTc of 416 ms. He has horizontal gaze-evoked nystagmus. What should
be done next?

Choices:
1. Order diphenhydramine and metoclopramide
2. Order ibuprofen and ondansetron
3. CT scan of the head
4. MRI of the head and neck
Answer: 3 - CT scan of the head
Explanations:
Given this patient’s recent diagnosis of a cerebellar stroke, his presentation is concerning
for worsening of his infarct potentially due to hemorrhagic conversion. He is at higher risk
for this having just received thrombolysis. This could be due to reactive edema or spreading
infarct.
Emergent CT imaging is indicated to rapidly assess any changes that may require
neurosurgical intervention.
MRI would be helpful to visualize the evolution of the infarct but is slow, and CT will give
most of the immediately useful information it provides.
Symptomatic care may be helpful, but a migraine in this patient would be a diagnosis of
exclusion. Giving an NSAID like ibuprofen with some antiplatelet activity would be
contraindicated.

Go to the next page if you knew the correct answer, or click the link image(s) below to further
research the concepts in this question (if desired).

Research Concepts:
Cerebellar Infarct

We update eBooks quarterly and Apps daily based on user feedback. Please tap flag to
report any questions that need improvement.
Question 44: A patient was brought in by her family for "being sleepy." During the
interview, she is drowsy and reports diarrhea, vomiting, weakness and frequent urination. On
exam, her hands shake mildly. Her medication includes HCTZ, verapamil, metformin, and
lithium. What is the best treatment for this probable overdose?

Choices:
1. Hemodialysis
2. Activated charcoal
3. D-Penicillamine
4. Ipecac
Answer: 1 - Hemodialysis
Explanations:
Lithium toxicity is difficult to treat and may present with vomiting, diarrhea, drowsiness,
muscular weakness, and lack of muscle coordination.
When the lithium levels are high, it is important to discontinue the medication. The best
way to deal with an overdose is hemodialysis.
Use of diuretics or activated charcoal usually does not help.
Ipecac should never be given as there is a risk of aspiration. Ipecac is also associated with a
cardiomyopathy in children.

Go to the next page if you knew the correct answer, or click the link image(s) below to further
research the concepts in this question (if desired).

Research Concepts:
Lithium Toxicity

We update eBooks quarterly and Apps daily based on user feedback. Please tap flag to
report any questions that need improvement.
Question 45: A 17-year old female from Connecticut presents with a 7-day history of fever
and chills. In the last 48 hours, she has also developed malaise, anorexia, and muscle pain. Her
physical exam is unremarkable except for a few petechiae. She was hiking in the woods recently,
where there were many ticks and insects. Testing for Lyme disease is negative, but the infectious
disease expert feels that she may have been bitten by the deer tick "Ixodes dammini." Which
other disease, besides Lyme, can be caused by this same tick?

Choices:
1. Babesiosis
2. Rocky mountain spotted fever
3. Typhoid fever
4. Q-fever
Answer: 1 - Babesiosis
Explanations:
Babesiosis is a zoonotic infection transmitted by the deer tick, Ixodes dammini.
The Ixodes tick is much smaller than the tick that causes Rocky Mountain Spotted Fever.
Babesiosis also occurs in the same geographical location as Lyme disease.
It is often mistaken for malaria and somewhat difficult to diagnose. This is because the
infection has no specific signs or symptoms.

Go to the next page if you knew the correct answer, or click the link image(s) below to further
research the concepts in this question (if desired).

Research Concepts:
Babesiosis

We update eBooks quarterly and Apps daily based on user feedback. Please tap flag to
report any questions that need improvement.
Question 46: A 17-year-old patient starts a new drug and now presents with fever, muscle
rigidity, altered mental status, and signs of autonomic instability. Treatment is started with
antipyretics, intravenous hydration, and dantrolene. What is the most likely drug involved in this
adverse reaction?

Choices:
1. Chlorpromazine
2. Diazepam
3. Phenytoin
4. Levodopa
Answer: 1 - Chlorpromazine
Explanations:
Chlorpromazine, a neuroleptic drug, can cause neuroleptic malignant syndrome (NMS).
NMS is a rare but potentially life-threatening reaction seen with the administration of
certain drugs.
NMS is most often associated with the administration of older neuroleptics, such as
haloperidol and chlorpromazine.
Less commonly, NMS can be precipitated by any of the antipsychotic agents and some non-
neuroleptic drugs such as lithium, metoclopramide, and amoxapine.
Although serious side effects can be seen with SSRIs, phenytoin, and levodopa, they are not
associated with NMS, even at toxic levels.

Go to the next page if you knew the correct answer, or click the link image(s) below to further
research the concepts in this question (if desired).

Research Concepts:
Neuroleptic Malignant Syndrome

We update eBooks quarterly and Apps daily based on user feedback. Please tap flag to
report any questions that need improvement.
Question 47: A 34-year-old male presents to the emergency department following a burn
injury sustained on chest and arms after accidental exposure to gas leaking vent at home. He
does not have any facial injury but has 2nd and 3rd-degree burn injury involving the upper chest
and both arms. Initial vitals show a temperature of 100 F, a heart rate of 100/min, respiratory rate
of 22/min, and mean arterial pressure (MAP) of 70 mmHg. Initial chest x-ray shows basal
atelectasis. Blood work shows acute leukocytosis with a WBC count of 16000/microL. He is
admitted and started on the appropriate course of management. He continues to improve until
day 3 when he spikes a fever again with increased shortness of breath and increased tan-colored
sputum. Vitals now show a fever of 102 F with a heart rate of 120/min, respiratory rate 26/min,
and a MAP of 65 mmHg. It is difficult to appreciate any new consolidation in chest X-ray with
preexisting basal atelectasis. Which combination of cytokines is most likely responsible for an
infectious complication resulting from immunosuppression in this scenario?

Choices:
1. IL-4 and IL-10
2. IL-6 and IL-8
3. IL-1 and TNF-alfa
4. Activated Protein C and Plasminogen Activator Inhibitor -1
Answer: 1 - IL-4 and IL-10
Explanations:
The proinflammatory state of SIRS can give way to a compensatory anti-inflammatory
response syndrome (CARS) when the body is in a state of immune dysregulation, relative
immunosuppression and therefore susceptible to nosocomial infections.
Compensatory anti-inflammatory response is mediated by interleukins IL-4 and IL-10.
They inhibit the production of TNF-alpha, IL-1, IL-6, and IL-8.
The individual thus becomes susceptible to nosocomial infection, which can thus reinitiate
the septic cascade.

Go to the next page if you knew the correct answer, or click the link image(s) below to further
research the concepts in this question (if desired).

Research Concepts:
Systemic Inflammatory Response Syndrome

We update eBooks quarterly and Apps daily based on user feedback. Please tap flag to
report any questions that need improvement.
Question 48: A 64-year-old male presents to the emergency department with chief
complaints of fever for 3 days, cough with sputum production, and chest pain for 2 days. His
vitals include a temperature of 103 F, respiratory rate of 35/min, pulse 84/min, and blood
pressure 104/70 mmHg. He is admitted for inpatient treatment and is started on empiric
antibiotic coverage based on the local resistance patterns. On the second day, his vitals continue
to deteriorate, and an arterial blood gas analysis shows a partial pressure of oxygen to be 54
mmHg. He is transferred to the intensive care unit and is ventilated with supplemental oxygen
therapy of 50% oxygen at a tidal volume of 7 ml/kg, and positive end-expiratory pressure of 10
mmHg. His condition improves, and his oxygen saturation is maintained at 89%. On the third
day, his fever recurs, and his oxygen saturation drops despite being on high FiO2. Chest
radiography shows new onset infiltrates. What is the best next step in the management of this
patient?

Choices:
1. CT scan of the chest
2. Continue empiric antibiotic therapy
3. Blood cultures
4. Mini bronchoalveolar lavage (Mini- BAL)
Answer: 4 - Mini bronchoalveolar lavage (Mini- BAL)
Explanations:
Ventilator-associated pneumonia (VAP) is any pneumonia observed 48 hours after being on
a ventilator in a hospital setting. VAP should be suspected in ventilated patients with new-
onset dyspnea, falling oxygen saturation, or new onset of lung infiltrates.
Management of VAP, unlike the management of community- or hospital-acquired
pneumonia, involves both radiological evidence and invasive sampling before the
administration of antibiotics. After the culture and sensitivity reports of bronchoalveolar
lavage are available, the antibiotic regimen should be tailored appropriately.
Mini-BAL is done by inserting a catheter through the endotracheal tube, instilling normal
saline, and aspirating the sample.
Mini-BAL is a less invasive, less expensive, and less-cumbersome technique than BAL.

Go to the next page if you knew the correct answer, or click the link image(s) below to further
research the concepts in this question (if desired).

Research Concepts:
Ventilator-associated Pneumonia

We update eBooks quarterly and Apps daily based on user feedback. Please tap flag to
report any questions that need improvement.
Question 49: A 17-year-old male with AIDS presents with worsening exertional dyspnea,
fever, and a nonproductive cough. His temperature is 102 F, heart rate 120 beats/min, and
respiratory rate 24. A physical exam reveals mild crackles and rhonchi bilaterally. A chest x-ray
shows diffuse bilateral infiltrates. PaO2 is 69 mmHg on room air. He has no known drug
allergies. What is the best initial treatment?

Choices:
1. Intravenous trimethoprim/sulfamethoxazole
2. Intravenous pentamidine
3. Intravenous corticosteroids followed by IV trimethoprim/sulfamethoxazole
4. Intravenous corticosteroids followed by IV levofloxacin
Answer: 3 - Intravenous corticosteroids followed by IV trimethoprim/sulfamethoxazole
Explanations:
Corticosteroids decrease mortality in patients with severe Pneumocystis jiroveci pneumonia
(PCP).
Trimethoprim/sulfamethoxazole is the drug of choice, but pentamidine is used for those
with sulfa allergy.
Levofloxacin does not cover PCP. However, if there are infiltrates, it can be added for
possible bacterial pneumonia.
Adjunctive corticosteroids can decrease the inflammatory response associated with PCP.
Also, they can reduce the decline of oxygenation and reduce the incidence of respiratory
failure.

Go to the next page if you knew the correct answer, or click the link image(s) below to further
research the concepts in this question (if desired).

Research Concepts:
Pneumocystis Jirovecii Pneumonia

We update eBooks quarterly and Apps daily based on user feedback. Please tap flag to
report any questions that need improvement.
Question 50: A 72-year-old white man with prostate cancer treated ten years ago with
resection who presents with the complaint of double vision and not able to completely open the
eyelid on the right side over the past two months. The patient denies any other focal weakness,
sensory loss or difficulty in breathing, eating or coughing. Vitals are normal. The neurological
exam showed restriction of right eye abduction, right side ptosis with miosis. The fundal exam is
unremarkable. Miosis is not changed after cocaine eye drop to the right eye but left eye dilated.
Basic serum blood workup and CT head are unremarkable. Which of the following structures are
involved to have these symptoms?

Choices:
1. Trochlear nerve and ophthalmic nerve
2. Abducens nerve and sympathetic fibers
3. Oculomotor nerve and sympathetic fibers
4. Maxillary Nerve and abducens nerve
Answer: 2 - Abducens nerve and sympathetic fibers
Explanations:
If anisocoria increases and one eye shows minimal dilation with cocaine eye drop, it
indicates Horner syndrome.
On examination, he had a right abduction deficit consistent with an abducens nerve palsy.
The combination of an abducens nerve palsy and a Horner syndrome indicated a cavernous
sinus localization. The history of slow progression over months indicated a compressive or
infiltrative lesion.
A biopsy of the cavernous sinus lesion is rarely needed for diagnosis because of the
proximity to critical neurologic structures makes difficult for any procedure like biopsy or
excision.

Go to the next page if you knew the correct answer, or click the link image(s) below to further
research the concepts in this question (if desired).

Research Concepts:
Cavernous Sinus Syndromes

We update eBooks quarterly and Apps daily based on user feedback. Please tap flag to
report any questions that need improvement.
Question 51: An 85-year-old female is brought to the emergency department for confusion.
Her family reports she has become depressed and is talking to her long-dead husband. She
complains of abdominal, back, and hip pain with constipation. The neurological exam is non-
focal. The cardiopulmonary exam is unremarkable. There is a 3-centimeter, fixed breast mass.
What is the most likely ECG finding?

Choices:
1. Flattened T waves
2. Prolonged QT interval
3. Shortened QT interval
4. Peaked T waves
Answer: 3 - Shortened QT interval
Explanations:
Given the fixed mass and symptoms, the patient is presenting with paraneoplastic
hypercalcemia secondary to metastatic breast cancer.
Hypercalcemia results in a shortened QT interval. Rarely, it can cause an arrhythmia.
Hypercalcemia also can cause calcium deposits in the coronary arteries and valves resulting
in increased cardiovascular morbidity.
Hypercalcemia can affect the gastrointestinal system, central nervous system, kidneys, and
musculoskeletal system. Symptoms may include abdominal pain, constipation, confusion,
depression, nephrolithiasis, polyuria, weakness, and myopathy.
The mnemonic for hypercalcemia is stones, bones, abdominal groans, and psychiatric
overtones.

Go to the next page if you knew the correct answer, or click the link image(s) below to further
research the concepts in this question (if desired).

Research Concepts:
Malignancy-Related Hypercalcemia

We update eBooks quarterly and Apps daily based on user feedback. Please tap flag to
report any questions that need improvement.
Question 52: A 36-year-old female presents to the emergency department with complaints
of productive cough, malaise, and fever. She has no known medical history and does not take
any medications. Her blood pressure is 105/92 mmHg, temperature 101.3 F, respiratory rate
24/min, and heart rate 96/min. Her BMI is 40 kg/m2. This patient's pulse oximetry is 93% on
room air. Examination shows jugular venous distention, clear lung fields. A portable chest x-ray
shows low lung volumes but is unremarkable otherwise. Laboratory results show hemoglobin of
15 g/l, platelets 290000/microL, and leukocytes 5000/microL. His sodium is 142 mEq/L,
potassium 4.3 mEq/L, chloride 99 mEq/L, bicarbonate 35 mEq/L, BUN 26 mg/dL, creatinine 1.0
mg/dL, and glucose 100 mg/dL. Which of the following is the best next step in managing this
patient?

Choices:
1. CT chest
2. Arterial blood gas
3. Acetazolamide
4. Ventilation/ perfusion scan
Answer: 2 - Arterial blood gas
Explanations:
The patient came to the emergency room with upper respiratory symptoms. She likely has
underlying obesity hypoventilation syndrome. Her serum bicarbonate is elevated.
An ABG on room air should be done on any patient with suspected OHS to look up for
daytime hypercapnia.
With her respiratory symptoms, it's possible she may not be well compensated.
An ABG will help determine if this patient needs positive pressure.

Go to the next page if you knew the correct answer, or click the link image(s) below to further
research the concepts in this question (if desired).

Research Concepts:
Obesity-Hypoventilation Syndrome

We update eBooks quarterly and Apps daily based on user feedback. Please tap flag to
report any questions that need improvement.
Question 53: A 48-year-old female with past medical history of B cell lymphoma colitis on
pembrolizumab presents with bilateral perianal abscesses. Patients states she was here 1 week
ago with the same and now increase in pain and erythema. What is the appropriate treatment for
this patient?

Choices:
1. Incision and drainage with a surgeon at the bedside
2. Addition of ciprofloxacin for recurrence of infection.
3. Obtain an endoanal ultrasound
4. Obtain blood cultures, start intravenous antibiotics and admit to the hospital for imaging and
possible surgical drainage
Answer: 4 - Obtain blood cultures, start intravenous antibiotics and admit to the hospital for
imaging and possible surgical drainage

Explanations:
Patients with recurrent or bilateral abscesses should be evaluated by surgery for
intraoperative drainage, including cases where internal draining may be necessary.
Ciprofloxacin should be combined with metronidazole for appropriate antibiotic coverage.
CT should be obtained in patients with significant comorbidities, complex suppurative
anorectal conditions, and when surgical consultation is not immediately available.
Patients who are immunocompromised with bilateral and recurrent abscesses should be
started on intravenous antibiotics after cultures are obtained and have a prompt surgical
evaluation for intraoperative drainage.

Go to the next page if you knew the correct answer, or click the link image(s) below to further
research the concepts in this question (if desired).

Research Concepts:
Perirectal Abscess

We update eBooks quarterly and Apps daily based on user feedback. Please tap flag to
report any questions that need improvement.
Question 54: A 67-year-old thin female has recurrent abdominal pain every time she eats.
She has lost 9 pounds (4 kg) in weight over the past 2 months. Examination reveals a soft
epigastric systolic abdominal murmur and her aorta is palpable but not pulsatile or enlarged.
What is her most likely diagnosis?

Choices:
1. Abdominal aortic aneurysm
2. Portal hypertension
3. Colon cancer
4. Mesenteric ischemia
Answer: 4 - Mesenteric ischemia
Explanations:
Patients with mesenteric ischemia have a very typical presentation. However, the diagnosis
is often missed because of vague complaints and signs.
Patients may present with a history of postprandial pain, typically starting 20 minutes after a
meal, which may last up to 90 minutes.
Because of the pain, they develop food fear and experience subsequent weight loss. Patients
may be severely malnourished upon presentation.
Chronic mesenteric ischemia usually results from long-standing atherosclerotic disease of 2
or more mesenteric vessels. Symptoms are caused by a progressive decrease in blood flow.
In normal people, there is a 15% increase in blood flow during food consumption.

Go to the next page if you knew the correct answer, or click the link image(s) below to further
research the concepts in this question (if desired).

Research Concepts:
Chronic Mesenteric Ischemia

We update eBooks quarterly and Apps daily based on user feedback. Please tap flag to
report any questions that need improvement.
Question 55: A 75-year-old male with a past medical history of diabetes mellitus type 2,
hypertension, peripheral vascular disease, hypothyroidism, and rheumatoid arthritis presents with
significant substernal chest pain and diaphoresis after acute emotional stress. His
electrocardiogram reveals anterior ST-segment elevations. His cardiac troponin levels are
slightly elevated. A coronary angiogram reveals 40% stenosis of the proximal right coronary
artery. The left anterior descending artery is wrapped around the apex of the heart. Left
ventriculography reveals apical ballooning in the mid, distal, and apical segments, and a
hypercontractile basal segment. What is the treatment for this disorder?

Choices:
1. Intravenous fluids
2. Inotropic agents
3. Beta-1 blockers, aspirin, angiotensin-converting-enzyme inhibitors, intravenous diuretics, and
direct thrombin inhibitors
4. Percutaneous transluminal coronary angioplasty
Answer: 3 - Beta-1 blockers, aspirin, angiotensin-converting-enzyme inhibitors, intravenous
diuretics, and direct thrombin inhibitors

Explanations:
If Takotsubo cardiomyopathy is diagnosed and there is no comorbidity in the patient,
medical treatment is supportive. This patient has cardiovascular and other system
comorbidities. Therefore, he should be treated initially as someone presenting with acute
coronary syndrome or myocardial infarction.
If the patient is hypotensive, pressors may be required as well as an intra-aortic balloon
pump and left ventricular assist device. Management should include continuous ECG
monitoring. Echocardiography should be recommended at approximately 4 to 6 weeks post-
presentation to document normalization of left ventricular function.
The patient should be monitored closely if receiving inotropic agents because these agents
can exacerbate symptoms and result in a dynamic left ventricular mid-cavity obstruction.
If a dynamic intraventricular pressure gradient develops, discontinue the inotropic agent and
administer intravenous beta-blockers to increase diastolic filling time and left ventricular
end-diastolic volume.

Go to the next page if you knew the correct answer, or click the link image(s) below to further
research the concepts in this question (if desired).

Research Concepts:
Takotsubo Cardiomyopathy

We update eBooks quarterly and Apps daily based on user feedback. Please tap flag to
report any questions that need improvement.
Question 56: A 52-year-old male presents to the office with three months of right-sided
chest pains, productive cough, and shortness of breath with associated low-grade fevers, chills,
and 10 lbs unintentional weight loss. He has no medical problems that he is aware of, does not
have a primary care provider, has never had any surgeries other than his wisdom teeth removed,
denies any sick contacts, refuses all vaccinations, does not drink any alcohol or use any tobacco
products or illicit drugs. He does report an extensive travel history in the Middle East and
Northern Africa over the last three years as a military contractor. He reports that he got home
about two weeks ago from Iraq, but his symptoms started while he was there and had been
getting progressively worse. Vital signs are temperature 38.2 C, heart rate 98 bpm, respiratory
rate 24/minute, blood pressure 122/78 mmHg, and oxygen saturation 92% on room air. He
appears acutely ill, no acute distress, and is normocephalic and atraumatic. Sclera are nonicteric,
and pupils equal round and reactive to light and accommodation. The neck nontender is with no
jugular venous distention. He is mildly tachycardic with regular rhythm and no murmur. Lungs
are clear to auscultation bilaterally. The abdomen is soft, nontender with normal bowel sounds.
There is no costovertebral angle tenderness or bladder distension. He has normal speech with no
neurologic deficits. His chemistry panel shows normal electrolytes, normal renal function, and
normal hepatic. His complete blood count shows mild leukocytosis with a WBC count of
12.6/microL and his differential 2.2% eosinophils. Chest x-ray shows there is a small cavitary
lung lesion in the right upper lobe. CT scan of the chest shows a 2 cm x 3 cm ring-enhancing
cavitary lung abscess in the right upper lobe with an air-fluid level. There is no hilar/mediastinal
lymphadenopathy, and there are no other lesions. The patient is diagnosed with sepsis secondary
to a lung abscess and admitted to the medical floor, and started on broad-spectrum antibiotics
with IV vancomycin and ampicillin-sulbactam. Over the next three days, the patient continues to
have low-grade fevers, and his cough continues. He undergoes bronchoscopy and
bronchoalveolar lavage on hospital day four. Cytology of the fluid is negative for malignancy but
does note a high preponderance of eosinophils and trematode eggs. His blood and sputum
cultures remain negative since admission. Serological antigen testing reveals that the eggs belong
to Fasciola giganticum. What is the next best step?

Choices:
1. Left upper lobectomy
2. Triclabendazole
3. Ivermectin and praziquantel
4. Diethylcarbamazine
Answer: 2 - Triclabendazole
Explanations:
The treatment for fasciola infection is oral triclabendazole 10 mg/kg daily taken after a meal
for two days. It is typically very well tolerated and effective.
Fasciola reportedly does not respond well to praziquantel, mebendazole, or albendazole.
Ivermectin is the drug of choice for strongyloidiasis and onchocerciasis but has some
activity against filarial worms. Praziquantel has activity against intestinal tapeworms,
schistosomes, neurocysticercosis, and other flukes besides fasciola.
Diethylcarbamazine has activity against loiasis, visceral larva migrans, and lymphatic
filariasis but does not have any activity against fasciola.

Go to the next page if you knew the correct answer, or click the link image(s) below to further
research the concepts in this question (if desired).

Research Concepts:
Fascioliasis

We update eBooks quarterly and Apps daily based on user feedback. Please tap flag to
report any questions that need improvement.
Question 57: A 30-year-old, previously good health, non-smoker, triathlete is helped out of
the water due to extreme exhaustion and dyspnea. While being examined, he begins coughing up
blood-tinged sputum. He is transported to the local emergency room on 100% oxygen due to
oxygen saturation of 91%. All other vitals are stable. What is expected on chest x-ray when he
arrives at the hospital?

Choices:
1. Cardiomegaly
2. Unilateral pleural effusion
3. Widened mediastinum
4. Kerley B lines
Answer: 4 - Kerley B lines
Explanations:
While there is some component of heart failure in immersion pulmonary edema (IPE) it
may or may not be significant enough to show up on chest x-ray.
Pleural effusions may be seen with IPE but it would be bilateral.
While there is a redistribution of blood centrally during submersion which may lead to
increased diameter of the inferior vena cava, it may not be radiographically significant, and
there is a rapid redistribution after removal from the water.
Kerley B lines are indicative of pulmonary edema.

Go to the next page if you knew the correct answer, or click the link image(s) below to further
research the concepts in this question (if desired).

Research Concepts:
Immersion Pulmonary Edema

We update eBooks quarterly and Apps daily based on user feedback. Please tap flag to
report any questions that need improvement.
Question 58: A 78-year-old female, history of coronary artery disease, hypertension,
diabetes mellitus, and hypothyroidism, presents to the emergency department with two days of
fever, headache, and neck stiffness. Her physical examination is significant for fever of 101.1 F
and heart rate of 110 bpm. She has a stiff neck, photophobia, and nuchal rigidity. Laboratory
values are significant for leukocytosis. Lumbar puncture is pending. What is the most
appropriate empiric regimen of antibiotics?

Choices:
1. Vancomycin and amoxicillin
2. Ceftriaxone and amoxicillin-clavulanic acid
3. Vancomycin, ceftriaxone, and ampicillin
4. Ceftriaxone and Ampicillin
Answer: 3 - Vancomycin, ceftriaxone, and ampicillin
Explanations:
Prompt initiation of antibiotics is critical in the management of suspected bacterial
meningitis, which has been shown to improve long-term outcomes.
Ampicillin is an important consideration in the empiric regimen in patients over the age of
50 and in neonates, given the increased prevalence of Listeria monocytogenes.
Ceftriaxone has excellent blood-brain barrier penetration and covers most organisms that
typically cause meningitis, such as Streptococcus pneumoniae, Neisseria meningitidis, and
Haemophilus influenzae.
The reason for coverage with vancomycin is less for methicilline resistant staphylococcus,
but rather for coverage of penicillin resistant S. pneumoniae

Go to the next page if you knew the correct answer, or click the link image(s) below to further
research the concepts in this question (if desired).

Research Concepts:
Bacterial Meningitis

We update eBooks quarterly and Apps daily based on user feedback. Please tap flag to
report any questions that need improvement.
Question 59: A 38-year-old male presents to the emergency department after a 3-day
history of right-sided neck pain and headache. He has no significant past medical history. His
headache started after a day at an amusement park where he rode several roller coasters. He had
taken aspirin without relief but became concerned when he had a 10-minute episode of confusion
associated with left-hand numbness and clumsiness. An exam shows mild, right ptosis,
anisocoria with the right pupil being smaller, and left arm drift. What is the most likely
diagnosis?

Choices:
1. Right vertebral artery dissection
2. Right internal carotid dissection
3. Hemiplegic migraine
4. Paroxysmal hemicrania
Answer: 2 - Right internal carotid dissection
Explanations:
The patient has Horner syndrome, neck pain, headache, and transient ischemic symptoms
on the right.
The most probable diagnosis is right internal carotid dissection.
The dissection could be spontaneous but was more likely secondary to neck movements on
the roller coasters.
Treatment is with anticoagulation or antiplatelet medication, but angioplasty and stent may
be needed if there are persistent ischemic symptoms.

Go to the next page if you knew the correct answer, or click the link image(s) below to further
research the concepts in this question (if desired).

Research Concepts:
Carotid Artery Dissection

We update eBooks quarterly and Apps daily based on user feedback. Please tap flag to
report any questions that need improvement.
Question 60: A 65-year-old female underwent transfemoral coronary angioplasty with stent
placement. 3 hours after the procedure, she is having hypotension and low back pain. Patient
denies any chest pain or dyspnea. On physical exam, heart sounds are within normal limits with
no jugular venous distention, and lungs are clear to auscultation. The dressing at femoral entry
site is clear. What is the underlying cause of patient’s hypotension?

Choices:
1. Acute myocardial infarction
2. Acute stent thrombosis
3. Retroperitoneal hemorrhage
4. Coronary artery perforation
Answer: 3 - Retroperitoneal hemorrhage
Explanations:
Retroperitoneal hemorrhage (RPH) is an infrequent but fatal complication of transfemoral
percutaneous coronary intervention.
RPH present with hypotension with or without back pain. Severe hypotension or shock may
need an urgent blood transfusion.
Female gender is associated with an increased risk for RPH. Females have smaller diameter
femoral arteries than men which may make access more difficult. This difference can lead
to a greater likelihood of multiple and posterior arterial wall punctures, thus leading to RPH.
A higher femoral arterial puncture site, such as above the inguinal ligament or above the
middle one-third of the femoral head on fluoroscopy is an important procedure-related risk
factor for RPH.

Go to the next page if you knew the correct answer, or click the link image(s) below to further
research the concepts in this question (if desired).

Research Concepts:
Angioplasty

We update eBooks quarterly and Apps daily based on user feedback. Please tap flag to
report any questions that need improvement.
Question 61: A 37-year-old patient with alcohol use disorder presents with 2 weeks of
stomach pain and fatigue. Physical findings are confusion, jaundice, normal bowel sounds, right
upper quadrant tenderness without rebound or guarding, liver edge 7 cm below the costal margin
at the midclavicular line, splenomegaly, and asterixis. Labs show a marked white blood cell
count without left shift and elevated transaminases. The patient has a calculated Maddrey
discriminant function of 48. An ultrasound of the liver shows hepatomegaly, increased
echogenicity, gallstones, and no ascites. A chest x-ray and urinalysis are normal. Which of the
following is false?

Choices:
1. This presentation could reflect concomitant hepatitis C infection
2. Lactulose therapy is indicated
3. This patient can be managed as an outpatient
4. Morbidity and mortality can be decreased by IV methylprednisolone
Answer: 3 - This patient can be managed as an outpatient
Explanations:
This patient's severe acute alcoholic hepatitis must be managed as an inpatient. A Maddrey
discriminant function score above 32 suggests increased disease severity and mortality.
Encephalopathy is an indication for lactulose therapy.
Concomitant hepatitis C infection increases morbidity synergistically.
This patient's 30-day mortality rate of 30% to 50% can be reduced with IV corticosteroids.

Go to the next page if you knew the correct answer, or click the link image(s) below to further
research the concepts in this question (if desired).

Research Concepts:
Alcoholic Hepatitis

We update eBooks quarterly and Apps daily based on user feedback. Please tap flag to
report any questions that need improvement.
Question 62: A 55-year-old male with a history of chronic obstructive pulmonary disease
and bronchiectasis, a current everyday smoker with a 50 pack-year history, was brought into the
emergency department due to unresponsiveness. His wife confirms that the patient was not
feeling well for the past four days with an upper respiratory viral infection. His symptoms
included generalized malaise, fevers, shortness of breath, and cough. His home medications
include fluticasone and salmeterol twice a day and albuterol as needed for shortness of breath.
He went to urgent care two days ago and was prescribed azithromycin, prednisone, albuterol, and
ipratropium nebulization which only provided temporary relief. In the emergency department,
vital signs show a blood pressure of 140/72 mmHg, heart rate of 98/min, respiratory rate of
10/min, temperature 38.3 C, and pulse oximetry of 88% on room air. On physical examination,
the patient is noted to be lethargic, mildly responsive to painful stimuli, and has decreased air
movement on auscultation of his chest. Arterial blood gas (ABG) was drawn, and the results are
as follows, pH 7.02, pO2 55 mmHg, pCO2 100 mmHg. Chest X-ray showed bilateral multifocal
infiltrates. The patient was endotracheally intubated and placed on mechanical ventilation. Initial
settings were a respiratory rate of 20/min, tidal volume 500 m, a fraction of inspired oxygen 1.0,
and positive end-expiratory pressure of 5 cm H20. He is started on inhaled bronchodilators,
intravenous methylprednisolone, broad-spectrum antibiotics, and oseltamivir. He becomes
asynchronous with the ventilator requiring sedation and later neuromuscular blockade. His repeat
ABG shows a pH of 6.95, pO2 70 mmHg, pCO2 120 mmHg, and he is currently becoming
hypotensive. Due to difficulty with ventilation, the patient is placed on a venovenous
extracorporeal membrane oxygenator (VV-ECMO). The patient is cannulated for VV-ECMO.
The inflow cannula is inserted in the right femoral vein and outflow cannula in the right internal
jugular vein with flows of 3 liters per minute, sweep four, and fraction of inspired oxygen 1.0.
ABG on these settings show pH 7.20, pO2 50 mmHg, pCO2 70 mmHg. What is the next best
step in management?

Choices:
1. Start the patient on inhaled nitric oxide
2. Increase the dose and frequency of intravenous steroids
3. Order a stat chest ultrasound
4. Increase flow to 4 liters/minute
Answer: 3 - Order a stat chest ultrasound
Explanations:
It is possible to perform VV-ECMO cannulation blindly, but it is recommended to cannulate
for ECMO under direct visualization, usually with fluoroscopy or ultrasound, to confirm
appropriate placement of the inflow and outflow cannulas.
In the double VV-ECMO cannulation, there are two ways to insert the cannulas. The inflow
cannula is usually inserted in the femoral vein and advanced to the junction of the inferior
vena cava (IVC), and the right atrium and the outflow cannula can be inserted into the right
internal jugular vein and advanced through the superior vena cava to the right atrium. The
second approach is to insert the inflow cannula into the femoral vein and advanced to the
mid-IVC, and the outflow cannula can be inserted in the contralateral femoral vein with the
tip ending in the right atrium.
The correct placement of the cannulas is important to avoid the recirculation phenomenon.
This is when the oxygenated blood that is being returned to the patient is drained by the
inflow cannula before being circulated through the body. This is usually due to the two
cannulas being in close proximity.
The flow rate, the fraction of inspired oxygen, the amount of recirculation of blood will all
affect the ability of the VV-ECMO circuit to oxygenate the blood. Increasing the flow when
pO2 is low is appropriate, but the placement of the cannulas should always be confirmed
after cannulation.

Go to the next page if you knew the correct answer, or click the link image(s) below to further
research the concepts in this question (if desired).

Research Concepts:
Extracorporeal Membrane Oxygenation In Adults

We update eBooks quarterly and Apps daily based on user feedback. Please tap flag to
report any questions that need improvement.
Question 63: A 47-year-old male is brought to the emergency department for 2 days of
fever, headache, and altered mental status. His wife reports that he has been confused and
forgetful since yesterday. The patient is an accountant with no significant medical history. He
has had no sick contacts and has not traveled recently. He does not use tobacco or illicit drugs
but drinks alcohol occasionally. Temperature is 38 C, blood pressure is 126/84 mmHg, and pulse
is 72/minute. Mucous membranes are moist with no lesions. No lymphadenopathy is present.
Lungs are clear to auscultation, and heart sounds are normal with no murmur. The abdomen is
soft and nontender, with no hepatosplenomegaly. He has nuchal rigidity and is alert and oriented
only to self. He has no motor or sensory deficits but can only follow simple commands.
Cerebrospinal fluid analysis shows a white blood cell count of 150/mm3, predominantly
lymphocytes, red blood cell count of 8/mm^3, protein concentration of 130 mg/dL, and glucose
concentration of 70 mg/dL. Which of the following is the best next step in management?

Choices:
1. Administer intravenous acyclovir
2. Obtain an MRI
3. Consult neurointerventional radiology
4. Administer intravenous vancomycin and ceftriaxone
Answer: 1 - Administer intravenous acyclovir
Explanations:
The patient’s presenting history, exam, and cerebrospinal fluid findings are consistent with
viral encephalitis. Red blood cells may be present in the cerebrospinal fluid of patients with
herpes simplex virus (HSV) encephalitis. Acyclovir should be started empirically before
further confirmatory testing.
Acyclovir has been shown to significantly decrease the mortality and morbidity of HSV
encephalitis if started early.
Acyclovir limits the severity of long-term behavioral and cognitive impairment seen with
HSV encephalitis.
The intravenous route has been shown to have greater efficacy than the oral form.

Go to the next page if you knew the correct answer, or click the link image(s) below to further
research the concepts in this question (if desired).

Research Concepts:
Viral Encephalitis

We update eBooks quarterly and Apps daily based on user feedback. Please tap flag to
report any questions that need improvement.
Question 64: A novel waste anesthetic gas disposal system is designed, with active capture
and closed features on the receiving element. The manufacturer’s intended use is to minimize
pollution and optimize resource recycling by attaching it in line with the ventilatory circuit,
between the Y-piece and the patient. A 61-year-old 59 kg male undergoes laparoscopic
cholecystectomy for acute calculous cholecystitis refractory to conservative therapy. His past
medical history is notable for uncontrolled hypertension, COPD, and active tobacco use. The
patient is induced with intravenous agents, and after uneventful intubation, he is mechanically
ventilated. Insufflation and appropriate positioning are achieved, requiring minimal ventilator
adjustments to maintain oxygenation. As the gallbladder is being dissected off of the liver, the
anesthesia alarms indicate high airway pressures. Vital signs show a blood pressure 82/40
mmHg, heart rate 118/min, and SpO2 87%. Which of the following features of the waste
anesthetic gas disposal system could have most likely prevented this patient’s pathology from
developing?

Choices:
1. Longer connection tubing
2. Presence of a pressure relief valve
3. Smaller-diameter tubing
4. Passive system mechanics
Answer: 2 - Presence of a pressure relief valve
Explanations:
Pressure relief valves are important safety features that minimize the potential for causing
barotrauma.
The novel system described here both bypasses the native disposal system and has an active
scavenging method, which utilizes pressure to achieve effective collection.
This combination in a patient with underlying pulmonary disease raises concern for tension
pneumothorax in the setting of compromised pulmonary compliance during a laparoscopic
procedure.
Longer connection tubing would not effectively reduce the potential for causing barotrauma
leading to tension pneumothorax. Resistance to flow increases as the radius of the conduit
decreases. This could potentially worsen already-elevated airway pressures in an
abdominally-insufflated patient. Although passive waste anesthetic gas disposal systems do
not utilize artificially-generated pressure to collect volatile agents, the patient’s underlying
comorbidities would likely be better served by a pressure-limiting valve.

Go to the next page if you knew the correct answer, or click the link image(s) below to further
research the concepts in this question (if desired).

Research Concepts:
Waste Gas Scavenging System

We update eBooks quarterly and Apps daily based on user feedback. Please tap flag to
report any questions that need improvement.
Question 65: A 25-year-old female has had a 4-month history of substernal chest pain and
dyspnea on exertion. Electrocardiogram shows right axis deviation. An arterial blood gas shows
pH 7.46, PO2 80 mmHg, and PCO2 32 mmHg. A chest x-ray shows enlarged pulmonary arteries
but no infiltrates. A spiral CT shows subsegmental defects not consistent with pulmonary
embolism. Echocardiogram shows no primary cardiac disease but there is right heart strain.
Which of the following would be the most appropriate next step in evaluation?

Choices:
1. Ventilation perfusion scan
2. Bronchoscopy
3. Right heart catheterization
4. Antineutrophil cytoplasmic antibody
Answer: 3 - Right heart catheterization
Explanations:
The patient most likely has primary pulmonary hypertension. Features on a chest x-ray
include large central pulmonary arteries, right ventricular hypertrophy, and clear lung fields.
An ECG typically shows right ventricular hypertrophy with right atrial enlargement, right
axis deviation, and increased amplitude of P waves due to right atrial enlargement (lead II).
Echocardiography is the most sensitive test, which helps assess right ventricular size and
pressure-volume overload. It also gives an estimate of pulmonary artery pressure. In
primary pulmonary hypertension, there is right atrial and ventricular enlargement and
tricuspid regurgitation. An arterial blood gas may reveal an increased A-a gradient and
hypoxia. Pulmonary function tests will show an impaired diffusing capacity of the lungs for
carbon monoxide (DLCO).
Right heart catheterization is the criterion standard diagnostic test to detect elevated
pressures and confirm pulmonary hypertension. Once diagnosed, additional studies should
be performed to evaluate for the etiology of pulmonary hypertension.
Causes of pulmonary hypertension include thromboembolism, chronic obstructive
pulmonary disease, obstructive sleep apnea, and heart disease.
In those from undeveloped countries, infections with filariasis and schistosomiasis should
be considered.

Go to the next page if you knew the correct answer, or click the link image(s) below to further
research the concepts in this question (if desired).

Research Concepts:
Idiopathic Pulmonary Artery Hypertension

We update eBooks quarterly and Apps daily based on user feedback. Please tap flag to
report any questions that need improvement.
Question 66: A 65-year-old male with a history of hypertension on multiple medications
with good compliance, progressively worsening chronic kidney disease (over the last 12 months,
creatinine increasing from 1.2 to 2.0 mg/dL, eGFR declining from 65 to 35 mL/min/m2) presents
to the emergency department with sudden onset dyspnea which started an hour prior to
presentation when he was asleep. He also reports an increased frequency of headaches over the
last 12 months. The examination reveals the patient to be in mild distress with tachypnea, blood
pressure of 230/132 mmHg, heart rate of 100/min, and oxygen saturation of 92% on room air
which improved to 98% on 2 L supplemental oxygen by the nasal cannula. Chest x-ray reveals
extensive pulmonary edema. Laboratory data reveals hemoglobin of 11 mg/dL, and creatinine
2.1 mg/dL. N terminal pro-brain-natriuretic peptide (NT-proBNP) is 1054 ng/L. EKG shows
normal sinus rhythm with left ventricular hypertrophic changes. A bedside echocardiogram
reveals normal ejection fraction with moderate left ventricular hypertrophy with no
hemodynamically significant valvular lesions. The patient was started on intravenous
nitroglycerin infusion, given IV furosemide, and transferred to the intensive care monitoring
(ICU) for close monitoring. Coronary angiography is performed, which shows nonobstructive
coronary disease. What is the next step to prevent further occurrences of this patient's pulmonary
edema?

Choices:
1. Change furosemide to a long-acting loop diuretic like torsemide
2. Renal sympathetic denervation
3. Renal angiography and endovascular stenting
4. Initiate therapy with mineralocorticoid receptor antagonist like spironolactone
Answer: 3 - Renal angiography and endovascular stenting
Explanations:
This patient has flash pulmonary edema secondary to renal artery stenosis. Renal
angiography is the gold-standard test to diagnose renal artery atherosclerosis and further,
stenosis.
Percutaneous renal artery stenting should be strongly considered in patients with labile
hypertension, progressively worsening renal function, and flash pulmonary edema.
A careful selection of patients for renal artery stenting must be conducted as some patients
can continue to have a rapid decline in renal function due to atheroembolic disease after
angioplasty.
Along with consideration of renal angioplasty, the patient's hypertension should be tightly
controlled to prevent further recurrences of flash pulmonary edema. The initiation of a long-
acting loop diuretic, as well as mineralocorticoid receptor antagonists, can be considered to
manage hypertension. Renal denervation therapy has not shown statistically significant
improvement in systolic blood pressure and is currently not utilized for the treatment of
resistant hypertension.

Go to the next page if you knew the correct answer, or click the link image(s) below to further
research the concepts in this question (if desired).

Research Concepts:
Pulmonary Edema

We update eBooks quarterly and Apps daily based on user feedback. Please tap flag to
report any questions that need improvement.
Question 67: A 56-year-old man is found with altered mental status by his wife. His history
is notable for back pain for which he takes opioid pain medication as well as clonidine for which
he takes for hypertension. No evidence of trauma. He is brought to the emergency department
and found to have miotic pupils, decreased respiratory status, bradycardia, and hypotension. In
addition to getting peripheral access, all pertinent labs including point of care glucose, and
maintaining his airway, what is the next medication that should be administered?

Choices:
1. Norepinephrine
2. Naloxone
3. Atropine
4. Dopamine
Answer: 2 - Naloxone
Explanations:
Naloxone is a very easy drug to give, and emergency departments should have this drug
readily available. Even if it turns out this patient overdosed on his clonidine, naloxone
should be given first as an opioid and alpha agonist overdose present very similarly.
Nonresponse to naloxone can be helpful in diagnosing an alpha agonist overdose.
Norepinephrine is the vasopressor of choice when dealing with a clonidine or alpha agonist
overdose as it will correct both the hypotension and the bradycardia.
The mainstay of treatment when treating an alpha agonist overdose is replacing the
catecholamines that are not being endogenously secreted because of central alpha-2 receptor
activation.

Go to the next page if you knew the correct answer, or click the link image(s) below to further
research the concepts in this question (if desired).

Research Concepts:
Alpha Receptor Agonist Toxicity

We update eBooks quarterly and Apps daily based on user feedback. Please tap flag to
report any questions that need improvement.
Question 68: A 75-year-old man with hypertension, diabetes mellitus, and hyperlipidemia
presents with severe epigastric abdominal pain for three days associated with vomiting. His
lipase is found to be extremely elevated, and CT shows acute necrotizing pancreatitis. Clinicians
find discoloration due to ecchymosis on his flanks. What is the name for this finding?

Choices:
1. Cullen sign
2. Grey Turner sign
3. Bryant sign
4. Fox sign
Answer: 2 - Grey Turner sign
Explanations:
Retroperitoneal or intraabdominal bleeding may track to the subcutaneous tissue and
present as discoloration or ecchymosis of the flanks known as the Grey Turner sign.
Although classically associated with severe acute pancreatitis almost any pathology that
causes intraabdominal or retroperitoneal bleeding may cause the Grey Turner sign.
Researchers have not established the sensitivity and specificity of the Grey Turner sign for
intraabdominal or retroperitoneal bleeding definitively. However, less than 5% of patients
with relatively severe pancreatitis have a Grey Turner sign showing that the sign itself is not
sensitive in detecting pancreatitis.
Other subcutaneous manifestations of intraabdominal or retroperitoneal bleeding include
ecchymosis around the umbilicus (Cullen sign), ecchymosis of the upper thigh just below
the inguinal ligament (Fox sign), and ecchymosis of the scrotum (Bryant sign).

Go to the next page if you knew the correct answer, or click the link image(s) below to further
research the concepts in this question (if desired).

Research Concepts:
Grey-Turner Sign

We update eBooks quarterly and Apps daily based on user feedback. Please tap flag to
report any questions that need improvement.
Question 69: A 71-year-old male reports chest pain and shortness of breath during a right
subclavian central venous catheter placement. After halting the procedure, you discover absent
right-sided lung sounds. His vitals are stable with mild tachypnea. What is the first step in the
management of this patient?

Choices:
1. Perform a needle decompression in the fifth intercostal space of the midclavicular line
2. Emergently place a tube thoracostomy in the seventh intercostal space of the midaxillary line
3. Call for chest radiography to confirm the diagnosis
4. Perform a needle decompression in the fifth intercostal space of the midaxillary line
Answer: 4 - Perform a needle decompression in the fifth intercostal space of the midaxillary
line

Explanations:
In recent studies, needle decompressions have been shown to be more effective in lateral
versus medial approaches in patients with pneumothorax. For this patient, a needle
decompression in the fifth intercostal space of the midaxillary line should be performed.
Needle decompressions are considered temporary measures with final management
consisting of a tube thoracostomy.
Correct tube thoracostomy placement is commonly in the fifth intercostal space of the
midaxillary line.
Iatrogenic pneumothorax is a patient safety indicator (PSI) condition. It is defined as a
traumatic pneumothorax that is caused secondary to an invasive procedure or surgery.

Go to the next page if you knew the correct answer, or click the link image(s) below to further
research the concepts in this question (if desired).

Research Concepts:
Iatrogenic Pneumothorax

We update eBooks quarterly and Apps daily based on user feedback. Please tap flag to
report any questions that need improvement.
Question 70: A 65-year-old female was admitted to the ICU after suffering head trauma and
had eventually a complicated course with acute respiratory distress syndrome that required using
Veno-venous extracorporeal membrane oxygenation (ECMO). A decision was made to perform
an apnea test to assess for brain death. PO2 was 40 mmHg on FiO2 of 100% before connecting
the patient to the ECMO machine. Which of the following is the most appropriate strategy to
maintain oxygenation during the test while allowing the rising of CO2?

Choices:
1. Decrease gas sweep flow rate to 0 L/min, while maintenance of oxygenation by a cannula
connected to the ET tube at 12 L/min
2. Decrease gas sweep flow rate to 0.5 L/min, and disconnect the mechanical ventilator
3. Decrease gas sweep flow rate to 2.0 L/min, while maintenance of oxygenation by a cannula
connected to the ET tube at 12 L/min
4. Decrease gas sweep flow to 1.0 L/min, while maintenance of oxygenation by a CPAP delivery
system at 10 cm H2O
Answer: 4 - Decrease gas sweep flow to 1.0 L/min, while maintenance of oxygenation by a
CPAP delivery system at 10 cm H2O

Explanations:
The gas sweep flow rate could be decreased to the range of (0.5-1.0 L/min) to allow the
rising of CO2 while preventing severe hypoxemia. CPAP delivery system was found to be
more effective than oxygen insufflation by a cannula through the ET tube to maintain
oxygenation during the apnea test for patients on ECMO.
Using ECMO machines isn't a contraindication to perform the apnea test. The apnea test
should be aborted if the patient develops severe hypoxia.
While performing the apnea test, using a CPAP delivery system for oxygenation is found to
be more effective than oxygen insufflation through a cannula especially for patients on
ECMO.
Decreasing the gas sweep flow rate to 0.5-1 L/min is found to be appropriate to avoid
hypoxia while allowing the rise of CO2 for patients on ECMO undergoing the apnea test.

Go to the next page if you knew the correct answer, or click the link image(s) below to further
research the concepts in this question (if desired).

Research Concepts:
Brain Death Criteria

We update eBooks quarterly and Apps daily based on user feedback. Please tap flag to
report any questions that need improvement.
Question 71: A 50-year-old woman survives a near-drowning episode. She is brought to the
emergency department. Initial 12-lead electrocardiogram demonstrates T-wave inversions in the
precordial leads. Her blood pressure is 70/40 mmHg, and the heart rate is 112/minute.
Emergency coronary angiography demonstrates no evidence of epicardial coronary artery
disease. Left ventriculography shows basal hyperkinesis with apical akinesis. She is admitted to
the coronary care unit, given her significant hypotension and tachycardia. Which of the
following vasopressors is most appropriate for this patient?

Choices:
1. Phenylephrine
2. Norepinephrine
3. Epinephrine
4. Dobutamine
Answer: 1 - Phenylephrine
Explanations:
This patient has Takotsubo cardiomyopathy which developed after stressful triggers, in this
case, the near-drowning episode. Furthermore, this patient has developed a complication of
Takotsubo cardiomyopathy, left ventricular outflow tract obstruction (LVOTO), which is
causing her hypotension. Phenylephrine, a pure alpha agonist is an optimal therapy here
because it reduces cardiac contractility and heart rate.
Intravenous beta blocker or calcium channel blocker and volume expansion can also help to
increase left ventricular end diastolic volume and relieve left ventricular outflow tract
obstruction here.
LVOT obstruction is an important complication to recognize in typical Takotsubo
cardiomyopathy which involves basal hyperkinesis with apical akinesis such as in this case.
It may result in hypotension, tachycardia, and cardiogenic shock.
Vasopressors or inotropes such as norepinephrine, epinephrine, and dobutamine will be
counterproductive as it will further increase contractility and worsen the clinical situation.

Go to the next page if you knew the correct answer, or click the link image(s) below to further
research the concepts in this question (if desired).

Research Concepts:
Takotsubo Cardiomyopathy

We update eBooks quarterly and Apps daily based on user feedback. Please tap flag to
report any questions that need improvement.
Question 72: A 52-year-old woman with a history of resistant hypertension presents to the
emergency department with complaints of chest tightness radiating to the left arm, dyspnea,
diaphoresis, and headache for the past 45 minutes. Physical examination reveals faint heart
sounds and a bruit in the right flank. Troponin levels are elevated. EKG shows normal sinus
rhythm with non-specific ST-T wave changes. The patient is admitted for emergent angiography
which reveals type 2 spontaneous coronary artery dissection (SCAD). Which of the following
condition is the most likely predisposing factor for SCAD in this patient?

Choices:
1. Systemic sclerosis
2. Systemic lupus erythematosus
3. Takayasu arteritis
4. Fibromuscular dysplasia
Answer: 4 - Fibromuscular dysplasia
Explanations:
Fibromuscular dysplasia (FMD) is a non-inflammatory and non-atherosclerotic disorder that
leads to arterial stenosis, occlusion, aneurysm, and dissection. It is a common predisposing
factor of spontaneous coronary artery dissection (SCAD).
Suspicion for FMD is high in this patient due to the history of resistant hypertension and the
bruit in the right flank
In patients with SCAD, up to 85% patients will have angiographic evidence of FMD. The
two angiographic subtypes of FMD are multifocal and focal. Multifocal is more common
and gives a "string of beads" appearance on angiography. Focal FMD is less common and
has the angiographic appearance of a "circumferential or tubular stenosis."
In patients presenting with SCAD with high suspicion for FMD, a full-body, cross-sectional
imaging study with CT angiography or MR angiography is recommended.

Go to the next page if you knew the correct answer, or click the link image(s) below to further
research the concepts in this question (if desired).

Research Concepts:
Coronary Artery Dissection

We update eBooks quarterly and Apps daily based on user feedback. Please tap flag to
report any questions that need improvement.
Question 73: A 17-year-old male is shot in the chest during an altercation. Emergency
medical services (EMS) have arrived on the scene and have found the patient to be in cardiac
arrest and begin advanced cardiac life support, including cardiopulmonary resuscitation (CPR).
The emergency medical services crew is 15 minutes away and have been on the scene doing
CPR for the last 10 minutes. The patient now arrives in the trauma bay, and EMS reports the
patient is in asystole with a downtime of approximately 25 minutes with no signs of life and no
return of spontaneous circulation. Which of the following would is the most appropriate
management at this time?

Choices:
1. Give an additional dose of intravenous epinephrine and perform immediate defibrillation
2. Perform an emergency department resuscitative thoracotomy with aortic cross-clamping and
internal cardiac massage
3. Place an echo probe on the patient's chest and if no cardiac motion is identified cease
resuscitative efforts
4. Initiate the massive-transfusion protocol and send emergently to the operating room
Answer: 3 - Place an echo probe on the patient's chest and if no cardiac motion is identified
cease resuscitative efforts

Explanations:
Patients who present in traumatic cardiac arrest are managed differently with respect to the
injury mechanism. Penetrating trauma, which leads to exsanguination, pericardial
tamponade, tension hemothorax, or tension pneumothorax, can be treated if done
expeditiously. The advanced cardiac life support (ACLS) and advanced trauma life support
(ATLS) courses promulgated by the American Heart Association and the American College
of Surgeons teach the 5 H's and T's (hypoxia, hypovolemia, hyperkalemia, hydrogen ions,
hypothermia, tension pneumothorax, thrombosis, toxins, thromboembolism, tamponade) for
potentially reversible causes of asystole. If not identified and treated early and return of
spontaneous circulation is restored, patients have a very dismal prognosis.
A patient who presents in traumatic cardiac arrest from a penetrating wound who arrives
after 20 minutes of known downtime should be declared dead on arrival after an echo probe
is placed on the patient's chest, and no cardiac motion is identified.
Cardiopulmonary resuscitation, when performed correctly, can restore about 25% of a
patient's normal cardiac output. Cerebral hypoxia/anoxia continues until circulation is
restored and data has shown that survival in normothermic traumatic arrest from a
penetrating injury approaches zero after about 20 minutes and resuscitative efforts can be
ceased.
Additional doses of epinephrine may be given to a patient in asystole from a penetrating
injury, but defibrillation is not warranted unless in ventricular fibrillation or ventricular
tachycardia without a pulse. Emergency department resuscitative thoracotomy with aortic
cross-clamping and internal cardiac massage is reserved for patients who have a penetrating
mechanism of injury leading to traumatic cardiac arrest with known downtimes of less than
20 minutes. When this time-frame is exceeded, performing this high-risk procedure is more
of a risk to the resuscitation team than benefit to the patient as the patient's survivability
nears 0%.

Go to the next page if you knew the correct answer, or click the link image(s) below to further
research the concepts in this question (if desired).

Research Concepts:
Asystole

We update eBooks quarterly and Apps daily based on user feedback. Please tap flag to
report any questions that need improvement.
Question 74: A 71-year-old male is brought to the emergency department (ED) by
emergency medical services (EMS) after being found unresponsive at home by family. Upon
discovery, the patient is not displaying spontaneous respiration and is found to be pulseless.
Basic life support protocols were performed by a friend until the arrival of EMS, at which time
advanced cardiac life support protocols (ACLS) were enacted. EMS obtained a history from a
friend who notes that the patient has a medical history of hypertension, diabetes mellitus, and
smoking cigarettes. She notes that the patient was recently discharged after being hospitalized for
chest pain and that during his stay, he underwent cardiac catheterization and had two drug-
eluting stents placed. Upon arrival to the ED, ACLS protocols are continued by ED staff.
Unfortunately, after 40 minutes of ACLS protocols, EMS and ER staff are unable to obtain a
return of spontaneous circulation, and the patient is declared deceased. The family is notified and
is asking what the most likely cause of death was. Which of the following best describes the
cause of death in this patient?

Choices:
1. Silent stent thrombosis
2. Possible stent thrombosis
3. Probable stent thrombosis
4. Definite stent thrombosis
Answer: 3 - Probable stent thrombosis
Explanations:
Any unexplained death within 30 days of stent placement should be considered as probable
stent thrombosis.
This diagnosis should be considered if there is active ischemia on electrocardiogram or
stress in the distribution of prior stent and absence of significant coronary lesion on
angiography.
Probable stent thrombosis should be considered in a patient with acute coronary syndrome
within 30 days of stent placement.
Diagnosis definite stent thrombosis requires confirmation of thrombosis with coronary
angiogram.

Go to the next page if you knew the correct answer, or click the link image(s) below to further
research the concepts in this question (if desired).

Research Concepts:
Stent Thrombosis

We update eBooks quarterly and Apps daily based on user feedback. Please tap flag to
report any questions that need improvement.
Question 75: A 62-year-old female is treated with a ventriculoperitoneal shunt for
hydrocephalus. One month later, she presents with a headache, unsteadiness and fever. She
attends the emergency department who discusses the case with the on-call neurosurgical team.
Following a normal set of labs, including inflammatory markers and CBC, a shunt tap is
performed, and a plain CT head and shunt radiographs are requested. Which of the following is
the most accurate statement about the patient's condition?

Choices:
1. The CT head is likely to diagnose the condition
2. The shunt tap is likely to diagnose the underlying condition
3. This lady needs a lumbar puncture to diagnose the underlying condition
4. The CT head will show ependymal enhancement and hydrocephalus.
Answer: 2 - The shunt tap is likely to diagnose the underlying condition
Explanations:
CSF in ventriculitis may show a raised cell count and protein, with low glucose.
CSF culture may be positive, although this is not always the case, especially is sample is
taken following antibiotic administration.
The presence of oligoclonal immunoglobulin G or M bands, CSF lactate, procalcitonin, and
lysozymes help make an early diagnosis.
Non-contrast CT head demonstrates non-specific findings, including dependent hyperdense
ventricular debris, hydrocephalus, periventricular low density as well as features of the
underlying abnormality. It does not offer a definitive diagnosis. A CSF sample should be
obtained. CSF sample can be obtained via a shunt tap, it does not need to be obtained via
lumbar puncture. The patient had a plain CT head. Enhancement may be seen on a contrast-
enhanced scan. A CSF sample will also be required. A wound infection needs to be
considered - and examination of her abdomen and cranial wound is imperative. A shunt
series radiograph may help rule out a blocked shunt. Comparison of the new and previous
CT scans may help decide if the patient has hydrocephalus, which may suggest a blocked
shunt.

Go to the next page if you knew the correct answer, or click the link image(s) below to further
research the concepts in this question (if desired).

Research Concepts:
Ventriculitis

We update eBooks quarterly and Apps daily based on user feedback. Please tap flag to
report any questions that need improvement.
Question 76: A 70-year-old male with a past medical history of diabetes mellitus, chronic
kidney disease, and hypertension recently admitted due to ST-segment elevation myocardial
infarction. After five days the patient developed low blood pressure and harsh systolic murmur.
Echocardiogram showed ventricular septal rupture (VSR). Intraaortic balloon pump (IABP) has
been placed for hemodynamically stability, and cardiac surgery has been called for intervention.
How does patient get benefit from IABP device in ventricular septal rupture?

Choices:
1. By decreasing preload
2. By decreasing afterload
3. By increasing heart rate
4. By dilating aorta
Answer: 2 - By decreasing afterload
Explanations:
VSR is one of the rare but lethal complications of acute myocardial infarction.
VSR can present with hemodynamic instability due to cardiogenic shock.
Ultimate treatment of VSR is surgical repair with a patch.
Due to hemodynamic instability and cardiogenic shock patient needs an IABP as a bridge
until the surgery. IABP reduce afterload so decrease left-to-right shunt through VSR.

Go to the next page if you knew the correct answer, or click the link image(s) below to further
research the concepts in this question (if desired).

Research Concepts:
Ventricular Septal Rupture

We update eBooks quarterly and Apps daily based on user feedback. Please tap flag to
report any questions that need improvement.
Question 77: A 42-year-old male is stabbed in the left chest during a bar-fight. Emergency
medical services (EMS) have arrived on the scene and have found the patient to be hypotensive
in hemorrhagic shock. En route to the hospital, the patient goes into cardiac arrest. Paramedics
intubate the patient and find him to be in asystole. EMS calls the local emergency department to
establish medical command. The medical command clinician is told that EMS is 5 minutes from
the hospital and that they are having problems oxygenating and ventilating the patient because
there is a lot of resistance when squeezing the bag-valve-mask. Which of the following would be
the most appropriate management at this time for the medical command clinician to give to the
paramedics?

Choices:
1. Continue to transport to the hospital with no further treatment advised
2. Extubate and reintubate the patient as the endotracheal tube is most likely displaced into the
esophagus
3. Perform immediate left-sided needle decompression as the patient may have a tension
pneumothorax
4. Perform immediate needle pericardiocentesis as the patient may have pericardial tamponade
Answer: 3 - Perform immediate left-sided needle decompression as the patient may have a
tension pneumothorax

Explanations:
Patients who present in traumatic cardiac arrest are managed differently with respect to the
injury mechanism. Penetrating trauma, which leads to exsanguination, pericardial
tamponade, tension hemothorax, or tension pneumothorax, can be treated if done
expeditiously. The advanced cardiac life support (ACLS) and advanced trauma life support
(ATLS) courses promulgated by the American Heart Association and the American College
of Surgeons teach the 5 H's and T's (hypoxia, hypovolemia, hyperkalemia, hydrogen ions,
hypothermia, tension pneumothorax, thrombosis, toxins, thromboembolism, tamponade) for
potentially reversible causes of asystole. If not identified and treated early and return of
spontaneous circulation is restored, patients have a very dismal prognosis.
Tension pneumothorax usually develops when the pleura of the chest is violated, and the
underlying pulmonary parenchyma is damaged. This leads to air being released between the
visceral and parietal pleura creating a true space in which further air leak leads to further
lung collapse or pneumothorax. With intubation and bag-mask ventilation, air is exchanged
via positive pressure ventilation. This positive pressure leads to rapid expansion of the
underlying pneumothorax. When intrapleural pressure exceeds the pressure within the
mediastinum, low-pressure structures such as the right atria and ventricle as well as the
venous system begin to collapse. Cardiac pre-load is then decreased drastically, which
diminishes left heart filling leading to a significant reduction in cardiac output and lower
coronary filling pressures during diastole. This, in turn, leads to cardiac ischemia and
arrhythmias such as asystole if not corrected. Needle thoracostomy is a rapid way to convert
a life-threatening tension pneumothorax into an open pneumothorax correcting the
underlying cardiac dysfunction.
Needle thoracostomy is performed via a large-bore 18-gauge or large needle at least 2
inches long placed in the mid-clavicular line over the third rib in the second intercostal
space. The second rib can be identified as it is adjacent to the sternomanubrial joint, which
can often be palpated. The third rib is then identified from there, and the needle is placed
over the top of the rib to avoid the neurovascular bundle that runs on the inferior aspect. The
needle is inserted until a rush of air is identified, and the tension pneumothorax is corrected
evidenced by a return of normal hemodynamics. Alternatively, in heavier set patients or
patients with large muscle mass of the anterior chest, needle insertion can be performed by
placement at the anterior axillary line using the same landmarks. Ultimately, a formal tube
thoracostomy must be performed as this is only a temporizing procedure.
Pericardial tamponade has similar underlying pathophysiology as that of a tension
pneumothorax except that blood is released into the pericardial sac leading to increased
pressures which compress the right heart structures and venous system. Needle
pericardiocentesis is an option to alleviate this pressure in an emergent situation, but
clinically, the patient would not present with issues in performing bag-mask ventilation and
meeting increasing resistance. It is commonly taught that patients with pericardial
tamponade present with Beck's triad: jugular venous distension (JVD), muffled heart sounds
and narrowed pulse pressure. However, this is seen in only about one-third of patients and is
less likely to be seen in a hypovolemic trauma patient who is in hemorrhagic shock as they
will not have high venous pressure to see JVD.

Go to the next page if you knew the correct answer, or click the link image(s) below to further
research the concepts in this question (if desired).

Research Concepts:
Asystole

We update eBooks quarterly and Apps daily based on user feedback. Please tap flag to
report any questions that need improvement.
Question 78: A 17-year-old male collapses after being hit in the chest with a lacrosse ball.
He has no known past medical history. Which of the following is most likely to improve his
long-term outcome?

Choices:
1. Intravenous epinephrine
2. Early defibrillation
3. Implanted cardiac defibrillator
4. Endotracheal intubation
Answer: 2 - Early defibrillation
Explanations:
Commotio cordis is ventricular fibrillation prompted by direct trauma to the chest wall
overlying the heart without structural cardiac injury.
Early defibrillation is the most effective means of promoting the return of spontaneous
circulation and improving long-term outcomes. Efforts to increase awareness, earlier
recognition and intervention, and increased access to automatic external defibrillators have
been associated with improved outcomes.
An implanted cardiac defibrillator is not indicated for the treatment of commotio cordis with
a structurally normal heart.
Successful reversal of the ventricular fibrillation from commotio cordis does not appear to
increase the long-term risk for adverse cardiac events. However, there may be secondary
ischemic injury from decreased cardiac output during the fibrillatory period, especially if
resuscitation is delayed.

Go to the next page if you knew the correct answer, or click the link image(s) below to further
research the concepts in this question (if desired).

Research Concepts:
Commotio Cordis

We update eBooks quarterly and Apps daily based on user feedback. Please tap flag to
report any questions that need improvement.
Question 79: A patient on 4 liters of oxygen by nasal cannula has an arterial blood gas
performed. The patient has normal work of breathing, with a respiratory rate of 23. The arterial
blood gas indicates a pH of 7.40, PaO2 = 92 mmHg, and PaCO2 = 42 mmHg. Which of the
following is most appropriate?

Choices:
1. Decrease the oxygen to 2 liters
2. Continue present management and record the data in the chart
3. Increase the oxygen to 6 liters
4. Encourage the patient to breathe faster
Answer: 2 - Continue present management and record the data in the chart
Explanations:
The arterial blood gas shows the patient is receiving adequate treatment, and no changes are
necessary.
A normal physiologic range of PaO2 is 75-100 mmHg.
If someone is requiring supplemental oxygen you need to asses WHY, instead of ONLY
increasing the oxygen. There are multiple reasons for hypoxemia: V/Q mismatch, right-to-
left shunt, diffusion impairment, hypoventilation, and low Fraction of inspired O2. at sea
level, low inspired O2 is not the cause for hypoxemia.
You can evaluate quickly an intubated patient with hypoxemia using the pneumonic DOPE.
Displacement (moved endotracheal tube), Obstruction (secretions or object to be suctioning
or removed from the airway), Pneumothorax, Equipment failure (detached ventilator, wrong
ventilator settings, malfunctioning ventilator, malfunctioning surveillance equipment).

Go to the next page if you knew the correct answer, or click the link image(s) below to further
research the concepts in this question (if desired).

Research Concepts:
Arterial Blood Gas

We update eBooks quarterly and Apps daily based on user feedback. Please tap flag to
report any questions that need improvement.
Question 80: A previously healthy 17-year-old boy presents to the emergency department
with a high-grade fever, malaise, neck rigidity, and a petechial skin rash. He reveals that he had
developed an upper respiratory tract infection three days ago. Today, in the emergency
department, he has a blood pressure of 70/40 mmHg, a heart rate of 120 beats per minute.
Further evaluation with labs show a white blood cell count of 22,000 per microliter, sodium 120
mEq/L, potassium 5.8 mEq/L, chloride 89 mEq/L, blood urea nitrogen 34 mg/dL, creatinine 2.0
mg/dL, and glucose 50 mg/dL. Which of the following is most likely to be associated with his
condition?

Choices:
1. Disseminated intravascular coagulation
2. Isolated thrombocytopenia
3. Respiratory acidosis
4. Metabolic alkalosis
Answer: 1 - Disseminated intravascular coagulation
Explanations:
The patient has hypotension with hyperkalemia, hyponatremia, and hypochloremia. These
laboratory abnormalities and petechial skin rash suggest that he has Waterhouse-
Friderichsen syndrome (WFS).
The syndrome is caused by meningococcus. Disseminated intravascular coagulation is a
complication associated with meningococcemia.
Other conditions associated with meningococcemia petechial rash, neurologic
manifestations, and pupura fulminans.
Metabolic acidosis due to adrenal insufficiency is seen on arterial blood gas analysis.

Go to the next page if you knew the correct answer, or click the link image(s) below to further
research the concepts in this question (if desired).

Research Concepts:
Waterhouse-Friderichsen Syndrome

We update eBooks quarterly and Apps daily based on user feedback. Please tap flag to
report any questions that need improvement.
Question 81: A 23-year-old man marathon runner, who was participating in a triathlon,
presented to the emergency department for nausea and vomiting. In the emergency department,
his vital signs showed a heart rate of 140/min and blood pressure 95/76 mmHg. Clinical
examination showed dry lips, mouth, and skin. His labs showed a white blood cell count of 7,000
cells/mm3, hemoglobin 15 g/dL, platelets 450,000/microL, sodium 141 mEq/L, potassium 5.1
mmol/L, creatinine 5 mg/dL, CO2 17 mEq/L. He was given one-liter fluid bolus and discharged
from the emergency department. The following day he came again to the hospital because of
worsening vomiting and bloody urine. His repeated labs show a white blood cell count of 12,000
cells/mm3, hemoglobin 16 g/dL, platelets 456,000/microL, sodium 142 mEq/L, potassium 5
mmol/L, creatinine 2.0 gm/dL, CO2 15 mEq/L. Urinalysis shows a red color, pH 4.8, 2-3 RBCs,
blood positive, nitrate negative and no bacteria. He is admitted to the hospital and give IV fluids.
A few days later, he became hypoxic and was eventually intubated. Clinical examination reveals
elevated jugular venous pressure, bilateral crackles and bilateral pitting edema. Which of the
following best explains his recent decline in cardiac function?

Choices:
1. Acute kidney injury
2. Vasculitis
3. Ischemia secondary to hypotension
4. Hematuria
Answer: 1 - Acute kidney injury
Explanations:
This patient has type 3 cardiorenal syndrome which is defined as a sharp decline in renal
function that results in a sharp reduction in cardiac function.
The pathophysiology of type 3 cardiorenal syndrome is still not completely understood.
However, it can be due to a direct effect of acute kidney injury on the heart through
releasing immunomodulators like interleukin-1 and tumor necrosis factor which will induce
cardiac apoptosis.
The other possibility that cardiac injury can be due to the direct toxic accumulation from
kidney failure (uremic cardiomyopathy).
The patient has hematuria secondary to rhabdomyolysis, and it is unlikely due to systemic
illness like vasculitis. Moreover, ischemic cardiac disease secondary to hypotension is
unlikely in a healthy adult patient with mean arterial pressure at 75.

Go to the next page if you knew the correct answer, or click the link image(s) below to further
research the concepts in this question (if desired).

Research Concepts:
Cardiorenal Syndrome

We update eBooks quarterly and Apps daily based on user feedback. Please tap flag to
report any questions that need improvement.
Question 82: A 67-year-old patient with a history of coronary artery disease and COPD, on
chronic steroid therapy, is admitted for coronary artery bypass graft. On the same day, the patient
self extubates in the ICU with the endotracheal tube cuff still inflated. Subsequently, he develops
hoarseness and dysphagia. Which of the following is the most likely reason for the development
of hoarseness and dysphagia?

Choices:
1. Chronic steroid therapy
2. Traumatic extubation with the endotracheal tube still inflated
3. COPD
4. Coronary artery bypass graft surgery
Answer: 2 - Traumatic extubation with the endotracheal tube still inflated
Explanations:
Arytenoid subluxation is a rare complication that typically occurs after traumatic injury to
the cricoarytenoid joint during laryngoscopy and intubation. They are usually associated
with prolonged and traumatic intubation in cases of unanticipated difficult airways. Chronic
steroid use may weaken the cricoarytenoid joint capsule and predispose to this complication
in the event of traumatic intubation.
Direct trauma by the laryngoscope during intubation is the proposed mechanism of trauma
for anteromedial arytenoid subluxation. Posterolateral displacement of the arytenoid
cartilage by a partially or fully inflated cuff is the proposed mechanism of trauma for
posterolateral arytenoid subluxation and is the most likely explanation for the hoarseness in
this patient.
In rare instances, a history of severe cough, or spontaneous events has been reported. This
patient had a history of COPD but no COPD exacerbation was reported so is unlikely to be
the cause of the hoarseness and dysphagia.
Major cardiac surgery involving the use of a transesophageal echocardiography probe may
increase the incidence of arytenoid subluxation. Coronary artery bypass graft surgery by
itself is not a risk factor for arytenoid subluxation.

Go to the next page if you knew the correct answer, or click the link image(s) below to further
research the concepts in this question (if desired).

Research Concepts:
Arytenoid Subluxation

We update eBooks quarterly and Apps daily based on user feedback. Please tap flag to
report any questions that need improvement.
Question 83: A 65-year-old homeless male is admitted with the chief complaints of fever,
shortness of breath, and weight loss. He has been having a productive cough with hemoptysis off
and on for one month. He is a known alcoholic. On examination, he is febrile with a blood
pressure of 110/80 mmHg, pulse 98/min regular, respiratory rate 24/min and oxygen saturation
of 90% on room air. On auscultation, there are coarse crackles bilaterally but more prominent on
the right side. You notice that there is a draining sinus on the chest wall. A chest radiograph
shows pulmonary infiltrates and a few cavitary lesions. Dark brown granules are seen on a frozen
section. Gram stain shows purple-staining fine filaments. CBC shows leukocytosis with elevated
neutrophils. C- reactive protein and erythrocyte sedimentation rate are elevated. Which of the
following is used to treat this condition?

Choices:
1. Incision and drainage
2. Radiation therapy
3. High-dose penicillin
4. Amphotericin
Answer: 3 - High-dose penicillin
Explanations:
Dark brown granules can be sulfur and represent actinomycosis. Histologically, sulfur
granules appear black.
Actinomyces can be cultured. The preliminary diagnosis usually is made by looking for
sulfur granules with microscopy. Imaging is usually needed to determine the extent of
infection. The gold standard test is a histological examination and bacterial culture of a
biopsy which can be obtained with bronchoscopy (pulmonologist), a CT-guided biopsy
(interventional radiologist), or video-assisted thoracoscopy surgery (VATS) performed by a
thoracic surgeon. Anaerobic cultures of pleural fluid almost never grow the organism.
Sputum culture will not be useful unless the patient has a cavitary disease.
The imaging for actinomycosis is nonspecific. In the acute presentation, it can look like any
other pneumonia. In chronic forms, it can present as a pulmonary mass or can cavitate. In
the case of a mass malignancy, Actinomycosis is an important diagnosis to consider.
Especially when it cavitates, tuberculosis is an important differential diagnosis to consider.
The computed tomography scan findings can vary depending upon the duration of illness
and can include consolidation, cavitation, pleural effusion, lymph node enlargement,
atelectasis, and ground glass opacification.
Treatment is with high-dose penicillin.

Go to the next page if you knew the correct answer, or click the link image(s) below to further
research the concepts in this question (if desired).

Research Concepts:
Actinomycosis

We update eBooks quarterly and Apps daily based on user feedback. Please tap flag to
report any questions that need improvement.
Question 84: A 65-year-old woman status post-three-vessel coronary artery bypass grafting
(CABG), and transcatheter aortic valve replacement (TAVR) is under the care of a critical care
team in the intensive care unit. She was put on a ventilator due to respiratory insufficiency. The
ventilator settings include a fraction of inspired oxygen (FiO2) of 70%, positive end-expiratory
pressure (PEEP) of 20 cm H2O, rate of 12 breaths/min. She loses pulse suddenly. The
electrocardiogram (EKG) shows narrow QRS complexes. There is absent breath sounds on the
left side with tracheal deviation on the right side. Which of the following is the next step in
management after initiating cardiopulmonary resuscitation (CPR)?

Choices:
1. Transcutaneous pacemaker
2. Needle decompression
3. Pericardiocentesis
4. Surgical evaluation
Answer: 2 - Needle decompression
Explanations:
Patients on high PEEP are at risk of ventilator-associated barotrauma. This patient most
likely had a tension pneumothorax induced pulseless electrical activity (PEA) due to high
PEEP. The best step is to decompress the chest with a needle. It can then be converted to a
chest tube if necessary.
A pacemaker is used in case of bradyarrhythmias or to pace rhythms. It does not have a role
in treating PEA.
Pericardiocentesis would be a good option if the patient had PEA due to tamponade. This
would show raised jugular venous pressure, hypotension, and muffled heart sounds.
Surgical evaluation can be done after the patient is stabilized, and if the cause of PEA
cannot be fixed with supportive medical treatment, for example, in case of a massive
pulmonary embolus.

Go to the next page if you knew the correct answer, or click the link image(s) below to further
research the concepts in this question (if desired).

Research Concepts:
Pulseless Electrical Activity

We update eBooks quarterly and Apps daily based on user feedback. Please tap flag to
report any questions that need improvement.
Question 85: A 75-year-old-female underwent percutaneous coronary intervention in the
left anterior descending artery after presenting with abdominal pain for two days. Two hours
after her procedure she developed an accelerated idioventricular rhythm. Medications are aspirin,
clopidogrel, metoprolol, atorvastatin, and lisinopril. The patient denies any active complaints and
hemodynamically stable. What should be the possible intervention?

Choices:
1. Add calcium channel blocker
2. Increase beta blocker
3. Defibrillator
4. No intervention needed
Answer: 4 - No intervention needed
Explanations:
Accelerated idioventricular rhythm is a common complication within 24 hours of
reperfusion. The patient is on optimum medical therapy so there is no need to add another
medication.
She is tolerating the rhythm well, so there is no need to increase the beta-blocker dose.
Accelerated idioventricular rhythm is a benign rhythm after reperfusion so no defibrillation
is needed.
Accelerated idioventricular rhythm (AIVR) is due to abnormal automaticity in
subendocardial Purkinje fibers and is observed in 15% of patients undergoing reperfusion.
There is no further intervention needed.

Go to the next page if you knew the correct answer, or click the link image(s) below to further
research the concepts in this question (if desired).

Research Concepts:
Silent Myocardial Ischemia

We update eBooks quarterly and Apps daily based on user feedback. Please tap flag to
report any questions that need improvement.
Question 86: An adult patient presents with the worst headache of her life. A noncontrast
CT of the head is negative. A lumbar puncture produces four bloody tubes, each with RBC
counts greater than 100,000/mm3. What is the next step in management?

Choices:
1. Repeat a noncontrast head CT the next day
2. Perform a CT angiogram
3. Perform a head CT with contrast
4. Administer mannitol
Answer: 2 - Perform a CT angiogram
Explanations:
A noncontrast head CT in the context of a nontraumatic, bloody tap will be positive in the
vast majority of patients with subarachnoid hemorrhage. However, a normal CT does not
mean a bloody tap can be ignored.
Unlike a traumatic tap, the blood cell count does not diminish from the first to the fourth
tube. Xanthochromia is the yellow appearance of cerebrospinal fluid caused by the
degradation of heme to bilirubin confirming a hemorrhage.
CT or MR angiography is needed to detect vascular pathology that might account for the
lumbar puncture findings of subarachnoid hemorrhage.
Initial management consists of preventing vasospasm, blood pressure control, fluid
management, anticonvulsant therapy, pain control, and many neurosurgeons recommend
early surgery.

Go to the next page if you knew the correct answer, or click the link image(s) below to further
research the concepts in this question (if desired).

Research Concepts:
Subarachnoid Hemorrhage

We update eBooks quarterly and Apps daily based on user feedback. Please tap flag to
report any questions that need improvement.
Question 87: A 65-year-old female has a left hemicolectomy for cancer. On postoperative
day seven, she develops a temperature of 39.2 C. Physical examination shows clear lungs, no
evidence of wound or line infections, and her calves are soft and nontender. Chest radiograph
shows mild bibasilar atelectasis. Two days later, blood cultures grow Bacteroides fragilis and
Escherichia coli, while urine culture is negative. Which of the following is the most appropriate
approach to this patient?

Choices:
1. Start triple antibiotics empirically
2. Computed tomography scan of the abdomen and pelvis
3. Repeat blood cultures
4. Replace all lines
Answer: 2 - Computed tomography scan of the abdomen and pelvis
Explanations:
The evidence of polymicrobial bacteremia indicates that the patient likely has an
intraabdominal abscess.
Postoperative intraabdominal abscess imaging is best after postoperative day seven because
tissue edema is reduced and nonsuppurative fluids are reabsorbed.
If present and accessible, percutaneous CT-guided drainage is indicated.
Ultrasound can sometimes reveal intraabdominal abscesses, but exams can be limited by
obesity, dressings, open wounds, stomas, and intervening viscera.

Go to the next page if you knew the correct answer, or click the link image(s) below to further
research the concepts in this question (if desired).

Research Concepts:
Abdominal Abscess

We update eBooks quarterly and Apps daily based on user feedback. Please tap flag to
report any questions that need improvement.
Question 88: A 32-year-old male presents for complaints of headache, dizziness, and
generalized weakness, that started this afternoon but is progressively worsening. He works as a
painter and was operating a gas-powered paint sprayer indoors for approximately 10 hours today.
He is alert, oriented, and has an unremarkable physical exam. He reports no medical problems
and smokes 1 pack of cigarettes per day as well as alcohol use socially. Carboxyhemoglobin
level obtained via blood gases is 12%. Which of the following is the next best step in the
management of this patient?

Choices:
1. Transfer to a hyperbaric treatment center
2. Treatment with high-flow oxygen until carboxyhemoglobin level is 3%
3. Treatment with high-flow oxygen for 6 hours
4. Cardiac monitoring, ECG, and troponin in addition to normobaric oxygen
Answer: 3 - Treatment with high-flow oxygen for 6 hours
Explanations:
Current recommendations for asymptomatic patients without high-risk features is treatment
with normobaric oxygen for 6 hours. Oxygen competes with carbon monoxide for binding
at the heme group of hemoglobin. The half-life of carboxyhemoglobin in room air is 4 hours
or more.
However, this is substantially decreased to just 40 to 80 minutes with high-flow oxygen. By
increasing the rate of oxygen flow up to 60 liters per hour, the high concentration of oxygen
molecules can displace carbon monoxide from the hemoglobin molecule more readily,
enhancing elimination.
Though hyperbaric oxygen hastens elimination twice as fast, high-flow oxygen is more
accessible, easier to initiate, and safer with much less concern for barotrauma.
Treatment with normobaric high-flow oxygen would be appropriate for this patient.
Recommendations currently suggest considering hyperbaric treatment in those with
neuropsychiatric complaints, acute coronary syndrome, those who are comatose or have an
altered level of consciousness, pregnant women, anyone over the age of 65, and those with
initial carboxyhemoglobin levels greater than 25%. Hyperbaric oxygen is not the standard
of care in any particular scenario, and any decision to initiate this should be made in
conjunction with a toxicologist and/or Poison Control, taking into consideration the patient's
age, comorbidities, mental status, as well as feasibility and distance from a hyperbaric
center. Despite hastening carboxyhemoglobin elimination by decreasing the half-life of
carboxyhemoglobin to just 23 minutes, the use of hyperbaric oxygen has not been translated
into improved outcomes or decreased mortality. Of note, hyperbaric oxygen may cause
barotrauma, pulmonary edema, and seizures as well as potentially increase oxidative stress
and free radical production. Although this patient should certainly receive high-flow
oxygen, the appropriate management for asymptomatic patients and without high-risk
features (chest pain, loss of consciousness, altered level of consciousness, focal neurologic
deficits) is treatment for 6 hours or until levels normalize. A small amount of carbon
monoxide is produced endogenously with the metabolism of heme, so a baseline
carboxyhemoglobin level is thought to be around 1 to 3 percent in non-smokers and as high
as 10 percent in smokers. For this reason, it would be not reasonable to treat the above
patient with a goal of 3 percent since their baseline is likely higher. ECG and troponin are
not routinely obtained in carbon monoxide toxicity but are warranted in the presence of
angina, dyspnea, or palpitations. Management should always include a carboxyhemoglobin
level obtained via blood gases, as well as a pregnancy test in females since this may change
management. Imaging with a chest x-ray or head CT is frequently obtained to rule out other
causes but is not routinely necessary. In cases of non-accidental poisonings, a full
toxicology panel including salicylate/acetaminophen levels and urine drug screen is
recommended.

Go to the next page if you knew the correct answer, or click the link image(s) below to further
research the concepts in this question (if desired).
Research Concepts:
Carboxyhemoglobin Toxicity

We update eBooks quarterly and Apps daily based on user feedback. Please tap flag to
report any questions that need improvement.
Question 89: A registered nurse is brought to the emergency department after a seizure. She
was found unresponsive with an undetectable blood sugar. Two ampules of 50% glucose were
administered with the elevation of her blood sugar to 42 mg/dL. She has a history of depression.
Which of the following finding would be consistent with exogenous insulin overdosage?

Choices:
1. Plasma insulin greater than 18 pmol/L, plasma glucose less than 55 mg/dL, and C-peptide
greater than 0.6 ng/ml
2. Plasma insulin greater than 18 pmol/L, plasma glucose less than 55mg/dL, and C-peptide
greater than 0.6 ng/ml
3. Plasma insulin greater than 18 pmol/L, plasma glucose less than 55 mg/dL, and C-peptide
undetectable
4. Plasma insulin less than 18 pmol/L, Plasma glucose less than 55 mg/dL, and C-peptide
undetectable
Answer: 3 - Plasma insulin greater than 18 pmol/L, plasma glucose less than 55 mg/dL, and
C-peptide undetectable

Explanations:
Exogenous insulin administration should be considered in patients with profound
hypoglycemia and no history of diabetes mellitus.
The measurement of C-peptide and insulin level is useful in differentiating between
exogenous and endogenous insulin administration. The C-peptide is a marker for
endogenous insulin. The C-peptide levels will be low in patients with insulin overdose and
high in patients with hypoglycemia resulting from an insulin secretagogue overdose.
The three needed tests are plasma glucose, plasma insulin, and C-peptide level.
C-peptide levels less than 0.6 ng/mL would suggest exogenous hyperinsulinemia if plasma
insulin measurement is high and hypoglycemia diagnosed.

Go to the next page if you knew the correct answer, or click the link image(s) below to further
research the concepts in this question (if desired).

Research Concepts:
Insulin

We update eBooks quarterly and Apps daily based on user feedback. Please tap flag to
report any questions that need improvement.
Question 90: A 28-year-old female presents with shortness of breath for 3 days, which is
progressively getting worse. In the emergency department, the blood workup shows a white cell
count of 14,000, with 78% neutrophils, 78%, and 11% eosinophils. Basic metabolic profile
shows sodium of 136 mmol/L, potassium 4.1 mmol/L, chloride 100 mmol/L, and bicarbonate 20
mmol/L. The patient has seasonal allergies, and family history is positive for asthma in her
mother. The chest x-ray is normal, EKG shows sinus tachycardia. Arterial blood gas (ABG)
shows pH of 7.2, PCO2 of 60, and bicarbonate of 30. Initially, the patient was given albuterol
and Ipratropium nebulization and intravenous corticosteroids, was put on BiPAP, but she did not
improve. Her shortness of breath increased, and she was using accessory muscles. She is
intubated and placed on a ventilator with settings AC mode tidal volume of 400, rate of 24,
PEEP of 5, and FiO2 40%. ABG was drawn after 1 hour, which shows pH 7.16, PCO2 66, and
bicarbonate of 25. What is the acid-base disorder patient has on last ABG, and how should the
ventilator settings be adjusted?

Choices:
1. The patient has metabolic acidosis and the respiratory rate should be increased to 26, flow
should be decreased
2. The patient has respiratory acidosis with metabolic compensation and respiratory rate should
be lowered to 20, the flow should be increased
3. The patient has respiratory and metabolic acidosis. The respiratory rate and flow rate should
not be changed
4. The patient has metabolic alkalosis with respiratory acidosis and respiratory rate should be
increased to 30 and flow should be increased
Answer: 2 - The patient has respiratory acidosis with metabolic compensation and
respiratory rate should be lowered to 20, the flow should be increased

Explanations:
The patient has respiratory acidosis with metabolic compensation. The respiratory rate
should be lowered to 20, flow should be increased. Respiratory acidosis is a state in which
there is usually a failure of ventilation and an accumulation of carbon dioxide. The primary
disturbance of elevated arterial PCO2 decreases the ratio of bicarbonate to arterial PCO2,
which leads to a lowering of the pH. In the presence of alveolar hypoventilation, two
features commonly seen are respiratory acidosis and hypercapnia.
The patient's acidosis worsened as she developed auto-peep.
To correct auto-peep her I:E ratio has to be adjusted. Lower the respiratory rate which will
give her more expiratory time. Increasing the flow will also help in getting more expiratory
time and will help in acidosis. ABG should be repeated in 30 to 45 minutes.
Carbon dioxide plays a remarkable role in the human body mainly through its role in pH
regulation of the blood. It is the primary stimulus to initiate ventilation. In its normal state,
the body maintains CO2 in a well-controlled range from 38 to 42 mmHg by balancing its
production and elimination.

Go to the next page if you knew the correct answer, or click the link image(s) below to further
research the concepts in this question (if desired).

Research Concepts:
Respiratory Acidosis

We update eBooks quarterly and Apps daily based on user feedback. Please tap flag to
report any questions that need improvement.
Question 91: An unidentified middle-aged male is brought to the hospital after being found
in a pool of his vomit at the side of the road along with a "will work for food" sign alongside
him. He has good chest excursions and pulse. Vital signs are within limits, and fingerstick
glucose is found to be 45 mg/dL. Intravenous access is obtained, and he is started on intravenous
saline and dextrose along with multivitamins. Routine lab work including toxicologic studies is
sent. He wakes up groggily and admits that he has been a long-term alcohol user and had a binge
last night as well. He is cachectic on exam with temporal hollowing but without any evidence of
chronic liver disease including jaundice or ascites. Suddenly, the patient starts to have
generalized tonic-clonic contractions, which are terminated by intravenous benzodiazepines but
the patient goes into a coma, becoming unresponsive but able to maintain his airway. Lab tests
show calcium level of 8 mEq/dL, magnesium level of 1.8 mg/dL, potassium level of 2.7 mEq/dL,
and phosphorus level of 1.5 mg/dL. Which electrolyte abnormality most likely can be attributed
to the clinical presentation and complications?

Choices:
1. Calcium
2. Magnesium
3. Potassium
4. Phosphorus
Answer: 4 - Phosphorus
Explanations:
The most likely cause of the patient's seizure and coma is hypophosphatemia. He has risk
factors including prolonged malnutrition and alcohol use disorder and goes into tonic-clonic
contractions on administration of glucose which stimulates insulin release which causes
further worsening of hypophosphatemia by cellular uptake as is seen in refeeding syndrome.
Calcium abnormalities would more often be associated with tetany, cardiac arrhythmias
instead of seizures and coma and less likely precipitated by intravenous dextrose.
There are risk factors including chronic alcohol use disorder, refeeding in the patient but
magnesium levels are within acceptable limits and hypomagnesemia would more likely
present with tetany, cardiac arrhythmias from resultant hypocalcemia and also respiratory
failure.
Potassium abnormalities would more often be associated with cardiac arrhythmias and
myopathy instead of seizures and coma.

Go to the next page if you knew the correct answer, or click the link image(s) below to further
research the concepts in this question (if desired).

Research Concepts:
Ketoacidosis

We update eBooks quarterly and Apps daily based on user feedback. Please tap flag to
report any questions that need improvement.
Question 92: A senior man living in an assisted living facility came to the emergency
department with cough and fever. He is diagnosed with pneumonia. Blood and sputum
specimens were sent for laboratory testing. Empiric therapy started at that time. Now the sputum
culture is back and positive with P. aeruginosa. Now therapy should be changed from previous
empiric therapy to which of the following?

Choices:
1. Ampicillin and clavulanic acid
2. Azithromycin
3. Vancomycin
4. Piperacillin and tazobactam
Answer: 4 - Piperacillin and tazobactam
Explanations:
The senior man with cough and fever is most likely has bacterial pneumonia.
He is living in an assisted living facility so most likely has healthcare-associated
pneumonia.
One of the most common causes of healthcare-associated pneumonia is Pseudomonas
aeruginosa.
Antipseudomonal therapy, for example, piperacillin and tazobactam, is the recommended
therapy for this infection.

Go to the next page if you knew the correct answer, or click the link image(s) below to further
research the concepts in this question (if desired).

Research Concepts:
Bacterial Pneumonia

We update eBooks quarterly and Apps daily based on user feedback. Please tap flag to
report any questions that need improvement.
Question 93: A 56-year old female is admitted to the intensive care unit (ICU) due to
hypotension and bleeding after she was involved in a motor vehicle accident. An ultrasound is
performed, which shows splenic rupture. A laparotomy is carried out successfully, and the
patient is stabilized. On the 3rd post-operative day, the patient complains of difficulty breathing,
and her abdomen is tense and distended. The patient is tachycardic, hypertensive, and has a
bladder pressure of 24 cm. Which of the following is the most appropriate next step in the
management of this patient?

Choices:
1. Nasogastric tube placement
2. Avoidance of muscle relaxants
3. Head elevation of more than 30 degrees
4. Inotropic support to achieve a perfusion pressure of 60 mmHg in the abdomen
Answer: 1 - Nasogastric tube placement
Explanations:
Non-surgical therapeutic options for the treatment of intra-abdominal hypertension involves
an overall goal to improve abdominal wall compliance.
The goals of treatment are to decrease muscle contractions, evacuate luminal contents by
decompression, and evacuate abdominal fluid by drainage.
Correction of positive fluid balance through goal-directed volume resuscitation, along with
placement of the nasogastric tube can aid in decompressing the abdomen.
Muscle relaxants are a helpful adjunct in the conservative management of abdominal
hypertension.

Go to the next page if you knew the correct answer, or click the link image(s) below to further
research the concepts in this question (if desired).

Research Concepts:
Abdominal Compartment Syndrome

We update eBooks quarterly and Apps daily based on user feedback. Please tap flag to
report any questions that need improvement.
Question 94: A patient with a history of cirrhosis of the liver presents with shortness of
breath in a standing position which gets better in a lying down position. A chest x-ray is negative
for pleural effusion. Ultrasound abdomen reveals mild ascites. Which of the following
mechanisms is most likely responsible for the patient's symptoms?

Choices:
1. Impaired metabolism of body hormone
2. Impairment in the urea cycle
3. Increased portal pressure
4. Low serum albumin concentration
Answer: 1 - Impaired metabolism of body hormone
Explanations:
Patients with cirrhosis of the liver are prone to a hyperestrogenic state due to impaired
estrogen metabolism.
It leads to a hyperestrogenic state with the formation of arteriovenous malformation in the
lungs, which leads to platypnea and orthodeoxia syndrome.
Cardiac catheterization is indicated to confirm the diagnosis.
The impaired urea cycle leads to hyperammonemia which does not cause shortness of
breath. Increased portal pressure causes ascites, but the US abdomen reveals mild ascites,
which does not explain her symptoms. Low albumin contributes to ascites which is mild in
this patient. It is less likely to cause shortness of breath.

Go to the next page if you knew the correct answer, or click the link image(s) below to further
research the concepts in this question (if desired).

Research Concepts:
Pulmonary Arteriovenous Malformation

We update eBooks quarterly and Apps daily based on user feedback. Please tap flag to
report any questions that need improvement.
Question 95: A 66-year-old male is admitted to the hospital for abdominal pain, diarrhea,
and fever of 2 days duration. The patient underwent stem cell transplantation three weeks ago.
He denies blood in the stool. On physical exam, his temperature was 38.5 C (101.3 F), blood
pressure 109/70 mmHg, pulse rate 109/mon, and respiratory rate 16/min. Abdominal exam
reveals tenderness in the left lower quadrant. His labs show hemoglobin (Hb) 7 mg/dL, white
blood cells (WBCs) 400/micoL, platelets 22000/micoL, and creatinine 1.3 mg/dL. The patient
was seronegative for CMV before transplant. Computed tomography (CT) scan shows thickening
of the bowel wall of the descending colon. What is the next best step in the management of this
patient?

Choices:
1. Granulocyte infusion
2. Stool culture
3. Chest CT scan
4. Testing for Clostridium difficile
Answer: 4 - Testing for Clostridium difficile
Explanations:
The patient presented with signs and symptoms concerning for colitis. He had a recent stem
cell transplant, and he is immunosuppressed and neutropenic. He needs prompt workup and
management to prevent complications. The CT findings confirm the diagnosis of colitis.
The cause of colitis still needs to be determined. Given his neutropenia, the patient might
have had neutropenic enterocolitis (NE). However, the location of inflammation on CT scan
(descending colon) is not typical for NE, but it still can occur anywhere in the colon.
However, before diagnosing NE, workup should be done to rule out other causes of colitis
that include infectious causes.
Stool panel, stool culture, and clostridium difficile testing should be done. Clostridium
difficile gives results faster than stool culture, and early testing for Clostridium difficile
allows earlier intervention with appropriate antibiotics.
Some studies suggest granulocyte infusion for the treatment of neutropenic enterocolitis.
However, to diagnose neutropenic enterocolitis, one must rule out other causes of colitis
first such as clostridium difficile infection which can give a similar presentation. Stool
culture is part of the workup done for a patient presenting with colitis symptoms, especially
in the setting of immune suppression such as the patient in the question. However, it takes a
long time for the results to come back as compared to Clostridium difficile testing. Early
testing for Clostridium difficile allows earlier intervention with appropriate antibiotics. The
main concern in this patient is colitis, which is likely the source of his symptoms and signs.
There is no evidence for a lung infection.

Go to the next page if you knew the correct answer, or click the link image(s) below to further
research the concepts in this question (if desired).

Research Concepts:
Neutropenic Enterocolitis (Typhlitis)

We update eBooks quarterly and Apps daily based on user feedback. Please tap flag to
report any questions that need improvement.
Question 96: A patient who is debilitated presents to the emergency department with fevers
and general malaise. A two-view chest x-ray shows a mass in the periphery of the right lung with
an air-fluid level. The patient is started on intravenous antibiotics. Over the next 2 days, the
patient looks more toxic and shows signs of sepsis. The lung mass appears to have increased and
the patient is not improving. What is the most appropriate next step in management?

Choices:
1. Thoracotomy
2. Video-assisted thoracoscopic surgery
3. Percutaneous decompression
4. Infiltration of the mass with antibiotics
Answer: 3 - Percutaneous decompression
Explanations:
Percutaneous tube drainage has been used for drainage in patients with an expanding lung
abscess, signs of unremitting sepsis, abscess under tension, failure to wean from mechanical
ventilation, and if there has been contamination of the contralateral lung.
In such a case, percutaneous decompression is the best procedure.
Studies have shown that the incidence of empyema after percutaneous drainage is low and
that patients improve significantly after decompression.
When these patients need surgery, a double lumen tube should be used to prevent
contamination of the unprotected lung.

Go to the next page if you knew the correct answer, or click the link image(s) below to further
research the concepts in this question (if desired).

Research Concepts:
Lung Abscess

We update eBooks quarterly and Apps daily based on user feedback. Please tap flag to
report any questions that need improvement.
Question 97: A 58-year-old patient presents to the emergency department via emergency
medical services following an apartment building fire. On primary survey, he is unconscious,
does not appear in respiratory distress but appears pale. The patient's vitals are blood pressure
110/65 mmHg, oxygen saturation 98% on room air, temperature 38.3 C (100.9 F), respiration
rate 18/min, and pulse 110 bpm. There are no external burn injuries noted, and physical exam
reveals no wheezing, no stridor, and no rhonchi but does show bright red retinal veins. Which of
the following is the most important next step?

Choices:
1. CT head without contrast
2. Emergent intubation and ventilation with 100% oxygen
3. Hydroxocobalamin
4. Initiate 100% FiO2 via nonrebreather
Answer: 2 - Emergent intubation and ventilation with 100% oxygen
Explanations:
Carbon monoxide (CO) poisoning should be considered in all patients exposed to fires in
confined spaces, especially those patients with complaints of delirium, altered
consciousness, hallucinations, fatigue, chest pain, or abdominal pain. Physical examination
findings suggesting CO poisoning are often nonspecific but include tachycardia,
hyperthermia, and pallor. "Cherry red" skin is a rare finding and most often seen
postmortem. Bright red retinal veins are an early finding.
Protection of the airway is of paramount importance in possible smoke inhalation injuries.
This patient is unconscious, thus having a Glasgow coma scale of less than 8. He should be
intubated and treated with 100% oxygen. Signs of upper airway edema and injury include
carbonaceous sputum, stridor, burns around the face, and singed facial hair.
Rapid intervention and delivery of oxygen is the basis of treatment for CO poisoning since
there is competition with oxygen to bind with hemoglobin. The binding of CO to
hemoglobin is reversible with high oxygen concentrations.
Hyperbaric oxygen therapy, if available, should be considered in patients suspected of
having carbon monoxide poisoning, but treatment should not be delayed if hyperbaric
therapy is not readily available.

Go to the next page if you knew the correct answer, or click the link image(s) below to further
research the concepts in this question (if desired).

Research Concepts:
Inhalation Injury

We update eBooks quarterly and Apps daily based on user feedback. Please tap flag to
report any questions that need improvement.
Question 98: A 50-year-old man is brought by emergency medical services (EMS) after
sustaining a gunshot injury. He has a past medical history of type 2 diabetes mellitus, coronary
artery disease, tobacco smoking, and chronic obstructive pulmonary disease. He undergoes an
exploratory laparotomy, during which he received 5 liters of IV normal saline during the surgery.
Soon after extubation, he develops acute respiratory distress. He is tachycardic. Lung
auscultation reveals bibasilar rales. Arterial blood gas analysis on room air reveals a PaO2 of 59,
pH 7.47, PaCO2 36, and bicarbonate 21. His arterial blood gas does not improve with the
administration of 100% oxygen. What is the most likely cause of his respiratory distress?

Choices:
1. Excessive anesthesia
2. Pulmonary edema
3. Pneumonia
4. Bronchial asthma exacerbation
Answer: 2 - Pulmonary edema
Explanations:
This patient has hypoxemia, and since he has an elevated A-a gradient leaves only shunting
and V/Q mismatch as a potential cause for his condition.
His failure to improve with 100% oxygen makes V/Q mismatch less possible, as might
occur in the case of pulmonary embolism or exacerbation of bronchial asthma.
Shunting would explain his hypoxemia. Pulmonary edema, pneumonia, and vascular shunt
are frequent causes of shunting.
His many risk factors make pulmonary edema the most likely explanation for his arterial
blood gases abnormalities and rales on the pulmonary exam.

Go to the next page if you knew the correct answer, or click the link image(s) below to further
research the concepts in this question (if desired).

Research Concepts:
Hypoxia

We update eBooks quarterly and Apps daily based on user feedback. Please tap flag to
report any questions that need improvement.
Question 99: A 67-year-old male is admitted with fever, general malaise, headaches, and
myalgias. He returned from a hiking trip in Connecticut 1 week ago. The patient is placed on
doxycycline therapy for possible anaplasmosis. Two days after admission he develops
hypotension and hemolysis. What is the next step in management?

Choices:
1. Increase doxycycline dose
2. Add intravenous vancomycin and cefepime for possible bacteremia
3. Send a peripheral smear and start clindamycin and quinine therapy
4. Consult hematology for hemolysis evaluation
Answer: 3 - Send a peripheral smear and start clindamycin and quinine therapy
Explanations:
The Ixodes tick transmits both anaplasmosis and babesiosis. Both diseases share the same
geographic distribution, and patients can be co-infected with both.
Anaplasmosis is treated with doxycycline, whereas Babesia is treated with clindamycin and
quinine in severe cases, such as this one.
Patients older than 60 years or splenectomized patients can have a severe presentation of
babesiosis. This patient is older than 60 years and was initially started on doxycycline for
possible anaplasmosis. Given the decline in his clinical status, he should be evaluated for
co-infection with Babesia and be initiated on proper therapy.
A peripheral smear should be obtained to look for the parasite Babesia inside red blood
cells. Anaplasma can be diagnosed by identifying the presence of morulae inside
neutrophils.

Go to the next page if you knew the correct answer, or click the link image(s) below to further
research the concepts in this question (if desired).

Research Concepts:
Anaplasma Phagocytophilum

We update eBooks quarterly and Apps daily based on user feedback. Please tap flag to
report any questions that need improvement.
Question 100: A 31-year-old male with previously diagnosed beta-thalassemia with a
history of multiple blood transfusions presents to the emergency department with ten days of
urinary and bowel incontinence. Vitals signs are within normal limits. The neurological exam is
normal except for hypoesthesia in the S1-S5 dermatomes bilaterally. Laboratory studies are only
remarkable for hemoglobin of 7 g/dL and a platelet count of 90,000/microL. MRI of the lumbar
spine was obtained which showed multiple enhancing lesions in the epidural space between L5-
S1 with compression of the cauda equina. Which of the following is the most likely cause of the
patient's clinical picture?

Choices:
1. Herniating disc
2. Epidural abscess
3. Tumor
4. Extramedullary hematopoiesis
Answer: 4 - Extramedullary hematopoiesis
Explanations:
Extramedullary hematopoiesis occurs in patients with thalassemia and chronic hemolytic
anemias as a compensatory mechanism. It typically occurs in sites such as the spleen, liver
and adrenal glands. It has been reported to occur in the spinal canal, causing compression of
the spinal cord and its associated structures.
A high index of suspicion must be kept for cauda equina and conus medullaris syndromes
(CES/CMS) in patients presenting with urinary or bowel retention or incontinence, as these
can be the only presenting complaints.
Laminectomy with evacuation of the hematopoietic tissue would be the appropriate
treatment to decompress the cauda equina in this case.
Fungal abscesses have also been reported as rare causes of CES/CMS.

Go to the next page if you knew the correct answer, or click the link image(s) below to further
research the concepts in this question (if desired).

Research Concepts:
Cauda Equina And Conus Medullaris Syndromes

We update eBooks quarterly and Apps daily based on user feedback. Please tap flag to
report any questions that need improvement.
Section 2
Question 101: A 25-year-old male presents with 2 days of fever, headache, altered mental
status, and involuntary movements. His wife reports that he recently returned from a summer
camp program, where he works as a camp counselor and was complaining of mosquito bites. He
has no significant medical history. He does not use tobacco or illicit drugs but drinks alcohol
occasionally. Temperature is 38C, blood pressure is 126/84 mmHg, and pulse is 72/minute. On
exam, he has involuntary chorea and writhing movements of his extremities. Which of the
following will most likely be revealed by serology testing?

Choices:
1. Herpesvirus
2. Enterovirus
3. Arbovirus
4. Rabies virus
Answer: 3 - Arbovirus
Explanations:
In the United States, arboviruses are one of the most common causes of viral encephalitis.
History, clinical presentation, and serology can be used to diagnose arbovirus encephalitis.
Arboviruses cause disease during the summer months when mosquitos are active.
Some arboviruses predominately affect the basal ganglia, causing motor symptoms such as
choreoathetosis and parkinsonian movements.

Go to the next page if you knew the correct answer, or click the link image(s) below to further
research the concepts in this question (if desired).

Research Concepts:
Viral Encephalitis

We update eBooks quarterly and Apps daily based on user feedback. Please tap flag to
report any questions that need improvement.
Question 102: A patient is being admitted to the hospital for further evaluation due to
concern for Hantavirus pulmonary syndrome after presenting with dyspnea, vomiting, and fever.
What laboratory abnormality is most significant on presentation demonstrating the need for
admission?

Choices:
1. Thrombocytopenia
2. Hypernatremia
3. Hypokalemia
4. Leukocytosis
Answer: 1 - Thrombocytopenia
Explanations:
Thrombocytopenia is often present at the stage of disease requiring admission for further
observation. This often precedes the descent into pulmonary failure by a couple of days.
Hyponatremia is more common than hypernatremia in Hantavirus pulmonary syndrome.
Potassium is not normally greatly affected in the early stages of Hantavirus pulmonary
syndrome.
Atypical lymphocytes and premature white blood cells may be present on admission, but
leukocytosis alone is not significant enough to warrant admission.

Go to the next page if you knew the correct answer, or click the link image(s) below to further
research the concepts in this question (if desired).

Research Concepts:
Hantavirus Syndrome

We update eBooks quarterly and Apps daily based on user feedback. Please tap flag to
report any questions that need improvement.
Question 103: A 65-year-old man with multiple vascular risk factors including
hypertension, diabetes, hyperlipidemia a 30-pack-year history of smoking is brought by
ambulance to the emergency department because he was found in bed unable to talk and move
his face or his extremities. He was last seen normal 5 hours before presentation. Examination
shows a blood pressure of 190/120 mmHg and pulse 90/min, regular. He is conscious and able to
respond to verbal commands by blinking. He has normal vertical eye movements but no
horizontal eye movements. He has quadriplegia with no facial movement. His NIH stroke scale
score is 28. His CT head is unremarkable. Which of the following is the next best step in the
management of this patient?

Choices:
1. Lower his blood pressure to prepare for IV thrombolysis
2. MRI brain to confirm the diagnosis of brainstem infarct
3. CT angiogram to confirm basilar thrombosis
4. Prepare for hospice and counsel the family
Answer: 3 - CT angiogram to confirm basilar thrombosis
Explanations:
The patient presented with classical locked in syndrome with basilar thrombosis as the
obvious cause. He presented beyond the 4.5 hours window.
His NIHSS score is high, also indicating large vessel occlusion. He should be a good
candidate for thrombectomy therapy.
The logical next step should be to confirm basilar thrombosis with CT angiogram in
preparation for thrombectomy.
His blood pressure needs to be lowered to below 180/110 mmHg only if he is an IV TPA
candidate. MRI brain is best in confirming brainstem infarction yet this is not as important
as CT angiogram. The patient does have a guarded prognosis; it is too soon to give up and
consider hospice.

Go to the next page if you knew the correct answer, or click the link image(s) below to further
research the concepts in this question (if desired).

Research Concepts:
Basilar Artery Thrombosis

We update eBooks quarterly and Apps daily based on user feedback. Please tap flag to
report any questions that need improvement.
Question 104: A 28-year-old female is accidentally kicked in the abdomen by her horse
and is brought to the emergency department complaining of severe abdominal pain. The triage
nurse notes her vitals are unstable and immediately consults with the emergency department
physician to assess the patient. The patient is found to have an acute abdomen, and the general
surgeon performs an immediate exploratory laparotomy. The patient has a lacerated liver,
ruptured spleen, and perforated transverse colon. The patient receives 22 units of packed red
blood cells, 12 units of fresh frozen plasma, 12 liters of crystalloid solution, and 10 units of
platelets. After surgery, she is taken to the ICU in relatively stable condition. Ten hours later, the
nurse calls the resident surgeon on call to inform them that the patient's abdomen appears tense.
Which of the following parameters warrants an immediate explorative laparotomy?

Choices:
1. Elevated intracranial pressure
2. Decreased jugular venous pressure
3. Decreased systemic vascular resistance
4. Decreased peak airway pressure
Answer: 1 - Elevated intracranial pressure
Explanations:
Physiologic cardiac sequelae include decreased cardiac output and increased central venous
pressure (CVP) due to inferior vena cava (IVC) and portal vein compression, as well as an
increased systemic vascular resistance which leads to hypotension.
Pulmonary involvement can include decreased thoracic volumes and elevated peak
pressures from compression of the diaphragm, a decrease in P/F ratio, and hypercarbia.
Renal compression can lead to decreased GFR and low urine output. Visceral blood flow is
also decreased.
Neurologic symptoms result from increased intracranial pressure from the elevated CVP
due to IVC compression; thus, contributing to a decrease in cerebral perfusion pressure
(CPP).

Go to the next page if you knew the correct answer, or click the link image(s) below to further
research the concepts in this question (if desired).

Research Concepts:
Abdominal Compartment Syndrome

We update eBooks quarterly and Apps daily based on user feedback. Please tap flag to
report any questions that need improvement.
Question 105: A 65-year-old male with a history of constipation (on laxatives) and
hyperlipidemia is admitted to the hospital with complaints of abdominal pain, vomiting, and
bright red blood per rectum. He was found to have a pseudopneumoperitoneum on imaging and
diagnosed with Chilaiditi syndrome with part of the cecum interposed between the liver and
diaphragm. He underwent nasogastric tube decompression and improved. He was discharged
with follow up. He returns to the emergency department three months later with similar
complaints and imaging confirmed cecal interposition between the liver and the diaphragm. He
undergoes nasogastric tube decompression with improvement and was again discharged with
follow up. Eight months later he was re-admitted with the same issue. Which of the following is
the next best step in the management of this patient?

Choices:
1. Exploratory laparotomy
2. Ceceopexy
3. Cecal resection
4. Colonoscopic decompression
Answer: 2 - Ceceopexy
Explanations:
Surgical intervention is indicated when the patient does not respond to conservative
management and either the obstruction fails to resolve or there is evidence of bowel
ischemia.
There is no clear consensus on the optimal surgical approach to correct bowel interposition.
Ceceopexy may be done to eliminate the possibility of recurrence in an uncomplicated cecal
volvulus unless surgical resection is indicated.
Colonic resection is the best option for volvulus of the transverse colon and attempts at
colonoscopic reduction are not recommended due to a high frequency of gangrene with
these types of volvulus.

Go to the next page if you knew the correct answer, or click the link image(s) below to further
research the concepts in this question (if desired).

Research Concepts:
Chilaiditi Syndrome

We update eBooks quarterly and Apps daily based on user feedback. Please tap flag to
report any questions that need improvement.
Question 106: A 24-year-old patient with asthma comes to the emergency department with
acute shortness of breath and wheezing. She was given nebulization, corticosteroids, and was
placed on BI-PAP. She continued to be in respiratory distress and was eventually intubated. Her
initial arterial blood gas (ABG) was pH 7, PCO2 100, and PO2 60. After two days, a sedation
vacation was attempted, and ABG was drawn on assist mode of ventilation which showed her pH
was 7.15, PCO2 80, and PO2 80 on 30% oxygen. All other labs and images are normal. The
patient is hemodynamically stable and following commands. Her negative inspiratory force
(NIF) was -40 H20. Which of the following would be considered contraindicated for pressure
support ventilation?

Choices:
1. pH-7.15, abnormal acid-base balance
2. NIF -40 cm H2O
3. Mental status
4. Normal blood pressure
Answer: 1 - pH-7.15, abnormal acid-base balance
Explanations:
Pressure support breathing is a mode of ventilation that is made up of patient-triggered,
pressure-limited, flow-cycled breaths. It is valuable to note that the patient has consistent
control over breath frequency, breath duration and flow while in a pressure support
ventilation environment. The volume of each breath is a direct result of set pressures,
patient effort, and potentially other mechanical settings that may oppose ventilation.
Patients must initiate his or her own breaths. Apneic patients are required to be in a
controlled breath environment in which the ventilator may fully sustain the respiratory
function.
It is essential that a patient be hemodynamically stable and have sufficient respiratory effort
to qualify for pressure support mode.
Patients who have a major acid-base imbalance on controlled setting should not be
considered for pressure support mode as they can become more unstable.

Go to the next page if you knew the correct answer, or click the link image(s) below to further
research the concepts in this question (if desired).

Research Concepts:
Pressure Support

We update eBooks quarterly and Apps daily based on user feedback. Please tap flag to
report any questions that need improvement.
Question 107: A seventy-eight-year-old male is noted to have right arm and right leg
weakness by his caretaker. Emergency medical services are called, and the patient is transported
to a nearby Acute Stroke Ready Hospital without neurovascular imaging capabilities. His last
known normal was 5 hours ago. Non-contrast brain CT is negative for hemorrhage. The
emergency physician suspects an acute ischemic stroke. There are no contraindications to
intravenous (IV) tPA if indicated. What is the most appropriate course of action?

Choices:
1. Admit for observation and physical therapy as the patient is outside the IV tPA treatment
window
2. Initiate IV tPA therapy and transfer to Primary Stroke Center for admission and ongoing care
3. Initiate IV tPA and transfer patient to Comprehensive Stroke Center for possible mechanical
thrombectomy
4. Transfer patient to Comprehensive Stroke Center for possible mechanical thrombectomy
without starting IV tPA
Answer: 4 - Transfer patient to Comprehensive Stroke Center for possible mechanical
thrombectomy without starting IV tPA

Explanations:
The American Heart Association and the American Stroke Association recommend IV tPA
be administered to all eligible patients as quickly as possible within 3 hours of last known
normal with an extended window of 4.5 hours in a more selective group of patients.
The American Heart Association and the American Stroke Association recommend
mechanical thrombectomy as quickly as possible to eligible patients with large vessel
occlusion within 6-16 hours of last known normal. The American Heart Association and the
American Stroke Association consider mechanical thrombectomy reasonable in select
patients within 6 to 24 hours of last known normal.
Large vessel occlusion strokes may be identified with neurovascular imaging including CT
angiography (CTA) and magnetic resonance angiography (MRA).
Patients should be transferred to a stroke center capable of identifying a large vessel
occlusion amenable to mechanical thrombectomy if they remain eligible for this
intervention.

Go to the next page if you knew the correct answer, or click the link image(s) below to further
research the concepts in this question (if desired).

Research Concepts:
Stroke Center Certification

We update eBooks quarterly and Apps daily based on user feedback. Please tap flag to
report any questions that need improvement.
Question 108: A 50-year-old male is brought to the emergency department (ED) after
being involved in a road traffic accident. On arrival, he is intubated immediately for airway
protection. His vitals on arrival include a temperature of 99.2 degrees Fahrenheit, a blood
pressure of 100/70 mmHg, a pulse of 110 beats per minute, a respiratory rate of 20 breaths per
minute, and oxygen saturation of 88% on a ventilator. CT abdomen and pelvis shows grade 4
splenic laceration with fluid in the left paracolic gutter. Laboratory investigations reveal a
hemoglobin of 9 g/dl, a white cell count of 12000/mm3, a platelet count of 250000/mm3, sodium
148 meq/dl, potassium 4 meq/dl, chloride 107 meq/dl, bicarbonate 24 meq/dl, creatinine 1 mg/dl,
and blood urea nitrogen of 20 mg/dl. In the ED, he receives 2 liters fluid and is taken to the
operating room. Open laparotomy is started under general anesthesia. After 15 minutes into the
laparotomy, patient oxygen saturation drops to 70%. The patient position is double-checked to
confirm that endotracheal tube is not kinked. What is the next best step?

Choices:
1. Flexible bronchoscopy
2. Rigid bronchoscopy
3. Deflating endotracheal tube balloon and adjust the position of the tube
4. Repeating chest x-ray
Answer: 1 - Flexible bronchoscopy
Explanations:
Flexible bronchoscopy is done to rule out any accidental intubation of aberrant bronchus.
Flexible bronchoscopy allows providers to see the relation of the endotracheal tube cuff to
the abnormal bronchus.
Based on how far the distance of the tracheal bronchus from the carina and the endotracheal
tube cuff, a decision is made to use a double-lumen endotracheal tube or use a bronchial
blocker.
After the appropriate adjustments are made, flexible bronchoscopy is repeated to confirm
the appropriate positions of the tube. Bilateral breath sounds should be audible on both sides
to ensure appropriate ventilation.

Go to the next page if you knew the correct answer, or click the link image(s) below to further
research the concepts in this question (if desired).

Research Concepts:
Tracheal Bronchus

We update eBooks quarterly and Apps daily based on user feedback. Please tap flag to
report any questions that need improvement.
Question 109: A 78-year-old female was diagnosed with deep vein thrombosis and started
on rivaroxaban seven weeks ago. Two weeks ago, she was admitted to the intensive care unit for
a gastrointestinal (GI) bleed and received andexanet alfa for rivaroxaban reversal. She is now
hemodynamically stable and is being discharged from the hospital. What is the most appropriate
next step in the management of this patient?

Choices:
1. The patient should never receive anticoagulation because she experienced a GI bleed.
2. The patient does not require anticoagulation because she completed the treatment course.
3. The patient should resume anticoagulation with rivaroxaban one month from now.
4. The patient should resume anticoagulation with rivaroxaban now.
Answer: 4 - The patient should resume anticoagulation with rivaroxaban now.
Explanations:
This patient had a recent deep vein thrombosis (DVT), requiring anticoagulation for at least
three months
The patient had a GI bleed, received a reversal agent, has stabilized, and is ready for
discharge from the hospital.
After the reversal, anticoagulation should be resumed as soon as medically appropriate to
avoid thromboembolic complications.
Given the history of DVT, this patient needs anticoagulation, which should be resumed now
that she is medically stable.

Go to the next page if you knew the correct answer, or click the link image(s) below to further
research the concepts in this question (if desired).

Research Concepts:
Andexanet Alfa

We update eBooks quarterly and Apps daily based on user feedback. Please tap flag to
report any questions that need improvement.
Question 110: A 35-year-old male with history of daily alcohol use for acute alcoholic
pancreatitis. This is his second admission to the hospital for this issue. He continues to have
abdominal pain, but his nausea and vomiting are well controlled at this time. Which of the
following interventions or treatments holds a strong level of recommendation based on moderate
quality evidence from the American Gastroenterological Association?

Choices:
1. Keep the patient on a clear liquids diet until abdominal pain completely resolves
2. Initiation of prophylactic antibiotics if his hospital stay exceeds 48 hours
3. Goal-directed fluid therapy with hydroxyethyl starch (HES) instead of normal saline or
lactated Ringer’s solution
4. Brief alcohol intervention during admission
Answer: 4 - Brief alcohol intervention during admission
Explanations:
The American Gastroenterological Association Institute guideline on initial management of
acute pancreatitis, published in 2018, recommends that patients admitted for alcoholic acute
pancreatitis receive brief alcohol intervention counseling during admission.
Studies have shown that repeated interventions at regularly scheduled periods have shown
to decrease alcohol use and decrease hospital admissions for recurrent attacks. These can be
performed at 6-month intervals during primary care or gastroenterology clinic visits by a
nurse or other trained medical professional.
These interventions should be employed along with smoking cessation counseling and
dietary optimization strategies, as these factors play a strong role in recurrence of disease
when associated with chronic alcohol use.
The most recent guidelines also recommend early initiation of enteral nutrition with a low-
fat, soft or solid diet, fluid replacement with normal saline or lactated Ringer’s and
avoidance of prophylactic antibiotics.

Go to the next page if you knew the correct answer, or click the link image(s) below to further
research the concepts in this question (if desired).

Research Concepts:
Alcoholic Pancreatitis

We update eBooks quarterly and Apps daily based on user feedback. Please tap flag to
report any questions that need improvement.
Question 111: A 38-year-old male with a significant past medical history of diabetes,
hypertension and end-stage renal disease (on dialysis) is brought to the emergency room with
shortness of breath. He is diagnosed with aspiration pneumonia and intubated due to respiratory
failure. He is on appropriate antibiotics and propofol for sedation. He was very dyssynchronous
with the ventilator, so midazolam infusion was started to achieve more sedation. After 2 days,
sedation vacation was attempted, but the patient did not wake up even after 8 hours after turning
all the sedatives off. Which of the following is the most likely explanation for the prolonged
sedation?

Choices:
1. Propofol remains for a longer time in renal failure patients
2. He developed encephalitis
3. He had a stroke while he was sedated
4. Midazolam remains for a longer time in renal failure patients
Answer: 4 - Midazolam remains for a longer time in renal failure patients
Explanations:
The medications used to initiate and maintain sedation within an intensive care unit setting
include benzodiazepines such as diazepam, lorazepam, and midazolam; opioid analgesics
such as fentanyl, hydromorphone, morphine, remifentanil, propofol, dexmedetomidine, and
ketamine; and antipsychotics such as haloperidol, quetiapine, and ziprasidone.
No sedative is found to be superior in efficacy or mortality.
However, The Society of Critical Care Medicine guidelines advises avoiding
benzodiazepines due to evidence of longer duration of intubation especially in patients with
renal impairment.
The choice of which sedative is best lies in the practitioner's clinical assessment of
individual patient scenarios, weighing the risk/benefit profile of the medicine to each
patient.

Go to the next page if you knew the correct answer, or click the link image(s) below to further
research the concepts in this question (if desired).

Research Concepts:
Sedation Vacation in the ICU

We update eBooks quarterly and Apps daily based on user feedback. Please tap flag to
report any questions that need improvement.
Question 112: A patient presents to the emergency department following an overdose of
her antihypertensive medications. She is noted to have profound bradycardia and hypotension.
Which of the following statements is true when differentiating calcium channel blocker toxicity
from beta-blocker toxicity?

Choices:
1. The electrocardiogram findings in each of the poisonings are completely different
2. An elevated serum calcium is indicative of calcium channel blocker poisoned patients
3. A normal mental status suggests calcium channel blocker overdose
4. A reliable marker for beta blocker overdose is an elevated serum glucose
Answer: 3 - A normal mental status suggests calcium channel blocker overdose
Explanations:
The vital signs and the EKG findings are not particularly helpful in differentiating between
the two classes of drugs in overdose.
In calcium channel overdose, the patient typically maintains a normal mental status. This
may be due to the neuroprotective features of calcium channel blockers.
In calcium channel overdose, the serum glucose is typically elevated, with beta blocker
overdose, the serum glucose may be high, low, or normal so this is not a reliable marker.
One of the first line treatments for calcium channel blocker overdose includes intravenous
calcium.

Go to the next page if you knew the correct answer, or click the link image(s) below to further
research the concepts in this question (if desired).

Research Concepts:
Calcium Channel Blockers

We update eBooks quarterly and Apps daily based on user feedback. Please tap flag to
report any questions that need improvement.
Question 113: A 73-year-old female is being evaluated in the clinic for four months history
of intermitted lightheadedness, dizziness, and presyncope. She has even had a syncopal episode 3
days that lasted briefly and was witnessed by her husband. The patient states that the frequency
of her symptoms is about once a week. She denies palpitations, chest pain, or trouble breathing.
Her past medical history is significant for hypertension and diabetes mellitus. Her medications
include amlodipine and hydrochlorothiazide. EKG shows sinus bradycardia with a rate of 51/min
and a normal PR interval. Which of the following is the next best step in diagnosing the patient's
underlying problem?

Choices:
1. The patient will need an event monitor for 4 weeks
2. The patient will need 48 hours Holter monitor
3. The patient will need an echocardiogram
4. No further evaluation, the patient has sinus bradycardia which is a benign finding
Answer: 1 - The patient will need an event monitor for 4 weeks
Explanations:
This patient most likely will have a diagnosis of sinus node dysfunction as she is having
inappropriate symptomatic bradycardia.
The key to establishing this diagnosis is to correlate the symptoms with abnormal EKG
tracing, which often necessitates continuous ambulatory EKG monitoring.
Most patients will require an event monitor; however, rarely select patients with frequent
symptoms can be diagnosed with a 48 hours Holter monitor.
Sinus bradycardia could be a normal finding in conditioned athletes. However, this patient
is having symptoms that are most likely related to bradycardia.

Go to the next page if you knew the correct answer, or click the link image(s) below to further
research the concepts in this question (if desired).

Research Concepts:
Sinus Node Dysfunction

We update eBooks quarterly and Apps daily based on user feedback. Please tap flag to
report any questions that need improvement.
Question 114: A 67-year-old patient with chronic obstructive pulmonary disease is on
mechanical ventilation with the following settings: assist-control (AC) 12, tidal volume (TV)
650, positive end-expiratory pressure (PEEP) 10, and FiO2 40%. The patient suddenly develops
desaturation to 75% and becomes hypotensive, diaphoretic, and tachycardic. What is the
definitive management for this patient?

Choices:
1. Needle decompression
2. Normal saline fluid bolus
3. Tube thoracostomy decompression
4. Reduce PEEP
Answer: 3 - Tube thoracostomy decompression
Explanations:
Given the high ventilator settings and clinical signs, the patient likely has an iatrogenic
pneumothorax.
Treatment should never be delayed for imaging studies.
Although needle decompression is regarded as a quick procedure that relieves air from the
pleural space and improves symptoms, it is never definitive management and should be
followed with a tube thoracostomy.
Tube thoracostomies often are placed in the fifth intercostal space above the rib to avoid
neurovascular injury and in the midaxillary line directed posterior and superior.

Go to the next page if you knew the correct answer, or click the link image(s) below to further
research the concepts in this question (if desired).

Research Concepts:
Iatrogenic Pneumothorax

We update eBooks quarterly and Apps daily based on user feedback. Please tap flag to
report any questions that need improvement.
Question 115: A 16-year-old white female presents to the emergency department with an
acute exacerbation of shortness of breath, chest tightness, and wheezing that has not responded to
her inhalers at home for over 4 hours. She is using accessory muscles to breathe and can only
speak in one or two words. She mentions that she is tired. The difference in systolic blood
pressure between inspiration and expiration is 10%. She needs 2 liters of oxygen to maintain
saturation at 94%. She is diffusely wheezy but does not have any secretions. Which of the
following treatment strategies is least likely to benefit her current disease process?

Choices:
1. Heliox with 40:60 parts of helium and oxygen
2. Use of BIPAP support
3. 125 mg of methylprednisolone followed by 60 mg every 6 hourly
4. 2 separate doses of 2 g magnesium sulfate IV an hour apart
Answer: 1 - Heliox with 40:60 parts of helium and oxygen
Explanations:
The patient appears to be in status asthmaticus. She is somewhat fatigued but still awake.
She does not have any significant secretions. Treatment with noninvasive ventilatory
support is appropriate as is methylprednisolone and magnesium at the mentioned dose.
Heliox is rarely used in status asthmaticus in patients with definitive evidence of pulsus
paradoxus with upper airway wheeze or stridor. Its ratio is 70:30 or 60:40 for oxygen to
helium.
The patient has no pulsus paradoxus which is interpreted as greater than 12% variation in
systolic blood pressure with respiration.
Heliox is avoided due to its prohibitive cost, infrequent indication and need for recalibration
of gas blenders and flow meters when used with mechanical ventilation.

Go to the next page if you knew the correct answer, or click the link image(s) below to further
research the concepts in this question (if desired).

Research Concepts:
Status Asthmaticus

We update eBooks quarterly and Apps daily based on user feedback. Please tap flag to
report any questions that need improvement.
Question 116: An 18-year-old male is brought in by emergency medical services for
confusion. He has dilated pupils, dry mucous membranes, blurry vision, tachycardia, and urinary
retention. He begins to seize. Which of the following is the most likely medication he ingested in
overdose?

Choices:
1. Acetaminophen
2. Doxylamine
3. Hydrocodone
4. Verapamil
Answer: 2 - Doxylamine
Explanations:
Pharmacologically, doxylamine has antihistamine and anticholinergic effects.
In overdose, doxylamine causes an anticholinergic toxidrome. Clinical signs and symptoms
may progress to hallucinations, rhabdomyolysis, seizures, and death.
The half-life of doxylamine is 10 to 12 hours, which may be increased in the elderly. The
median lethal dose (LD50) is 50 to 500 mg/kg.
The toxidrome for a doxylamine overdose is the same as a diphenhydramine overdose. An
acetaminophen level must always be checked initially and at four hours, as these
medications may be combined with acetaminophen in over-the-counter medications
targeted at analgesia and insomnia.

Go to the next page if you knew the correct answer, or click the link image(s) below to further
research the concepts in this question (if desired).

Research Concepts:
Doxylamine

We update eBooks quarterly and Apps daily based on user feedback. Please tap flag to
report any questions that need improvement.
Question 117: A 17-year-old male with leukemia presents to the emergency room with four
days of vomiting and voluminous diarrhea. He has had several episodes per day. He has also had
reduced oral intake but denies fever or any additional symptomatology. Serum chemistry reveals
a sodium level of 145 mmol/L, a potassium level of 1.4 mmol/L, a chloride level of 90 mmol/L,
a bicarbonate level of 20 mmol/L, and a magnesium level of 1.9 mg/dL. The physical exam does
not reveal any significant findings, and his ECG is normal. What is the next step in management?

Choices:
1. Administer potassium chloride of 40 mEq orally every 3 to 4 hours for 3 doses
2. Administer magnesium sulfate 1 gram IV over 1 to 2 hours
3. Prescribe spironolactone 25 mg orally every 8 hours
4. Administer potassium chloride of 40 mEq intravenously every 3 to 4 hours for 3 doses
Answer: 4 - Administer potassium chloride of 40 mEq intravenously every 3 to 4 hours for 3
doses

Explanations:
Rapid replacement therapy is warranted with severe hypokalemia or when clinical
symptoms are present. Potassium chloride of 40 mmol given every 3 to 4 hours for 3 doses
is preferred.
Rapid correction can be provided via oral and IV formulation. Oral supplementation is
preferred because it is associated with a lower risk of rebound hyperkalemia. However,
intravenous (IV) repletion is administered if oral therapy is not tolerated. IV administration
is also preferred in the setting of cardiac dysrhythmias, digitalis toxicity, and recent or
ongoing cardiac ischemia.
A potassium-sparing diuretic should be considered when the etiology of hypokalemia
involves renal potassium wasting as potassium replacement therapy alone may not suffice.
Pain and phlebitis usually occur with peripheral IV infusions when infusion rates exceed 10
mEq/hour. There is also a risk of rebound hyperkalemia when rates exceed a dose of 20
mEq/hour. Careful monitoring of serum potassium levels is required as the development of
hyperkalemia is common in hospitalized patients.

Go to the next page if you knew the correct answer, or click the link image(s) below to further
research the concepts in this question (if desired).

Research Concepts:
Hypokalemia

We update eBooks quarterly and Apps daily based on user feedback. Please tap flag to
report any questions that need improvement.
Question 118: A 56-year-old female with renal failure was hospitalized because she missed
two of her dialysis days due to lack of transport. The patient received her first dialysis treatment
the day before and is doing much better in the hospital, but she developed a rash on her flank that
is tender to the touch. She states it started four days ago and was much smaller than it was now.
Physical exam shows a 4 cm x 5 cm black-purple retiform purpura with central eschar on the left
flank that is tender to touch. Which of the following is the next best step in the management of
this patient?

Choices:
1. Abdominal x-ray
2. Consult dermatology for an incisional biopsy to include subcutaneous tissue
3. Consult general surgery for wide local excision
4. Initiate sodium thiosulfate
Answer: 2 - Consult dermatology for an incisional biopsy to include subcutaneous tissue
Explanations:
A consult to dermatology for an incisional biopsy to include subcutaneous fat is necessary
to make the diagnosis.
The treatment approach is interprofessional to include medicine, nephrology, and
dermatology.
Calciphylaxis has high morbidity and mortality, and early recognition is important.
Other causes of retiform purpura are important to consider, including vasculitis.

Go to the next page if you knew the correct answer, or click the link image(s) below to further
research the concepts in this question (if desired).

Research Concepts:
Calcinosis Cutis

We update eBooks quarterly and Apps daily based on user feedback. Please tap flag to
report any questions that need improvement.
Question 119: A 60-year-old male has a sudden onset of extreme dizziness, nausea, and
vomiting while at work. He is taken to the hospital for a stroke workup. On exam, he opens his
eyes to voice, is oriented to conversation, follows commands with all extremities and has
profound bilateral dysmetria of the upper and lower extremities. His blood pressure is 180/100
mmHg, heart rate of 102 bpm, and oxygen saturation of 98% on room air. Imaging reveals a
cerebellar hemorrhage of the bilateral cerebellar hemispheres with 40 mL of estimated blood
volume and effacement of the fourth ventricle with lateral ventricular enlargement. There does
not appear to be any blood in the ventricular system. What is his 30-day mortality based on the
intracerebral hemorrhage (ICH) score?

Choices:
1. 26%
2. 72%
3. 97%
4. 100%
Answer: 1 - 26%
Explanations:
The intracerebral hemorrhage (ICH) score predicts 30-day mortality based on patient with
non-traumatic intracerebral hemorrhage.
The score is based on Glasgow coma score (GCS), age, volume of hemorrhage,
intraventricular hemorrhage, and infratentorial origin of hemorrhage.
The point distribution is GCS 13 to 15 is 0 points, GCS 5 to 12 is 1 point, GCS 3 to 4 is 2
points; age 80 or greater is 1 point, ICH volume 30 mL or more is 1 point, intraventricular
hemorrhage is 1 point, and infratentorial origin of hemorrhage is 1 point.
30-day mortality is 0% for 0 points, 13% for 1 point, 26% for 2 points, 72% for 3 points,
97% for 4 points, 100% for 5 points, and assumed to be 100% for 6 points.

Go to the next page if you knew the correct answer, or click the link image(s) below to further
research the concepts in this question (if desired).

Research Concepts:
Intracranial Hemorrhage

We update eBooks quarterly and Apps daily based on user feedback. Please tap flag to
report any questions that need improvement.
Question 120: A 65-year-old male patient is being treated in the emergency department. He
is experiencing congestive heart failure. The patient has had obesity and type 2 diabetes for much
of his adult life. His diabetes is not currently being medically addressed. He has an arrhythmia
which is being managed with disopyramide. The patient has no known allergies. During the
maintenance dose of inamrinone, the patient’s blood pressure drops significantly from 149/95 to
78/55 mmHg. What is the next most appropriate course of treatment in this situation?

Choices:
1. Stop administration of inamrinone
2. Maintain the rate and concentration of inamrinone administration
3. Reduce rate and maintain concentration of inamrinone administration
4. Stop administration of disopyramide
Answer: 1 - Stop administration of inamrinone
Explanations:
Inamrinone administration should be stopped when there are significant drops in blood
pressure and the patient becomes hypotensive.
Inamrinone may be interacting with disopyramide, indicating that inamrinone
administration should be stopped.
Even a smaller dose of inamrinone may have the capacity to compound the hypotensive
symptoms.
Disopyramide is an antiarrhythmic drug taken orally. Stopping its administration would not
result in the acute improvement of blood pressure as required.

Go to the next page if you knew the correct answer, or click the link image(s) below to further
research the concepts in this question (if desired).

Research Concepts:
Inamrinone

We update eBooks quarterly and Apps daily based on user feedback. Please tap flag to
report any questions that need improvement.
Question 121: A patient presents with sudden loss of vision in his right eye that lasted 45
seconds. He felt weakness in his left arm at the same time, but the symptoms improved rapidly.
He is on metoprolol, aspirin, and atorvastatin. A workup reveals a bruit in the neck and a duplex
ultrasound reveals 90% stenosis at the right carotid artery bifurcation. Also noted is that the
internal carotid artery is severely calcified starting at the bifurcation, and this calcification
extends distally for 5 cm. If this patient undergoes angioplasty and stenting, what hemodynamic
complication is most likely?

Choices:
1. Premature ventricular complexes
2. Hypotension
3. Supraventricular tachycardia
4. Wolff-Parkinson-White syndrome
Answer: 2 - Hypotension
Explanations:
During stenting and angioplasty of the carotid artery, the patient is at increased risk for
hypotension during the procedure if there is extensive calcification.
The hypotension may persist for 24 to 72 hours. Some patients may need IV fluids and
pseudoephedrine to overcome the hypotension.
Most surgeons do not discharge these patients home until their blood pressure has
stabilized. In the intensive care unit, inotropes may need to be started.
Patients also can experience bradycardia as an effect of the stenting.

Go to the next page if you knew the correct answer, or click the link image(s) below to further
research the concepts in this question (if desired).

Research Concepts:
Carotid Artery Stenting

We update eBooks quarterly and Apps daily based on user feedback. Please tap flag to
report any questions that need improvement.
Question 122: A 65-year-old female from Southeast Asia traveled by air to Toronto and
was found to have acid-fast bacteria in the sputum. She was immediately started on rifampin.
Four days later, she was seen in the emergency department with a swollen right leg and started
on heparin for 3 days and then placed on oral warfarin. Despite immediate treatment for her
pathology, she expired 7 days later. At autopsy, she was found to have pulmonary emboli. Which
of the following most likely triggered the chain of events that led to her death?

Choices:
1. Miliary tuberculosis
2. Starting rifampin
3. Overdosing warfarin
4. Underdosing heparin
Answer: 2 - Starting rifampin
Explanations:
Rifampin is often a first-line drug used to treat tuberculosis. It is frequently used in
combination with other tuberculosis agents like isoniazid and pyrazinamide.
Rifampin is also one of the most potent inducers of the cytochrome P450 enzymes and,
thus, is often associated with drug interactions.
Rifampin can increase the breakdown of warfarin, and thus the levels become
subtherapeutic and lead to deep vein thrombosis and pulmonary emboli.
When a patient on warfarin is started on drugs like rifampin, more frequent monitoring of
the INR is mandatory or the rifampin should be discontinued.

Go to the next page if you knew the correct answer, or click the link image(s) below to further
research the concepts in this question (if desired).

Research Concepts:
Rifampin

We update eBooks quarterly and Apps daily based on user feedback. Please tap flag to
report any questions that need improvement.
Question 123: A 62-year-old male with a positive past medical history for DM, HTN, CKD
stage 3 and COPD comes in with left-sided chest pain and shortness of breath for the past 2
hours. The pain is pressure like and is radiating to the jaw. Physical examination is positive for
tachycardia, otherwise negative — Vitals, BP 160/45, Respiratory rate 32/min, Pulse 110/min,
and afebrile. Labs show mild leukocytosis and elevated cardiac enzymes. EKG shows ST-
segment elevation in lateral leads. STEMI alert is called, and the patient undergoes cardiac
catheterization. He got 2 stents in LAD and was started on appropriate medical treatment. After 3
days he is discharged home and was told to follow up with the primary care physician. Two
weeks later he undergoes sudden cardiac arrest and dies. What is the likely cause of patients
sudden cardiac death?

Choices:
1. Ventricular fibrillation
2. Pulseless electrical activity
3. Atrial fibrillation
4. Supraventricular tachycardia
Answer: 2 - Pulseless electrical activity
Explanations:
After a myocardial infarction, the risk of sudden cardiac death is highest during the first
months due to fatal tachyarrhythmias, re-infarction, or myocardial rupture.
Ventricular fibrillation (VF) and ventricular tachycardia (VT) were initially thought to be
the most common causes of out-of-hospital cardiac arrest.
More recent studies show pulseless electrical activity (PEA), and asystole as the most
frequent cause of sudden post-MI death. Approximately 50% of patients initially have
asystole, and 19% to 23% have PEA as the first identifiable rhythm.
Following the hospital discharge, the PAE leads to the sudden cessation of the blood flow to
the brain leading to the death.

Go to the next page if you knew the correct answer, or click the link image(s) below to further
research the concepts in this question (if desired).

Research Concepts:
Sudden Cardiac Death

We update eBooks quarterly and Apps daily based on user feedback. Please tap flag to
report any questions that need improvement.
Question 124: A 55-year-old obese male undergoes a diagnostic cardiac catheterization
procedure for complaints of chest pain. His cardiac catheterization revealed moderate non-
obstructive disease, and he was recommended aggressive medical therapy. Common femoral
access performed the procedure, and hemostasis was secured by manual compression. Baseline
vital signs revealed a blood pressure of 130/76 mmHg and heart rate of 70 beats per minute. Two
hours after the procedure, the patient complains of feeling nauseous, is diaphoretic and has
complaints of back pain. His blood pressure is 70/50 mmHg, and heart rate is 120 bpm. Which of
the following is the likely diagnosis?

Choices:
1. Acute stroke
2. Acute myocardial infarction
3. Retroperitoneal bleeding
4. Arteriovenous fistula
Answer: 3 - Retroperitoneal bleeding
Explanations:
Vascular access site complications are the most common complications following cardiac
catheterization procedures.
Bleeding that extends into the retroperitoneal space cause result in significant bleeding
resulting in hemodynamic instability and increases morbidity and mortality.
Retroperitoneal bleeding usually happens when the artery is accessed above the inguinal
ligament. Computed tomographic imaging or angiography may be needed for quick
diagnosis.
Treatment included reversal of anticoagulation, volume resuscitation with fluids, identifying
the bleeding vessel and with coiling or use of stent grafts. Sometimes patients may need
vascular surgical consultation and surgical repair of the bleeding vessel.

Go to the next page if you knew the correct answer, or click the link image(s) below to further
research the concepts in this question (if desired).

Research Concepts:
Cardiac Catheterization Risks and Complications

We update eBooks quarterly and Apps daily based on user feedback. Please tap flag to
report any questions that need improvement.
Question 125: A 75-year-old chronically ill male is admitted to the intensive care unit for
septic shock for presumed vancomycin-resistant enterococci (VRE) bacteremia and is started on
daptomycin. Seven days into treatment the patient reports severe muscle aches. What is the
appropriate lab test to order to determine the etiology of the patient’s pain?

Choices:
1. Procalcitonin
2. C-reactive protein (CRP)
3. Creatine kinase (CK)
4. Basic metabolic panel (BMP)
Answer: 3 - Creatine kinase (CK)
Explanations:
Patients should be evaluated for myopathy while on daptomycin by obtaining weekly
creatine kinase levels.
Daptomycin is not effective for pulmonary infections as it is inactivated by surfactant.
VRE is a rare cause of pneumonia.
Daptomycin is one of the few drugs that is bactericidal against VRE.

Go to the next page if you knew the correct answer, or click the link image(s) below to further
research the concepts in this question (if desired).

Research Concepts:
Daptomycin

We update eBooks quarterly and Apps daily based on user feedback. Please tap flag to
report any questions that need improvement.
Question 126: A 66-year-old female presents with fever, chills, hematuria, dysuria,
abdominal pain, nausea, and vomiting. Vital signs show temperature 37.8 °C, heart rate 100,
blood pressure 120/80 mmHg, and respirations 18. The cardiopulmonary exam is normal. Her
abdomen is soft with normal bowel sounds. There is significant left flank pain with suprapubic
tenderness. What is the most likely diagnosis?

Choices:
1. Diverticulitis
2. Acute pyelonephritis
3. Gastroenteritis
4. Cholecystitis
Answer: 2 - Acute pyelonephritis
Explanations:
Dysuria, fever, chills, and abdominal or flank pain are characteristic of acute pyelonephritis.
The elderly are at higher risk for urosepsis and subsequent mortality.
Pyonephrosis will need to be ruled out by imaging.
The classic features of pyelonephritis may not be seen in children or the elderly. In the
elderly, there may also be mental status changes. Gross hematuria may be seen in young
women but in males, this is unusual.

Go to the next page if you knew the correct answer, or click the link image(s) below to further
research the concepts in this question (if desired).

Research Concepts:
Acute Pyelonephritis

We update eBooks quarterly and Apps daily based on user feedback. Please tap flag to
report any questions that need improvement.
Question 127: A 72-year-old male in the intensive care unit (ICU) for respiratory failure is
receiving total parenteral nutrition (TPN) and has signs of sepsis. He has multiple risk factors for
fungemia including age, ICU stay, central venous catheter, and TPN. Cultures are positive for
Candida and treatment is started with caspofungin. The central line is removed and he responds
well to the caspofungin. What other intervention should be considered?

Choices:
1. Abdominal CT to rule out deep-seated tissue infection
2. Abdominal MRI to rule out deep-seated tissue infection
3. Ophthalmology consult and evaluation
4. Early discontinuation of therapy as the patient has improved
Answer: 3 - Ophthalmology consult and evaluation
Explanations:
All patients with candidemia should undergo an ophthalmologic evaluation to rule out
endophthalmitis.
Recommended duration of therapy is for 2 weeks after documented clearing of the infection
and resolution of symptoms.
When candidemia has been confirmed by blood cultures, treatment with an echinocandin
and central venous catheter removal should be the first steps to take.
CT scans only are warranted if deep-seated tissue infection is strongly suspected.

Go to the next page if you knew the correct answer, or click the link image(s) below to further
research the concepts in this question (if desired).

Research Concepts:
Fungemia Candidiasis

We update eBooks quarterly and Apps daily based on user feedback. Please tap flag to
report any questions that need improvement.
Question 128: A 17-year-old previously healthy female in Hawaii developed fever, chills,
pharyngitis, muscle pain, scleral injection, headache, cervical lymphadenopathy, and
photophobia lasting 7 days and was improving. The patient then seemed to relapse with
additional symptoms of nausea, headache, emesis, and moderate neck stiffness. She raises pigs.
Lumbar puncture showed 180 WBCs per microliter that were all monocytes and elevated protein.
Which of the following is true?

Choices:
1. More than half of cases are severe
2. Involvement of the central nervous system is common
3. Hepatic and renal involvement is common
4. The most common vector is the mosquito
Answer: 2 - Involvement of the central nervous system is common
Explanations:
The patient has leptospirosis, a disease acquired from rats, livestock, cats, dogs, or other
pets.
Most patients have anicteric courses, but 50% to 90% have meningeal involvement.
Fewer than 10% of cases are icteric, and these patients may also have renal involvement.
Both anicteric and icteric illnesses start with the septicemic phase, as described in the
question.

Go to the next page if you knew the correct answer, or click the link image(s) below to further
research the concepts in this question (if desired).

Research Concepts:
Leptospirosis

We update eBooks quarterly and Apps daily based on user feedback. Please tap flag to
report any questions that need improvement.
Question 129: A 72-year-old male with a past medical history significant for ischemic
cardiomyopathy with reduced ejection fraction of 25% to 30%, essential hypertension, mixed
hyperlipidemia, coronary artery disease, type II diabetes, gout, and chronic kidney disease stage
III presented with worsening dyspnea, at rest and with exertion, orthopnea, and increased lower
extremity swelling. Per his cardiologist, the patient is compliant with a low sodium diet, cardiac
rehabilitation, and his diuretic regimen. Home cardiac medications include 80 mg of oral
furosemide twice a day. He was recently started on metolazone after hospitalization for acute
decompensated heart failure last month. Despite this adjustment, the patient continues to
complain of progressive worsening of his symptoms. His vital signs showed a blood pressure of
110/70 mmHg, heart rate of 65 bpm, respiratory rate of 19/minute, temperature of 98.1 F, and
oxygen saturation was 89% on room air. Pertinent physical exam findings were elevated jugular
venous pressure with positive hepatojugular reflex and +3 pitting edema in the bilateral lower
extremities. Chest radiograph was significant for an increased cardiopulmonary ratio greater than
60% and bilateral pleural effusions with increased cephalization consistent with pulmonary
edema. Laboratory studies show sodium 132 mEq/L, potassium 3.4 mEq/L, Chloride 112
mEq/L, carbon dioxide 19 mEq/L, BUN 31 mg/dL, creatinine 1.4 mg/dL pro-BNP of 30,000
pg/mL. The patient was admitted to the hospital and was started on intravenous furosemide 120
mg BID, and his home metolazone was continued. The patient experienced minimal
improvement in symptoms. Input and output was net –200 ml in 24 hours. What would be the
next most effective way to remove fluids in the clinical scenario mentioned above?

Choices:
1. Increase the dose of intravenous loop diuretics
2. Switch scheduled IV furosemide to a continuous IV furosemide drip
3. Add IV positive inotropic agents
4. Ultrafiltration
Answer: 4 - Ultrafiltration
Explanations:
The patient is already on a high dose of IV furosemide so increasing the dose of loop
diuretics would unlikely have any further diuretic effect.
The patient is already to a high dose of IV furosemide, and therefore switching to a
furosemide drip would unlikely have any further diuretic effect.
Positive inotropic agents would have less effect that ultrafiltration in removing fluids.
Ultrafiltration is the most effective way in removing fluids among all answers in this
patient.

Go to the next page if you knew the correct answer, or click the link image(s) below to further
research the concepts in this question (if desired).

Research Concepts:
Ultrafiltration in Acute Decompensated Heart Failure

We update eBooks quarterly and Apps daily based on user feedback. Please tap flag to
report any questions that need improvement.
Question 130: A 17-year-old female arrives at the emergency department via ambulance.
On exam, she is agitated and confused. Her blood pressure is 160/110 mmHg, heart rate 140
beats/min, and temperature 39C. She is diaphoretic, tremulous, and hyper reflexive but otherwise
has a non-focal neurological exam. She is unable to give a detailed history. She has with her a
bottle of phenelzine. Her friends report that she "wanted to celebrate her eighteenth birthday"
with "something new." What other drugs may she have taken to cause this presentation?

Choices:
1. MDMA (ecstasy)
2. Marijuana
3. Phencyclidine (PCP)
4. Lorazepam
Answer: 1 - MDMA (ecstasy)
Explanations:
She is exhibiting neuromuscular effects, autonomic effects, and mental status changes
associated with serotonin syndrome.
She was taking phenelzine, a monoamine oxidase inhibitor (MAOI). The addition of
methylenedioxymethamphetamine (MDMA), also known as ecstasy, or cocaine may
precipitate serotonin syndrome.
Serotonin syndrome occurs following the use of serotonergic drugs. The degree of
symptoms can range from mild to severe. Symptoms include hyperthermia, agitation,
increased reflexes, tremors, sweating, dilated pupils, and diarrhea. Temperature can increase
to greater than 41.1C.
Serotonin syndrome typically is caused by a combination of an SSRI with other
serotonergic substances such as serotonin-norepinephrine reuptake inhibitors, MAOIs,
tricyclic antidepressants, amphetamines, buspirone, triptans, St. John's wort, ecstasy, or
cocaine. It is a predictable consequence of excess serotonin. The onset of symptoms occurs
within a day of serotonin toxicity. Treatment consists of stopping the serotonin. In those
who are agitated, benzodiazepines and a serotonin antagonist such as cyproheptadine may
be used. In those with hyperthermia, active cooling measures may be needed. With
treatment, the risk of death is less than 1%.

Go to the next page if you knew the correct answer, or click the link image(s) below to further
research the concepts in this question (if desired).

Research Concepts:
Serotonin Syndrome

We update eBooks quarterly and Apps daily based on user feedback. Please tap flag to
report any questions that need improvement.
Question 131: A 17-year-old male presents to the emergency department in Arizona with
vomiting, dyspnea, fever, and generalized myalgias. The vital signs demonstrate fever,
tachypnea, tachycardia, and mild hypoxia. The patient is found to have thrombocytopenia,
metamyelocytes, leukocytosis, and bilateral pulmonary infiltrates on chest x-ray. Which
organism is the most likely etiology?

Choices:
1. Influenza A
2. Hantavirus
3. Human immunodeficiency virus (HIV)
4. Pneumocystis jiroveci
Answer: 2 - Hantavirus
Explanations:
Hantavirus presenting as cardiopulmonary syndrome, such as in this patient, is common in
the Southwestern United States. Thrombocytopenia is the most common significant
laboratory abnormality on presentation when patients are reaching admission criteria.
Metamyelocytes may also be seen in the setting of significant leukocytosis.
Bilateral infiltrates seen on the chest x-ray may have an acute respiratory distress syndrome
appearance due to the fluffy nature due to increased capillary permeability.
Hantavirus often presents with fever, chills, diarrhea, malaise, and dyspnea.
Hantavirus is predominantly acquired through aerosolization of rodent urine and droppings.

Go to the next page if you knew the correct answer, or click the link image(s) below to further
research the concepts in this question (if desired).

Research Concepts:
Hantavirus Syndrome

We update eBooks quarterly and Apps daily based on user feedback. Please tap flag to
report any questions that need improvement.
Question 132: A 17-year-old female is brought to the emergency department by emergency
medical services. She is too delirious to give a history but alert enough to complain of stomach
pain. She is noted to be hypotensive and tachycardic. She is afebrile. Her exam is unremarkable
except for dry mucous membranes. Intravenous fluids are started. Serum chemistry shows that
the potassium is elevated and sodium is decreased. Blood glucose is borderline decreased. The
electrolytes are otherwise within normal limits. A CBC is unremarkable. After 3 liters of fluids,
the patient is still hypotensive. What is the definitive treatment for this patient's condition?

Choices:
1. Vasopressors and broad-spectrum antibiotics
2. Insulin drip
3. Fluids and corticosteroids
4. CT abdomen and pelvis
Answer: 3 - Fluids and corticosteroids
Explanations:
This presentation, lab abnormalities, and hypotension unresponsive to fluid resuscitation are
most consistent with an acute adrenal crisis. The definitive treatment for adrenal crisis is
fluids and steroids.
An insulin drip would be indicated in a patient with diabetic ketoacidosis (DKA). A patient
in DKA would be expected to have elevated potassium and decreased sodium, but the
glucose would most likely be elevated. Additionally, the sodium bicarbonate would be low
in DKA.
Vasopressors and broad-spectrum antibiotics would be indicated in a septic patient. While
this patient is hypotensive and tachycardic, she was afebrile and did not have leukocytosis.
The patient had altered mental status and complaining of abdominal pain. While it would be
reasonable to consider getting a CT of the abdomen and pelvis, it would not be the
definitive treatment of the underlying condition.

Go to the next page if you knew the correct answer, or click the link image(s) below to further
research the concepts in this question (if desired).

Research Concepts:
Adrenal Crisis

We update eBooks quarterly and Apps daily based on user feedback. Please tap flag to
report any questions that need improvement.
Question 133: A 65-year-old female, with a 30-pack-year smoking history, presents to the
emergency department with acute onset of crampy lower abdominal pain associated with nausea
and vomiting. She also has a history of rheumatoid arthritis for 15 years and has been off
treatment for the last 4. She denies any fever or weight loss or rectal bleeding. On examination,
she appears nontoxic but shows distress from the pain. The abdomen is diffusely tender, with
hypoactive bowel sounds. She has deformities in her bilateral hand joints, with some
erythematous swelling and tenderness in the metacarpophalangeal and interphalangeal joints.
Laboratory testing reveals a total leukocyte count of 11,000/microL, hemoglobin of 10.5 g/dL,
serum creatinine of 1.2 mg/dL and slightly elevated serum lactate of 2.5 mmol/L. Inflammatory
markers, including C-reactive protein and erythrocyte sedimentation rate, are elevated. CT
abdomen with contrast reveals multiple segmental stenoses and narrowing in the mesenteric
arteries. There is no evidence of any bowel necrosis or perforation. Supportive management with
IV fluids and antiemetics is started. What is the next best step in the management of this patient?

Choices:
1. Laparotomy and focal excision of the part of intestines supplied by the affected arteries
2. Laparoscopy with small intestinal and mesenteric biopsy
3. Supportive treatment only, no further treatment required
4. High-dose glucocorticoids and cyclophosphamide
Answer: 4 - High-dose glucocorticoids and cyclophosphamide
Explanations:
This patient has mesenteric vasculitis related to her rheumatoid arthritis. In patients with
rheumatoid arthritis, vasculitis is a rare but serious complication that is usually seen in
advanced stages of the disease. It is a significant cause of morbidity and mortality in these
patients.
Mesenteric vasculitis can be seen as a manifestation of rheumatoid vasculitis. These patients
usually exhibit evidence of active rheumatoid disease and systemic vasculitis. In any patient
with advanced rheumatoid arthritis presenting with abdominal pain, mesenteric vasculitis
needs to be considered in the differential diagnosis.
CT of the abdomen reveals the involvement of medium-sized arteries and arterioles, with
areas of segmental narrowing. If blood flow to the intestines is critically affected, patient's
can present with intestinal ischemia. Significant abdominal tenderness with signs of
peritonitis and lower gastrointestinal bleeding should raise clinical suspicion of the same.
In these patients, surgery is indicated if there is evidence of acute mesenteric artery
occlusion or intestinal gangrene or perforation. In the absence of the above complications,
immunosuppression is the mainstay of therapy. Supportive treatment alone, without
immunosuppressive therapy, will result in continued inflammation and worsening of
symptoms.

Go to the next page if you knew the correct answer, or click the link image(s) below to further
research the concepts in this question (if desired).

Research Concepts:
Mesenteric Vasculitis

We update eBooks quarterly and Apps daily based on user feedback. Please tap flag to
report any questions that need improvement.
Question 134: A 48-year-old female presents with epigastric pain radiating through to the
back and on physical exam has epigastric tenderness. She denies any use of alcohol or tobacco
products. Her heart rate is 115 beats/min, respirations 16 breaths/min, blood pressure 130/85
mmHg, and oxygen saturation 98% on room air. Her lipase is elevated to 13,800 units/L. It has
been decided to admit her to the hospital. Which of the following is true concerning her
prognosis at this time?

Choices:
1. Her severely elevated lipase level indicates a poor prognosis
2. CT of the abdomen is needed for accurate prognostication
3. Ranson criteria can be calculated to estimate mortality
4. The Bedside Index of Severity for Acute Pancreatitis can be used to calculate mortality
Answer: 4 - The Bedside Index of Severity for Acute Pancreatitis can be used to calculate
mortality

Explanations:
Estimating mortality of acute pancreatitis can be important at admission to guide disposition
decisions for the patient.
Lipase levels have not been shown to be an accurate indicator of the severity of acute
pancreatitis.
The Ranson criteria at admission include age, WBC, blood glucose, serum aspartate
aminotransferase, and serum lactate dehydrogenase. The criteria at 48 hours include
calcium, hematocrit fall, oxygen, BUN, base deficit, and sequestration of fluid. These can
be used to predict the severity of acute pancreatitis but cannot be used at admission to
estimate mortality.
The Bedside Index of Severity for Acute Pancreatitis (BISAP) score uses five aspects to
calculate the mortality of a patient and guide disposition. It is more commonly used now
than Ranson or APACHE II criteria because it is less cumbersome and can be used at the
time of admission. Increasing scores on the BISAP indicate higher mortality with a full
score of 5 points indicating a mortality of 22%. The five aspects of the BISAP score are as
follows: (1) BUN greater than 25 mg/dL, (2) Impaired mental status, (3 ) Fulfills two or
more of the systemic inflammatory response syndrome (SIRS) criteria, (4) Age over 60
years, and (5) Presence of pleural effusions.

Go to the next page if you knew the correct answer, or click the link image(s) below to further
research the concepts in this question (if desired).

Research Concepts:
Acute Pancreatitis

We update eBooks quarterly and Apps daily based on user feedback. Please tap flag to
report any questions that need improvement.
Question 135: A 78-year-old man with a past medical history of stroke, diabetes mellitus,
hypertension, and chronic obstructive pulmonary disease comes in from a nursing home for
evaluation of increased effort necessary for him to breathe. On initial evaluation, he appears
distressed and is placed on noninvasive positive pressure ventilation for an oxygen saturation of
80% on room air. He is found to be tachycardic and hypotensive, with a blood pressure of 80/40
mmHg. A chest X-ray shows a left lower lobe consolidation. What is the best initial antibiotic
coverage?

Choices:
1. Azithromycin
2. Levofloxacin
3. Ceftriaxone and azithromycin
4. Cefepime and vancomycin
Answer: 4 - Cefepime and vancomycin
Explanations:
Azithromycin would cover atypical pathogens but is not broad enough coverage given that
the patient is a nursing home resident.
Levofloxacin would be an appropriate treatment if he appeared well and would be treated as
an outpatient.
Ceftriaxone and azithromycin is the correct treatment for community-acquired pneumonia.
This patient is in extremis and requires broader antibiotic coverage given the risk of
multidrug-resistant organisms.
Cefepime and vancomycin are the appropriate broad-spectrum coverage. He would be
broadly covered for gram-negative and gram-positive organisms, including Pseudomonas
and methicillin-resistant Staphylococcus aureus (MRSA). In patients requiring intensive
care unit admission with risk factors for multidrug-resistant organisms, broad-spectrum
antibiotics are appropriate first-line treatment.

Go to the next page if you knew the correct answer, or click the link image(s) below to further
research the concepts in this question (if desired).

Research Concepts:
Nursing Home Acquired Pneumonia

We update eBooks quarterly and Apps daily based on user feedback. Please tap flag to
report any questions that need improvement.
Question 136: A 43-year-old male patient with past medical history notable for chronic
obstructive pulmonary disease and moderate hereditary spherocytosis presents complaining of
increased cough with sputum production for the previous week. On physical evaluation, the
patient has a blood pressure of 84/57 mmHg, heart rate 102 bpm, and respirations 24 per minute.
Auscultation reveals rales in the right lower base. Chest x-ray shows new consolidation on
bilateral lung bases. Vital signs normalize after aggressive intravenous fluid resuscitation, and
the patient is admitted for treatment of COPD. His bloodwork shows that troponin I levels
increased to 1.2 ng/mL and then normalized within the same day. Hemoglobin is 9.4 g/dL (his
baseline). His EKG did not show any acute abnormalities. Which of the following management
decisions is most adequate considering his increase in troponin?

Choices:
1. The patient should be started on treatment for acute coronary syndrome, including aspirin,
high-intensity statins, and anticoagulation
2. His increase in troponin is most likely due to demand ischemia from his underlying sepsis, and
he does currently warrant treatment for acute coronary syndrome
3. Patient warrants immediate catheterization due to his TIMI score
4. Patient requires a blood transfusion
Answer: 2 - His increase in troponin is most likely due to demand ischemia from his
underlying sepsis, and he does currently warrant treatment for acute coronary syndrome

Explanations:
This patient has a low hemoglobin baseline due to a history hereditary spherocytosis. In this
setting, an acute decrease in blood pressure may decrease the supply of oxygen to the
myocardium, which is the most likely cause of his transient increase in troponin.
Patients hereditary spherocytosis may remain asymptomatic even with low hemoglobin
levels. They may, however, have a higher predisposition to myocardial ischemia in acute
conditions where coronary blood supply decreases, such as in sepsis.
His hemodynamic status improved with administration of intravenous fluids.
Administration of blood transfusion is warranted in hemodynamically stable patients with
hemoglobin greater than 8 g/dL and without a history of underlying coronary disease.
For patients with acute coronary syndrome in the setting of anemia, an optimal transfusion
threshold is unclear. Some authors advocate for transfusing when hemoglobin is below 8
g/dL.

Go to the next page if you knew the correct answer, or click the link image(s) below to further
research the concepts in this question (if desired).

Research Concepts:
Hereditary Spherocytosis

We update eBooks quarterly and Apps daily based on user feedback. Please tap flag to
report any questions that need improvement.
Question 137: A 64-year-old male presents to the emergency department with complaints
of shortness of breath and chest palpitations. He has a significant history of hypertension, chronic
obstructive pulmonary disease (COPD), and is an active tobacco smoker. He states that he
recently had upper respiratory symptoms about one week ago, and since has progressively had
worsening shortness of breath. Medications include an albuterol inhaler, maintenance
combination inhaler, and hydrochlorothiazide. On exam, vitals are blood pressure 150/85 mmHg,
heart rate 160 bpm, respiratory rate 24/minute, and oxygen saturation 94% on 2 liters by nasal
cannula. Cardiac exam reveals tachycardia and regular rhythm. Lung exam reveals bilateral end-
expiratory wheezes. ECG reveals narrow complex tachycardia at 160 bpm with electrical
alternans. Chest x-ray reveals hyperinflated lung volumes. Which of the following is the next
step to control the patient's heart rate?

Choices:
1. Adenosine given at an initial decreased 3 mg bolus secondary to COPD
2. Vagal maneuvers
3. Diltiazem
4. Digoxin
Answer: 2 - Vagal maneuvers
Explanations:
Active wheezing is a contraindication for adenosine administration as it can cause
bronchospasm, worsening respiratory failure in these patients.
ACC/AHA/HRS guideline therapy for first-line management of supraventricular
tachycardia (SVT) in a hemodynamic stable adult is vagal maneuvers.
Other pharmacologic therapies for SVT include nondihydropyridine calcium channel
blockers and beta blockers are considered as second-line therapy in patients who fail first-
line therapy or have a contraindication for use.
Adenosine, as well as dipyridamole, are also contraindicated for pharmacologic stress
testing in patients with active wheezing secondary to the risk of bronchospasm subsequent
respiratory failure.

Go to the next page if you knew the correct answer, or click the link image(s) below to further
research the concepts in this question (if desired).

Research Concepts:
Supraventricular Tachycardia

We update eBooks quarterly and Apps daily based on user feedback. Please tap flag to
report any questions that need improvement.
Question 138: A male patient with a mechanical aortic valve is on warfarin. He now
presents to the emergency department with life-threatening bleeding. His blood pressure is in the
90s and his heart rate is 130 bpm. His INR is 6 and he has already lost about 700 ml following a
lower gastrointestinal bleed. His hematocrit is 18 and the hemoglobin is 5.4. Besides
resuscitation, what is the best approach to reverse his bleeding?

Choices:
1. Immediate 10 mg intravenous vitamin K
2. 12 units of intravenous fresh frozen plasma
3. Prothrombin complex concentrate
4. Desmopressin followed by cryoprecipitate
Answer: 3 - Prothrombin complex concentrate
Explanations:
Prothrombin complex concentrate is the ideal way to reverse serious bleeding in patients on
warfarin.
The concentrate is not always available and should only be used in life-threatening cases of
bleeding. It is used in hemophiliacs and in patients who need emergency surgery while on
warfarin.
The prothrombin complex is not used in patients with known heparin-induced
thrombocytopenia as it does contain heparin.
The complex contains clotting factors 2, 9 and 10 and some versions also contains factor 7.
Minor bleeding should be managed with vitamin K.

Go to the next page if you knew the correct answer, or click the link image(s) below to further
research the concepts in this question (if desired).

Research Concepts:
Warfarin Toxicity

We update eBooks quarterly and Apps daily based on user feedback. Please tap flag to
report any questions that need improvement.
Question 139: A 45-year-old female lawyer presents one hour after experiencing constant
projectile vomiting lasting 10 to 20 minutes just before a court hearing. She had trouble walking
out of the bathroom at the courthouse and called 911 using her cell phone for help. On arrival to
the emergency department, she is sitting up in the stretcher with her eyes closed but awake, alert,
oriented, and appropriate. Her vital signs are heart rate of 82 bpm, blood pressure 122/85 mmHg,
respiratory rate of 15/min, temperature of 98.2F, and a point-of-care glucose is 105. She says she
still feels nauseous but has not vomited in the last 30 minutes. She also complains that she feels
as though she’s spinning constantly, and her voice is slightly slurred but still readily
understandable. On exam, she has some dysmetria, and her gait demonstrates mild ataxia. She
has intact extraocular movements, no facial weakness, and vertical nystagmus. She complains of
discomfort when you palpate her abdomen, worst in the epigastrium, but no guarding or rebound.
What is the most appropriate imaging modality to identify the cause of her symptoms?

Choices:
1. Unenhanced magnetic resonance imaging (MRI) of the head
2. MRI of the head with diffusion-weighted (DWI) and angiographic (MRA) sequences
3. Unenhanced computed tomography (CT) of the head
4. Computed tomographic arteriogram (CTA) of the head and neck
Answer: 2 - MRI of the head with diffusion-weighted (DWI) and angiographic (MRA)
sequences

Explanations:
Of these options, magnetic resonance imaging including diffusion-weighted and
angiographic sequences would be most helpful to guide the evaluation of a possible cerebral
infarct because it can localize an area of perfusion deficit, a vessel occlusion, and possibly
also the presence of collateral feeding vessels. Without these sequences, MRI alone will
likely miss an early stroke.
While CTA of the head and neck can be helpful, especially if the patient had other
ipsilateral signs suggesting a larger vessel in the neck, a separate CT perfusion sequence
would also be required to provide comparable information.
CT of the head without contrast is often an important part of emergent evaluation look for a
hemorrhagic infarct, but has comparatively poor sensitivity in the posterior fossa due to
surrounding osseous structures, especially in the early phases of an infarct.
Though the patient has some epigastric tenderness, this may be due to her extensive
vomiting, and as she has normal vital signs at this time, the potential for a neurologic
emergency should be ruled out prior to an abdominal one so imaging of the abdomen would
not be indicated first.

Go to the next page if you knew the correct answer, or click the link image(s) below to further
research the concepts in this question (if desired).

Research Concepts:
Cerebellar Infarct

We update eBooks quarterly and Apps daily based on user feedback. Please tap flag to
report any questions that need improvement.
Question 140: A 45-year-old male presented with an intentional overdose of his home
regimen of antihypertensive medication. He is otherwise healthy apart from a history of
hypertension. The initial presentation was with profound refractory shock and bradycardia
requiring initiation of norepinephrine ( running at eight mcg/minute) and vasopressin. His mean
arterial pressure is 59 mm Hg. A minimally invasive solution (arterial line) that provides
dynamic and flow-based hemodynamic monitor was used to assess hemodynamics at the bedside
and his cardiac output is 5.1 liters/min with a stroke volume variability of 11%. 6 hours after the
presentation he starts becoming rapidly short of breath and was therefore electively intubated. On
low tidal volume ventilator settings with a PEEP of 5 and FIO2 of 100%, his PO2 is 82 mm Hg.
Chest x-ray shows diffuse bilateral infiltrates worse than prior. Which intervention will be most
effective at his condition to improve his outcome?

Choices:
1. Additional IV fluids as his stroke volume variation is elevated
2. Addition of milrinone as an inotrope
3. Increasing dose of insulin dextrose infusion
4. Referral for extracorporeal membrane oxygenation
Answer: 4 - Referral for extracorporeal membrane oxygenation
Explanations:
With a normal cardiac output, and significantly low P/F ratio and bilateral alveolar
infiltrates, this individual is likely developing acute respiratory distress syndrome with non-
cardiogenic pulmonary edema, a manifestation seen in rare cases of calcium channel
blocker overdose
Beyond low tidal volume mechanical ventilation, strategies that would improve clinical
outcome if instituted early will be prone ventilation or referral to extracorporeal membrane
oxygenation (ECMO).
If cardiac function is well preserved V- V ECMO should be considered without further
delay
His stroke volume variation is within 13%. cardiac output is normal. Neither additional
fluids nor inotropes will be helpful in the scenario. Fluids may worsen his pulmonary edema
and respiratory failure.

Go to the next page if you knew the correct answer, or click the link image(s) below to further
research the concepts in this question (if desired).

Research Concepts:
Calcium Channel Blocker Toxicity

We update eBooks quarterly and Apps daily based on user feedback. Please tap flag to
report any questions that need improvement.
Question 141: A 68-year-old male presents with sudden onset of back pain. He says he has
never had back pain before and denies any trauma. He says he just does not feel good and feels
like he is going to die. On examination, he looks ill with a blood pressure of 70/54 mmHg, a
pulse of 110 beats/min, and respirations 18. His abdomen is tender to palpation. There is a large
pulsatile mass. What is the next step in his management?

Choices:
1. Fast track ultrasound
2. CT scan of the abdomen
3. Angiogram of abdomen
4. Surgery
Answer: 4 - Surgery
Explanations:
In any patient who is hypotensive with signs of abdominal distension, the diagnosis of a
ruptured aneurysm must be at the top of the list.
These patients need urgent surgery. The individual should never be sent to a CT scan suite.
An ultrasound can be performed in the operating room while the patient is receiving general
anesthesia.
A ruptured aortic aneurysm has a mortality of 90%.

Go to the next page if you knew the correct answer, or click the link image(s) below to further
research the concepts in this question (if desired).

Research Concepts:
Abdominal Aortic Aneurysm

We update eBooks quarterly and Apps daily based on user feedback. Please tap flag to
report any questions that need improvement.
Question 142: A 56-year-old female is in the hospital 3 days after an aneurysmal
subarachnoid hemorrhage. She has hyponatremia with serum sodium of 130 meq/L with
decreased skin turgor and elevated hematocrit. Her urine sodium is elevated. Her vital signs are
unremarkable and chloride is within the reference range. The patient is conscious, alert, and well
oriented to time, place, and person. What is the most rational initial step in the management of
the patient presenting with such clinical characteristics?

Choices:
1. Fluid supplementation with isotonic saline
2. Fluid restriction
3. Nimodipine
4. Hypertonic saline
Answer: 1 - Fluid supplementation with isotonic saline
Explanations:
Cerebral salt wasting syndrome is characterized by hyponatremia with elevated urine
sodium in a hypovolemic state, as evidenced by decreased skin turgor, hypotension,
decreased central venous pressure, and elevated hematocrit.
Syndrome of inappropriate secretion of antidiuretic hormone (SIADH) is characterized by
hyponatremia with elevated urine sodium in a hypervolemic to euvolemic state.
Cerebral salt wasting syndrome can be differentiated from SIADH by fluid status. In
cerebral salt wasting, the patient is hypovolemic. In SIADH, the patient is hypervolemic to
euvolemic.
Cerebral salt wasting syndrome is treated with fluid resuscitation. SIADH is treated with
fluid restriction.

Go to the next page if you knew the correct answer, or click the link image(s) below to further
research the concepts in this question (if desired).

Research Concepts:
Cerebral Salt Wasting Syndrome

We update eBooks quarterly and Apps daily based on user feedback. Please tap flag to
report any questions that need improvement.
Question 143: A 50-year-old male presented with hemoptysis and melena. He had
following vitals. Blood pressure 90/60 mmHg and pulse 105 bpm. CBC shows a hemoglobin 9.8
g/dL, platelet count 120,000/microL. He was managed for a variceal bleed. Endoscopy and band
ligation were done. Which of the following drug could be used as prophylaxis in this patient to
stop further episodes?

Choices:
1. Octreotide
2. Terlipressin
3. Omeprazole
4. Propranolol
Answer: 4 - Propranolol
Explanations:
Propranolol is used as prophylaxis for variceal bleed.
It has least cardiac side effects.
Octreotide and terliprresin are used for acute management of variceal bleed.
Other vasopressors can cause coronary vasospasm and cannot be used for long term.

Go to the next page if you knew the correct answer, or click the link image(s) below to further
research the concepts in this question (if desired).

Research Concepts:
Chronic Liver Disease

We update eBooks quarterly and Apps daily based on user feedback. Please tap flag to
report any questions that need improvement.
Question 144: A 73-year-old African American female with past medical history of
congestive heart failure, atrial fibrillation, hypertension, type 2 diabetes mellitus, and
hyperlipidemia presents to the emergency department with dizziness and palpitations. The
patient gives a history of being on digoxin. On physical examination, she has a regular heart rate
of 50 bpm; blood pressure is 133/72 mmHg, respiratory rate of 18/minute, and temperature of
98.4 F. Her electrocardiogram shows junctional tachycardia with no specific ST-T wave changes.
Her laboratory testing shows toxic levels of digoxin. What is the next step in the management of
this patient?

Choices:
1. Give 1 liter of intravenous normal saline bolus and monitor the patient
2. Administer digoxin-specific antibody
3. Administer intravenous atropine and digoxin-specific antibody
4. Intravenous phenytoin
Answer: 3 - Administer intravenous atropine and digoxin-specific antibody
Explanations:
This patient is currently suffering from digoxin toxicity.
Junctional tachycardia is a manifestation of digoxin toxicity, and the rhythm should be
terminated with intravenous medications.
Intravenous atropine and digoxin-specific antibody should be administered to a patient
presenting with digoxin toxicity and bradycardia.
Intravenous phenytoin is used to treat patients who are refractory to other pharmacological
treatments, and it should be given in only monitored setting due to its side-effect of
hypotension.

Go to the next page if you knew the correct answer, or click the link image(s) below to further
research the concepts in this question (if desired).

Research Concepts:
Digoxin Toxicity

We update eBooks quarterly and Apps daily based on user feedback. Please tap flag to
report any questions that need improvement.
Question 145: A 65-year-old male presented to the emergency department with shortness
of breath. Initial arterial blood gas (ABG) showed a pH of 7.31 with a partial pressure of carbon
dioxide (PCO2) at 44 mmHg, and partial pressure of oxygen (PO2) at 45 mmHg. A chest x-ray
showed bilateral pulmonary infiltrates. Non-invasive ventilation with a bilevel positive airway
pressure mask (BiPAP) was tried without success. His blood pressure dropped to 80/40 mmHg.
He was intubated immediately. He was treated with antibiotics, vasopressors, and corticosteroids
for septic shock due to bilateral pneumonia. Over the next few days, the patient ‘s clinical status
improved, and he was no longer in need of vasopressors. The patient was placed on a
spontaneous breathing trial. After 40 minutes, his ABG shows a pH 7.39, PCO2 of 38 mmHg,
and PO2 of 63 mmHg on 60% oxygen at 5 cm of water of positive end-expiratory pressure
(PEEP). He is following commands and has mild to moderate airway secretions. Which of the
following is the next best step in the management of this patient?

Choices:
1. Avoid extubation due to high oxygen requirements and attempt weaning trial the next morning
2. Extubate patient to non-rebreather mask and consider non-invasive ventilation at night and
during daytime naps
3. Extubate patient to venturi face mask 50% and consider high flow nasal cannula if clinical
status or gas exchange deteriorates
4. Extubate patient to high flow nasal cannula
Answer: 4 - Extubate patient to high flow nasal cannula
Explanations:
High flow nasal cannula offers alternative therapy for patients with acute hypoxic
respiratory failure in the immediate post-extubation phase as it has been linked to positive
clinical and physiologic outcomes.
High flow nasal cannula allows for high oxygen supplementation without concerns for
airway secretions dryness, which could impair mucosal cilia clearance and pulmonary
toilette. Low flow Oxygen delivery systems such as face mask do not deliver humidified
oxygen, leading to drying of the secretions and potentially compromising clearance of this
patient's secretions during the immediate post-extubation period.
High flow nasal cannula, different from non-invasive ventilation, will not interfere and
facilitate airway secretion clearance.
High flow oxygen systems offer the advantage of providing PEEP to improve oxygenation
and alveolar recruitment. This mechanism of oxygenation improvement is not found with
low flow oxygen systems.

Go to the next page if you knew the correct answer, or click the link image(s) below to further
research the concepts in this question (if desired).

Research Concepts:
High Flow Nasal Cannula

We update eBooks quarterly and Apps daily based on user feedback. Please tap flag to
report any questions that need improvement.
Question 146: Clindamycin is often added to antibiotic coverage for necrotizing fasciitis
which already includes carbapenems or combinations like vancomycin with
piperacillin/tazobactam. Why is clindamycin used to treat necrotizing fasciitis?

Choices:
1. To cover Clostridium butyricum
2. To decrease the effects of tissue necrosis factor
3. To act as a bacterial protein synthesis/toxin inhibitor
4. To improve gram-negative antibiotic coverage
Answer: 3 - To act as a bacterial protein synthesis/toxin inhibitor
Explanations:
Protein synthesis inhibitors, such as clindamycin, may help by inhibiting toxin production,
which can be crucial for controlling the inflammatory response in patients with necrotizing
fasciitis.
Clindamycin’s main role is with necrotizing fasciitis caused by Group A Streptococcus.
Because the causative organism is not known at the time of antibiotic initiation,
clindamycin should be used initially in all cases of necrotizing fasciitis.
Clindamycin has been found to be beneficial even for infections caused by clindamycin-
resistant strains. It is likely due to the virulence factor-inhibiting properties of clindamycin
rather than a directly lethal effect on group A Streptococcus. Clindamycin has no Gram-
negative activity and is effective only against Gram-positives.
Clostridium species are often found in necrotizing fasciitis. Most commonly Clostridium
perfringens and Clostridium sordellii, but occasionally others. Clostridium butyricum rarely
causes disease in humans.

Go to the next page if you knew the correct answer, or click the link image(s) below to further
research the concepts in this question (if desired).

Research Concepts:
Necrotizing Fasciitis

We update eBooks quarterly and Apps daily based on user feedback. Please tap flag to
report any questions that need improvement.
Question 147: A 68-year-old male with a history of hypertension presents to the emergency
department with a sudden onset headache, nausea, vomiting, and dizziness. A CT scan of the
head is unremarkable with no intracranial hemorrhage and no acute ischemic lesions seen. He
has a non-focal neurologic exam while lying down. As soon as he stands up, he has severe
unsteadiness and falls to the floor. He is unable to walk. What is the most appropriate next step
in the management of this patient?

Choices:
1. Discharge the patient with an anti-emetic and follow up with his primary care provider the
next day
2. Order an MRI brain and MR angiogram of the intracranial and neck vessels
3. Admit him under "observation" with a diagnosis of vertigo, with neurological evaluation every
8 hours
4. Perform an urgent lumbar puncture looking for a subarachnoid hemorrhage
Answer: 2 - Order an MRI brain and MR angiogram of the intracranial and neck vessels
Explanations:
The patient has symptoms suggestive of either an acute vestibular disorder or an acute
cerebellar ischemic event, for which a CT is commonly unrevealing.
An MRI brain with diffusion-weighted imaging and an MR angiogram are the best way of
imaging the posterior fossa and posterior circulation when there is a negative CT.
Lumbar puncture is contraindicated in cases where the posterior fossa pressure gradient is
likely to be increased, as in ischemic vertebrobasilar stroke. The risk of herniation of the
edematous cerebellum in this context is high.
This patient should be admitted, with frequent neurological evaluations to observe for
mental status changes from obstructive hydrocephalus. However, the best initial step is
ordering an MRI brain and MR angiogram.

Go to the next page if you knew the correct answer, or click the link image(s) below to further
research the concepts in this question (if desired).

Research Concepts:
Ischemic Stroke

We update eBooks quarterly and Apps daily based on user feedback. Please tap flag to
report any questions that need improvement.
Question 148: A 48-year-old male comes in with altered mental status. The patient is very
agitated, and dexmedetomidine is started. After 1 hour he spikes a fever of 102 degrees F and is
still very agitated and trying to climb out of bed. He was intubated, and lumbar puncture is done.
Lumbar puncture shows no WBCs and meningitis is ruled out. His labs show acute kidney injury
with elevated CK levels. As the patient takes SSRI and baclofen, a working diagnosis of
serotonin syndrome was established. After 3 days of sedation, vacation is attempted, and the
patient becomes extremely agitated. The patient was started on antipsychotics and sedation
vacation was again attempted for the next 3 days with no difference in the patient’s mental status
and agitation. He was started on dexmedetomidine and sedation vacation was attempted again
with no help. What’s the next best step?

Choices:
1. Restart baclofen
2. Extubate on ketamine
3. Extubate on fentanyl
4. Extubate on propofol
Answer: 1 - Restart baclofen
Explanations:
The patient is likely withdrawing from baclofen, and it should be restarted. The reason for
sedation vacation or breathing trial failing should be identified and addressed accordingly.
If the patient is getting agitated and is delirious, antipsychotics can be tried; if anxious,
anxiolytics can be tried; or if in pain, low dose fentanyl drip/morphine drip or patch can be
tried.
Patient delirium and agitation can be very well controlled on dexmedetomidine, and as the
medication does not suppress the respiratory center, the patient can be extubated while on
continuous infusion of dexmedetomidine.
Attention should be paid to home medications, and if clinically permitted, a patient who is
on any anxiolytics, antipsychotics, or any chronic pain medications should continue them
while sedated or restart them in a timely fashion to prevent withdrawal.

Go to the next page if you knew the correct answer, or click the link image(s) below to further
research the concepts in this question (if desired).

Research Concepts:
Sedation Vacation in the ICU

We update eBooks quarterly and Apps daily based on user feedback. Please tap flag to
report any questions that need improvement.
Question 149: A 61-year-old woman with a history of focal seizures and right parietal
cerebrovascular accident with no notable deficits presents to the emergency department after
being found down in her apartment for an unknown length of time. She was found by a friendly
neighbor who "heard a thud." She has no evidence of head trauma but appears confused. The
patient is seen and evaluated the patient. She can follow simple commands, and her neurologic
exam reveals 5/5 strength excluding her left upper extremity (LUE) which is 1/5. It is otherwise
unremarkable. There is no evidence of swelling, pain or trauma to the extremity. CT brain
showed no evidence of intracranial hemorrhage. On rounds the next day her confusion seems to
have resolved, but her LUE weakness remains and is unchanged. The MRI brain is complete but
is unavailable for review due to what IT calls "technical difficulties." The following day on
rounds, approximately 34 hours after being found, the patient has regained 5/5 strength in her
LUE. IT has gone on vacation starting yesterday and has not fixed the "technical issue" yet.
Which of the following is the most likely cause of the patient's findings?

Choices:
1. Transient ischemic attack
2. Autoimmune myositis flare
3. Conversion disorder
4. Todd paresis
Answer: 4 - Todd paresis
Explanations:
Todd paresis is the most likely cause of her transient LUE weakness. The patient had a
history of focal seizures which are strongly associated with the condition. She had a history
of right parietal CVA which can cause scarring and may act as a focus for focal seizure
activity. Her symptoms resolved completely in 34 hours which rules out TIA as this is a less
than a 24-hour condition.
Symptoms of Todd paresis resolve, on average, by about 15 hours but can range from 30
minutes to 36 hours before resolution.
Conversion disorder is incorrect. While possible, the patient had no history given relating
psychiatric disorders of any kind or increased stressors, making this not the best answer.
Autoimmune myositis is not associated with focal, transient extremity weakness. A
transient ischemic attack is incorrect as, by definition, it is limited by 24 hours duration.

Go to the next page if you knew the correct answer, or click the link image(s) below to further
research the concepts in this question (if desired).

Research Concepts:
Todd Paresis

We update eBooks quarterly and Apps daily based on user feedback. Please tap flag to
report any questions that need improvement.
Question 150: A patient post coronary artery bypass graft (CABG) surgery presents with
shortness of breath, fatigue, peripheral edema, and ascites. ECG reveals non-specific ST changes.
The chest x-ray shows cardiomegaly with flecks of calcium around the pericardium. What is the
best treatment for this patient?

Choices:
1. Non-steroidal anti-inflammatory agents
2. Corticosteroids
3. Pericardiectomy
4. Pericardial drainage
Answer: 3 - Pericardiectomy
Explanations:
Constrictive pericarditis can occur after coronary artery bypass graft (CABG) surgery. It
causes dense adhesions of the pericardium to the epicardium.
This results in progressive dyspnea, fever, peripheral edema, and fatigue. The jugular
venous pressure (JVP) is elevated but pulsus paradoxus is rare.
Echocardiogram will show pericardial thickening but a right heart catheterization will show
equilibration of diastolic pressures and demonstrate a diastolic dip (square root sign) in right
ventricular pressure.
Pericardiectomy is the best treatment for this patient.

Go to the next page if you knew the correct answer, or click the link image(s) below to further
research the concepts in this question (if desired).

Research Concepts:
Constrictive Pericarditis

We update eBooks quarterly and Apps daily based on user feedback. Please tap flag to
report any questions that need improvement.
Question 151: A 93-year-old male is being evaluated in the emergency department after an
episode of syncope. The last thing he remembers is shopping at the grocery store and pushing the
cart. He did not sustain any injuries. Medical history is significant for hypertension. Otherwise,
he is healthy and independent. His physical exam is significant for a crescendo-decrescendo
murmur at the right sternal border with radiation to carotids bilaterally. EKG shows a right
bundle branch block (RBBB). 2D-echocardiogram confirms the diagnosis of severe aortic
stenosis. After detailed consideration, a transcatheter aortic valve replacement (TAVR) is
planned. Which of the following statements is most accurate about this treatment for this patient?

Choices:
1. He is at a higher risk for further conductions disturbance and the need for a permanent
pacemaker
2. Worsening hypertension after TAVR would worsen the overall survival
3. The mortality rate would have been the same if this patient undergoes aortic balloon
valvuloplasty
4. His risk for further conductions disturbance and the need for a permanent pacemaker is similar
to other patients without RBBB
Answer: 1 - He is at a higher risk for further conductions disturbance and the need for a
permanent pacemaker

Explanations:
Studies have shown that RBBB at baseline is associated with a high incidence of permanent
pacemaker implantation post-TAVR.
Symptomatic aortic stenosis has a high mortality if left untreated.
TAVR is a great alternative treatment in patients with symptomatic aortic stenosis.
Development or worsening of hypertension after TAVR is associated with improved
survival despite an increase in postoperative heart failure exacerbations. Patients who
undergo valvuloplasty have a higher mortality rate and it can be used as a bridge to surgical
aortic valve replacement or TAVR.

Go to the next page if you knew the correct answer, or click the link image(s) below to further
research the concepts in this question (if desired).

Research Concepts:
Transcatheter Aortic Valve Replacement

We update eBooks quarterly and Apps daily based on user feedback. Please tap flag to
report any questions that need improvement.
Question 152: A 67-year-old female is admitted to the hospital with a 1-week history of
fatigue, fever, nausea and abdominal pain. She has history of hypertension, hyperlipidemia,
diabetes mellitus and osteoarthritis. She takes metoprolol, simvastatin, glyburide, and ibuprofen
at home. She was started on amoxicillin-clavulanate for sinusitis and received a total of 10 days
of treatment one month ago. There is no history of travel in the last year. Examination reveals
jaundice. The liver is palpable 3 cm below the right subcostal margin. Lab investigations reveals
WBC count 13,400/L, hemoglobin 11.2 g/dL, platelet count 156x10^6, sodium 135 mEq/L,
potassium 4.4 mEq/L, albumin 3.5 mg/dl, aspartate aminotransferase (AST) 150U/L, alkaline
phosphatase 345 IU/L, bilirubin 1.8 mg/dl, and INR 1.0. Hepatitis panel is negative. Epstein Barr
virus and Cytomegalovirus are negative. Ultrasound reveals normal a liver and common bile
duct. Which of the following is the most likely cause?

Choices:
1. Simvastatin
2. Glyburide
3. Amoxicillin-clavulanate
4. Metformin
Answer: 3 - Amoxicillin-clavulanate
Explanations:
Simvastatin can cause elevation of alanine aminotransferase (ALT) but usually resolves
even after the continuation of the treatment. Very rarely it causes liver disease 3 years after
the start of treatment.
Glyburide can cause drug-induced hepatitis. It causes hepatotoxicity in less than 1% of
patients. This usually appears within 3 to 12 weeks of starting therapy. It can cause
hepatotoxicity months to years after changing the dose.
Metoprolol can cause elevation of aminotransferase which is transient and improves even
after ongoing treatment.
Amoxicillin-clavulanate is the most common cause of drug-induced liver toxicity and
usually presents 3 to 8 weeks after the start of treatment.

Go to the next page if you knew the correct answer, or click the link image(s) below to further
research the concepts in this question (if desired).

Research Concepts:
Liver Toxicity

We update eBooks quarterly and Apps daily based on user feedback. Please tap flag to
report any questions that need improvement.
Question 153: A 17-year-old patient was involved in a motor vehicle accident. He was
found to be hypotensive at the scene of the accident and had to be resuscitated. He suffered
multiple injuries to his lower extremities and required numerous surgeries and prolonged
mechanical ventilation. He was started on a high concentration of enteral glucose feeds on a 24-
hour protocol. Four weeks later, he is still dependent on the mechanical ventilator, but his chest
x-ray remains clear. He has profound respiratory muscle weakness and the MRI shows
significant thinning of the diaphragm. Which of the following is the most likely cause of his
muscle weakness?

Choices:
1. Hypocalcemia
2. Hypophosphatemia
3. Zinc deficiency
4. Hypernatremia
Answer: 2 - Hypophosphatemia
Explanations:
Low phosphate levels can cause significant weakness of skeletal and smooth muscle, which
can affect the eyes as well as the diaphragm.
Respiratory insufficiency in patients on ventilators is common after hypophosphatemia.
Impaired cardiac contractility can also occur and usually reverses when phosphate is
reinstituted in the diet.
Administering high carbohydrate diets can lead to a lowering of phosphate by stimulating
insulin release. Insulin moves phosphate into the cells.
Diabetic ketoacidosis is another important cause of hypophosphatemia. However, routine
replacement of phosphate in these patients has not been shown to reduce morbidity or
mortality.

Go to the next page if you knew the correct answer, or click the link image(s) below to further
research the concepts in this question (if desired).

Research Concepts:
Hypophosphatemia

We update eBooks quarterly and Apps daily based on user feedback. Please tap flag to
report any questions that need improvement.
Question 154: A patient is rushed to the medical facility in Surinam from a local small-
scale gold mine. He was working on separating the gold from amalgam when he suddenly started
coughing and became short of breath. Upon arrival, he is in respiratory distress. His vitals are
heart rate 144 bpm, respiratory rate 26/minute, blood pressure 146/92 mmHg, and pulse oximetry
82%. He is intubated. His chest x-ray is concerning for acute respiratory distress syndrome.
Resuscitation is started but what other therapy would be indicated?

Choices:
1. Sodium bicarbonate
2. British anti-Lewisite (BAL)
3. Methylprednisolone
4. Activated charcoal
Answer: 2 - British anti-Lewisite (BAL)
Explanations:
Mercury is used in the mining of gold by binding gold creating an amalgam. The mercury is
then separated from the gold by heating and creating mercury vapor.
Small-scale gold mining using mercury affects 10 to 15 million miners worldwide.
Eleven percent of human-generated mercury and 5 percent of all mercury released is
generated from gold production.
Following an inhalation, patients develop acute shortness of breath and can go into acute
respiratory distress (ARDS) rapidly. Patients should be treated aggressively and may need
to be suctioned if it is an acute aspiration.

Go to the next page if you knew the correct answer, or click the link image(s) below to further
research the concepts in this question (if desired).

Research Concepts:
Mercury Toxicity

We update eBooks quarterly and Apps daily based on user feedback. Please tap flag to
report any questions that need improvement.
Question 155: A 17-year-old, previously healthy, female presents with an acute flu-like
illness, fever, myalgias, shortness of breath, and chest pain. Her ECG shows sinus tachycardia.
Her labs show elevated white blood cells and an abnormally elevated troponin. Her chest x-ray
shows mild pulmonary edema with findings consistent with congestive heart failure. What is her
likely diagnosis?

Choices:
1. Hypertrophic obstructive cardiomyopathy
2. Brugada syndrome
3. Acute myocarditis
4. Prolonged QT
Answer: 3 - Acute myocarditis
Explanations:
Hypertrophic obstructive cardiomyopathy is an inherited, usually autosomal dominant,
cardiac disorder that may lead to sudden death, typically in young people. The chief
abnormality found is left ventricular hypertrophy with asymmetrical thickening of the
anterior interventricular septum. Classically, patients will present with exertional syncope
caused by acute left ventricular outlet obstruction and ventricular dysrhythmias with the
potential for sudden death.
Brugada syndrome is a well-known cause of sudden death secondary to a genetic mutation
in cardiac sodium channels. Also known as a sodium channelopathy, over 60 mutations
have been described. Classic ECG findings include the "Brugada sign" in type 1 Brugada
syndrome with a coved ST-segment elevation greater than 2 mm in more than one of V1-V3
followed by a negative T wave. Other types have been described as well. This abnormal
sodium channel with corresponding ECG findings can lead to sudden death. It presents as
ventricular dysrhythmias such as ventricular fibrillation and ventricular tachycardias.
Myocarditis, by definition, is an inflammatory process of the myocardium that is caused
mostly by viruses. Patients typically will present with a flu-like illness, with symptoms
including fever, malaise, myalgias, vomiting, and even diarrhea. Adults typically will
present with dyspnea, chest pain, and arrhythmias. Vital signs typically are abnormal,
including fever, tachycardia, tachypnea, and sometimes hypotension. No single sign or
symptom is specific to make the diagnosis, but a presentation with chest pain or congestive
heart failure often indicates a poor prognosis.
The QT interval is measured from the beginning of the Q wave to the end of the T wave.
The corrected QT or QTc interval is considered long if greater than 440 milliseconds in men
and 460 milliseconds in women. Any QTc greater than 500 milliseconds poses a
significantly increased risk for torsades de pointes. Causes of prolonged QT include
hypokalemia, hypomagnesemia, hypocalcemia, certain drugs, and congenital prolonged QT
syndrome. Patients often are asymptomatic but also may present with syncope or ventricular
dysrhythmias such as torsades de pointes.

Go to the next page if you knew the correct answer, or click the link image(s) below to further
research the concepts in this question (if desired).

Research Concepts:
Viral Myocarditis

We update eBooks quarterly and Apps daily based on user feedback. Please tap flag to
report any questions that need improvement.
Question 156: A 51-year-old man comes to the emergency department with worsening
shortness of breath for the past two weeks. He reports a history of human immunodeficiency
virus (HIV) infection, but he is not compliant with his antiviral or prophylactic therapy. On
physical examination, he is in tachycardic, tachypneic, and using accessory muscles with
prolonged expiratory phase and bilateral wheezing. A chest x-ray shows bilateral pulmonary
infiltrates. A computed tomogram of the chest reveals bilateral ground-glass opacities scattered
throughout both lung fields. His arterial blood gas shows pH 7.45, the partial pressure of carbon
dioxide (PCO2) of 35 mmHg, the partial pressure of oxygen (PO2) at 45 mmHg, and bicarbonate
level of 24 mmol/L. As the patient is extremely dyspneic but alert and awake, it is decided to
attempt non-invasive mechanical ventilation. A few minutes after the institution of therapy, the
patient becomes agitated, removes the non-invasive mask, and states, “ I don’t like it, I feel
claustrophobic, and it makes my breathing worse.” Which of the following is the next best step
in the management of this patient?

Choices:
1. Provide sedation and place the patient back on non-invasive ventilation while trying different
mask sizes
2. Place the patient on high flow nasal cannula and monitor his gas exchange and mental status
closely
3. Place the patient on high flow nasal cannula for preoxygenation while setting for endotracheal
intubation
4. Place the patient on Venturi face mask at 50% and continue to coach the patient
Answer: 2 - Place the patient on high flow nasal cannula and monitor his gas exchange and
mental status closely

Explanations:
High flow nasal cannula is an effective alternative for the treatment of non-hypercarbic
respiratory failure. This intervention could potentially avoid endotracheal intubation in a
highly selected patient population.
High flow nasal cannula enhances comfort as well as compliance with therapy. High flow
nasal cannula offers an alternative intervention for patients who cannot tolerate non-
invasive mechanical ventilation.
High flow nasal cannula improves oxygenation prior to intubation when compared to other
conventional oxygen supplementation systems.
High flow nasal cannula provides decrease work of breathing by decreasing airway
resistance, providing positive end-expiratory pressure (PEEP), and increase carbon dioxide
washout.

Go to the next page if you knew the correct answer, or click the link image(s) below to further
research the concepts in this question (if desired).

Research Concepts:
High Flow Nasal Cannula

We update eBooks quarterly and Apps daily based on user feedback. Please tap flag to
report any questions that need improvement.
Question 157: A 52-year-old man presents to the emergency department via ambulance
with altered mental status. His vital signs reveal blood pressure 80/55 mmHg, heart rate 127/min,
temperature 103.5 F, respiratory rate 16/min, and SpO2 95% on room air. He is unable to
provide any reliable history. Nursing staff report previous homelessness and alcohol-related
emergency department visits from his electronic medical record. Physical examination is
remarkable for a confused, disheveled male with fine diffuse tremors, severe tenderness,
erythema, crepitus, and purulent drainage from a 3 cm eschar on the ventral surface of the
scrotum. After initial resuscitation with intravenous fluids and antibiotics, what is the next best
step in the management of this patient?

Choices:
1. CT scan of the abdomen/pelvis with IV contrast
2. Surgical debridement
3. Intravenous immunoglobin (IVIG)
4. Treatment for alcohol withdrawal
Answer: 2 - Surgical debridement
Explanations:
Broad-spectrum antibiotics and intravenous fluids should be started initially with emergent
surgical consultation that coincides with resuscitation efforts.
Radiographic imaging can be helpful, demonstrating air in the soft tissues as well as the
depth of involvement but they should not be obtained if they delay the definitive surgical
treatment.
Fournier gangrene is usually due to polymicrobial infection, which includes facultative
organisms and anaerobes.
Surgical intervention, along with antibiotics, takes precedence in this case. A CT scan of the
abdomen/pelvis is not currently indicated at this time, would delay care, and increase
mortality. The administration of IVIG has been shown to confer some benefit in cases of
necrotizing soft tissue infections related to streptococcal toxic shock syndrome, a
monomicrobial infection. Clinical suspicion of alcohol dependence, which could manifest
as alcohol withdrawal syndrome, in this case, is understandable. However, management
should be focused on definitive care for the Fournier gangrene.

Go to the next page if you knew the correct answer, or click the link image(s) below to further
research the concepts in this question (if desired).

Research Concepts:
Fournier Gangrene

We update eBooks quarterly and Apps daily based on user feedback. Please tap flag to
report any questions that need improvement.
Question 158: A patient has been receiving mechanical ventilation with a balloon-cuffed
tracheostomy tube for the past 3 weeks after suffering a severe blunt laryngeal injury. On two
occasions within 24 hours, about 30 ml of blood have been suctioned from the tracheostomy
tube. What is the most likely source of the bleeding?

Choices:
1. Lung pathology
2. Granulation tissue at the tracheostomy site
3. Erosion into the internal jugular vein
4. Erosion into the innominate artery
Answer: 4 - Erosion into the innominate artery
Explanations:
Any patient with tracheal bleeding that amounts to more than blood streaking of the tracheal
aspirate, 48 hours or more after tracheostomy, must be assumed to have tracheal erosion
into the innominate artery until proven otherwise.
Episodes of such minimal blood loss tend to precede or herald the development of a
massive and frequently fatal hemorrhage. Recognition of the significance of such seemingly
minor bleeding episodes, with appropriate subsequent management, offers the best chance
for survival.
Patients with severe bleeding should be immediately re-intubated and corrective surgery
should be undertaken if the diagnosis is confirmed. Massive bleeding from the
tracheostomy site invariably comes from an erosion of the innominate artery and is usually
fatal unless rapidly controlled.
In most cases, bleeding stops temporarily with further inflation of the balloon cuff until the
patient is taken to the operating room. Sometimes the innominate artery can be compressed
against the back of the sternum with a finger introduced through the tracheostomy incision,
between the trachea and the artery.

Go to the next page if you knew the correct answer, or click the link image(s) below to further
research the concepts in this question (if desired).

Research Concepts:
Tracheo Innominate Artery Fistula

We update eBooks quarterly and Apps daily based on user feedback. Please tap flag to
report any questions that need improvement.
Question 159: A 17-year-old male is brought into the emergency department by
paramedics after jumping off a two-story building. When found, he was unresponsive on a
cement parking lot. On arrival to the emergency department, the Glasgow coma scale is 6 and the
patient is immediately intubated for airway protection. On initial evaluation, there is a large area
of ecchymosis behind his right ear. Which of the following sets of vital signs indicates
impending brain herniation?

Choices:
1. A heart rate of 130 beats per minute, blood pressure of 90/50 mmHg, and respiratory rate of
30 breaths per minute
2. A heart rate of 115 beats per minute, blood pressure of 90/50 mmHg, and respiratory rate of
22 breaths per minute
3. A heart rate of 70 beats per minute, blood pressure of 205/120 mmHg, and respiratory rate of
23 breaths per minute
4. A heart rate of 48 beats per minute, blood pressure of 165/70 mmHg, and respiratory rate of
10 breaths per minute
Answer: 4 - A heart rate of 48 beats per minute, blood pressure of 165/70 mmHg, and
respiratory rate of 10 breaths per minute

Explanations:
Cushing reflex is a physiological response to increased intracranial pressure, which is seen
in severe traumatic brain injury.
Cushing triad is a late sign of impending herniation.
Cushing triad includes bradycardia, hypertension, and respiratory depression. In children, it
can cause either bradycardia or tachycardia.
The presence of Cushing triad is an ominous condition and requires urgent evaluation and
subsequent intervention.

Go to the next page if you knew the correct answer, or click the link image(s) below to further
research the concepts in this question (if desired).

Research Concepts:
Cushing Reflex

We update eBooks quarterly and Apps daily based on user feedback. Please tap flag to
report any questions that need improvement.
Question 160: A 41-year-old male presents to the emergency department for acute onset
chest palpitations and discomfort. He states he has had similar episodes in the past; however,
they usually resolve without intervention, and he has never seen a provider for it. He has no other
known medical history and takes no medications. On examination, his blood pressure is 150/90
mmHg, heart rate 190 beats/min with regular rhythm, respiratory rate 24, and oxygen saturation
99% on room air. He appears to be in mild distress. His EKG shows significant tachycardia, a
narrow complex at a ventricular rate of 180, and evidence of electrical alternans. Adenosine is
planned to be administered. Which of the following should be discussed with the patient before
administration?

Choices:
1. Adenosine can cause bowel irritation
2. Adenosine has been known to cause permanent central nervous injury
3. Most patients experience facial swelling and chest palpitations over the next 24 hours
4. He can expect to have facial flushing and chest discomfort, but symptoms are transient and
rapidly resolve
Answer: 4 - He can expect to have facial flushing and chest discomfort, but symptoms are
transient and rapidly resolve

Explanations:
Facial flushing and chest pain are common with adenosine treatment. However, given the
very short half-life of adenosine, these symptoms subside rapidly.
Tremors and sympathetic overactivity occur with adrenergic agonist use.
Because the symptoms experienced with the administration of adenosine can be frightening,
patients should be informed of the side effects prior to administration.
Other side effects of adenosine include dizziness, difficulty breathing, diaphoresis, and jaw
discomfort.

Go to the next page if you knew the correct answer, or click the link image(s) below to further
research the concepts in this question (if desired).

Research Concepts:
Adenosine

We update eBooks quarterly and Apps daily based on user feedback. Please tap flag to
report any questions that need improvement.
Question 161: Three days after admission for an acute myocardial infarction, a 76-year-old
man complains of severe, unremitting abdominal pain. Physical examination demonstrates pain
that is out of proportion to the findings. Laboratory testing is significant for serum lactate level
of 30 mg/dL (normal: 4.5-19.8 mg/dL) and white blood cell count 23,000 cells/uL. Serum
creatinine level is 1.1 mg/dl. Which of the following is the most appropriate next step in
management?

Choices:
1. Duplex ultrasonography
2. Mesenteric CT-angiography
3. Video laparoscopy
4. Abdominal CT with and without IV and oral contrast
Answer: 2 - Mesenteric CT-angiography
Explanations:
Intestinal ischemia is characterized by severe abdominal pain without significant findings
on physical examination. The ischemia may be secondary to mesenteric vasospasm,
occlusion of the mesenteric vessels by thrombotic or embolic events, or a low cardiac index.
Mesenteric angiography is the gold standard for diagnosis. Direct infusion of vasodilators or
thrombolytic therapy can be used if there is suspicion of ischemic bowel.
Embolectomy or vascular bypass may be needed. If there is no occlusion, there should be
attempts to increase cardiac output and dilate the mesenteric vessels.
Abdominal CT should not be performed with oral contrast. Oral contrast will obscure the
mesenteric vessels and bowel wall, causing a delayed diagnosis.

Go to the next page if you knew the correct answer, or click the link image(s) below to further
research the concepts in this question (if desired).

Research Concepts:
Mesenteric Artery Ischemia

We update eBooks quarterly and Apps daily based on user feedback. Please tap flag to
report any questions that need improvement.
Question 162: A patient in the ER was noted to have a significant renal contusion on
imaging after he fell from a ladder. His vitals are stable and his labs are normal. What is the next
step in management?

Choices:
1. Observation in the ICU with serial clinical exams, vital signs, and hematocrits every 6 to 8
hours
2. Discharge home with outpatient follow up with his primary care doctor
3. Admit patient to the floor for observation with follow-up hematocrit every 12 hours
4. Admit the patient to the floor and obtain a CT scan 12 hours after admission
Answer: 1 - Observation in the ICU with serial clinical exams, vital signs, and hematocrits
every 6 to 8 hours

Explanations:
Non-operative management in the hemodynamically stable renal trauma patient has a better
overall outcome.
Non-operative management initially entails supportive care in an intensive care unit (ICU)
setting, serial clinical exams, serial hematocrits every 6 to 8 hours, transfusion of blood
products and angioembolization or drain placement for urine leakage as indicated.
Initial non-operative approach renal function can be preserved and unnecessary
nephrectomies avoided.
Patients with grade IV or V injuries, if managed non-operatively, should have a repeat
contrast-enhanced CT 48 to 72 hours after the initial scan or earlier if clinically warranted to
evaluate for the common complications of bleeding or urinoma in these high-grade injuries.

Go to the next page if you knew the correct answer, or click the link image(s) below to further
research the concepts in this question (if desired).

Research Concepts:
Kidney Trauma

We update eBooks quarterly and Apps daily based on user feedback. Please tap flag to
report any questions that need improvement.
Question 163: A 17-year-old male immigrant presents with a 6-month history of dyspnea
on exertion. He complains of swollen legs. He had tuberculosis as a child that was treated with
four drugs. ECG is normal. CT of the chest shows pericardial calcifications. Which of the
following would not be expected on an exam?

Choices:
1. Pulsus paradoxus
2. Lack of decline in jugular pressure with inspiration
3. Ascites
4. Cannon A waves in the jugular veins
Answer: 4 - Cannon A waves in the jugular veins
Explanations:
The patient has congestive heart failure secondary to constrictive pericarditis.
Physical examination findings in constrictive pericarditis include pulsus paradoxus and lack
of decline in jugular pressure on inspiration.
Ascites can be present with right-sided heart failure.
Cannon A waves are seen with atrioventricular dissociation when the atria contracts against
a closed valve.

Go to the next page if you knew the correct answer, or click the link image(s) below to further
research the concepts in this question (if desired).

Research Concepts:
Constrictive Pericarditis

We update eBooks quarterly and Apps daily based on user feedback. Please tap flag to
report any questions that need improvement.
Question 164: A 65-year-old man with a history of bitemporal hemianopsia is undergoing a
surgical evaluation. MRI of the head and neck reveals a craniopharyngioma. Transsphenoidal
hypophysectomy is performed without apparent complications. Two hours after surgery, he
develops polyuria (375 mL/h) and hypernatremia (155 mEq/L). The patient describes severe
thirst and seems lethargic and disoriented. Which of the following is the next best step in the
management of this patient?

Choices:
1. Indomethacin
2. Amiloride
3. Intravenous desmopressin
4. Hydrochlorothiazide
Answer: 3 - Intravenous desmopressin
Explanations:
Diabetes insipidus is a common complication following pituitary surgery. Diabetes
insipidus (DI) is a disease process that results in either decreased release of or response to
antidiuretic hormone (ADH, also known as vasopressin or AVP), which can cause
electrolyte imbalances.
Most cases of diabetes insipidus following transsphenoidal hypophysectomy are transient.
About 20% of patients who undergo neurosurgery will present with diabetes insipidus.
Treatment is with fluid replacement and desmopressin. The most common findings in
patients with diabetes insipidus are polydipsia, polyuria, and nocturia.
Risk factors for developing diabetes insipidus due to transsphenoidal hypophysectomy
include craniopharyngioma or extensive intraoperative handling of the pituitary gland.

Go to the next page if you knew the correct answer, or click the link image(s) below to further
research the concepts in this question (if desired).

Research Concepts:
Diabetes Insipidus

We update eBooks quarterly and Apps daily based on user feedback. Please tap flag to
report any questions that need improvement.
Question 165: A 17-year-old female is admitted with status asthmaticus. She is treated with
continuous albuterol nebulization, intravenous corticosteroids, and bilevel positive airway
pressure mechanical respiration. The patient's respiratory status improves, but she develops
generalized weakness, fatigue, and myalgias. The cardiac monitor shows prolonged QT interval,
flattened T waves, and ST depression. What is the most likely etiology of these changes?

Choices:
1. Hypocalcemia
2. Hypokalemia
3. Corticosteroid-induced myopathy
4. Cardiac ischemia
Answer: 2 - Hypokalemia
Explanations:
Beta-agonists, such as albuterol, can cause hypokalemia.
This is secondary to insulin secretion and its action on the NaK ATPase. This increases the
cellular uptake of potassium.
Initially, this can cause fatigue, muscle weakness, and myalgias but can progress to
complete paralysis and hypoventilation.
Intravenous replacement of potassium is limited by pain and sclerosing of veins.

Go to the next page if you knew the correct answer, or click the link image(s) below to further
research the concepts in this question (if desired).

Research Concepts:
Status Asthmaticus

We update eBooks quarterly and Apps daily based on user feedback. Please tap flag to
report any questions that need improvement.
Question 166: A 54-year-old man working in the sanitation department presents with
acute-onset shortness of breath. He says that over the past two days, he has suddenly become
very short of breath. He denies smoking or any recent illness. His past medical history is
unremarkable. He is admitted and requires immediate mechanical ventilation. Chest x-rays show
diffuse, bilateral air-space opacities. Two days later, he expires. The pathology shows diffuse
alveolar damage and fibrosis. What is the most likely diagnosis?

Choices:
1. Vaping associated pulmonary injury
2. Acute interstitial pneumonia
3. Cryptogenic organizing pneumonia
4. Bird flu
Answer: 2 - Acute interstitial pneumonia
Explanations:
Acute interstitial pneumonia (also known as Hamman-Rich syndrome) is an acute, rapidly
progressive idiopathic pulmonary disease that often leads to fulminant respiratory failure
and acute respiratory distress syndrome (ARDS).
It can be distinguished clinically from other types of interstitial pneumonia by the rapid
onset of respiratory failure in a patient without preexisting lung disease. Chest x-ray usually
shows a pattern that is similar to ARDS, which is diffuse, bilateral air-space opacities.
Acute interstitial pneumonia (AIP) has the histopathological pattern of diffuse alveolar
damage (DAD) that is indistinguishable from the histologic pattern found in ARDS.
The histologic discerption depends on the timing of the biopsy. The early phase (within a
week of the initial tissue injury) is exudative, characterized by edema in the interstitium and
alveolus. After that, a late phase, which is also called the organizing phase, shows
fibroblastic proliferation and type 2 cell hyperplasia.

Go to the next page if you knew the correct answer, or click the link image(s) below to further
research the concepts in this question (if desired).

Research Concepts:
Acute Interstitial Pneumonia

We update eBooks quarterly and Apps daily based on user feedback. Please tap flag to
report any questions that need improvement.
Question 167: A 49-year-old female presents with 8 hours of progressive epigastric pain
that worsens when recumbent and with radiation to her back with associated nausea. The patient
is well known at your institution and has a well-documented history of alcohol abuse. On your
evaluation, the patient’s vitals show temperature of 100.6 F, heart rate of 123 beats per minute,
blood pressure of 95/60 mmHg, respiratory rate of 16 breaths per minute and saturating at 90%
on ambient air. The physical exam reveals an uncomfortable and ill-appearing patient from
whom it is difficult to elicit a history as she drifts off between sentences. Breath sounds are
slightly diminished over the left posterior lung fields, and she displays epigastric tenderness with
guarding but without rigidity or rebound tenderness. Bowel sounds are decreased but present.
The patient’s laboratory results show hemoglobin of 17.0 g/dL, hematocrit of 58.0%, white
blood cell count of 17 x 10^9/microL and platelets of 135,000/microL. The metabolic shows
alanine aminotransferase (ALT) 62 U/L, aspartate aminotransferase (AST) 128 U/L, and alkaline
phosphatase of 135 U/L with blood urea nitrogen of 34 mg/dL, and serum creatinine of 1.20
mg/dL. Lipase is 2524 U/L. The chest x-ray is significant for mild to moderate left pleural
effusion without visualization of free air under the diaphragm and abdominal ultrasound without
dilatation of the common bile duct or presence of any stones in the visualized biliary ducts but is
unable to visualize the pancreas due to overlying bowel gas. What is the next best step in the
management of this patient?

Choices:
1. Order CT of the abdomen with contrast to establish diagnosis of acute pancreatitis
2. Administer fluid bolus and admit to keep nil per os and for pain control
3. Start patient on piperacillin/tazobactam for empiric coverage of likely pancreatic necrosis
4. Admit patient to the intensive care unit for hydration with monitoring of fluid resuscitation
with complete blood count and metabolic panel every 6 hours
Answer: 4 - Admit patient to the intensive care unit for hydration with monitoring of fluid
resuscitation with complete blood count and metabolic panel every 6 hours

Explanations:
The Bedside Index of Severity for Acute Pancreatitis (BISAP) score can be used to predict
mortality in acute pancreatitis and has been shown to be similar in prognosticating mortality
no other previous more complicated scoring systems such as Ranson's criteria or the
APACHE II score. High scoring patients require intensive care unit admission.
This patient scores high on the Bedside Index of Severity for Acute Pancreatitis (BISAP)
with 4/5 points indicating a high risk of mortality and the patient will require close
monitoring.
Patients with 0 points have a less than a 1% risk of mortality while mortality increases
significantly with a score of 3 or more. A score of five points is associated with a 22%
mortality. In this question, the patient scores 4 out of 5 points and has a high risk for
mortality
In severe pancreatitis, BUN and hematocrit should be followed every 6 hours to ensure
adequate fluid resuscitation. The first 12 to 24 hours of resuscitation are most crucial in
improving outcomes.

Go to the next page if you knew the correct answer, or click the link image(s) below to further
research the concepts in this question (if desired).

Research Concepts:
Acute Pancreatitis

We update eBooks quarterly and Apps daily based on user feedback. Please tap flag to
report any questions that need improvement.
Question 168: A 65-year-old female presents with orthopnea, anorexia, and bilateral lower
extremity edema. Physical exam reveals cachexia, jugular venous distention, a holosystolic,
high-pitched, blowing murmur, bilateral rales, and hepatomegaly. Blood work shows an elevated
total bilirubin, aspartate aminotransferase, and alanine aminotransferase. What is the best
treatment for the suspected condition?

Choices:
1. Metoprolol
2. Digoxin
3. Furosemide
4. Nifedipine
Answer: 3 - Furosemide
Explanations:
Diuretics are the first-line option for cardiac cirrhosis.
Aggressive diuresis is often required to reduce the edema.
However, one must treat the primary cause of cardiac failure.
There is no role of the use of beta blockers acutely in the setting of acute decompensated
cardiac cirrhosis, especially if acute heart failure or cardiogenic shock could be the potential
cause of cardiac cirrhosis.

Go to the next page if you knew the correct answer, or click the link image(s) below to further
research the concepts in this question (if desired).

Research Concepts:
Cardiac Cirrhosis

We update eBooks quarterly and Apps daily based on user feedback. Please tap flag to
report any questions that need improvement.
Question 169: A 50-year-old male is admitted to the intensive care unit with acute
respiratory distress syndrome. The patient is intubated and on mechanical ventilation. Chest x-
ray shows bilateral infiltrates with subcutaneous emphysema, pneumomediastinum, and
pneumopericardium. Vital signs are within normal limits with no desaturation. What is the
clinical intervention necessary at present?

Choices:
1. Pericardiocentesis
2. Needle thoracostomy
3. Observe and monitor with serial chest x-rays
4. Extubate patient
Answer: 3 - Observe and monitor with serial chest x-rays
Explanations:
The patient has subcutaneous emphysema, pneumomediastinum, and pneumopericardium.
The air leak syndrome is not causing any hemodynamic instability. The best therapeutic
option at present is clinical observation and monitoring with chest x-rays.
Pneumomediastinum radiologically can be seen as a column of air in the mediastinum.
Pneumopericardium is seen as a collection of air around the pericardial sac on routine chest
X-ray
Needle thoracostomy is the treatment for tension pneumothorax.
Pericardiocentesis is the evacuation of fluid from the pericardial space. This is not indicated
for pneumopericardium, which just requires monitoring.

Go to the next page if you knew the correct answer, or click the link image(s) below to further
research the concepts in this question (if desired).

Research Concepts:
Air Leak

We update eBooks quarterly and Apps daily based on user feedback. Please tap flag to
report any questions that need improvement.
Question 170: A 50-year-old male with a history of hypertension, diabetes mellitus, and
chronic tobacco use presents to the emergency department with shortness of breath. The patient's
symptoms have been progressively worsening over the past week. Associated symptoms include
fevers, chills, cough, and fatigue. His wife was having similar symptoms, but hers have resolved.
Vital signs show a blood pressure of 113/68 mmHg, respiratory rate of 25/min, heart rate
110/min, temperature 39 C, and pulse oximetry 88% on a non-rebreather mask. On physical
exam, the patient is tachypneic with increased work of breathing. Lung auscultation reveals
diffuse crackles on bilateral lung fields. The rest of the physical exam was unremarkable. Chest
X-ray shows extensive bilateral infiltrates. The patient is intubated and placed on mechanical
ventilation, with tidal volume 500 mL, respiratory rate 20/min, FiO2 100%, and PEEP 5. Arterial
blood gas on these settings shows pH 7.20, pO2 100 mmHg, pCO2 60 mmHg. The next morning
patient is noted to have worsening hypoxemia, and arterial blood gas shows pO2 50 on FiO2
100% and PEEP of 10. The patient is placed on a venovenous extracorporeal membrane
oxygenator (VV-ECMO) at a flow of 3.5 L/min, a sweep of 3, and a repeat arterial blood gas is
drawn, which shows a pH 7.35, pO2 120 mmHg, pCO2 42 mmHg. What is the next best step in
the management of this patient?

Choices:
1. Continue current management, no changes necessary
2. Decrease the tidal volume
3. Increase the PEEP to 15
4. Make arrangements for prone ventilation
Answer: 2 - Decrease the tidal volume
Explanations:
VV-ECMO is not therapeutic on its own, but it is temporary support to help with
oxygenation as the patient has severe ARDS and is unable to oxygenate or ventilate
properly. While the patient is on this support, the ventilator should be placed on minimum
settings to decrease the amount of insult to the lungs due to the ventilator. The patient
should be on low tidal volume ventilation, between 4 and 6 ml/kg of ideal body weight.
This will help with the healing process of the lungs by avoiding further insults from the
ventilator.
The partial pressure of carbon dioxide (pCO2) can be managed directly with the VV-ECMO
by changing the sweep gas as needed to increase the removal of CO2. This means that we
do not need to increase the tidal volumes on the ventilator, which could lead to volutrauma
and worsening of ARDS.
While on VV-ECMO, we still follow guidelines from the ARDSnet trial regarding
oxygenation. The aim is to target a pO2 of 55mmHg or pulse oximetry of 88%. As long as
the patient is above these parameters, there is no need to increase the fraction of inspired
oxygen, which leads to the formation of harmful free radicals, and can avoid increasing
PEEP, which could lead to barotrauma.
VV-ECMO is the last resort for patients with worsening respiratory failure that is refractory
to medical treatment. Before placement on VV-ECMO patient should have been having
worsening respiratory parameters while on optimal management, including low lung
volume ventilation strategy, neuromuscular blockade, and while prone. If this fails, then the
patient is a candidate for VV-ECMO. There is no advantage to proning the patient while he
is on VV-ECMO.

Go to the next page if you knew the correct answer, or click the link image(s) below to further
research the concepts in this question (if desired).

Research Concepts:
Extracorporeal Membrane Oxygenation In Adults

We update eBooks quarterly and Apps daily based on user feedback. Please tap flag to
report any questions that need improvement.
Question 171: A 16-year-old female sustains a circumferential burn of the left forearm in a
campfire accident, with additional burned areas along the left side of her body. There was no
significant impact. While assessing her total body surface area affected and evaluating her need
for formal fluid resuscitation, she states that her left hand is developing “excruciating, deep,
achy” pain. The initial inspection is normal. The radial or ulnar pulses on the left wrist are not
palpable. Upon touching the patient’s hand, she screams in agony, and the posterior forearm is
taut to palpation. What is the next best step in the management of this patient?

Choices:
1. Proceed with fluid management before addressing the extremity
2. Escharotomy is indicated before fluid resuscitation begins
3. Warm and cool compresses should be initiated after fluids are started
4. The forearm needs to be splinted after fluids are started
Answer: 2 - Escharotomy is indicated before fluid resuscitation begins
Explanations:
Compartment syndrome is a limb-threatening complication of circumferential burns.
In situations of compartment syndrome, all restrictive dressings such as splints or casts
should be removed immediately.
Although occult fracture can cause compartment syndrome, the patient’s injury was not
traumatic, and inflammation alone will not cause fracture.
Escharotomy and fasciotomy should be performed before starting fluid resuscitation
because the resulting edema will worsen the compartment syndrome.

Go to the next page if you knew the correct answer, or click the link image(s) below to further
research the concepts in this question (if desired).

Research Concepts:
Acute Compartment Syndrome

We update eBooks quarterly and Apps daily based on user feedback. Please tap flag to
report any questions that need improvement.
Question 172: A 55-year-old male was admitted 2 weeks ago for medical management of
pancreatic ascites after presenting with new onset abdominal distention in the setting of a
personal history of chronic pancreatitis and alcohol abuse. The patient has shown very little
improvement and continues to require repeated therapeutic paracentesis. At this time, it is
decided to proceed with endoscopic retrograde cholangiopancreatography (ERCP) and
cholangiography to better characterize the presumed pancreatic duct leak. During
cholangiography the pancreatic duct displays opacification throughout the head and neck of the
pancreas upon injection of contrast, however, in the tail of the pancreas, there is significant
extravasation of the contrast and no further visualization of the pancreatic duct beyond that point.
What is the next best step in management at this time?

Choices:
1. Attempt stent placement to bridge the ruptured pancreatic duct
2. Continue with medical management as the pancreatic duct rupture is distal enough that the
smaller caliber duct has high chances of successful healing without intervention
3. The patient needs surgical intervention. Consult surgery for further management
4. Endoscopically inject N-butyl-2-cyanoacrylate (endoscopic sealant) to obstruct the fistula
communicating with the peritoneum
Answer: 3 - The patient needs surgical intervention. Consult surgery for further management
Explanations:
Management of pancreatic ascites usually begins with medical management for up to two
weeks if the patient is tolerating it well. Longer than two weeks, it is thought that the patient
begins to be at higher risk for mortality and morbidity, particularly from infectious causes.
Once a patient has undergone up to 2 weeks of conservative management without
improvement, ERCP is recommended to further characterize the pancreatic duct leak and in
most places to place a transpapillary stent to bridge the duct disruption or to drain any
communicating pseudocysts.
Disconnected pancreatic duct syndrome is where there is complete pancreatic duct
disruption with no further reconstitution of ductal structures distal to the point of contrast
extravasation on cholangiography. In this case, surgical intervention is required for
anastomosis of the distal pancreatic duct to the jejunum to allow for drainage of pancreatic
juices.
Endoscopically bridging pancreatic duct lesions which are completely disrupted via stenting
is an area where there is little available data and therefore is not currently recommended.
Use of endoscopic sealants is also an area of ongoing research and is not broadly
recommended, though has been used with some success to avoid surgery by some European
groups. Use of these sealants would not be recommended in a completely disrupted duct,
however.

Go to the next page if you knew the correct answer, or click the link image(s) below to further
research the concepts in this question (if desired).

Research Concepts:
Pancreatic Ascites

We update eBooks quarterly and Apps daily based on user feedback. Please tap flag to
report any questions that need improvement.
Question 173: A 50-year-old man presents for evaluation of confusion for 1 day. He has a
past medical history of hypertension and seizure disorder. Vital signs are heart rate 75 bpm,
blood pressure 130/85 mmHg, respiratory rate 8/minute, and pulse oximetry 92% on room air.
The patient is somnolent but arousable. He is disoriented. Neurologic exam shows nystagmus.
Lab workup and CT head without IV contrast are within normal limits. He is observed for 24
hours with gradual improvement in mental status. He later admits to taking his entire monthly
dose of levetiracetam in a suicide attempt. What is the appropriate treatment for acute
levetiracetam toxicity?

Choices:
1. Benzodiazepines
2. Supportive care
3. Dialysis
4. L-carnitine
Answer: 2 - Supportive care
Explanations:
The therapeutic serum concentration of levetiracetam is 12 to 46 mg/L. The toxic
concentration is not well defined.
Levetiracetam toxicity commonly presents with central nervous system depression,
nystagmus, respiratory depression. Supportive care including fluid resuscitation,
supplemental oxygen, and intubation is the primary method of treatment.
Levetiracetam is not metabolized by the CYP P450 system. It has little to no interaction
with other anticonvulsants.
Thorough laboratory and imaging workup are necessary in the setting of levetiracetam
toxicity as the presentation can mimic other neurologic and metabolic conditions.

Go to the next page if you knew the correct answer, or click the link image(s) below to further
research the concepts in this question (if desired).

Research Concepts:
Anticonvulsants Toxicity

We update eBooks quarterly and Apps daily based on user feedback. Please tap flag to
report any questions that need improvement.
Question 174: A 16-year-old male presents to the emergency department 12-hours after
having his right forearm run over by a car. His blood pressure is 135/90 mmHg, and heart rate is
110 beats/minute. Radiographs of the right forearm show no fractures. His forearm is very
swollen but soft and compressible. A pressure monitor is used, which shows intra-compartmental
pressure of 35 mmHg. The patient does not have pain with passive extension of his fingers or
wrist. Which of the following is the next best step in the management of this patient?

Choices:
1. Immediate transfer to the operating room for the release of compartments
2. Ice and elevation with serial clinical compartment checks in the hospital
3. Soft compression bandage application, discharge home with instructions to ice and elevate
once the patient is comfortable
4. Repeat pressure measurement in 1 hour
Answer: 2 - Ice and elevation with serial clinical compartment checks in the hospital
Explanations:
This patient's clinical exam is not concerning for compartment syndrome. His absolute
pressure is greater than 30 mmHg, but delta pressure is not less than 30 mmHg.
The delta pressure is the difference between diastolic pressure and compartment pressure. A
delta pressure less than 30 mmHg is more indicative of compartment syndrome than an
absolute pressure greater than 30 mmHg.
The patient's exam is not consistent with compartment syndrome. However, it can evolve
over several hours so the patient should be admitted and monitored with serial clinical
exams. Ice and elevation of the forearm will provide symptomatic relief.
An invasive pressure needle to measure compartment pressures should only be undertaken
when clinical exams are not reliable.

Go to the next page if you knew the correct answer, or click the link image(s) below to further
research the concepts in this question (if desired).

Research Concepts:
Acute Compartment Syndrome

We update eBooks quarterly and Apps daily based on user feedback. Please tap flag to
report any questions that need improvement.
Question 175: A 65-year-old woman comes to the emergency department because of
abdominal pain and mild shortness of breath. Her past medical history is significant for chronic
liver disease and two episodes of variceal bleeding. On physical examination, her vital signs are
normal, and ascites is noticed. A paracentesis is performed, and a total of 4.5 liters is removed.
The ascitic fluid shows 299 polymorphonuclear neutrophils (PMN) cells/ml. Which of the
following would be the most appropriate option in her management?

Choices:
1. No indication of intravenous albumin
2. Albumin 5% infusion
3. Albumin 25% infusion
4. Crystalloids
Answer: 3 - Albumin 25% infusion
Explanations:
Administration of albumin 1.5 g/kg within 6 hours and 1 g/kg on day three along with
antibiotics have a better effect in preventing renal impairment and reducing mortality in
patients with spontaneous bacterial peritonitis (SBP).
The use of albumin 25% solution to treat spontaneous bacterial peritonitis is not an FDA
approved medication, but multiple clinical trials have shown benefits when used in
conjunction with antibiotics.
SBP is a common cause of mortality in patients with cirrhosis. The use of intravenous
albumin has reduced that mortality.
The diagnosis of SBP is established by a positive ascitic fluid bacterial culture and an
elevated ascitic fluid absolute PMN count of 250 cells/ml.

Go to the next page if you knew the correct answer, or click the link image(s) below to further
research the concepts in this question (if desired).

Research Concepts:
Albumin Colloid

We update eBooks quarterly and Apps daily based on user feedback. Please tap flag to
report any questions that need improvement.
Question 176: A 17-year-old female with diabetes mellitus presents with pain in her left
calf after a fall from her bicycle. She is febrile, tachycardic, looks ill, and her calf is tender to
touch. The skin overlying the left leg has many bullae with black necrosis. There is a
serosanguinous discharge. The distal pulses are decreased and there is crepitus. What is the next
step in management?

Choices:
1. Broad-spectrum antibiotics and emergent operative debridement
2. Get a stat angiogram
3. Obtain x-rays to look for subcutaneous gas
4. Send patient for duplex ultrasound to rule out deep venous thrombosis
Answer: 1 - Broad-spectrum antibiotics and emergent operative debridement
Explanations:
Gas gangrene is a disorder where there is necrotizing myositis. The hallmarks of the
disorder are rapid onset muscle swelling, severe pain, sepsis, and gas in tissues. This is a
life-threatening surgical emergency.
Clostridium species are gram-positive, spore-forming, and are a common cause of gas
gangrene. The findings will reveal bulla, crepitus, necrotic skin, and serosanguinous
discharge. Pulses may be present or absent. There may be no pain because the nerves are
destroyed.
The combination of aggressive surgical debridement and effective antibiotic therapy is the
determining factor for successful treatment.
Gas may appear on x-rays, but the patient must be resuscitated and taken to the operating
room. Fasciotomy plus radical debridement may be required.

Go to the next page if you knew the correct answer, or click the link image(s) below to further
research the concepts in this question (if desired).

Research Concepts:
Gas Gangrene

We update eBooks quarterly and Apps daily based on user feedback. Please tap flag to
report any questions that need improvement.
Question 177: A previously healthy 55-year-old male with non-insulin dependent diabetes
mellitus presents with neck swelling, 10 out of 10 pain, and fever. He states that he has a small
lesion on the side of his neck, which has progressively increased in size. On examination, he has
a temperature of 102.2F and a white blood cell count of 22,000 cells/mm3. The pain is not easily
controlled with multiple doses of pain medication. In addition to antibiotics, the management of
this patient includes which of the following?

Choices:
1. Admission to the intensive care unit
2. Surgical exploration and debridement
3. Moist compresses
4. Observation on antibiotics
Answer: 2 - Surgical exploration and debridement
Explanations:
In patients with diabetes mellitus, infections can progress rapidly and become fulminant.
Necrotizing fasciitis is more common in patients with diabetes mellitus, patients who are
obese, and patients who are immunocompromised.
Pain out of proportion to physical findings or pain that is difficult to control should warrant
further investigation and raise the possibility of a necrotizing infection.
The surgery may need to be done in the operating room under general anesthesia, because
the infection may be more extensive than is initially apparent.

Go to the next page if you knew the correct answer, or click the link image(s) below to further
research the concepts in this question (if desired).

Research Concepts:
Necrotizing Fasciitis

We update eBooks quarterly and Apps daily based on user feedback. Please tap flag to
report any questions that need improvement.
Question 178: A 65-year-old female with a past medical history of hypertension and a large
ischemic stroke 5 years ago with resultant left-sided hemiparesis presents to the emergency
department with acute onset of pleuritic chest pain with shortness of breath. She is tachypneic,
diaphoretic, and in visible distress. Her vital signs show a heart rate of 125 beats/min, respiratory
rate of 25/min, blood pressure of 100/60, and pulse oximetry of 85% on room air. CT
angiography of the chest shows a filling defect in the proximal pulmonary vasculature bilaterally.
What is the best treatment option for this patient?

Choices:
1. Argatroban infusion
2. Catheter-directed thrombolysis
3. Inferior vena cava filter placement
4. Heparin infusion
Answer: 4 - Heparin infusion
Explanations:
This patient presents with a massive to submassive pulmonary embolism (PE). Catheter-
directed thrombolysis (CDT) is indicated for massive PE or for submassive PE with
hemodynamic compromise. However, due to the risk of complications, contraindications for
the use of CDT must be reviewed prior to consideration of this therapy.
Though thrombolysis is administered locally during CDT, the thrombolytic agent can
disperse systemically. Contraindications to CDT include a prior ischemic stroke, cerebral
bleed, cerebral mass, vascular deformation, a recent bleeding ulcer in the gastrointestinal
tract, recent brain/spine surgery, major abdominal or pelvic surgery, or any source of active
hemorrhage.
The most common and most feared complication of CDT is a hemorrhagic stroke. This
patient's history of prior stroke puts her at a higher risk of intracranial bleeding following
thrombolytic agent use. This is a relative contraindication to CDT for her. She should be
treated with intravenous anticoagulation with close monitoring of her neurologic status.
Inferior vena cava placement is indicated in patients who have contraindications to
anticoagulation. Given her remote stroke and current hemodynamically devastating
pulmonary embolism, a trial of anticoagulation is warranted. Argatroban is indicated in
cases of heparin-induced thrombocytopenia.

Go to the next page if you knew the correct answer, or click the link image(s) below to further
research the concepts in this question (if desired).

Research Concepts:
Catheter Directed Thrombolysis Of Pulmonary Embolism

We update eBooks quarterly and Apps daily based on user feedback. Please tap flag to
report any questions that need improvement.
Question 179: A 58-year-old man is seen in the emergency department after suffering a
smoke inhalation injury following a house fire. He was intubated for dyspnea with stridor and
currently is on a ventilator with 100% FiO2. He has copious secretions that are being treated
with aggressive pulmonary hygiene. His carboxyhemoglobin level is 10% and arterial blood gas
shows pH 7.3. What is the best next step in management?

Choices:
1. Nebulized albuterol
2. Intravenous methylprednisolone
3. Transfer to a facility with hyperbaric oxygen therapy
4. Empiric broad-spectrum antibiotics
Answer: 1 - Nebulized albuterol
Explanations:
Nebulized beta-2-adrenergic agonists and muscarinic receptor antagonists have been proven
in animal models to improve respiratory function following inhalation injury.
Corticosteroids have not been proven to be beneficial in inhalation injury patients.
Hyperbaric oxygen therapy (HBOT) is indicated for carboxyhemoglobin levels greater than
25%, unconscious patients, those with neurologic findings, or in cases of severe metabolic
acidosis, pH less than 7.1. Treatment often requires transfer to a facility with HBOT
capabilities, but treatment should not be delayed.
Antibiotics should be started in patients with inhalation injury who have complications of
pneumonia. Pneumonia is a common short-term complication of severe inhalation injury
and usually presents 4 to 5 days after the initial injury.

Go to the next page if you knew the correct answer, or click the link image(s) below to further
research the concepts in this question (if desired).

Research Concepts:
Inhalation Injury

We update eBooks quarterly and Apps daily based on user feedback. Please tap flag to
report any questions that need improvement.
Question 180: A 50-year-old male with no past medical history presents to the emergency
department with signs and symptoms of ischemic stroke. Symptoms have been present for the
past 3 hours, and he is visibly upset because he states that he works out almost daily and eats a
healthy diet. He also states that he goes to his primary care physician every year, and has always
had a "normal" physical. His family history is non-contributory. He takes no medication except
for a daily multi-vitamin and turmeric powder. Vital signs and physical exam are within normal
limits. CBC, comprehensive metabolic profile, urine toxicology screen, and blood alcohol
content all came back within normal limits. EKG showed normal sinus rhythm. CT scan of the
brain did not show any evidence of bleeding. What is the next best step in evaluating for
cryptogenic stroke?

Choices:
1. Transthoracic echocardiogram (TTE)
2. TTE followed by transesophageal echocardiogram with agitated saline contrast
3. Transesophageal echocardiogram
4. Pelvic magnetic resonance venography
Answer: 2 - TTE followed by transesophageal echocardiogram with agitated saline contrast
Explanations:
Patients less than 55 years old without risk factors for cardiac embolism or
thromboembolism and who have had a negative workup for the standard evaluation for
stroke should undergo a more advanced workup.
When a cryptogenic stroke is suspected, the workup should include a transesophageal
echocardiogram with injected agitated saline, prolonged cardiac monitoring, coagulopathy,
and vasculitis workup.
Transesophageal echocardiogram with agitated saline contrast should be injected at rest,
with Valsalva maneuver, and with cough. This is because the Valsalva maneuver can
transiently increase the intracardiac pressure turning a left to right shunt into a right to left
shunt.
Up to 45% of strokes in young patients are said to be cryptogenic. Patent foramen ovale
(PFO) can be found in 30% of the population. Although PFO is a diagnosis of exclusion, it
should be strongly considered in the setting of a cryptogenic stroke.

Go to the next page if you knew the correct answer, or click the link image(s) below to further
research the concepts in this question (if desired).

Research Concepts:
Paradoxical Embolism

We update eBooks quarterly and Apps daily based on user feedback. Please tap flag to
report any questions that need improvement.
Question 181: A 33-year-old farmer was referred to the tertiary hospital due to fever,
shortness of breath, and chest pain for two weeks. He had no history of chronic disease, and
before the present illness, he had been perfectly healthy. He also did not have any history of
medication or travel for the past 3 months. During the first hospitalization, he was diagnosed
having pneumonia of the left upper lobe and treated with antibiotics. Ten days after the
treatment, the pneumonia progressed. The differential white blood count revealed 46% of
eosinophils, and the serum IgE was elevated. The sputum culture was negative for any bacteria,
the direct smear of the sputum was negative for fungi or acid bacilli, and blood culture was
negative for any culturable bacteria. Stool examination was negative for ova or any parasites.
Surprisingly, during hospitalization, the chest X-ray revealed migratory infiltrates of the lungs.
What could be the possible condition in this patient?

Choices:
1. Interstitial lung disease due to bacterial infection
2. Acute respiratory distress syndrome due to viral infection
3. Flu-like symptoms due to viral infection
4. Loeffler syndrome due to helminths infection
Answer: 4 - Loeffler syndrome due to helminths infection
Explanations:
The patient is a farmer who has a higher risk to contract with soil-transmitted helminths.
Treatment with antibiotics does not improve the patient condition, thus organism other than
bacteria should be suspected.
Eosinophilia and elevated serum IgE along with pulmonary symptoms define the Loeffler
syndrome.
The patient had no history of consuming drugs prior to hospitalization. Based on history
taking and laboratory examination, Loeffler syndrome due to helminths infection is the
possible cause.

Go to the next page if you knew the correct answer, or click the link image(s) below to further
research the concepts in this question (if desired).

Research Concepts:
Ancylostoma

We update eBooks quarterly and Apps daily based on user feedback. Please tap flag to
report any questions that need improvement.
Question 182: A 17-year-old female presents confused and uncooperative. Her vital signs
are normal except for a respiratory rate of 32/minute. Blood work reveals a sodium of 137
mEq/L, potassium 2.5 mEq/L, bicarbonate 13 mEq/L, chloride 119 mEq/L, BUN 16 mg/dL,
creatinine 0.6 mg/dL, glucose 80 mg/dL, calcium 9.2 mg/dL, urine pH 7.7, urine sodium 38
mEq/L, urine potassium 45 mEq/L, and urine chloride 20 mEq/L. Her arterial blood gas shows a
pH of 7.27, PCO2 28 mmHg, and PO2 94 mmHg on room air. What is the most likely diagnosis?

Choices:
1. Proximal renal tubular acidosis
2. Distal renal tubular acidosis
3. Lactic acidosis
4. Malabsorption with diarrhea
Answer: 2 - Distal renal tubular acidosis
Explanations:
The patient has hyperchloremic, non-anion gap metabolic acidosis with appropriate
respiratory compensation. This can be due to gastrointestinal bicarbonate losses such as
from diarrhea or renal causes, including proximal renal tubular acidosis (RTA) type II,
distal RTA (type I), or hypoaldosteronism RTA (type IV). Urine anion gap in diarrhea may
be positive.
A positive urine anion gap (urine sodium plus urine potassium minus urine chloride) points
towards a renal etiology. The fact that the patient has an elevated urine pH and hypokalemia
in the setting of acidosis points to distal RTA because the normal response would be a
lowering of urine pH in the setting of metabolic acidosis.
Distal RTA presents with rickets, growth failure, and osteomalacia due to metabolic
acidosis. Hypercalciuria, hypocitraturia, and alkaline urine lead to nephrocalcinosis,
specifically calcium phosphate stones and recurrent urinary tract infections. End-stage renal
failure due to nephrocalcinosis may result.
Incomplete distal RTA should be considered in any patient with calcium stones and a urine
pH persistently 5.5 or higher in the absence of infection.

Go to the next page if you knew the correct answer, or click the link image(s) below to further
research the concepts in this question (if desired).

Research Concepts:
Renal Tubular Acidosis

We update eBooks quarterly and Apps daily based on user feedback. Please tap flag to
report any questions that need improvement.
Question 183: A 24-year-old primigravida presents to the emergency department with
fatigue, nausea, vomiting, and abdominal pain. She is ten weeks pregnant, and so far the
pregnancy has been uneventful. The physical exam is significant for pallor, tachycardia, and mild
tenderness in the epigastrium. Other vital parameters show a blood pressure of 98/56 mmHg and
a temperature of 97.5 F. Labs show hemoglobin of 11.1 g/dL, leukocyte count of 12100/microL,
platelet count of 74000/microL, creatinine of 3.2 mg/dL, and blood urea nitrogen (BUN) of 45
mg/dL. Review of the peripheral smear shows schistocytes, giant platelets, thrombocytopenia,
Burr cells, and anisocytosis. No atypical white cells are seen on peripheral smear. Following this,
more tests are ordered, and results are significant for lactate dehydrogenase of 1563 U/L,
haptoglobin is undetectable, and absolute reticulocyte index is 3.2. What is the next best step in
the management of this patient?

Choices:
1. Send direct Coombs test and start high dose steroids
2. Send ADAMTS-13 and wait for results
3. Send ADAMTS-13 and start plasma exchange without waiting for results
4. Send testing for genetic mutation for atypical HUS and start eculizumab without waiting for
results
Answer: 3 - Send ADAMTS-13 and start plasma exchange without waiting for results
Explanations:
The patient presented with microangiopathic hemolytic anemia (hemolysis with
schistocytes on peripheral smear along with thrombocytopenia and renal failure).
Thrombotic microangiopathy is rare in pregnancy, but it is associated with high maternal
and neonatal morbidity.
Once the diagnosis of microangiopathic hemolytic anemia is established, plasma exchange
should be initiated without any further delay. Confirmatory tests, especially ADAMTS-13,
should be sent before start plasma exchange, but there is no need to wait for results.
ADAMTS-13, mutations for atypical HUS, Shiga toxin should be sent in a patient with
microangiopathic hemolytic anemia for evaluation of thrombotic thrombocytopenic
purpura, atypical HUS, and typical HUS respectively.
Eculizumab is started only if a patient with microangiopathic hemolytic anemia does not
respond to plasma exchange. Direct Coombs test is used to determine the cause of antibody-
mediated hemolysis. This patient presentation is quite characteristic for microangiopathic
hemolytic anemia.

Go to the next page if you knew the correct answer, or click the link image(s) below to further
research the concepts in this question (if desired).

Research Concepts:
Thrombocytopenia in Pregnancy

We update eBooks quarterly and Apps daily based on user feedback. Please tap flag to
report any questions that need improvement.
Question 184: A 75-year-old patient with a history of atrial fibrillation, on apixaban for
anticoagulation, is brought to the ER following a syncopal fall. She has been compliant with
apixaban, and her last dose was this morning. She reports bright red blood per rectum since
yesterday. Heart rate is 114 bpm, and her blood pressure is 86/54 mmHg. Lab work shows a
hemoglobin of 5.2 g/dL. Which of the following is the best next step in managing this patient?

Choices:
1. Administer normal saline
2. Administer fresh frozen plasma (FFP)
3. Transfuse packed red blood cells (PRBCs) and vitamin K
4. Transfuse PRBC and prothrombin complex concentrate (PCC)
Answer: 4 - Transfuse PRBC and prothrombin complex concentrate (PCC)
Explanations:
Patients taking newer anticoagulants and presenting with a life-threatening bleeding
emergency can be treated with 4 factor PCC.
Vitamin K is not indicated in this patient taking apixaban. It is indicated for the reversal of
warfarin-induced anticoagulation.
Normal saline alone will be inadequate in this patient with life-threatening gastrointestinal
bleed with a hemoglobin of 5.2 g/dL.
FFP can be used to reverse anticoagulation and also has the benefit of providing volume
resuscitation. But FFP needs to be procured from the blood bank, thawed, and takes several
hours to administer. PCC can be administered in minutes with no need to thaw, making it a
better agent in this scenario.

Go to the next page if you knew the correct answer, or click the link image(s) below to further
research the concepts in this question (if desired).

Research Concepts:
Prothrombin Complex Concentrate

We update eBooks quarterly and Apps daily based on user feedback. Please tap flag to
report any questions that need improvement.
Question 185: A 73-year-old white man undergoes emergency surgery for a perforated
duodenal ulcer. The surgery went smoothly without any event. The patient comes out of
anesthesia and is awake and alert. He receives post-operative pain management. Approximately
8 hours later, the patient develops a complication and requires immediate intubation with
concurrent administration of naloxone. Which of the following presentation is the most specific
for the complication?

Choices:
1. Pupillary miosis
2. Hypotension
3. Conjunctival injection
4. Respiratory depression
Answer: 4 - Respiratory depression
Explanations:
Opioid toxicity typically presents with a depressed level of consciousness, respiratory
depression, and pupillary miosis. It is important to be aware that opioid exposure does not
always result in miosis and that respiratory depression is the most specific sign.
Drowsiness, euphoria, and conjunctival injection are seen frequently. Other vital presenting
signs are ventricular arrhythmias, acute mental status changes, and seizures.
Naloxone is indicated for the treatment of opioid toxicity, specifically to reverse respiratory
depression from opioid use. Naloxone is a competitive opioid antagonist with a high affinity
for the mu-opioid receptor, allowing for the reversal of the effects of opioids.
Reliance on pupillary miosis to diagnose opiate overdose can be misleading. If sufficiently
severe, the central hypoxia may present with hypertension and pupillary dilation.

Go to the next page if you knew the correct answer, or click the link image(s) below to further
research the concepts in this question (if desired).

Research Concepts:
Opioid Overdose

We update eBooks quarterly and Apps daily based on user feedback. Please tap flag to
report any questions that need improvement.
Question 186: A 25-year-old schizophrenic and asthmatic patient is brought to the
emergency room with severe dyspnea. On physical examination, he was found to be wheezing.
He was nebulized with IV magnesium and IV steroids and was put on bi-level positive pressure
ventilation. He did not improve after 1 hour on BiPAP and was intubated. His psychiatric
medications were held, and the Propofol infusion is initiated. After 3 days, his wheezing is
better, and sedation cessation was attempted. After 30 minutes he became extremely agitated and
was sedated again. Which of the following is the best plan for the next attempt at extubation?

Choices:
1. Extubate the patient on propofol
2. Extubate the patient on fentanyl
3. Restart the psychiatric medications and add dexmedetomidine for sedation
4. Extubate the patient on ketamine
Answer: 3 - Restart the psychiatric medications and add dexmedetomidine for sedation
Explanations:
The reason for sedation vacation or breathing trial failing should be identified and addressed
accordingly.
If the patient is getting agitated and is delirious, antipsychotics can be tried; if anxious,
anxiolytics can be tried; or if in pain, low dose fentanyl drip/morphine drip or patch can be
tried.
Patient delirium and agitation can be very well controlled on dexmedetomidine, and as the
medication does not suppress the respiratory center, the patient can be extubated while on
continuous infusion of dexmedetomidine.
Attention should be paid to home medications, and if clinically permitted, a patient who is
on any anxiolytics, antipsychotics, or any chronic pain medications should continue them
while sedated or restart them in a timely fashion to prevent withdrawal.

Go to the next page if you knew the correct answer, or click the link image(s) below to further
research the concepts in this question (if desired).

Research Concepts:
Sedation Vacation in the ICU

We update eBooks quarterly and Apps daily based on user feedback. Please tap flag to
report any questions that need improvement.
Question 187: A 68-year-old female with a history of decompensated cirrhosis and ascites
presents to the hospital with a decreased urine output for the past three days. She currently takes
furosemide and spironolactone. Labs are significant for serum creatinine of 3.5 mg/dL and BUN
50 mg/dL. Her blood pressure was noted to be 96/65 mmHg. Her diuretics were held, and she
was started on IV fluids. 2 days later, her serum creatinine did not improve, and serum potassium
was noted to be 4.2 mEq/L. A renal ultrasound did not show any evidence of hydronephrosis.
Which of the following is the next best step in the management of this patient?

Choices:
1. Furosemide
2. Hemodialysis
3. Observation
4. Midodrine
Answer: 4 - Midodrine
Explanations:
Patients with cirrhosis and ascites are prone to develop hepatorenal syndrome secondary to
systemic hypotension and renal vasoconstriction, causing the underfilling phenomenon.
Splanchnic vasodilation in cirrhosis leads to decreased effective blood flow to the kidneys,
which activates the RAAS system, leading to retention of sodium and water and renal
vascular constriction. However, this effect is not enough to overcome the systemic
vasodilation caused by cirrhosis, leading to renal hypoperfusion and worsened by renal
vasoconstriction with the end point of renal failure.
Acute renal injury in patients with cirrhosis is usually secondary to dehydration and it is
managed by stopping the diuretics and starting the patients on IV fluids. If the kidney
function does not improve after adequate hydration then the diagnosis of hepatorenal
syndrome is made.
Patients with hepatorenal syndrome are usually managed with a systemic vasoconstrictor
given with albumin. Midodrine has shown a good response and is first-line for the treatment
of hepatorenal syndrome.
Diuretics should be held in cirrhotic patients who develop acute kidney injury.
Hemodialysis is indicated in patients who develop refractory hypervolemia or
hyperkalemia. Observation is inappropriate as the patient’s kidney function did not improve
with IV fluids.

Go to the next page if you knew the correct answer, or click the link image(s) below to further
research the concepts in this question (if desired).

Research Concepts:
Hepatic Cirrhosis

We update eBooks quarterly and Apps daily based on user feedback. Please tap flag to
report any questions that need improvement.
Question 188: A 65-year-old female presents to the emergency department with a chief
complaint of abdominal pain. The patient appears uncomfortable in the room and complains of
severe pain that started suddenly less than an hour before arrival. On physical exam, her
abdomen is noted to be soft and non-tender. Her pulse is 150 bpm and irregular, with a blood
pressure of 140/95 mmHg, and an increased respiratory rate. The patient has no prior history of
atrial fibrillation and is not currently anticoagulated. What is the best next test to confirm the
suspected diagnosis?

Choices:
1. Abdominal ultrasound
2. KUB plain film x-ray
3. CTA of the abdomen and pelvis
4. Angiography
Answer: 3 - CTA of the abdomen and pelvis
Explanations:
Bowel necrosis is a life-threatening emergency that has many presentations and equally
many underlying etiologies. One such etiology of bowel necrosis, acute mesenteric
ischemia, frequently presents with abdominal pain out of proportion to physical
examination findings. Rapid diagnosis of this disease process is vital to both mortality and
disease course.
Patients presenting with severe abdominal pain and a new diagnosis of atrial fibrillation are
a high-risk group for acute mesenteric ischemia and bowel necrosis secondary to
thromboembolism. Approximately half of all cases of acute mesenteric ischemia are thought
to be secondary to an underlying embolic disease process.
There are multiple imaging modalities available to test for acute bowel necrosis and
mesenteric ischemia. In this clinical scenario, the next best imaging test would be a CTA of
the abdomen and pelvis. CTA has a reported sensitivity between 94% to 96% and
specificity between 96% to 98%. Although angiography is still considered the gold standard
for diagnosis, this procedure is invasive and time intensive in a disease that is time
sensitive.
Patients with a diagnosis of bowel necrosis are managed according to a variety of factors
including the underlying etiology of the disease process and comorbid conditions. The final
pathway for management for many cases involves surgical consultation and intervention. In
a high probability of patient presentation, emergent consultation on an unstable patient with
a suspected diagnosis of acute mesenteric ischemia and bowel necrosis should not be
delayed by imaging or laboratory results.

Go to the next page if you knew the correct answer, or click the link image(s) below to further
research the concepts in this question (if desired).

Research Concepts:
Bowel Necrosis

We update eBooks quarterly and Apps daily based on user feedback. Please tap flag to
report any questions that need improvement.
Question 189: A 55-year-old male who recently flew from India is brought to the
emergency department for hemoptysis. The patient was first seen at the primary care clinic where
he complained by fevers and chills as well as profuse sweating at night. He then proceeded to
cough up a large volume of blood and was sent to the emergency department. The patient was
tried on a course of inhaled tranexamic acid. This did not control the bleeding. Finally, the
decision was made to intubate the patient. After, the patient was sent to the CT which showed a
left upper lobe cavitary lesion and also a bronchial artery source of the bleeding. The patient has
a blood pressure of 101/70 mmHg, a heart rate of 98 bpm, oxygen saturation of 100%, and
currently being ventilated at a respiratory rate of 15 breaths per minute. What is the next step in
management?

Choices:
1. Perform bronchoscopy and use cold saline lavage in order to control the source of bleeding
and isolate the etiology of the hemoptysis
2. Take the patient to the operating room for surgical lobectomy to bleeding control
3. Replace the single-lumen endotracheal tube with a double lumen endotracheal tube
4. Perform bronchial artery embolization to stop bleeding
Answer: 4 - Perform bronchial artery embolization to stop bleeding
Explanations:
CT scan is as accurate as bronchoscopy in localizing source of bleeding, and CT is superior
for finding the cause of the bleeding. The patient has risk factors that point to tuberculosis
as the source of bleeding, and bronchoscopy can further exacerbate hemoptysis by
stimulating coughing.
Surgical lobectomy has high mortality rates in emergent cases. Although the patient may
need a lobectomy in the future, a stable patient would benefit from embolization before
going through scheduled lobectomy.
Double-lumen endotracheal tubes are technically difficult to place and do not allow optimal
source control due to the smaller diameter of the lumens.
Immediate success rates of bronchial artery embolization are reported to be 82% to 98%,
and would most benefit a stable patient in the immediate term. While surgical lobectomy
may be required down the line due to recurrence, the patient can be further worked up and
medically optimized during the interim.

Go to the next page if you knew the correct answer, or click the link image(s) below to further
research the concepts in this question (if desired).

Research Concepts:
Pulmonary Hemorrhage

We update eBooks quarterly and Apps daily based on user feedback. Please tap flag to
report any questions that need improvement.
Question 190: A 65-year-old female with a history of hypertension on hydrochlorothiazide
presents with severe chest pain that radiates to her back and dyspnea. She is found to have a
blood pressure of 205/98 mmHg, pulmonary rales, an ECG with ST changes, and a chest x-ray
with a widened mediastinum. What the best initial management?

Choices:
1. Immediate cardiac catheterization with likely stent placement
2. Intravenous nitroprusside and urgent CT angiography with three-dimensional reconstruction
3. Intravenous morphine and nitrates, oxygen, and aspirin
4. Serial cardiac enzymes, intravenous beta-blockers, and heparin
Answer: 2 - Intravenous nitroprusside and urgent CT angiography with three-dimensional
reconstruction

Explanations:
This scenario is suggestive of an aortic dissection that can be diagnosed with urgent CT
angiography with three-dimensional reconstruction. CT is useful in patients who are
hemodynamically stable.
Beta-blockers are used to decrease cardiac contractility but will aid in bringing the blood
pressure down as well. Calcium-channel blockers can be used if beta-blockers are
contraindicated.
Nitroprusside is used to reduce blood pressure.
A proximal, type A, dissection is associated with aortic insufficiency and hypertension and
requires urgent repair, while a distal dissection is associated with arterial obstruction leading
to renal failure and bowel ischemia.

Go to the next page if you knew the correct answer, or click the link image(s) below to further
research the concepts in this question (if desired).

Research Concepts:
Aortic Dissection

We update eBooks quarterly and Apps daily based on user feedback. Please tap flag to
report any questions that need improvement.
Question 191: A 91-year-old female becomes acutely unresponsive in her assisted living
care facility. She is taken to the hospital for a stroke workup and found to have a right frontal
intraparenchymal hemorrhage with intraventricular extension. The estimated blood volume of the
hemorrhage is 65 mL. She takes warfarin for non-valvular atrial fibrillation and has an INR of
2.1. She has hypertension, diabetes mellitus, asthma, gastroesophageal reflux disease, and stress
incontinence. On initial exam, her eyes do not open to painful stimuli, she moans and
withdrawals to noxious stimuli in the right arm and has extensor posturing in the left arm with an
extension of bilateral legs. What is her 30-day mortality based on the intracerebral hemorrhage
score?

Choices:
1. 26%
2. 72%
3. 97%
4. 100%
Answer: 3 - 97%
Explanations:
The intracerebral hemorrhage (ICH) score predicts 30-day mortality based on patients with
non-traumatic intracerebral hemorrhage.
The score is based on Glasgow coma score (GCS), age, the volume of hemorrhage,
intraventricular hemorrhage, and infratentorial origin of hemorrhage.
The point distribution is GCS 13 to 15 is 0 points, GCS 5 to 12 is 1 point, GCS 3 to 4 is 2
points; age 80 or greater is 1 point, ICH volume 30 mL or more is 1 point, intraventricular
hemorrhage is 1 point, and infratentorial origin of hemorrhage is 1 point.
30-day mortality is 0% for 0 points, 13% for 1 point, 26% for 2 points, 72% for 3 points,
97% for 4 points, 100% for 5 points, and assumed to be 100% for 6 points.

Go to the next page if you knew the correct answer, or click the link image(s) below to further
research the concepts in this question (if desired).

Research Concepts:
Intracranial Hemorrhage

We update eBooks quarterly and Apps daily based on user feedback. Please tap flag to
report any questions that need improvement.
Question 192: A 50-year-old male with alcohol use disorder presents to the emergency
department. His medical history includes only alcohol use disorder, and he is not currently taking
any medication. On presentation, he complains of nausea, vomiting, and abdominal pain. He says
that he has been drinking heavily for several days but over the last day he has been unable to
“keep anything down.” His vital signs show respirations 28, heart rate 126 beats/min, blood
pressure 106/62 mmHg, and temperature of 98.5 F. On physical exam, he has dry mucous
membranes, mild diffuse abdominal tenderness, no peritoneal signs, and clear lungs. His
extremities are without edema. He has a visible resting tremor which is worsened with finger-to-
nose maneuvers. His bedside blood glucose is 70 mg/dL. He has the following arterial blood gas
results: pH 7.2, pCO2 25, pO2 98, and bicarbonate 15 mEq/L. His basic metabolic panel shows
sodium 138 mEq/L, potassium 4 mEq/L, chloride 100 mEq/L, bicarbonate 15 mEq/L, BUN
29mg /dL, and creatinine 1.6 mg/dL. Anion gap is 23. What is the patient’s acid-base status?

Choices:
1. Metabolic acidosis with adequate respiratory compensation
2. Metabolic alkalosis without respiratory compensation
3. Metabolic acidosis without adequate respiratory compensation
4. Metabolic acidosis without respiratory compensation
Answer: 3 - Metabolic acidosis without adequate respiratory compensation
Explanations:
The patient has metabolic acidosis without adequate respiratory compensation. The
presence of large amounts of ketoacids (beta-hydroxybutyrate) lowers the pH and causes an
increased respiratory rate to decrease the pCO2.
By using Winter's formula one can tell if the patient is adequately compensating for the
metabolic acidosis.
In this case, the lungs are not adequately compensating for the metabolic acidosis.
The patient uses up existing bicarbonate ions by combining them with the hydrogen ions to
make carbonic acid (H2CO3). This is an example of buffering an acid. At the level of the
lungs, the respiratory rate increases to release carbon dioxide in an attempt to decrease the
amount of acid in the system and bring the pH back up.

Go to the next page if you knew the correct answer, or click the link image(s) below to further
research the concepts in this question (if desired).

Research Concepts:
Alcoholic Ketoacidosis

We update eBooks quarterly and Apps daily based on user feedback. Please tap flag to
report any questions that need improvement.
Question 193: Several workers from a local government office present to the emergency
department shortly after being exposed to a white powder from an envelope that was mailed to
their address. After undergoing thorough decontamination, they are brought into the department,
and they are noted to be coughing, wheezing, and complaining of feeling short of breath.
Responders on the scene giving a preliminary report that the white powder has been identified as
ricin. Which of the following best describes an appropriate plan of care and likely clinical course
for these patients?

Choices:
1. Their wheezing and shortness of breath will probably resolve after treatment with nebulized
albuterol and oral corticosteroids, and they can probably be discharged after a few hours of
observation if their symptoms are improving
2. They will probably progress to angioedema and anaphylaxis unless treated with intramuscular
epinephrine quickly, but if treatment is initiated promptly and there are no signs of airway
compromise, they could probably be admitted to a floor bed for observation
3. They need to be given prophylactic antibiotics and admitted to an isolation room
4. Their symptoms may quickly progress to pulmonary edema, pneumonia, respiratory failure,
seizures, and organ failure. They should be admitted to the intensive care unit
Answer: 4 - Their symptoms may quickly progress to pulmonary edema, pneumonia,
respiratory failure, seizures, and organ failure. They should be admitted to the intensive care unit

Explanations:
While it is not incorrect to treat their symptoms, under no circumstances should these
patients be discharged home because they could quickly progress to pulmonary edema and
respiratory failure regardless of their initial response to treatment.
Inhalational exposure to ricin does cause wheezing and difficulty breathing, but these
symptoms are not because of an anaphylaxis reaction, and because these patients may
rapidly deteriorate into respiratory failure, admitting them to a floor bed when they are
already symptomatic is unwise.
Prophylactic antibiotics are not indicated in ricin exposure, and once the patients have been
thoroughly decontaminated there is no risk that healthcare workers will experience
symptoms by coming into contact with these patients. Thus an isolation room is
unnecessary.
Patients exposed to ricin by inhalation can rapidly deteriorate and may develop all of these
symptoms; because they are already symptomatic and may quickly progress to being
critically ill, admitting them to the intensive care unit is the best choice.

Go to the next page if you knew the correct answer, or click the link image(s) below to further
research the concepts in this question (if desired).

Research Concepts:
Ricin Toxicity

We update eBooks quarterly and Apps daily based on user feedback. Please tap flag to
report any questions that need improvement.
Question 194: A 48-year-old male presents to the emergency department with self-report of
suicidal ideations. He states that he drank a bottle of windshield washer fluid. He is
asymptomatic with normal vital signs. His basic metabolic panel is unremarkable, and his
ethanol concentration is negative. Methanol concentration is drawn and your hospital's laboratory
has to send it to another laboratory for gas chromatography. The turn around time is 24 hours.
What is the most appropriate next step?

Choices:
1. Check a serum osmolality
2. Calculate the osmolality gap
3. Check serial basic metabolic panels every 3 hours for 12 hours monitoring for acidosis while
the patient remains hydrated and nourished
4. Give fomepizole
Answer: 3 - Check serial basic metabolic panels every 3 hours for 12 hours monitoring for
acidosis while the patient remains hydrated and nourished

Explanations:
The patient is asymptomatic with normal acid-base status. Fomepizole should be given
when there is evidence of a toxic concentration of a toxic alcohol.
Methanol is a toxic alcohol that is metabolized by alcohol dehydrogenase to formaldehyde,
and then by aldehyde dehydrogenase to formic acid. The parent compound, methanol causes
the serum osmol gap, while formic acid causes the anion gap acidosis and retinal toxicity.
As the methanol is metabolized, the osmolal gap decreases while the anion gap increases.
A serum osmolality gap can be helpful in this situation. If it is elevated, it may support the
decision to give fomepizole. However, it cannot be used to rule out the ingestion of a toxic
alcohol.
This patient will require fomepizole to cease the progression of acidosis if it were to
develop. Fomepizole should be given if bicarbonate falls below 15 and then resuscitation
should ensue. To help avoid any form of iatrogenic acidosis, the patient should remain
hydrated and nourished during the observation period.

Go to the next page if you knew the correct answer, or click the link image(s) below to further
research the concepts in this question (if desired).

Research Concepts:
Methanol Toxicity

We update eBooks quarterly and Apps daily based on user feedback. Please tap flag to
report any questions that need improvement.
Question 195: A 17-year-old male presents with hypothermia, malaise, anorexia, a
semicomatose state, and hypotension. A chest x-ray shows findings consistent with pneumonia.
The blood culture shows gram-negative rods. Which is the presumptive diagnosis and which
cytokines may be involved?

Choices:
1. B-lymphocyte cell growth: Interleukin (IL)-2 and interferon gamma are involved
2. Septic shock: IL-1 and tumor necrosis factor are involved
3. Systemic autoimmune disease: IL-4 and IL-5 are involved
4. AIDS: IL-3 and IL-7 are involved
Answer: 2 - Septic shock: IL-1 and tumor necrosis factor are involved
Explanations:
Septic shock generated by gram-negative bacteria is mediated by cytokines, mainly tumor
necrosis factor (TNF) along with other cytokines including interleukin (IL)-1 and IL-6.
The similarities between IL-1 actions and those of TNF appear surprising at face value
because the cytokines and their receptors are structurally different.
Toxins, including lipopolysaccharide that is present in the outer membrane of the gram-
negative bacterium, are involved in the pathogenesis of the TNF-mediated septic shock.
IL-1 does not induce the apoptosis. Even at high systemic concentrations, IL-1 by itself it
does not cause the pathophysiologic changes of septic shock.

Go to the next page if you knew the correct answer, or click the link image(s) below to further
research the concepts in this question (if desired).

Research Concepts:
Interleukin

We update eBooks quarterly and Apps daily based on user feedback. Please tap flag to
report any questions that need improvement.
Question 196: A 32-year-old male with a history of acute myeloid leukemia (AML) is
admitted after a recurrence of AML following initial treatment with chemotherapy. During his
admission, he develops perianal pain and has a complaint of severe pain with bowel movements.
Digital rectal examination is performed and yields only tenderness to palpation at the anal verge
without identifying fullness or erythema. What is the best diagnostic test to further assess his
pain?

Choices:
1. Computed tomography (CT) of the pelvis
2. Endorectal ultrasound
3. No further imaging is needed, continue treatment for AML
4. Magnetic resonance imaging (MRI) of the pelvis
Answer: 4 - Magnetic resonance imaging (MRI) of the pelvis
Explanations:
CT is not the best imaging technique available in an immunocompromised patient with the
suspicion of perianal abscess. CT may miss small abscesses.
Endorectal ultrasound is extremely painful and often unnecessary in patients with perianal
abscesses.
Immunocompromised patients may present only with anal pain. It should not be dismissed
when there are no outward signs of inflammation. These patients will need evaluation and
treatment of any perianal abscesses identified on MRI before beginning their
immunosuppressive regimen. They may not be able to combat this infection in their
immunocompromised state.
MRI should be utilized as this imaging modality can detect very small amounts of fluid in
the perianal space, which may be causing discomfort in this patient.

Go to the next page if you knew the correct answer, or click the link image(s) below to further
research the concepts in this question (if desired).

Research Concepts:
Perianal Abscess

We update eBooks quarterly and Apps daily based on user feedback. Please tap flag to
report any questions that need improvement.
Question 197: A 17-year-old female with asthma presents to the emergency department
with right upper quadrant abdominal pain, nausea, and jaundice for two weeks. Laboratory
analysis shows alanine aminotransferase of 1048, aspartate aminotransferase of 1120, alkaline
phosphatase of 212, and bilirubin of 4.1. INR is 2.1. She appears slightly confused and lethargic.
Hepatitis C RNA by polymerase chain reaction (PCR) is not detected, HCV antibody is negative,
hepatitis A IgM is positive, hepatitis B core antibody IgM is negative, hepatitis B core antibody
IgG is positive, and hepatitis B surface antigen is negative. What is the next step in management?

Choices:
1. Admit to the hospital and initiate liver transplantation evaluation
2. Discharge the patient home as she has an acute hepatitis A infection which is a self-limited
illness
3. Treat the patient for chronic hepatitis B infection with oral lamivudine
4. Admit the patient to the hospital for intravenous fluids, IV N-acetylcysteine, and close
monitoring
Answer: 1 - Admit to the hospital and initiate liver transplantation evaluation
Explanations:
This patient has acute hepatitis A as indicated by the positive hepatitis A IgM.
Acute hepatitis A infection is typically a mild, uncomplicated and self-limited illness in this
young and healthy patient. However, this patient meets criteria for acute liver failure with
coagulopathy and encephalopathy without preexisting cirrhosis, and with an illness of, 2
weeks' duration. Therefore, she should be admitted, and evaluation for liver transplantation
should be initiated if she continues to clinically decompensate.
Acute hepatitis B is diagnosed when patients have positive hepatitis B core antibody IgM
plus or minus positive hepatitis B surface antigen.
Patients with chronic hepatitis B will have persistence of hepatitis B surface antigen
positivity beyond 6 months' duration.

Go to the next page if you knew the correct answer, or click the link image(s) below to further
research the concepts in this question (if desired).

Research Concepts:
Hepatitis A

We update eBooks quarterly and Apps daily based on user feedback. Please tap flag to
report any questions that need improvement.
Question 198: A 65-year-old man presents with complaints of painless visual blurring,
which has happened twice in the last day. He has a history of hypertension, hyperlipidemia, and
type 2 diabetes mellitus. He has a 30 pack-year smoking history, drinks alcohol occasionally, and
has never used illicit drugs. He mentions that one day earlier, he lost his speech for a few
minutes. On examination, it is determined that he has again lost vision in his left eye. What is the
most appropriate next step in the management of this patient?

Choices:
1. Heparin infusion
2. Fibrinolytic therapy
3. CT brain scan without contrast
4. Acetylsalicylic acid
Answer: 3 - CT brain scan without contrast
Explanations:
Before beginning lytic therapy for a developing stroke, it is most essential to obtain CT
brain imaging to ensure that there is no hemorrhagic component.
Only embolic and ischemic strokes can be treated with lytic therapy. It is very dangerous to
treat hemorrhagic strokes with lytic therapy.
In this clinical scenario, painless loss of vision may be amaurosis fugax and may be
associated with carotid artery stenosis.
Thus, another recommended study is a duplex ultrasound of the neck to look at the carotid
bifurcation. Alternatively, one could follow the unenhanced brain CT with a head and neck
CT angiogram.

Go to the next page if you knew the correct answer, or click the link image(s) below to further
research the concepts in this question (if desired).

Research Concepts:
Stroke Imaging

We update eBooks quarterly and Apps daily based on user feedback. Please tap flag to
report any questions that need improvement.
Question 199: A 72-year-old female with a history of diabetes mellitus, hypertension,
smoking, and previous myocardial infarction and multivessel disease presents with a history of
central chest pain for 1 hour, severe in intensity, associated with nausea, vomiting diaphoresis,
shortness of breath, and radiating to left jaw. Vital signs are BP 165/90 mmHg, pulse 127/min,
RR 26/min, and afebrile. The chest is clear, and no significant findings on the cardiovascular
exam are noted. ECG shows anterolateral myocardial infarction. Thrombolytics were given, and
the patient improved. One day later she develops shock with BP 80/60 mmHg, pulse 140/min,
RR=32/min, and SpO2=86% on room air. Auscultation of the chest reveals a blowing, high
pitched pan-systolic murmur best heard at the apex, radiating to left axilla and accentuating on
expiration with soft S1. There are fine inspiratory crackles to the mid-zone of the lungs. What is
the initial treatment in this case?

Choices:
1. Intra-aortic balloon pump
2. Emergency surgical intervention
3. Intravenous nitrates
4. Intravenous diuretics
Answer: 1 - Intra-aortic balloon pump
Explanations:
The patient is suffering from acute pulmonary edema after papillary muscle rupture and
acute mitral regurgitation. As the patient is in cardiogenic shock, the initial treatment of
choice is intra-aortic balloon pump which increases the cardiac output and decreases the
oxygen consumption/demand heart.
Emergency surgical intervention the treatment of choice if the patient is hemodynamically
stable. Mitral valve repair is done if the papillary muscles are not necrotic and mitral valve
replacement if the muscles become necrotic.
Intravenous nitrates are used to decrease the workload on the heart by decreasing both the
preload and afterload when the blood pressure is within normal range. They cannot be used
initially when the patient is hemodynamically stable.
Intravenous diuretics are used for acute pulmonary edema but should be used with caution
as they also tend to lower the blood pressure. As in this case, the patient is in cardiogenic
shock, the initial step should be to resuscitate the patient and make him hemodynamically
stable.

Go to the next page if you knew the correct answer, or click the link image(s) below to further
research the concepts in this question (if desired).

Research Concepts:
Cardiogenic Pulmonary Edema

We update eBooks quarterly and Apps daily based on user feedback. Please tap flag to
report any questions that need improvement.
Question 200: A 65-year-old male is found at home febrile and unresponsive. He is
brought to the emergency department. His blood pressure is 60/40 mmHg, pulse 45 beats/min,
temperature 38.4C, and respiratory rate 22/min. Upon administering fluids, a physical exam is
conducted. There is marked tenderness at the costovertebral angle. Urinalysis is positive for
leukocyte esterase and WBCs. Which of the following is the best medication to resuscitate him?

Choices:
1. Dobutamine
2. Epinephrine
3. Dopamine
4. Lidocaine
Answer: 2 - Epinephrine
Explanations:
Epinephrine is FDA-approved in adults with hypotension associated with septic shock.
Pyelonephritis leading to septic shock is a medical emergency and needs to be treated as
soon as possible.
Epinephrine stimulates the body's sympathetic nervous system and plays a role in acute
stress response.
Another treatment for septic shock is norepinephrine.

Go to the next page if you knew the correct answer, or click the link image(s) below to further
research the concepts in this question (if desired).

Research Concepts:
Epinephrine

We update eBooks quarterly and Apps daily based on user feedback. Please tap flag to
report any questions that need improvement.
Section 3
Question 201: A 28-year-old African American female is brought to the emergency
department with acute onset shortness of breath that has progressively worsened over the past 12
hours. According to her boyfriend at the bedside, she has not slept all night and has been using
her inhaler every hour without any benefit. Symptoms suddenly became much worse over the
past 3 hours. On arrival she appears anxious, in visible respiratory distress, and unable to speak
more than a couple of words at a time. She is using her accessory muscles to breathe. The
calculated difference between systolic blood pressure at end inspiration and expiration is 14
mmHg. She is promptly started on nebulized albuterol and ipratropium for continuous hour-long
treatment and administered a dose of 2 grams magnesium sulfate, 125 mg methylprednisolone,
and 2 liters oxygen supplementation. Two hours later, she is finally lying in bed and sleeping
without distress. Boyfriend feels reassured that she is finally able to catch some rest. Her wheeze
is less than before. Difference between systolic blood pressure at inspiration and expiration is
now 5 mmHg. Which of the following is the next best step in the management of this patient?

Choices:
1. Wake her up and check her peak expiratory flow rate and FEV1
2. With a reduced respiratory variation of blood pressure and lack of wheeze get a chest xray to
evaluate for pneumothorax
3. Get an ABG immediately and prepare for endotracheal intubation
4. Continue hourly nebulization treatment for 4 more hours and transition to 60 mg solumedrol
every 6 hourly.
Answer: 3 - Get an ABG immediately and prepare for endotracheal intubation
Explanations:
While the ability to lie supine on a status asthmaticus patient who was sitting upright may
reflect a treatment response it has to be assessed in relation to the entire clinical picture. She
appears sleepy but could be lethargic indicative of respiratory fatigue. Lack of wheezing
could be a sign of worsened air entry as well. Similarly, a sudden reduction in pulsus
paradoxus can happen in the progression of respiratory failure and hypoventilation from
respiratory muscle fatigue. Preparation for mechanical ventilatory support seems
appropriate in the circumstance.
Pulsus paradoxus is caused by a reduction in left ventricular outflow with air hunger and
deep inspiration causing an increase in LV and RV afterload and ventricular
interdependence. Greater than 12% is considered significant. However, in worsening status
asthmaticus patient may be too fatigued to take deep enough breaths to result in a large
variation of pressure.
An ABG performed in a clinical setting as above may show normal PCO2 which again
should be interpreted in light of prior carbon dioxide partial pressure. Patient in status
asthmaticus presents initially with respiratory alkalosis and low PCO2. Normalization may,
therefore, be an ominous sign indicating rising level and worsening fatigue of respiratory
apparatus.
If a patient does not excessive secretions, is somnolent but arousable, and has no ongoing
nausea, noninvasive ventilatory support with BIPAP can also be tried for a short duration to
be reassessed in 3-4 hours.

Go to the next page if you knew the correct answer, or click the link image(s) below to further
research the concepts in this question (if desired).

Research Concepts:
Status Asthmaticus

We update eBooks quarterly and Apps daily based on user feedback. Please tap flag to
report any questions that need improvement.
Question 202: A 25-year-old male, who has been HIV positive for about 8 years, is
admitted to the hospital with respiratory distress. The patient had focal neurological deficits
earlier and now is in a comatose state. The pulmonary and central nervous system biopsies show
dichotomously branching septate hyphae. What is the most likely underlying condition?

Choices:
1. Severe neutropenia
2. CD4+ cell count less than 600
3. Diabetic ketoacidosis
4. Multiple myeloma
Answer: 1 - Severe neutropenia
Explanations:
Invasive aspergillosis is found primarily in patients with neutrophil counts less than
500/mm3.
It also is seen with chronic granulomatous disease and cystic fibrosis.
The use of antifungals and antimicrobials is needed.
Immunological support includes the administration of IV purified immunoglobulins and
antiretroviral therapy. Frequent evaluation of CD4+ count should be done.

Go to the next page if you knew the correct answer, or click the link image(s) below to further
research the concepts in this question (if desired).

Research Concepts:
Neutropenia

We update eBooks quarterly and Apps daily based on user feedback. Please tap flag to
report any questions that need improvement.
Question 203: A 78-year-old lady presents to the emergency department with acute
symptoms of lightheadedness and dizziness for past 4 hours. She called her son home for not
feeling well. He found her blood pressure to be 80/40 mmHg and brought her to the hospital
promptly. She had only taken 6 of her prednisone pills as prescribed after dinner. However, he
found the pill bottle with the medication name as verapamil. Depending on the severity of
manifestations, which of the following is not a recommended therapeutic intervention?

Choices:
1. Urgent hemodialysis
2. Intravenous calcium chloride
3. High dose insulin with dextrose
4. Intravenous lipid emulsion
Answer: 1 - Urgent hemodialysis
Explanations:
Calcium channel blockers are highly protein bound and have a very large volume of
distribution. Hemodialysis or hemofiltration are therefore ineffective in their toxicities.
Hemodialysis may, however, be necessary if a patient develops acute renal failure due to
acute tubular necrosis from hypoperfusion.
Intravenous calcium chloride is the recommended first step in treatment with careful
monitoring of serum calcium level.
Severe toxicity is managed by incremental doses of intravenous insulin infusion.

Go to the next page if you knew the correct answer, or click the link image(s) below to further
research the concepts in this question (if desired).

Research Concepts:
Calcium Channel Blocker Toxicity

We update eBooks quarterly and Apps daily based on user feedback. Please tap flag to
report any questions that need improvement.
Question 204: An 18-year-old female patient is admitted to the hospital with refractory
nausea and vomiting. About three days ago she had an upper respiratory tract infection and has
been progressively worse since. History is significant for amenorrhea, repeated candida
infections of the oral cavity, and debilitating cramps of her hands. Heart rate is 110/min,
respiratory rate 18/min, temperature 99.6 F, and blood pressure 90/58 mmHg. On physical exam,
she is noted to have hyperpigmentation of the knuckles, the absence of axillary hair, and white
patches in the oral cavity. What is the next best step in the management of this patient?

Choices:
1. Obtain plasma ACTH and cortisol at 8 AM
2. Obtain MRI of the pituitary gland
3. Obtain CT scan of the adrenal glands
4. Start stress doses of hydrocortisone
Answer: 4 - Start stress doses of hydrocortisone
Explanations:
Plasma and cortisol can be drawn at the time patient is being evaluated in the emergency
department or the hospital prior given the first shot of high dose corticosteroids, but given
patient is acutely ill, waiting until 8 am to give steroids is not appropriate. Very high levels
of ACTH may be seen in patients.
Based on the clinical presentation the patient most likely has primary adrenal insufficiency,
there is no need to obtain an MRI of the pituitary at this time.
Imaging of the adrenals at this time is not needed given she can be diagnosed clinically and
biochemically first. Furthermore, this would delay treatment and will not likely establish the
diagnosis.
Adrenal insufficiency can present with fatigue, abdominal pain, nausea, vomiting, and
dizziness. Adrenal crisis is treated by starting stress dose hydrocortisone which results in
marked improvement of the patient’s status. A history of chronic candidiasis and
hypocalcemia as evidenced by cramps in her hands indicate a diagnosis of APS-1.

Go to the next page if you knew the correct answer, or click the link image(s) below to further
research the concepts in this question (if desired).

Research Concepts:
Polyglandular Autoimmune Syndrome Type I

We update eBooks quarterly and Apps daily based on user feedback. Please tap flag to
report any questions that need improvement.
Question 205: A 44-year-old female presents after a motor vehicle collision with an
isolated tibial fracture of her right leg. She has swelling and bruising at the site of trauma. She
has increasing pain medication requirements in the emergency department, and her pain is worse
when her leg is passively stretched. She now reports tingling and numbness. On re-examination,
the dorsalis pedis pulse is not palpable, and she cannot feel light touch to her foot. Which history
or exam finding in acute compartment syndrome is most likely to present first?

Choices:
1. Loss of sensation and motor function in her foot
2. Loss of pulses of the affected leg and foot
3. Inability to regulate temperature with abnormal sweating
4. Worsening pain of the leg made worse with passive stretching
Answer: 4 - Worsening pain of the leg made worse with passive stretching
Explanations:
Acute compartment syndrome classically presents with the 6 P’s: pain, paresthesia, paresis,
pallor, poikilothermia, and pulselessness. However, this is not the typical clinical picture.
Most patients are going to have worsening pain out of proportion to the exam which is also
made worse with passive stretching.
Loss of sensation and motor function to the foot can be seen, but typically will not present
first.
Pulselessness is typically a late finding.
Poikilothermia or inability to regulate temperature is a late finding and probably the least
likely to be obvious by history or exam.

Go to the next page if you knew the correct answer, or click the link image(s) below to further
research the concepts in this question (if desired).

Research Concepts:
Tibial Anterior Compartment Syndrome

We update eBooks quarterly and Apps daily based on user feedback. Please tap flag to
report any questions that need improvement.
Question 206: A 25-year-old male is admitted to the hospital for rectal bleeding, diarrhea,
purulent rectal discharge, tenesmus, and anal pain. He has no past history of similar rectal
bleeding. Examination shows erythema around the perianal region and the perineum and few
ulcers and vesicles. A colonoscopy reveals erythema, friability of rectal mucosa, ulcerations,
vesicles, and mucopurulent exudates. Rectal biopsies show negative PCR for Neisseria
gonorrhoeae and Chlamydia trachomatis. Tests for Treponema pallidum and HIV are all
negative. Which of the following is the next best step in the management of this patient?

Choices:
1. Viral cultures
2. Multidetector CT scan of pelvis and abdomen
3. PCR for Mycobacterium tuberculosis
4. Microscopic examination of swabs
Answer: 1 - Viral cultures
Explanations:
The patient’s presentation (rectal bleeding, diarrhea, purulent rectal discharge, tenesmus,
and anal pain together with skin erythema around the perianal region and the perineum and
the presence of few ulcers and vesicles) are consistent with herpes simplex 2 infection.
Other sexually transmitted infections including Neisseria gonorrhoeae, Chlamydia
trachomatis, Treponema pallidum, and HIV were excluded.
Therefore, viral cultures or swabs from rectal vesicles or ulcers to detect virus DNA by PCR
should be ordered to confirm the diagnosis
Multidetector CT scan of pelvis and abdomen may help in differentiating between bacterial
colitis and inflammatory bowel disease. However, these scans cannot confirm herpes
simplex-2 infection. Viral cultures or swabs from rectal vesicles or ulcers to detect virus
DNA by PCR should be ordered to confirm the diagnosis of herpes simplex 2. The patient’s
clinical presentation is not consistent with tuberculous infection. He is not HIV positive and
there are no indications that he is immunocompromised or at risk of developing
tuberculosis. Viral cultures or swabs from rectal vesicles or ulcers to detect virus DNA by
PCR should be ordered to confirm the diagnosis of herpes simplex 2. The patient’s
presentation is not consistent with bacterial colitis and Microscopic examination of swabs
will not help. Viral cultures or swabs from rectal vesicles or ulcers to detect virus DNA by
PCR should be ordered to confirm the diagnosis of herpes simplex 2.

Go to the next page if you knew the correct answer, or click the link image(s) below to further
research the concepts in this question (if desired).

Research Concepts:
Infectious Colitis

We update eBooks quarterly and Apps daily based on user feedback. Please tap flag to
report any questions that need improvement.
Question 207: A 35-year-old female is brought to the emergency department with
complaints of abdominal pain. Her blood pressure was initially measured as 70/40 mmHg by
Emergency Medical Service with a heart rate of 135 and a temperature of 39 C at her home. She
was given 2 L normal saline en route resulting in an improvement in blood pressure to 95/50 and
HR to 110 upon arrival to the emergency department. Physical examination shows lower
abdominal tenderness. Her blood work is significant for lactate of 4.2 mmol/L. A urinalysis
comes reports a high number of pus cells and bacteria while urine culture is pending. Despite
repeated efforts for about an hour, the phlebotomist is unable to draw blood cultures. The patient
has been in the hospital for about three hours now. Which of the following is the best next step in
the management of this patient?

Choices:
1. Keep attempting blood cultures for another hour and defer antibiotics
2. Administer antibiotics despite the possibility of cultures being negative thereafter
3. Give another saline bolus while deferring antibiotics and retry cultures afterward
4. Administer antibiotics and cancel blood cultures as urine cultures were already drawn
Answer: 2 - Administer antibiotics despite the possibility of cultures being negative
thereafter

Explanations:
Delay to antibiotic administration has been associated with increased in-hospital mortality.
Waiting to draw blood cultures in this instance would be inappropriate.
Delaying antibiotic administration compared between 1 hour and 3 hours of sepsis
identification has not been shown to affect patient-centered outcomes or mortality. However
the time to antibiotic administration has been shown to increase in-hospital mortality, so
delaying antibiotic administration beyond the 3-hour point in this patient would be
inappropriate.
While intravenous hydration is indicated, it should not delay cultures further.
Blood cultures should be drawn as this patient’s presentation is consistent with
pyelonephritis and acquiring urine cultures alone would not be enough. This can be from
bacteremia seeding the kidney rather than from an ascending urinary tract infection.

Go to the next page if you knew the correct answer, or click the link image(s) below to further
research the concepts in this question (if desired).

Research Concepts:
Bacterial Sepsis

We update eBooks quarterly and Apps daily based on user feedback. Please tap flag to
report any questions that need improvement.
Question 208: An elderly male with long-standing chronic obstructive pulmonary disease
(COPD) is rushed to the emergency room with exacerbation of his COPD. He is severely short of
breath and gasping. Quick blood work reveals an arterial blood gas of pH 7.15, PCO2 55, PO2
54 and HCO3 15. He is admitted to the intensive care unit and the pulmonologist intubates him.
He remains on the mechanical ventilator for 13 days and is eventually extubated. According to
the 2015 Global Initiative for Chronic Obstructive Lung Disease, what is the mortality of such
patients at 12 months after discharge?

Choices:
1. Less than 5%
2. 10% to 15%
3. 40% to 60%
4. 90%
Answer: 3 - 40% to 60%
Explanations:
COPD carries a high morbidity and mortality.
The GOLD report indicates that patients with COPD who require mechanical ventilation
have a high mortality at 12 months.
Any acute exacerbation should be promptly treated because delays often result in protracted
hospital admissions. The aim is to reduce the burden of COPD.
Factors associated with negative prognosis include hypercapnia, low body mass indices,
advanced age, low serum albumin, long-term use of corticosteroids and poor functional
status.

Go to the next page if you knew the correct answer, or click the link image(s) below to further
research the concepts in this question (if desired).

Research Concepts:
Chronic Obstructive Pulmonary Disease

We update eBooks quarterly and Apps daily based on user feedback. Please tap flag to
report any questions that need improvement.
Question 209: A 64-year-old male with a past medical history of diabetes mellitus,
hypertension, obesity, and tobacco abuse presents to the emergency department in a rural area
with crushing 10/10 substernal chest pain of 30 minutes duration. He receives 162 mg of aspirin
and 0.4 mg of sublingual nitroglycerin en route to the hospital. Emergency medical services
report ST elevations in V3-V6 which is corroborated by the EKG acquired upon arrival to the
emergency department. Initial troponin is 14.6 ng/mL. What is the most appropriate next step in
treatment?

Choices:
1. Transfer to percutaneous coronary intervention (PCI) capable facility 4 hours away
2. Thrombolytic therapy followed by transfer to a PCI capable facility 4 hours away
3. Thrombolytic therapy followed by a heparin drip
4. Start patient on 1 mg/kg of low molecular weight heparin
Answer: 2 - Thrombolytic therapy followed by transfer to a PCI capable facility 4 hours
away

Explanations:
In patients where percutaneous coronary intervention (PCI) cannot be performed in under 2
hours, the decision should be made to begin fibrinolytic therapy.
Patients who receive thrombolytic therapy should still be transferred to a PCI capable
facility to undergo PCI in the first 3 to 24 hours after the ST-elevation myocardial
infarction.
Thrombolytic therapy alone is only effective in 50% of patients.
Thrombolysis or PCI treatment is the treatment for ST-elevation myocardial infarction,
anticoagulation alone is not sufficient.

Go to the next page if you knew the correct answer, or click the link image(s) below to further
research the concepts in this question (if desired).

Research Concepts:
Acute ST Elevation Myocardial Infarction

We update eBooks quarterly and Apps daily based on user feedback. Please tap flag to
report any questions that need improvement.
Question 210: You are seeing a 64-year-old obese male who was admitted with some chest
pain. He says that the chest pain lasted a few minutes and he was admitted to ensure that he had
not suffered a myocardial infarction. The cardiac enzymes turned out to be normal and the ECG
is normal sinus rhythm. Today, you are asked to do the easy thing first. Measure the blood
pressure. You measure the blood pressure and notice that it is 90/80 mmHg in the right arm and
95/75 mmHg in the left arm. You measure the blood pressure three more times and get the same
result. What is the most likely cause of this patient's pathology?

Choices:
1. Smoking
2. Diabetes
3. Degenerative calcification
4. Rheumatic fever
Answer: 3 - Degenerative calcification
Explanations:
When the pulse pressure is narrow, think of heart failure, significant blood loss, or severe
aortic stenosis.
A low pulse pressure is commonly seen in shock and cardiac tamponade
The most common cause of aortic stenosis is degenerative, meaning that calcium starts to
deposit on the valves.
Other things that can cause low pulse pressure include reduced preload and dehydration.

Go to the next page if you knew the correct answer, or click the link image(s) below to further
research the concepts in this question (if desired).

Research Concepts:
Aortic Stenosis

We update eBooks quarterly and Apps daily based on user feedback. Please tap flag to
report any questions that need improvement.
Question 211: A 16-year-old patient with uncontrolled severe asthma presents to the
emergency department with fever, breathlessness, and cough with brownish expectoration for the
last seven days. His Laboratory findings show hemoglobin of 11.7 g/dL, white blood cells
5600/cmm, platelets 2,56,000/cmm, total IgE level 4000 kU/L and peripheral blood eosinophil
count 2540 cells/microL. Sputum examination shows fungal hyphae, fibrin, Charcot-Leyden
crystals, Curschmann spirals & dichotomous branching of fungal hyphae seen at 45-degree
angles. HRCT shows bronchiectasis in the right upper lobe. Which of the following organism is
most likely responsible for the patient's condition?

Choices:
1. Aspergillus niger
2. A. flavus
3. A. nidulans
4. A. fumigatus
Answer: 4 - A. fumigatus
Explanations:
As per patient's history, laboratory findings and radiological finding, most probable
diagnosis is allergic bronchopulmonary aspergillosis (ABPA). Aspergillus fumigatus is the
most common ubiquitous airborne fungus causative organism for ABPA.
Aspergillus conidia, because of its small diameter (2-3 µm), easily reach to the pulmonary
alveoli and deposits there. A. fumigatus species need dead organic matters to survive, so
they thrive on fallen leaves and its compost so infection present most profusely in the winter
throughout the world.
ABPA usually found in asthmatic patients mostly in severe asthmatic and people with cystic
fibrosis.
Aspergillus flavus, Aspergillus niger, Aspergillus terreus, and Aspergillus nidulans have
emerged as important pathogens in immunocompromised individuals. Aspergillus nidulans
is the second most encountered mold in CGD patients, causing almost exclusively invasive
infections.

Go to the next page if you knew the correct answer, or click the link image(s) below to further
research the concepts in this question (if desired).

Research Concepts:
Allergic Bronchopulmonary Aspergillosis

We update eBooks quarterly and Apps daily based on user feedback. Please tap flag to
report any questions that need improvement.
Question 212: A 48-year-old female patient in the ICU recovering from a brain surgery
presents with multiple small lesions on her legs. She has multiple 1 mm non-blanching lesions
throughout her legs, with the majority concentrated on her shins. The patient has a past medical
history significant for hypertension, diabetes insipidus, and glioblastoma. She has not regained
consciousness since her surgery two days ago. Her family states that the patient has had these
lesions before, but they are not sure of what precipitated them in the past. The patient is currently
receiving lisinopril, hydrochlorothiazide, and desmopressin. She is also receiving trimethoprim-
sulfamethoxazole for a urinary tract infection she developed in the hospital. Which of the
following is the most likely cause of this patient's symptoms?

Choices:
1. Desmopressin
2. Hydrochlorothiazide
3. Trimethoprim-sulfamethoxazole
4. Lisinopril
Answer: 1 - Desmopressin
Explanations:
Desmopressin is used in patients suffering from diabetes insipidus.
A rare side effect of desmopressin is the occurrence of thrombocytopenic purpura.
Patients with a history of thrombocytopenic purpura in the past are more at risk for
developing purpuras when on desmopressin.
Desmopressin acts as an ADH analog in patients suffering from diabetes insipidus to help
retain water in these patients.

Go to the next page if you knew the correct answer, or click the link image(s) below to further
research the concepts in this question (if desired).

Research Concepts:
Desmopressin

We update eBooks quarterly and Apps daily based on user feedback. Please tap flag to
report any questions that need improvement.
Question 213: An 81-year-old male presents to the emergency department with a chief
complaint of a severe headache. He reports the onset of his headache occurred approximately 1
hour after dinner at a friend's house where he ate charcuterie, assorted cheeses, pasta with
meatballs, and had one glass of red wine. The patient is alert and oriented to person, place, time,
and situation. He denies loss of consciousness or shortness of breath. His past medical history is
remarkable for myocardial infarction 3 years ago treated with emergent angioplasty and a history
of depression on and off for the past 20 years. The patient appears well groomed, well nourished,
and in distress. A Cincinnati stroke assessment is negative. His vitals are a heart rate of 52
beats/min, blood pressure 210/120 mmHg, respiratory rate 26, and SaO2 is 97% on room air.
The patient takes low dose aspirin, clopidogrel, metoprolol, and phenelzine daily. The patient
states that he was recently seen by a psychiatrist for the return of his depression prompting a
discontinuation of sertraline and the initiation of phenelzine. While waiting for a CT of the brain,
which of the following is the most beneficial medication to administer to the patient at this time?

Choices:
1. Verapamil
2. Propanolol
3. Norepinephrine
4. Fenoldopam
Answer: 4 - Fenoldopam
Explanations:
Fenoldopam has an onset of action of 10 minutes in adults and 5 minutes in children.
Peripheral vasodilation and blood pressure lowering will cause a compensatory tachycardia.
Routine vitals such as blood pressure and heart rate, in addition to serial ECGs, renal and
hepatic function tests, and serum potassium, should be monitored during fenoldopam
infusion.
A detailed evaluation of the chief complaint and inciting events can prove to be beneficial in
this patient. The patient's past medical history of depression being treated with phenelzine
and the recent ingestion of tyramine-containing food and drinks caused a hypertensive
crisis. Tyramine is found in many types of cured meat, cheeses, beers, and red wine. When
ingested in excess in a patient taking monoamine oxidase inhibitors (MAOIs), a
catecholamine surge occurs, causing an increase in blood pressure and heart rate. Because
this patient is taking a cardioselective beta blocker (metoprolol), the catecholamine surge is
not affecting his heart rate. A reflexive bradycardia can be seen as a normal compensatory
mechanism in patients with high blood pressure, depending on the etiology. However, it is
exaggerated in this patient due to the unopposed parasympathetic effect of vagal stimulation
from the carotid sinus baroceptors. In this case, fenoldopam, a dopamine D1 receptor
agonist, is the best medication to give, as it has a short onset of action and will not further
worsen the patient's bradycardia.

Go to the next page if you knew the correct answer, or click the link image(s) below to further
research the concepts in this question (if desired).

Research Concepts:
Fenoldopam

We update eBooks quarterly and Apps daily based on user feedback. Please tap flag to
report any questions that need improvement.
Question 214: A 32-year-old female who is obese and has a history of migraine headaches
flew from Los Angeles, CA to Aspen, CO and spent a day at a local spa. While in the sauna, she
became dizzy and sluggish and then developed a dull, moderately severe bilateral frontal
headache and nausea. She tried to stand but fell and was found oriented but lethargic. What is the
appropriate treatment?

Choices:
1. Acetazolamide, oxygen, and corticosteroids
2. Oral rehydration and bedrest
3. Have the patient take her own sumatriptan
4. Have the patient fly home to Los Angeles
Answer: 1 - Acetazolamide, oxygen, and corticosteroids
Explanations:
The patient has a high altitude headache with some features of acute mountain sickness.
The patient ascended from sea level to 7,900 feet of elevation, was likely dehydrated, and is
obese. These are all risk factors.
A high-altitude headache occurs within 24 hours after assent.
Acute mountain sickness can be life-threatening with resulting cerebral edema. Treatment
with acetazolamide, oxygen, and corticosteroids are indicated.

Go to the next page if you knew the correct answer, or click the link image(s) below to further
research the concepts in this question (if desired).

Research Concepts:
High Altitude Cerebral Edema

We update eBooks quarterly and Apps daily based on user feedback. Please tap flag to
report any questions that need improvement.
Question 215: An 85-year-old man was hospitalized because of acute exacerbation of
chronic obstructive lung disease secondary to lung infection. He has a history of chronic
bronchitis, heart failure with reduced ejection fraction, hypertension and coronary artery disease.
On the following night, his blood pressure dropped to 80/65 mm/Hg. He was started on a one-
liter normal saline fluid bolus. After 30 minutes, his repeated blood pressure was 82/64 mm/Hg.
On evaluation, the patient is feeling dizzy and not-fully comprehensive but denies any ongoing
chest pain. Pulse is 85 /min and oxygen saturation 84% on room air. Chest examination shows
bilateral fine crackles and jugular venous distention. The rest of the examination is
unremarkable. The patient is started on oxygen therapy and his oxygen saturation improved to
90%. He is transferred to the intensive care unit, and bedside ultrasound shows dilated inferior
vena cava. Bloodwork reveals worsened kidney function and elevated troponin. Which of the
following is the next best step in the management of this patient?

Choices:
1. Continue fluid resuscitation with another 1 L fluid bolus
2. Start him on loading dose loop diuretic followed by a diuretic infusion
3. Start him on aspirin and heparin drip
4. Proceed with venous ultrafiltration
Answer: 2 - Start him on loading dose loop diuretic followed by a diuretic infusion
Explanations:
Patients with a history of heart failure with reduced ejection fraction are more susceptible to
develop acute cardiorenal syndrome. This patient was admitted because of acute
exacerbation of his chronic bronchitis which resulted in acute cardiorenal syndrome, (type-1
cardiorenal syndrome).
Physical examination revealed bilateral crackles and jugular venous distention after 1 L of
fluid resuscitation with ultrasound finding of dilated inferior vena cava would be a sign of
fluid overload status secondary to the cardiorenal syndrome.
The primary treatment of cardiorenal syndrome to target fluid removal from the body either
with diuretic or ultrafiltration. Using inotropes are kept for resistance cases. There are
different types of diuretics: loops, thiazide, and potassium-sparing diuretics. Loop diuretics
including furosemide, torsemide, and bumetanide are the most potent diuretic they can be
used alone or conjugated with other types of diuretic.
Proceeding with further fluid therapy would result in worsening respiratory function status
as the patient has elevated venous pressure. He has mild elevation in troponin with no active
chest pain at the moment. It would be more reasonable to repeat the troponin for now before
starting him on a heparin drip. Finally, venous ultrafiltration can be used in resistance cases,
but recent studies showed that diuretic therapy was better than ultrafiltration on symptoms
control and creatinine level decline in the initial approach toward euvolemic.

Go to the next page if you knew the correct answer, or click the link image(s) below to further
research the concepts in this question (if desired).

Research Concepts:
Cardiorenal Syndrome

We update eBooks quarterly and Apps daily based on user feedback. Please tap flag to
report any questions that need improvement.
Question 216: A patient with end-stage renal disease who has missed her last scheduled
dialysis appointments presents to the emergency department with weakness, dyspnea, rales, and
jugular venous distension. The monitor shows a wide complex bradycardia. Which medication is
not indicated for her suspected electrolyte imbalance?

Choices:
1. Magnesium IV
2. Calcium gluconate IV
3. Polystyrene sulfonate PO or rectal
4. Glucose and regular insulin IV
Answer: 1 - Magnesium IV
Explanations:
Magnesium is not indicated in the treatment of suspect hyperkalemia, which is causing the
wide complex bradycardia.
Calcium gluconate stabilizes the cell membrane.
Polystyrene sulfonate helps to excrete potassium.
Glucose and insulin drive potassium back into the cells.

Go to the next page if you knew the correct answer, or click the link image(s) below to further
research the concepts in this question (if desired).

Research Concepts:
Hyperkalemia

We update eBooks quarterly and Apps daily based on user feedback. Please tap flag to
report any questions that need improvement.
Question 217: A 16-year-old female presented with fever, a target-shaped rash on her
thigh, and episodes of dizziness three days after a hike through the woods in New Hampshire.
Her electrocardiogram revealed a complete heart block with a junctional rhythm, and she
received one dose of ceftriaxone for Lyme carditis. Five hours after the ceftriaxone dose, she
became unresponsive with non-palpable pulses, telemetry showed ventricular tachycardia,
oxygen saturation dropped to 82%, and auscultation revealed bilateral crackles. She was
resuscitated with defibrillation and a 300 mg bolus of amiodarone, following which she was
started on inotropic support. An echocardiogram revealed a severely impaired systolic function
of the left and the right ventricles and a left ventricular ejection fraction of 12%. Coronary
angiography revealed patent vessels, and a biopsy revealed multiple foci of lymphocytic
infiltration (CD 3+) with associated myonecrosis. Serology was only positive for anti-Borrelia
burgdorferi IgM. The patient continued to be in a complete heart block with episodes of pauses
and ventricular tachycardia that persisted even after the placement of a temporary wire. Serial
CRP revealed a worsening trend over three days. What is the next best step?

Choices:
1. Increase the dose of ceftriaxone
2. Permanent pacemaker
3. Switch to doxycycline
4. Start corticosteroid therapy
Answer: 4 - Start corticosteroid therapy
Explanations:
Spirochetal diseases like Lyme disease can display a Jarisch-Herxheimer reaction after
initiation of antimicrobial therapy.
It is caused by the widespread release of antigens from microbial killing, causing cytokine
release.
This triggers a systemic inflammatory response which may cause worsening of the
myocarditis.
Steroid therapy can suppress inflammation and improve cardiac function.

Go to the next page if you knew the correct answer, or click the link image(s) below to further
research the concepts in this question (if desired).

Research Concepts:
Nonviral Myocarditis

We update eBooks quarterly and Apps daily based on user feedback. Please tap flag to
report any questions that need improvement.
Question 218: A 63-year-old female patient who was found confused with altered mental
status at home was brought to the emergency department which initial chest x-ray showed
suspected aspiration pneumonia with right lower lobe infiltrate and was admitted to intensive
care unit. Due to low hemoglobin of 5.1 g/dL, 2 packed red blood cells (PRBCs) were ordered,
and 1 PRBCs transfusion was immediately started. After 6 hours posttransfusion, oxygen
saturation dropped and required a non-rebreather mask from previous 2 L oxygen by nasal
cannula to keep oxygen saturation up to 91%. Blood pressure dropped from 132/87 mmHg to
87/62 mmHg. Chest auscultation revealed coarse breath sounds from the anterior chest with
bilateral crackles. Pretransfusion CBC had WBC of 10.2 and platelet of 180. Repeat CBC
showed WBC of 2.1 and platelet of 78. Intravenous furosemide 40 mg was given with no effect.
What is the most likely the diagnosis?

Choices:
1. Flash pulmonary edema
2. Transfusion-related acute lung injury (TRALI)
3. Acute respiratory distress syndrome (ARDS)
4. Transfusion-associated circulatory overload (TACO)
Answer: 2 - Transfusion-related acute lung injury (TRALI)
Explanations:
This case is most likely TRALI as, within 6 hours post-transfusion, the patient developed
hypotension.
TRALI does not respond to diuretic as this is noncardiogenic pulmonary edema as opposed
to flash pulmonary edema and TACO.
Other answer choices are possibilities, but this happened after transfusion
Leukopenia and thrombocytopenia happen with TRALI as TACO, flash pulmonary edema
and ARDS do not change WBC and platelet values.

Go to the next page if you knew the correct answer, or click the link image(s) below to further
research the concepts in this question (if desired).

Research Concepts:
Transfusion-related Acute Lung Injury

We update eBooks quarterly and Apps daily based on user feedback. Please tap flag to
report any questions that need improvement.
Question 219: An 83-year-old male developed a cholecysto-cutaneous fistula after
prolonged untreated cholecystitis. He has mild pain and asymptomatic otherwise, except the
drainage site at the right upper quadrant. Which of the following is the next best step in the
management of this patient?

Choices:
1. Pressure dressing to minimize the drainage amount
2. Routine wet to dry dressing
3. Stoma bag well fitted on the fistula opening
4. Suture closure of the fistula opening
Answer: 3 - Stoma bag well fitted on the fistula opening
Explanations:
The appropriate management is to apply a stoma bag to collect and quantitate the drainage
conveniently. It should be well fitted to prevent skin irritation.
The management of cholecyst-cutaneous fistula requires initial drainage of any associated
abscess and administration of appropriate antibiotics.
Before the definitive surgery is performed, an effort must be taken to ensure that there is no
associated bile duct obstruction or stones. An indication of an unresectable disease is if a
gallbladder fistula occurs in the setting of cancer with an associate dated mass. In this case,
the overall outcome is dismal because gallbladder cancer has a predilection for seeding and
spreading to the adjoining anatomical structures.
A pressure dressing is useful to stop bleeding sites. Fistula opening should not be sealed.
This will facilitate creating other alternative tracts of the fistula. A regular routine dressing
can be used. But it becomes wet and soaked easily which needs to be changed very
frequently. This makes impractical for nursing care. Closing the fistula opening by suturing
is contra-indicated. It promotes uncontrolled drainage through alternative tracts.

Go to the next page if you knew the correct answer, or click the link image(s) below to further
research the concepts in this question (if desired).

Research Concepts:
Cholecystocutaneous Fistula

We update eBooks quarterly and Apps daily based on user feedback. Please tap flag to
report any questions that need improvement.
Question 220: A 29-year-old male patient was admitted to the hospital after presenting an
erythematous rash with central clearing on his back and flu-like symptoms followed by
palpitations and dyspnea on exertion. There was evidence of third-degree atrioventricular block
on the electrocardiogram at the time of admission. Immediate treatment with intravenous
ceftriaxone was started. A temporary pacemaker was also placed. On day four of admission
patient is feeling better, palpitations and shortness of breath have subsided. The repeat
electrocardiogram shows no further P waves and QRS dissociation, heart rate of 76/min, and the
PR interval of 350 ms. The patient is complaining that he feels uncomfortable with the
pacemaker attached to his body and thinks he no longer needs it as his symptoms have improved.
What is the next best step in the management of this patient?

Choices:
1. Remove pacemaker and continue treatment with intravenous ceftriaxone and evaluation with
daily electrocardiogram
2. Remove pacemaker and discharge the patient on oral doxycycline to complete a total course of
21 days
3. Explain to the patient that pacemaker needs to continue attached to his body and that he will
continue daily evaluations with an electrocardiogram
4. Place permanent pacemaker
Answer: 1 - Remove pacemaker and continue treatment with intravenous ceftriaxone and
evaluation with daily electrocardiogram

Explanations:
A temporary pacemaker in patients with Lyme carditis and atrioventricular block can be
removed once second-degree or third-degree heart block has resolved.
Treatment with intravenous antibiotics should be continued until the PR interval shortens to
less than 300 ms.
After PR interval shortens to less than 300 ms, treatment can be switched to oral amoxicillin
or doxycycline to complete 14 to 21 days.
The recovery time in patients with third-degree heart block has a median of 6 days with
adequate antibiotic treatment. They do not usually need a permanent pacemaker.

Go to the next page if you knew the correct answer, or click the link image(s) below to further
research the concepts in this question (if desired).

Research Concepts:
Lyme Carditis

We update eBooks quarterly and Apps daily based on user feedback. Please tap flag to
report any questions that need improvement.
Question 221: A patient in respiratory distress presents to the emergency department. Her
pulse oximeter reads 88%, and her respiratory rate is 22/minute. A severe asthma exacerbation is
suspected. The other vital signs are normal, and a normal perfusion status is presumed. The only
other history comes from the patient's adult daughter who tells you that she also has type 2
diabetes mellitus and takes an oral medication which she took 3 hours earlier. Arterial blood gas
shows a high anion gap metabolic acidosis. How is this metabolic acidosis best classified?

Choices:
1. Diabetic ketoacidosis
2. Ethanol poisoning
3. Type A lactic acidosis
4. Type B lactic acidosis
Answer: 4 - Type B lactic acidosis
Explanations:
While diabetic ketoacidosis typically presents as a high anion gap metabolic acidosis in
diabetes mellitus type 1, this diagnosis is unlikely in a non-insulin dependent patient with
type 2 diabetes.
Nothing in this patient's history suggests that she is intoxicated.
Type A lactic acidosis is seen in low perfusion states such as shock. This patient's history is
not suggestive of a shock state.
Metformin-induced lactic acidosis is rare and often associated with hypoxic states. This
patient likely suffers from type B lactic acidosis which is that which is not associated with
low perfusion states.

Go to the next page if you knew the correct answer, or click the link image(s) below to further
research the concepts in this question (if desired).

Research Concepts:
Lactic Acidosis

We update eBooks quarterly and Apps daily based on user feedback. Please tap flag to
report any questions that need improvement.
Question 222: A 50-year-old male with alcohol use disorder is admitted with acute
pancreatitis. He admits to drinking a fifth of vodka daily and is having epigastric pain, nausea,
and has vomited once but denies hematemesis or hematochezia. Vital signs are pulse 115
beats/min, respirations 16 breaths/min, and blood pressure 160/80 mmHg. The abdomen reveals
decreased bowel sounds and is tympanic. There is tenderness to palpation at the mid epigastrium
without rebound, masses, or splenomegaly. The liver is 16 cm at the midclavicular line. Lipase is
1200 units/L, amylase 550 units/L, alanine aminotransferase 140 units/L, aspartate
aminotransferase 270 units/L, alkaline phosphatase 90 units/L, and albumin 2.5 grams/L. What is
the best management of this patient?

Choices:
1. Intravenous fluids and nasogastric tube
2. Analgesia, intravenous fluids, nothing by mouth, and antiemetics
3. Analgesia, nasogastric tube, intravenous fluids, and total parenteral nutrition
4. Analgesia, intravenous fluids, and clear liquids
Answer: 2 - Analgesia, intravenous fluids, nothing by mouth, and antiemetics
Explanations:
The patient will most likely recover in 3 to 7 days if he receives fluids and takes nothing by
mouth.
A nasogastric tube may need to be placed if there is protracted emesis. In mild cases, a
patient can be treated with antiemetics if there are no contraindications.
Nasojejunal feeding tubes are preferable to total parental nutrition in cases of severe
pancreatitis when oral feeds are not tolerated but nutritional support is needed. This reduces
the risk of infectious complications of total parental nutrition.
In mild cases of acute pancreatitis, the patient initially should take nothing by mouth. Oral
feeding with clear fluids or a low-fat diet can begin as soon as nausea, vomiting, and
abdominal pain has resolved.

Go to the next page if you knew the correct answer, or click the link image(s) below to further
research the concepts in this question (if desired).

Research Concepts:
Acute Pancreatitis

We update eBooks quarterly and Apps daily based on user feedback. Please tap flag to
report any questions that need improvement.
Question 223: A 17-year-old male is struck in the chest by a hard-driven ball while playing
baseball and immediately collapses. He is pulseless, and CPR is initiated by his coach, and EMS
is called. What is the most likely rhythm EMS will observe upon arrival?

Choices:
1. Ventricular fibrillation
2. Ventricular tachycardia
3. Asystole
4. Pulseless electrical activity (PEA)
Answer: 1 - Ventricular fibrillation
Explanations:
This presentation is most consistent with an episode of commotio cordis, which most
commonly present with ventricular fibrillation after impact to the chest during ventricular
repolarization (T-wave).
Commotio cordis is ventricular fibrillation precipitated by blunt trauma to the heart, not
attributable to structural damage to the heart or surrounding structures.
A history consistent with commotio cordis involves a sudden impact with the anterior chest
overlying the heart, followed by immediate cardiac arrest. This is most commonly a
baseball; however, any impact may be present in the appropriate circumstances. Ventricular
fibrillation may be observed if monitoring or an AED is available. Patients generally have
no history of structural heart disease to explain the dysrhythmia, and the injury is not
attributable to physical damage to the heart, cardiac contusion, or rupture. Penetrating injury
is not the cause for arrest.
Ventricular tachycardia is less likely than ventricular fibrillation in a structurally normal
heart with commotio cordis. Ventricular fibrillation is most likely the initial rhythm,
although it may degenerate into asystole over time if perfusion is not maintained. The
patient received immediate bystander CPR, so hopefully, his rhythm will still be shockable
ventricular fibrillation when EMS arrives. Ventricular fibrillation is most likely the initial
rhythm, although it may degenerate into PEA over time if perfusion is not maintained

Go to the next page if you knew the correct answer, or click the link image(s) below to further
research the concepts in this question (if desired).

Research Concepts:
Commotio Cordis

We update eBooks quarterly and Apps daily based on user feedback. Please tap flag to
report any questions that need improvement.
Question 224: A 72-year-old female presents with fever, cough, and moderate dyspnea
since last five days. She is tachypneic with a respiratory rate of 34/min. Chest radiographs reveal
lobar pulmonary infiltrates. Hospital admission can be decided based on the CURB-65 score.
What score indicates the need for intensive care unit admission?

Choices:
1. Greater than 2
2. Greater than 3
3. Greater than 4
4. Greater than 5
Answer: 3 - Greater than 4
Explanations:
CURB-65 and pneumonia severity index help stratify patients and determine if a patient
needs hospitalization for treatment of pneumonia.
CURB-65 Score of greater than or equal to 2 need hospitalization.
A CURB-65 score greater than 4 indicates the need for intensive care unit admission and
more intense therapy.
CURB-65 assigns one point for each factor including confusion, uremia (BUN greater than
20 mg/dL), respiratory rate greater than 30/min, hypotension (systolic less than 90 mmHg
and diastolic less than 60 mmHg), and, age more than 65 years.

Go to the next page if you knew the correct answer, or click the link image(s) below to further
research the concepts in this question (if desired).

Research Concepts:
Typical Bacterial Pneumonia

We update eBooks quarterly and Apps daily based on user feedback. Please tap flag to
report any questions that need improvement.
Question 225: A 52-year-old male patient was brought in 2 hours after an intentional
overdose with 20 diltiazem pills. He was intubated on arrival due to poor airway protection. He
has already received 3 liters of intravenous crystalloids and intravenous calcium chloride. High
dose insulin infusion has just been started. Blood pressure continues to remain low with a MAP
of 45 mmHg. Heart rate is 55/minute with sinus bradycardia and slightly prolonged QRS. A
monitor was attached to a radial arterial line and shows the following parameters: stroke volume
variability = 8%, cardiac output = 3.0 L/min, cardiac index= 2.1 L/min, systemic vascular
resistance = 90 dynes/sec/cm5. What will be the most appropriate intervention for his
hypotension?

Choices:
1. Start norepinephrine infusion
2. Start dobutamine infusion
3. Increase the dose of insulin infusion
4. Administer 1 liter of crystalloid bolus
Answer: 2 - Start dobutamine infusion
Explanations:
From the available hemodynamic parameters, his stroke volume variability (desired less
than 12%) and systemic vascular resistance (800 to1200 dynes/sec/cm5 ) are both within
normal limits. Neither intravenous fluids nor vasopressors will be as useful.
It is important to identify the etiology of shock in a setting of severe calcium channel
blocker toxicity. If reduced inotropic action is the root cause (reduced cardiac output and
index) initiation of an inotrope is the next best intervention.
Norepinephrine does have some inotropic action but dobutamine will be more potent in this
setting.
In refractory shock despite vasopressors and inotropes, intra-aortic balloon pump may need
to be considered. (IABP).

Go to the next page if you knew the correct answer, or click the link image(s) below to further
research the concepts in this question (if desired).

Research Concepts:
Calcium Channel Blocker Toxicity

We update eBooks quarterly and Apps daily based on user feedback. Please tap flag to
report any questions that need improvement.
Question 226: A 17-year old is hit in the neck area during a football game. He is
immediately brought to the emergency department because of shortness of breath. A lateral chest
x-ray of the neck reveals subcutaneous emphysema. The hyoid bone appears to be high in the
neck. What is the diagnosis?

Choices:
1. Lacerated thyroid gland
2. Transection of the cervical trachea
3. Ruptured thyroglossal duct cyst
4. Tracheoesophageal fistula
Answer: 2 - Transection of the cervical trachea
Explanations:
X-ray findings are important in patients with a tracheobronchial injury.
If there is subcutaneous emphysema and the hyoid bone is elevated, one must suspect
transection of the cervical trachea.
Other features on an x-ray of a tracheal injury include tracheal deviation or deformity.
Up to 20% of people with tracheobronchial injury may have no signs on the x-ray.

Go to the next page if you knew the correct answer, or click the link image(s) below to further
research the concepts in this question (if desired).

Research Concepts:
Tracheal Trauma

We update eBooks quarterly and Apps daily based on user feedback. Please tap flag to
report any questions that need improvement.
Question 227: A 17-year-old presented to the emergency room with chest pain, dyspnea,
and fever. He is an intravenous drug user. His vitals are temperature: 103 F, pulse: 109
beats/minute, blood pressure: 86/56 mm Hg, and respiratory rate of 22 breaths/minute.
Auscultation reveals findings suggestive of tricuspid regurgitation. Echocardiography supported
the clinical findings. Blood culture samples were sent, and the patient is started on broad-
spectrum antibiotics. On day 7, the sample grew a fungus. Which of the following feature would
suggest the fungus could be Aspergillus?

Choices:
1. Cutaneous macronodules
2. Black hemorrhagic skin lesions
3. Gram stain shows gram-positive cocci, arranged in grape-like clusters
4. Germ tube visible in high power microscopy of a centrifuged blood sample
Answer: 2 - Black hemorrhagic skin lesions
Explanations:
There are certain clinical features unique to certain fungal infections. Cutaneous
macronodules suggest Candida infection while black hemorrhagic skin lesions suggest
Aspergillus infection. These findings are unique but rare.
The right-sided heart valve endocarditis is common in the intravenous drug users.
Draw blood culture samples before starting broad-spectrum antibiotics.
The patient of infective endocarditis often needs fluid resuscitation so initial stabilization of
the patient is a key to the management.

Go to the next page if you knew the correct answer, or click the link image(s) below to further
research the concepts in this question (if desired).

Research Concepts:
Fungal Endocarditis

We update eBooks quarterly and Apps daily based on user feedback. Please tap flag to
report any questions that need improvement.
Question 228: A 17-year-old female presents to the emergency department with two days
of vomiting, abdominal pain, and bloody diarrhea. In questioning him, she notes that she recently
returned from a trip to the Louisiana coast where she visited a friend and says they ate a large
quantity of seafood, including some raw oysters. Which of the following is the most likely
infectious agent causing her current issue?

Choices:
1. Entamoeba histolytica
2. Campylobacter jejuni
3. Vibrio parahaemolyticus
4. Schistosomiasis
Answer: 3 - Vibrio parahaemolyticus
Explanations:
Infectious causes of bloody diarrhea include Campylobacter, Enterohemorrhagic strains of
Escherichia coli, Salmonella, Shigella, Yersinia, Clostridium difficile, Aeromonas, Vibrio,
Entamoeba, and Schistosomiasis.
Vibrio parahaemolyticus is associated with raw or undercooked seafood and most often
oysters. Seafood infected is most commonly found in warm coastal waters.
Potential symptoms include nausea, vomiting, watery or bloody diarrhea, abdominal pain or
cramping, and fever.
Onset is usually within 24 hours of ingestion. The illness can last up to 7 days and is
undercooked self-limiting.

Go to the next page if you knew the correct answer, or click the link image(s) below to further
research the concepts in this question (if desired).

Research Concepts:
Gastrointestinal Bleeding

We update eBooks quarterly and Apps daily based on user feedback. Please tap flag to
report any questions that need improvement.
Question 229: An adult patient is febrile, has a pulse of 120 beats/min, and a blood
pressure of 65/40 mmHg. The patient has been given 4 liters of crystalloids with minimal
response. What is the best next treatment for this patient?

Choices:
1. Norepinephrine
2. Dopamine
3. Digoxin
4. Vasopressin
Answer: 1 - Norepinephrine
Explanations:
Norepinephrine is more effective than dopamine in treating hypotension associated with
sepsis. Urine output is greater and lactic acid levels are lower. There also is better perfusion
of the splanchnic tissues. Dopamine can cause tachycardia and arrhythmias. Dopamine is
used only in highly specific situations.
Care should be given in identifying the right sympathomimetic agent by assessing perfusion
before initiation. In early septic shock, the patient may be in a warm, vasodilatory shock
state wherein norepinephrine is appropriate; however, patients who present in later stages of
shock may be in a cool, vasoconstrictive shock state wherein epinephrine is appropriate.
Most of the time, in fluid-refractory shock states, such as severe sepsis or septic shock, a
combination of norepinephrine and epinephrine is used.
Individuals with sepsis, who do not respond to early resuscitative fluid therapy in the
emergency department, show evidence of end-organ hypoperfusion and hemodynamic
monitoring and should be admitted to an intensive care unit.

Go to the next page if you knew the correct answer, or click the link image(s) below to further
research the concepts in this question (if desired).

Research Concepts:
Septic Shock

We update eBooks quarterly and Apps daily based on user feedback. Please tap flag to
report any questions that need improvement.
Question 230: A 66-year old male with emphysema had a sigmoid resection for cancer ten
days ago. He now returns with complaints of mild shortness of breath which started a few days
ago. He states that he is still not able to tolerate a solid diet and is only drinking liquids. He did
have bouts of nausea and vomiting for the first three days, but now these symptoms subsided
after he started to pass flatus. Because of the incisional pain, he has limited himself to the home.
Vitals reveals that he is afebrile and his blood pressure is 90/45 mmHg, heart rate 90 bpm and his
room air pulse oximetry is 87%. His electrocardiogram shows non-specific ST changes. Which
of the following tests should be ordered immediately?

Choices:
1. Chest x-ray
2. Arterial blood gas
3. Cardiac enzymes
4. CT of the chest
Answer: 4 - CT of the chest
Explanations:
Specific historical features that assist in the diagnosis of a deep venous thrombosis (DVT)
are those related to DVT risk factors and include a history of cancer, exogenous estrogen
therapy, recent surgery, smoking tobacco, previous history of DVT, immobility, age, history
of a hypercoagulable state, and other comorbidities.
Patients often will present with a chief complaint of unilateral leg swelling and discomfort.
Be mindful to ask about symptoms related to a pulmonary embolism (PE) as well, such as
chest pain, shortness of breath, and syncope.
Rapid diagnosis or rule-out by the emergency provider can expedite necessary treatment,
reduce the length of stay, and is particularly useful where access to 24-hour ultrasound is
unavailable.
CT of the chest should be done to determine if the patient has had a PE.

Go to the next page if you knew the correct answer, or click the link image(s) below to further
research the concepts in this question (if desired).

Research Concepts:
Deep Vein Thrombosis

We update eBooks quarterly and Apps daily based on user feedback. Please tap flag to
report any questions that need improvement.
Question 231: A 17-year-old male presents to the emergency department via emergency
medical services for altered mental status after intravenous use of phencyclidine. The patient is
extremely agitated requiring physical restraint. He has a heart rate of 145 bpm, blood pressure
170/90 mmHg, respiratory rate of 22/minute, and oxygen saturation of 90% on room air.
Fingerstick blood glucose is 102 mg/dL. He is warm to the touch, diaphoretic, and has rotary
nystagmus. The patient then begins to have a generalized seizure. The patient has no IV access.
What is the next most appropriate step?

Choices:
1. Administer 1000 g of IM fosphenytoin
2. Administer 2 to 4 mg of IM lorazepam
3. Administer 10 mg of IM midazolam
4. Place the patient in a quiet dark room
Answer: 3 - Administer 10 mg of IM midazolam
Explanations:
Intravenous benzodiazepines are the drug of choice to treat phencyclidine-induced seizures.
Lorazepam 2 to 4 mg IM or IV, or diazepam 5 to 10 mg IM or IV can be given. Both of
these medications take 15 to 20 minutes for onset when given IM. Therefore, IM midazolam
is preferred when there is no IV access.
Seizures, rhabdomyolysis, coma, hypertensive crisis, myocardial infarction, and
intraparenchymal hemorrhage are all potential complications of phencyclidine use.
Phencyclidine is available as a powder, crystal liquid, or tablet. It can be ingested orally,
injected intravenously, inhaled, or smoked.
Intravenous phencyclidine use can produce symptoms in 2 to 5 minutes.

Go to the next page if you knew the correct answer, or click the link image(s) below to further
research the concepts in this question (if desired).

Research Concepts:
Phencyclidine Toxicity

We update eBooks quarterly and Apps daily based on user feedback. Please tap flag to
report any questions that need improvement.
Question 232: A 45-year-old man presents for a six-month history of progressive dyspnea
and nonproductive cough. He has smoked one pack per day of cigarettes for 20 years, and he
works as a plumber. Current medications include antacids and acetaminophen as needed. His
oxygen saturation is 92% on room air. Bibasilar crackles without wheezing is noted on
auscultation. The rest of the exam is within normal limits. Pulmonary function tests show the
following: forced vital capacity (FVC) 62% of predicted, forced expiratory volume at 1 second
(FEV1) 59% of predicted, total lung capacity 70% of predicted, vital capacity 50% of predicted,
residual volume 70% of predicted, and diffusion lung capacity (DLCO) 49% of predicted. High-
resolution CT (HRCT) shows diffuse ground-glass disease bilaterally with no enlarged
mediastinal or hilar lymphadenopathy. What is the best next step in the management of this
patient?

Choices:
1. HRCT findings should be sufficient to confirm the diagnosis
2. Bronchoscopy
3. Surgical lung biopsy
4. Initiate antifibrotic therapy
Answer: 3 - Surgical lung biopsy
Explanations:
While high-resolution CT findings are helpful in the initial diagnostic workup, it is often
nonspecific when classic radiological findings of idiopathic pulmonary fibrosis (IPF) are
lacking. Classic findings include honeycombing with or without peripheral traction or
bronchiectasis and subpleural basal predominant with heterogeneous distribution.
Bronchoscopy is used to rule out infectious etiology; however, it is not recommended to
confirm the diagnosis due to the inability to obtain adequate lung samples. Transbronchial
biopsy specimens are considered inadequate to diagnose desquamative interstitial
pneumonia (DIP) or to distinguish it clearly from overlapping disease patterns like
respiratory bronchiolitis-associated interstitial lung disease (RB-ILD) in early stages and
nonspecific interstitial pneumonitis (NSIP) in late stages.
Surgical lung biopsy is considered the criterion standard in diagnosing DIP and other types
of interstitial lung disease (ILD). In DIP, open lung biopsy shows infiltration of alveoli with
macrophages rich in eosinophilic cytoplasm and a brown pigment called "smoker's
pigment."
The initiation of antifibrotics is only indicated in eligible patients with a confirmed
diagnosis of idiopathic pulmonary fibrosis (IPF).

Go to the next page if you knew the correct answer, or click the link image(s) below to further
research the concepts in this question (if desired).

Research Concepts:
Desquamative Interstitial Pneumonia

We update eBooks quarterly and Apps daily based on user feedback. Please tap flag to
report any questions that need improvement.
Question 233: A 55-year-old female presents with 3 days of fever, headache, and
progressive cognitive decline. She has no significant medical history. She has no sick contacts
and no recent travel history. She does not use tobacco, or illicit drugs, but drinks alcohol
occasionally. Temperature is 38 C, blood pressure is 126/84, and pulse is 72/min. Mucous
membranes are moist with no lesions. No lymphadenopathy is present. Lungs are clear to
auscultation and heart sounds are normal with no murmur. The abdomen is soft and nontender
with no hepatosplenomegaly. She has nuchal rigidity and is alert and oriented only to self. She
has no motor or sensory deficits but can only follow simple commands. Cerebrospinal fluid
(CSF) analysis shows a white blood cell count of 150/mm3, red blood cell count of 3/mm3,
protein concentration of 130 mg/dL, and glucose concentration of 40 mg/dL. CSF polymerase
chain reaction (PCR) testing is positive for herpes simplex virus 1. Magnetic resonance imaging
is most likely to show involvement of which part of the brain?

Choices:
1. Parietal lobe
2. Temporal lobe
3. Occipital lobe
4. Corpus callosum
Answer: 2 - Temporal lobe
Explanations:
Herpes simplex virus (HSV) encephalitis usually involves the temporal and frontal lobes,
hence causing predominately psychiatric features, memory disturbance, and aphasia.
Magnetic resonance imaging is the most sensitive and specific imaging modality for HSV
encephalitis.
Temporal lobe abnormalities on brain imaging are strongly suggestive of HSV encephalitis.
Temporal lobe lesions are usually unilateral, but extra-temporal abnormalities may also be
seen.

Go to the next page if you knew the correct answer, or click the link image(s) below to further
research the concepts in this question (if desired).

Research Concepts:
Viral Encephalitis

We update eBooks quarterly and Apps daily based on user feedback. Please tap flag to
report any questions that need improvement.
Question 234: A middle-aged male with long-standing chronic obstructive pulmonary
disease (COPD) presents with dyspnea, fatigue, and copious sputum production to the
emergency department. Twenty-four hours ago, he noted a low-grade fever and a runny nose.
Physical exam reveals respiratory distress with the use of accessory muscles. He has marked
wheezing and no air entry bilaterally. He is immediately treated with oxygen and nebulizer
therapy but fails to improve. The pulmonologist feels that the patient may need a bronchoscopy
for diagnosis and therapeutic reasons. During bronchoscopy, what percentage of COPD patients
with exacerbations have microorganisms in the lower airways?

Choices:
1. Less than 3%
2. About 10%
3. About 50%
4. More than 90%
Answer: 3 - About 50%
Explanations:
Bronchoscopy is sometimes performed to sample the airways for the bacteria in patients
who are not responding to treatment.
Studies suggest that at least 50% of patients have microorganisms in the lower airway
during exacerbation of COPD. B. R. Celli, P. J. Barnes. Exacerbations of chronic
obstructive pulmonary disease European Respiratory Journal Jun 2007, 29 (6) 1224-1238;
DOI: 10.1183/09031936.00109906 https://erj.ersjournals.com/content/29/6/1224
At the same time, a significant number of patients also have colonization of the airways,
making it difficult to interpret the data. However, the one feature is that the bacterial burden
increases in the airways during an exacerbation.
New organisms are most likely in patients with symptoms of wet cough and purulent
sputum.

Go to the next page if you knew the correct answer, or click the link image(s) below to further
research the concepts in this question (if desired).

Research Concepts:
Chronic Obstructive Pulmonary Disease

We update eBooks quarterly and Apps daily based on user feedback. Please tap flag to
report any questions that need improvement.
Question 235: A patient with head and abdominal trauma was admitted to the surgical
intensive care unit and eventually was started on total parenteral nutrition (TPN). After three
weeks, the area around the central line became red and indurated. The patient had no fever, and
the white blood cell count was normal. Cultures and the Gram stain from the site reveal Gram-
positive bacterium. Based on this information, the clinician has ordered the removal of the line,
and TPN stopped. The tip was sent for culture. Four hours later, the patient is very lethargic and
obtunded. Which of the following is the next best step in the management of this patient?

Choices:
1. Obtain blood cultures and start antibiotics
2. Give glucagon intravenously
3. Give glucose intravenously
4. Obtain a CT scan of the head
Answer: 3 - Give glucose intravenously
Explanations:
Complications of total parenteral nutrition (TPN) vary, but the most common complication
is glucose abnormalities. To avoid hyperglycemia, one should monitor plasma glucose and
adjust insulin levels in the TPN solution as needed. Alternately, the sudden discontinuation
of the dextrose solution can precipitate sudden hypoglycemia. Abrupt removal of a
continuous dextrous source causes the circulating insulin to induce hypoglycemia.
Management of TPN therapy includes monitoring of access site and changing of solution
and tubing per protocol every 24 hours. The nurse should record daily weights and accurate
intake and output in the patient's chart.
The dietitian collaborates with members of the interdisciplinary team, using a calculation
based on the Harris-Benedict equation to start feedings based on individualized nutrient
needs.
One orders TPN therapy for patients whose gastrointestinal tract is non-functional because
of either traumatic injury or medical conditions. Central venous access is best for TPN to
allow for higher concentrations of glucose.

Go to the next page if you knew the correct answer, or click the link image(s) below to further
research the concepts in this question (if desired).

Research Concepts:
Total Parenteral Nutrition

We update eBooks quarterly and Apps daily based on user feedback. Please tap flag to
report any questions that need improvement.
Question 236: A patient presents to the emergency department with witnessed prehospital
cardiac arrest. The initial rhythm was ventricular fibrillation. After multiple defibrillation
attempts, high-quality compressions, and medication administration return of spontaneous
circulation (ROSC) has occurred. Hemodynamic stability is present, the comatose state
continues, and the core temperature is 37.0 degrees C. Definitive airway established and suitable
placement confirmed. What is the following appropriate treatment modality to consider in this
patient?

Choices:
1. Continue high quality compressions
2. Targeted Temperature Management (Therapeutic hypothermia)
3. Needle decompression
4. Aggressive rewarming techniques
Answer: 2 - Targeted Temperature Management (Therapeutic hypothermia)
Explanations:
Targeted temperature management is recommended by the American Heart Association as
long as the patient is still unresponsive. The purpose of targeted temperature management is
to reduce cerebral metabolism and improve oxygen stores, as well as reversing ischemic
depolarization of CNS to stabilize cell membranes. Monitor closely for complications such
as hypotension, potassium abnormalities, hyperglycemia, and ongoing ischemia.
Temperature is monitored by more than one probe. Bladder probes are not accurate if there
is no urine output. Esophageal, rectal probes or pulmonary artery temperature probes may
be better options for primary and secondary monitoring. Shivering should be avoided
because it raises the body temperature. Transfer these patients to a critical care bed quickly.
Targeted temperature management, formerly known as therapeutic hypothermia, has been
proven to be of benefit in adults with witnessed, documented, prehospital ventricular
fibrillation arrest with the deliberate reduction in body temperature to 32 degrees C to 36
degrees C. This hypothermic state has been proven quite beneficial in maintaining and
restoring normal neurological function and is huge in the advancement of medicine. Many
clinical trials are underway for its use in other arrest situations and the pediatric population.
It is showing promise and of benefit in neonatal hypoxic-ischemic encephalopathy or
asphyxiated newborns. More data is needed before cooling becomes the standard of care in
children. A practitioner may decide to initiate cooling in patients with return of spontaneous
circulation after pulseless electrical activity, asystole, or even an in-house cardiac arrest but
the cause of the arrest will be a consideration. Sepsis patients are not good candidates for
cooling due to compromised immune systems already, and hypothermia will further
complicate the condition.
The American Heart Association recommends cooling in adults for at least 24 hours post
prehospital ventricular fibrillation arrest with a return of spontaneous circulation (ROSC)
that remain unconscious. If hypothermia exists already, > 33 degrees C and
hemodynamically stable after ROSC, do not actively rewarm. Cool as soon as possible by
starting in the emergency department and maintain neuroprotective treatment such as
elevating the head of the bed 30 degrees and seizure precautions.
Hypothermia is contraindicated in patients with sepsis, impaired clotting systems, recent
major surgery in the past 14 days, ingestion drug coma, or who have active bleeding, as in
trauma patients. A targeted temperature goal usually is reached by the 3-4 hour mark after
cooling initiated. Rewarming will start around 24 hours later. Do not delay cooling after
ROSC. Cooling early preserves brain cells. Keep MAP over 80. Osbourn waves may be
present with cooling. Bradycardia with a heart rate of less than 40 is common and does not
need treatment unless an unstable patient. Rewarm slowly at a rate of 0.3-0.5 degrees C
every hour. This process could last around 8 hours or more. Paralytics originally started to
control shivering during cooling, should be discontinued for the start of rewarming.

Go to the next page if you knew the correct answer, or click the link image(s) below to further
research the concepts in this question (if desired).
Research Concepts:
Targeted Hypothermia Temperature Management

We update eBooks quarterly and Apps daily based on user feedback. Please tap flag to
report any questions that need improvement.
Question 237: A 45-year-old trauma patient with no comorbidities is diagnosed with blunt
cardiac injury (BCI) due to abnormalities on ECG and an elevated cardiac troponin I level.
However, the patient is stable and can be admitted to the floor under telemetry for continuous
cardiac monitoring. What further studies should be ordered?

Choices:
1. ECG with cardiac troponin I level every 6 hours and an echocardiogram
2. ECG with cardiac troponin I level every 6 hours
3. ECG with cardiac troponin I level every 6 hours, an echocardiogram, and a cardiology consult
4. Cardiac troponin I level every 6 hours and an echocardiogram
Answer: 1 - ECG with cardiac troponin I level every 6 hours and an echocardiogram
Explanations:
Abnormal ECG findings warrant admission for continuous cardiac monitoring for 24 to 48
hours.
Following an abnormal ECG and cTnI level, echocardiography is usually obtained to
further characterize the injury. If there are wall motion abnormalities, cardiology should be
consulted.
Patients with BCI can be admitted to the intensive care unit or under telemetry depending
on concurrent injuries, type of ECG change, and the grade of hemodynamic imbalance.
Patients with a normal ECG in conjunction with normal levels of cardiac troponin I can be
discharged home safely if there are no other clinical concerns.

Go to the next page if you knew the correct answer, or click the link image(s) below to further
research the concepts in this question (if desired).

Research Concepts:
Blunt Cardiac Injury

We update eBooks quarterly and Apps daily based on user feedback. Please tap flag to
report any questions that need improvement.
Question 238: A 65-year-old male patient is hospitalized for acute alcohol withdrawal.
During the hospitalization, he becomes increasingly confused, combative, and has hallucinations
of bugs crawling on him. He is tachycardic, hypertensive, febrile, and has a whole body tremor.
What is the next best step in the management of this patient?

Choices:
1. IV antipsychotic
2. Immediate brain CT scan
3. IV vasodilator
4. IV benzodiazepine
Answer: 4 - IV benzodiazepine
Explanations:
Delirium tremens (DTs) is a severe form of alcohol withdrawal that can be life-threatening.
DTs occur between 10 and 72 hours after the last drink and are associated with fever,
nausea, hallucinations, agitation, hypertension, tachycardia, diaphoresis, and tremor.
Benzodiazepines are the drug of choice in managing all degrees of alcohol withdrawal and
are used for prophylaxis.
Any patient suspected of alcohol use disorder should be placed on thiamine to prevent
Wernicke encephalopathy.

Go to the next page if you knew the correct answer, or click the link image(s) below to further
research the concepts in this question (if desired).

Research Concepts:
Delirium Tremens

We update eBooks quarterly and Apps daily based on user feedback. Please tap flag to
report any questions that need improvement.
Question 239: A patient was admitted to the ICU with an acute ischemic stroke involving
the right middle cerebral artery. She did not receive embolectomy. After 12 hours in the ICU, she
began to demonstrate erratic respirations, tachycardic arrhythmias, with decreased
responsiveness, followed by what appeared to be tonic-clonic activity. Assuming no hemorrhage
was present on CT of the head, what strategy can be used to rapidly stabilize, excluding surgical
interventions, this severely symptomatic patient?

Choices:
1. Furosemide drip
2. Dexamethasone 4 mg IVP every 8 hours
3. Blood pressure reduction with nitroprusside drip
4. Short term hyperventilation via mechanical ventilation
Answer: 4 - Short term hyperventilation via mechanical ventilation
Explanations:
Hyperventilation of an intubated patient to a goal PCO2 of 26 to 30 mmHg can reduce
intracranial blood flow, and thereby pressures, by 3% for every 1 mmHg drop in PCO2.
Hyperventilation strategies should not be continued beyond the first 24 hours to prevent the
risk of vasoconstriction and worsened brain ischemia.
Hyperventilation strategies should be used only in patients with intracranial pressure (ICP)
monitoring.
Care must be taken with patients on mechanical ventilation to ensure adequate blood
pressure is maintained in order to maintain adequate cerebral perfusion pressure.
Vasopressors may be used if necessary, as these are not shown to increase ICP as they
increase blood pressure if blood pressure drops as a result of sedation used for the ventilated
patient.

Go to the next page if you knew the correct answer, or click the link image(s) below to further
research the concepts in this question (if desired).

Research Concepts:
Cerebral Edema

We update eBooks quarterly and Apps daily based on user feedback. Please tap flag to
report any questions that need improvement.
Question 240: A patient who suffered a multiorgan injury is intubated in the intensive care
unit. His ventilator settings rate is 14, FiO2 60%, total volume 450 ml, and positive end-
expiratory pressure of 12. At night, the nurse notices a significant amount of subcutaneous
emphysema and elevated peak pressures. What should be the next step in the management of this
patient?

Choices:
1. Blood gas
2. CT scan
3. Place chest tubes
4. Bronchoscopy
Answer: 3 - Place chest tubes
Explanations:
In any ventilated patient with a high positive end-expiratory pressure (PEEP), barotrauma
can often present with a pneumothorax. Sometimes it may present as subcutaneous
emphysema.
Many patients on the ventilator with high PEEP have prophylactic chest tubes inserted.
PEEP can also cause a decline in cardiac output.
In such a scenario the placement of chest tubes can be life saving, as hemodynamic
compromise can occur anytime. No time should be wasted obtaining chest x-rays.
In rare cases, the endotracheal tube can cause a posterior tear in the trachea and cause
subcutaneous emphysema.

Go to the next page if you knew the correct answer, or click the link image(s) below to further
research the concepts in this question (if desired).

Research Concepts:
Iatrogenic Pneumothorax

We update eBooks quarterly and Apps daily based on user feedback. Please tap flag to
report any questions that need improvement.
Question 241: An 81-year-old man presents to the emergency department with four days of
epigastric abdominal pain and non-bloody vomiting. He is afebrile but has a blood pressure of
90/50 mmHg and a heart rate of 130 bpm. On examination, he has a tender upper abdomen and
bilateral ecchymosis of the flanks. Which of the following is the most likely diagnosis in this
setting?

Choices:
1. Acute inferior myocardial infarction
2. Acute ascending thoracic aortic dissection
3. Acute necrotizing pancreatitis
4. Upper gastrointestinal bleeding
Answer: 3 - Acute necrotizing pancreatitis
Explanations:
Although it is an uncommon finding the appearance of ecchymotic discoloration on the
flanks points to intraabdominal or retroperitoneal bleeding, classically due to severe acute
necrotizing pancreatitis. The exam finding is referred to as the Grey Turner sign named after
a British surgeon who published the finding in 1920.
In a few patients with severe necrotizing pancreatitis, the liberated enzymes destroy
peripancreatic tissues causing bleeding that tracks through various tissue planes to reach the
subcutaneous tissues causing the discoloration on the flanks.
The vast majority of patients with pancreatitis do not show subcutaneous signs of
retroperitoneal bleeding. Thus, clinicians cannot use the absence of a Grey Turner sign to
rule out severe pancreatitis.
Almost any process that causes intraabdominal or retroperitoneal bleeding may result in
blood tracking to the subcutaneous tissues resulting in the Grey Turner sign. Therefore, it is
not specific to pancreatitis.

Go to the next page if you knew the correct answer, or click the link image(s) below to further
research the concepts in this question (if desired).

Research Concepts:
Grey-Turner Sign

We update eBooks quarterly and Apps daily based on user feedback. Please tap flag to
report any questions that need improvement.
Question 242: A 70-year-old male with a past medical history of hypertension and
hyperlipidemia is brought to the emergency department by for complaints of left-sided weakness
and numbness. He states that he passed out after having some beers and cigarettes and woke up
two hours later with left-sided weakness. A CT scan of the head without contrast shows no
evidence of hemorrhage. The patient is given IV TPA. His initial lab work was positive for an
initial troponin of 2.96 ng/ml, and his second troponin is 3.55 ng/ml. His ECG shows a 0.5mm T-
wave inversion in lead V2 and nonspecific ST-T wave changes in other precordial leads. What is
the most likely causes of the elevated troponins and what medication(s) should be started next?

Choices:
1. The patient has a stroke; start aspirin and statin now
2. The patient has a stroke; start aspirin, clopidogrel and statin now
3. The patient has a stroke; start aspirin and clopidogrel at least 24 hours later and start statin
now
4. The patient has a non-ST elevation myocardial infarction; start heparin infusion and statin now
Answer: 3 - The patient has a stroke; start aspirin and clopidogrel at least 24 hours later and
start statin now

Explanations:
The elevated troponins are likely due to the patient’s stroke. Starting aspirin alone right
away would not be appropriate. Instead, dual antiplatelet therapy with aspirin and
clopidogrel should be held for at least 24 hours after giving TPA and then started.
Dual antiplatelet therapy with aspirin and clopidogrel for 21 days has been proven superior
to aspirin alone and should be given with a statin for secondary stroke prevention, but
antiplatelet agents should be held for at least 24 hours after giving TPA.
As the troponin elevation is due to stroke and the patient has no noted history of bleeding,
dual antiplatelet therapy with aspirin and clopidogrel should be started and continued for 21
days for secondary stroke prevention followed by continuation on aspirin and careful
outpatient follow up care. No contraindication to starting a statin right away is present.
The elevation in troponins, in this case, is not an indicator of non-ST elevation myocardial
infarction but is instead more likely a consequence of the patient’s stroke, which is part of
the differential for medical conditions associated with positive troponins. As a result, dual
antiplatelet therapy would be the more appropriate choice.

Go to the next page if you knew the correct answer, or click the link image(s) below to further
research the concepts in this question (if desired).

Research Concepts:
Cardiac Enzymes

We update eBooks quarterly and Apps daily based on user feedback. Please tap flag to
report any questions that need improvement.
Question 243: A 40-year-old woman with a history of seizure disorder, bipolar disorder
type 1, and newly diagnosed atrial fibrillation presents to the emergency department for
evaluation of altered mental status. The patient has a list of medications that was updated this
week after starting a new medication. She has taken carbamazepine for a seizure disorder for ten
years with no recent dosage changes. Vital signs are heart rate 60/bpm, blood pressure 105/55
mmHg, respiratory rate 10/minute, and oxygen saturation 96% on room air. The patient is
somnolent but arousable. She is oriented to self only. Her physical exam is otherwise
unremarkable. CBC, basic metabolic profile, urinalysis, and CT of the head without contrast are
within normal limits. Carbamazepine level is 45 mcg/ml. What medication has likely contributed
to the patient's current condition?

Choices:
1. Diltiazem
2. Lithium
3. Metoprolol
4. Warfarin
Answer: 1 - Diltiazem
Explanations:
The toxic concentration of carbamazepine is 40 mcg/ml. The risk of seizure increases
significantly at level greater than 40 mcg/ml.
Carbamazepine (CBZ) is metabolized by the cytochrome P450 enzyme CYP3A4. Many
medications, including allopurinol, diltiazem, fluoxetine, isoniazid, and valproic acid,
inhibit the activity of the CYP3A4 enzyme, which can lead to elevated serum concentration
of CBZ. A serum CBZ level and current medication list should be obtained when evaluating
mental status changes in a patient treated with CBZ.
Carbamazepine is also a potent CYP 3A4 inducer and can cause decreased serum levels of
other anticonvulsants such as valproic acid and phenytoin) and can induce seizures
secondary to decreased serum levels of these medications.
Dialysis is the definitive treatment for CBZ toxicity in the setting of high serum
concentrations and significant central nervous system depression.

Go to the next page if you knew the correct answer, or click the link image(s) below to further
research the concepts in this question (if desired).

Research Concepts:
Anticonvulsants Toxicity

We update eBooks quarterly and Apps daily based on user feedback. Please tap flag to
report any questions that need improvement.
Question 244: A 17-year-old female with no significant past medical history was
hospitalized after a gunshot wound to the abdomen. She had a complicated postoperative course
requiring multiple re-interventions for anastomotic leakage. She is receiving total parenteral
nutrition via a central venous catheter and is on broad-spectrum antibiotics for the bowel
perforation. After 8 days in the intensive care unit, the patient became febrile and hypotensive.
Urinalysis and chest x-ray show no clear focus of infection. Blood cultures are pending. Beta-d-
glucan is positive for candida of unknown species. What is the best initial therapy?

Choices:
1. Fluconazole
2. Amphotericin B
3. Micafungin
4. No additional medication, wait for the blood cultures
Answer: 3 - Micafungin
Explanations:
An echinocandin is the first-line treatment for candidemia. Echinocandin resistance is rare.
Fluconazole is an alternative first-line treatment in non-critically ill patients or if the
cultures show azole-sensitive candida species.
Amphotericin B also is a therapeutic option in patients who are neutropenic or unable to
tolerate echinocandins or azoles.
The central venous catheter should be removed as early as possible if it is the presumed
source of candidemia.

Go to the next page if you knew the correct answer, or click the link image(s) below to further
research the concepts in this question (if desired).

Research Concepts:
Fungemia Candidiasis

We update eBooks quarterly and Apps daily based on user feedback. Please tap flag to
report any questions that need improvement.
Question 245: A 32-year-old male is admitted to the intensive care unit for progressive
change in mental status, delirium, and rapidly progressive cognitive impairment. His medical
history is significant for advanced HIV disease. MRI of the brain shows evidence of
periventricular inflammation and meningeal enhancement. What is the recommended treatment?

Choices:
1. Acyclovir 10 mg/kg IV every 8 hours for 14 to 21 days
2. Acyclovir 10 to 15 mg/kg IV every 8 hours for 10 to 14 days
3. Ganciclovir 5 mg/kg IV every 12 hours
4. Ganciclovir 5 mg/kg IV every 12 hours and foscarnet 60 mg/kg IV every 8 hours for 21 days
Answer: 4 - Ganciclovir 5 mg/kg IV every 12 hours and foscarnet 60 mg/kg IV every 8
hours for 21 days

Explanations:
Immunocompromised individuals, such as those with advanced HIV disease, are at
increased risk of developing cytomegalovirus (CMV) encephalitis.
Neuroimaging of patients with CMV encephalitis can show periventricular inflammation
and meningeal enhancement.
Combination therapy of ganciclovir and foscarnet is shown to be more effective than
ganciclovir alone.
Recommended therapy for CMV encephalitis is ganciclovir 5 mg/kg IV every 12 hours and
foscarnet 60 mg/kg IV every 8 hours for 21 days or ganciclovir 5 mg/kg IV every 12 hours
and foscarnet 90 mg/kg IV every 12 hours for 21 days.

Go to the next page if you knew the correct answer, or click the link image(s) below to further
research the concepts in this question (if desired).

Research Concepts:
Viral Encephalitis

We update eBooks quarterly and Apps daily based on user feedback. Please tap flag to
report any questions that need improvement.
Question 246: Transesophageal echocardiography reveals an unruptured sinus of valsalva
aneurysm in a patient evaluated for widened mediastinum on chest x-ray. What is the next best
step in management?

Choices:
1. Chest MRI for confirmation of the aneurysm, and, if confirmed, immediate surgical
consultation
2. Follow up echocardiogram in 6 weeks
3. Holter monitor
4. Referral for outpatient cardiothoracic surgery consultation
Answer: 1 - Chest MRI for confirmation of the aneurysm, and, if confirmed, immediate
surgical consultation

Explanations:
Sinus of valsalva aneurysms can cause compression of adjacent coronary arteries resulting
in myocardial ischemia and infarction.
Aneurysms should be confirmed with cardiac MRI.
Urgent surgical intervention is generally warranted if the aneurysm is confirmed.
Patients will undergo coronary angiography before cardiac surgery to assess coronary
anatomy. Patients with low risk of coronary disease may have an evaluation of coronary
anatomy using cardiac computed tomography, and may not need to undergo cardiac
catheterization. Patients with intermediate or high risk for coronary artery disease will
usually undergo cardiac catheterization for assessment of possible bypass grafting at the
time of cardiac surgery.

Go to the next page if you knew the correct answer, or click the link image(s) below to further
research the concepts in this question (if desired).

Research Concepts:
Sinus Of Valsalva Aneurysm

We update eBooks quarterly and Apps daily based on user feedback. Please tap flag to
report any questions that need improvement.
Question 247: A 78-year-old female is admitted to the intensive care unit after a
hemicolectomy for a perforated diverticulum. She has a temperature 39.5C, blood pressure of
75/45 mmHg, and pulmonary arterial wedge pressure of 22 mmHg after 2 liters of normal saline.
Which of the following would be the most appropriate treatment for this patient?

Choices:
1. Expand intravascular volume with pentastarch
2. Begin dobutamine drip
3. Begin IV norepinephrine drip
4. Start antibiotics and obtain CT abdomen and pelvis
Answer: 3 - Begin IV norepinephrine drip
Explanations:
Colloids are not the recommended fluids for a hypotensive patient. The patient does not
appear to be intravascularly depleted. She appears to be in septic shock.
Dobutamine is not the recommended vasopressor for septic shock.
Norepinephrine is the recommended vasopressor in septic shock. The patient needs further
evaluation with an EKG and a transesophageal echocardiogram.
The patient is unstable and requires vasopressors. Although antibiotics are also needed, the
patient should be stabilized first.

Go to the next page if you knew the correct answer, or click the link image(s) below to further
research the concepts in this question (if desired).

Research Concepts:
Inotropes And Vasopressors

We update eBooks quarterly and Apps daily based on user feedback. Please tap flag to
report any questions that need improvement.
Question 248: A 72-year-old female with a BMI of 22 is ready for extubation as she has
passed a spontaneous breathing trial as per the intensive care unit ventilator liberation protocol.
Patient has been intubated for 11 days due to acute respiratory distress syndrome related to
community-acquired pneumonia. Review of the record showed that intubation was difficult and
multiple attempts had to be made to successfully intubate her. The patient is placed patient back
on volume control with a tidal volume of 500 and the cuff deflated. An average of the lowest 3
tidal volumes was taken and none of them were less than 390 ml. What is the next best step for
the management of this patient?

Choices:
1. Go ahead and extubate as the patient had a positive cuff leak test
2. Place patient back on full support and call otolaryngology for evaluation
3. The patient will most likely need a tracheostomy placement
4. Give a stat dose of methylprednisolone IV 20 mg now and then every 4 hourly for 4 total
doses before re-assessing cuff leak test
Answer: 4 - Give a stat dose of methylprednisolone IV 20 mg now and then every 4 hourly
for 4 total doses before re-assessing cuff leak test

Explanations:
The cuff leak test is used to predict airway laryngeal edema and post-extubation stridor. It is
not recommended in every patient but should be used in selected patients who are at risk.
This group includes patients with difficult intubation, airway trauma, and prolonged
intubation. Edema is most common in female patients with low BMI who may have a
narrow airway in the first place.
Cuff leak can be assessed qualitatively by listening to the leak while the cuff is deflated. It
can also be done quantitatively by placing the patient on volume control ventilation and
calculating the difference between inspired and expired volume while the cuff is deflated. If
the difference is less than 110 ml or less than 20% of delivered volume, it suggests that cuff
leak is absent.
The best strategy for absent cuff leak is to initiate steroid therapy. 2 common regimen
includes one dose of 40 mg methylprednisolone 4 hours before extubation or 20 mg every 4
hourly for 4 doses.
If despite a course of corticosteroids, cuff leak is still absent, the patient should be extubated
over an airway exchange catheter which can facilitate re-intubation or extubation should be
done in the operating room with otolaryngology consultation.

Go to the next page if you knew the correct answer, or click the link image(s) below to further
research the concepts in this question (if desired).

Research Concepts:
Extubation

We update eBooks quarterly and Apps daily based on user feedback. Please tap flag to
report any questions that need improvement.
Question 249: A 16-year-old female has been transferred following a coral snake bite.
Upon arrival, a dose of North American coral snake antivenin was initiated. The patient is
currently on the third out of five vials but is now complaining of a red, itchy rash and severe
shortness of breath. What is the next best step in management of this patient?

Choices:
1. Continue antivenom infusion until completion, he will feel better afterward
2. Continue antivenom infusion, but order oral antihistamines
3. Stop the infusion, order IM epinephrine, IV corticosteroids, IV antihistamines, IV fluids, and
nebulized albuterol.
4. Stop the infusion and observe the patient
Answer: 3 - Stop the infusion, order IM epinephrine, IV corticosteroids, IV antihistamines,
IV fluids, and nebulized albuterol.

Explanations:
In patients exhibiting hypersensitivity reactions, infusion of antivenin should be
discontinued immediately, not paused or continued to completion.
In patients exhibiting hypersensitivity reactions treatment with epinephrine, steroids,
antihistamines, and albuterol should be initiated at the first sign of allergic reaction without
delay.
North American coral snake antivenin (NACSA) is an equine-derived IgG medication that
commonly causes mild acute allergic reactions and delayed serum sickness. Severe acute
allergic reactions and anaphylaxis is uncommon but possible.
Any delay in treatment of hypersensitivity reactions is inappropriate as they are acute
allergic reactions and patients may quickly deteriorate, requiring additional respiratory or
cardiovascular supportive care.

Go to the next page if you knew the correct answer, or click the link image(s) below to further
research the concepts in this question (if desired).

Research Concepts:
Coral Snake Toxicity

We update eBooks quarterly and Apps daily based on user feedback. Please tap flag to
report any questions that need improvement.
Question 250: A 35-year-old male is being evaluated for shortness of breath and chest pain.
He is found to have a pulmonary embolism and started on anticoagulation. In his nine months'
follow up appointment, he is found to have signs of right heart failure and pulmonary
hypertension on echocardiogram. Right heart catheterization confirms pulmonary hypertension.
His clot burden is unchanged on pulmonary angiography. Which of the following is the next best
step in the management of this patient?

Choices:
1. Initiate a different anticoagulant
2. Surgery-pulmonary thromboendarterectomy
3. Admission to the hospital for intravenous anticoagulation such as unfractionated heparin or
subcutaneous low molecular weight heparin
4. Double-lung transplantation
Answer: 2 - Surgery-pulmonary thromboendarterectomy
Explanations:
Surgery is the only definitive therapy for chronic thromboembolic pulmonary hypertension.
The evaluation for surgery should occur early in the disease course for chronic
thromboembolic pulmonary hypertension, even if the symptoms or hemodynamic
abnormalities are mild because early surgery may prevent the development of an
irreversible lung and heart disease.
Medical therapy is not curative, and its effects are relatively modest. There are no studies
that compare anticoagulation.
Double-lung transplantation is an alternative surgical option for patients who are not
candidates for pulmonary thromboendarterectomy.

Go to the next page if you knew the correct answer, or click the link image(s) below to further
research the concepts in this question (if desired).

Research Concepts:
Chronic Thrombo-Embolic Pulmonary Hypertension

We update eBooks quarterly and Apps daily based on user feedback. Please tap flag to
report any questions that need improvement.
Question 251: A 60-year-old patient is admitted to the intensive care unit after thoracic
surgery. They are intubated with a left-sided double lumen tube. There is a concern for tube
migration during routine positioning of the patient. The vital signs are stable and patients oxygen
saturation decreased from 98 to 94. What's the next step in management?

Choices:
1. Send patient for CT scan
2. Extubate the patient
3. Confirm tube position with a flexible bronchoscopy at the bedside
4. Change to right-sided double lumen tube
Answer: 3 - Confirm tube position with a flexible bronchoscopy at the bedside
Explanations:
Tube malposition can be life-threatening due to compromise in oxygenation and ventilation.
Whenever malposition is suspected, tube position should be reconfirmed with auscultation,
Flexible bronchoscopy is the most important means to confirm the position of a double
lumen tube (DLT).
Patient with suspected endotracheal tube malposition should not be transported before
confirming tube position. Transporting a critically ill patient carries risks of significant
hemodynamic and respiratory compromise.
Left-sided double lumen tube has a better "margin of error " compared to right-sided tube
due to right upper lobe origin and shorter length of the right main bronchus. Thus right side
DLT are more prone to malposition.
With an unstable patient, priority is to maintain vitals and ensure oxygenation and
ventilation. Treating hypoxia hypercarbia during ventilation with DLT involves confirming
tube position, optimizing ventilator settings of tidal volume, respiratory rate, inspired
oxygen concentration FiO2 and positive end-expiratory pressure PEEP, and the ratio of
inspiratory to expiratory time; I:E ratio.

Go to the next page if you knew the correct answer, or click the link image(s) below to further
research the concepts in this question (if desired).

Research Concepts:
Double Lumen Endobronchial Tubes

We update eBooks quarterly and Apps daily based on user feedback. Please tap flag to
report any questions that need improvement.
Question 252: A 28-year-old male with no prior history is admitted to the intensive care
unit for acute hypoxic respiratory failure. He has no prior medical problems but is a current
smoker. He had recently started smoking a month back due to anxiety issues. His wife at the
bedside denied any use of medications recently or illicit substances. She reports that he was well
until about 2 days ago when he started feeling short of breath on walking, and today he could
barely move without getting short of breath and gasping for air. When he arrived in the
emergency department, he was found to have pulse oximetry of 65% on room air with a
respiratory rate of 35/minute, so he was intubated. He is currently intubated on assist control
setting of tidal volume 400, rate of 22, FiO2 of 80% and PEEP of 14. He is sedated and
synchronous with the ventilator. On examination, he has bilateral crackles, and his plateau
pressure on the ventilator is 28. S1 and S2 are heard, no murmurs or rubs, the abdomen is soft,
nontender with normal bowel sounds, extremities are warm with good peripheral perfusion, no
rashes are seen. Chest x-ray that shows bilateral opacities occupying most of the lung fields
bilaterally, no lobar consolidation or pleural effusions are seen. CBC shows an elevated WBC
count of 14.3 with 83% neutrophils. Electrolytes, renal function, and liver function tests are all
within normal limits. A urine drug screen was done which was negative. He is started on
ceftriaxone, azithromycin, pantoprazole, and enoxaparin. A bronchoalveolar lavage is done
which shows RBC 176, WBC 652 with 35% eosinophils, 43% neutrophils and 22%
macrophages. The gram stain is negative for organisms. What would be the best next step in
management?

Choices:
1. Intravenous methylprednisolone
2. Chest CT without contrast
3. Upgrade antibiotics to vancomycin and cefepime
4. Consult thoracic surgery for a VATS guided lung biopsy
Answer: 1 - Intravenous methylprednisolone
Explanations:
Idiopathic acute eosinophilic pneumonia is a rare disease, predominantly occurring in young
male smokers
There is a temporal association with cigarette smoking.
Most patients with acute eosinophilic pneumonia have a severe hypoxemic respiratory
failure, with at least 70% patients needing mechanical ventilation.
Intravenous corticosteroids are the mainstay of therapy with a clinical response seen as
early as 12 hours, and almost always within 48 hours.

Go to the next page if you knew the correct answer, or click the link image(s) below to further
research the concepts in this question (if desired).

Research Concepts:
Pulmonary Eosinophilia

We update eBooks quarterly and Apps daily based on user feedback. Please tap flag to
report any questions that need improvement.
Question 253: Two days after abdominal surgery, a 66-year-old female patient develops
left side chest pain, dyspnea, and ECG changes in the anterolateral leads. Myocardial infarction
is confirmed by elevations of cardiac biomarkers. The patient's blood pressure is 85/45 mm Hg,
pulse 125 bpm, respiratory rate 20/min, and she is afebrile. Pulse oximetry at room air is 94%.
Which of the following treatment is not recommended?

Choices:
1. Thrombolytic therapy
2. Aspirin
3. Intra-aortic balloon pump
4. Coronary artery bypass surgery
Answer: 1 - Thrombolytic therapy
Explanations:
Postoperative myocardial infarction carries a grave prognosis with mortality exceeding
50%. Postoperative myocardial infarction is a well-known phenomenon, and it usually
occurs within 48 hours of surgery.
Postoperative myocardial infarcts tend to be silent in most cases and may be identified by
ECG and blood work.
If available these patients should undergo urgent coronary reperfusion with either medical
therapy or coronary angioplasty. However, timely reperfusion is critical. Cardiogenic shock
is a life-threatening condition that can occur secondary to massive myocardial infarction.
Using a mechanical counterpulsation strategy, a device called the Intra-aortic balloon pump
(IABP) could be effective in augmenting pressure and flow.
Thrombolytic therapy is usually contraindicated in patients who have had recent surgery
and an intervention-based procedure may be the best choice. If a fresh surgical patient is
given thrombolytic drugs, there is higher mortality associated with it.

Go to the next page if you knew the correct answer, or click the link image(s) below to further
research the concepts in this question (if desired).

Research Concepts:
Myocardial Infarction

We update eBooks quarterly and Apps daily based on user feedback. Please tap flag to
report any questions that need improvement.
Question 254: A 75-year-old female with a 50-pack-year history of tobacco use presents
with a 2-week history of worsening dyspnea on exertion. She has had a dry cough but no fever,
night sweats, or chills. Her neck shows no jugular venous distention. Heart tones are distant but
regular without murmurs. A lung exam reveals right lower lung dullness to percussion and
decreased breath sounds. A chest radiograph shows a pleural effusion but no infiltrates or
lymphadenopathy. Laboratories are normal. What is the appropriate management?

Choices:
1. Broad-spectrum antibiotics
2. CT of the chest
3. Bronchoscopy
4. Thoracentesis
Answer: 4 - Thoracentesis
Explanations:
There is no evidence of infection, so antibiotics would not be appropriate.
CT of the chest may have a small yield, but thoracentesis will provide the most rapid
diagnosis.
Sampling the pleural fluid by thoracentesis will determine if the fluid is exudate or
transudate in nature and aids in diagnosis.
Thoracentesis also will provide symptomatic relief.

Go to the next page if you knew the correct answer, or click the link image(s) below to further
research the concepts in this question (if desired).

Research Concepts:
Pleural Effusion

We update eBooks quarterly and Apps daily based on user feedback. Please tap flag to
report any questions that need improvement.
Question 255: A 36-year-old female with a past medical history of type 1 diabetes mellitus
and end-stage renal disease on hemodialysis presented with complaints of fever, chills, nausea,
and vomiting. On examination, her temperature was 101F and heart rate 120 beats/min. The
patient appeared to be in mild distress, and there was tenderness to palpation of the right chest
wall permacath. An empiric antibiotic regimen was started for a presumed line infection. Blood
cultures were positive for Staphylococcus aureus. On day 6 of antibiotic therapy, the patient
remains febrile. A contrast-enhanced CT shows a 5 cm right perinephric abscess. What is the
next step in the treatment of this patient?

Choices:
1. Continue current antibiotic regimen
2. Percutaneous catheter drainage
3. Surgical drainage
4. Nephrectomy
Answer: 2 - Percutaneous catheter drainage
Explanations:
Perinephric abscesses greater than 3 mm should be drained.
Abscesses less than 3 mm should initially be treated with antibiotics.
Initially, percutaneous catheter drainage should be done for abscesses greater than 3 mm.
Surgical intervention might be indicated when percutaneous drainage is unsuccessful.

Go to the next page if you knew the correct answer, or click the link image(s) below to further
research the concepts in this question (if desired).

Research Concepts:
Perinephric Abscess

We update eBooks quarterly and Apps daily based on user feedback. Please tap flag to
report any questions that need improvement.
Question 256: A 74-year-old diabetic male presents to the emergency department for left-
sided chest pain. His past medical history is significant for type 2 diabetes, major depressive
disorder, and congestive heart failure. A bedside EKG is performed and shows a left bundle
branch block. The patient takes doxepin for his depression. Which of the following is the next
best step in the management of this patient?

Choices:
1. Advise patient to continue current antidepressant medication
2. Advise patient to stop his antidepressant medication
3. Increase the dosage of doxepin
4. Switch to another class of antidepressant
Answer: 4 - Switch to another class of antidepressant
Explanations:
Tricyclics have serious side effects such as cardiotoxicity, conduction abnormalities,
orthostatic hypotension, syncope, dysrhythmias, bradycardia, and tachycardia. Thorough
history taking including ruling out coronary artery disease risk factors should be gathered
from a patient before prescribing an antidepressant. This patient has risk factors such as
diabetes and a history of heart failure. It is recommended that the patient stay away from
tricyclics as they can worsen his symptoms.
Tricyclic antidepressants have serious adverse effects especially in those patients who have
cardiovascular risk factors. It has been documented that TCAs can cause conduction
abnormalities, heart blocks, as well as PR, QRS, and QT interval changes.
It is advised to stay away from these types of medications in patients who have risks factors
or coronary artery disease.
Other antidepressants such as SSRIs are less potent and toxic. Risks and benefits must be
assessed as SSRI can still cause dysrhythmias like atrial fibrillation, bradycardia or syncope.

Go to the next page if you knew the correct answer, or click the link image(s) below to further
research the concepts in this question (if desired).

Research Concepts:
Doxepin

We update eBooks quarterly and Apps daily based on user feedback. Please tap flag to
report any questions that need improvement.
Question 257: A 17-year-old patient presented to the emergency department following a
crush injury to their thorax and abdomen. They undergo a CT scan as part of their trauma
workup. The radiologist is concerned that the patient has a diaphragmatic hernia. Which of the
following symptoms is most likely?

Choices:
1. Constipation
2. Shortness of breath
3. Shoulder tip pain
4. Visual disturbance
Answer: 2 - Shortness of breath
Explanations:
A diaphragmatic hernia can result in increased thoracic pressure which can lead to
respiratory symptoms, including shortness of breath.
Shoulder tip pain could potentially occur as this result from diaphragmatic irritation but in
the case of crushing thoracic and abdominal trauma, this would likely to not be noticed by
the patient.
It is important to note that shortness of breath is not a specific sign of a diaphragmatic
hernia especially in a trauma case, and there are more common causes that need to be
explored and considered first. In this case, there is CT evidence of diaphragmatic herniation.
Additional signs to look for include tracheal deviation and reduced chest expansion on the
side of the herniation.

Go to the next page if you knew the correct answer, or click the link image(s) below to further
research the concepts in this question (if desired).

Research Concepts:
Diaphragmatic Hernia

We update eBooks quarterly and Apps daily based on user feedback. Please tap flag to
report any questions that need improvement.
Question 258: A 57-year-old female was admitted to the medical intensive care unit
(MICU) for septic shock and acute respiratory failure requiring endotracheal intubation
secondary to multilobar pneumonia. She was started on appropriate therapy; however her MICU
course was complicated with the development of intermittent rapid, narrow complex tachycardia
episodes that usually resolve with vagal maneuvers; however, it is not refractory. On physical
exam, vitals are blood pressure 110/75 mmHg, heart rate 180 bpm, respiratory rate 18/minute,
and oxygen saturation 90% on FiO2 of 70%. Her only access is a right internal jugular triple
lumen central line in which she is administered intravenous fluids and antibiotics. Which of the
following is the next step to treat the patient's tachycardia?

Choices:
1. Start patient on esmolol drip starting at 50 mcg/kg/min and titrate accordingly
2. Administer adenosine at 6 mg IV bolus followed by 20 mL of a saline flush
3. Administer adenosine at 3 mg IV bolus followed by 20 mL of a saline flush
4. Load patient with digoxin at 10 mcg/kg IV over the next 24 hours
Answer: 3 - Administer adenosine at 3 mg IV bolus followed by 20 mL of a saline flush
Explanations:
ACC/AHA/HRS guidelines state that first-line therapy for supraventricular tachycardia
(SVT) in an adult is vagal maneuvers or adenosine.
Adenosine should be administered at a lower dose when given through central access,
starting at a dose of 3 mg IV bolus. Other causes to decrease the initial dose to 3 mg are in
cardiac transplant patients or patients on dipyridamole or carbamazepine drugs.
Any patient receiving adenosine should be on a form of cardiac monitoring. Patients treated
for SVT are often on a 12-lead electrocardiogram rhythm monitoring to assess the
underlying rhythm while adenosine is actively affecting the atrioventricular (AV) node.
Adenosine can be further classified as a miscellaneous antiarrhythmic drug outside the
Vaughan-Williams classification scheme. It acts on receptors in the cardiac AV node,
significantly reducing conduction time through hyperpolarization.

Go to the next page if you knew the correct answer, or click the link image(s) below to further
research the concepts in this question (if desired).

Research Concepts:
Adenosine

We update eBooks quarterly and Apps daily based on user feedback. Please tap flag to
report any questions that need improvement.
Question 259: A multiplace chamber facility is contacted by another medical facility 10
miles away via a major highway. The contacting practitioner states that they have a patient who
has just finished an intraoperative debridement for a necrotizing fasciitis infection. They want to
send the patient for adjuvant hyperbaric oxygen therapy. She originally presented with
tachypnea, developed septic shock, and acute respiratory failure. The patient has a medical
history of uncontrolled diabetes mellitus, hypertension, hyperlipidemia, and congestive heart
failure with a most recent ejection fraction of 25%. She is currently intubated, and her vitals
show temperature 38.3 C (101 F), heart rate 110 beats/min, respiratory rate 20/minute, blood
pressure 100/60 mmHg, and oxygen saturation 98% on a ventilator. Is this patient appropriate for
treatment?

Choices:
1. No, the patient is too unstable for treatment.
2. No, the patient has an ejection fraction under 35%, which is a contraindication for treatment.
3. Yes, treat the patient daily at 3 ATA for up to 30 treatments.
4. Yes, treat the patient initially twice a day until the spread of infection has ceased.
Answer: 4 - Yes, treat the patient initially twice a day until the spread of infection has
ceased.

Explanations:
In necrotizing fasciitis, it is recommended to treat twice a day. Most patients will only need
seven to 10 treatments for the spread of infection to cease.
The patient's ejection fraction is lower than 35%, but this is only a relative contraindication,
not an absolute contraindication.
This patient does have an indicated diagnosis, does not have any absolute contraindications,
and a multiplace facility has the capabilities for providing critical care during treatment if
necessary.
Hyperbaric oxygen therapy slows the progression of necrosis in these patients. Also, it
augments the efficacy of antimicrobials, demarcating viable tissue from nonviable tissue
and enhancing limb salvage.

Go to the next page if you knew the correct answer, or click the link image(s) below to further
research the concepts in this question (if desired).

Research Concepts:
Necrotizing Fasciitis

We update eBooks quarterly and Apps daily based on user feedback. Please tap flag to
report any questions that need improvement.
Question 260: A 56-year-old female presents to the hospital with shortness of breath. She
has a past medical history of hypertension, hyperlipidemia, type 2 diabetes mellitus, and COPD.
She also notes nausea but denies vomiting, chest pain, or abdominal pain. Physical examination
shows a distressed female with deep, prolonged respirations; however, lungs are clear to
auscultation. The abdomen is non-tender and non-distended. Vital signs show a heart rate of
120/min, respiratory rate 28/min, blood pressure 120/70 mmHg, temperature 37.2 C, and Sp02
99% on room air. Lab work reveals pH 7.1, pCO2 13 mmHg, pO2 90 mmHg, HCO3 less than 6
mEq/L, sodium 130 mEq/L, potassium 4.0 mEq/L, chloride 100 mEq/L, creatinine 1.3 mg/dL,
BUN 24 mg/dL, and glucose 200 mg/dL. Urinalysis shows 3+ ketones. Which of the following
medications most likely caused this patient's symptoms?

Choices:
1. Metformin
2. Glyburide
3. Insulin
4. Canagliflozin
Answer: 4 - Canagliflozin
Explanations:
Canagliflozin is an SGLT-2 inhibitor that has been shown to be associated with DKA with
euglycemia.
SGLT-2 inhibitors affect the proximal convoluted tubule of the nephron to prevent
reabsorption of glucose.
Over time, this can result in severe dehydration and acidosis, while maintaining euglycemia.
Metformin can cause severe lactic acidosis, however, this patient is in DKA, which is not
precipitated by metformin. Glyburide can cause hypoglycemia but is unlikely to result in
severe acidosis. Insulin can result in hypoglycemia but is unlikely to cause severe acidosis.

Go to the next page if you knew the correct answer, or click the link image(s) below to further
research the concepts in this question (if desired).

Research Concepts:
Euglycemic Diabetic Ketoacidosis

We update eBooks quarterly and Apps daily based on user feedback. Please tap flag to
report any questions that need improvement.
Question 261: A 56-year-old female is referred to the clinic from her gynecologist where
she had gone to get cervical cancer screening. During the office visit there, she fell asleep mid-
conversation. Given her body habitus, the gynecologist was concerned for obstructive sleep
apnea and made the referral. Blood work done at the OBGYN visit is significant for elevated
serum bicarbonate at 32 mEq/L. She denies smoking. There is some concern for obstructive
sleep apnea (OSA) and obesity hypoventilation syndrome (OHS). Which of the following is most
accurate about OHS?

Choices:
1. An alternative neuromuscular, mechanical or metabolic explanation for hypoventilation can be
coexistent
2. Presence of awake alveolar hypoventilation characterized by daytime hypercapnia (arterial
PCO2 greater than 45 mmHg)
3. A consequence of diminished ventilatory drive and capacity related to individual patient being
overweight (BMI greater than 25)
4. Presence of awake hypoxia characterized by arterial PO2 less than 65 mmHg
Answer: 2 - Presence of awake alveolar hypoventilation characterized by daytime
hypercapnia (arterial PCO2 greater than 45 mmHg)

Explanations:
OHS is defined as the presence of awake alveolar hypoventilation characterized by daytime
hypercapnia (arterial PCO2 greater than 45 mm Hg [5.9 kPa]) that is thought to be a
consequence of diminished ventilatory drive and capacity related to obesity (BMI greater
than 30) in the absence of an alternative neuromuscular, mechanical or metabolic
explanation for hypoventilation.
Daytime Hypercapnia is the hallmark of OHS, differentiating it from OSA alone. Although
OHS/Pickwickian syndrome could have a component of hypoxia coexistent in the more
severe cases, it is not required for defining OHS.
Other causes of hypoventilation have to be ruled out before making the final diagnosis.
Obesity is required at a range of BMI greater than 30.

Go to the next page if you knew the correct answer, or click the link image(s) below to further
research the concepts in this question (if desired).

Research Concepts:
Pickwickian Syndrome

We update eBooks quarterly and Apps daily based on user feedback. Please tap flag to
report any questions that need improvement.
Question 262: A 56-year-old female who had a total abdominal hysterectomy via
laparoscopy for fibroids and heavy vaginal bleeding is being evaluated on the second post-op
day. Her pain is well-controlled, and she is eating a normal diet. Her major complaint is bilateral
ankle swelling, stating, "my socks are too tight." What is the most likely cause of this symptom?

Choices:
1. Excessive IV fluids in the operating room
2. Physiologic vasopressin surge
3. Deep vein thrombosis
4. Body dysmorphic syndrome
Answer: 2 - Physiologic vasopressin surge
Explanations:
Vasopressin (or antidiuretic hormone) is secreted by the body in response to stress or shock.
Increase of vasopressin is a well-known phenomenon after surgical procedures.
It can cause water retention in postoperative patients and can explain the mild lower
extremity swelling in this patient.
While it is possible this patient got more fluid than necessary in the operating room, this is
not the best answer available. Signs and symptoms of a DVT include unilateral lower
extremity swelling, redness, and pain, especially with dorsiflexion of the foot. Because this
swelling is bilateral, a DVT is not the highest differential diagnosis.

Go to the next page if you knew the correct answer, or click the link image(s) below to further
research the concepts in this question (if desired).

Research Concepts:
Insensible Fluid Loss

We update eBooks quarterly and Apps daily based on user feedback. Please tap flag to
report any questions that need improvement.
Question 263: A 45-year-old male patient from Guatemala presents to the emergency
department with a complaint with "coughing up blood." A chest x-ray was performed that shows
what seems like left upper lobe cavitary lesions with associated hilar lymphadenopathy. While
reevaluating the patient, he suddenly starts coughing up large quantities of bright red blood. The
patient was placed in the left lateral decubitus position and Trendelenburg position, but the
patient is still having large amounts of hemoptysis. Oxygen saturation for the patient continues to
drop, and the patient becomes lethargic. The decision is made to intubate the patient. The patient
is intubated and currently hemodynamically stable. The pulmonologist is at bedside ready for
bronchoscopy and hospital transport is also at bedside ready to take the patient to CT scan. The
surgical team is also on their way for evaluation of the patient. What is the best plan of action?

Choices:
1. Wait for surgery to evaluate the patient for possible emergent lobectomy
2. Take the patient to CT before performing bronchoscopy
3. Perform bronchoscopy for definitive care before performing CT
4. Wait for all consultants to be at the bedside for further discussion
Answer: 2 - Take the patient to CT before performing bronchoscopy
Explanations:
In a stable patient such as this who is already intubated, CT has demonstrated equal ability
as bronchoscopy for localization of the source of bleeding.
CT scan is able to determine the cause of the bleeding more often than bronchoscopy and
allows for guidance of embolization.
Surgical lobectomy is considered last line treatment given the high rates of mortality and is
significantly safer when performed non-emergently.
Bronchoscopy has the advantage of being able to be performed at the bedside without
transportation of the patient and allows for pulmonary isolation techniques if hemostasis has
not been achieved.

Go to the next page if you knew the correct answer, or click the link image(s) below to further
research the concepts in this question (if desired).

Research Concepts:
Pulmonary Hemorrhage

We update eBooks quarterly and Apps daily based on user feedback. Please tap flag to
report any questions that need improvement.
Question 264: A 76-year-old patient with past medical history significant for heart failure
with reduced ejection fraction (HFrEF) with 30% to 35%, chronic obstructive pulmonary
disease, and iron deficiency anemia was admitted for acute dyspnea on exertion with wheezing.
CBC revealed hemoglobin of 6.2 g/dL and hematocrit of 20%. A packed red blood cell
transfusion was initiated. 7 hours later, the patient develops worsening of dyspnea with oxygen
saturation of 72% on room air. Body temperature is reported as 101.2 F. Lung sounds show rales
bilaterally. Neck vein is distended, and the patient developed hypertension. CBC showed WBC
of 10.1 and platelet count of 237/microliter. What is the most likely diagnosis?

Choices:
1. Transfusion associated circulatory overload (TACO)
2. Hemolytic transfusion reaction
3. Acute exacerbation of chronic obstructive pulmonary disease
4. Transfusion related acute lung injury (TRALI)
Answer: 1 - Transfusion associated circulatory overload (TACO)
Explanations:
TACO and TRALI both can have rales, but TACO has neck distended vein due to
circulatory overload and develops hypertension.
TRALI develops hypotension and fever.
Patients with systolic dysfunction are more vulnerable to TACO than TRALI with
cardiogenic pulmonary edema.
TRALI likely to show leukopenia and thrombocytopenia.

Go to the next page if you knew the correct answer, or click the link image(s) below to further
research the concepts in this question (if desired).

Research Concepts:
Transfusion-related Acute Lung Injury

We update eBooks quarterly and Apps daily based on user feedback. Please tap flag to
report any questions that need improvement.
Question 265: A 46-year-old female with a past medical history of cirrhosis of liver
presents with shortness of breath and bilateral pedal edema. On examination, JVD is raised,
spider angioma and gynecomastia are present. A chest x-ray is normal, and echocardiography
reveals pulmonary artery pressure of 40 mmHg and a dilated right ventricle. Which of the
following is the next best step in the management of this patient?

Choices:
1. Computed tomogram of chest
2. Diuretics and discharge
3. Cardiac catheterization
4. Stress test
Answer: 3 - Cardiac catheterization
Explanations:
Patients with cirrhosis of the liver are prone to arteriovenous malformation.
Since chest x-ray is normal, pleural effusion and hepatopulmonary syndrome are ruled out.
Echo is suggestive of pulmonary artery hypertension and right ventricular volume overload.
Cardiac catheterization is diagnostic to rule out coronary cameral fistula formation or
interatrial shunt formation due to hyperestrogenic state due to liver cirrhosis.

Go to the next page if you knew the correct answer, or click the link image(s) below to further
research the concepts in this question (if desired).

Research Concepts:
Coronary Cameral Fistula

We update eBooks quarterly and Apps daily based on user feedback. Please tap flag to
report any questions that need improvement.
Question 266: A patient has suffered an out-of-hospital cardiac arrest. Upon arrival, the
patient is hemodynamically stable. On evaluation, the patient has a regular rate and rhythm.
Coarse breath sounds are heard throughout both lungs; the patient is intubated and mechanically
ventilated. The patient currently has a Glasgow coma scale score (GCS) of 3T, is areflexic, does
not arouse, and is currently on propofol and fentanyl for sedation. The sedatives are temporarily
held, with no change in his neurologic exam. The abdomen is benign. The patient's pupils are
pinpoint and non-reactive. What can be determined about the possible use of targeted
temperature management or therapeutic hypothermia in this patient?

Choices:
1. Therapeutic hypothermia is indicated immediately due to the neurologic examination
2. Targeted temperature management is indicated immediately due to the neurologic exam
3. Neither temperature option is initially indicated with these examination findings
4. The patient has progressed to brain death and further care is futile
Answer: 2 - Targeted temperature management is indicated immediately due to the
neurologic exam

Explanations:
Targeted temperature management is the process of maintaining the temperature at or below
36 degrees Celsius, whereas therapeutic hypothermia involves maintaining the temperature
within the range of 32 to 34 degrees Celsius.
Historically, the suggested temperature range was 32 to 34 degrees Celsius; however, newer
trials suggest similar benefits at 36 degrees compared to 33 degrees giving rise to targeted
temperature management.
Cessation of sedation and neurologic evaluation, if stable, should be performed to assess for
anoxic brain injury.
It is not possible to determine brain death at this time due to sedation as well as the
timeframe of illness. After cessation of sedation, an accurate neurologic examination should
be obtained, and the patient may require temperature guidelines. It is recommended that, in
addition to daily assessments, reapproaching this possibility at 72 hours is more appropriate.

Go to the next page if you knew the correct answer, or click the link image(s) below to further
research the concepts in this question (if desired).

Research Concepts:
Anoxic Encephalopathy

We update eBooks quarterly and Apps daily based on user feedback. Please tap flag to
report any questions that need improvement.
Question 267: A 17-year-old male patient with a history of intravenous drug use complains
of a few days history of pain in his left arm, double vision, fever, and difficulty swallowing. He
is awake, alert, and oriented to person, place and time, and denies any headache or neck stiffness.
Exam shows a blood pressure of 110/81 mmHg, a temperature of 37.4 C (99.3 F), heart rate of
110 beats/min, and respiratory rate of 18. Kernig and Brudzinski tests are negative. A large
erythematous, warm, and indurated rash on the left arm is noted with several track marks and
pustular lesions. Cranial nerves II through XII are grossly intact, and the bladder is distended.
Which of the following is the next best step in the management of this patient?

Choices:
1. CT head, antibiotics, admit to intensive care unit (ICU)
2. CT head, lumbar puncture, antibiotics, admit to floor
3. Antitoxin, antibiotics, admit to ICU
4. Antitoxin, antibiotics, admit to floor
Answer: 3 - Antitoxin, antibiotics, admit to ICU
Explanations:
The description is classic for wound botulism, most likely from Clostridium botulinum
cellulitis secondary to subcutaneous injection of spores from contaminated drugs.
Respiratory failure may occur suddenly, thus the admission to intensive care.
Botulism can be acquired through exposure to the pre-formed toxin via improperly-stored
food, iatrogenic injection, or bioterrorism, or it can result from the systemic release of the
toxin in vivo, as in the cases of infant and wound botulism.
Antitoxin should be administered as soon as botulism is suspected. Antibiotics for wound
botulism include penicillin or metronidazole if penicillin-allergic.

Go to the next page if you knew the correct answer, or click the link image(s) below to further
research the concepts in this question (if desired).

Research Concepts:
Botulism

We update eBooks quarterly and Apps daily based on user feedback. Please tap flag to
report any questions that need improvement.
Question 268: A 61-year-old male with a history of kidney transplantation on
immunosuppressant medications presents to the clinic for a 6-month follow-up after he had an
episode of acute hepatitis E infection. He denies having any symptoms. His vitals are normal.
BMI 23. Labs are significant for ALT 140 IU/L, AST 127 IU/L with normal serum bilirubin and
ALP. Serum creatinine and GFR are at baseline. He denies any changes to his medications.
Hepatitis A, B, and C serology are negative. CMV serology and PCR are negative. Hepatitis E
PCR is positive. RUQ ultrasound is within normal limits. Liver biopsy shows diffuse
inflammation in the portal and periportal areas. His immunosuppressant medications doses were
decreased, however, without success in clearing his viremia. What is the best next step in the
management of this patient?

Choices:
1. Orlistat
2. Ursodiol
3. Observation
4. Ribavirin
Answer: 4 - Ribavirin
Explanations:
Patients with a history of solid organ transplantation on immunosuppressant medications
who develop acute hepatitis E infection have a high risk of progression to chronic hepatitis
E.
This patient’s liver biopsy is consistent with chronic viral hepatitis, and his hepatitis E
viremia is consistent with chronic hepatitis E virus. Decreasing the doses of
immunosuppressant medications has been shown to clear the virus in about 30% of patients;
however, it was unsuccessful in this case.
Ribavirin and pegylated interferon have shown to be effective in treating chronic hepatitis E
infection and should be considered in this patient.
Orlistat is used for weight loss and is not indicated in this patient. Ursodiol is used in
patients with primary biliary cholangitis; however, it is not indicated in this patient. This
patient can benefit from ribavirin to clear his chronic infection and should be considered.

Go to the next page if you knew the correct answer, or click the link image(s) below to further
research the concepts in this question (if desired).

Research Concepts:
Hepatitis E

We update eBooks quarterly and Apps daily based on user feedback. Please tap flag to
report any questions that need improvement.
Question 269: A 16-year-old male presents to the emergency department with difficulty
breathing. His symptoms and clinical presentation are consistent with status asthmaticus. WIthin
3 hours, he rapidly deteriorates with evidence fo fatigue, altered mental status and combined
respiratory and metabolic acidosis. He was therefore intubated and placed on mechanical
ventilation. Initial ventilator settings had to be adjusted due to asynchrony on ventilator and
hypotension from intrinsic PEEP. He is still diffusely bronchospastic and is getting albuterol via
a metered dose inhaler attached to the inspiratory limb of vent circuit. He is on assist-control
volume-control mode ventilation. Which of the following is most accurate regarding his ongoing
treatment strategy?

Choices:
1. Ventilation mode should be changed to pressure regulated volume control or VC+
2. Ongoing method of albuterol administration is appropriate as MDI has a better effect on
inspiratory flow-resistive pressure
3. Albuterol administration should always be via a nebulizer in a ventilated patient
4. There is no definitive appropriate strategy of albuterol administration but an assessment of
airway peak to pause pressure gradient can decide the appropriate therapy in an individual
patient
Answer: 4 - There is no definitive appropriate strategy of albuterol administration but an
assessment of airway peak to pause pressure gradient can decide the appropriate therapy in an
individual patient

Explanations:
There is an ongoing debate about the use of MDI versus nebulizers in ventilated patients.
While small volume nebulizers have been shown to have reduced aerosol percentage
delivery to lungs, MDI has been demonstrated to have a poor effect on inspiratory flow-
resistive pressure.
Assessment of airway peak to pause pressure gradient can be a rational indicator to use
when either one of the delivery modes is used. A 15% or greater decline in the gradient is
considered to be a favorable response to be aimed for.
A higher dose of the medication is needed in both MDI or nebulized delivery in an
intubated patient.

Go to the next page if you knew the correct answer, or click the link image(s) below to further
research the concepts in this question (if desired).

Research Concepts:
Status Asthmaticus

We update eBooks quarterly and Apps daily based on user feedback. Please tap flag to
report any questions that need improvement.
Question 270: A 65-year-old female presents to his healthcare provider for the evaluation
of fatigue and general malaise that has been present for the past few days. On further
questioning, she reports a recent head cold with some sinus congestion that has since improved.
She, however, claims that she is now having some mild chest discomfort and some increased
work in breathing. Her vital signs show a temperature of 99.2 F, pulse 89/min, respiratory rate
20/min, SpO2 95%, and blood pressure 118/76 mmHg. Her ECG shows a peculiar pattern of
varying levels of the height of her QRS complex. If left untreated, which of the following
combination of symptoms is most likely to occur?

Choices:
1. S1Q3T3 complex on ECG, tachycardia, inverted T waves in lateral leads
2. Hypotension, jugular venous distension, hypoxia
3. Hypertension, bradycardia, decreased respiratory rate
4. Hypotension, distended neck vein, muffled heart sounds
Answer: 4 - Hypotension, distended neck vein, muffled heart sounds
Explanations:
Hypotension, JVD and muffled heart sounds is Beck's triad which is associated with cardiac
tamponade which can result in the electrical alternans appearance on ECG
Electrical alternans is defined as alternating QRS amplitude that is seen in any or all leads
on an electrocardiogram (ECG) with no additional changes to the conduction pathways of
the heart.
This rhythm is typically associated with pericardial effusion via the “swinging heart” from
the fluid surrounding the heart. However, electrical alternans is also associated with other
pathologies including but not limited to ventricular tachycardia, Wolff-Parkinson-White
(WPW), accelerated idioventricular rhythm, and supraventricular tachycardia.
S1Q3T3 and T wave inversion may be seen in pulmonary embolism. Hypotension, JVD,
and hypoxia are more likely related to pulmonary congestion. Hypertension, bradycardia,
and decreased respiratory rate is Cushing triad, which is associated with herniation.

Go to the next page if you knew the correct answer, or click the link image(s) below to further
research the concepts in this question (if desired).

Research Concepts:
Electrical Alternans

We update eBooks quarterly and Apps daily based on user feedback. Please tap flag to
report any questions that need improvement.
Question 271: A 17 year-old-male mountain climber and his colleagues are attempting to
summit Mount Everest. After ascending to an altitude of about 5500 meters on the second day,
he complained of headaches, anorexia, nausea, and malaise. On day three of the expedition, he
developed ataxia, impaired cognition, irrational behavior and errors in reading his map. What is
the most likely diagnosis?

Choices:
1. Acute mountain sickness
2. High-altitude cerebral edema (HACE)
3. High-altitude pulmonary edema (HAPE)
4. Meningitis
Answer: 2 - High-altitude cerebral edema (HACE)
Explanations:
Acute mountain sickness (AMS) is a syndrome characterized by headaches, anorexia,
nausea, and malaise. AMS occurs when exposed to altitudes of 4000 meters, typically for 2
or more days. Headache must be present for a diagnosis of acute mountain sickness.
Treatment includes NSAIDs and supplemental oxygen. Prophylaxis is with acetazolamide.
High-altitude cerebral edema (HACE) is thought to be the end stage form of acute mountain
sickness. The hallmark symptom of HACE is ataxia, but patients may also experience
seizure, altered mental status, or other neurologic symptoms.
This patient has symptoms of acute mountain sickness and ataxia. Hence, he has HACE, not
AMS. Treatment is descent from altitude, corticosteroids, and oxygen. Acetazolamide may
also be started.
Symptoms of high-altitude pulmonary edema (HAPE) are breathlessness, chest pain,
headache, fatigue, and dizziness. It occurs at an altitude of about 2000 to 2500 meters, with
significant elevation of pulmonary artery pressure leading to capillary breakdown and
subsequent edema.

Go to the next page if you knew the correct answer, or click the link image(s) below to further
research the concepts in this question (if desired).

Research Concepts:
High Altitude Cerebral Edema

We update eBooks quarterly and Apps daily based on user feedback. Please tap flag to
report any questions that need improvement.
Question 272: A 59-year-old man is admitted for pulmonary embolism and rivaroxaban is
initiated. Three days later he has a hypotensive episode, and his hemoglobin and hematocrit drop
significantly. He is transfused with four units of packed red blood cells and given 2 liters of
normal saline and prothrombin complex concentrate (PCC) therapy. Two days later, hemoglobin
and hematocrit are within the normal range, and the patient is hemodynamically stable. What is
the most appropriate next step in management?

Choices:
1. Administer andexanet alfa to completely reverse rivaroxaban effects
2. Administer a second dose of PCC to completely reverse rivaroxaban effects
3. Initiate aspirin and monitor the patient for signs of bleeding
4. Resume rivaroxaban and monitor the patient for signs of bleeding
Answer: 4 - Resume rivaroxaban and monitor the patient for signs of bleeding
Explanations:
Andexanet alfa is a reversal agent that is an FDA-approved factor Xa inhibitor (i.e.,
rivaroxaban) in patients with life-threatening or uncontrolled bleeding.
Prothrombin complex concentrate (PCC) is a blood coagulation factor replacement product
that is FDA-approved for the reversal of vitamin K antagonists. PCC has been used off-
label to reverse the effects of direct oral anticoagulants, like rivaroxaban.
It is important to resume anticoagulation therapy as soon as it is medically appropriate to
reduce the risk of thrombotic events.
This patient has received PCC and is hemodynamically stable; therefore, he no longer needs
a reversal agent. Anticoagulation with rivaroxaban should be resumed, and the patient
should be monitored for signs of bleeding.

Go to the next page if you knew the correct answer, or click the link image(s) below to further
research the concepts in this question (if desired).

Research Concepts:
Andexanet Alfa

We update eBooks quarterly and Apps daily based on user feedback. Please tap flag to
report any questions that need improvement.
Question 273: A 22-year-old man admitted to the hospital complaining of diarrhea and
fever of 2 days duration. The patient underwent stem cell transplantation five weeks ago. He
denies blood in the stool. On physical exam, his temperature is 37.5 C (99.5 F), blood pressure
112/75 mmHg, pulse rate 107/min, and respiratory rate is 15/min. Abdominal exam reveals
tenderness more on the left lower quadrant. A CT scan shows thickened bowel wall of the
descending colon, and he is diagnosed with neutropenic enterocolitis. The patient is started on
intravenous fluid support with bowel rest. Testing for Clostridium difficile is still pending. What
is the next best step in the management of this patient?

Choices:
1. Ampicillin-sulbactam
2. Intravenous vancomycin and ceftriaxone
3. Piperacillin/ tazobactam and oral vancomycin
4. Piperacillin/ tazobactam and intravenous vancomycin
Answer: 4 - Piperacillin/ tazobactam and intravenous vancomycin
Explanations:
The patient's clinical presentation and CT findings correlate with a diagnosis of colitis,
which can be neutropenic enterocolitis or other infectious colitis. Although there is no
consensus on the treatment neutropenic enterocolitis, the treatment should cover for the
most common organisms that may seed from the gastrointestinal tract - i.e., gram-negative
bacilli including pseudomonas and anaerobic bacteria.
Piperacillin/ tazobactam plus intravenous vancomycin is the empirical treatment suggested
for NE by multiple studies.
Although there is no consensus on the best treatment protocol, any regimen should cover
gram-negative bacilli, anaerobic bacteria, and pseudomonas because who has NE is a high-
risk patient because of neutropenia. Studies also found that adding coverage for gram-
positive bacteria decreases mortality.
Ampicillin-sulbactam covers for gram-negative bacilli and anaerobic bacteria, but it doesn't
cover for pseudomonas. So without adding anti-pseudomonal agent, the treatment will be
insufficient. For the same reason, intravenous vancomycin plus ceftriaxone regimen would
bot cover for pseudomonas or anaerobic bacteria. Clostridium difficile still pending for the
patient. Treatment for C. difficile infection should not be given empirically to every patient
presenting with neutropenic enterocolitis unless highly suspected, i.e., if the patient has risk
factors for C. difficile infection which is not mentioned in the question.

Go to the next page if you knew the correct answer, or click the link image(s) below to further
research the concepts in this question (if desired).

Research Concepts:
Neutropenic Enterocolitis (Typhlitis)

We update eBooks quarterly and Apps daily based on user feedback. Please tap flag to
report any questions that need improvement.
Question 274: A 58-year-old male patient with rheumatoid arthritis and coronary artery
disease managed is admitted for coronary artery bypass graft. Induction and intubation are
uneventful. A blind attempt at transesophageal echocardiography (TEE) probe placement was
unsuccessful after 2 attempts and was successfully placed with laryngoscopic assistance on the
third attempt. The left ventricular ejection fraction (LVEF) was 50%, and no wall motion
abnormalities were noted. The patient is transferred to the intensive care unit after surgery and is
successfully extubated 2 hours later. He complains of hoarseness on postoperative day 2 but has
no chest pain, dysphagia or shortness of breath. Which of the following is the most likely
etiology for the hoarseness?

Choices:
1. Intubation
2. Coronary artery disease
3. Placement of a transesophageal echocardiography probe
4. Extubation
Answer: 3 - Placement of a transesophageal echocardiography probe
Explanations:
Airway management in patients with rheumatoid arthritis can be challenging due to limited
mandibular motion and mouth opening, a narrowed glottis and cervical instability. In this
patient, intubation was uneventful and is not likely to be the cause of the hoarseness.
Coronary artery disease is not a predisposing factor for arytenoid subluxation.
Placement of transesophageal echocardiography (TEE) probe. Major cardiac surgery
involving the use of TEE may be associated with an increased incidence of arytenoid
subluxation. TEE probes are usually inserted blindly and are only placed under direct vision
with the help of a laryngoscope or video laryngoscope when resistance to placement is
encountered.
Traumatic extubation with a fully or partially inflated endotracheal tube cuff may also lead
to the posterolateral displacement of the arytenoid leading to postoperative hoarseness. In
this patient extubation was successful and no complications were noted.

Go to the next page if you knew the correct answer, or click the link image(s) below to further
research the concepts in this question (if desired).

Research Concepts:
Arytenoid Subluxation

We update eBooks quarterly and Apps daily based on user feedback. Please tap flag to
report any questions that need improvement.
Question 275: An adult patient presents with complaints of paresthesias and occasional
nausea after having been recently started on a new drug. He is found to have hyperchloremic
metabolic acidosis. Which diuretic is the patient taking?

Choices:
1. Hydrochlorothiazide
2. Furosemide
3. Acetazolamide
4. Mannitol
Answer: 3 - Acetazolamide
Explanations:
Acetazolamide is a carbonic anhydrase inhibitor used to treat some types of glaucoma as
well as epilepsy.
It is also effective for prevention and treatment of acute altitude illness and treatment of
congestive heart failure.
It is a mild diuretic that causes hyperchloremic metabolic acidosis.
Common adverse effects of acetazolamide include paraesthesia, depression, drowsiness,
nausea, and vomiting. Less common adverse effects include Stevens-Johnson syndrome,
anaphylaxis, and blood dyscrasias.

Go to the next page if you knew the correct answer, or click the link image(s) below to further
research the concepts in this question (if desired).

Research Concepts:
Acetazolamide

We update eBooks quarterly and Apps daily based on user feedback. Please tap flag to
report any questions that need improvement.
Question 276: A 42-year-old female from Mexico presents with a 1-week history of fever,
chills, and right upper quadrant pain. She is tender in the right upper quadrant. Ultrasound shows
cholelithiasis but no thickening of the wall of the gallbladder or pericholecystic fluid. There is a
2 cm x 2 cm mass in the liver at the right lobe that is hypoechoic. She is started on cefoxitin. A
CT guided drain is placed and only yields 15 mL of fluid over the next 5 days. She has persistent
fever and leukocytosis. Repeat CT shows the abscess has not changed. What is the best next step
in management?

Choices:
1. Catheter irrigation, catheter upsizing, insertion of another drain, or a combination thereof
2. Laparoscopic surgery
3. Open surgery
4. Change of antibiotics
Answer: 1 - Catheter irrigation, catheter upsizing, insertion of another drain, or a
combination thereof

Explanations:
Persistence of the liver abscess indicates there is not adequate drainage.
Options include catheter irrigation with saline or tPA, switching out the initial catheter for a
larger catheter, insertion of another catheter, or a combination thereof.
Depending on the site of the abscess, a persistent abscess can be treated with laparoscopic
or open drainage.
Antibiotics, even adjusted for sensitivities will not be adequate.

Go to the next page if you knew the correct answer, or click the link image(s) below to further
research the concepts in this question (if desired).

Research Concepts:
Liver Abscess

We update eBooks quarterly and Apps daily based on user feedback. Please tap flag to
report any questions that need improvement.
Question 277: A 53-year-old woman who has a past medical history of atrial fibrillation
who is brought to the emergency department with dizziness, light-headedness, and blood in her
stool. She has been taking rivaroxaban 20 mg once a day for approximately one year. She
received her last dose of rivaroxaban approximately 14 hours ago. Her hemoglobin is 6.7 g/dL,
and blood pressure is 78/49 mmHg. She is given fluid boluses with normal saline, is transfused
with two units of packed red blood cells and started on a norepinephrine infusion. The provider
decides to use andexanet alfa. Which of the following is an appropriate order for andexanet alfa
for this patient?

Choices:
1. 400 mg IV; target infusion rate of 30 mg/min followed by an IV infusion of 4 mg/min IV for
up to 2 hours
2. 400 mg oral dose followed by 240 mg given orally every hour for up to 2 hours
3. 800 mg administered orally
4. 400 mg IV bolus followed by two doses of 240 mg tablets administered orally every hour
Answer: 1 - 400 mg IV; target infusion rate of 30 mg/min followed by an IV infusion of 4
mg/min IV for up to 2 hours

Explanations:
Andexanet alfa is available as a lyophilized powder in 100 mg vials. Unopened vials should
be refrigerated at 2 to 8 C. Vials are reconstituted with sterile water for injection, and the
calculated dose is administered intravenously as a bolus followed by an infusion.
Reconstituted andexanet alfa in vials is stable at room temperature for up to 8 hours and at 2
to 8 C or up to 24 hours.
Reconstituted andexanet alfa in IV bags is stable at room temperature for up to 8 hours and
2 to 8 C or up to 16 hours.
Dosing of andexanet alfa is based on the factor Xa inhibitor to be reversed, the dose last
taken, and the timing of the last dose.

Go to the next page if you knew the correct answer, or click the link image(s) below to further
research the concepts in this question (if desired).

Research Concepts:
Andexanet Alfa

We update eBooks quarterly and Apps daily based on user feedback. Please tap flag to
report any questions that need improvement.
Question 278: A 40-year-old man with no significant past medical history presents to the
emergency department with fever, malaise, and right-sided pleuritic chest pain. Two weeks
before admission, he complained of sore throat with low-grade fever but did not seek medical
attention. His chest imaging reveals a right-sided empyema, and his neck ultrasound revealed a
right internal jugular vein thrombus. Thoracentesis and cultures of the pleural fluid reveal growth
in the anaerobic culture. What is the best next step in management?

Choices:
1. Starting anticoagulant therapy for the management of his thrombus
2. Intravenous ampicillin /sulbactam for 4-6 weeks
3. Intravenous imipenem for one week
4. Oral Metronidazole for 6 weeks
Answer: 2 - Intravenous ampicillin /sulbactam for 4-6 weeks
Explanations:
Lemierre syndrome presents with fever and a brief upper respiratory illness later develops
septic embolization and distant infection, e.g., lung empyema and internal jugular vein
septic thrombophlebitis. The treatment plan is usually intravenous antibiotics targeted
toward Fusobacterium necrophorum the most common causative organism.
Fusobacterium necrophorum is usually the causative organism for Lemierre syndrome, but
due to the concern of mixed infection with other oral microbial flora, Monotherapy with
Metronidazole is not recommended.
Fusobacterium necrophorum is an uncommon cause of pharyngitis, untreated is usually
complicated by suppurative thrombophlebitis of Lemierre syndrome. Due to possible beta-
lactamase production, it is recommended to treat with a combination of a ß-lactam antibiotic
and a ß-lactamase inhibitor or carbapenem.
Fusobacterium necrophorum a common inhabitant of oral mucosa that can cause
oropharyngeal infection and result in complications of suppurative thrombophlebitis of
Lemierre syndrome and distant infections and require prolonged antibiotic treatment from
4-6 weeks.

Go to the next page if you knew the correct answer, or click the link image(s) below to further
research the concepts in this question (if desired).

Research Concepts:
Septic Emboli

We update eBooks quarterly and Apps daily based on user feedback. Please tap flag to
report any questions that need improvement.
Question 279: A 16-year-old patient underwent placement of an external ventricular drain
for the management of acute hydrocephalus secondary to tubercular meningitis. Two days later
he developed a high-grade fever associated with chills and rigor. He had neck stiffness on
clinical examination. The CSF culture showed the growth of Pseudomonas aeruginosa sensitive
to amikacin only. The treating clinician planned to start intraventricular amikacin therapy. What
is the safe daily dosage of amikacin that can be administered intraventricularly in the patient?

Choices:
1. 10 mg
2. 30 mg
3. 60 mg
4. 100 mg
Answer: 2 - 30 mg
Explanations:
Intraventricular instillation of antibiotics is a valid option in treating patients with
ventriculitis.
The dose of amikacin through the intraventricular route is 1/20th of the intravenous dose.
The average daily dose of amikacin through the intraventricular route is 30 mg. The
maximum daily dose of amikacin should not exceed 50 mg.
The dosage of amikacin in excess of 30 mg daily has shown to be associated with a high
incidence of adverse effects, including choroid plexitis and the risk of an intraventricular
bleed.

Go to the next page if you knew the correct answer, or click the link image(s) below to further
research the concepts in this question (if desired).

Research Concepts:
Ventriculitis

We update eBooks quarterly and Apps daily based on user feedback. Please tap flag to
report any questions that need improvement.
Question 280: A 28-year-old female is admitted with jaundice for 1 week with associated
confusion. Her initial labs show hemoglobin 12.1 g/dL, WBC 5400/microL, platelets
543000/microL, Alanine aminotransferase (ALT) 3200 U/L, aspartate aminotransferase (AST)
1079 U/L , bilirubin 4.3 mg/dL, alkaline phosphatase 345 U/L, INR 4.5, and creatinine 1.02
mg/dL. Blood culture and urine culture are negative, and ultrasound of the abdomen showed
normal liver echotexture with ascites. Which of the following is less likely associated with poor
prognosis in this lady with acute liver failure?

Choices:
1. Coagulopathy
2. Duration of jaundice
3. Encephalopathy
4. Ascites
Answer: 4 - Ascites
Explanations:
Coagulopathy is the most important prognostic factor in acute liver failure (ALF).
Derangement of clotting mechanism suggest poor prognosis and need for referral for liver
transplant
Duration of jaundice to hepatic encephalopathy has been related to prognosis. Subacute
acute liver failure (ALF) of less than 1 month duration from jaundice with encephalopathy
had poorer survival compared to hyperacute of duration less than 7 days and acute 1 week
to 1 month duration between jaundice to encephalopathy ALF.
Encephalopathy suggests cerebral edema and it suggests worst prognosis in ALF.
Development of ascites is rare in ALF as ascites favor underlying chronic liver disease like
cirrhosis. Ascites has no correlation to survival in ALF.

Go to the next page if you knew the correct answer, or click the link image(s) below to further
research the concepts in this question (if desired).

Research Concepts:
Hepatic Failure

We update eBooks quarterly and Apps daily based on user feedback. Please tap flag to
report any questions that need improvement.
Question 281: A critically ill 65-year-old male patient with a past medical history of
diabetes, hypertension, congestive heart failure, obstructive sleep apnea, and chronic obstructive
pulmonary disease is undergoing a pelvic fracture external reduction with internal fixation. The
patient was placed on an insulin drip to control blood sugars. Last blood sugar prior to being
taken back to the operating room was 180 mg/dL. When blood sugar was checked 2 hours into
the case, it dropped to 65 mg/dL. What is the best next step to take in this case?

Choices:
1. Stop insulin drip
2. Start Dextrose-5 solution at a rate of 100cc/ hour
3. Give Dextrose-50 and stop insulin drip
4. Do not titrate the drip and follow up blood sugar in 15 minutes
Answer: 3 - Give Dextrose-50 and stop insulin drip
Explanations:
Hypoglycemia is defined as any blood sugar less than 70 mg/dL. Severe hypoglycemia is
defined as a blood sugar less than 40 mg/dL.
Hypoglycemia is an independent risk factor for morbidity and mortality in hospitalized
patient's.
If a patient is experiencing hypoglycemia in the operating room, give the patient Dextrose-
50, stop the insulin drip, and check blood sugar every 15 minutes until blood sugar trending
upwards.
It is best practice to flush out the carrier line that is connected to the insulin drip to ensure
insulin that is stuck in the line is not inadvertently administered to the patient and further
exaggerating hypoglycemia.

Go to the next page if you knew the correct answer, or click the link image(s) below to further
research the concepts in this question (if desired).

Research Concepts:
Diabetes Intraoperative Management

We update eBooks quarterly and Apps daily based on user feedback. Please tap flag to
report any questions that need improvement.
Question 282: A 46-year-old female is rushed to the emergency department following a
motor vehicle accident in which she fell asleep at the wheel and ran her car into a light pole.
While still trapped in the wreckage, she received third-degree electrical burns from an active
wire, exposed because of the crash, over the dorsal surface of her right arm and shoulder.
Besides the burns, the patient is suffering from several broken ribs. An ultrasound-guided FAST
exam reveals one of the broken ribs punctured the right lung causing the patient to experience
respiratory distress. Vital signs are a weight of 156 pounds (71 kg), blood pressure 100/53
mmHg, heart rate 110 beats/min, respiratory rate 35, temperature 99.1F, and oxygen saturation of
75% that continues to drop. On physical exam, the patient appears obtunded, her lips show blue
discoloration, and hypertonicity of the neck muscles is appreciated when palpating the trachea.
The emergency care provider decides to intubate to stabilize the patient’s breathing while waiting
for the trauma team to arrive. The emergency care provider infuses 35 mg IV rocuronium before
intubating but has difficulty securing the airway due to continued muscle tightness appreciated in
the larynx. Why would the administration of rocuronium not be the best choice for facilitating
intubation in this patient?

Choices:
1. Rocuronium only works well in patients with a lower body mass.
2. The patient is suffering from third-degree burns.
3. Steroidal non-depolarizing neuromuscular blockers should not be used for intubation, only
benzylisoquinolines.
4. Rocuronium was the right choice; the patient is a fast metabolizer.
Answer: 2 - The patient is suffering from third-degree burns.
Explanations:
Rocuronium is infused IV at 0.45 to 0.90 mg/kg for intubation with 0.15 mg/kg boluses if
needed for maintenance.
Burn patients exhibit resistance to non-depolarizing neuromuscular blockers due to being in
hyperkalemic states. Hyperkalemia causes resistance, while hypokalemia augments activity.
Other electrolyte abnormalities to keep in mind are hypocalcemia and hypermagnesemia,
both of which augment blockade.
Both steroidal (rocuronium, vecuronium, pancuronium) and benzylisoquinoline (atracurium,
cisatracurium, mivacurium) agents exhibit the same mechanism of action. What
differentiates them is their chemical structure and reversal; the steroidal agents are reversed
with sugammadex and the benzylisoquinoline agents with neostigmine/glycopyrrolate.
Rocuronium is one of the few agents in this drug class that is not metabolized but instead
cleared mainly by the liver and slightly by the kidneys in its original form. Its volume of
distribution is not affected by the renal disease; however, its action is prolonged by severe
hepatic failure and pregnancy.

Go to the next page if you knew the correct answer, or click the link image(s) below to further
research the concepts in this question (if desired).

Research Concepts:
Non-depolarizing Neuromuscular Blockers

We update eBooks quarterly and Apps daily based on user feedback. Please tap flag to
report any questions that need improvement.
Question 283: A 70-year-old female with chronic obstructive pulmonary disease is placed
on mechanical ventilation for respiratory failure. Intubation required rocuronium for paralysis.
The ventilator was placed on assist-control (AC) mode with a rate of 12, a fraction of inspired
oxygen (FIO2) of 1.0, a tidal volume of 500, and positive end-expiratory pressure (PEEP) of 0.
Arterial blood gas after intubation demonstrates pH 7.23, PaCO2 75 mmHg, and PO2 350
mmHg, so FIO2 is decreased to 0.70. Half an hour later the patient becomes hypotensive with a
blood pressure of 75/40 mmHg, heart rate 135 beats per minute, and respiratory rate 26/minute.
The trachea is midline, and there are breath sounds in both lung fields. Bilateral wheezing
persists until the next inspiration. The high-pressure alarm has triggered. What is the preferred
initial management?

Choices:
1. Disconnect the ventilator and manually ventilate the patient
2. Administer intravenous fluid bolus
3. Perform bilateral needle decompression of thorax
4. Start dopamine infusion
Answer: 1 - Disconnect the ventilator and manually ventilate the patient
Explanations:
Mechanical ventilation of patients with chronic obstructive pulmonary disease may develop
intrinsic positive end-expiratory pressure also known as auto-positive end-expiratory
pressure (PEEP).
Obstructive lung disease requires a longer time for expiration to avoid the development of
auto-PEEP and dynamic hyperinflation. This can be avoided by having a low inspiratory:
expiratory ratio (I:E).
As auto-PEEP and dynamic hyperinflation progress there is an increase in transthoracic
pressure. This results in decreased preload into the right ventricle. As preload decreases you
can get resulting hypotension. The increased alveolar pressure can also result in
pneumothorax, but this has not yet happened in this patient.
The ventilator settings will need to be adjusted. There will need to be a prolonged expiration
time. As the patient is overbreathing the ventilator, she will need to be highly sedated to
allow for ventilator synchrony. If needed the inspiratory flow rate can be increased to help
lower the I:E ratio.

Go to the next page if you knew the correct answer, or click the link image(s) below to further
research the concepts in this question (if desired).

Research Concepts:
Mechanical Ventilation

We update eBooks quarterly and Apps daily based on user feedback. Please tap flag to
report any questions that need improvement.
Question 284: A 42-year-old man was admitted two days back after an intentional
overdose of his medications at home. His chronic medical conditions include hypertension, mild
pulmonary hypertension, and Raynaud syndrome. He does not smoke or drink alcohol. Initial
presentation was with acute shock and hypoxic respiratory failure requiring emergent
endotracheal intubation. Initial lab work did not show any renal or hepatic dysfunction. He has
since been on vasopressors with maximal norepinephrine dose of 10 micrograms per minute 24
hours back and slowly being weaned. He has also received insulin and dextrose infusion and
several grams of calcium chloride within the first 24 hours. On the third day, the nurse noticed
new skin rashes over extremities. He does not have any fever. Bloodwork shows normal
coagulation parameters and platelets. However, glomerular filtration rate shows an acute drop to
32. In addition, transaminases are four times elevated from baseline. To what could one attribute
these changes in his clinical picture?

Choices:
1. Progression of shock and multiorgan failure
2. Vasopressor toxicity with poor end-organ perfusion
3. Calcium chloride effect
4. Acute leptospirosis
Answer: 3 - Calcium chloride effect
Explanations:
The patient likely has a calcium channel blocker overdose, as his medical conditions of
hypertension and Raynaud syndrome require treatment with a calcium channel blocker
agent. He has no fever or leukocytosis and had normal renal and liver function. This is
therefore unlikely an infection like leptospirosis
Calcium chloride or gluconate are often used in the initial treatment of calcium channel
blocker overdose. Overaggressive use in the first 24 hours is often seen and can rarely result
in calciphylaxis acutely presenting with skin necrosis, rash, acute kidney injury, and
abnormal liver function.
Progression of multiorgan failure or vasopressor adverse effects is unlikely to develop now
when his needs are slowly improving.
Although hemodialysis is not indicated in the treatment of calcium channel blocker toxicity,
it may be indicated with worsening toxicity of calciphylaxis and kidney injury in the setting
of calcium channel blocker overdose.

Go to the next page if you knew the correct answer, or click the link image(s) below to further
research the concepts in this question (if desired).

Research Concepts:
Calcium Channel Blocker Toxicity

We update eBooks quarterly and Apps daily based on user feedback. Please tap flag to
report any questions that need improvement.
Question 285: A 65-year-old male was admitted to the hospital with fever, productive
cough, and shortness of breath. Initial evaluation revealed leukocytosis with a left shift, and chest
radiograph showed a left lower lobe infiltrate. He was started on appropriate intravenous
antibiotics with oxygen support via nasal cannula. After 2 days of hospitalization, the patient
experienced worsening respiratory distress with an increased requirement of oxygen, and he
became more confused. Repeat chest radiograph showed the development of new-onset right-
sided infiltrate and worsening of the left side infiltrate. Arterial blood gas, while the patient is
breathing on 6 L/min via a face mask, showed a pH of 7.32, PaCO2 of 38 mmHg, and a PaO2 of
53 mmHg. What is the most appropriate next step in management?

Choices:
1. Transfer to the intensive care unit and start non-invasive ventilation with a serial assessment of
the patient’s clinical condition
2. Transfer to intensive care unit, intubate and start volume assist-control ventilation with tidal
volume 6 ml/kg of ideal body weight target plateau pressure of 30 cm H2O
3. Transfer to intensive care unit, intubate and start pressure support ventilation
4. Transfer to intensive care unit, intubate and start volume assist-control ventilation with tidal
volume 10 ml/kg of ideal body weight, and target plateau pressure of 45 cm H2O.
Answer: 2 - Transfer to intensive care unit, intubate and start volume assist-control
ventilation with tidal volume 6 ml/kg of ideal body weight target plateau pressure of 30 cm H2O

Explanations:
Pneumonia is a common cause of acute respiratory distress syndrome (ARDS) in
hospitalized patients. Early recognition of worsening clinical conditions and the
development of ARDS is essential in the management of these patients.
Patients who developed ARDS should be transferred to ICU, intubated and mechanical
ventilation should be started with volume assist-control with a tidal volume of 6 ml/kg of
ideal body weight with a target inspiratory plateau pressure of 30 cm of H2O to avoid
barotrauma and further lung injury.
Low tidal volume and low-pressure ventilation are associated with a significant reduction in
mortality and various clinical outcomes when compared with the higher tidal volume of 12
ml/kg and higher plateau pressures of 45 cm H2O.
Management of ARDS should also involve treatment of the underlying cause, prevention of
complications associated with a hospital stay, including nosocomial infections, venous
thromboembolism, stress ulcers, and aspiration.

Go to the next page if you knew the correct answer, or click the link image(s) below to further
research the concepts in this question (if desired).

Research Concepts:
Respiratory Distress Syndrome

We update eBooks quarterly and Apps daily based on user feedback. Please tap flag to
report any questions that need improvement.
Question 286: A 65-year-old patient is brought to the hospital by his family after he was
found in bed, not talking, and not moving his right side. He is admitted to the hospital for a
stroke work up. His initial head CT is negative, and the plan was to proceed to MRI brain and
MRA of the head and neck since the patient had an iodine contrast allergy (rash and throat
itching). While waiting to get an MRI, the patients mental status decompensates, and he becomes
comatose requiring intubation. His exam off sedation reveals quadriplegia and no response to
noxious stimuli. What is the most important next step in the management of this patient?

Choices:
1. Admit the patient to a neurological ICU and obtain an EEG of the brain to rule out subclinical
seizures as the cause
2. Wait for the MRI brain and MRA head of the neck for the diagnosis
3. Repeat CT head and send the patient to cerebral angiography to evaluate for large vessel
occlusion and possible treatment
4. Obtain a CTA of the head and neck
Answer: 3 - Repeat CT head and send the patient to cerebral angiography to evaluate for
large vessel occlusion and possible treatment

Explanations:
The patient presented with focal neurological deficits that progressed to coma and
quadriplegia. This is a classical presentation of locked-in syndrome caused by basilar artery
occlusion. The patients can have waxing and waning symptoms prior to the full onset of the
syndrome.
Also, the patients can have alternating hemiparesis, which is seen with tumbling embolism
as goes up the basilar artery before it causes full occlusion and infarction. In this situation,
the deficits happened acutely and would, therefore, need emergent intervention with an
attempted thrombectomy.
The iodine allergy should not have delayed the CTA in someone with suspected large vessel
occlusion as these patients can be pre-medicated with steroids and antihistamines.
The patient will eventually need to be admitted to the neurological ICU, and subclinical
seizures are on the differential. However, this is not the most likely scenario as the patient
did not have a generalized tonic-clonic seizure and the biggest clinical concern here is a
basilar occlusion causing locked-in syndrome. If vessel evaluation is completed and
unremarkable other etiologies such as infection, inflammatory, herniation or seizures should
be considered. Waiting for the MRI brain and MRA head and neck can delay the diagnosis.
These two studies can take up to 30-40 minutes to complete. Stroke diagnosis is made
clinically and treatment is initiated before confirmation by MRI. Obtaining a CTA of the
head and neck is reasonable. However, in this case, the suspected diagnosis is basilar
occlusion, and delays in treatment should be avoided. Cerebral angiography can be
diagnostic and therapeutic. Sending the patient directly to the angiography suite would
avoid giving the patient contrast twice.

Go to the next page if you knew the correct answer, or click the link image(s) below to further
research the concepts in this question (if desired).

Research Concepts:
Foville Syndrome

We update eBooks quarterly and Apps daily based on user feedback. Please tap flag to
report any questions that need improvement.
Question 287: Assessing a child's neurologic function during an emergency can be
challenging. However, there are standardized evaluation techniques that can be done quickly by
any member of the healthcare team. They involve checking the child’s pupillary response to light
and use of either the Alert, Verbal, Pain, Unresponsive (AVPU) pediatric response scale or the
Glasgow coma scale (GCS). What are the three components of the GCS?

Choices:
1. eye opening, motor response, and memory
2. eye opening, motor response, and pain response
3. eye opening, verbal response, and motor response
4. memory, verbal response, and eye opening
Answer: 3 - eye opening, verbal response, and motor response
Explanations:
A Glasgow coma scale (GCS) score should be assigned to every child with significant head
trauma. This scale assesses eye opening as well as motor and verbal responses.
The GCS helps categorize neurologic disability, and serial measurements can help identify
improvement or deterioration over time.
The score is broken up into three sections with a maximum of 4 points for eye-opening, 5
for verbal response and 6 for motor response for a total score of 15. The higher the score,
the better the chances of improvement and normal brain function. A child with severe
neurological impairment with a GCS score of 8 or less should be intubated.
The GCS provides a practical method for assessing consciousness in response to defined
stimuli.

Go to the next page if you knew the correct answer, or click the link image(s) below to further
research the concepts in this question (if desired).

Research Concepts:
Glasgow Coma Scale

We update eBooks quarterly and Apps daily based on user feedback. Please tap flag to
report any questions that need improvement.
Question 288: A patient who is 8 weeks pregnant ate some unpasteurized cheese 2 days
ago and now presents with muscle aches, diarrhea, low-grade fever, and malaise. She says the
pregnancy has been uneventful, but she has persistent nausea all the time. Blood work and
urinalysis are all normal. Examination of stools reveals a gram-positive organism with "tumbling
motility." What is the most likely diagnosis?

Choices:
1. Hyperemesis gravidarum
2. Hepatitis A
3. Listeria
4. Escherichia coli
Answer: 3 - Listeria
Explanations:
Listeria is a gram-positive, facultative anaerobic, non-spore-forming bacillus with a
characteristic "tumbling" motility.
Listeria primarily affects older individuals, pregnant women, newborns, and
immunosuppressed individuals.
Individuals with listeriosis usually have fever and myalgia, sometimes preceded by diarrhea
and other gastrointestinal symptoms.
Unlike most bacteria, Listeria can grow and multiply in some foods in the refrigerator.

Go to the next page if you knew the correct answer, or click the link image(s) below to further
research the concepts in this question (if desired).

Research Concepts:
Listeria Monocytogenes

We update eBooks quarterly and Apps daily based on user feedback. Please tap flag to
report any questions that need improvement.
Question 289: A young female is brought to the emergency department with an acute onset
of crushing substernal pain radiating to the left arm and shortness of breath. Significant past
medical history includes hospitalization for a heart problem at age 3 years that was treated with
aspirin and gamma globulin. Her electrocardiogram shows 3 mm of ST elevation in the anterior
leads. She has followed up for yearly echocardiograms since this time. Select the most likely
cause of the patient's symptoms.

Choices:
1. Vasospasm after cocaine ingestion
2. Thrombosis of a coronary artery aneurysm
3. Vasculitis of the left anterior descending artery
4. Dissection of the aortic root compromising blood flow into the left coronary ostium
Answer: 2 - Thrombosis of a coronary artery aneurysm
Explanations:
Kawasaki disease is a vasculitis of medium and small vessels that can lead to coronary
artery aneurysms. Intravenous immunoglobulin (IVIG) treats the vasculitis and decreases
the risk of aneurysm from 25% to 3%. These patients are also treated initially with high
dose aspirin.
Patients with large coronary artery aneurysms are treated with antithrombotic agents (low-
molecular-weight heparin or warfarin) and antiplatelet agents (aspirin, clopidogrel or
dipyridamole).
Most aneurysms regress, especially if fusiform, small and distal, but those greater than 8
mm can persist and develop rupture, thrombosis, recanalization, and stenosis at the outflow
area.
Cardiac stress testing for reversible ischemia is recommended for KD children with known
coronary aneurysms.

Go to the next page if you knew the correct answer, or click the link image(s) below to further
research the concepts in this question (if desired).

Research Concepts:
Kawasaki Disease

We update eBooks quarterly and Apps daily based on user feedback. Please tap flag to
report any questions that need improvement.
Question 290: Purpura fulminans is the skin manifestation of disseminated intravascular
coagulation, necrotizing fasciitis, and overwhelming sepsis. Aside from antibiotics, resuscitation
and source control, what specific therapy can be started which directly addresses purpura
fulminans?

Choices:
1. The 4-factor prothrombin complex concentrate (4F-PCC)
2. Activated protein C concentrate
3. Methylprednisolone sodium succinate
4. Recombinant activated clotting factor VII (rFVIIa)
Answer: 2 - Activated protein C concentrate
Explanations:
Purpura fulminans is an acute purpuric rash characterized by coagulation of the
microvasculature which leads to purpuric lesions and skin necrosis. It is a true
dermatological emergency and requires immediate diagnosis and management.
Treating the underlying cause of purpura fulminans is the most important aspects of
treatment. Early antibiotics, fluid resuscitation, and in certain cases early surgery are first-
line therapy.
Purpura fulminans is associated with a relative deficiency of protein C, an important
regulator of blood coagulation, leading to consumption coagulopathy, intravascular fibrin
deposition, downregulated fibrinolysis, and clotting of the microcirculation.
Activated protein C is believed to decrease tissue damage and speed recovery. Use of
activated protein C has been shown to improve mortality over predicted historical mortality,
but no randomized controlled trials have been performed.

Go to the next page if you knew the correct answer, or click the link image(s) below to further
research the concepts in this question (if desired).

Research Concepts:
Purpura Fulminans

We update eBooks quarterly and Apps daily based on user feedback. Please tap flag to
report any questions that need improvement.
Question 291: A 56-year-old is admitted to the hospital with bleeding rectal varices. She is
on the liver transplant list with a MELD score of 30 with a 3-month predicted mortality of
52.6%. Her Child-Pugh score is 8, making her a Child B with a 1-year mortality of 20%. Which
of the following, if present, best explains the difference in mortality predicted by the two scoring
systems?

Choices:
1. Creatinine of 6.0 mg/dL, on dialysis
2. Bilirubin 2.5 mg/dL
3. INR 1.8
4. Lack of ascites
Answer: 1 - Creatinine of 6.0 mg/dL, on dialysis
Explanations:
The MELD score incorporates renal function into the scoring system, including both serum
creatinine and whether the patient is currently on dialysis.
One main criticism of the MELD system is the heavy emphasis on renal function without
incorporating factors such as ascites, encephalopathy, and nutrition status.
Currently, the transplant allocation system uses the MELD score for organ allocation due to
the wider range of continuous variables and lack of subjective scoring items.
The main criticism of the Child-Pugh score is the use of subjective variables, smaller range
of continuous variables, and exclusion of renal function.

Go to the next page if you knew the correct answer, or click the link image(s) below to further
research the concepts in this question (if desired).

Research Concepts:
Use Of The Child Pugh Score In Liver Disease

We update eBooks quarterly and Apps daily based on user feedback. Please tap flag to
report any questions that need improvement.
Question 292: A 58-year-old man with chronic obstructive pulmonary disease is brought to
the emergency department for disorientation. He has been taking theophylline for six months,
and the bottle is empty. His theophylline level is 51 mcg/mL. Which action should be taken first?

Choices:
1. Load immediately with phenobarbital
2. Give prochlorperazine to prevent vomiting and decrease the risk of aspiration
3. Administer 1 amp of calcium gluconate
4. Contact the on-call nephrologist to discuss arranging urgent hemoperfusion.
Answer: 4 - Contact the on-call nephrologist to discuss arranging urgent hemoperfusion.
Explanations:
Determining if a patient has an acute or chronic theophylline overdose is vital. Those with
acute overdose are at higher risk for gastrointestinal and cardiac complications.
Theophylline toxicity often present with seizures that are most effectively treated with a
benzodiazepine, not a long-acting barbiturate such as phenobarbital.
Additional medications with anticholinergic or antihistaminic properties, such as
prochlorperazine, should be avoided.
Neurologic complications are more likely in chronic overdose situations. This patient likely
has chronic ingestion, and levels greater than 40 mcg/mL should be considered for urgent
hemoperfusion.

Go to the next page if you knew the correct answer, or click the link image(s) below to further
research the concepts in this question (if desired).

Research Concepts:
Theophylline Toxicity

We update eBooks quarterly and Apps daily based on user feedback. Please tap flag to
report any questions that need improvement.
Question 293: A 47-year-old male with no relevant past medical history presents to the
hospital complaining of dyspnea on exertion and chest pain. He states symptoms started one
week ago and have progressively worsened. Upon further questioning, he also reports an episode
of flu-like symptoms 4 weeks ago. He lives in central Pennsylvania, owns a farm, and spends
most of his time working on the field. An electrocardiogram taken in the emergency department
shows S-T depression of the anterior leads, but two sets of troponins have been negative. Lyme
myocarditis is suspected. What would a transthoracic echocardiogram most likely show?

Choices:
1. Severe mitral valve regurgitation
2. Diffuse ventricular hypokinesis
3. Focal wall motion abnormalities
4. Increased right ventricular pressure
Answer: 3 - Focal wall motion abnormalities
Explanations:
Lyme myocarditis may present with symptoms and electrocardiographic findings similar to
acute coronary syndrome.
T wave inversion or ST-segment depression are usually seen; however, high troponin level
and/or ST-segment elevation is uncommon.
In acute coronary syndrome, echocardiography reveals diffuse ventricular hypokinesis.
Echocardiography will show focal wall motion abnormalities in myocarditis.

Go to the next page if you knew the correct answer, or click the link image(s) below to further
research the concepts in this question (if desired).

Research Concepts:
Lyme Carditis

We update eBooks quarterly and Apps daily based on user feedback. Please tap flag to
report any questions that need improvement.
Question 294: A 50-year-old patient with chronic alcohol used disorder presents with the
report of drinking windshield washer fluid after running out of vodka. He smells of alcohol and
is intoxicated but is otherwise asymptomatic with normal vital signs. An arterial blood gas
reveals a pH of 7.3 and bicarbonate of 22 mmol/L. His ethanol concentration is 200 mg/dL. A
methanol concentration will take 24 hours to result. What is the next most appropriate step?

Choices:
1. Administer fomepizole
2. Perform serial basic metabolic panels monitoring for acidosis starting now
3. Admit to the intensive care unit
4. Perform serial metabolic panels monitoring for acidosis starting in 8 to 10 hours.
Answer: 4 - Perform serial metabolic panels monitoring for acidosis starting in 8 to 10 hours.
Explanations:
The patient is intoxicated with ethanol. He is not at risk of developing an acidosis from
metabolizing any methanol while ethanol is competing for alcohol dehydrogenase.
Ethanol's elimination rate is about 15 to 25 mg/dL/hr, depending on the individual. At
approximately 20 mg/dL/hr, it can be anticipated that the patient's ethanol concentration
will be approaching 0 in 8 to 10 hours. At that time, metabolism of methanol will begin and
monitoring for acidosis should occur. Monitoring should continue for 12 hours after ethanol
clears.
Fomepizole should not be given empirically. In this situation, it would prolong the patient's
length of stay as it prolongs the half-life of ethanol. Once ethanol clears, fomepizole should
be given if an acidosis occurs.
Provided the patient's mental status and vital signs are normal, the patient does not require
intensive care monitoring.

Go to the next page if you knew the correct answer, or click the link image(s) below to further
research the concepts in this question (if desired).

Research Concepts:
Methanol Toxicity

We update eBooks quarterly and Apps daily based on user feedback. Please tap flag to
report any questions that need improvement.
Question 295: A 60-year-old male with a history of stable coronary artery disease,
hypertension, hyperlipidemia, and constipation presents with complaints of a large amount of
painless bright red bleeding per rectum for the last two days, which appears to have now
subsided. He is lightheaded upon standing and appears somewhat pale. You suspect he has a
large diverticular bleed. Supportive treatment is initiated. Intravenous access is obtained, and
resuscitative fluids started. CBC shows a hemoglobin of 5 g/dL. What is the recommended goal
hemoglobin level in this patient?

Choices:
1. Greater than 6 g/dL
2. Greater than 7 g/dL
3. Greater than 8 g/dL
4. Greater than 9 g/dL
Answer: 2 - Greater than 7 g/dL
Explanations:
The American College of Physicians recommends a restrictive transfusion threshold with a
goal hemoglobin greater than 7 g/dL in anemic patients.
Multiple studies have shown that a liberal transfusion strategy with a goal hemoglobin
greater than 10 g/dL confers no mortality benefit and may be harmful to patients.
The American College of Gastroenterology also recommends transfusing packed red cells
to maintain a hemoglobin level above 7 g/dL but says a threshold of 9 g/dL should be
considered in patients with massive bleeds, active ischemia, or delay in receiving
therapeutic interventions.
Transfusion strategies and thresholds do not differentiate between upper and lower
gastrointestinal bleeds.

Go to the next page if you knew the correct answer, or click the link image(s) below to further
research the concepts in this question (if desired).

Research Concepts:
Gastrointestinal Bleeding

We update eBooks quarterly and Apps daily based on user feedback. Please tap flag to
report any questions that need improvement.
Question 296: A female patient presents with bilateral hydroureteronephrosis and acute
renal failure secondary to retroperitoneal fibrosis. Bilateral percutaneous nephrostomies are
placed by interventional radiology. Her urine output has been greater than 200 mL/hour for the
past four hours, and she is tolerating oral intake well at this time. What is the best next step in
management?

Choices:
1. With the patient’s excellent urine output, nothing more needs to be done.
2. Order frequent basic metabolic panels (BMPs) to monitor the patient’s electrolytes and renal
function.
3. Order frequent BMPs to monitor the patient’s electrolytes and renal function; instruct the
nurse to have the patient replace the fluid loss with oral hydration if the patient can tolerate and
keep up with losses.
4. Order frequent BMPs to monitor the patient’s electrolytes and renal function; replace the fluid
loss with an equal volume of intravenous half normal saline.
Answer: 3 - Order frequent BMPs to monitor the patient’s electrolytes and renal function;
instruct the nurse to have the patient replace the fluid loss with oral hydration if the patient can
tolerate and keep up with losses.

Explanations:
After renal decompression, the patient must be monitored closely for post-obstructive
diuresis, renal function status, and appropriate replacement of fluids and electrolytes.
Post-obstructive diuresis is a prominent polyuria that occurs immediately after relieving
severe urinary obstruction. Patients are often fluid overloaded initially, and urine output
often exceeds 200 mL/hr. Patients are at risk for severe dehydration, electrolyte imbalances,
and hypovolemic shock. The condition usually lasts less than 48 hours.
Treatment includes close monitoring of serum BUN, creatinine, and electrolytes, and
replacement of fluid. Oral replacement is preferred, but if the patient cannot tolerate oral
liquids or is unable to keep up with the loss, intravenous half normal saline can be used to
replace half of the fluid loss (500 mL for every 1 liter lost).
After initial management, it is necessary to identify the potential cause of retroperitoneal
fibrosis. Any potential inciting drug should be discontinued immediately.

Go to the next page if you knew the correct answer, or click the link image(s) below to further
research the concepts in this question (if desired).

Research Concepts:
Retroperitoneal Fibrosis

We update eBooks quarterly and Apps daily based on user feedback. Please tap flag to
report any questions that need improvement.
Question 297: A 57-year-old male with alcoholic liver cirrhosis presents to the emergency
department after 2 days of hematemesis. He currently is not taking any medications. A previous
chart shows esophageal varices listed in the history. Today's CBC shows a hemoglobin of 5.8
g/dL and hematocrit of 17.8%. The prothrombin time and INR are 22.5 and 1.7, respectively.
One unit of packed red blood cells and one unit of fresh frozen plasma are ordered to transfuse.
Within 3 hours of transfusion, the patient becomes dyspneic, and the lungs sound very
congested. His blood pressure is 95/62 mmHg and heart rate is 112 beats/min. An emergent chest
x-ray and arterial blood gas are ordered. What is the best next step?

Choices:
1. Intubate the patient
2. Stop the transfusion and notify the blood bank
3. Continue the transfusion and give IV furosemide 40 mg emergently
4. Slow the rate of blood transfusion
Answer: 2 - Stop the transfusion and notify the blood bank
Explanations:
The best treatment for transfusion-related acute lung injury (TRALI) is to stop the
transfusion. The blood transfusion will worsen the patient's condition. TRALI usually is
associated with plasma components such as platelets and fresh frozen plasma.
Supportive care is the mainstay of TRALI treatment. Oxygen supplementation is needed.
Also, IV fluids and vasopressors are needed for blood pressure support.
Intubating the patient and protecting the airway is next step if the patient's respiratory status
worsens.
Diurectics should be avoided in TRALI treatment.

Go to the next page if you knew the correct answer, or click the link image(s) below to further
research the concepts in this question (if desired).

Research Concepts:
Transfusion-related Acute Lung Injury

We update eBooks quarterly and Apps daily based on user feedback. Please tap flag to
report any questions that need improvement.
Question 298: A 66-year-old female is admitted for a five days history of fever, bloody
diarrhea, and abdominal pain. The patient has acute myeloid leukemia, and the last
chemotherapy she received was two weeks ago as an inpatient. While the patient was in the
hospital two weeks ago, she had a UTI and was treated with IV antibiotics. On physical
examination, her temperature is 38.5 C (101.3 F), blood pressure 95/50 mmHg, pulse rate
103/min, and respiratory rate is 15/min. The abdominal exam shows diffuse tenderness. Imaging
showed thickened bowel wall of the cecum and ascending colon, fat stranding, and air under the
diaphragm. Her labs show hemoglobin (Hb) 7 mg/dL, white blood cells (WBC) 300/micoL, and
platelets 15000/microL. What is the next best step in the management of this patient?

Choices:
1. Stool analysis and culture
2. Bowel rest and Nasogastric suctioning
3. Evaluation for surgery
4. Oral vancomycin
Answer: 3 - Evaluation for surgery
Explanations:
The patient presented with symptoms of colitis (fever, abdominal pain, and diarrhea) and in
such patient stool analysis and culture is part of the workup.
However, the patient has concerning signs, including the hemodynamic instability and the
air under diaphragm seen on CT imaging.
So the most important next step in management is to be evaluated by surgery for possible
bowel perforation.
Bowel rest and NG suctioning is part of the management of any patient with colitis, but the
most appropriate next step the patient needs is to be evaluated by surgery for early
intervention. She might have had neutropenic enterocolitis or colitis due to other infectious
etiology. Regardless of the cause, the patient has hemodynamic instability, and CT imaging
shows a sign of bowel perforation (air under diaphragm) and surgical evaluation is needed
to do intervention as soon as possible to decrease complications and mortality. The patient
has colitis of which the cause is still unknown. Clostridium difficile testing should be done
as part of the workup, and if positive, she will be given oral vancomycin. However, the
patient has complicated colitis that needs to be evaluated by surgery urgently.

Go to the next page if you knew the correct answer, or click the link image(s) below to further
research the concepts in this question (if desired).

Research Concepts:
Neutropenic Enterocolitis (Typhlitis)

We update eBooks quarterly and Apps daily based on user feedback. Please tap flag to
report any questions that need improvement.
Question 299: A 17-year-old female presents to the emergency department with a fever and
a headache. She has no significant past medical history. Her temperature is measured to be 39 C
while blood pressure is 102/74 mmHg. Physical examination shows petechiae on her legs. Neck
stiffness is present as well. Lumbar puncture is performed after which appropriate antibiotics and
fluids are started. The next day, she develops severe back pain, perineal anesthesia and is unable
to urinate. An MRI confirms spinal cord compression. Which of the following is the most likely
cause of the compression in this patient?

Choices:
1. Spinal abscess
2. Epidural hematoma
3. Disc herniation
4. Tumor
Answer: 2 - Epidural hematoma
Explanations:
Cauda equina syndrome (CES) is a known complication of lumbar puncture procedures
involving direct trauma or an epidural hematoma.
Medicolegal litigation is high in these syndromes due to missed diagnoses and a high degree
of residual functional impairment.
Surgical decompression within 48 hours is paramount in CES and conus medullaris
syndrome.
Saddle anesthesia is one of the most common presenting symptoms. The most common
cause of cauda equina syndrome is disk herniation.

Go to the next page if you knew the correct answer, or click the link image(s) below to further
research the concepts in this question (if desired).

Research Concepts:
Cauda Equina And Conus Medullaris Syndromes

We update eBooks quarterly and Apps daily based on user feedback. Please tap flag to
report any questions that need improvement.
Question 300: A patient arrives with severe shoulder pain for one day. There is redness and
swelling and limited range of motion but no history of trauma. The patient has a fever and looks
toxic. While waiting for his x-ray, the patient notices the skin has become numb to the touch and
bulla has started to form on the affected area. Examination shows irregular hemorrhagic necrosis.
What is the name of this characteristic rash?

Choices:
1. Purpura fulminans
2. Erysipelas
3. Bullous pemphigoid
4. Staphylococcal scalded skin syndrome
Answer: 1 - Purpura fulminans
Explanations:
Purpura fulminans is a disorder characterized by rapidly progressing skin lesions with a
typical morphology. As this disease is associated with severe illness and rapid progression,
it is important to keep a high degree of suspicion.
The skin findings of purpura fulminans have a characteristic appearance and evolution.
Purpura fulminans begins with erythema which develops irregular hemorrhagic necrosis and
in some cases, vesicles and bulla form.
The affected skin initially is painful and indurated but in the later stages, there may be total
loss of sensation.
As the later stages of purpura fulminans may include large areas of necrosis, close
monitoring and multiple surgeries for debridement are often required.

Go to the next page if you knew the correct answer, or click the link image(s) below to further
research the concepts in this question (if desired).

Research Concepts:
Purpura Fulminans

We update eBooks quarterly and Apps daily based on user feedback. Please tap flag to
report any questions that need improvement.
Section 4
Question 301: A 17-year-old male who lives in a homeless shelter is brought to the
emergency department by emergency medical services for significant respiratory distress. He has
a history of alcohol use disorder and intravenous drug use. He has had hemoptysis for several
weeks. He is admitted to the intensive care unit and intubated for respiratory failure. A chest x-
ray shows a left-sided, moderate size pleural effusion with mediastinal shift and perihilar
lymphadenopathy. What is the most probable cause of his pleural effusion?

Choices:
1. Haemophilus influenzae
2. Staphylococcus aureus
3. Mycobacterium tuberculosis
4. Pseudomonas aeruginosa
Answer: 3 - Mycobacterium tuberculosis
Explanations:
Pleural effusion is the accumulation of fluid in the pleural cavity. The mechanism by which
fluid accumulates in the pleural cavity includes infection producing exudates, increased
capillary permeability causing the production of exudative fluid in the pleural cavity, and
increased hydrostatic pressure or decreased oncotic pressure leading to the formation of
transudates.
In this vignette, all of the organisms listed can cause pleural effusion. The combination of
the patient being homeless and using intravenous drugs make him more likely to have
tuberculosis.
The most common cause of pleural effusion in the pediatric population is infection. The
infectious agents that can cause pleural effusion include Streptococcus pneumoniae,
Hemophilus influenza type B, Staphylococcus aureus, Mycobacterium tuberculosis, and
Pneumocystis jiroveci in immunocompromised patients.
Diagnosis can be confirmed by an acid-fast bacilli (AFB) smear and culture, interferon-
gamma release assay, or pleural fluid adenosine deaminase. According to the Centers for
Disease Control and Prevention, culture remains the criterion standard for laboratory
confirmation of tuberculosis disease.

Go to the next page if you knew the correct answer, or click the link image(s) below to further
research the concepts in this question (if desired).

Research Concepts:
Parapneumonic Pleural Effusions And Empyema Thoracis

We update eBooks quarterly and Apps daily based on user feedback. Please tap flag to
report any questions that need improvement.
Question 302: A 40-year-old woman with fevers, cough, and productive sputum for five
days, is admitted to the intensive care unit requiring intubation and mechanical ventilation. She
presented to the emergency department with acute respiratory distress syndrome (ARDS)
secondary to pneumonia. Oxygenation saturation had been satisfactory on an FIO2 of 50% and
positive end-expiratory pressure (PEEP) of 6 cm H2O but dropped to the low 80% despite an
increase in FIO2 to 100%. On physical examination, the patient is intubated and sedated. Lung
exam reveals diffuse rales and rhonchi. The examination of the heart is unremarkable except for
tachycardia (heart rate of 115 beats/min). She has 2+ peripheral edema. A chest x-ray shows
diffuse bilateral infiltrates. She is being ventilated using an assist/control mode with a tidal
volume of 6 mL/kg and a plateau pressure of 25 cm H2O. Which of the following is the most
appropriate strategy for PEEP in this patient?

Choices:
1. Increase PEEP in 2- to 3-cm H2O increments and lower FIO2 to at most 60%, if possible.
Arterial oxygen saturation of at least 88% and 95% or less must be maintained
2. Set the PEEP below the lower inflection point on a pressure-volume curve of the lung
3. Maintain PEEP to correspond to the expiratory pressure that minimizes compliance of the lung
4. Maintain PEEP of at least 14 cm H2O and increase up to 20 cm H2O for FIO2 of 0.5 to 0.8.
Monitor cardiac output using a pulmonary artery catheter
Answer: 1 - Increase PEEP in 2- to 3-cm H2O increments and lower FIO2 to at most 60%, if
possible. Arterial oxygen saturation of at least 88% and 95% or less must be maintained

Explanations:
In this patient with ARDS secondary to pneumonia, increasing PEEP in 2- to 3-cm H2O
increments and subsequently lowering FIO2 to at most 60% while maintaining an arterial
oxygen saturation of at least 88% and 95% or less is appropriate.
This strategy has been used by ARDSnet and associated with improved outcomes. It is a
"lung protective strategy".
Lower PEEP is associated with reduced barotrauma.
There is no utility for a pulmonary artery catheter in the management of PEEP.

Go to the next page if you knew the correct answer, or click the link image(s) below to further
research the concepts in this question (if desired).

Research Concepts:
Pulmonary Edema

We update eBooks quarterly and Apps daily based on user feedback. Please tap flag to
report any questions that need improvement.
Question 303: A 55-year-old male is presenting for follow up after recently being
diagnosed with non-small cell lung cancer. He has no new complaints on this visit. No prior
medical or surgical history and does not take any medications. He continues to smoke one pack
per day for past 30 years. Staging workup for his cancer reveals a normal MRI of the brain.
Contrast-enhanced CT of abdomen and pelvis does not reveal any evidence of distant metastasis
but a left renal vein thrombus extending to the inferior vena cava. Both kidneys appear normal in
size. Routine labs including complete blood count, renal and liver function panel are
unremarkable. INR is 1. Urinalysis and microscopy are unremarkable. Which is the best
treatment option for the patient’s renal vein thrombus?

Choices:
1. Aspirin 81mg and clopidogrel 75 mg daily
2. Enoxaparin 1mg/kg Q12 hours
3. Rivaroxaban
4. No treatment is necessary
Answer: 2 - Enoxaparin 1mg/kg Q12 hours
Explanations:
Patients with asymptomatic renal vein thrombosis are best treated with anticoagulation. The
reason is to prevent progression of the thrombus or development of a new thromboembolic
event such as pulmonary embolism.
Initial treatment consists of starting unfractionated or low-molecular-weight heparin and
then bridged to warfarin, for a goal INR of 2 to 3.
Anticoagulation is generally given for 6 to 12 months
Novel oral anticoagulants (NOAC) are not approved for the treatment of renal vein
thrombosis.

Go to the next page if you knew the correct answer, or click the link image(s) below to further
research the concepts in this question (if desired).

Research Concepts:
Renal Vein Thrombosis

We update eBooks quarterly and Apps daily based on user feedback. Please tap flag to
report any questions that need improvement.
Question 304: A 47-year-old female patient with a history of antiphospholipid antibody
syndrome who has been noncompliant with warfarin presents with a deep vein thrombosis and
dyspnea. On examination, her blood pressure is 90/60 mmHg, respiratory rate is 22/min, and her
heart rate is 92/min. Computerized tomography of the chest shows a saddle embolus. She does
not respond to heparin and fluids. An echocardiogram shows right ventricular hypokinesis. What
is the most appropriate next step?

Choices:
1. Urgent referral for surgical embolectomy
2. Recombinant tissue plasminogen activator
3. Continue administration of fluids and heparin
4. Add lepirudin
Answer: 2 - Recombinant tissue plasminogen activator
Explanations:
Anticoagulation alone is an inadequate treatment for this patient. Current guidelines for the
management of unstable patients recommend immediate treatment with anticoagulation.
Fibrinolytic treatment with recombinant tissue plasminogen activator is indicated unless
there are major contraindications.
If the patient does not improve or fibrinolysis is not possible, the best choice is surgical
embolectomy. Surgery is the last resort and never the first consideration. Placing such a
patient on bypass is associated with mortality in excess of 50%. Furthermore, even those
who survive are faced with several complications like acute respiratory distress syndrome
(ARDS), bleeding, and shock.
After surgery, the patient will need lifelong treatment with warfarin. Today, tPA is
recommended as the treatment of choice. A dose of 100 mg is given over 2 hours, and there
is a rapid resolution of the elevated pulmonary artery pressure.

Go to the next page if you knew the correct answer, or click the link image(s) below to further
research the concepts in this question (if desired).

Research Concepts:
Antiphospholipid Syndrome

We update eBooks quarterly and Apps daily based on user feedback. Please tap flag to
report any questions that need improvement.
Question 305: A 23-year-old male with no significant past medical history is brought to the
emergency department after being involved in a motor vehicle collision. He is found to have
multiple injuries, including bilateral rib fractures, left pneumohemothorax, splenic injury, and
cervical vertebral fractures. He is emergently intubated, and a left-sided chest tube is placed. He
is admitted to the intensive care unit for further care. Two days later, the patient develops
worsening hypoxemia. A chest X-ray shows worsening bilateral infiltrates compared to the
previous day, and there is no effusion. The chest tube has been draining about 100 mL of
serosanguinous fluid for the past two days. He is placed on a fraction of inspired oxygen of 1.0,
positive end-expiratory pressure 15 mmHg, low tidal volume ventilation 6 ml/kg, and respiratory
rate 30/min. Peak pressure is 40 cmH20, and plateau pressure is 38 cmH20. Arterial blood gas
analysis on these settings shows pH 7.20, pO2 60 mmHg, and pCO2 68 mmHg. What is the best
next step in management?

Choices:
1. Increase the tidal volume to facilitate the clearance of carbon dioxide
2. Initiate prone ventilation
3. Initiate venovenous extracorporeal membrane oxygenation (VV-ECMO)
4. Computed tomography angiography of the chest
Answer: 3 - Initiate venovenous extracorporeal membrane oxygenation (VV-ECMO)
Explanations:
In patients with respiratory distress, the mainstay of therapy is low tidal volume ventilation
of 4-6 ml/kg of ideal body weight while maintaining plateau pressures of less than 30
cmH20 and permissive hypercapnia with an acceptable pH. If the patient is not a candidate
for or fails other therapies for acute respiratory distress syndrome, the next step is
venovenous extracorporeal membrane oxygenation (VV-ECMO).
In patients with a purely gas exchange problem, whether it is hypoxemia or inability to clear
carbon dioxide, VV-ECMO is indicated as opposed to venoarterial ECMO unless the
patient has evidence of cardiogenic shock or pulmonary hypertension with a failing right
ventricle.
Prone ventilation is contraindicated in patients with unstable vertebral fractures; therefore,
VV-ECMO is the best next step in this patient.
In the absence of active bleeding, patients will need to be anticoagulated while on ECMO to
prevent thrombosis. Hemorrhage is one of the complications of ECMO, and therefore, the
risks of bleeding and thrombosis should be weighed on an individual basis.

Go to the next page if you knew the correct answer, or click the link image(s) below to further
research the concepts in this question (if desired).

Research Concepts:
Extracorporeal Membrane Oxygenation In Adults

We update eBooks quarterly and Apps daily based on user feedback. Please tap flag to
report any questions that need improvement.
Question 306: An 18-year-old with intellectual disability and cerebral palsy, who is
tracheostomy and ventilator dependent had two days of increased secretions with color change to
green. The worried parents bring him to the emergency department (ED) with one day of
increased ventilator settings and oxygen requirement. In the ED his temperature is 102.5 F, heart
rate of 140 beats/minute, blood pressure of 75/50 mmHg, and the oxygen saturation is 82% on
room air. A chest x-ray shows a right lower lobe infiltrate. The mother states that he recently
finished a 14-day course of levofloxacin. Blood and tracheal cultures are obtained, and it is
decided to start antibiotic treatment. His last tracheal culture which was obtained five months ago
grew methicillin-resistant Staphylococcus aureus (MRSA). Based on this information which of
the following antibiotic/antibiotic combinations will provide adequate coverage while awaiting
the blood and tracheal cultures?

Choices:
1. Vancomycin
2. Clindamycin
3. Vancomycin plus piperacillin-tazobactam plus gentamicin
4. Cefepime
Answer: 3 - Vancomycin plus piperacillin-tazobactam plus gentamicin
Explanations:
Empiric treatment for ventilator-associated pneumonia should include coverage against
methicillin-resistant Staphylococcus aureus, Pseudomonas, and other gram negatives.
Earlier treatment reduces the risk of complications. However, treatment should be tailored
based on the identification of the culprit organisms on tracheal and blood cultures because
prolonged broad-spectrum antibiotics are associated with development of resistant
infections as well as side effects.
This patient is presenting in septic shock and was on antibiotics over the past three months,
which are regarded as a risk factor for starting broad-spectrum antibiotics. Other risk factors
include acute respiratory distress syndrome, five days or more days of hospitalization before
development of ventilator-associated pneumonia, acute renal replacement therapy before
development of ventilator-associated pneumonia. In addition, one should use the local
antibiotic susceptibility pattern as well as the patient's past microbiology data.
In a patient with more than one risk factors for a multidrug-resistant organism, the empiric
treatment should cover methicillin-resistant Staphylococcus aureus with either vancomycin
or linezolid plus two anti-Pseudomonas agents such as piperacillin-tazobactam, cefepime,
meropenem, and aztreonam plus an aminoglycoside such as gentamicin, amikacin, or
tobramycin. In the absence of risk factors, only one anti-Pseudomonal agent should be used.
Monotherapy with aminoglycosides should be avoided.
The combination of antibiotics should be given intravenously initially. It should be tailored
once the antibiotic susceptibility results return. The decision to tailor antibiotics should also
take into consideration the progression of clinical and laboratory markers of infection such
as fevers, white blood cell, and procalcitonin.

Go to the next page if you knew the correct answer, or click the link image(s) below to further
research the concepts in this question (if desired).

Research Concepts:
Ventilator-associated Pneumonia

We update eBooks quarterly and Apps daily based on user feedback. Please tap flag to
report any questions that need improvement.
Question 307: A 31-year-old male presents to the emergency department with a high fever
for the past two days, vomiting, back pain, and dyspnea. The patient was staying in a rural cabin
noted to have rodents in North Korea and recently returned. Upon admitting the patient to the
intensive care unit for respiratory distress, thrombocytopenia, hypotension, and marked
leukocytosis with premature white blood cells, the patient asks if there is a point that will predict
a good outcome. What phase would predict good long-term recovery?

Choices:
1. Oliguric
2. Polyuric
3. Convalescent
4. Febrile
Answer: 2 - Polyuric
Explanations:
Hantavirus hemorrhagic fever and renal syndrome is found to have 5 distinct phases:
febrile, hypotensive, oliguric, polyuric, and convalescent.
The febrile period lasts around 3 to 7 days which can be accompanied by conjunctival
hemorrhages and fine palatal petechiae in the mouth. This phase is followed by the
hypotensive phase which yields one-third of the mortality of hemorrhagic fever and renal
syndrome.
The oliguric phase may need treatment with dialysis if renal function deteriorates
significantly. One-half of the fatalities in hemorrhagic fever and renal syndrome caused by
Hantavirus occur in this phase. The onset of the polyuric phase is a positive prognostic
indicator.
The convalescent phase is a return towards the patient's clinical and laboratory baseline that
may take up to 6 months.

Go to the next page if you knew the correct answer, or click the link image(s) below to further
research the concepts in this question (if desired).

Research Concepts:
Hantavirus Syndrome

We update eBooks quarterly and Apps daily based on user feedback. Please tap flag to
report any questions that need improvement.
Question 308: A 17-year-old male presents to the emergency department with a 2-day
history of fevers, headache, photophobia, and neck pain. Of note, the patient is 14 days status
post craniotomy for a gunshot wound to the head. On exam, the patient is lethargic but oriented
to person, place, and time. Workup is significant for a white blood cell count of 26,000
cells/microL and a lactate of 5.2 mmol/L. What should be the initial antibiotic therapy for this
patient?

Choices:
1. Vancomycin 20 mg/kg and cefepime 2 g intravenously
2. Vancomycin 20 mg/kg, ceftriaxone 2 g, and ampicillin 2 g intravenously
3. Vancomycin 20 mg/kg and ceftriaxone 2 g intravenously
4. Vancomycin 20 mg/kg and piperacillin-tazobactam 4.5g intravenously
Answer: 1 - Vancomycin 20 mg/kg and cefepime 2 g intravenously
Explanations:
Cefepime is a fourth-generation cephalosporin with good Pseudomonas coverage.
It is recommended in the setting of meningitis with recent neurosurgery or penetrating head
trauma.
Its structure allows better and more rapid penetration through the cell wall of aerobic gram-
negative bacilli.
Ampicillin is primarily used for Listeria coverage. Listeria should be suspected in the
neonatal and elderly populations

Go to the next page if you knew the correct answer, or click the link image(s) below to further
research the concepts in this question (if desired).

Research Concepts:
Bacterial Meningitis

We update eBooks quarterly and Apps daily based on user feedback. Please tap flag to
report any questions that need improvement.
Question 309: A 17-year-old male presents to the emergency department via emergency
medical services (EMS) with report of combativeness and encephalopathy that required chemical
sedation and intubation en route. On arrival, he was tachycardic with a heart rate of 124 bpm,
hypertensive at 190/110 mmHg, with mydriatic pupils and diaphoresis. Before administration of
further sedation, he became combative and had a seizure. His friend that called EMS reports that
they were only smoking cannabis. Your evaluations include a negative urine drug screen. Abuse
of which agent is most consistent with this presentation.

Choices:
1. Cocaine
2. Synthetic cannabinoids
3. Cannabis
4. Atropa belladonna
Answer: 2 - Synthetic cannabinoids
Explanations:
The patient's presentation is most consistent with abuse of synthetic cannabinoids.
Synthetic cannabinoids are sold as plant or plant-like substances that are laced with
chemicals that are designed to agonize cannabinoid receptors. These chemicals are vast and
newer ones are constantly engineered.
Synthetic cannabinoids are known to cause similar effects as cannabis but are often
associated with increased toxicity, including encephalopathy, seizures, and death.
Urine drug screens do not test for synthetic cannabinoids.

Go to the next page if you knew the correct answer, or click the link image(s) below to further
research the concepts in this question (if desired).

Research Concepts:
Cannabinoid Toxicity

We update eBooks quarterly and Apps daily based on user feedback. Please tap flag to
report any questions that need improvement.
Question 310: A 48-year-old male with no significant past medical history came to the
emergency room with a chief complaint of palpitations. EKG showed atrial fibrillation with a
rapid ventricular rate. Intravenous diltiazem was given with improvement in rate control. The
urine drug screen was positive for amphetamines. Transesophageal echocardiogram showed no
atrial thrombus and electrical cardioversion was subsequently performed. After successful
cardioversion, sotalol was started. An EKG showed QT duration of 400 ms, potassium 4.2
mEq/L, and magnesium 2.3 mEq/L. How long should he stay in the hospital for monitoring?

Choices:
1. He can go home today after two hours after the first dose of sotalol
2. One day after staring sotalol if serial EKGs are normal
3. He cannot be discharged due to the positive drug screen
4. Three days monitoring is required in the hospital with serial EKGs
Answer: 4 - Three days monitoring is required in the hospital with serial EKGs
Explanations:
Sotalol is recommended younger patients post cardioversion for maintenance for sinus
rhythm due to its superior side effect profile as compared with amiodarone.
QT prolongation is one of the most important adverse effects of sotalol as it also has class
III effect along with beta-adrenergic blockade properties.
QT prolongation can lead to torsade de pointes which is a lethal arrhythmia if not treated.
To prevent these adverse events it is recommended to observe the patient in the hospital for
three days with serial EKGs to monitor QT.

Go to the next page if you knew the correct answer, or click the link image(s) below to further
research the concepts in this question (if desired).

Research Concepts:
Sotalol

We update eBooks quarterly and Apps daily based on user feedback. Please tap flag to
report any questions that need improvement.
Question 311: The patient is a 75-year-old male with a past medical history of
hypertension, diabetes mellitus, coronary artery disease status post coronary artery bypass grafts,
and history of inferior vena cava (IVC) filter placement who presents to the emergency
department with a chief complaint of shortness of breath. Physical examination reveals lungs that
are clear to auscultation bilaterally, cardiac exam shows regular rate and rhythm, normal S1, S2,
no murmurs. EKG was obtained showing sinus tachycardia. CT of the chest with contrast was
obtained showing bilateral pulmonary emboli. The patient was started on intravenous heparin.
What is the next step in management?

Choices:
1. Doppler ultrasound bilateral lower extremities
2. CT abdomen and pelvis
3. Abdominal ultrasound evaluating the IVC
4. MRI of abdomen and pelvis
Answer: 3 - Abdominal ultrasound evaluating the IVC
Explanations:
IVC thrombosis can be a bland thrombosis which is an isolated thrombus most commonly
arising from a deep venous thrombosis or due to a current IVC filter in place. IVC filter
thrombus can range from small, clinically insignificant to an extensive clot extending to the
lower extremities or causing complete occlusion of the IVC, ultimately leading to stasis. It
is important to determine if the IVC filter is a permanent or retrievable device. If it is
retrievable many times, intravascular thrombectomy will be performed, and the IVC filter
will be removed.
If the IVC filter thrombus is extensive, it can go on to cause the postthrombotic syndrome.
The patient will present with lower extremity pain and edema, claudication and stasis ulcers.
It can also progress to potential pulmonary emboli which could be fatal. There have been
studies where the IVC filters were retrieved, and thrombosis ranges from 0.6 to 8%. There
is thought that this percentage may be higher as this study was only on IVC filters that were
retrieved after one year, it did not assess the older permanent filters that were placed.
It is known that IVC filter placement is associated with increased risk for thrombus
development within the filter, ultimately leading to pulmonary emboli. The ultrasound
imaging will show echogenic material within its lumen, with an incomplete filling of color
when Doppler is applied to the vessel. Pharmacologic anticoagulation is normally the
mainstay treatment, but many patients with IVC filters are unable to have anticoagulation,
and therefore endovascular therapies are considered.
If there is a question on the extent of a clot within the filter, endovascular ultrasound can
better evaluate the degree of stenosis and thrombus size to help target management.
Endovascular management consists of the catheter directly thrombolysis or thrombectomy,
balloon angioplasty, and stent placement if possible removal of the filter is considered.

Go to the next page if you knew the correct answer, or click the link image(s) below to further
research the concepts in this question (if desired).

Research Concepts:
Inferior Vena Caval Thrombosis

We update eBooks quarterly and Apps daily based on user feedback. Please tap flag to
report any questions that need improvement.
Question 312: A 42-year-old male is admitted to the hospital for increasing shortness of
breath with palpitations. The patient requires supplemental oxygen at this time, and his chest x-
ray on admission is suspicious for bilateral alveolar infiltrates. On admission, it was also noted
that he had been admitted multiple times in the past for similar findings and symptoms. On his
last admission, he had undergone bronchoscopy with bronchoalveolar lavage which showed
hemosiderin filled alveolar macrophages. He was discharged home on a steroid taper. Now he
has been found to have new-onset atrial fibrillation with rapid ventricular response. His blood
pressure is 155/85 mmHg, heart rate is ranging 120s to 140s per minute, respiratory rate is
19/min, and his oxygen saturation is 98% on 2 L nasal cannula. Which of the following
medications has the strongest relative contraindication in this patient?

Choices:
1. Diltiazem
2. Amiodarone
3. Furosemide
4. Metoprolol
Answer: 2 - Amiodarone
Explanations:
Pulmonary hemosiderosis is typically grouped into three categories based on disease
characteristics. Group 1 pulmonary hemosiderosis is defined by pulmonary hemorrhage
associated with circulating anti-GBM antibodies. Group 2 pulmonary hemosiderosis is
defined by pulmonary hemorrhage with immune complex disease. Group 3 pulmonary
hemosiderosis (idiopathic pulmonary hemosiderosis [IPH]) is defined as pulmonary
hemorrhage without immunologic association. As previously noted, repeated episodes of
diffuse alveolar hemorrhage result in accumulation of iron in the form of hemosiderin inside
pulmonary macrophages. These recurrent episodes also lead to thickening of alveolar
basement membranes and interstitial fibrosis. Therefore, one can consider IPH is a
diagnosis of exclusion after having ruled out primary and secondary causes of pulmonary
hemosiderosis. Idiopathic pulmonary hemosiderosis most commonly occurs in children less
than ten years of age.
Due to the rare nature of IPH, the incidence and prevalence of which is relatively unknown.
Many patients previously reported to have IPH are likely misdiagnosed and have diffuse
alveolar hemorrhage.
Amiodarone should be avoided in these patients. Amiodarone is known to have various
pulmonary side effects. One of the more serious side effects of amiodarone is interstitial
lung disease, pulmonary fibrosis, and it has been associated with increased rates of diffuse
alveolar hemorrhage.
There has been no correlation with adverse effects of diltiazem, furosemide, or metoprolol
and IPH.

Go to the next page if you knew the correct answer, or click the link image(s) below to further
research the concepts in this question (if desired).

Research Concepts:
Idiopathic Pulmonary Hemosiderosis

We update eBooks quarterly and Apps daily based on user feedback. Please tap flag to
report any questions that need improvement.
Question 313: An 87-year-old female is admitted to the intensive care unit for severe sepsis
secondary to a urinary tract infection. Despite appropriate fluid resuscitation and antibiotic
therapy, she continues to be lethargic and is intubated due to her inability to maintain her airway.
The ventilator is set to a tidal volume of 360 mL (6 mL/kg), a rate of 12 breaths per min, an FiO2
of 50%, and a PEEP of 15. Soon after intubation, her blood pressure drops, and she requires the
initiation of inotropic agents. Which of the following maneuvers is most likely to help improve
her blood pressure?

Choices:
1. Place the patient in a prone position
2. Decrease PEEP to 5
3. Increase FiO2 to 100%
4. Increase the tidal volume to 8 mL/kg
Answer: 2 - Decrease PEEP to 5
Explanations:
Patients undergoing mechanical ventilation undergo a change in their respiratory
physiology. In normal respiration, negative intrathoracic pressure caused by contraction of
the diaphragm generates air entry into the lungs. At the same time, the negative pressure
increases venous return to the right atrium. This, in turn, increases the preload, stroke
volume, and cardiac output. In mechanically ventilated patients, this negative pressure is
turned into positive pressure through the ventilator. Positive intrathoracic pressure
effectively "squeezes" the right atrium, decreasing venous return and diminishing cardiac
output.
In our case of a septic patient, diminishing cardiac output by any minute margin can have
significant deleterious effects on blood pressure. The higher the PEEP, the higher the
intrathoracic pressure and the higher the effect on cardiac output.
Diminishing PEEP to a more physiologic level can help restore some of the cardiac output
and improve blood pressure in this patient.
Of the given options, only decreasing PEEP would have an effect on blood pressure. None
of the other displayed answers would have any effect on blood pressure.

Go to the next page if you knew the correct answer, or click the link image(s) below to further
research the concepts in this question (if desired).

Research Concepts:
Ventilator Management

We update eBooks quarterly and Apps daily based on user feedback. Please tap flag to
report any questions that need improvement.
Question 314: A 34-year-old male with no significant past medical history is involved in a
motor vehicle collision with a suspected cervical spine injury. The patient is intubated in the field
for respiratory failure and altered mental status and receives 1 liter of normal saline en route to
the hospital. The patient’s vitals upon arrival were a blood pressure of 106/68 mmHg, heart rate
74 beats/min, temperature 98.4F, SpO2 99% on room air, and respiratory rate 15. He was started
on a low-dose propofol infusion for sedation upon arrival. He was induced with 200 mg of
ketamine and paralyzed with 8 mg of vecuronium approximately 30 minutes ago. A primary
survey reveals no major injuries aside from some superficial abrasions and lacerations. While in
the CT scanner, the patient’s blood pressure begins to drop with a sustained mean arterial
pressure less than 60 mmHg. There are no other changes to the patient’s vital signs. Which of the
following describes the vasopressor of choice to increase this patient’s mean arterial pressure?

Choices:
1. It functions as both a hormone and neurotransmitter
2. It may be used as a decongestant
3. It functions mainly as a beta-1 agonist
4. It is a PDE3 inhibitor
Answer: 2 - It may be used as a decongestant
Explanations:
Patients with high cervical injuries can experience a host of hemodynamic changes,
including impairment of diaphragmatic and intercostal muscles resulting in respiratory
failure, as in this case. Additionally, sympathetic tone decreases and a state of vasodilatory
shock can ensue. It is imperative to ensure adequate resuscitation and initiate vasopressor
therapy to increase mean arterial pressure (MAP) and thus spinal perfusion pressure. Most
sources recommend MAPs greater than 85 mmHg, which in the setting of deep sedation or
general anesthesia, will often require vasopressor support.
Unlike septic shock, there is no clear consensus on the first-line or ideal vasopressor therapy
for neurogenic shock resulting in significant institutional and geographic variations. It is
pertinent to evaluate every patient individually and assess the primary underlying problem.
In this case, the patient is young, with presumably normal cardiac function, was given 1 liter
of fluids before arrival, and had no other major injuries. He has a suspected C-spine injury
with a normal heart rate. Most likely, his preload is adequate. His contractility should not be
impaired, but his afterload and systemic vascular resistance (SVR) are significantly reduced.
Choosing an agent like phenylephrine, an alpha-1 adrenergic agonist, would be ideal in
increasing SVR without unneeded ionotropic or chronotropic effects. It is also used as a
decongestant in various nasal sprays.
With the loss of cardiac accelerator fibers (T1-T4) in spinal trauma, it is possible to have a
bradycardic response. If the patient were bradycardic, phenylephrine would not be advised.
Reflex bradycardia is less likely to be seen in infusion dosing compared to bolus dosing.
It is difficult to discern the hypotensive contribution of the patient’s sedation. The question
was worded "low dose propofol infusion" to focus on the primary pathology, but propofol is
a profound vasodilator. The first step in any hypotensive crisis is to decrease or eliminate
anesthetic or sedative medications until the patient stabilizes. Because propofol also causes
hypotension by a reduction in SVR, phenylephrine remains the best choice listed.

Go to the next page if you knew the correct answer, or click the link image(s) below to further
research the concepts in this question (if desired).

Research Concepts:
Phenylephrine

We update eBooks quarterly and Apps daily based on user feedback. Please tap flag to
report any questions that need improvement.
Question 315: A 17-year-old male presents with fever and chills that were not relieved with
acetaminophen. According to parents, the fever started three days ago when the patient returned
from school. He was fine prior to the fever. One of the patient's friends had upper respiratory
tract infection the day he developed a fever. Past medical history is hemoglobin SC disease. He
is in no acute distress. Cardiac, lung, and abdominal exam are unremarkable. After two days of
hospitalization, the patient developed altered mental status and respiratory failure. He was
immediately started on mechanical ventilation and IV antibiotics. The patient died 48 hours later
due to cardiorespiratory failure. What would the peripheral blood of this patient show?

Choices:
1. Bite cells
2. Howell-Jolley bodies and sickle cells
3. Hairy cells and sickle cells
4. Sickle cells and bite cells
Answer: 2 - Howell-Jolley bodies and sickle cells
Explanations:
The peripheral smear of asplenic patients show Howell-Jolley bodies, pitted erythrocytosis,
monocytosis, and lymphocytosis.
After the peripheral blood shows these cells, spleen scintigraphy can be done to confirm the
diagnosis
If spleen scintigraphy is not available to confirm asplenia, sonogram of the abdomen, CT
abdomen, and MRI could be done.
Howell-Jolly bodies are present because the spleen has the inability to remove them.

Go to the next page if you knew the correct answer, or click the link image(s) below to further
research the concepts in this question (if desired).

Research Concepts:
Asplenia

We update eBooks quarterly and Apps daily based on user feedback. Please tap flag to
report any questions that need improvement.
Question 316: A 43-year-old male is brought in by emergency medical services to the
emergency department after being involved in the house fire. On initial exam, the patient has
deep, full-thickness burns covering approximately 50% of his body, including a circumferential
burn around his entire upper left arm. The patient also has singed nasal hairs, as well as
inspiratory and expiratory stridor. Which of the following is the most appropriate next step in
management?

Choices:
1. Start 2 liters of lactated Ringer's through peripheral IVs
2. Perform rapid sequence intubation and mechanical ventilation
3. Administer IV fentanyl for pain
4. Perform emergent escharotomy of the left upper extremity
Answer: 2 - Perform rapid sequence intubation and mechanical ventilation
Explanations:
This patient is exhibiting signs of inhalation injury and impending airway compromise.
Inhalation injury is a significant cause of mortality in critical burn patients and can lead to
severe laryngeal edema, pulmonary damage, and respiratory failure. Physical exam findings
include significant facial burns, singed nasal or eyebrow hair, soot around mouth and nose,
stridor, wheezing, cough, and hoarse or muffled voice. Early intubation for airway
protecting is the first step in management.
This patient has large, full-thickness burns of greater than 20% of his total body surface
area. Thus he will require aggressive fluid resuscitation during the first 24 hours of his care.
However, as airway compromise can quickly lead to respiratory failure, the patient’s airway
must be addressed before calculating fluid requirements. After the patient is intubated and
ventilated, he will require IV fluid resuscitation with lactated Ringer's calculated using the
Parkland formula.
If the setting of full circumferential burns, emergent escharotomy may be indicated if there
is evidence of vascular compromise to the limb, however, the patient’s airway, breathing,
and circulation must be managed first.
Although severe burns are extremely painful, dramatic, and distracting, the first steps in the
management of any critical burn patient must be to address the patient’s airway, breathing,
and circulation. Once this patient is intubated, he will require sedation and pain
management while on mechanical ventilation.

Go to the next page if you knew the correct answer, or click the link image(s) below to further
research the concepts in this question (if desired).

Research Concepts:
Parkland Formula

We update eBooks quarterly and Apps daily based on user feedback. Please tap flag to
report any questions that need improvement.
Question 317: A 27-year-old female with systemic lupus erythematosus on daily
prednisone presents to the emergency department with a 10-day history of headaches and low-
grade fever. Her head CT is normal, and a lumbar puncture shows a normal leukocyte count
(2/microliter) with mostly monocytes; high proteins (55 mg/dL); normal glucose (50 mg/dL);
and no erythrocytes. What test of the cerebrospinal fluid is most likely to provide a rapid
diagnosis?

Choices:
1. Cryptococcal antigen detected by latex agglutination
2. Cryptococcal culture
3. Gram staining
4. Immunofluorescence
Answer: 1 - Cryptococcal antigen detected by latex agglutination
Explanations:
The patient is immunosuppressed and the normal cerebrospinal fluid findings support the
investigation of cryptococcal meningitis. In one study of patients with AIDS, 26% of
patients with cryptococcal meningitis had normal CSF findings; 40% had high protein
levels, low glucose levels, and pleocytosis; and 55% had fewer than 10 lymphocytes/mL.
The CSF may have a clear or turbid appearance in cryptococcal meningitis. Variable
mononuclear pleocytosis is observed, and the white blood cell (WBC) count may be over 20
x 109/L. A high CSF opening pressure is present in about two-thirds of patients and is a
poor prognostic sign.
Rapid diagnosis is made by India ink stain, latex agglutination, or PCR. Cryptococcal
antigen is present several weeks before overt signs of meningitis develop; therefore, its
detection provides an opportunity to catch the infection early, and, hence, screening persons
with HIV for cryptococcal infection when they access health care can identify
asymptomatic infected patients, allowing for prompt treatment and prevention of death The
initial diagnostic sensitivity of cryptococcal CSF antigen is 94.1%, followed by the serum
antigen of 93.6%; however, this tool is unreliable in assessing the point of discontinuation
of antifungal therapy
Gram staining is valid for bacteria but does not provide a rapid diagnosis.
Immunofluorescence is a forgotten cryptococcal-diagnostic practice.

Go to the next page if you knew the correct answer, or click the link image(s) below to further
research the concepts in this question (if desired).

Research Concepts:
Cryptococcal Meningitis

We update eBooks quarterly and Apps daily based on user feedback. Please tap flag to
report any questions that need improvement.
Question 318: A 28-year old was admitted with sudden onset of breath while walking. He
is a heavy smoker. Chest x-ray reveals a 90% pneumothorax on the right side. The emergency
room resident placed a size 32F tube to evacuate the pneumothorax. An hour later the patient
complained of severe shortness of breath and his pulse oximeter revealed oxygen saturation of
89%. A portable chest x-ray revealed vascular congestion in the right lung. He was taken to the
intensive care unit and placed on the ventilator because of severe respiratory distress. Which of
the following is true about this condition?

Choices:
1. It only occurs in elderly people
2. The incidence is very high
3. It affects patients with extensive pneumothorax
4. It occurs only with exudate but not transudate effusion evacuations
Answer: 3 - It affects patients with extensive pneumothorax
Explanations:
Pulmonary re-expansion edema occurs when there is rapid emptying of the pleural cavity.
It has a low incidence but has a high mortality.
It can occur in people with effusions or lung collapse that has been present for more than 7
days.
When effusions of more than 3 liters are rapidly evacuated the condition can occur.

Go to the next page if you knew the correct answer, or click the link image(s) below to further
research the concepts in this question (if desired).

Research Concepts:
Pneumothorax

We update eBooks quarterly and Apps daily based on user feedback. Please tap flag to
report any questions that need improvement.
Question 319: A 64-year-old male with a long history of back problems was in the
swimming pool when he suddenly noticed that his legs had become weak. He struggled out of
the pool and called emergency medical services. In the emergency department, he reports that his
legs suddenly had no strength, and he was not able to move them. He denies any trauma, loss of
consciousness, or visual or speech loss. His only other complaint is an inability to pass urine. On
examination, he has significant motor weakness in both legs with numbness all the way from the
buttocks to the soles of the feet. He has no sensation in the perineal area and the rectal tone is
absent. A Foley catheter is placed and it puts out 800 ml of urine. Which of the following is best
for the management of this patient?

Choices:
1. CT scan of the head
2. Complete bed rest for 24 hours
3. Nerve conduction studies
4. Emergency surgery
Answer: 4 - Emergency surgery
Explanations:
This patient presents with acute symptoms of spinal cord compression. The features of
absent rectal tone and urinary retention, together with motor and sensory loss in the
extremities, suggest cauda equina syndrome. This is a medical emergency and a surgeon
should be notified as soon as possible.
The patient needs a quick MRI to decipher the site of compression or fracture. Surgery
should follow. This is a common scenario in males with prostate cancer metastases to the
vertebral column.
For chronic back pain with leg pain, numbness, or paresthesias bed rest with pain control is
recommended. When acute symptoms arise, such as loss of rectal tone, an acute
compression injury is treated with surgery rather than bed rest.
Nerve conduction studies are used in a chronic situation to decipher diseases affecting the
neuromuscular junction and to differentiate them from primary muscle myopathy or from
disorders due to denervation. Nerve conduction studies are not indicated in this patient,
whose etiology lies somewhere below L1-L2.

Go to the next page if you knew the correct answer, or click the link image(s) below to further
research the concepts in this question (if desired).

Research Concepts:
Cauda Equina And Conus Medullaris Syndromes

We update eBooks quarterly and Apps daily based on user feedback. Please tap flag to
report any questions that need improvement.
Question 320: A 25-year-old male is admitted to the medical intensive care unit after
presenting to the hospital with acute shortness of breath due to multifocal pneumonia. The
patient has a medical history of asthma and anxiety disorder, for which he is chronically on
inhaled corticosteroids and clonazepam therapy. He is in severe respiratory distress with oxygen
saturation 84% despite several nebulized bronchodilator therapies and 100% oxygen
supplementation. Etomidate and rocuronium are given for rapid sequence intubation.
Postintubation, his oxygen saturation improves to 95%. A chest x-ray confirms adequate
positioning of the endotracheal tube. Twenty minutes later, the patient is tachycardic and
hypertensive, with high pressures alarming on the ventilator. Oxygen saturation remains 95%.
What is the next step in the management of this patient?

Choices:
1. Start nicardipine
2. Start fentanyl
3. Start propofol
4. Start metoprolol
Answer: 3 - Start propofol
Explanations:
This patient is still most likely paralyzed from the rocuronium but without sedation as the
etomidate has most likely worn off by now.
Analgesia and sedation are paramount in the intubated patient. It ensures ventilator
synchrony, reduces patient agitation and improves outcomes.
Analgesia-first sedation should be provided. Fentanyl is a great first choice for the
mechanically ventilated patient due to its hemodynamic profile.
If analgesia sedation is insufficient then propofol or dexmedetomidine should be added.
Benzodiazepines should be avoided in most cases. However in patients chronically on
benzodiazepine therapy fentanyl would not prevent benzodiazepine withdrawal. Propofol
would be a better choice for this patient.

Go to the next page if you knew the correct answer, or click the link image(s) below to further
research the concepts in this question (if desired).

Research Concepts:
Mechanical Ventilation

We update eBooks quarterly and Apps daily based on user feedback. Please tap flag to
report any questions that need improvement.
Question 321: A 42-year-old-male was intubated the day before due to status epilepticus.
He is currently in a nursing home on synchronized intermittent mandatory ventilation. The
nursing home is employing state-of-the-art nursing interventions for liberating a patient from a
ventilator. Continuous assessment of what two items is commonly used, backed by evidence-
based bundle for nursing interventions?

Choices:
1. Pain and delirium
2. Sedation and antibiotic usage
3. Endotracheal tube placement and cuff pressure
4. Partial pressure of oxygen on a blood gas and fraction of inspired oxygen
Answer: 1 - Pain and delirium
Explanations:
The ABCDEF bundle was originally proposed by the Society of Critical Care Medicine in a
campaign to help wean and liberate patients from ventilators.
The bundle stands for: Assess, prevent, and manage pain; to have daily spontaneous
awakening and breathing trials; choice of analgesia and sedation; to assess, prevent, and
manage delirium; early mobility and exercise; and family engagement.
Closely monitoring pain and delirium will help the patient receive the maximum benefit of
mechanical ventilation while ensuring they are not in excruciating pain or too delirious to
participate in ventilator weaning protocols.
Sedation is part of the ABCDEF bundle, but antibiotics are not. Endotracheal tube
placement and cuff pressure can be part of a respiratory therapist's ventilator checklist but is
not a nursing intervention. PaO2 and FiO2 are the two variables required to diagnose acute
respiratory distress syndrome.

Go to the next page if you knew the correct answer, or click the link image(s) below to further
research the concepts in this question (if desired).

Research Concepts:
Synchronized Intermittent Mandatory Ventilation

We update eBooks quarterly and Apps daily based on user feedback. Please tap flag to
report any questions that need improvement.
Question 322: A 17-year-old male patient with a history of intravenous drug use complains
of a few days history of pain to his left arm, double vision, fever, and difficulty swallowing. He
is awake, alert, and oriented to person, place and time, and denies any headache or neck stiffness.
Exam shows a blood pressure of 110/81 mmHg, temperature of 99.3 F, heart rate of 110
beats/min, and respiratory rate of 18. Kernig and Brudzinski testing are within normal limits. A
large erythematous, warm, and indurated rash on the left arm is noted with several track marks
and pustular lesions. Cranial nerves II through XII are grossly intact, and the bladder is
distended. Which of the following is the next best step?

Choices:
1. CT head, lumbar puncture, antibiotics, admit to intensive care unit (ICU)
2. CT head, lumbar puncture, antibiotics, admit to floor
3. Antitoxin, antibiotics, admit to ICU
4. Antitoxin, antibiotics, admit to floor
Answer: 3 - Antitoxin, antibiotics, admit to ICU
Explanations:
The description is classic for wound botulism, most likely from Clostridium botulinum
cellulitis secondary to subcutaneous injection of spores from contaminated drugs.
Respiratory failure may occur suddenly, thus the admission to intensive care.
Botulism is seen in wounds, food, and in the infantile form.
Antitoxin should be administered as soon as botulism is suspected. Antibiotics for wound
botulism include penicillin or metronidazole if penicillin allergic.

Go to the next page if you knew the correct answer, or click the link image(s) below to further
research the concepts in this question (if desired).

Research Concepts:
Botulism

We update eBooks quarterly and Apps daily based on user feedback. Please tap flag to
report any questions that need improvement.
Question 323: A patient with cirrhosis of liver presents with anginal chest pain. A stress
test is positive. The patient undergoes cardiac catheterization, and a selective coronary
angiogram is negative for coronary artery disease. But the dyes injected into the right and left
coronary ostia filled the ventricles. Which of the following mechanisms is most likely
responsible for the patient's symptoms?

Choices:
1. Steal phenomenon
2. Increased oxygen demand due to chamber hypertrophy
3. Increased blood flow to ventricles
4. Due to hypoalbuminemia
Answer: 1 - Steal phenomenon
Explanations:
Patients with cirrhosis of the liver are prone to coronary cameral fistula due to
hyperestrogenic state, which leads to blood emptying into the veins or chambers without
perfusing capillaries. Impairment in supply-demand mismatch leads to anginal chest pain.
Chamber hypertrophy is not found in a cirrhotic patient unless they have concomitant
valvular heart disease.
Increased blood flow to ventricles leads to hypervolemia and congestive heart failure. It is
not known to cause angina.
Hypoalbuminemia leads to ascites but no angina.

Go to the next page if you knew the correct answer, or click the link image(s) below to further
research the concepts in this question (if desired).

Research Concepts:
Coronary Cameral Fistula

We update eBooks quarterly and Apps daily based on user feedback. Please tap flag to
report any questions that need improvement.
Question 324: A 65-year-old patient has been admitted to the intensive care unit (ICU) for
the management of acute respiratory failure needing intubation related to chronic obstructive
pulmonary disease (COPD) exacerbation. The following morning the patient has been on
spontaneous weaning trial for 4 hours since. The patient is getting very restless and anxious and
pointing to his endotracheal tube (ETT) and wants to take it out. The patient is sitting upright,
awake, alert and pointing to his ETT. His vital signs are BP 150/95 mmHg, HR 110 bpm, RR
26/minute, and SpO2 99%. He is on 25% FiO2. His vent screen showed VT in the range of 500-
600, RR 20-26, PEEP 5 and pressure support of 5. He has no respiratory distress. The patient has
a nasogastric tube with tube feeding has been running at 65 cc/hour. What is the best strategy for
this patient to be removed from the mechanical ventilator?

Choices:
1. Place back on full support for now. Hold tube feeds at midnight and then consider extubation
if the patient able to pass SBT.
2. Go ahead and extubate patient while tube feeding is going at the current rate.
3. Hold tube feeding now and extubate one hour later.
4. suction all residual gastric contents through NG tube and proceed with extubation.
Answer: 4 - suction all residual gastric contents through NG tube and proceed with
extubation.

Explanations:
The concern about extubation is tube feeding. There is no other contraindication so delaying
extubation for another day will prolong ventilator days and ICU days. The patient is already
very anxious and may need to be sedated again which can prolong ventilator days even
more. Extubation should not be delayed if the patient has passed SBT.
It is possible that patient can be extubated with tube feeds going, and he may do just fine,
but there is some risk of failure of extubation trial and need for reintubation. In that
situation, the stomach should be empty. Also, the patient is awake, and he may not need
ongoing tube feeding as he may be able to take oral nutrition very soon.
Ideally, tube feeding should be held when a patient appears to do well on a spontaneous
breathing trial (SBT). For another patient, it may have been ok to delay extubation for one
more hour while holding tube feeding. But this patient has been on an SBT for 4 hours now
which has led to anxiety and frustration. Extubation should not be delayed any further.
The best strategy would be to suction all tube feed contents from the stomach and then
proceed with extubation. It is possible as the patient has an NG tube. It may not have been
possible in the presence of nasoduodenal or nasojejunal tube, but in that case, the stomach
may not have many contents either. Before extubation, always consider enteral access needs
for ongoing care. If the patient is likely to pass swallow, remove the NG tube and start a
diet. If the patient is at risk for failure, keep the NG tube and do a swallow test and the
decide about an ongoing need for the tube. For high-risk patient for extubation trial failure,
be very careful about advancing the diet.

Go to the next page if you knew the correct answer, or click the link image(s) below to further
research the concepts in this question (if desired).

Research Concepts:
Extubation

We update eBooks quarterly and Apps daily based on user feedback. Please tap flag to
report any questions that need improvement.
Question 325: A 52-year-old male involved in a motor vehicle collision presents in the
trauma bay with no complaints. He is awake, alert, Glasgow coma scale 15, hemodynamically
stable with no abnormalities noted on primary and secondary survey. ECG was noted to have
premature ventricular contractions. What is the next best course of action?

Choices:
1. Discharge the patient home with cardiology follow up
2. Admit the patient to the hospital for 24 to 48 hours of cardiac monitoring
3. Obtain a cardiology consult STAT
4. Discharge the patient home
Answer: 2 - Admit the patient to the hospital for 24 to 48 hours of cardiac monitoring
Explanations:
Patients with abnormal ECG and/or troponin I (cTnI) levels should be admitted for cardiac
monitoring for 24 to 48 hours because life-threatening arrhythmias or cardiac failure will
present during this time.
Patients can be admitted to the intensive care unit or under telemetry depending on
concurrent injuries, type of ECG changes, and the grade of hemodynamic imbalance.
The subset of patients with isolated ECG and cTnI elevations are the most prevalent of the
blunt cardiac injury (BCI) and usually have a benign course that usually resolves in a few
hours and with rare occurrence of long-term functional impairment.
Management of dysrhythmias should be approached and treated as in the patient without
BCI. Replete electrolytes accordingly, avoid hypoxia and acidosis, and utilization of
antidysrhythmics and defibrillation if needed.

Go to the next page if you knew the correct answer, or click the link image(s) below to further
research the concepts in this question (if desired).

Research Concepts:
Blunt Cardiac Injury

We update eBooks quarterly and Apps daily based on user feedback. Please tap flag to
report any questions that need improvement.
Question 326: A 35-year-old woman presents to the hospital complaining of abdominal
pain, bloody diarrhea, and fever of 2 days duration. The patient is known to have systemic lupus
erythematosus with renal involvement, and she is on immunosuppressive therapy. She has
known penicillin allergy. On physical exam, her temperature was 39.5 C (103.1 F), blood
pressure 100/70 mmHg, and pulse rate 112/min. The patient looks dehydrated and sick.
Abdominal exam shows diffuse tenderness that is more prominent on the left lower quadrant.
Her labs show hemoglobin (Hb) 9 mg/dL, white blood cells (WBC) 500/microL, and platelets
90000/microL. CT scan shows bowel wall thickening with fat stranding. No air under the
diaphragm is noted. What is the next best step in the management of this patient?

Choices:
1. Piperacillin/ tazobactam
2. Vanco + ceftriaxone + metronidazole
3. Piperacillin/ tazobactam + vancomycin
4. Ceftriaxone + vancomycin
Answer: 2 - Vanco + ceftriaxone + metronidazole
Explanations:
The patient presented with a clinical picture of neutropenic enterocolitis confirmed by
imaging findings.
Although there is no consensus on the best treatment of NE, it is usually treated by
piperacillin/ tazobactam to cover for bacteria originating from the bowels (gram-negative
and anaerobic bacteria) in addition to IV vancomycin in sick or hemodynamically unstable
patient to include gram-positive bacteria coverage.
However, the patient is allergic to penicillins and cannot be given piperacillin/ tazobactam.
So of the choices listed in the question, the most appropriate choice with a spectrum that
covers both gram-negative bacteria and anaerobes is ceftriaxone + metronidazole along with
vancomycin.

Go to the next page if you knew the correct answer, or click the link image(s) below to further
research the concepts in this question (if desired).

Research Concepts:
Neutropenic Enterocolitis (Typhlitis)

We update eBooks quarterly and Apps daily based on user feedback. Please tap flag to
report any questions that need improvement.
Question 327: A 33-year-old alcoholic drank some bootlegged alcohol and within a few
hours presented to the emergency department with epigastric pain, nausea, and vomiting. He also
complains of blurred vision that just started. Blood work reveals a CBC 4.5, WBC 9,
Hemoglobin 11, Cr 1.2, BUN 33, Na 138, K 4.8, glucose 150 mg/dl elevated levels of amylase.
Arterial blood gas reveals a pH of 7.23, PO2 77, PCO2 37, HCO3 17. His calculated osmolar gap
was increased. Which gastrointestinal problem is he likely to develop?

Choices:
1. Gastric perforation
2. Mesenteric ischemia
3. Hemorrhagic pancreatitis
4. Small bowel obstruction
Answer: 3 - Hemorrhagic pancreatitis
Explanations:
Hemorrhagic pancreatitis has been described in as many as two-thirds of the patients with
methanol poisoning.
Even though blindness is of major concern, most people forget about the potential
abdominal problems like hemorrhagic pancreatitis that can be severe.
Hemorrhagic pancreatitis often presents with bruising and superficial edema in the
subcutaneous fatty tissue around the umbilicus. This is Cullen sign.
The treatment is the same as for regular methanol toxicity- this includes the use of
fomepizole and hydration. Pancreatic hemorrhage is more likely to occur in patients with
chronic pancreatitis or alcohol abuse.

Go to the next page if you knew the correct answer, or click the link image(s) below to further
research the concepts in this question (if desired).

Research Concepts:
Methanol Toxicity

We update eBooks quarterly and Apps daily based on user feedback. Please tap flag to
report any questions that need improvement.
Question 328: A 55-year-old male in septic shock secondary to pneumonia is brought to
the intensive care unit (ICU) for further stabilization. A right internal jugular central line was
placed in the emergency department and started on pressor support. A portable chest x-ray shows
the line terminating 3 cm above the junction of the superior vena cava and the right atrium. The
patient is hemodynamically unstable at this time. What should be done about the current
placement of the central line?

Choices:
1. Carefully clip the sutures and advance the central line 3 cm into the internal jugular (IJ) vein
then resuture the central line into place
2. Leave the line in place as it is still in the internal jugular vein and is able to draw and flush
3. After donning appropriate attire, sterile gloves, and using sterile technique, clip the sutures and
advance the central line 3 cm further into the IJ then resuture the line into place
4. Ask the emergency room doctor to fix the line that they placed
Answer: 2 - Leave the line in place as it is still in the internal jugular vein and is able to draw
and flush

Explanations:
Advancing a central line without the use of proper sterile technique places the patient at risk
of developing a catheter-based infection such as cellulitis, abscess, or bacteremia.
The correct placement of an internal jugular central line tip is in the superior vena cava or at
the cavo-atrial junction (i.e., at the level of 1st anterior intercostal space above carina). This
can best be visualized and confirmed with a portable chest x-ray. This line can be used
without being adjusted. If needed the line can be changed over a guidewire, or a new line
may be inserted at a different site.
At this time it would be unsafe to transport the patient down to the emergency department to
ask the performing physician to correct the placement. Also, the patient is now under the
care of the ICU, and the correction can be simply made with appropriate sterile technique.
After placing internal jugular or subclavian central lines, portable chest x rays should
always be performed immediately after to ensure correct placement and absence of
pneumothorax.

Go to the next page if you knew the correct answer, or click the link image(s) below to further
research the concepts in this question (if desired).

Research Concepts:
Central Line Management

We update eBooks quarterly and Apps daily based on user feedback. Please tap flag to
report any questions that need improvement.
Question 329: Following a motor vehicle accident, a 35-year-old male is agitated and has
multiple cerebral contusions on CT of the head. Hemodynamic measurements show a heart rate
of 85 beats/min and a mean arterial pressure (MAP) of 84 mmHg. An intracranial pressure
monitor reveals a pressure of 28 mmHg. What is the most appropriate management regimen?

Choices:
1. Hyperventilation to maintain a cerebral PCO2 of 25 to 30 mmHg
2. Administration of phenylephrine to increase MAP
3. Administer hypertonic saline
4. Placement of patient in Trendelenburg position
Answer: 3 - Administer hypertonic saline
Explanations:
Cerebral perfusion pressure (CPP) is the difference between mean arterial pressure (MAP)
and intracranial pressure (ICP). It should be greater than or equal to 70 mmHg. ICP should
be 20 mmHg or less.
Methods for reducing ICP include elevation of the head of the bed (reverse Trendelenburg),
administration of hypertonic saline, mannitol, other diuretics, sedatives to control agitation,
prevention of hypovolemia, maintenance of adequate CPP, and keeping pCO2 in the proper
range.
Hypertonic saline has been shown to be superior to mannitol in patients with hypotension
severe brain trauma. The patient also will require aggressive fluids management.
Hypercapnia causes vasodilation of the cerebral vessels, which increases intracranial
volume and pressure. Hyperventilation may be used for impending herniation to acutely
lower ICP. However, prolonged hyperventilation decreases perfusion secondary to the
vasoconstriction that occurs in an injured, ischemic brain.

Go to the next page if you knew the correct answer, or click the link image(s) below to further
research the concepts in this question (if desired).

Research Concepts:
Increased Intracranial Pressure

We update eBooks quarterly and Apps daily based on user feedback. Please tap flag to
report any questions that need improvement.
Question 330: A 16-year-old white female presents to the emergency department with
acute severe worsening of her asthma symptoms that would not respond to home inhalers over
the past 6 hours. Medical history is significant for anxiety and bipolar disorder, irritable bowel
syndrome, hypothyroidism, GERD and a recent diagnosis of bronchial asthma. This is her 4th
episode of severe asthma exacerbation in the past 4 months. Each one of them required
endotracheal intubation. She also gave birth to a healthy boy by a caesarian section last year.
Symptoms escalate rapidly in requiring intubation again. Her bloodwork shows an absolute
eosinophil count of 320/microL. Alarmed by these recurrent episodes of severe asthma, a CT
angiogram of the chest is ordered as well. It does not show pulmonary embolism or chronic
bronchiectasis. What further investigations may be most helpful in treating her condition?

Choices:
1. Send aspergillus specific antibody and IgE level
2. Send 5-hydroxy indole acetic acid level and chromogranin assay
3. Perform bronchoscopy when extubated
4. Speech therapy evaluation once extubated
Answer: 3 - Perform bronchoscopy when extubated
Explanations:
Recurrent status asthmaticus requiring and reversed by endotracheal intubation should
always raise concern for tracheal pathologies like tracheomalacia or excessive dynamic
airway collapse (EDAC) which can be diagnosed by evaluation of upper airway post-
extubation.
Risk factors for EDAC include prior intubation, GERD, chronic respiratory inflammation,
and thyroid disease.
CT scan may appear normal particularly if done in the inspiratory phase. Treatment once
diagnosed may involve placement of a tracheal stent.
Evaluation by speech therapy can be effective for pure vocal cord dysfunction once tracheal
collapse or tracheomalacia has been ruled out by airway evaluation.

Go to the next page if you knew the correct answer, or click the link image(s) below to further
research the concepts in this question (if desired).

Research Concepts:
Status Asthmaticus

We update eBooks quarterly and Apps daily based on user feedback. Please tap flag to
report any questions that need improvement.
Question 331: A 67-year-old female presents with abdominal pain that occurs minutes to a
few hours after every meal. This has been going on for the past 2 months, and she has lost about
15 pounds. She has become afraid to eat. Also, she has a history of a stroke. The physical exam
is unremarkable, and the blood work is normal. What is the most likely condition?

Choices:
1. Abdominal aortic aneurysm
2. Chronic mesenteric ischemia
3. Gallstones
4. Chronic pancreatitis
Answer: 2 - Chronic mesenteric ischemia
Explanations:
Mesenteric ischemia is a rare disorder but carries high mortality if the diagnosis is missed.
Chronic mesenteric ischemia typically presents with postprandial abdominal pain and
weight loss. Patients also may develop a fear of eating.
Patients often have a history of vascular disease that involves other organs including
cerebral vascular disease, coronary artery disease, or peripheral vascular disease.
Arteriography is the gold standard for making the diagnosis. Open or endovascular
revascularization is the treatment of choice.

Go to the next page if you knew the correct answer, or click the link image(s) below to further
research the concepts in this question (if desired).

Research Concepts:
Chronic Mesenteric Ischemia

We update eBooks quarterly and Apps daily based on user feedback. Please tap flag to
report any questions that need improvement.
Question 332: A 65-year-old male smoker with a past medical history of chronic
obstructive pulmonary disease (COPD) presents with worsening shortness of breath and cough
with expectoration. On examination, he is awake but in severe distress and using accessory
muscles. Chest auscultation reveals prolonged expiration and bilateral wheezes. Arterial blood
gas analysis reveals a pH of 7.20, pCO2 of 72 mmHg, and pO2 of 50 mmHg. Pulmonary
function tests show FEV1/FVC 60% of predicted. Chest x-ray shows flattened diaphragm and
widened intercostal spaces. Inhaled short-acting beta-agonists, steroids, oxygen, and non-
invasive ventilation with Bilevel positive airway pressure are tried, but the condition does not
improve. The patient is intubated, and on day three does well on a spontaneous breathing trial.
What is the best next step in the management of this patient?

Choices:
1. Extubate to nasal cannula
2. Room air
3. Tracheostomy and percutaneous endoscopic gastrostomy
4. Extubate to bilevel positive airway pressure
Answer: 4 - Extubate to bilevel positive airway pressure
Explanations:
Prophylactic non-invasive ventilation like bilevel positive airway pressure (BiPAP)
following extubation has been shown to decrease the reintubation rate in chronic respiratory
disease especially in hypercarbic chronic obstructive pulmonary disease (COPD) patients
who were mechanically intubated for respiratory failure. Hypercapnia in this patient points
towards the chances of extubation failure. To prevent extubation failure, this patient should
be extubated to BiPAP.
The early use of non-invasive ventilation (NIV) post-extubation also has reduced hospital
mortality in such high-risk intensive care unit patients.
Noninvasive ventilation post-extubation in mechanically ventilated patients with COPD has
also has shown to reduce the duration of mechanical ventilation, length of intensive care
unit stay and, decreased incidence of ventilator-associated pneumonia.
Since the mechanically ventilated patients with COPD has passed the spontaneous breathing
trial and is hypercapnic, the best next step is extubating to non-invasive ventilation. NIV
prevents acute respiratory failure and decreases mortality in such patients. With BiPAP,
such patients are likely to show a higher PaO2, lower PaCO2, respiratory rate, especially
when compared with patients using an oxygen mask.

Go to the next page if you knew the correct answer, or click the link image(s) below to further
research the concepts in this question (if desired).

Research Concepts:
Emphysema

We update eBooks quarterly and Apps daily based on user feedback. Please tap flag to
report any questions that need improvement.
Question 333: A 60-year-old male with a past medical history significant for hypertension
and diabetes mellitus presents for a physical examination. He complains of some shortness of
breath with walking at a moderate pace and while he is mowing his lawn. On physical
examination, his blood pressure is 160/60 mm Hg with a heart rate of 84 bpm. The cardiac exam
reveals two murmurs. The first is a decrescendo, blowing diastolic murmur at the 3rd intercostal
space at the sternal border. The second is a diastolic murmur at the apex and is low pitched and
rumbling. Carotid pulses are bounding. What ECG finding is consistent with the underlying
diagnosis?

Choices:
1. ST depression in leads II, III, and AVF
2. Q waves in the anterior leads
3. LVH with strain
4. Biphasic P wave
Answer: 3 - LVH with strain
Explanations:
This patient with wide pulse pressure has a murmur consistent with chronic aortic
regurgitation (AR).
In AR, a high frequency, blowing, decrescendo, diastolic murmur is heard best third
intercostal space along the left sternal border. The low pitched rumbling mid-diastolic
murmur heard best at the apex is thought to be caused by premature closure of the mitral
valve due to the jet of AR.
AR can cause LVH over time, which can be seen on an ECG.
The test of choice to assess aortic regurgitation is an echocardiogram.

Go to the next page if you knew the correct answer, or click the link image(s) below to further
research the concepts in this question (if desired).

Research Concepts:
Aortic Regurgitation

We update eBooks quarterly and Apps daily based on user feedback. Please tap flag to
report any questions that need improvement.
Question 334: A 37-year-old male crashes his motorcycle at a very high speed and strikes
his chest into a guardrail. Emergency medical services (EMS) have arrived on the scene and have
found the patient to be in cardiac arrest and begin advanced cardiac life support, including
cardiopulmonary resuscitation (CPR). The emergency medical services crew also performed
endotracheal intubation, bilateral chest decompression, and established humeral intraosseous
access with intravenous crystalloid resuscitation. The patient now arrives in the trauma bay, and
EMS reports the patient is in asystole for downtime of approximately 10 minutes with signs of
life initially present in the field. Which of the following is the most appropriate management at
this time?

Choices:
1. Give an additional dose of intravenous epinephrine and electrically defibrillate the patient
2. Place an ultrasound on the patient's chest and if no cardiac motion is identified cease
resuscitative efforts
3. Perform an emergency department resuscitative thoracotomy with aortic cross-clamping and
internal cardiac massage
4. Initiate the massive-transfusion protocol and send emergently to the operating room
Answer: 3 - Perform an emergency department resuscitative thoracotomy with aortic cross-
clamping and internal cardiac massage

Explanations:
Patients who present in traumatic cardiac arrest are managed differently with respect to the
injury mechanism. Blunt trauma often leads to uncontrollable exsanguination, severe
hypoxia, and ventilatory derangements or catastrophic neurologic injuries which often lead
to the death of patients in the field. The advanced cardiac life support (ACLS) and advanced
trauma life support (ATLS) courses promulgated by the American Heart Association and
the American College of Surgeons teach the 5 H's and T's (hypoxia, hypovolemia,
hyperkalemia, hydrogen ions, hypothermia, tension pneumothorax, thrombosis, toxins,
thromboembolism, tamponade) for potentially reversible causes of asystole. If not identified
and treated early and return of spontaneous circulation is restored, patients have a very
dismal prognosis. Unfortunately, blunt traumatic injuries that lead to traumatic cardiac
arrest are often not amenable to these treatments.
A patient who presents in traumatic cardiac arrest from a blunt mechanism of injury who
arrives with less than 10 minutes of known downtime with confirmed signs of life in the
field would be a candidate for an emergency department resuscitative thoracotomy.
For blunt traumatic arrest, total downtime with CPR of 10 minutes or less, and signs of life
noted in the field, an emergency department resuscitative thoracotomy (EDT) could be
performed. The first step is to perform a left-sided thoracotomy from the lateral sternal
border to the posterior thorax as far as possible in the 4-6th intercostal space. The chest is
then opened with a retractor exposing the lung. Blood and air are evacuated from the pleural
cavity. The lung is swept out of the way anteriorly by first taking down the inferior
pulmonary ligament, ensuring not to damage the inferior pulmonary vein, which lies just
superiorly. Afterexposing the heart, internal cardiac massage is performed while further
resuscitation is performed to perfuse the heart and brain. If the patient has a return of
spontaneous circulation, they are often brought to the operating room for definitive repair of
their injuries.
Additional doses of epinephrine may be given to a patient in asystole from a blunt
mechanism of injury, but defibrillation is not warranted unless in ventricular fibrillation or
ventricular tachycardia without a pulse. Placement of an ultrasound probe on the patient's
chest would be prudent to cease resuscitative efforts if the patient does not meet the strict
criteria to perform an EDT.

Go to the next page if you knew the correct answer, or click the link image(s) below to further
research the concepts in this question (if desired).

Research Concepts:
Asystole
We update eBooks quarterly and Apps daily based on user feedback. Please tap flag to
report any questions that need improvement.
Question 335: A 40-year-old male with a history of alcohol use disorder is admitted to the
hospital for skin lacerations following a motor vehicle accident. Urgent care is given with
intravenous fluids and sutures where required. On physical examination, the blood pressure is
130/82 mmHg. The pulse is 86/min, respiratory rate is 18/min, and temperature is 37 C (98.7 F).
After 5 hours, the nurse goes to draw blood, and the patient refuses, citing restlessness and
agitation. The patient yells and states that his heart is pounding and he would not let anyone
come close to him. He has a previous history of hepatitis C that has been treated. What is the
next step in management?

Choices:
1. Administer haloperidol
2. Administer oxazepam
3. Administer lactulose
4. Administer diazepam
Answer: 2 - Administer oxazepam
Explanations:
Benzodiazepines bind as an allosteric modulator and increase the number of chloride ions
crossing the cell membrane. Once benzodiazepines bind to the receptor, the gamma-
aminobutyric acid (GABA) receptor changes conformation and thereby begins to have an
increased affinity for GABA.
Oxazepam is an FDA-approved benzodiazepine used for the treatment of alcohol
withdrawal, management of anxiety disorders, and agitation.
The convention on psychotropic substances classifies oxazepam as a schedule IV substance,
meaning that oxazepam has a lower potential for abuse relative to substances in schedule III
substances (ex. ketamine).
For alcohol withdrawal in adults, administer oxazepam 15 mg PO TID. Diazepam should
not be used in this patient because it is metabolized in the liver. Oxazepam, on the other
hand, is excreted through the kidneys.

Go to the next page if you knew the correct answer, or click the link image(s) below to further
research the concepts in this question (if desired).

Research Concepts:
Oxazepam

We update eBooks quarterly and Apps daily based on user feedback. Please tap flag to
report any questions that need improvement.
Question 336: A 36-years-old female presents to the hospital for back pain and fever.
Symptoms started four days ago and are getting progressively worse. She was seen in an urgent
care center and was given pain medication and antibiotics with no improvement. She had a skin
infection some three weeks ago that lasted a few days but disappeared without treatment.
Otherwise, she is healthy and takes no medication. Physical exam shows enderness over the
thoracic spine, numbness in both thighs and low-grade fever. Her blood count shows
leukocytosis. Blood cultures are drawn. The emergency department does not have the capability
for emergency MRI. Which of the following is the next best step in the management of this
patient?

Choices:
1. MRI of the spine with contrast to be done outpatient for better sensitivity and wait for results
of culture to be available
2. CT of the spine with contrast followed by neurosurgical evaluation; start antibiotics
(vancomycin and cefotaxime)
3. CT scan of the spine without contrast followed neurosurgical evaluation; start antibiotics
(vancomycin and ceftriaxone)
4. CT scan of the spine with contrast followed by neurosurgical evaluation; start antibiotics
(vancomycin and piperacillin/tazobactam)
Answer: 2 - CT of the spine with contrast followed by neurosurgical evaluation; start
antibiotics (vancomycin and cefotaxime)

Explanations:
Spinal epidural abscess (SEA) is a true medical emergency. 1/3 of patients have no clear
etiology for acquiring the infection. Of the other 2/3, the more common site of origin is skin
infection followed by complications of spinal surgery.
Staphylococcus is the organism in the majority of cases followed by gram-negative bacilli.
CT and MRI with contrast are needed for SEA diagnosis. MRI is preferred, but if not
immediately available, CT is a feasible alternative.
Vancomycin plus a later generation cephalosporin (cefotaxime, ceftriaxone, cefepime) is the
empiric antibiotic treatment of choice. If Pseudomonas is suspected (prior surgery or
invasive procedures) then ceftazidime is preferred.

Go to the next page if you knew the correct answer, or click the link image(s) below to further
research the concepts in this question (if desired).

Research Concepts:
Epidural Abscess

We update eBooks quarterly and Apps daily based on user feedback. Please tap flag to
report any questions that need improvement.
Question 337: A 16-year-old patient returns home after traveling to Africa and is seen in
the emergency department. His vitals reveal a blood pressure of 90/60 mmHg, respiratory rate of
22, pulse of 43 beats/min, and temperature 103 degrees F. He appears slightly jaundiced and has
numerous petechiae. The patient may have which of the following conditions?

Choices:
1. Tuberculosis
2. Yellow fever
3. Leishmaniasis
4. Lymphoma
Answer: 2 - Yellow fever
Explanations:
With fever, most people develop tachycardia.
In yellow fever, the patient develops bradycardia.
A fever with bradycardia is known as Faget sign and is often seen in yellow fever.
The diagnosis requires a thorough travel history and record of immunizations. Patients may
present with headache, malaise, and myalgia. A physical exam may reveal Faget sign or
pulse fever dissociation, facial flushing, and conjunctival injection. Blood work may reveal
leukopenia with elevated transaminase levels. If the liver is involved, a coagulation profile
may be abnormal. Yellow fever can be diagnosed using enzyme-linked immunosorbent
assay (ELISA) and polymerase chain reaction assay. Other investigations depend on what
organ is involved. If there is evidence of altered mentation, a lumbar puncture and a CT
scan are performed.

Go to the next page if you knew the correct answer, or click the link image(s) below to further
research the concepts in this question (if desired).

Research Concepts:
Yellow Fever

We update eBooks quarterly and Apps daily based on user feedback. Please tap flag to
report any questions that need improvement.
Question 338: An unidentified patient is brought to the nearest rural hospital following a
traumatic car accident. An emergent exploratory laparotomy for hypovolemic shock is performed
via a predominantly halothane based anesthetic plan. On day 11 of hospitalization, he reports
jaundice, abdominal swelling, nausea, vomiting, malaise, and confusion. The following labs
were obtained: Complete blood count reveals hematocrit levels of 42 %, platelets are
160,000/mm3, and leukocytes are 9000/mm3. Serum chemistry shows blood urea nitrogen 32
mg/dL and creatinine 1.9 mg/dL. Liver function studies reveal total bilirubin 3.3 mg/dL, alkaline
phosphatase 220 U/L, aspartate aminotransferase 3207 U/L, alanine aminotransferase 4180 U/L
and INR 1.6 (normal 0.8-1.2). What is the next most appropriate step in the management of this
patient?

Choices:
1. Reassurance
2. NPO and IV fluids
3. Emergent liver transplant
4. Antibiotics and surgical exploration
Answer: 3 - Emergent liver transplant
Explanations:
This is a case of type 2 hepatotoxicity based on timing and severity. Type 1 is the self
remitting variation of halothane toxicity.
In mild forms of liver injury, supportive therapy is the correct answer. However, this patient
is in acute liver failure indicated by encephalopathy (confusion), LFT's >1000, and liver
synthetic dysfunction (abnormal INR).
A liver transplant is needed for acute liver failure to prevent death.
Even with a liver transplant, the mortality rate is over 50% for type two hepatotoxicity with
acute liver failure.

Go to the next page if you knew the correct answer, or click the link image(s) below to further
research the concepts in this question (if desired).

Research Concepts:
Halothane Toxicity

We update eBooks quarterly and Apps daily based on user feedback. Please tap flag to
report any questions that need improvement.
Question 339: A 66-year-old male patient is currently in a hospital recovering from a
cardiac revascularization procedure. He reports that he suddenly feels unusually weak and
fatigued, even when considering his recent operation. The patient has had obesity for
approximately 25 years and was diagnosed with type 2 diabetes 12 years ago, which is managed
with metformin. Recently, the patient was diagnosed with ischemic heart disease, which required
a revascularization procedure. The patient has no known allergies or hypersensitivities to any
medications. Physical examination reveals a substantial decrease in the patient’s cardiac output
and that the patient is being impacted by, for the first time, congestive heart failure. The patient
has not yet received any treatment for his symptoms of congestive heart failure. Treatment with
inamrinone is being considered. Which of the following is the most appropriate strategy for
treating the patient with this medication?

Choices:
1. The clinician should initially administer digitalis, diuretics and/or vasodilators before
administering inamrinone
2. The clinician should, initially, administer undiluted inamrinone at 0.75 mg/kg over 2 to 3
minutes
3. The clinician should, initially, administer diluted (with 0.9% saline) inamrinone at a dose of 5
to 10 mcg/kg/min
4. The clinician should, initially, administer diluted (with a dextrose [glucose]-containing
solution) inamrinone at a dose of 5 to 10 mcg/kg/min
Answer: 1 - The clinician should initially administer digitalis, diuretics and/or vasodilators
before administering inamrinone

Explanations:
The administration of inamrinone is only indicated if the patient's cardiovascular function
has not initally responsed positively to the administration of digitalis, diuretics and/or
vasodilators.
A loading dose of inamrinone at 0.75 mg/kg over 2 to 3 minutes will next be indicated if
digitalis, diuretics and/or vasodilators have failed to sufficiently address the symptoms of
congestive heart failure.
A maintenance dose of inamrinone, diluted with 0.9 or 0.45% saline, at 5 to 10 mcg/kg/min
is only administered after the loading dose of inamrinone.
Inamrinone should never be diluted with a dextrose [glucose]-containing substance due to
the chemical reaction that can occur.

Go to the next page if you knew the correct answer, or click the link image(s) below to further
research the concepts in this question (if desired).

Research Concepts:
Inamrinone

We update eBooks quarterly and Apps daily based on user feedback. Please tap flag to
report any questions that need improvement.
Question 340: A 59-year-old man, with the renal transplant, comes to the emergency
department complaining of abdominal cramps, diarrhea, and nausea from 3 days. Today he also
developed moderate fever and decreased appetite. The diarrhea is non-bloody and non-fatty. He
is on prophylactic antibiotics for prevention of opportunistic infections. The temperature is 101.4
F, blood pressure is 100/65 mmHg, and pulse is 100/min. On complete blood count, his white
blood cell count is 18,000/microL, serum creatinine is 2 mg/dL, and serum albumin level is
4g/dL. Stool comes positive for Clostridium difficile toxin. Doctors started oral vancomycin and
after which ileus developed. What is the next step in management?

Choices:
1. Add oral metronidazole
2. Switch to intravenous metronidazole
3. Add intravenous metronidazole
4. Switch to rectal metronidazole
Answer: 3 - Add intravenous metronidazole
Explanations:
This patient's presentation is consistent with Clostridium difficile colitis. Advance age and
antibiotic use are the risk factors he has been for C. difficile colitis.
For mild C. difficile colitis oral metronidazole is used.
In severe cases, oral vancomycin is preferred.
In severe Clostridium difficile colitis, after oral vancomycin, if ileus develops there are two
options. Add intravenous metronidazole or switch to rectal vancomycin.

Go to the next page if you knew the correct answer, or click the link image(s) below to further
research the concepts in this question (if desired).

Research Concepts:
Bacterial Gastroenteritis

We update eBooks quarterly and Apps daily based on user feedback. Please tap flag to
report any questions that need improvement.
Question 341: A 70-year-old woman complains of difficulty breathing towards the end of
receiving 1 unit of packed red blood cell transfusion following right knee replacement. She has a
history of hypertension, chronic kidney disease stage 4, and heart failure with preserved ejection
fraction. Medications currently in the hospital are lisinopril, chlorthalidone, and subcutaneous
heparin for deep venous thrombosis (DVT) prophylaxis. Before the transfusion, she had also
been receiving lactated ringer's solution at 75 ml/hr. On evaluation, the patient is afebrile, blood
pressure is 172/85 mm Hg, heart rate is 122/min, respiratory rate is 24/min, and saturation is
88% on room air. Jugular venous distension noted to be 7 cm H2O. Tachycardia with no
murmurs is heard on the cardiac exam with extensive rales auscultated in both lung fields. There
is 1+ bilateral pedal edema present. EKG reveals sinus tachycardia with left ventricular
hypertrophic changes. No ST-T changes noted. A chest x-ray reveals pulmonary edema with
bilateral airspace opacities. Laboratory data show hemoglobin of 8.0 g/dL (7.1 g/dL prior to
transfusion), creatinine 3.3 mg/dL (3.2 mg/dL prior to transfusion), total bilirubin 1.0 mg/dL,
lactate dehydrogenase 75 U/L. Urinalysis is negative for protein and blood. A brain natriuretic
peptide (BNP) level is pending. Intravenous furosemide was administered with symptomatic
improvement noted within 30 minutes. Which of the following clinical information can be used
to distinguish between transfusion-related complications?

Choices:
1. Absence of fever
2. Favorable response to diuretics
3. Absence of hypotension
4. Presence of pulmonary edema on chest x-ray
Answer: 2 - Favorable response to diuretics
Explanations:
This patient has transfusion-associated circulatory overload (TACO). It is diagnosed when
at least 3 of the following clinical features are identified within 6 hours of receiving blood
transfusion: symptoms of respiratory distress, raised central venous pressure, high B-type
natriuretic peptide, x-ray findings of pulmonary edema, presence of left heart failure and
presence of positive fluid balance.
Patients diagnosed with TACO show significant improvement in symptoms with the use of
diuretics. However, this is not true in patients with transfusion-associated lung injury
(TRALI).
Risk factors for TACO include age more than 60 years, presence of chronic kidney disease,
heart failure, number of blood transfusions, and the rate of transfusion. Since the etiology
for the development of TACO is a positive fluid balance secondary to administration of
blood, sometimes in addition to the intravenous fluids, diuretics remain the mainstay of
treatment.
Diuretics do not help patients with TRALI as these patients do not present with features
suggestive of volume overload such as elevated central venous pressure, pedal edema).
Fever and hypotension can be seen in TRALI, although only present in about 30% of cases.
Evidence of pulmonary edema on chest x-ray is present in both TACO and TRALI
however; the mechanism is cardiogenic in TACO and non-cardiogenic in TRALI.

Go to the next page if you knew the correct answer, or click the link image(s) below to further
research the concepts in this question (if desired).

Research Concepts:
Pulmonary Edema

We update eBooks quarterly and Apps daily based on user feedback. Please tap flag to
report any questions that need improvement.
Question 342: A 72-year-old female with end-stage renal disease on dialysis presents to her
primary care physician for pain in the left upper extremity that began over the last week. The
patient has a hemodialysis catheter in place on the left upper extremity without any signs of
surrounding erythema, drainage, or warmth surrounding the catheter. She reports feeling like her
upper extremity is "heavy." Which of the following imaging modalities is the preferred initial
imaging study for her likely condition?

Choices:
1. Magnetic resonance imaging
2. Computed tomographic venography
3. Plain film
4. Compression duplex ultrasonography
Answer: 4 - Compression duplex ultrasonography
Explanations:
Magnetic resonance imaging (MRI) has a high sensitivity for the diagnosis of deep venous
thrombosis (DVT) at almost 100%. It is not the gold standard for initial imaging due to its
high costs, length of time to complete the study, and/or availability.
Computed tomographic venography also has a high sensitivity for the diagnosis of DVT,
however, is not the gold standard for initial imaging due to higher costs, radiation exposure,
time, and/or availability. If ultrasonography is negative, but the clinical suspicion remains
high for a DVT, venography should be obtained.
Plain films would not adequately show a venous thrombosis but could help evaluate any
bony abnormalities that could lead to venous stenosis and resultant thrombosis.
Ultrasonography is the preferred initial imaging study for diagnosis of DVT and has a
sensitivity of 97%, which is lower than MRI but is non-invasive, less expensive, widely
available, and fast.

Go to the next page if you knew the correct answer, or click the link image(s) below to further
research the concepts in this question (if desired).

Research Concepts:
Upper Extremity Deep Venous Thrombosis

We update eBooks quarterly and Apps daily based on user feedback. Please tap flag to
report any questions that need improvement.
Question 343: A 16-year-old is seen in the emergency department with a hoarse voice and
dysphagia. His symptoms started 12 hours ago and have progressed. He has a fever and appears
ill. He is unable to speak and thus the mother provides the history. The tonsils are 4+/4 enlarged
with grey exudates. There is bilateral cervical adenopathy. While waiting for the patient to be
seen by an ENT surgeon, what one study could be ordered for this child?

Choices:
1. Blood culture
2. Lateral neck x-ray
3. Throat swab
4. Arterial blood gas
Answer: 2 - Lateral neck x-ray
Explanations:
The treatment of routine tonsillitis which is mild is usually pain control and antibiotics
(macrolides or amoxicillin). For recurrent tonsillitis, tonsillectomy is recommended.
The one imaging study sometimes performed in patients with enlarged tonsils is the lateral
neck x-ray.
The lateral neck x-ray will reveal the degree of airway obstruction.
When the airway obstruction is severe, the patient should be admitted and a tracheostomy
set must be available at the bedside.

Go to the next page if you knew the correct answer, or click the link image(s) below to further
research the concepts in this question (if desired).

Research Concepts:
Tonsillitis

We update eBooks quarterly and Apps daily based on user feedback. Please tap flag to
report any questions that need improvement.
Question 344: A 45-year-old male is admitted to the hospital after presenting with
epigastric pain with nausea and vomiting and initial laboratories showing an elevated lipase
consistent with a diagnosis of acute pancreatitis. A further review of laboratory values shows
aspartate aminotransferase of 75 units/L, alanine aminotransferase of 270 units/L, alkaline
phosphatase of 202 international units/L, and total bilirubin of 2.1 mg/dL. Which of the
following is the most likely cause of his pancreatitis?

Choices:
1. Gallstones
2. Medication-induced
3. Hypertriglyceridemia
4. Alcohol
Answer: 1 - Gallstones
Explanations:
Basic laboratory values can be used to guide the determine the etiology of acute
pancreatitis.
Suspected obstruction should be confirmed with abdominal imaging, initially with an
abdominal ultrasound.
An elevated aspartate aminotransferase (AST) is more consistent with alcohol-related
pancreatitis.
Laboratory values suggesting pancreatitis secondary to bile duct obstruction are an
elevation of alanine aminotransferase (ALT) greater than AST as well as an elevation of
alkaline phosphatase. Also, total bilirubin levels likely will be elevated with full biliary
obstruction.

Go to the next page if you knew the correct answer, or click the link image(s) below to further
research the concepts in this question (if desired).

Research Concepts:
Acute Pancreatitis

We update eBooks quarterly and Apps daily based on user feedback. Please tap flag to
report any questions that need improvement.
Question 345: A 30-year-old male with a history of sickle cell disease and ischemic stroke
1 year ago, undergoes a left hip replacement for aseptic necrosis of the left femoral head. He
presents to the emergency department 2 weeks after the surgery with difficulty breathing and
chest pain. Prior to complete evaluation the patient becomes unresponsive and has a cardiac
arrest. Despite adequate cardiopulmonary resuscitation, the patient does not survive. The autopsy
results show multiple fat emboli in the lung with pathologic evidence of ischemic stroke. When
filling out the death certificate, which of the following will be noted as the immediate cause of
death?

Choices:
1. Cardiorespiratory arrest
2. Air embolism
3. Fat embolism
4. Ischemic stroke
Answer: 3 - Fat embolism
Explanations:
The sudden onset of chest pain and dyspnea is highly suggestive of an embolic process in
the lung in this patient. This is the most attributable immediate cause of death.
Sickle cell disease causes chronic hypoxia which can lead to fragments of necrotic bone
marrow tissue being dislodged from the marrow. This embolus can subsequently occlude
the pulmonary artery.
Gurd and Wilson proposed criteria for the diagnosis of fat embolism. This condition can be
seen in patients with sickle cell disease and has a high mortality rate.
The cause of death section in the death certificate should be filled out as specifically as
possible. In Part I, one reports the causal events that lead to death and consists of Line Ia to
Line Id. The most “immediate” or “recent“ event that leads to the demise is listed on Line
Ia. The other conditions are listed on Line Ib to Line Id in a sequential manner. The last and
most remote condition leading to death is listed as the “underlying” cause of death. All
conditions that fall between the immediate and underlying cause of death are known as
intermediate or intermediary.

Go to the next page if you knew the correct answer, or click the link image(s) below to further
research the concepts in this question (if desired).

Research Concepts:
Death Certification

We update eBooks quarterly and Apps daily based on user feedback. Please tap flag to
report any questions that need improvement.
Question 346: A 55-year-old female presents to the clinic with progressive shortness of
breath and chest pain for the past 3 weeks. She is a smoker and social drinker. She is on a
bronchodilator and takes several herbal medications. On the physical examination, she has a
decrescendo diastolic murmur that is best heard while leaning forward. Palpation of the groin
reveals abrupt distension and quick collapse of the femoral artery pulse. On further physical
examination, which of the following is unlikely to be observed?

Choices:
1. A narrowed pulse pressure
2. Bobbing motion of the patient's head with each heartbeat
3. Visible pulsations of the uvula
4. Inferior displacement of the apical pulse
Answer: 1 - A narrowed pulse pressure
Explanations:
Severe aortic regurgitation usually presents with wide pulse pressure.
Corrigan pulse is also known as a water hammer pulse. Quincke sign is heard over the
femoral artery. Traube sign is hearing ‘pistol shots' over the groin.
The point of maximal impulse is diffuse and often displaced inferiorly and towards the
axilla.
The murmur of aortic regurgitation is high pitched and correlates with the severity of the
disorder.

Go to the next page if you knew the correct answer, or click the link image(s) below to further
research the concepts in this question (if desired).

Research Concepts:
Aortic Regurgitation

We update eBooks quarterly and Apps daily based on user feedback. Please tap flag to
report any questions that need improvement.
Question 347: A 78-year-old male presents via emergency medical services for ingestion of
200 tabs of 300 mg propafenone. On arrival, the patient moans to painful stimuli but makes no
effort to remove the stimuli. Blood pressure is 80/40 mmHg, pulse 130 bpm, respiratory rate
6/minute, and pulse oximetry 89% on nonrebreather mask. What is the most appropriate next
intervention?

Choices:
1. Bilevel positive airway pressure (BiPAP)
2. Airway support
3. Normal saline bolus
4. Atropine
Answer: 2 - Airway support
Explanations:
The patient has a Glasgow coma scale less than 8, so he is unable to effectively protect his
airway necessitating airway support.
BiPAP should be avoided due to the patient's significantly depressed mental status, and
inability to protect his airway. One of the contraindications to BiPAP is the inability to
protect the airway or remove the device themselves in the event of emesis.
It is important to initiate fluid boluses as the patient is hypotensive, but the management of
the patient's airway should be the priority.
When managing critically patient's address the ABC's first.

Go to the next page if you knew the correct answer, or click the link image(s) below to further
research the concepts in this question (if desired).

Research Concepts:
Sodium Channel Blocker Toxicity

We update eBooks quarterly and Apps daily based on user feedback. Please tap flag to
report any questions that need improvement.
Question 348: A 30-year-old white female who is diagnosed with acute promyelocytic
leukemia was started on all-trans retinoic acid (ATRA). On the third day of the therapy, the
patient developed fever, peripheral edema, and shortness of breath. Her vital signs include a
temperature of 102.3 F, blood pressure 90/58 mmHg, pulse 112 beats/min, pulse oximetry 84%
on 10 L of high flow oxygen, and respiratory rate 24 breaths/min. Pertinent laboratory studies
include WBC 31,000 cells/mL, hemoglobin 7 grams/dL, platelet count 50,000/mL, serum
creatinine 2.4 mg/dL (baseline 0.9 mg/dL). A chest x-ray suggests pulmonary edema. She was
transferred to the intensive care unit for further management. Which of the following is correct
regarding the management of this condition?

Choices:
1. Add arsenic trioxide to ATRA
2. Supportive therapy and continue ATRA
3. Continue ATRA and start hydroxyurea
4. Discontinue ATRA and start the patient on IV dexamethasone 10 mg twice daily
Answer: 4 - Discontinue ATRA and start the patient on IV dexamethasone 10 mg twice daily
Explanations:
Differentiation syndrome is a cytokine release syndrome seen after initiation of
differentiating agents characterized by fever, peripheral edema, pulmonary edema, and
multi-organ dysfunction. It can be fatal, if not promptly recognized and treated.
High dose systemic corticosteroids are helpful in these cases.
All-trans retinoic acid (ATRA) can be continued in mild cases in addition to IV
corticosteroids.
ATRA must be discontinued in severe cases with multi-organ dysfunction and can be
restarted when the differentiation syndrome resolves.

Go to the next page if you knew the correct answer, or click the link image(s) below to further
research the concepts in this question (if desired).

Research Concepts:
Acute Promyelocytic Leukemia

We update eBooks quarterly and Apps daily based on user feedback. Please tap flag to
report any questions that need improvement.
Question 349: A 55-year-old male with a medical history of hypothyroidism presented to
the emergency department with complaints of severe insomnia, confusion, sweating, fever, and
palpitations. On exam, blood pressure and heart rate were elevated, temperature 101 F, and
generalized continuous fine muscle twitches movements in extremities were noted. The patient
was admitted for further evaluation. Basic blood work including thyroid function panel was
within normal limits. Brain MRI was normal. Cerebrospinal fluid analysis was also within
normal limits. Next day patient developed agitation and hallucinations. Serum VGKC antibodies
were found to be elevated. What is the most likely diagnosis?

Choices:
1. Morvan syndrome
2. Limbic encephalitis
3. Myxedema coma
4. Neuromyotonia
Answer: 1 - Morvan syndrome
Explanations:
The symptoms described are highly suggestive of Morvan syndrome which is predominant
in men. It is characterized by central, autonomic and peripheral nervous system
hyperactivity. Morvan syndrome is a clinical diagnosis. Insomnia, disorientation, confusion,
amnesia, hallucinations, agitation, myokymia, neuropathic pain, loss of deep tendon
reflexes, sensory deprivation, excessive sweating, pruritus, drooling, severe constipation,
excessive lacrimation, cardiac arrhythmias, weight loss, palmoplantar erythema,
hyponatremia due to SIADH comprise most of the symptoms caused by Morvan syndrome.
Elevated serum antibodies against voltage-gated potassium channel (VGKC) complex
proteins such as CASPR2 and LGI1 are quite diagnostic of MoS.
Limbic encephalitis (LE) is a close differential of Morvan syndrome. However, subtle
differences do exist. Severe insomnia, hyperhidrosis, and myokymia are characteristic of
Morvan syndrome. Whereas LE has amnesia, seizures and temporal lobe structural
abnormalities on imaging. LE differs from MoS with the conspicuous absence of
neuromyotonia, dysautonomia. There are other biochemical and radiological differences
also.
Myxedema coma is a state of severely decompensated hypothyroidism. The patients are in a
hypothyroid state. Unlike the above-mentioned patient, patients in myxedema coma show
altered mentation, severe hypothermia, hypoglycemia, hypoxia, hypercapnia, bradycardia,
hypotension, hyponatremia, and also hypoventilation. Thyroid functions are also abnormal.
Neuromyotonia is characterized by peripheral nerve hyperexcitability only without overt
central or autonomic nervous system dysfunction.

Go to the next page if you knew the correct answer, or click the link image(s) below to further
research the concepts in this question (if desired).

Research Concepts:
Morvan Syndrome

We update eBooks quarterly and Apps daily based on user feedback. Please tap flag to
report any questions that need improvement.
Question 350: A 38-year-old male with a past medical history of diabetes mellitus,
hypertension, and endstage renal disease on hemodialysis presents with shortness of breath. He
was found to have aspiration pneumonia and was intubated for severe hypoxemia, not
responding to high flow nasal cannula oxygen. He was started on appropriate antibiotics and
propofol for sedation. He was very dyssynchronous with the ventilator, so a midazolam drip was
started to achieve more sedation. After two days, a sedation vacation was attempted, but the
patient was not waking up even after 8 hours after turning all the sedatives off. What is the most
likely explanation?

Choices:
1. Propofol remains for a longer time in patients with renal failure
2. He is likely sleeping
3. He likely had a stroke while he was sedated
4. Midazolam's metabolite is renally excreted and so it should be avoided in patients with end-
stage renal disease
Answer: 4 - Midazolam's metabolite is renally excreted and so it should be avoided in
patients with end-stage renal disease

Explanations:
The medications used to initiate and maintain sedation within an intensive care unit setting
include benzodiazepines such as diazepam, lorazepam, and midazolam; opioid analgesics
such as fentanyl; hydromorphone, morphine, remifentanil, propofol, dexmedetomidine, and
ketamine; and antipsychotics such as haloperidol, quetiapine, and ziprasidone.
No sedative is found to be superior in efficacy or mortality.
The Society of Critical Care Medicine guidelines recommends avoiding benzodiazepines
due to evidence of the longer duration of intubation, especially in patients with renal
impairment. While midazolam is metabolized by the liver, its main metabolite, alpha
hydroxymidazolam, is renally excreted.
The choice of which sedative is best lies in the practitioner's clinical assessment of
individual patient scenarios, weighing the risk/benefit profile of the medicine for each
patient.

Go to the next page if you knew the correct answer, or click the link image(s) below to further
research the concepts in this question (if desired).

Research Concepts:
Sedation Vacation in the ICU

We update eBooks quarterly and Apps daily based on user feedback. Please tap flag to
report any questions that need improvement.
Question 351: A 55-year-old male with a history of decompensated cirrhosis secondary to
cryptogenic cirrhosis and recurrent ascites on furosemide and spironolactone presents with a
worsening abdominal distention and diffuse abdominal pain. He also reports a decrease in urine
output over the past week. His physical exam is significant for ascites and mild abdominal
tenderness. His vitals reveal a blood pressure of 93/59 mmHg and pulse of 85/min. Labs are
significant for serum creatinine 2 mg/dL (from normal baseline one month ago) and potassium
3.9 mEq/L. Urine analysis is negative for casts or proteinuria. Renal ultrasound is negative for
hydronephrosis. Paracentesis is suggestive of spontaneous bacterial peritonitis, and he is started
on ceftriaxone. For his kidney injury, he is resuscitated with IV albumin 1 g/kg for two days
without improvement in his urine output or his kidney function. What is the next best step in the
management of this patient?

Choices:
1. Continue albumin
2. Hemodialysis
3. Transjugular intrahepatic portosystemic shunt
4. Start octreotide and midodrine
Answer: 4 - Start octreotide and midodrine
Explanations:
This patient likely has hepatorenal syndrome (HRS). The majority of patients with cirrhosis
and acute kidney injury are dehydrated, and their diuretics should be held, and they should
be started on IV hydration.
Lack of improvement in renal function after volume expansion with intravenous albumin (1
g/kg/day) for at least two days and withdrawal of diuretics with the exclusion of other
etiologies of kidney injury makes the diagnosis of HRS.
Patients with the diagnosis of HRS should be started on albumin for volume expansion and
octreotide and midodrine to increase and maintain systemic vascular resistance, blood
pressure, and eventually renal perfusion.
Albumin alone will not be enough to maintain his systemic vascular resistance and increase
his renal perfusion. There is no indication for hemodialysis at this point. Patients who do
not respond to medical therapy and are considered well enough can be considered for
transjugular intrahepatic portosystemic shunt as it is sometimes successful in improving
kidney function.

Go to the next page if you knew the correct answer, or click the link image(s) below to further
research the concepts in this question (if desired).

Research Concepts:
Hepatorenal Syndrome

We update eBooks quarterly and Apps daily based on user feedback. Please tap flag to
report any questions that need improvement.
Question 352: An African American female is brought to the emergency department with
acute onset shortness of breath that has progressively worsened over the past 4 hours at home.
She has a peak flow meter at home, and the number prior to arrival was 220 L, which is 51% of
predicted. She has known bronchial asthma but hasn't had an exacerbation requiring an ED visit
or hospitalization before. Her FEV1 in spirometry six months back was at 89%. After 2 hours of
management in the emergency department, she feels significantly better and is able to speak in
short sentences. Last breathing treatment was 15 minutes back. Repeat PEFR is 310 L/min,
which is 68% of predicted. Her wheezing has improved, and she wants to go home. Which of the
following is the next best step in the management of this patient?

Choices:
1. Discharge her home as her initial PEFR was above 50% and 200 L/min
2. DIscharge her home as PEFR improved by 12% and above 300 L/min
3. Assess one more peak flow and symptoms in another 2 hours in the ED
4. Discharge her home as she has normal FEV1 and no prior history of severe exacerbation
Answer: 3 - Assess one more peak flow and symptoms in another 2 hours in the ED
Explanations:
Kelsen and colleagues in a study showed a 50% relapse rate in patients treated for 2 hours
or less in a facility as opposed to 4% in those treated and observed for an additional 2 to 4
hours. Therefore even though she feels better, she should be requested to stay back in ED
for at least 4 hours.
In a study by Stein and Cole, an adequate response to treatment in ED was characterized as
visual improvement in symptoms which sustains 30 minutes or beyond the last
bronchodilator dose, and a PEFR greater than 70% of predicted. Her response immediately
post-treatment was 67%.
In the same study by Stein and Cole, initial PEFR on presentation did not predict the need
for hospitalization.
An adequate response to treatment was also characterized by an improvement in FEV1 by
10% or above 70% of predicted, although it may not be the most practical approach at the
bedside.

Go to the next page if you knew the correct answer, or click the link image(s) below to further
research the concepts in this question (if desired).

Research Concepts:
Status Asthmaticus

We update eBooks quarterly and Apps daily based on user feedback. Please tap flag to
report any questions that need improvement.
Question 353: A 41-year-old female with a past medical history of hypertension and type 2
diabetes mellitus presents to the emergency department in status epilepticus. Seizures are
eventually relieved per seizure protocol though the patient does not regain consciousness. She is
intubated and mechanically ventilated due to a Glasgow coma scale of 6. Her mother who came
with her in the ambulance explains that the patient has no history of seizures and that she had a
cough for the past few weeks. The patient has also had a few recent episodes of headaches and
confusion. The patient’s mother explains that the patient has one healthy sister and two brothers
who died when they were babies. The patient’s ammonia level is found to be 525 micromol/L
and a blood sugar of 140 mg/dL. What is the next step to care for this patient?

Choices:
1. Genetic consultation
2. Endocrinology consultation
3. Dietary consultation
4. Nephrology consultation
Answer: 4 - Nephrology consultation
Explanations:
For suspected ornithine transcarbamylase deficiency (OTCD), this patient needs urgent
hemodialysis, making nephrology consult the best next step.
With ammonia levels above 500 micromol/L, hemodialysis should be done emergently.
Ammonia levels above 800 micromol/L are associated with severe neuralgic damage,
limiting treatment options.
Using arginine as well as a combination of sodium benzoate and sodium phenylbutyrate
help reduce ammonia by using alternative pathways nitrogen elimination.

Go to the next page if you knew the correct answer, or click the link image(s) below to further
research the concepts in this question (if desired).

Research Concepts:
Ornithine Transcarbamylase Deficiency

We update eBooks quarterly and Apps daily based on user feedback. Please tap flag to
report any questions that need improvement.
Question 354: A patient is involved in a motor vehicle accident from which he was thrown
through the windshield and landed flush on the pavement. He suffered multiple injuries and
requires intubation at the scene. His vitals are labile, and he is rushed to the hospital. Because of
poor IV access, an intraosseous line is inserted in the right lower leg. A surgeon is called to
perform a cutdown on a neck vein at the same time. The patient is resuscitated and stabilized. A
CT scan reveals a fracture of the pelvis and four ribs in the left chest. Radiographs of the lower
extremity show a comminuted tibial shaft fracture. After application of an external fixator, the
patient is taken to the intensive care unit. Late in the night, the nurse notes no pulses in the right
leg which is now tense, ecchymotic, and he is unable to dorsiflex his toes and ankle. Which of
the following muscles is not involved in his pathology?

Choices:
1. Extensor hallucis longus
2. Peroneus brevis
3. Extensor digitorum longus
4. Tibialis anterior
Answer: 2 - Peroneus brevis
Explanations:
Along with other muscles of the anterior compartment that include extensor hallucis longus,
peroneus tertius, and extensor digitorum longus, the tibialis anterior is prone to acute
compartment syndrome.
Trauma to the leg is the most common cause of acute compartment syndrome.
Other causes include surgery, IV fluids, intraosseous infusion, intravenous
chemotherapeutic agents, military anti-shock trousers, and tight splints.
If the compartment syndrome is unrecognized, it can lead to ischemia and necrosis of the
muscle and nerves.

Go to the next page if you knew the correct answer, or click the link image(s) below to further
research the concepts in this question (if desired).

Research Concepts:
Tibial Anterior Compartment Syndrome

We update eBooks quarterly and Apps daily based on user feedback. Please tap flag to
report any questions that need improvement.
Question 355: A 17-year-old female who was in a motorcycle accident comes to the
emergency department. She is groaning and speaking in a low voice. She has multiple abrasions,
a blood pressure of 80/50 mmHg, and a respiratory rate of 22. She is cold to the touch, and there
is extensive subcutaneous emphysema in the right upper chest and neck. A FAST exam is
performed. What is the next step in management?

Choices:
1. Chest x-ray
2. Arterial blood gas
3. Tube thoracostomy
4. Fluid resuscitation
Answer: 3 - Tube thoracostomy
Explanations:
Airways should be the top priority.
One should always be suspicious of a reversible cause of respiratory distress.
Subcutaneous emphysema with respiratory distress is a pneumothorax until proven
otherwise. If available FAST imaging should be done. If not, the tube is placed at the fifth
intercostal and the midaxillary line.
In the field, needle decompression can be done. In the emergency department, tube
thoracostomy is preferred. An arrow catheter can be used and later converted.

Go to the next page if you knew the correct answer, or click the link image(s) below to further
research the concepts in this question (if desired).

Research Concepts:
Tension Pneumothorax

We update eBooks quarterly and Apps daily based on user feedback. Please tap flag to
report any questions that need improvement.
Question 356: A 65-year-old male presents to the emergency department with a chief
complaint of weakness following mowing the yard. The patient appears alert and oriented, well-
nourished, well dressed, and slightly agitated. Vitals include a blood pressure of 106/58 mmHg,
heart rate of 180 bpm, respiratory rate of 24/minutes, and oxygen saturation of 94% on room air.
An ECG performed shows 2:1 atrial flutter. Past medical history is remarkable for a history of
recurrent atrial fibrillation. The patient is currently taking simvastatin, clopidogrel, and
metoprolol. The provider administers ibutilide, 1 mg over 10 minutes, without a change in
rhythm and decides to switch antiarrhythmics. Which antiarrhythmic is contraindicated in this
patient?

Choices:
1. Amiodarone
2. Dofetilide
3. Digoxin
4. Propafenone
Answer: 2 - Dofetilide
Explanations:
Amiodarone is a class 3 antiarrhythmic that has been shown to have no significant effect on
the proarrhythmic potential of ibutilide and may be used safely in combination.
Dofetilide is a class 3 antiarrhythmic with a mechanism of action similar to ibutilide. The
administration of dofetilide in the presence of ibutilide is contraindicated due to the
potential for excessive prolongation of the QT interval.
Digoxin can be used safely in the presence of ibutilide.
Propafenone is a class 1C antiarrhythmic that can be used safely in the presence of ibutilide.

Go to the next page if you knew the correct answer, or click the link image(s) below to further
research the concepts in this question (if desired).

Research Concepts:
Ibutilide

We update eBooks quarterly and Apps daily based on user feedback. Please tap flag to
report any questions that need improvement.
Question 357: A 56-year-old presented with fever, chills, dyspnea, and fatigue. The
temperature is documented as high as 103.5 degrees Fahrenheit. He has a history of rheumatic
heart disease with valve replacement surgery done three years ago with the left-sided mechanical
valve in situ. The blood samples are drawn, and he is started on empirical antibiotics coverage
for infective endocarditis. Blood samples showed candidal growth on day 4. Echocardiography
showed an ejection fraction of 35% with mitral regurgitation. What should be the next step in the
management of the patient?

Choices:
1. Continue antibiotics coverage
2. Start antifungal therapy
3. Start antifungal therapy and plan for early surgery
4. Wait for 1 week to see if culture come back positive for bacteria
Answer: 3 - Start antifungal therapy and plan for early surgery
Explanations:
From an initial presentation, it is very difficult to differentiate fungal endocarditis from
bacterial endocarditis. The sign and symptoms often coincide.
Prosthetic valve endocarditis caused by fungi or other highly resistant organisms warrants
early surgery. Explanted valve or vegetations obtained from surgery is sent for further
histopathological examination and culture.
The initial antifungal treatment for Candida spp. endocarditis should be lipid formulation of
amphotericin B with or without flucytosine or a high dose echinocandin (caspofungin,
micafungin, or anidulafungin).
Plan for surgery in case of persistent bacteremia in patients with prosthetic valve despite the
use of appropriate antibiotics for 5 to 7 days. Rule out other sources of infection.

Go to the next page if you knew the correct answer, or click the link image(s) below to further
research the concepts in this question (if desired).

Research Concepts:
Fungal Endocarditis

We update eBooks quarterly and Apps daily based on user feedback. Please tap flag to
report any questions that need improvement.
Question 358: A 16-year-old female presents to the hospital with high-grade fever and
headache for the last 2 days. She has had an upper respiratory infection for the last 2 weeks as
well. On examination, she has right-side weakness and numbness. MRI brain shows cortical and
deep grey matter enhancing lesions. Lumbar puncture revealed lymphocytic pleocytosis without
any oligoclonal bands. Which medication has been shown to shorten the duration of neurological
symptoms and halt the progression of the disease?

Choices:
1. Acetaminophen
2. Corticosteroids
3. Ibuprofen
4. Antibiotics
Answer: 2 - Corticosteroids
Explanations:
Acute disseminated encephalomyelitis (ADEM), also referred to as post-infectious
encephalomyelitis, is an acute, rapidly progressive, autoimmune process that occurs in the
central nervous system. ADEM is characterized by demyelination in the brain and spinal
cord (and occasionally the optic nerve), as a result of inflammation that occurs in response
to a preceding infection or immunization.
Corticosteroids shorten the duration of neurological symptoms and halt the progression of
the disease in acute disseminated encephalomyelitis.
ADEM is treated with immunosuppression with high-dose intravenous glucocorticoids.
These can be started simultaneously with acyclovir and/or antibiotics on the initial patient
presentation. If a patient is not improving or is responding poorly to glucocorticoid
treatment, try intravenous immune globulin (IVIG), plasma exchange, or
cyclophosphamide.
Ibuprofen is used for body pains, but it does not shorten the duration of neurological
symptoms. Acetaminophen is used for fever management, but it does not shorten the
duration of neurological symptoms.

Go to the next page if you knew the correct answer, or click the link image(s) below to further
research the concepts in this question (if desired).

Research Concepts:
Acute Disseminated Encephalomyelitis

We update eBooks quarterly and Apps daily based on user feedback. Please tap flag to
report any questions that need improvement.
Question 359: A 65-year-old Vietnam veteran with no medical comorbidities was admitted
to intensive care unit in a tertiary care center following the sudden onset of high-grade fever,
intense headache, cough, and hemoptysis. He was profoundly hypoxic at admission needing
intubation and mechanical ventilation. Inotropes were needed for septic shock. Bloodwork
showed leukocytosis with 13% bands, hemoglobin 12.4 g/dL, mild thrombocytopenia, serum
creatinine, electrolytes, and blood urea were normal. Liver enzymes included aspartate
aminotransferase 70 U/L, alanine aminotransferase 85 U/L, alkaline phosphatase and serum total
bilirubin were normal. CT chest with contrast revealed multilobar infiltrates, most pronounced in
the right middle and lower lobes, associated with a moderate sized effusion. A week ago he
returned from a week-long stay in Martha's Vineyard at his son's house, where he also mowed
the lawn for one full day and pulverized a rabbit. Blood cultures were drawn and he was started
on vancomycin, cefepime, and azithromycin. In 48 hours his FiO2 was increased from 40% to
70%, platelet counts were 42,000/microliter and infiltrates increased on chest x-ray. What is the
most appropriate next step?

Choices:
1. Add IV doxycycline
2. Change cefepime to meropenem
3. Thoracentesis to rule out empyema
4. Add IV gentamicin
Answer: 4 - Add IV gentamicin
Explanations:
Tularemia pneumonia must be included in the differential diagnosis of severe community-
acquired pneumonia because of recent travel to Martha's Vineyard and using a lawn mower,
both of which have been classically associated with previous outbreaks of pneumonic
tularemia.
Necrotizing pneumonia occurs following inhalation of aerosolized Francisella tularensis,
either from accidental pulverization of flea infested or infected rabbits or rodents by a lawn
mower or from soil containing excreta of an infected rodent.
Streptomycin or gentamicin are first-line agents for the treatment of tularemia. Empiric
therapy is indicated soon after suspicion arises.
Diagnosis is usually made by serology, due to the high risk of inhalational hazard to the
microbiologist by culturing this organism.

Go to the next page if you knew the correct answer, or click the link image(s) below to further
research the concepts in this question (if desired).

Research Concepts:
Tularemia

We update eBooks quarterly and Apps daily based on user feedback. Please tap flag to
report any questions that need improvement.
Question 360: A 65-year-old female who is diagnosed with liver chronic liver disease
presents with fever, generalized abdominal pain, and confusion. Abdominal examination reveals
abdominal distension with marked abdominal tenderness. Ultrasound confirms the presence of
fluid in the peritoneal cavity. Which of the following is the most likely diagnosis?

Choices:
1. Fulminant hepatic failure
2. Abdominal tuberculosis
3. Spontaneous bacterial peritonitis
4. Perforated peptic ulcer
Answer: 3 - Spontaneous bacterial peritonitis
Explanations:
Spontaneous bacterial peritonitis (SBP) is an infection of a peritoneal cavity in a patient
with ascites.
It presents with fever and abdominal pain.
SBP can precipitate hepatic encephalopathy resulting in confusion.
The most important diagnostic investigation is a diagnostic tap with subsequent biochemical
and cytological analysis of the ascitic fluid.

Go to the next page if you knew the correct answer, or click the link image(s) below to further
research the concepts in this question (if desired).

Research Concepts:
Chronic Liver Disease

We update eBooks quarterly and Apps daily based on user feedback. Please tap flag to
report any questions that need improvement.
Question 361: A patient presents for an electrophysiology study following the incidental
finding of nonsustained ventricular tachycardia in an ambulatory surgery center. The
electrophysiologist decides to administer a nonselective beta agonist during the exam. Which of
the following would be most appropriately categorized as a relative contraindication to the use of
this medication?

Choices:
1. Hypotension
2. Acute myocardial infarction
3. Pulmonary hypertension
4. Digoxin intoxication
Answer: 1 - Hypotension
Explanations:
Isoproterenol is a nonselective beta agonist that will induce hypotension via its effects on
beta 2 receptors. Hypotension is a relative contraindication to isoproterenol administration.
Acute myocardial infarction is an absolute contraindication to the use of isoproterenol.
Isoproterenol can cause elevated blood glucose levels due to the induction of glycogenolysis
via beta-2 receptors in the liver.
Digoxin intoxication is an absolute contraindication to the usage of isoproterenol.

Go to the next page if you knew the correct answer, or click the link image(s) below to further
research the concepts in this question (if desired).

Research Concepts:
Isoproterenol

We update eBooks quarterly and Apps daily based on user feedback. Please tap flag to
report any questions that need improvement.
Question 362: A 17-year-old female presents with a diagnosis of mononucleosis made
three weeks ago by her provider. Her mother reports she has steadily worsened since that time
despite corticosteroids and one additional visit for intravenous fluids. More recently, her
daughter has developed a productive cough, and she has noticed some right lateral neck swelling.
She has had near continuous fever, and also a persistent sore throat. Vital signs show a fever of
102 F, pulse 120 bpm, blood pressure of 115/70 mmHg, and 100% oxygen saturation on room
air. Exam shows a sick but non-toxic appearing female with a "cord sign" present deep to the
anterior border of the sternocleidomastoid on the right side. Also, she has palpable
lymphadenopathy present bilaterally. What imaging study listed below is most sensitive for
confirming the diagnosis?

Choices:
1. Chest x-ray
2. Non-contrast CT of the neck
3. CT of the neck with intravenous contrast
4. Doppler ultrasound of the neck
Answer: 3 - CT of the neck with intravenous contrast
Explanations:
The patient presents with Lemierre syndrome. This is a rare condition, with an estimated
incidence of 1/1,000,000. This diagnosis can be very difficult to make early in the course of
the illness when the symptoms are very similar to common self-limited oropharyngeal
infections. As the disease progresses, tenderness in the lateral neck becomes more
prominent. A "cord sign," which represents thrombosis of the internal jugular vein, and
presents as tenderness and swelling deep to the anterior border of the sternocleidomastoid,
may be present.
The imaging modality most used to establish the diagnosis of Lemierre syndrome is CT
with IV contrast. The chest should be included as septic emboli are likely. It has the
advantage of being able to detect internal jugular vein thrombosis as well as pulmonary
septic emboli. Magnetic resonance venography (MRV) is likely more sensitive in the
diagnosis, but it is a more difficult study to obtain and requires a hemodynamically stable
patient. Ultrasound is useful to follow clot progression, but may miss acute clot and is not as
sensitive secondary to the anatomic constraints of the neck and the lack of fibrin in a fresh
clot.
In addition to internal jugular vein thrombosis shown on imaging studies, other findings
indicative of Lemierre syndrome include (1) Isolation of the causative organism from blood
cultures. Fusobacterium necrophorum is the most common. (2) Evidence for septic emboli.
These are most commonly seen in the lung but may affect almost any organ. There may be
laboratory evidence of end-organ dysfunction or abscess. (3) Precedent oropharyngeal
infection. (4) Evidence of sepsis.
A high degree of clinical suspicion must be used to make a diagnosis of Lemierre
syndrome. A healthy patient who is clinically deteriorating after a precedent oropharyngeal
infection, especially with evidence of septic emboli should prompt the clinician to order the
appropriate study to evaluate for internal jugular vein thrombosis.

Go to the next page if you knew the correct answer, or click the link image(s) below to further
research the concepts in this question (if desired).

Research Concepts:
Lemierre Syndrome

We update eBooks quarterly and Apps daily based on user feedback. Please tap flag to
report any questions that need improvement.
Question 363: A 50-year-old female polytrauma patient is admitted to the ICU. The patient
is intubated and sedated. You get called by the nurse for a swollen left forearm. You order X-
rays which show a both-bone forearm fracture. The compartments are soft and not tense, but
very swollen. The patient is not arousable. Her blood pressure measures 100/60 mm Hg, and her
heart rate is 105 bpm. Which of the following represents the correct logical sequence of events
for the management in this patient?

Choices:
1. Immediate splinting of the fracture, followed by ice and elevation and serial exams
2. Stryker measurement shows forearm compartment pressures of 35 mm Hg. You take the
patient immediately to the operating for the release of compartments because of an absolute
pressure greater than 30 mm Hg.
3. Stryker measurement shows forearm compartment pressures of 35 mm Hg. You take the
patient immediately to the operating for the release of compartments because of a delta pressure
(delta p) less than 30.
4. Ice and elevation, followed by closed reduction and splinting and rechecking the compartment
pressures every hour
Answer: 1 - Immediate splinting of the fracture, followed by ice and elevation and serial
exams

Explanations:
The delta pressure or difference between compartment pressure and diastolic blood pressure
is less than 30 mm Hg, which is more specific to compartment syndrome than an absolute
pressure of greater than 30 mm Hg.
This patient has both a delta pressure less than 30 mm Hg and absolute compartment
pressure greater than 30 mm Hg; there is no role for continued exams. She needs to be sent
down to the OR immediately for the release of compartments.
In patients whose clinical exam is limited, Stryker monitoring of compartment pressures is
vital.
In an awake and alert patient, a clinical exam, especially pain with passive stretch of the
digits in extension is sufficient for the diagnosis of compartment syndrome.

Go to the next page if you knew the correct answer, or click the link image(s) below to further
research the concepts in this question (if desired).

Research Concepts:
Forearm Compartment Syndrome

We update eBooks quarterly and Apps daily based on user feedback. Please tap flag to
report any questions that need improvement.
Question 364: A 65-year-old female with a history of diabetes mellitus type 2 and alcohol
use disorder presents with new onset jaundice and abdominal distention. She has been drinking
5-6 alcohol beverages per day for the past 20 years. Her vitals are significant for a blood pressure
of 98/67 mmHg with a normal heart rate. She is afebrile. Her exam is significant for
hepatomegaly and jaundice. Her labs are significant for WBC count of 6000/microL, platelet
count 130000/microL, ALT 140 IU/L, AST 220 IU/L, ALP 340 IU/L, and total bilirubin 8
mg/dL with a direct fraction of 6.9 mg/dL. Abdominal ultrasound shows large ascites with
hepatomegaly. She undergoes paracentesis that is significant for a serum-ascites albumin
gradient (SAAG) of 1.3, an ascitic protein of 1.5 mg/dL and neutrophils of 67/microL. What is
the most likely cause of her elevated SAAG?

Choices:
1. Increased blood flow within the splanchnic circulation
2. Decreased resistance within the liver
3. Increased right-sided cardiac pressure
4. Increase in systemic vascular resistance
Answer: 1 - Increased blood flow within the splanchnic circulation
Explanations:
This patient likely has alcoholic hepatitis with evidence of portal hypertension given the
development of ascites with elevated SAAG and thrombocytopenia. Portal hypertension
usually develops secondary to the increased resistance within the liver due to structural or
dynamic changes. Structural changes are due to the alteration of the hepatic
microcirculation such as hepatic stellate cell activation and the resultant fibrosis,
regenerative nodules, vascular occlusion, and angiogenesis.
The increased production of endothelial vasoconstrictors and decreased release of
vasodilators within the liver leads to sinusoidal constriction. Portal hypertension stemming
from this is augmented and perpetuated by the increased blood flow within the splanchnic
circulation.
This increased blood flow is due to the increased release of splanchnic vasodilators because
of increased shear stress and reduced effective arterial volume. Thus, portal hypertension is
a result of both increased resistance to portal venous flow and increased portal blood flow
due to splanchnic vasodilation.
Increase resistance within the liver is associated with portal hypertension. There is no
evidence of elevated right-sided cardiac pressures given the normal JVP, and the ascitic
protein is less than 2.5, which indicates a hepatic etiology if the ascites than cardiac. There
is a decrease in the systemic vascular resistance in portal hypertension due to the release of
splanchnic vasodilators.

Go to the next page if you knew the correct answer, or click the link image(s) below to further
research the concepts in this question (if desired).

Research Concepts:
Portal Hypertension

We update eBooks quarterly and Apps daily based on user feedback. Please tap flag to
report any questions that need improvement.
Question 365: A 54-year-old female with acute on chronic systolic congestive heart failure
and acute hypoxemic respiratory failure is now day 3 on mechanical ventilation. Diuretics and
inotropes prescribed resulted in diuresis and improvement in hypoxemia. Vital signs show a
heart rate of 94/min, blood pressure 112/68 mmHg, respiratory rate 16/min, and SpO2 98%. She
is sedated with fentanyl and propofol and opens her eyes to voice but does not follow commands.
Current ventilator settings are pressure control mode, respiratory rate 16/min, driving pressure 20
cmH2O, PEEP 8 cmH2O, and FiO2 40%. Average tidal volume is 300 mL on these settings. An
arterial blood gas shows pH 7.47, pCO2 38 mmHg, and pO2 88 mmHg. Sedation is stopped and
20 minutes later, the patient is alert and following commands. PSV mode is started, with driving
pressure 20 cmH2O, PEEP 8 cmH2O, and FiO2 40%. After 10 minutes on these settings, the
respiratory rate is now 28/min with average tidal volume 420 mL. Current vital signs are a pulse
rate of 120/min, blood pressure 108/58 mmHg, respiratory rate 28/min, and SpO2 94%. The
patient appears anxious. Chest X-ray shows stable cardiomegaly, clear lungs, and endotracheal
tube in the appropriate position. An arterial blood gas shows pH 7.52, pCO2 30 mmHg, and pO2
68 mmHg. What is the best explanation for the patient's tachypnea?

Choices:
1. Oxygen consumption has increased
2. Airway resistance has increased
3. Oxygen delivery has decreased
4. Lung compliance has decreased
Answer: 1 - Oxygen consumption has increased
Explanations:
Work of breathing and thus oxygen consumption is higher in PSV than in control modes of
ventilation. Patients with shock or low cardiac output may need more respiratory support.
The flow delivered by the driving pressure can provide a tidal volume and minute
ventilation higher than the patient could achieve without ventilator support. This higher
minute ventilation improves oxygen delivery and carbon dioxide offloading.
PSV is relatively contraindicated in patients who have a depressed respiratory drive, very
high oxygen consumption, or high airway resistance.
After PSV is initiated, the patient should be directly observed for several minutes to ensure
that the goals of ventilation, oxygenation, and patient comfort are met.

Go to the next page if you knew the correct answer, or click the link image(s) below to further
research the concepts in this question (if desired).

Research Concepts:
Pressure Support Ventilation

We update eBooks quarterly and Apps daily based on user feedback. Please tap flag to
report any questions that need improvement.
Question 366: A 55-year-old male with PMH of asthma/ COPD overlap syndrome was
admitted to the ICU for acute hypercapnic respiratory failure. He failed to non-invasive positive
pressure ventilation and had to be emergently intubated for increased work of breathing. He was
treated with intravenous steroids, bronchodilators, and his exam revealed audible wheezing. On
Day 3 of being on a mechanical ventilator, he started following commands during the
spontaneous awakening trial. Still, he became tachypneic with a respiratory rate of 32 during the
spontaneous breathing trial (SBT) on a CPAP of 5 cm H2O. His rapid shallow breathing index
was 116 breaths/minute/liter. What is the best next step in the management of this patient?

Choices:
1. Extubate the patient to CPAP
2. Obtain ABG
3. Administer a dose of succinylcholine and extubate the patient
4. Place the patient back on the assist control mode of mechanical ventilation
Answer: 4 - Place the patient back on the assist control mode of mechanical ventilation
Explanations:
A variety of respiratory, cardiac, psychological, neurologic, neuromuscular, and equipment
related causes leads to difficulty in weaning patients from ventilation.
Patients with RSBI less than 105 are likely to be successfully extubated and RSBI more
than 105 are at increased risk for extubation failure.
SBT should be performed only once a day. Several SBTs a day offer no additional benefit
compared with one.
The above patient with audible wheezing and tachypneic on SBT is most likely to fail
extubation and should not be extubated until his underlying condition improves.

Go to the next page if you knew the correct answer, or click the link image(s) below to further
research the concepts in this question (if desired).

Research Concepts:
Ventilator Weaning

We update eBooks quarterly and Apps daily based on user feedback. Please tap flag to
report any questions that need improvement.
Question 367: An 80-year-old female presents to the emergency department with
worsening shortness of breath over the past 2 days. She denies associated fevers or cough. She
has a past medical history of idiopathic pulmonary fibrosis, chronic hypoxemic respiratory
failure, chronic kidney disease, and ischemic cardiomyopathy. Her home medications include
atorvastatin, aspirin, metoprolol, nintedanib, and nitroglycerin as needed or chest pain. She uses
3 L of oxygen at home. On examination, she is in mild respiratory distress. Her jugular venous
pressure is elevated. Crackles are heard bilaterally. She has 2+ pitting edema. Her vitals are the
following: BP 134/82 mmHg, HR 100 bpm, T 99.1 F (37.3 C), and sat 88% on 3L oxygen. She
has a 6 lb (2.7 kg) weight increase from her baseline weight. A chest x-ray showed diffused
interstitial chronic fibrotic changes. A CT scan shows stable interstitial fibrotic changes with
superimposed edema, and an echocardiogram shows reduced ejection fraction. What is the best
next step in management?

Choices:
1. High dose corticosteroids
2. Referral for a lung transplant
3. Bronchodilators
4. Furosemide
Answer: 4 - Furosemide
Explanations:
Managing a patient with interstitial lung disease (ILD) with other comorbidities can be
difficult. Differentiating an acute exacerbation of ILD from any other causes comprises a
challenge.
Entities such as pulmonary edema or pneumonia should be excluded especially in the
appropriate clinical setting.
The patient's clinical history, physical exam, and imaging support a diagnosis of fluid
overload secondary to heart failure.
The most appropriate treatment is diuretics.

Go to the next page if you knew the correct answer, or click the link image(s) below to further
research the concepts in this question (if desired).

Research Concepts:
Interstitial Lung Disease

We update eBooks quarterly and Apps daily based on user feedback. Please tap flag to
report any questions that need improvement.
Question 368: A 38-year-old man presents to the emergency department after a 3-day
history of right-sided neck pain and headache. He has no significant past medical history. He
took aspirin without relief but became concerned when he had a 10-minute episode of confusion
associated with left-hand numbness and clumsiness. The exam shows mild right ptosis,
anisocoria with the right pupil being smaller, and left arm drift. Doppler ultrasonography reveals
a thrombus and possible right-sided internal carotid artery dissection. Which of the following is
the next best step in the management of this patient?

Choices:
1. Intravenous heparin
2. Oral clopidogrel
3. Arterial stenting
4. Reassurance
Answer: 1 - Intravenous heparin
Explanations:
This presentation warrants emergent CT angiography of the internal carotid artery.
When a thrombus is detected, anticoagulant therapy should be initiated. Once hemorrhage
has been ruled out, intravenous heparin followed by warfarin is typically used to prevent
thromboembolic complications.
If systemic anticoagulation is contraindicated, antiplatelet therapy can be used.
Angioplasty and stenting may be needed if there are persistent ischemic symptoms.

Go to the next page if you knew the correct answer, or click the link image(s) below to further
research the concepts in this question (if desired).

Research Concepts:
Carotid Artery Dissection

We update eBooks quarterly and Apps daily based on user feedback. Please tap flag to
report any questions that need improvement.
Question 369: A 32-year-old pregnant female presents to the emergency department with
bleeding gums and skin rashes, which started 24 hours ago. She is gravida 3 with two previous
live births. She is currently 24 weeks pregnant. She was diagnosed with immune
thrombocytopenia during her first pregnancy when she presented with similar symptoms. Her
second pregnancy was uneventful. Physical exam shows bleeding gums and petechiae in the oral
mucosa, groin area, and in the feet bilaterally. Labs are significant for a platelet count of
2000/microL. Other labs are unremarkable. Review of peripheral smear shows marked
thrombocytopenia and occasional giant platelets. She is again diagnosed with immune
thrombocytopenia. She had responded to steroids and intravenous immunoglobulins in the
previous pregnancy, however, this time she does not respond to steroids and intravenous
immunoglobulins. What is the next best step in the management of this patient?

Choices:
1. Thrombopoietin receptor agonist
2. Rituximab
3. Mycophenolate
4. Platelet transfusion
Answer: 2 - Rituximab
Explanations:
Rituximab is an acceptable option as a second-line treatment for ITP in pregnancy. It is
labeled category C by the FDA, but several reports have established its safety in pregnancy.
It is associated with transient B-cell lymphopenia in neonate which resolves by the 6th
month of life.
Splenectomy is also safe in pregnancy, but medical management is preferred over invasive
methods.
Platelet transfusion is almost never useful in a patient with immune thrombocytopenia.
Mycophenolate is contraindicated in pregnancy.

Go to the next page if you knew the correct answer, or click the link image(s) below to further
research the concepts in this question (if desired).

Research Concepts:
Thrombocytopenia in Pregnancy

We update eBooks quarterly and Apps daily based on user feedback. Please tap flag to
report any questions that need improvement.
Question 370: A 66-year-old female patient with a history of obstructive sleep apnea,
hypertension, diabetes mellitus, class 3 obesity, and asthma presents with five days of worsening
dyspnea and associated wheezing and cough. Her vital signs on presentation are blood pressure
168/70 mmHg, oxygen saturation 85% on room air, respiratory rate 32 breaths per minute, and
pulse 110 beats per minute. On exam, she has diffuse expiratory wheezing. A chest x-ray does
not show any significant infiltrates. Her labs are grossly normal, except her arterial blood gas
(ABG) shows a pH of 7.2, pCO2 56 mmHg, pO2 55 mmHg, and bicarbonate of 21 mmol/L.
Nebulization and corticosteroids are started. She is placed on BiPAP and transferred to a step-
down unit. Repeat ABG shows a pH of 7.1 and pCO2 of 70 mmHg. She is prepared to be
intubated and moved to the ICU. Rapid sequence intubation is attempted but fails after four
attempts. The patient develops bradycardia then cardiac arrest. Which of the following could
have been the cause of intubation failure?

Choices:
1. Placement of two fingers in the superior laryngeal notch
2. Placement of three fingers on the floor of the mandible between the mental angle and the neck
near the hyoid bone
3. Placement of two fingers between the incisors
4. Placement of three fingers between the incisors
Answer: 3 - Placement of two fingers between the incisors
Explanations:
Measurement of three fingers between the upper and lower teeth of the open mouth of a
patient indicates the ease of access to the airway through the oral opening. A typical patient
can open their mouth sufficiently to permit the placement of three of their fingers between
the incisors. Adequate mouth opening facilitates both insertions of the laryngoscope and
obtaining a direct view of the glottis.
Measurement of three fingers from the anterior tip of the mandible to the anterior neck
provides an estimate of the volume of the submandibular space. A typical patient can place
three fingers on the floor of the mandible between the mental angle and the neck near the
hyoid bone. Normally this distance should measure close to 7 cm. If this distance is less
than three finger-widths, the laryngeal axis will be at a more acute angle with the
pharyngeal axis, indicating that alignment of the oral opening to the pharyngeal opening
will be difficult. It also indicates that there will be less space to displace the tongue within
the throat. The rule has limitations as the distance can vary according to height and
ethnicity. For this reason, an alternative in the form of a ratio of height to thyromental
distance (RHTMD) has been suggested.
Measurement of two fingers between the floor of the mandible to the thyroid notch on the
anterior neck identifies the location of the larynx relative to the base of the tongue. A typical
patient can place two fingers in the superior laryngeal notch.
If the larynx is too high in the neck, measuring less than two fingers, direct laryngoscopy
will be difficult and potentially impossible; this is because the angle between the base of the
tongue to the larynx is too acute to be negotiated for direct visualization of the larynx easily.

Go to the next page if you knew the correct answer, or click the link image(s) below to further
research the concepts in this question (if desired).

Research Concepts:
3-3-2 Rule

We update eBooks quarterly and Apps daily based on user feedback. Please tap flag to
report any questions that need improvement.
Question 371: A 60-year-old male with history of hypertension, hyperlipidemia, and
insulin dependent diabetes mellitus presents to the emergency department for substernal chest
pain with radiation to the left side of his jaw. He is found to have ST segment elevations on EKG
with a new right bundle branch block. Which artery is most likely affected?

Choices:
1. Left marginal coronary artery
2. Left anterior descending coronary artery
3. Sinoatrial node artery
4. Right marginal coronary artery
Answer: 2 - Left anterior descending coronary artery
Explanations:
The right bundle receives most of its blood supply from the left anterior descending
coronary artery
In the setting of acute myocardial infarction, a right bundle branch block is associated with
increased mortality.
ST segment elevations are not affected by a right bundle branch block.
The right bundle also receives collateral blood supply from the right coronary artery or left
circumflex coronary artery, depending on the dominance of the heart.

Go to the next page if you knew the correct answer, or click the link image(s) below to further
research the concepts in this question (if desired).

Research Concepts:
Right Bundle Branch Block

We update eBooks quarterly and Apps daily based on user feedback. Please tap flag to
report any questions that need improvement.
Question 372: A 70-year-old female was found unresponsive on the floor in her apartment.
During a phone conversation two days ago with her daughter, her voice was loud, and she had
reported myalgias. According to her daughter, she is allergic to penicillin, has recurrent
methicillin-resistant Staphylococcus aureus skin infections, and consumes alcohol regularly.
Two weeks ago, she returned from a cruise trip to the Bahamas. Emergency services noted
vomitus soiling the floor adjacent to her, and she was intubated for airway protection. In the
emergency department, her temperature was 39 C, heart rate 105 beats/min, blood pressure 90/50
mmHg, oxygen saturation 96% on FiO2 70%, pressure support mode ventilation with PEEP 10,
pressure support 14 with spontaneous rate 28/min and minute ventilation 10 L/min. Blood
alcohol level was 155 mg/dL, white cell count 24,000/microliter, serum creatinine 2 mg/dL, and
serum sodium 130 mg/dL. The liver functions were normal. A chest x-ray confirmed correct
placement of endotracheal tube and a dense right lower lobe infiltrate. CT of the brain was
normal, and cerebrospinal fluid analysis demonstrated no white cells and normal glucose and
protein. Blood cultures were sent, intravenous vancomycin 1000 mg and levofloxacin 500 mg
were given in the emergency department. The patient was transferred to the medical intensive
care unit. Which of the following changes to her antibiotic regimen would be most appropriate?

Choices:
1. Change vancomycin and levofloxacin to linezolid and aztreonam
2. Add ampicillin-sulbactam, increase levofloxacin to 750 mg, continue vancomycin
3. Continue vancomycin, increase levofloxacin to 750 mg daily, and add ertapenem or
meropenem
4. Stop vancomycin and levofloxacin and treat with intravenous clindamycin only
Answer: 3 - Continue vancomycin, increase levofloxacin to 750 mg daily, and add
ertapenem or meropenem

Explanations:
Her risk factors comprising of alcohol use disorder and circumstantial suspicion for
aspiration, necessitate a regimen targeting gram-negative, anaerobic, and oral gram-positive
pathogens.
Although the patient has an allergy to penicillin, the risk of cross-allergenicity with
carbapenem is only 5%. Aztreonam has no activity against gram-positive or anaerobic
pathogens.
Levofloxacin has minimal activity against anaerobes, but the recent cruise trip to the
Bahamas placed her at risk for Legionella that is usually susceptible to fluoroquinolones.
Severe sepsis and a history of recurrent methicillin-resistant Staphylococcus aureus skin
infections, require the use of vancomycin.

Go to the next page if you knew the correct answer, or click the link image(s) below to further
research the concepts in this question (if desired).

Research Concepts:
Aspiration Pneumonia

We update eBooks quarterly and Apps daily based on user feedback. Please tap flag to
report any questions that need improvement.
Question 373: A 26-year-old man presented with a 1-week history of shortness of breath
and cough. He used over-the-counter cold medication and felt better. But for 2 days, he has been
getting a high fever, productive cough, and chills at night. In the emergency department, his
vitals were blood pressure 110/56 mmHg, pulse 130 bpm, temperature 39.6 C (103.2 F). Chest x-
ray showed left pleural effusion. He was started on intravenous fluids and antibiotics for
community-acquired pneumonia given after drawing blood cultures. Pulmonology service was
consulted, and a bedside pleural ultrasound showed a left moderate pleural effusion with
septations. Diagnostic thoracentesis findings were pH 7.0, lactate 649 mg/dL, and glucose of 2
mg/dL. What is the next step in management?

Choices:
1. Continue with intravenous antibiotics
2. Indwelling intrapleural catheter placement
3. Consult cardiothoracic surgery for video-assisted thoracoscopic surgery
4. Small-bore chest tube placement with intrapleural tPA and DNAase
Answer: 4 - Small-bore chest tube placement with intrapleural tPA and DNAase
Explanations:
This patient has community-acquired pneumonia, probably post-influenza. Staphylococcus
aureus is the likely causative organism in these patients. S. aureus is notorious for causing
empyema and abscesses.
Thoracentesis confirmed a parapneumonic effusion. High lactate, low glucose, and pH of
less than 7.2 is an indication for drainage of the parapneumonic effusion. The current
recommendation is to place a small-bore chest tube followed by 3 days of intrapleural tPA
and DNAase to lyse all the septations and adhesions.
An indwelling intrapleural catheter may be indicated for chronic infective pleural effusions
in highly selected cases. They have no role in the acute management of empyemas or
parapneumonic effusions.
Early video-assisted thoracoscopic surgery (VATS) has been shown to improve outcomes in
a few small studies and the moment recommended by CT surgery guidelines, but there is no
consensus among the societies. However, VATS should be done within 7 to 8 days of chest
tube placement. Doing VATS later than 8 days increases the risk of a trapped lung.

Go to the next page if you knew the correct answer, or click the link image(s) below to further
research the concepts in this question (if desired).

Research Concepts:
Intrapleural Catheter

We update eBooks quarterly and Apps daily based on user feedback. Please tap flag to
report any questions that need improvement.
Question 374: A 40-year-old female presents to an emergency department (ED) for
evaluation of recurrent abdominal pain, nausea, vomiting, diarrhea, and progressive bilateral
lower extremity weakness. She has been hospitalized with similar complaints at least ten times in
the last one and half years without a specific diagnosis. During her last admission, she was
treated for acute gastroenteritis with empiric antibiotics. Her evaluation was negative, including
an abdominal CT and upper and lower endoscopy. She also was suspected to have a psychiatric
illness that might be contributing to her symptoms. Today, she had a witnessed seizure in the ED
that lasted for 2 minutes. Her blood chemistry is notable for severe hyponatremia with a sodium
of 120 mEq/L and mildly elevated serum transaminases. She is tachycardic with a heart rate of
125 beats/min, and her blood pressure is 165/95 mmHg. The physical exam is notable for
generalized, mild abdominal tenderness without any guarding, rigidity, or rebound tenderness.
The neurologic exam is significant for disorientation and 2/5 strength and decreased sensation in
her lower extremities. Her CT head is negative for acute injury. An EEG has been ordered, and
neurology has been consulted. She is admitted to the intensive care unit for further management.
Besides correcting electrolytes abnormalities and management of acute problems, what else
should be considered?

Choices:
1. MRI brain to evaluate for the cause of her seizure
2. Urine test for porphobilinogen (PBG)
3. Consult psychiatry for evaluation of a psychiatric cause
4. Repeat EGD (esophagogastroduodenoscopy) and colonoscopy
Answer: 2 - Urine test for porphobilinogen (PBG)
Explanations:
This patient presented with a "classic triad" of abdominal pain, central nervous system
abnormalities, and peripheral neuropathy that should suggest acute porphyria. Therefore,
screening for acute porphyrias by measuring urine porphobilinogen should be considered.
Treatment for acute porphyria requires confirmation that the patient indeed has acute
porphyria, based on the finding of elevated urinary porphobilinogen (PBG), either at present
or previously. Knowing the exact type of acute porphyria is not required for treatment. IV
hemin should be administered immediately once the diagnosis of hepatic porphyria is
established.
This patient had a negative head CT, and EEG results and neurology evaluation are
pending. MRI brain is unnecessary at this stage of evaluation. It can be considered later if
necessary.
This patient had a normal EGD (esophagogastroduodenoscopy) and colonoscopy during her
last hospital admission. Repeat evaluation will not be helpful. In addition, she is not stable
enough for a procedure due to her severe hyponatremia.

Go to the next page if you knew the correct answer, or click the link image(s) below to further
research the concepts in this question (if desired).

Research Concepts:
Acute Hepatic Porphyria

We update eBooks quarterly and Apps daily based on user feedback. Please tap flag to
report any questions that need improvement.
Question 375: A 54-year-old man presents to the emergency department with dark stools.
He is on warfarin for atrial fibrillation. He feels weak, lethargic and is unable to complete
sentences. His vitals show a heart rate of 120 bpm, temp of 37.3 C, respiratory rate of 22 per
minute, blood pressure of 88/49 mm Hg. CBC shows a WBC count of 6,000 cells/microliter,
hemoglobin of 8.5 g/dL with a hematocrit of 28%, and a platelet count of 400,000/microliter, and
his INR is 5.4. Which of the following methods would the best way to control the coagulopathy?

Choices:
1. Platelet transfusion
2. Prothrombin complex concentrate
3. Plasma transfusion
4. Observation
Answer: 2 - Prothrombin complex concentrate
Explanations:
Due to life-threatening bleeding and a supratherapeutic INR. The patient should receive
both intravenous vitamin K and prothrombin complex concentrate. If PCC is unavailable,
plasma transfusion can be used.
While plasma which contains factors II, VII, IX, X in addition to other coagulation factors
is useful, PCC can correct bleeds quicker.
Vitamin K administration is necessary to permanently reverse the INR as PCC is a
transfusion of clotting factors that is used for short-term correction.
This patient would also benefit from a blood transfusion due to the brisk bleed and unstable
vital signs.

Go to the next page if you knew the correct answer, or click the link image(s) below to further
research the concepts in this question (if desired).

Research Concepts:
Anticoagulation Safety

We update eBooks quarterly and Apps daily based on user feedback. Please tap flag to
report any questions that need improvement.
Question 376: A 65-year-old man arrives at the emergency department with severe, watery
diarrhea and altered mental status. His Glasgow coma scale score is 12, and vital signs are a heart
rate of 110 beats/min, respiratory rate 24/min, and blood pressure 70/50 mmHg. His past medical
history is significant for hypertensive heart disease and diabetes mellitus type 2. His stool tests
are positive for Cyclospora cayetanensis. What is the most appropriate treatment in such a case
and why?

Choices:
1. Crystalloids, as they are readily available, cheap and are no less in mortality benefit than
albumin
2. Albumin 25% to avoid fluid overload because of his medical history of hypertensive heart
disease
3. Albumin 25% as he may be septic, and based on his blood pressure, he is in shock
4. Albumin 25% to correct his dehydration status
Answer: 1 - Crystalloids, as they are readily available, cheap and are no less in mortality
benefit than albumin

Explanations:
Crystalloids are considered the first-line treatment because albumin colloids have not
proved a better outcome in the 90-day survival rate in critically ill patients compared to
crystalloids.
Albumin 5% would be an option to avoid fluid overload because of his past medical history
of hypertensive heart disease. The recommended dosage is 500 mL of albumin, 5% repeated
every 30 minutes as needed.
Although albumin colloids have shown a slight difference in the outcome of more free days
of ventilation and vasopressor therapy than the crystalloid group, crystalloids are less
expensive than albumin colloids.
Albumin colloid solutions are more efficient than crystalloids as the amount of fluid that
remains in the intravascular space is more because of the oncotic effect. Therefore, less
fluid is required when using colloids versus crystalloids to achieve reanimation. However,
in clinical practice, there is no strong evidence to choose one over the other, in terms of
mortality outcomes.

Go to the next page if you knew the correct answer, or click the link image(s) below to further
research the concepts in this question (if desired).

Research Concepts:
Albumin Colloid

We update eBooks quarterly and Apps daily based on user feedback. Please tap flag to
report any questions that need improvement.
Question 377: A 50-year-old female presents to the emergency department after a motor
vehicle collision in which she was the unrestrained driver. Her vital signs include a heart rate of
110 bpm, stable blood pressure, and a respiratory rate in the high 20s. She looks uncomfortable
and has a deformity of the medial clavicle, prominent sternum and no palpable SC joint on the
left. What management is indicated?

Choices:
1. Stat chest x-ray followed by a stat CT angiogram of the chest
2. Chest radiographs and if normal, no further workup
3. Thoracotomy to evaluate pathology directly
4. Treat her pain, place her in a sling and refer to outpatient orthopedic follow up
Answer: 1 - Stat chest x-ray followed by a stat CT angiogram of the chest
Explanations:
This patient’s history and physical exam are concerning for a posterior sternal dislocation.
This is a potentially life-threatening injury associated with high morbidity and mortality.
Potentially injuries include esophageal or tracheal compression, pneumothorax,
hemothorax, tracheoesophageal fistula, ipsilateral acromioclavicular dislocation or clavicle
fracture, compression of subclavian artery or vein, compression of the innominate artery,
late subclavian vein obstruction, laceration of the innominate vein, or brachial plexus
compression.
Symptoms suggesting mediastinal injuries include dyspnea, stridor, dysphagia, or
paresthesia. The clinician may observe tachypnea or hypoxia, respiratory distress, difficulty
controlling secretions, discoloration, swelling or decreased pulses of the affected limb.
Management includes protecting the patient’s airway if necessary and emergent surgical
consultation with an orthopedic surgeon. Open reduction and internal fixation in the
operating room with vascular or cardiothoracic surgery as backup is indicated.

Go to the next page if you knew the correct answer, or click the link image(s) below to further
research the concepts in this question (if desired).

Research Concepts:
Sternoclavicular Joint Injury

We update eBooks quarterly and Apps daily based on user feedback. Please tap flag to
report any questions that need improvement.
Question 378: A 38-year-old healthy male, a software engineer with no known medical
history, admitted to the step-down unit with supraventricular tachycardia. The patient also has
continuous muscle twitching, agitation, fever, excessive sweating, and hallucinations. He had not
slept in 7 days. On exam sensory loss and areflexia in the lower extremities was noted. There
was no history of travel or exposure to sick contacts. Brain imaging including CT/MRI was
unremarkable. Cerebrospinal fluid analysis also within reasonable limits. EEG showed findings
consistent with encephalopathy. Serum antibody titers against CASPR2 and LGI1 proteins were
significantly elevated. What is the next best step in the management of this patient?

Choices:
1. Anticonvulsant therapy
2. Intravenous corticosteroids
3. Evaluate for thymoma
4. Immunosuppression with intravenous immunoglobulin (IVIG) or plasma exchange
Answer: 4 - Immunosuppression with intravenous immunoglobulin (IVIG) or plasma
exchange

Explanations:
Elevated antibody titers against CASPR2 and LGI1 proteins are diagnostic of Morvan
syndrome. Seizures are not a feature of Morvan syndrome and anticonvulsant therapy such
as carbamazepine, phenytoin, phenobarbital would not be beneficial.
Corticosteroids both oral and intravenously have been tried with variable but not very
significant clinical responses.
Morvan syndrome has a strong association with thymoma and could help in establishing a
diagnosis. However, CASPR2 and LGI1 antibodies are even more diagnostic. Investigations
to find a thymoma would not change the course of management at this time.
Morvan syndrome is an autoimmune disease caused by autoantibodies to voltage-gated
potassium channel complex proteins such as CASPR2 and LGI1. Immunosuppression with
IVIG or plasma exchange has been shown to decrease antibody titers and significantly
improve clinical outcomes.

Go to the next page if you knew the correct answer, or click the link image(s) below to further
research the concepts in this question (if desired).

Research Concepts:
Morvan Syndrome

We update eBooks quarterly and Apps daily based on user feedback. Please tap flag to
report any questions that need improvement.
Question 379: A 50-year-old Asian female with history of alcohol use disorder presents to
the emergency department with severe abdominal pain and vomiting that began 3 hours ago.
While working the patient up for pancreatitis, a CT scan of the abdomen is ordered. The image
showed a high-density pancreatic tail fluid collection consistent with a pseudoaneurysm of a high
flow vessel. The most likely structure is which of the following?

Choices:
1. Branch of the celiac trunk
2. Splenic artery
3. Short gastric artery
4. Left gastric artery
Answer: 2 - Splenic artery
Explanations:
Splenic artery pseudoaneurysm is a potential complication of pancreatitis as the release of
pancreatic enzymes is capable of damaging the splenic artery which travels just superior to
the pancreas.
The fact that the CT finding showed high-density fluid should raise concern for the presence
of blood within a pseudoaneurysm as opposed to a pancreatic pseudocyst which would
more commonly have homogenous low attenuation on CT scan.
Treatment options for a pseudoaneurysm include embolization of splenic artery, ligating the
splenic artery, and splenectomy with or without distal pancreatectomy.
The definitive test to confirm the presence of splenic artery pseudoaneurysm is CT
angiogram.

Go to the next page if you knew the correct answer, or click the link image(s) below to further
research the concepts in this question (if desired).

Research Concepts:
Pancreatic Pseudoaneurysm

We update eBooks quarterly and Apps daily based on user feedback. Please tap flag to
report any questions that need improvement.
Question 380: A 42-year-old male in the northeast United States presents to the emergency
department with confusion and leg weakness. On exam, he is noted to have a fever of 39 C,
diminished strength in bilateral lower extremities, and altered mental status. A head CT is
unremarkable. Cerebrospinal fluid is notable for pleocytosis (250 white blood cells/mm3), with
no red blood cells, mildly elevated protein, and normal glucose. There are no organisms seen on
Gram stain, and bacterial culture has no growth. Which of the following is the most likely vector
of his illness?

Choices:
1. Mosquito bite
2. Fecal contamination of food
3. Deer tick bite
4. Respiratory secretions
Answer: 1 - Mosquito bite
Explanations:
West Nile Virus is spread by mosquitoes.
West Nile Virus can cause viral meningitis.
West Nile Virus should be considered in adults presenting with cerebrospinal fluid
pleocytosis and lower motor neuron involvement.
A travel history should be obtained in patients with viral meningitis as many viruses have
specific geographical distributions.

Go to the next page if you knew the correct answer, or click the link image(s) below to further
research the concepts in this question (if desired).

Research Concepts:
Viral Meningitis

We update eBooks quarterly and Apps daily based on user feedback. Please tap flag to
report any questions that need improvement.
Question 381: The Grey Turner sign classically refers to ecchymosis or discoloration of the
flanks related to retroperitoneal bleeding from acute necrotizing pancreatitis though other
conditions may cause flank ecchymosis. Which of the following best describes the test
characteristics of the Grey Turner sign for detecting acute necrotizing pancreatitis?

Choices:
1. The Grey Turner sign has very low sensitivity and negative predictive value for the detection
of acute pancreatitis
2. The Grey Turner sign has high sensitivity but low specificity in the detection of acute
pancreatitis
3. The Grey Turner sign has both high sensitivity and specificity for detecting acute pancreatitis
4. The Grey Turner sign has very low sensitivity but high negative predictive value for the
detection of acute pancreatitis
Answer: 1 - The Grey Turner sign has very low sensitivity and negative predictive value for
the detection of acute pancreatitis

Explanations:
The Grey Turner sign is an uncommon finding among patients with acute severe necrotizing
pancreatitis found in less than 5% of patients with known severe pancreatitis. Thus, both the
sensitivity and negative predictive value of the sign for detecting pancreatitis is low.
Discoloration of the flanks due to ecchymosis may occur from direct trauma to the flanks
and not from intraabdominal or retroperitoneal blood dissecting through subcutaneous tissue
planes. Clinicians must determine by history and physical if direct trauma occurred and if
that explains the flank discoloration more consistently than other processes. Thus, flank
discoloration is not specific to acute necrotizing pancreatitis.
Authors have detailed multiple intraabdominal processes resulting in Grey Turner sign
including ruptured ectopic pregnancy, spontaneous splenic rupture, ruptured abdominal wall
varices, perforated duodenal ulcer, hemorrhagic transformation of intraabdominal cancers,
ruptured abdominal aortic aneurysms, amoebic liver abscesses, and several others. Thus, the
specificity of a Grey Turner sign for necrotizing pancreatitis is low and may occur for many
reasons.
Although the test characteristics of the Grey Turner sign show low sensitivity and
specificity for acute necrotizing pancreatitis the presence of flank ecchymosis not due to
direct trauma of the flank should lead the clinician to consider several serious and
potentially life-threatening diagnoses. Clinicians often require rapid bedside ultrasound or
CT to exclude these dangerous causes.

Go to the next page if you knew the correct answer, or click the link image(s) below to further
research the concepts in this question (if desired).

Research Concepts:
Grey-Turner Sign

We update eBooks quarterly and Apps daily based on user feedback. Please tap flag to
report any questions that need improvement.
Question 382: A 35-year-old Asian American male presents to the emergency department
with a complaint of multiple episodes of syncope over the past week. Initially, he feels his heart
racing and then he loses consciousness. His ECG shows an incomplete right bundle branch
block, and ST-segment elevations only lead V2, along with an inverted T wave. J wave
amplitude is 2 mm, ST-T configuration is cove-type, and the terminal portion of the ST segment
is gradually descending. Transthoracic echocardiography is normal. He is unsure of his family
history as he was adopted. Which of the following is the best therapy for this patient?

Choices:
1. Administer amiodarone at a dose of 1000 mg IV over 24 hours
2. Ablation therapy
3. Placement of an automatic implantable cardioverter-defibrillator
4. Infuse flecainide 2 mg/kg with a maximum of150 mg over 10 minutes
Answer: 3 - Placement of an automatic implantable cardioverter-defibrillator
Explanations:
Brugada syndrome is a genetic disorder that is associated with ventricular tachycardia and
can lead to syncope, cardiac arrest, or sudden cardiac death. This patient has a type 1 pattern
on EKG. Challenge with sodium channel blockers in type 1 pattern Brugada syndrome is
not necessary as it adds no additional information.
Radiofrequency ablation is a new mode of therapy that has promising results for some
Brugada patients. However, the best-proven current treatment is the insertion of an
automatic implantable cardioverter-defibrillator (AICD).
To date, no drugs have been shown to decrease mortality and reduce the occurrence of
ventricular arrhythmias or sudden death. There are conflicting results about using quinidine
instead of AICD as a treatment for Brugada syndrome, but it has proven to be useful in
patients with an AICD who experience multiple shocks or those who have a
contraindication for ICD placement.
In asymptomatic patients with no family history of sudden cardiac death, a close and
frequent follow-up with conservative management can be considered. However, a
personalized, interprofessional approach is suggested for those patients depending on each
patient's risk factors and co-morbidities, if the decision is not to undergo AICD placement.

Go to the next page if you knew the correct answer, or click the link image(s) below to further
research the concepts in this question (if desired).

Research Concepts:
Brugada Syndrome

We update eBooks quarterly and Apps daily based on user feedback. Please tap flag to
report any questions that need improvement.
Question 383: A 46-year-old man with a history of chronic lower back pain and
hypertension presents to the office for evaluation of worsening in his chronic back pain for the
last six days. He denies any trauma, fever, chills, urinary incontinence or worsening of his
chronic left lower extremity numbness. He has had some more difficulty ambulating due to back
pain and has started using his cane again in the last two days. His blood pressure is 164/95
mmHg, pulse rate 108 bpm, respiratory rate 16/minute, temperature 98.4 F. Examination reveals
point tenderness to the L3-L4 spinal region. Osteomyelitis is suspected. What imaging study has
the highest sensitivity and specificity for the detection of osteomyelitis within one week of
symptom onset?

Choices:
1. X-ray
2. CT scan
3. MRI
4. PET scan
Answer: 3 - MRI
Explanations:
MRI has the highest combined sensitivity and specificity for early detecting osteomyelitis at
78% to 90% and 60% to 90% respectively.
MRI can detect bone infection within 3 to 5 days of the disease onset.
MRI has a high negative predictive value, so a negative result is sufficient to exclude
infection in a patient if symptoms have been present for more than one week.
MRI has limited utility for detecting osteomyelitis in the presence of orthopedic hardware.

Go to the next page if you knew the correct answer, or click the link image(s) below to further
research the concepts in this question (if desired).

Research Concepts:
Osteomyelitis

We update eBooks quarterly and Apps daily based on user feedback. Please tap flag to
report any questions that need improvement.
Question 384: An 80-year-old male with multiple comorbidities presents to the emergency
department from a nursing home with a fever. He has a chronic urinary catheter that is draining
cloudy, foul-smelling urine. His nursing aide reports that he had a recent admission for
vancomycin-resistant Enterococcus faecium. Prior cultures show that the organism has been
resistant to ampicillin. Which of the following therapies would provide initial empiric coverage
for the patient’s urinary tract infection?

Choices:
1. Linezolid
2. Ceftriaxone
3. Trimethoprim/sulfamethoxazole
4. Aztreonam
Answer: 1 - Linezolid
Explanations:
Linezolid can be given through intravenous or oral routes, as it has 100% bioavailability.
Linezolid is bacteriostatic and acts through the inhibition of the pre-translational initiation
complex formation. This, in turn, stops production of proteins needed for bacterial
reproduction.
While a patient is taking linezolid, the patient should be monitored for thrombocytopenia,
anemia, and peripheral neuropathy.
Linezolid should be used with caution in patients taking other serotonergic drugs, as it has
been associated with serotonin syndrome.

Go to the next page if you knew the correct answer, or click the link image(s) below to further
research the concepts in this question (if desired).

Research Concepts:
Vancomycin-Resistant Enterococci

We update eBooks quarterly and Apps daily based on user feedback. Please tap flag to
report any questions that need improvement.
Question 385: A 67-year-old female with a history of hypertension, migraines, and
nephrolithiasis is admitted from the emergency department for 4 days of flank pain, dysuria, and
fever. She has no known drug allergies and takes 240 mg of propranolol daily for which she
endorsed compliance at admission. A CT scan showed a mildly dilated right renal pelvis and
proximal ureter with a 7 mm obstructing kidney stone. Her laboratory studies are remarkable for
a serum WBC count of 14,000/microliter, urine dipstick positive for leukocyte esterase and
WBCs, a creatinine of 1.3 mg/dL, and glomerular filtration rate 60. Her vital signs at the time of
admission were heart rate 44 beats/min, blood pressure 116/72 mmHg, temperature 101.2F, and
respiratory rate 18. She was started on broad-spectrum antibiotics after cultures were obtained
and scheduled for a ureteroscopy the following morning. She was given multiple fluid boluses
for hypotension overnight, totaling 3 liters. The following morning she is more somnolent with a
heart rate of 47 beats/min, blood pressure 88/54 mmHg, temperature 103.4F, and respiratory rate
22. Which of the following inotrope is most appropriate for this patient?

Choices:
1. Norepinephrine
2. Phenylephrine
3. Epinephrine
4. Dobutamine
Answer: 1 - Norepinephrine
Explanations:
The first step in treating septic shock is adequate fluid resuscitation (30ml/kg) within the
first 3 hours, sending cultures to the lab for evaluation, and early broad-spectrum
antibiotics. If fluid resuscitation is unable to keep the mean arterial pressure greater than
65mmHg, vasopressor or ionotropic support should be initiated.
Per the Surviving Sepsis Campaign (SSC), the vasopressor of choice after adequate fluid
resuscitation is norepinephrine.
The patient is not mounting a tachycardic response because she is on a high dose of
propranolol. Propranolol has a half-life of approximately 12 hours, and the patient is still
bradycardic. Phenylephrine would not be an appropriate choice because it could potentially
worsen her bradycardia.
If the patient is severely tachycardic or experiences a tachyarrhythmia in response to
norepinephrine, phenylephrine would be an appropriate choice after volume resuscitation.

Go to the next page if you knew the correct answer, or click the link image(s) below to further
research the concepts in this question (if desired).

Research Concepts:
Phenylephrine

We update eBooks quarterly and Apps daily based on user feedback. Please tap flag to
report any questions that need improvement.
Question 386: A 17-year-old male presents to the emergency department with generalized
weakness, nausea, and epigastric pain. On exam, he appears dehydrated and has no focal
abdominal tenderness. Vital signs are stable. Basic metabolic panel reveals bicarbonate of 10
mmol/L and a glucose of 68 mg/dL, with an anion gap of 24. Venous blood gas reveals a pH of
7.0. Urine is positive for ketones. Serum ethanol concentration is 80 mg/dL. The patient was
given two ampules of dextrose and two liters of normal saline. A repeat basic metabolic panel
then showed bicarbonate of 14 mmol/L and an anion gap of 15. What is the most likely
diagnosis?

Choices:
1. Methanol toxicity
2. Ethylene glycol toxicity
3. Isopropyl toxicity
4. Alcoholic ketoacidosis
Answer: 4 - Alcoholic ketoacidosis
Explanations:
Methanol and ethylene glycol toxicity both cause an anion gap acidosis. They do not
generally improve with resuscitation, especially if the parent compound is continuously
metabolizing.
Isopropyl alcohol toxicity causes ketosis without acidosis. Symptomatically, a patient
should be intoxicated after ingesting isopropyl, with possible hemorrhagic gastritis.
Alcoholic ketoacidosis is not treated with fomepizole. It can prolong the half-life of ethanol.
Treatment of alcoholic ketoacidosis requires replacement of dextrose and resuscitation.
Patients respond much better to resuscitation than when intoxicated with methanol or
ethylene glycol.
Oftentimes, when serum ethanol is elevated, it is unlikely that the patient would be acidotic
from a toxic alcohol unless they ingested the toxic alcohol long before the ethanol.

Go to the next page if you knew the correct answer, or click the link image(s) below to further
research the concepts in this question (if desired).

Research Concepts:
Methanol Toxicity

We update eBooks quarterly and Apps daily based on user feedback. Please tap flag to
report any questions that need improvement.
Question 387: Following a left lung lobectomy, a 66-year-old patient develops
hypotension, tachycardia, and hypoxia. Fleischner sign is noted on chest x-ray. A loud P2 is
heard on auscultation. The EKG shows variable ST-segment changes in the anterior leads.
Which of the following is the diagnostic test of choice for this patient's suspected condition?

Choices:
1. Cardiac enzymes
2. Time of flight MRI
3. Contrast-enhanced CT chest
4. Bronchoscopy
Answer: 3 - Contrast-enhanced CT chest
Explanations:
Pulmonary embolism should be the first consideration in a postoperative patient who
suddenly decompensates.
A contrast-enhanced CT chest is the diagnostic test of choice for evaluating pulmonary
artery emboli. CT typically demonstrates filling defects in the pulmonary arteries, including
the main pulmonary arterial trunks or the segmental and subsegmental arteries.
Radiographs often demonstrate nonspecific findings. Fleischner sign on a chest radiograph
can be seen, which corresponds to an enlarged pulmonary artery.
A Hampton hump, which corresponds to a peripheral lung infarction, can also be seen as a
peripheral wedge-shaped opacity. Westermark sign corresponds to regional decreased blood
flow with difficult to visualize peripheral pulmonary vasculature (oligemia).

Go to the next page if you knew the correct answer, or click the link image(s) below to further
research the concepts in this question (if desired).

Research Concepts:
Acute Pulmonary Embolism

We update eBooks quarterly and Apps daily based on user feedback. Please tap flag to
report any questions that need improvement.
Question 388: A 47-year-old male presents to the emergency department complaining of
severe epigastric pain radiating to the back and associated nausea and vomiting. He has a
prolonged history of alcohol use disorder and states that he often drinks 500 mL of vodka per
day with many failed attempts at quitting. The patient appears to be in distress and using
accessory muscles to breathe. His vitals show a temperature of 38.5 C, heart rate 115 beats/min,
blood pressure 90/45 mmHg, and respiratory rate 42. Laboratory studies are significant for lipase
of 5534 units/L and mildly elevated transaminases with aspartate aminotransferase greater than
alanine aminotransferase. Hematocrit is 42%. A chest x-ray is obtained and shows bilateral,
patchy opacities throughout the lung fields. Which of the following indicates a poor prognosis?

Choices:
1. Lipase greater than 950 units/L
2. PaO2 less than 60 mmHg
3. Hematocrit greater than 40%
4. Age greater than 45 years
Answer: 2 - PaO2 less than 60 mmHg
Explanations:
Poor prognostic indicators for acute pancreatitis are shock, hypoxemia, hemoconcentration
with hematocrit greater than 44%, gastrointestinal bleeding, and renal failure.
Ranson criteria include age greater than 55 years, WBC greater than 16,000/microliter,
glucose greater than 200 mg/dL, lactate dehydrogenase greater than 350 units/L, and
aspartate aminotransferase greater than 250 U/L.
Ranson criteria measured at admission and at 48 hours correlate with risk of death.
Ranson criteria can be used only to calculate mortality once at 48 hours after admission and
cannot be used to determine response to therapy.

Go to the next page if you knew the correct answer, or click the link image(s) below to further
research the concepts in this question (if desired).

Research Concepts:
Acute Pancreatitis

We update eBooks quarterly and Apps daily based on user feedback. Please tap flag to
report any questions that need improvement.
Question 389: A 65-year-old man with a history of chronic hepatitis C-induced liver
disease is brought to the emergency department for fatigue, worsening jaundice, and progressive
abdominal distention. The patient was found on the floor at home by his daughter difficult to
arouse in the afternoon. He responds to noxious stimuli but not to verbal command. On exam,
flank fullness with shifting dullness is elicited. CT head is negative. Diagnostic paracentesis is
performed, and blood is drawn for labs. The accuracy of serum ascites albumin gradient (SAAG)
is most diminished in which of the following?

Choices:
1. Fever
2. Arterial hypotension
3. Anemia
4. Jaundice
Answer: 2 - Arterial hypotension
Explanations:
SAAG is based on oncotic-hydrostatic balance.
Calculating SAAG entails simply subtracting the ascitic fluid value from the serum value.
SAAG greater than 1.1 g/dL (11 g/L), is suggestive of the presence of portal hypertension
and value less than 1.1 g/dL or 11 g/L may exclude it.
SAAG does not provide any information about the pathogenesis of the ascites. In the
presence of arterial hypotension, portal pressure is decreasing, causing narrowing of the
SAAG gradient. Fever, jaundice, and anemia have not been shown to affect the SAAG
value.

Go to the next page if you knew the correct answer, or click the link image(s) below to further
research the concepts in this question (if desired).

Research Concepts:
Ascites

We update eBooks quarterly and Apps daily based on user feedback. Please tap flag to
report any questions that need improvement.
Question 390: A 65-year-old male with history of stage IIIb squamous cell cancer of the
lung presents with increasing dyspnea on exertion and orthopnea ongoing for four weeks with
gradual worsening. On presentation, he is noted to be 90% on ambient air and is tachypneic to a
respiratory rate of 30/minute. Labs are at baseline. A chest x-ray demonstrates large pleural
effusion with contralateral mediastinal shift. He undergoes large-volume thoracentesis of 3 liters
with immediate relief of dyspnea. The next morning, he develops respiratory distress with new
hypoxia requiring supplemental oxygen. What is the most likely cause of the patient's newly
developed respiratory distress?

Choices:
1. Pneumothorax ex vacuo
2. Hemothorax
3. Reexpansion pulmonary edema
4. Pulmonary embolism
Answer: 3 - Reexpansion pulmonary edema
Explanations:
Reexpansion pulmonary edema occurs after the removal of fluid or air from the pleural
space quickly over a short period of time.
The mechanism of edema is believed to be increased capillary permeability. Risk factors for
this condition include young age, a long duration of lung collapse, and rapid reexpansion.
Treatment is largely supportive.
Thoracentesis operators should not perform large-volume thoracentesis. No more than 1.5
liters of fluid should be removed in any instance.

Go to the next page if you knew the correct answer, or click the link image(s) below to further
research the concepts in this question (if desired).

Research Concepts:
Malignant Effusion

We update eBooks quarterly and Apps daily based on user feedback. Please tap flag to
report any questions that need improvement.
Question 391: A 27-year-old female presents to the emergency department complaining of
rhinorrhea and excessive lacrimation for the last 30 minutes. One hour ago, she was cleaning her
windows with an ammonia-containing cleaner when the bottle slipped and fell on the floor. She
attempted to clean the fluid away despite the strong irritating pungent smell. Which of the
following findings on evaluation would best predict the development of chronic lung disease in
this patient?

Choices:
1. Reduced oxygen saturation on presentation to the emergency department
2. Presence of chemical burns on the nose
3. Reduced peak expiratory flow rate on presentation to the emergency department
4. Injury to the bronchial lining, as evident on bronchoscopy
Answer: 4 - Injury to the bronchial lining, as evident on bronchoscopy
Explanations:
Prolonged inhalation of ammonia fumes or anhydrous ammonia gas can cause injury to the
bronchial epithelium.
Since ammonia is a light and water-soluble gas, most of the injury is limited to the upper
respiratory tract if the exposure is of short duration.
Prolonged exposure results in damage to the basal lining of lower bronchial and pulmonary
epithelium and this can lead to the development of chronic lung disease.
Reduced peak expiratory flow on presentation to the emergency department could be due to
bronchoconstriction from the irritant action of the fumes. This is usually short-lived, and the
patient experiences symptomatic relief on supportive management.

Go to the next page if you knew the correct answer, or click the link image(s) below to further
research the concepts in this question (if desired).

Research Concepts:
Ammonia Toxicity

We update eBooks quarterly and Apps daily based on user feedback. Please tap flag to
report any questions that need improvement.
Question 392: A 66-year-old female was admitted to the ICU after a motor vehicle accident
causing a severe head injury. Brain death diagnosis was uncertain by physical examination, and
the apnea test could not be performed due to CO2 retention. A transcranial Doppler (TCD)
ultrasound on the middle cerebral arteries bilaterally did not detect any flow tracing. Which of
the following is the next best step in the management of this patient?

Choices:
1. Declare brain death
2. Order somatosensory evoked potentials
3. Repeat TCD after one day
4. Continue medical management
Answer: 2 - Order somatosensory evoked potentials
Explanations:
TCD can confirm brain death if it shows absent diastolic pulsations or small peaked systolic
pulsations. The absence of flow tracing is inconclusive of brain death, as it may be due to an
unsuitable window.
Somatosensory evoked potentials may be ordered to determine brain death as TCD is
inconclusive for this patient.
The absence of flow tracing by TCD does not confirm brain death, as it may be due to an
unsuitable window. Another ancillary test should be performed.
Normal PCO2 is a prerequisite to performing the apnea test.

Go to the next page if you knew the correct answer, or click the link image(s) below to further
research the concepts in this question (if desired).

Research Concepts:
Brain Death Criteria

We update eBooks quarterly and Apps daily based on user feedback. Please tap flag to
report any questions that need improvement.
Question 393: A patient presents with weight loss, anemia, and fatigue. Blood work reveals
a white blood cell count of 88,000 cells/mm3 and a hemoglobin of 9 grams/dL. He has a mildly
enlarged spleen and liver. Cytogenetic testing reveals a chromosomal translocation resulting in a
BCR-ABL1 fusion gene. What is the most likely diagnosis?

Choices:
1. Acute myeloid leukemia (AML)
2. Acute lymphoblastic leukemia (ALL)
3. Chronic myeloid leukemia (CML)
4. Chronic lymphocytic leukemia (CLL)
Answer: 3 - Chronic myeloid leukemia (CML)
Explanations:
The translocation is t(9;22)(q34;22q11) and results in the Philadelphia chromosome. This
results in the BCR (breakpoint cluster region) gene from chromosome 22 fusing with the
ABL1 gene of chromosome 9. Abl is from "Abelson," a leukemia virus that has a similar
protein.
The Philadelphia chromosome is seen in 95% of patients with chronic myeloid leukemia
(CML), which is detected by traditional karyotype.
The remainder are positive for cryptic BCR-ABL1 fusion gene, which is detected by
fluorescence in situ hybridization or reverse transcriptase-polymerase chain reaction.
The Philadelphia chromosome also is found in B-cell acute lymphoblastic leukemia (B-
ALL), acute myeloid leukemia (AML), and mixed phenotype acute leukemia (MPAL).

Go to the next page if you knew the correct answer, or click the link image(s) below to further
research the concepts in this question (if desired).

Research Concepts:
Chronic Myelogenous Leukemia

We update eBooks quarterly and Apps daily based on user feedback. Please tap flag to
report any questions that need improvement.
Question 394: A 65-year-old Asian male with known history of rheumatic mitral valve
disease presents with a 1-week history of new-onset gnawing pain and swelling of both arms and
wrist joints. He also has a high-grade fever for past four days with increased shortness of breath.
According to his son, he is mildly confused since this morning. What would be the most obvious
acute condition to suspect?

Choices:
1. Acute bacterial meningitis
2. Miliary tuberculosis
3. Acute infective endocarditis
4. Acute deep vein thrombosis
Answer: 3 - Acute infective endocarditis
Explanations:
Clinical findings of hypertrophic osteoarthropathy should trigger suspicion for a new
complication on a damaged valve, likely a vegetation.
Milliary tuberculosis does not present with acute confusion but with a generalized decline
over months, fever, weight loss and intestinal symptoms.
Acute bacterial meningitis causes meningeal signs in the axial, not appendicular skeleton.
Bilaterally symmetrical upper extremity deep venous thrombosis is highly unlikely.

Go to the next page if you knew the correct answer, or click the link image(s) below to further
research the concepts in this question (if desired).

Research Concepts:
Secondary Hypertrophic Osteoarthropathy

We update eBooks quarterly and Apps daily based on user feedback. Please tap flag to
report any questions that need improvement.
Question 395: A 48-year-old left-handed white collar worker is transported to the
emergency room with left sensory loss, aphasia, agitation, mild left-sided weakness, and a
headache. The patient has no past medical history. The patient was recently hospitalized for
transient speech problems, headache, and left-sided sensory loss. MRI/MRA of the head and
neck were normal. The patient had been discharged as symptoms had resolved. Which of the
following is most likely on lumbar puncture?

Choices:
1. Elevated opening pressure
2. Moderately elevated protein
3. Pleocytosis with lymphocyte predominance
4. Any of the above
Answer: 4 - Any of the above
Explanations:
The patient could have viral infection or headache with neurologic deficits and CSF
lymphocytosis.
If the patient's condition resolved HaNDL is most likely.
This condition is characterized by 1 to 20 episodes of transient neurologic deficits with
headache over 3 months.
The syndrome of neurological deficit with CSF lymphocytosis may be associated with a
viral infection. In some patients, the symptoms may resolve spontaneously but in others, the
symptoms may persist for an indefinite period.

Go to the next page if you knew the correct answer, or click the link image(s) below to further
research the concepts in this question (if desired).

Research Concepts:
Headache With Neurological Deficits and CSF Lymphocytosis

We update eBooks quarterly and Apps daily based on user feedback. Please tap flag to
report any questions that need improvement.
Question 396: A 21-year-old male with a past medical history of severe asthma, anxiety,
and obesity presents to the emergency department with acute onset dyspnea. A review of the
medical records shows that he has been hospitalized four times in the last year and required
intubation once when he was 18 years of age. On the initial exam, he was found to be tachypneic
with a respiratory rate of 34, diaphoretic, and using accessory muscles for respiration. An arterial
blood gas (ABG) done in the emergency department showed a pH of 7.35, PCO2 45, and PaO2
92 on room air. He was intubated and paralyzed due to difficulty syncing with the event. He was
placed on AC mode ventilation with a tidal volume of 8cc/kg, respiratory rate 23, PEEP of 5, and
FiO2 40%. A repeat ABG shows a pH of 7.19, PCO2 60, and PaO2 109. Apart from adjusting
tidal volume and respiratory rate, what else can be done to improve his ventilation?

Choices:
1. Increase inspiration time
2. Increase FiO2
3. Increase PEEP
4. Square the flow curve (increase inspiratory flow)
Answer: 4 - Square the flow curve (increase inspiratory flow)
Explanations:
This is a young patient with asthma who has been difficult to ventilate requiring him to be
paralyzed. His arterial blood gas (ABG) shows a respiratory acidosis even though he has a
good tidal volume and a rapid respiratory rate on the ventilator. Increasing his respiratory
rate could lead to auto-PEEP. One way to prevent this is by increasing the flow rate of air
into the respiratory system allowing for faster delivery of the tidal volume and more time
for exhaling. This is called squaring of the flow curve.
Decreasing, not increasing, the inspiratory time (decrease in the I:E ratio) will allow for
more time for exhalation, thereby helping to prevent auto-PEEP and allowing for an
increased respiratory rate.
Increasing FiO2 will not lead to improvement in the PCO2. The patient's oxygenation is
already good without needing further improvement.
Increasing PEEP will improve oxygenation but will not affect his CO2.

Go to the next page if you knew the correct answer, or click the link image(s) below to further
research the concepts in this question (if desired).

Research Concepts:
Ventilation Assist Control

We update eBooks quarterly and Apps daily based on user feedback. Please tap flag to
report any questions that need improvement.
Question 397: A 17-year-old female presents to the emergency department after ingesting
an unknown amount of a food warming fuel. She has nausea, epigastric discomfort, and blurred
vision. A physical exam reveals epigastric tenderness, sluggish pupillary response to light, and
decreased visual acuity. Her basic metabolic panel demonstrates a bicarbonate of 4 mmol/L.
What is the most appropriate treatment plan?

Choices:
1. Dialysis
2. Fomepizole, resuscitation, admit for dialysis
3. Fomepizole and resuscitation
4. Bicarbonate administration while awaiting confirmatory test results
Answer: 2 - Fomepizole, resuscitation, admit for dialysis
Explanations:
This patient ingested a food warming fuel, and food warming fuels are known to contain
methanol.
Symptoms of end-organ toxicity and acidosis are indications for dialysis in methanol
toxicity. Dialysis has been shown to markedly decrease the length of stay.
Fomepizole should be given immediately in this scenario so that the patient can respond to
resuscitation. Normal saline should be administered.
Confirmatory test results may take hours to days in some facilities and should not delay
treatment. Bicarbonate infusion may be of some benefit. Bicarbonate alone will not be an
effective treatment.

Go to the next page if you knew the correct answer, or click the link image(s) below to further
research the concepts in this question (if desired).

Research Concepts:
Methanol Toxicity

We update eBooks quarterly and Apps daily based on user feedback. Please tap flag to
report any questions that need improvement.
Question 398: A 72-year-old female who lives independently is brought in by emergency
medical services after being found down at home by a friend who became suspicious when the
patient had not arrived for their weekly book club. On initial assessment, the patient is
tachycardic and hypotensive and is on supplemental oxygen via nasal cannula at 5 L. She is
moaning and uncomfortable, non-verbal, but opens her eyes to command and moves all
extremities. An exam reveals abdominal tenderness and ecchymosis around the umbilicus.
Which of the following conditions is associated with periumbilical ecchymosis?

Choices:
1. Pancreatitis
2. Cholecystitis
3. Renal stone
4. Appendicitis
Answer: 1 - Pancreatitis
Explanations:
Cullen sign is a classic sign seen in some cases of hemorrhagic pancreatitis.
The sign is characteristic of severe, acute pancreatitis, ectopic pregnancy, and
hemoperitoneum.
Cullen sign appears as a bluish to purplish discoloration around the umbilicus and can occur
in conjunction with Turner sign.
Cullen sign is most often present in severe pancreatitis where inflammation has caused the
erosion of blood vessels leading to hemoperitoneum.

Go to the next page if you knew the correct answer, or click the link image(s) below to further
research the concepts in this question (if desired).

Research Concepts:
Acute Pancreatitis

We update eBooks quarterly and Apps daily based on user feedback. Please tap flag to
report any questions that need improvement.
Question 399: A 66-year-old female is admitted to the hospital for severe iron deficiency
secondary to chronic kidney disease. After receiving her first two infusions of parenteral iron
therapy, she begins to complain of muscle, and joint pain and her blood pressure drops to 92/65
mmHg. In addition to intravenous fluids, a drug with which of the following mechanism of
action is most appropriate for this patient?

Choices:
1. Chelation
2. Neutralization
3. Oxidation-reduction
4. Isomerization
Answer: 1 - Chelation
Explanations:
Deferoxamine is a chelating agent that works by binding to iron in the bloodstream, creating
a stable complex that is excreted renally.
While the body has innate processes to actively absorb iron, it does not have a way of
actively excreting iron.
Only a certain amount of iron can be actively absorbed at any given time; excess is taken up
passively via the paracellular route.
Chelation is also a useful therapy for treating other types of heavy metal toxicity including
copper in patients with Wilson Disease.

Go to the next page if you knew the correct answer, or click the link image(s) below to further
research the concepts in this question (if desired).

Research Concepts:
Iron

We update eBooks quarterly and Apps daily based on user feedback. Please tap flag to
report any questions that need improvement.
Question 400: A 70-year-old male presents to the hospital with an altered mental status.
Physical examination is remarkable for bradycardia, anasarca, and hypothermia. His family
reveals that he had thyroid cancer and a thyroidectomy. He has not been seen by a healthcare
provider in 5 years. Which of the following is the most accurate regarding this condition?

Choices:
1. Increased respiratory rate and decreased blood pressure
2. Decreased respiratory rate and increased blood pressure
3. Decreased serum potassium level and decreased serum sodium level
4. Decreased serum sodium level and decreased blood pressure
Answer: 4 - Decreased serum sodium level and decreased blood pressure
Explanations:
The diagnosis is myxedema crisis, which is a medical emergency. The patients usually
present with hyponatremia, hypotension, bradycardia.
Myxedema coma occurs in patients with long-standing hypothyroidism and usually is
precipitated by drugs, infection, trauma, cerebrovascular disease, or heart failure.
Hypotension can indicate systemic infection, hemorrhage, and further cardiovascular
deterioration. Prompt initiation of fluid resuscitation with broad-spectrum antibiotics ,
hydrocortisone, and levothyroxine is of paramount importance
Additional complications from myxedema crisis include respiratory failure, coma, and
hyponatremia. Mechanical ventilation is required if there is evidence of impending
respiratory failure.

Go to the next page if you knew the correct answer, or click the link image(s) below to further
research the concepts in this question (if desired).

Research Concepts:
Myxedema

We update eBooks quarterly and Apps daily based on user feedback. Please tap flag to
report any questions that need improvement.
Section 5
Question 401: A 32-year-old woman with a past medical history of acute myelocytic
leukemia presents complaining of cough with a small amount of bright red blood. She had a one-
week history of cough productive of thick brown sputum, fever, and pleuritic chest pain. A chest
x-ray done 5 days ago revealed right upper lobe infiltrate; she was immediately started on oral
antibiotics by her primary care provider. However, the symptoms persist. The patient underwent
allogeneic stem cell transplantation for his acute myelocytic leukemia 5 weeks ago, which was
complicated by acute graft-versus-host disease and neutropenia. Temperature is 39.5 C (103. 1
F), blood pressure is 100/62 mmHg, the pulse is 110/min, and respiratory rate is 20/min. The
physical exam is significant for right-sided crackles. Laboratory results show leukocytes
1500/microL, hematocrit 28%, and platelets 138,000/microL. Chest x-ray shows a right upper
lobe infiltrate, increased in size when compared to the previous x-ray. Chest CT-scan reveals
several nodular lesions with surrounding ground-glass opacities in the right upper lobe. Sputum
gram stain shows no organisms. What is the best initial therapy for this patient?

Choices:
1. Voriconazole
2. Voriconazole + caspofungin
3. Caspofungin
4. Embolization
Answer: 2 - Voriconazole + caspofungin
Explanations:
Caspofungin is effective against invasive aspergillosis in patients who are refractory to or
not tolerating voriconazole.
The main population at risk of developing invasive aspergillosis is immunocompromised
patients. These patients will typically present with a triad of fever, chest pain, and
hemoptysis. Chest CT reveals pulmonary nodules with the "halo" sign. Treatment consists
of voriconazole with or without caspofungin.
Caspofungin is not recommended as first-line therapy for invasive aspergillosis.
Patients on caspofungin should be monitored for hepatotoxicity, as it has been associated
with increased liver enzymes.

Go to the next page if you knew the correct answer, or click the link image(s) below to further
research the concepts in this question (if desired).

Research Concepts:
Caspofungin

We update eBooks quarterly and Apps daily based on user feedback. Please tap flag to
report any questions that need improvement.
Question 402: A 30-year-old female presents to the hospital for severe nausea and
vomiting along with a two-day history of fever and cough. She has type 1 diabetes mellitus and
has been using insulin since diagnosis 14 years ago. Her regimen is glargine insulin 25 U SQ
BID and also uses an insulin pump. On examination, the patient has dry mucous membranes and
appears somewhat lethargic. She has a bout of severe abdominal pain and that is relieved by
some morphine. Two intravenous lines with normal saline and insulin are started. Labs are drawn
showing blood glucose of 500 mg/dL. The laboratories show sodium 131 mEq/dL, potassium 3.1
mEq/dL, chloride 90 mEq/dL, bicarbonate 13 mEq/dL, BUN 28 mg/dl, and creatinine 0.88
mg/dL. Arterial blood gas shows pH 6.98, PO2 92, and PCO2 28. What are the changes to the
management needed?

Choices:
1. Continue normal saline and insulin, supplement potassium, and start broad-spectrum
antibiotics
2. Replace normal saline with a bicarbonate drip, continue insulin, supplement potassium, and
start broad-spectrum antibiotics
3. Continue normal saline, stop insulin, supplement potassium, and start broad-spectrum
antibiotics
4. Replace normal saline with a bicarbonate drip, stop insulin, supplement potassium, and start
broad-spectrum antibiotics
Answer: 3 - Continue normal saline, stop insulin, supplement potassium, and start broad-
spectrum antibiotics

Explanations:
Once the potassium level falls below 3.3, it is recommended that the insulin is stopped and
reinitiated once potassium is supplemented adequately. If the insulin is continued, the
potassium levels fall further, and there is a risk of cardiac arrhythmias with severe
hypokalemia.
There is no indication of bicarbonate replacement above a pH of 6.9 and may be associated
with increased harm including cerebral edema
Broad-spectrum antibiotics are indicated in this patient with a possible infectious trigger for
ketoacidosis
Potassium supplementation should be done on low and normal potassium levels in
anticipation of the transcellular shifts lowering the levels during treatment of diabetic
ketoacidosis with insulin.

Go to the next page if you knew the correct answer, or click the link image(s) below to further
research the concepts in this question (if desired).

Research Concepts:
Ketoacidosis

We update eBooks quarterly and Apps daily based on user feedback. Please tap flag to
report any questions that need improvement.
Question 403: A 68-year-old male is in the intensive care unit after being in a severe motor
vehicle accident. At 9 pm on the third day of his hospital stay, he pulls out his intravenous line
and urine catheter and begins screaming that people are trying to hurt him. Later that evening, he
is found to be disoriented and sedated in appearance. What is the most likely diagnosis?

Choices:
1. New-onset dementia
2. Post-traumatic stress disorder
3. Psychotic break
4. Delirium
Answer: 4 - Delirium
Explanations:
Delirium is very common in the hospital setting and includes a disturbance of consciousness
and changes in cognition.
A diagnosis of delirium requires knowledge of an individual's baseline cognitive function.
By definition, delirium must be caused by an organic process.
Treatment of delirium requires treating the underlying cause.

Go to the next page if you knew the correct answer, or click the link image(s) below to further
research the concepts in this question (if desired).

Research Concepts:
Delirium

We update eBooks quarterly and Apps daily based on user feedback. Please tap flag to
report any questions that need improvement.
Question 404: A 72-year-old male with a history of hypertension, hyperlipidemia, diabetes
mellitus, and congestive heart failure is brought to the hospital by his son for weakness, perioral
numbness, vomiting, and abdominal cramping for 2 days. He denies any travel history, but he ate
fish at a restaurant a few days ago. On examination, his blood pressure is 120/90 mmHg, pulse
52 beats/min, respirations 19, temperature 98.9 F, and oxygen saturation 96% on room air. An
EKG shows sinus bradycardia without any ST abnormalities. A chest x-ray does not show any
active cardiopulmonary processes. A CT head does not show any acute intracranial normality.
He is admitted to the hospital and started on conservative treatment with IV fluids, antiemetics,
and bedrest. Blood and urine cultures have been negative so far. After 3 days of hospitalization,
the patient continues to have the neurologic symptoms, but the bradycardia and gastrointestinal
symptoms have resolved. What is the best next step in management?

Choices:
1. MRI of the brain
2. Levetiracetam twice daily
3. One dose of mannitol
4. Acetazolamide three times daily
Answer: 3 - One dose of mannitol
Explanations:
Neurologic symptoms of ciguatera fish poisoning can last for some time and can be
debilitating.
Neurologic symptoms of ciguatera fish poisoning include perioral numbness, paresthesias,
painful urination, the sensation of loose of teeth, cold allodynia, and pruritus without any
skin lesions.
Conservative management with volume repletion, antiemetics, and bedrest is the mainstay
of treatment.
One dose of mannitol can be tried for patients with neurologic symptoms despite
conservative management. The role of mannitol for long-term use is controversial, but one
dose of mannitol can be tried without serious adverse effects. The patient has to be
euvolemic before mannitol can be administered, as it leads to fluid loss.

Go to the next page if you knew the correct answer, or click the link image(s) below to further
research the concepts in this question (if desired).

Research Concepts:
Ciguatera Toxicity

We update eBooks quarterly and Apps daily based on user feedback. Please tap flag to
report any questions that need improvement.
Question 405: A 56-year-old male with a medical history of hypertension, acute
lymphocytic leukemia, non-ischemic cardiomyopathy underwent an automatic implantable
cardioverter-defibrillator (AICD) placement to prevent sudden cardiac death due to a low
ejection fraction of 20%. Next day he developed significant chest pain, shortness of breath and
diaphoresis. Blood pressure was low, heart rate was high, but the temperature was normal. WBC
counts were normal, troponins were elevated, and EKG showed ST-segment elevations in
anterior and lateral chest leads. Emergent bedside transthoracic echocardiogram showed left
ventricular hypokinesis. Urgent CT angiogram of the chest showed some pulmonary vascular
congestion, but otherwise, it was normal. The patient was taken for an emergent coronary
angiogram which was nonobstructive. However, nonspecific dilated coronary sinus was noted on
the angiogram. What is the most likely diagnosis?

Choices:
1. Coronary sinus thrombosis
2. Pulmonary embolism
3. Cardiac tamponade
4. Endocarditis
Answer: 1 - Coronary sinus thrombosis
Explanations:
Coronary sinus thrombosis is a rare but severe post-procedural complication following
invasive right heart instrumentation. This patient underwent AICD placement which
involves instrumentation of coronary sinus for ventricular lead placement which most likely
caused endothelial injury and triggered thrombosis of the coronary sinus. This patient is at a
higher risk for development of coronary sinus thrombosis also due to the preexisting history
of leukemia. A high index of clinical suspicion and emergent treatment with thrombectomy
followed by long-term anticoagulation is needed to prevent mortality.
Pulmonary embolism can present with chest pain and shortness of breath. However, ST-
segment elevation on EKG is not common. Echocardiogram shows right ventricular strain.
Ct angiogram of the chest is diagnostic of Pulmonary Embolism. This patient’s most likely
diagnosis is coronary sinus thrombosis, mainly due to the finding of bulging coronary sinus
and a negative CT angiogram of the chest. A thrombus in the coronary sinus, especially if
chronic can get further complicated by rupturing and embolising to the pulmonary
vasculature. However, CT angiogram of the chest would be positive at that time.
Cardiac tamponade can result due to many cardiac complications like myocardial infarction
including acute worsening coronary sinus thrombosis. However cardiac tamponade can be
diagnosed on echocardiogram. There no pericardial effusion or tamponade noted in this
patient on echocardiogram.
Due to the absence of fever, normal WBC, normal echocardiogram, and finding of coronary
sinus dilation on a cardiac angiogram, endocarditis is less likely in this patient.

Go to the next page if you knew the correct answer, or click the link image(s) below to further
research the concepts in this question (if desired).

Research Concepts:
Coronary Sinus Thrombosis

We update eBooks quarterly and Apps daily based on user feedback. Please tap flag to
report any questions that need improvement.
Question 406: A 62-year-old male with a history of hypertension, diabetes mellitus,
chronic obstructive pulmonary disease (COPD) on 2L of home oxygen, congestive heart failure,
and atrial fibrillation on warfarin presents to the emergency department with shortness of breath.
His initial vitals are temperature 101.3 F, heart rate 121 bpm, blood pressure 150/90 mmHg,
respiratory rate 24/minute, and pulse oximetry (SpO2) 80% on 10 L non-rebreather mask. His
exam is significant for rales in the left lower lung field and decreased air movement diffusely as
well as 1+ pitting edema to his lower extremities bilaterally. Chest x-ray shows hyperinflated
lungs, and a focal infiltrate in the left lower lobe. EKG shows atrial fibrillation with a rapid
ventricular response without signs of ischemia. He is intubated for hypoxemic respiratory failure
secondary to pneumonia. However, his oxygenation does not improve with standard ventilator
settings, and inverse ratio ventilation (IRV)is being considered. Which of the following patient
factors is most likely to complicate this patient's management if he were to be placed on IRV?

Choices:
1. Respiratory failure secondary to pneumonia
2. History of COPD
3. History of heart failure
4. Hemodynamic status
Answer: 2 - History of COPD
Explanations:
This patient's history of severe COPD, indicated by his need for home oxygen, decreased air
movement on exam, and hyper-inflated lungs, put him at increased risk for auto-positive
end-expiratory pressure (PEEP) if placed on IRV.
Typical management of mechanical ventilation for patients with obstructive lung disease
often involves increasing the expiratory time to allow adequate exhalation time, not
decreasing it as is seen in IRV.
Decreased expiratory time will lead to incomplete exhalation in this patient with
subsequently decreased ventilation and increasing intrathoracic pressures resulting in
barotrauma.
In severe cases, intrathoracic pressure may continue to build with subsequent breaths as
each additional breath creates additional volume which is unable to be exhaled. This may
result in hemodynamic compromise or pneumothorax.

Go to the next page if you knew the correct answer, or click the link image(s) below to further
research the concepts in this question (if desired).

Research Concepts:
Inverse Ratio Ventilation

We update eBooks quarterly and Apps daily based on user feedback. Please tap flag to
report any questions that need improvement.
Question 407: A 35-year-old male presents to the Emergency Department in respiratory
distress. On exam, you note some venous distention on his neck and have difficulty with
auscultating heart sounds. Vital signs at this time show hypotension at 62/30 mmHg, heart rate
53 bpm, respiratory rate 26 per minute, and SpO2 92%. While evaluating the patient, he
suddenly becomes unresponsive, and no pulses are detected. An ECG performed in triage shows
the differing amplitude of the QRS complex. Following CPR and airway management, what is
the next appropriate step?

Choices:
1. Give the patient an IV fluid bolus
2. Perform emergent bedside pericardiocentesis
3. Obtain bedside trans-thoracic echocardiogram (TTE)
4. Consult cardiology for emergent cardiac catheterization
Answer: 2 - Perform emergent bedside pericardiocentesis
Explanations:
This patient is suffering from cardiac tamponade physiology as seen in exam findings
including hypotension, venous distension, distant heart sounds and ECG findings consistent
with electrical alternans.
This patient needs emergent management in reversal of cardiac arrest from tamponade
physiology. Bedside pericardiocentesis is the next step in managing this patient.
In a stable patient, both intravenous fluids and TEE are reasonable prior to
pericardiocentesis as increased volume promoting filling and increasing cardiac output
temporarily. Bedside TTE will also be useful in confirming cardiac tamponade.
Cardiac tamponade is one the T's in the H's and T's list when evaluating pulseless electrical
activity.

Go to the next page if you knew the correct answer, or click the link image(s) below to further
research the concepts in this question (if desired).

Research Concepts:
Electrical Alternans

We update eBooks quarterly and Apps daily based on user feedback. Please tap flag to
report any questions that need improvement.
Question 408: A 62-year-old man with a medical history of hypertension and pulmonary
embolism. He takes amlodipine 5 mg a day and apixaban 5 mg BID. He presents to the
emergency department with a chief complaint of a severe headache. He last took his medications
5 hours ago at 8 am. CT scan reveals intracranial hemorrhage. Apixaban therapy is held, and a
decision is made to treat with andexanet alfa. What is the recommended dose of andexanet alfa
for this patient?

Choices:
1. 400 mg IV infusion at 30 mg/min
2. 400 mg IV bolus at a rate of 30 mg/min, followed by an IV infusion at 4 mg/min for up to 2
hours
3. 400 mg IV bolus followed by a drip at 2mg/min
4. 800 mg IV infusion at 4 mg/min
Answer: 2 - 400 mg IV bolus at a rate of 30 mg/min, followed by an IV infusion at 4 mg/min
for up to 2 hours

Explanations:
Dosing of andexanet alfa for reversal is dependent on the factor Xa inhibitor given, the
dose, and the time of the last dose.
There are two dosing regimens, low and high. Low dose is 400 mg IV bolus at a rate of 30
mg/min followed by an IV infusion at 4 mg/min for up to 2 hours. High dose is 800 mg IV
bolus at a rate of 30 mg/min followed by an IV infusion at 8 mg/min for up to 2 hours.
If the last dose of apixaban was 5 mg or less and was given less than 8 hours ago, give the
low dose.
Administering more than one dose has not been evaluated in clinical trials.

Go to the next page if you knew the correct answer, or click the link image(s) below to further
research the concepts in this question (if desired).

Research Concepts:
Andexanet Alfa

We update eBooks quarterly and Apps daily based on user feedback. Please tap flag to
report any questions that need improvement.
Question 409: A 65-year-old male with a history of hypertension and depression on
enalapril and citalopram presents with abdominal pain for 4 days. Exam reveals a blood pressure
of 156/85 mmHg, pulse 115 bpm, temperature of 101.5 F (38.6 C), respirations of 16, abdominal
rebound tenderness in the left lower quadrant, and a white blood cell count of 34,000. What is
the next best step?

Choices:
1. Contrast enhanced abdominal and pelvic CT
2. Stool cultures
3. Flexible sigmoidoscopy
4. Plain radiographs
Answer: 1 - Contrast enhanced abdominal and pelvic CT
Explanations:
The best study would be an abdominopelvic CT, preferably with contrast. CT findings
include fat stranding, bowel wall thickening, and presence of diverticula. CT scans are fast
and readily available
Colonoscopy has no role in either diagnosis or management of acute diverticulitis, as it can
put the patient at further risk for perforation or worsening of inflammation. It is
recommended that a colonoscopy be performed approximately 6 to 8 weeks after symptoms
have resolved to rule out malignancy if the patient has not been scoped in the recent past.
Plain radiographs can show nonspecific findings such as ileus or small bowel obstruction.
They can also show free air if a perforation has occurred. However, none of these findings
are specific to diverticulitis.
Abdominal ultrasonography, when properly performed have a comparative sensitivity to
CT. However, it is very operator-dependent. The findings associated with diverticulitis
include a hypoechoic peridiverticular inflammatory reaction, mural and peridiverticular
abscess formation with or without gas bubbles, bowel wall thickening at the point of
maximal tenderness, and presence of diverticula in the surrounding segments.

Go to the next page if you knew the correct answer, or click the link image(s) below to further
research the concepts in this question (if desired).

Research Concepts:
Acute Diverticulitis

We update eBooks quarterly and Apps daily based on user feedback. Please tap flag to
report any questions that need improvement.
Question 410: A previously healthy 16-year-old male presents with fever and rash after
recently having nasal packing placed following surgery. He is tachycardic with an initial blood
pressure of 80/50 mmHg and a temperature of 102 F. The rash is diffuse, red, and painful, and he
has ulceration of the oral mucosa. Which of the following is the appropriate management and
disposition?

Choices:
1. Aggressive fluid hydration, antibiotics, and admission to the intensive care unit
2. Aggressive fluid hydration, removal of the packing, IV antibiotics, and admission to the
intensive care unit
3. Removal of the packing, intravenous fluids, oral antibiotics, and discharge home
4. Intravenous hydration, removal of the packing, and admission to the floor
Answer: 2 - Aggressive fluid hydration, removal of the packing, IV antibiotics, and
admission to the intensive care unit

Explanations:
This patient has toxic shock syndrome and requires intensive care unit admission.
While intravenous fluids and antibiotics also are appropriate, any source of infection and
toxin production needs to be removed.
Toxic shock syndrome can be caused by group A Streptococcus or Staphylococcus aureus.
Intravenous penicillin should be administered in addition to a beta-lactamase resistant
antibiotic.

Go to the next page if you knew the correct answer, or click the link image(s) below to further
research the concepts in this question (if desired).

Research Concepts:
Toxic Shock Syndrome

We update eBooks quarterly and Apps daily based on user feedback. Please tap flag to
report any questions that need improvement.
Question 411: An 89-year-old female is undergoing a cecopexy to repair a cecal volvulus
at a rural hospital. The anesthesiology provider wants to give her general anesthesia, inducing
with both IV and inhaled anesthetics. The patient’s surgical history is significant for an
appendectomy when she was 12 years old. The hospital pharmacy calls and informs the
anesthesiology provider that there is a national shortage of sevoflurane and desflurane. The
anesthesiology provider orders halothane to be given because he knows based on the patient’s
past surgical history she tolerated halothane induction with no complications. While recovering
on the medical-surgical floor, the patient develops jaundice, periodic episodes of blood oozing
from her IV site, and later in her hospital course, increasingly severe altered mental status. What
could be causing this symptomology?

Choices:
1. Ascending cholangitis
2. Viral-induced fulminant hepatitis
3. Drug-induced fulminant hepatitis
4. Disseminated intravascular coagulation (DIC)
Answer: 3 - Drug-induced fulminant hepatitis
Explanations:
Fulminant hepatic failure symptoms include jaundice, nausea, vomiting, dizziness,
coagulopathy, and later development of altered mental status due to hepatic encephalopathy.
Halothane is a volatile inhaled anesthetic, one of the original gases used as an
amnestic/anesthetic agent in surgical procedures. Like all volatile gases, its believed
mechanism of action is the augmentation of GABA signaling, which decreases excitatory
signaling in the central nervous system to induce sedation, amnesia, anesthesia.
Halothane-induced fulminant hepatitis is an uncommon occurrence seen in those who have
had past exposure to halothane with either no liver dysfunction or well-controlled liver
disease.
Another adverse effect seen with halothane administration is malignant hyperthermia. This
presents as muscle rigidity, hyperthermia, rapid onset tachycardia, hypercapnia,
hyperkalemia, and metabolic acidosis.

Go to the next page if you knew the correct answer, or click the link image(s) below to further
research the concepts in this question (if desired).

Research Concepts:
Anesthetic Gases

We update eBooks quarterly and Apps daily based on user feedback. Please tap flag to
report any questions that need improvement.
Question 412: A 44-year-old previously healthy female presents to the emergency
department in septic shock. Initial vitals are temperature 101.5 F, heart rate 115 bpm, blood
pressure 110/50 mmHg, respiratory rate 24/minute, and pulse oximetry (SpO2) 78% on a non-
rebreather mask. She is intubated for hypoxemic respiratory failure and acute respiratory distress
syndrome secondary to pneumonia. She is placed on inverse ratio ventilation (IRV) to improve
her oxygenation. Over the next 5 minutes, her SpO2 steadily increases to 95%. However, her
repeat blood pressure has decreased to 70/40 mmHg. What is the most likely etiology of her
hemodynamic compromise?

Choices:
1. Pneumothorax
2. Increased systemic inflammatory response syndrome (SIRS) response and vasodilation
secondary to inflammatory mediators released in the lungs in response to barotrauma
3. Distributive shock secondary to sepsis
4. Decreased venous return
Answer: 4 - Decreased venous return
Explanations:
Hemodynamic compromise secondary to decreased venous return is a known complication
of inverse ratio ventilation (IRV).
The increased mean airway pressures used in IRV translate to increased intrathoracic
pressure, which may cause an iatrogenic obstructive shock picture.
Patients with preexisting hemodynamic instability are at increased risk for shock when
placed on IRV.
Patients with sepsis may have an element of distributive shock that goes unrecognized due
to compensation. However, if an additional insult occurs, such as an obstruction to right
ventricular preload, this shock may rapidly decompensate.

Go to the next page if you knew the correct answer, or click the link image(s) below to further
research the concepts in this question (if desired).

Research Concepts:
Inverse Ratio Ventilation

We update eBooks quarterly and Apps daily based on user feedback. Please tap flag to
report any questions that need improvement.
Question 413: A 22-year-old healthy male presents to the hospital with abdominal pain,
nausea, vomiting, and yellowish discoloration of his skin. He recently was on vacation in
Mexico. His vitals are within normal limits. Labs were significant for ALT 1100 IU/L and AST
1000 IU/L with total bilirubin 9 mg/dl. Hepatitis A, B, and C serology is negative. Hepatitis E
IgM is negative. He is HIV negative and not sexually active. Ceruloplasmin levels within normal
limits. He denies taking any prescribed or over-the-counter medications. What is the best next
step in the management of this patient?

Choices:
1. Liver biopsy
2. Hepatitis E PCR
3. Anti-tissue transglutaminase antibody
4. Ferritin
Answer: 2 - Hepatitis E PCR
Explanations:
The patient’s symptoms and labs are consistent with hepatitis. Given his history of travel to
Central America, he is at risk of acquiring hepatitis E virus (HEV) infection. With negative
hepatitis A, B, C, HIV, normal ceruloplasmin levels, and no history of new medications, he
likely has acute HEV infection.
HEV serology is used for the evaluation of HEV, however, the sensitivity and specificity
vary between different lab assays.
Patients with high clinical suspicion of HEV infection and negative HEV IgM then HEV
PCR should be checked as viremia arises early during the incubation period.
Liver biopsy is not indicated as this patient likely has acute HEV infection. Anti-tissue
transglutaminase antibody is used in the evaluation of celiac disease. The patient did not
report any diarrhea and celiac disease does not cause high elevations in serum
transaminases such as in this patient that are consistent with acute viral hepatitis. Ferritin is
an acute phase reactant that is elevated during inflammation but is not specific.

Go to the next page if you knew the correct answer, or click the link image(s) below to further
research the concepts in this question (if desired).

Research Concepts:
Hepatitis E

We update eBooks quarterly and Apps daily based on user feedback. Please tap flag to
report any questions that need improvement.
Question 414: A 65-year-old female with rheumatoid arthritis is on methotrexate 12.5 mg
weekly and prednisone 7.5 mg daily. She is admitted for urosepsis. Her vital signs are blood
pressure 86/52 mmHg, heart rate 120 beats per minute, temperature 38.9 C, respiratory rate 22
breaths per minute, and oxygen saturation 95% on room air. A complete blood count shows
white blood cells 22,000 with a left shift and a hematocrit of 24%. Her electrolytes are normal,
but the BUN is 45 mg/dL and creatinine is 2.2 mg/dL. She is started on appropriate antibiotics,
but after 2 liters of fluids, she is still hypotensive with a central venous pressure of 20 cmH2O.
What is the most appropriate treatment?

Choices:
1. Transfuse two units of packed red blood cells
2. Continue crystalloids at 500 mL/hour
3. Start dopamine
4. Start intravenous hydrocortisone
Answer: 4 - Start intravenous hydrocortisone
Explanations:
The patient is having an Addisonian crisis.
She is steroid dependent and cannot adequately respond to the stress of sepsis with
endogenous glucocorticoids.
Continuing fluids would not be the best option. Her central venous pressure is high and
fluids could precipitate congestive heart failure.
If the hydrocortisone is not effective, pressors such as norepinephrine or dopamine may be
needed.

Go to the next page if you knew the correct answer, or click the link image(s) below to further
research the concepts in this question (if desired).

Research Concepts:
Addisonian Crisis

We update eBooks quarterly and Apps daily based on user feedback. Please tap flag to
report any questions that need improvement.
Question 415: A 75-year-old man is admitted for abdominal pain. He is febrile to 103 F.
His blood pressure falls to 90/60 mmHg with a pulse of 130 beats/min. He seems restless and
agitated. A urinalysis shows many bacteria and WBCs. He also has thick, brown discharge in his
Foley catheter. CT abdomen confirms active diverticulitis with a colovesical fistula. What is the
next step in management?

Choices:
1. Intravenous fluids and antibiotics
2. Surgery
3. Bowel rest
4. Abdominal MRI
Answer: 1 - Intravenous fluids and antibiotics
Explanations:
Fluid resuscitation and intravenous antibiotics are the cornerstones of managing sepsis,
regardless of the source.
Antibiotic regimens for diverticulitis must cover gram-negative rods and anaerobes, the
dominant bacteria within the gastrointestinal tract. These include ciprofloxacin plus
metronidazole, trimethoprim/sulfamethoxazole, ampicillin/sulbactam, or moxifloxacin only
if intolerant to the previous three regimens. If improving, the patient may be switched from
intravenous therapy to oral therapy after approximately 3 to 5 days. They should complete a
10 to 14-day course in total.
All cases of complicated diverticulitis including abscess, fistula formation, perforation, and
obstruction require inpatient admission. Patients with uncomplicated diverticulitis plus any
of the following also require inpatient admission, sepsis, immunosuppression, high fevers,
severe abdominal pain, advanced age, decreased oral intake, noncompliance with
medications, or those who have failed an outpatient therapy.
Surgery, commonly bowel resection, is indicated if a patient’s condition continues to
deteriorate despite appropriate medical therapy. In addition, complicated diverticulitis often
requires surgical intervention for abscess drainage or resection of the affected colon if
obstructed, perforated, or fistulized.

Go to the next page if you knew the correct answer, or click the link image(s) below to further
research the concepts in this question (if desired).

Research Concepts:
Acute Diverticulitis

We update eBooks quarterly and Apps daily based on user feedback. Please tap flag to
report any questions that need improvement.
Question 416: A male presents with new onset of bilateral leg pain and weakness. Three
weeks prior the patient was hospitalized for severe gastroenteritis and dehydration. He describes
the leg pain as a pins and needles sensation in both legs. Recently he just got a new job in a metal
foundry and noticed that his symptoms began soon after. Physical exam reveals decreased
vibratory sense in his ankle joint bilaterally, tachycardia and garlic odor from his mouth. Which
study would not be useful if the exposure occurred more than a few hours ago?

Choices:
1. Plasma levels
2. Electrocardiogram
3. Abdominal x-ray
4. 24-hour urinary levels
Answer: 1 - Plasma levels
Explanations:
Serum arsenic levels have not been shown to be an effective or reliable marker for arsenic
toxicity due to rapid clearance of arsenic from the bloodstream.
Arsenic serum levels are only detectable for a few hours after ingestion. However, urinary
levels last for days to weeks after ingestion depending on dosage and oxidative state.
Whole blood is excellent for detecting other metal toxins such as lead secondary to tight
binding of intracellular proteins.
Treatment for suspected arsenic toxicity should be started prior to any laboratory
confirmation.

Go to the next page if you knew the correct answer, or click the link image(s) below to further
research the concepts in this question (if desired).

Research Concepts:
Arsenic Toxicity

We update eBooks quarterly and Apps daily based on user feedback. Please tap flag to
report any questions that need improvement.
Question 417: A 36-year-old female is admitted to the hospital for gastroenteritis. On the
second day after admission, she develops petechiae on her legs. Blood work reveals hemoglobin
of 8 mg/dL, down from 14 mg/dL, and platelet count of 56,000, down from 250,000/ microL.
Her creatinine is also mildly elevated from 0.8 to 1.4. She appears drowsy and has also
developed a fever of 39 C. WBC count is 8000/microL which is the same as admission. A
pregnancy test is negative. She denies any diarrhea. Which of the following is the next best step
in the management of this patient?

Choices:
1. Watchful waiting and supportive treatment
2. Start plasma exchange and methylprednisolone
3. Start methylprednisolone alone
4. Obtain kidney ultrasound and further workup before starting management
Answer: 2 - Start plasma exchange and methylprednisolone
Explanations:
The suspected diagnosis for this patient is thrombotic thrombocytopenic purpura (TTP).
Given the high suspicion of TTP from this patient’s labs, presentation, and lack of other
clear causes including pregnancy leading to DIC, and eclampsia; and the high mortality of
TTP, it is essential to start plasma exchange therapy (PEX) along with a glucocorticoid
immediately as this reduces mortality.
Key factors for distinguishing between hemolytic syndromes are age, the time course of the
disease, and the degree of kidney injury.
It is also associated with diarrhea and gastrointestinal discomfort, which can lead to worse
kidney function.
PEX is often used with a glucocorticoid to reduce the production of ADAMTS13 inhibitor
antibody as well as reduce cytokine production. Rituximab, an antibody against CD20 of B
cells, is also commonly used with PEX as this produces better outcomes.

Go to the next page if you knew the correct answer, or click the link image(s) below to further
research the concepts in this question (if desired).

Research Concepts:
Acute Anemia

We update eBooks quarterly and Apps daily based on user feedback. Please tap flag to
report any questions that need improvement.
Question 418: A 51-year-old male presents with fever, productive cough, and worsening
chronic right shoulder pain. He has a history of interstitial lung disease and dermatomyositis. He
admits he is undergoing immunosuppressive therapy and was admitted one month ago for
pneumonia. He denies trauma or recent corticosteroid injections to the shoulder. On exam, there
is evidence of dermatomyositis, and his right shoulder is limited in all planes of motion with a
fluctuant mass in the superior and posterior aspects of the joint. His lungs are clear to
auscultation. Bedside ultrasound reveals a large subacromial bursal fluid collection, and a
radiograph of the chest shows interstitial infiltrates. What is the most appropriate treatment for
this patient?

Choices:
1. Ethambutol orally and rifampin, isoniazid, and pyridoxine intravenously
2. High dose corticosteroids and levofloxacin intravenously
3. Incision and drainage of the bursal fluid collection followed by doxycycline orally
4. Surgical debridement of the right shoulder and vancomycin intravenously
Answer: 1 - Ethambutol orally and rifampin, isoniazid, and pyridoxine intravenously
Explanations:
This patient is immunocompromised and at risk for fungal and atypical infections. His
likely diagnosis is septic bursitis and pulmonary infection likely from Mycobacterium. The
patient exhibits pulmonary, and extrapulmonary signs and symptoms, including pulmonary
infiltrates, evidence of dermatomyositis, and deep bursal infection. Diagnosis of
Mycobacterial infection involves sputum and bursal fluid culture in this patient and
histopathology. Treatment for disseminated disease is the same for pulmonary and includes
ethambutol, rifampin, isoniazid, and pyridoxine.
High-dose corticosteroid is the treatment for his dermatomyositis and levofloxacin for
pneumonia in an immunosuppressed individual. However, the patient also has septic
bursitis, and levofloxacin would be inadequate.
Given the patient is immunocompromised, this patient should be admitted for intravenous
antibiotics. If the patient was immunocompetent, then doxycycline could cover both septic
bursitis and pneumonia.
Given the patient's history, he likely has a Mycobacterium infection, which requires a 3 to 4
drug regimen.

Go to the next page if you knew the correct answer, or click the link image(s) below to further
research the concepts in this question (if desired).

Research Concepts:
Septic Bursitis

We update eBooks quarterly and Apps daily based on user feedback. Please tap flag to
report any questions that need improvement.
Question 419: A 70-year-old man undergoes surgical resection for prostate cancer. During
the early postoperative course, he develops hypotension and congestive heart failure requiring
intubation. His echocardiogram demonstrates akinesis of the basal segment and hyperkinesis of
the apical segment. Which of the following forms of cardiomyopathy does he most likely have?

Choices:
1. Reverse or inverted stress cardiomyopathy
2. Dilated cardiomyopathy
3. Idiopathic cardiomyopathy
4. Stress cardiomyopathy
Answer: 1 - Reverse or inverted stress cardiomyopathy
Explanations:
The clinical history and echocardiographic images are consistent with reverse (or inverted)
Takotsubo (stress cardiomyopathy) in which there are basal akinesis and hyperkinetic apex.
The stressor, in this case, was the recent surgery.
Several variants of Takotsubo cardiomyopathy have been described in the literature
including the classical apical ballooning (akinesis) variant with basal hyperkinesis, mid-
cavity ballooning, and basal–mid ballooning with apical hyperkinesis.
There are also variations with right ventricular apical involvement. This patient has left
ventricular involvement on the echocardiogram with the reverse variant.
Although males have much less predisposition for the development of Takotsubo
cardiomyopathy, they often have a worse prognosis because of the increased comorbidities
burden.

Go to the next page if you knew the correct answer, or click the link image(s) below to further
research the concepts in this question (if desired).

Research Concepts:
Takotsubo Cardiomyopathy

We update eBooks quarterly and Apps daily based on user feedback. Please tap flag to
report any questions that need improvement.
Question 420: A patient presents to the emergency department with symptoms of vomiting,
diarrhea, abdominal cramping, bronchospasm, miosis, bradycardia, excessive salivation, and
sweating. He also exhibits muscle fasciculations and weakness and has difficulty breathing. He is
treated with multiple doses of atropine and pralidoxime. This patient was most likely poisoned
with which of the following?

Choices:
1. Aldicarb
2. Aldrin
3. Parathion
4. Physostigmine
Answer: 3 - Parathion
Explanations:
Parathion is an organophosphate insecticide.
It acts on the enzyme acetylcholine esterase.
Symptoms of toxicity include blurred vision, diarrhea, excess salivation, tremor, dyspnea,
and unconsciousness.
Atropine is often given to block the excess acetylcholine activity. Pralidoxime is given to
reactivate the cholinesterase.

Go to the next page if you knew the correct answer, or click the link image(s) below to further
research the concepts in this question (if desired).

Research Concepts:
Organophosphate Toxicity

We update eBooks quarterly and Apps daily based on user feedback. Please tap flag to
report any questions that need improvement.
Question 421: A 66-year-old female with a past medical history of liver cirrhosis
complicated by ascites, hypertension, and type 2 diabetes presents to the emergency department
with a 3-day history of abdominal pain. She describes the pain as 6/10, limited to her right upper
quadrant, and associated with some nausea but no episodes of vomiting. A plain abdominal x-ray
shows free air under the diaphragm. An abdominal CT shows right bowel distension but no free
air. Which of the following is the next best step in the management of this patient?

Choices:
1. Exploratory laparotomy
2. Nasogastric tube decompression
3. Endoscopy
4. Colonoscopy
Answer: 2 - Nasogastric tube decompression
Explanations:
The absence of free air under the diaphragm on CT scan indicates that initial findings are
consistent with pneumoperitoneum.
This is classic of Chilaiditi syndrome, in which a segment of a large bowel loop or small
intestine is interposed between the liver and a diaphragm.
Treatment is conservative with nasogastric tube decompression, bed rest, IV fluids.
Surgical intervention is indicated when the patient does not respond to conservative
management, and either the obstruction fails to resolve, or there is evidence of bowel
ischemia.

Go to the next page if you knew the correct answer, or click the link image(s) below to further
research the concepts in this question (if desired).

Research Concepts:
Chilaiditi Syndrome

We update eBooks quarterly and Apps daily based on user feedback. Please tap flag to
report any questions that need improvement.
Question 422: A 78-year-old female came in the emergency with confusion and high-grade
fever. After her urine and blood specimens were sent to the laboratory, her blood pressure drops
to 89/60 mmHg. Suddenly she becomes out of breath with a respiratory rate of 30/min. Providers
are considering acute respiratory distress syndrome secondary to sepsis and pneumonia. They
ordered a chest x-ray and oxygen therapy, but they also wanted to start an empiric therapy for
pneumonia. Which of the following is the most likely recommended therapy for her?

Choices:
1. Macrolides
2. Doxycycline
3. Beta-lactam
4. Beta-lactam and a fluoroquinolone
Answer: 4 - Beta-lactam and a fluoroquinolone
Explanations:
In elderly patients sometimes confusion is the earliest manifestation of pneumonia.
Patients with community-acquired pneumonia are often risk stratified using the pneumonia
severity index or CURB-65 criteria to help empiric treatment and treatment location.
This patient with a score of 4 on CURB-65 criteria with confusion, age greater than 65
years, blood pressure less than 90/60 mmHg, and respiratory rate of 30/min, likely needs
admission in the intensive care unit and empiric therapy.
The recommended therapy for patients in the ICU is a beta-lactam and a fluoroquinolone or
a beta-lactam and a macrolide.

Go to the next page if you knew the correct answer, or click the link image(s) below to further
research the concepts in this question (if desired).

Research Concepts:
Bacterial Pneumonia

We update eBooks quarterly and Apps daily based on user feedback. Please tap flag to
report any questions that need improvement.
Question 423: A 23-year-old male patient is brought to the emergency department by his
family after he “drank something” in a suicide attempt. On arrival to the ED, he is confused,
sweaty, salivating, and has pinpoint pupils. His confusion, vomiting, excessive salivation, and
copious bronchorrhea make it hard to manage him. His GCS is 7. He is prepared to be intubated.
Which of the following medications is most appropriate to facilitate intubation?

Choices:
1. Etomidate and rocuronium
2. Etomidate and succinylcholine
3. Midazolam and succinylcholine
4. Pyridostigmine
Answer: 1 - Etomidate and rocuronium
Explanations:
Usual airway procedures are used with one exception. During intubation, succinylcholine
should not be used because the lack of acetylcholinesterase caused by the poisoning will
cause prolonged paralysis.
Depolarizing paralytics, such as succinylcholine, should not be used in cholinergic toxicity.
Non-polarizing paralytics, such as vecuronium and rocuronium, may be used safely.
The choice of induction agent is not changed when treating cholinergic toxicity. Etomidate,
midazolam, and propofol may all be used.
Rapid sequence intubation would give the best chance for first pass intubation in this patient
who is not completely comatose.

Go to the next page if you knew the correct answer, or click the link image(s) below to further
research the concepts in this question (if desired).

Research Concepts:
Cholinergic Toxicity

We update eBooks quarterly and Apps daily based on user feedback. Please tap flag to
report any questions that need improvement.
Question 424: Which of the following is the safest positioning to preserve neurologic
function prior to reduction and stabilization of a T12-L1 dislocation fracture with an incomplete
neurologic deficit?

Choices:
1. Supine
2. Lateral decubitus with lower extremities extended
3. Lateral decubitus with knees flexed
4. Prone with rolls under chest and thighs

Photo:Contributed by Scott Dulebohn, MD


Answer: 3 - Lateral decubitus with knees flexed
Explanations:
Patients in the supine position have an extension of the thoracolumbar spine at the level of a
fixed kyphosis secondary to the dislocation fracture.
This causes further narrowing of an already compromised spinal canal.
Placing the patient in the lateral decubitus position with the knees flexed may provide
enough decompression to allow improvement in an individual who is neurologically
incomplete.
The change in position required to turn a patient prone causes unnecessary manipulation of
the spinal column.

Go to the next page if you knew the correct answer, or click the link image(s) below to further
research the concepts in this question (if desired).

Research Concepts:
Thoracolumbar Spine Fracture

We update eBooks quarterly and Apps daily based on user feedback. Please tap flag to
report any questions that need improvement.
Question 425: A 58-year-old male is brought to the emergency department with the
complaint of palpitations and dizziness. Past medical history is significant for diabetes. Vital
signs are pulse rate 132/min, blood pressure 102/55 mm Hg, respiratory rate 21/min, and
temperature 98.6 F (37C). EKG shows polymorphic QRS complexes around an isoelectric line.
IV magnesium is given, but there is no improvement. What is the next step in the management of
this patient?

Choices:
1. Amiodarone
2. Beta-blocker
3. Isoproterenol
4. Adenosine

Photo:Contributed by Wikimedia Commons, Panthro (Public Domain-Self)


Answer: 3 - Isoproterenol
Explanations:
Torsades de Pointes is a type of polymorphic ventricular tachycardia characterized by a
gradual change in amplitude and twisting of the QRS complexes around an isoelectric line
on the electrocardiogram. Intravenous magnesium is the first-line pharmacologic therapy in
Torsades de Pointes.
For a patient that continues to have Torsades de Pointes, despite treatment with magnesium,
increasing the heart rate may also help. This can be done pharmacologically with
medications such as isoproterenol.
Isoproterenol has been shown to help prevent Torsades de Pointes in patients with
prolonged QT that is refractory to magnesium. It is a non-selective beta-agonist, which
increases the heart rate and shortens the QT interval. This lowers the likelihood of an R-on-
T phenomenon that can lead to TdP.
Isoproterenol can be given as an IV push of 10 mcg to 20 mcg or an infusion titrated to
maintain a heart rate of 100 bpm. Isoproterenol is, however, contraindicated in patients with
congenital prolonged QT, because it can paradoxically lengthen the QT interval.

Go to the next page if you knew the correct answer, or click the link image(s) below to further
research the concepts in this question (if desired).

Research Concepts:
Torsade de Pointes

We update eBooks quarterly and Apps daily based on user feedback. Please tap flag to
report any questions that need improvement.
Question 426: A 32-year-old woman presents after a fall on an icy sidewalk where she hit
her head and lost consciousness for several minutes. Her vital signs show oxygen saturation 98%
on room air, respiratory rate 20 per minute, heart rate 55 beats per minute, blood pressure 140/90
mmHg, and temperature 98 F. On examination, she is awake, alert, and complains of a severe
headache. There are no cranial nerve, motor, or sensory deficits. Babinski sign is positive on the
right side. The patient undergoes a CT scan (see below). In which cranial nerve would you
expect the patient to develop a deficit?

Choices:
1. CN I
2. CN III
3. CN IV
4. CN VI

Photo:Contributed by Scott Dulebohn, MD


Answer: 2 - CN III
Explanations:
The CT scan shows a subdural hematoma with subarachnoid hemorrhage on the right. The
complication would include an increase in the size of hemorrhage as well as edema of the
right temporal lobe. This may predispose to herniation of the medial temporal lobe leading
to uncal herniation.
Uncal herniation compresses CN III. The clinical presentation consists of a dilated
ipsilateral pupil which does not respond to light. On further compression, the affected eye
will be looking down and out.
Further herniation may compress the brainstem and cause a decrease in sensation.
The uncus and the adjacent temporal lobe slide down the tentorial incisura and compress the
posterior cerebral arteries and the brain stem. It carries a poor prognosis.

Go to the next page if you knew the correct answer, or click the link image(s) below to further
research the concepts in this question (if desired).

Research Concepts:
Uncal Herniation

We update eBooks quarterly and Apps daily based on user feedback. Please tap flag to
report any questions that need improvement.
Question 427: An 81-year-old man is brought to the emergency department due to several
hours of confusion, lethargy, vomiting, and abdominal pain. He resides in a nursing home, and
his oral intake has been poor over the last 2 days. He has a history of aplastic anemia and
underwent a bone marrow transplant recently. He also has a history of hypertension, type 2
diabetes mellitus, chronic kidney disease, and coronary artery disease. He has been taking her
medications without issue, but his long-acting insulin was held last night. Vital signs show
temperature 35.7 C (96.3 F), blood pressure 90/40 mmHg, pulse 98/min, and respiratory rate
18/min. Pulse oximetry shows 94% on room air. The patient is unable to answer questions or
follow commands. Cardiopulmonary examination is unremarkable. No edema is present in the
extremities, and the skin is warm to touch. There is mild pain on deep palpation on the right
lower quadrant. Neurologic examination is limited due to the patient's mental status, but no
obvious focal abnormalities are noted. EKG reveals normal sinus rhythm with minor T-wave
abnormalities. Laboratory results are shown below. The patient is given a 500mL bolus of
normal saline. Repeat vital signs reveal blood pressure 85/40 mmHg and heart rate 95/min.
Which of the following is the next best step in the management of this patient?

Choices:
1. CT scan of the abdomen
2. Intravenous albumin and broad-spectrum antibiotics
3. Emergency exploratory laparotomy
4. Normal saline boluses and broad-spectrum antibiotics

Photo:Contributed by Evelyn E Inga


Answer: 4 - Normal saline boluses and broad-spectrum antibiotics
Explanations:
This elderly patient with hypothermia, hypotension, and leukocytosis likely has an ongoing
infection, despite being an undetermined source, the physical examination suggests a septic
abdomen. The next step is to determine if the patient has sepsis, as early sepsis intervention
is associated with significant reductions in mortality.
This patient was under immunosuppression due to bone marrow transplantation after being
diagnosed with aplastic anemia. These antecedents plus the physical y laboratory findings
suggest neutropenic enterocolitis (NE). Timely conservative treatment frequently allows the
resolution of neutropenic enterocolitis without operation since this patient doesn't present
peritoneal signs. Although the conservative treatment follows a classic case of NE, the
diagnosis of sepsis demands aggressive intravenous fluid resuscitation and the early
initiation of antibiotics since these measures have been shown to reduce mortality.
Sepsis screening in the emergency department is most often performed with the qSOFA
score, a simple bedside calculation that assigns 1 point each for respiratory rate >22/min,
altered mentation, and systolic blood pressure 100 mm Hg. Patients with qSOFA scores >2
(as in this patient with confusion and hypotension) are likely to be septic. Two major initial
interventions reduce the risk of sepsis-related mortality: aggressive intravenous fluids and
early antibiotic therapy.
Intravenous albumin is expensive and has not been shown to be superior to crystalloid fluids
for the treatment of early sepsis. As such, albumin is not recommended. Colonoscopy is
contraindicated in neutropenic enterocolitis, but endoscopy may be life-saving in other
patients with GI bleeding. Surgery is reserved in cases of persistent decline in the patient's
evolution or any finding suggesting bowel perforation. This patient signs, symptoms, and
past medical history suggest sepsis - neutropenic enterocolitis. Sepsis is associated with
high rates of death. Aggressive intravenous fluid resuscitation and the early initiation of
antibiotics have been shown to reduce mortality. Empiric broad-spectrum antibiotics should
be administered within 1 hour of presentation (ideally after blood cultures have been
drawn).

Go to the next page if you knew the correct answer, or click the link image(s) below to further
research the concepts in this question (if desired).

Research Concepts:
Neutropenic Enterocolitis

We update eBooks quarterly and Apps daily based on user feedback. Please tap flag to
report any questions that need improvement.
Question 428: A 26-year-old woman presents with a six-month history of shortness of
breath. This symptom started one week after she was diagnosed with lung cancer. All her vital
signs are within normal limits. Chest x-ray shows moderate to large left-sided pleural effusion.
The patient states she has had fluid in her lung drained before. The appropriate procedure will be
performed. Which of the following is the most appropriate position for this intervention?

Choices:
1. Sitting
2. Supine
3. Left lateral decubitus
4. Right lateral decubitus

Photo:Contributed by National Heart, Lung and Blood Institute ( PD-US NIH)


Answer: 1 - Sitting
Explanations:
Thoracentesis is performed to remove fluid from the pleural cavity. Often even with stable
vital signs, patients may have orthopnea.
The preferred body position is sitting, with the patient leaning slightly forward and the arms
supported on a bedside table. In the sitting position, ultrasound can readily visualize large
fluid pockets much easier as well as identify organs of interest such as the diaphragm.
For most patients, the sitting position is well tolerated and provides ample access to the
back.
The lateral decubitus position can be used to insert a chest tube. The arm must be over the
shoulder to open up the intercostal space.

Go to the next page if you knew the correct answer, or click the link image(s) below to further
research the concepts in this question (if desired).

Research Concepts:
Thoracentesis

We update eBooks quarterly and Apps daily based on user feedback. Please tap flag to
report any questions that need improvement.
Question 429: A 35-year-old man is brought in by emergency services after an industrial
accident. There was a 60% total body surface area full-thickness burn, which was covering the
face, both hands, and the trunk. The patient was intubated en route and was started on
mechanical ventilation on arrival. A chest x-ray shows bilateral alveolar infiltrates, and a
fiberoptic bronchoscopy is performed that shows severe blisters and ulcers of the mucosa of the
bronchus. Which of the following can be an immediate complication in cases such as these?

Choices:
1. Subglottic stenosis
2. Bronchiectasis
3. Laryngeal edema
4. Acute respiratory distress syndrome (ARDS)

Photo:Contributed by Wikimedia Commons (CC BY 2.0) https://creativecommons.org/licenses/by/2.0/deed.en


Answer: 3 - Laryngeal edema
Explanations:
In inhalational injury, the severity of respiratory tract injury is directly proportional to the
length and intensity of exposure. Mild to moderate injury is largely self-limited with
patients having no complications. In these cases, symptoms will often resolve within 2 to 3
days.
Laryngeal edema, secondary to inhalation injury, can obstruct the airway and may
necessitate tracheostomy in severe cases.
Short-term complications are seen in more severe injuries within 4 to 5 days, and the most
common issue is pneumonia. Acute respiratory distress syndrome and pulmonary edema are
also seen in the short term.
Long-term complications from smoke inhalation injury are much less common. They
include subglottic stenosis, bronchiectasis, and bronchiolitis obliterans.

Go to the next page if you knew the correct answer, or click the link image(s) below to further
research the concepts in this question (if desired).

Research Concepts:
Inhalation Injury

We update eBooks quarterly and Apps daily based on user feedback. Please tap flag to
report any questions that need improvement.
Question 430: A 44-year-old man is brought to the emergency department from a paint
factory, where he was overcome by fumes. An oropharyngeal airway (OPA) is in place. Breath
sounds are present bilaterally. Which of the following statements is true?

Choices:
1. If upper airway bleeding occurs, the OPA should be left in place to control bleeding
2. The OPA should not be used in apneic comatose patients
3. Ventilation is as efficient and controlled with an OPA as with an endotracheal tube
4. OPA is contraindicated when the patient has a gag reflex

Photo:Contributed by Tammy J. Toney-Butler, RN, AS, CEN, TCRN, CPEN


Answer: 4 - OPA is contraindicated when the patient has a gag reflex
Explanations:
Since the airway is not protected by an oropharyngeal airway (OPA), upper airway bleeding
will allow blood into the airways.
OPAs are not designed to control bleeding.
The gag reflex generally causes bleeding and therefore is a contraindication to use of an
OPA.
An oropharyngeal airway (oral airway, OPA) is an airway adjunct used to maintain or open
an airway by preventing the tongue from covering the epiglottis. When a patient is
unconscious, the muscles in their jaw relax and allow the tongue to obstruct the airway.
Oropharyngeal airways come in sizes for infants to adults, and are used commonly in pre-
hospital emergency care and for short-term airway management post or when manual
methods are inadequate to maintain an open airway. Oropharyngeal airways are indicated
only in unconscious patients because of the likelihood they will stimulate a gag reflex and
vomiting in a conscious person. Nasopharyngeal airways are used instead as they do not
stimulate a gag reflex. Oropharyngeal airways must be sized and inserted correctly to
maximize effectiveness and minimize complications. The correct OPA size is chosen by
measuring from the first incisors to the angle of the jaw. The airway is inserted into the
mouth upside down. Once contact is made with the back of the throat, the airway is rotated
180 degrees, allowing insertion, and assuring the tongue is secured. The device is removed
when the patient regains swallow reflex and can protect their own airway. It is removed
pulling on it without rotation. Use of an OPA does not remove the need for the recovery
position and ongoing airway assessment, and it does not prevent obstruction by liquids or
the closing of the glottis.

Go to the next page if you knew the correct answer, or click the link image(s) below to further
research the concepts in this question (if desired).

Research Concepts:
Oropharyngeal Airway

We update eBooks quarterly and Apps daily based on user feedback. Please tap flag to
report any questions that need improvement.
Question 431: A 65-year-old man presents to the emergency department with complaints of
severe crushing central chest pain radiating to the left arm, sweating, and vomiting. He has a
history of hypertension, for which he takes amlodipine 10 mg once a day. On examination, his
blood pressure is 80/55 mmHg, and there are fine bibasal crackles on auscultation of the chest.
An electrocardiogram is obtained which is shown below. Despite oxygen supplementation, he is
still dyspneic. What is the next best step in his management?

Choices:
1. Dopamine
2. Transcutaneous pacing
3. Digoxin
4. Internal defibrillator

Photo:By MoodyGroove at English Wikipedia - Own work, Public Domain, https://commons.wikimedia.org/w/index.php?curid=18055698


Answer: 2 - Transcutaneous pacing
Explanations:
Dopamine infusion can be attempted but is often not successful in third-degree
atrioventricular block.
In patients with symptomatic bradycardia, transcutaneous pacing is indicated without delay.
Although digoxin can cause bradycardia, there is no role for digoxin in treating AV node
block with symptomatic bradycardia.
The patient should be stabilized before a permanent pacemaker is placed.

Go to the next page if you knew the correct answer, or click the link image(s) below to further
research the concepts in this question (if desired).

Research Concepts:
Third-Degree Atrioventricular Block

We update eBooks quarterly and Apps daily based on user feedback. Please tap flag to
report any questions that need improvement.
Question 432: A 72-year-old man presents to the hospital with shortness of breath. Past
medical history is significant for COPD on 2 L/min of oxygen by nasal cannula. The patient is
hemodynamically stable, saturating 94% on 2 L/min of the nasal cannula. CT thorax shows a
right-sided pleural effusion. The echocardiogram is unremarkable. The patient undergoes
diagnostic and therapeutic thoracocentesis. Light's criteria is suggestive of transudate effusion.
The patient says his symptoms are not improved. Repeat CT thorax post thoracocentesis is
shown below. Which of the following is the most likely diagnosis?

Choices:
1. Lung cancer
2. Aspergillosis
3. Pneumothorax
4. Trapped lung

Photo:Contributed by Albores J, Wang T. Images in clinical medicine. Trapped lung. N Engl J Med. 2015 May 7;372(19):e25. doi: 10.1056/NEJMicm1404964.
PMID: 25946304. (Open Access)
Answer: 4 - Trapped lung
Explanations:
The CT is showing chronic atelectasis with thickened visceral pleura. Following drainage of
the effusion, the lung failed to expand, creating a vacuum and an air-fluid level.
Malignancy is one of the causes of recurrent pleural effusions and trapped lung.
Trapped lung can present as one of the complications of pneumothorax.
Complicated pulmonary infections causing loculated effusions can cause trapped lung.

Go to the next page if you knew the correct answer, or click the link image(s) below to further
research the concepts in this question (if desired).

Research Concepts:
Trapped Lung

We update eBooks quarterly and Apps daily based on user feedback. Please tap flag to
report any questions that need improvement.
Question 433: In which electrolyte abnormality in the advanced stage would you expect to
see a widening of the QRS and decreased P wave amplitude?

Choices:
1. Hypocalcemia
2. Hypercalcemia
3. Hyperkalemia
4. Hypomagnesemia

Photo:Contributed by Wikimedia Commons,"Medical gallery of Mikael Häggström 2014" (Public Domain)


Answer: 3 - Hyperkalemia
Explanations:
The presence of peaked T waves on an ECG should suggest hyperkalemia.
Severe hyperkalemia may also be associated with wide QRS complexes and reduction in the
size of P waves.
Hyperkalemia causes an overall membrane depolarization that inactivates many sodium
channels.
Peaked T waves are the earliest changes on the ECG in patients with hyperkalemia. This is
usually associated with a short QT interval and ST segment depression. As the
hyperkalemia worsens, one will develop widening of the QRS and decreased P wave
amplitude.

Go to the next page if you knew the correct answer, or click the link image(s) below to further
research the concepts in this question (if desired).

Research Concepts:
Hyperkalemia

We update eBooks quarterly and Apps daily based on user feedback. Please tap flag to
report any questions that need improvement.
Question 434: A 31-year-old male is seen in the emergency department and undergoes a
quick neurological evaluation following head trauma. He is unable to open his eyes, has no
motor movement, and makes no verbal sounds. What is his Glasgow coma scale score?

Choices:
1. Seven
2. Five
3. Three
4. Zero

Photo:Contributed by Tammy Toney-Butler, AS, RN, CEN, TCRN, CPEN


Answer: 3 - Three
Explanations:
The Glasgow coma scale is a neurological measure that gives a reliable estimate of the
conscious state of an individual.
The lowest score is three and the highest is 15.
The scale assesses the best eye, best verbal, and best motor responses.
A severe score is any number less than 8. A moderate score is between 9 and 12. A minor
score is greater than 13.

Go to the next page if you knew the correct answer, or click the link image(s) below to further
research the concepts in this question (if desired).

Research Concepts:
Glasgow Coma Scale

We update eBooks quarterly and Apps daily based on user feedback. Please tap flag to
report any questions that need improvement.
Question 435: A 52-year-old man is brought in by emergency medical services (EMS) after
being involved in a motor vehicle accident. When performing the Glasgow coma scale (GCS),
there is slow flexion of the elbow, wrist, and fingers with adduction and internal rotation at the
shoulder. The legs are extended and internally rotated at the hip, with the extension of the knee
and plantar flexion of the feet. Which of the following is the most likely position of the toes in
this posturing?

Choices:
1. Adducted and hyperextended
2. Abducted and hyperextended
3. Abducted and flexed
4. Adducted and flexed

Photo:Contributed by Wikimedia Commons (Public Domain)


Answer: 2 - Abducted and hyperextended
Explanations:
Decorticate posturing is described as abnormal flexion of the arms with the extension of the
legs.
It involves slow flexion of the elbow, wrist, and fingers with adduction and internal rotation
at the shoulder.
The lower limbs show extension and internal rotation at the hip, extending the knee and
plantar flexion of the feet. Toes are typically abducted and hyperextended.
Decorticate posturing is a clinical diagnosis and should be considered a pathological sign of
a neurological injury.

Go to the next page if you knew the correct answer, or click the link image(s) below to further
research the concepts in this question (if desired).

Research Concepts:
Decerebrate And Decorticate Posturing

We update eBooks quarterly and Apps daily based on user feedback. Please tap flag to
report any questions that need improvement.
Question 436: A 60-year-old male complains of left-sided weakness that started ten hours
ago. The patient cannot provide a clear, detailed medical history because his speech is slurred.
According to the patient's medical file that he brought with him, he has a past medical history of
hypertension and diabetes mellitus type 2. His medications include hydrochlorothiazide and
metformin. Further information cannot be elicited. The patient was admitted to the hospital for
further evaluation. On examination, temperature is 36.3 C, blood pressure is 184/100 mmHg,
pulse is 84 beats/min, and respiratory rate is 17 breaths/min. He cannot move the left side of his
body. Sensation is lost on the left side of his body. Cardiovascular examination is unremarkable,
and there are no carotid bruits. CT scan of the brain is shown below. An electrocardiogram
shows left ventricular hypertrophy with non-specific T wave changes. Blood tests show WBC of
6,000/mm3, hemoglobin of 13.8 g/dL, platelets of 270,000/UL, calcium of 9 mg/dL, blood urea
nitrogen of 21 mg/dL, creatinine of 90 U/L, and an INR of 1.1. A bedside swallowing screening
test revealed no abnormalities. The patient was given oral aspirin. What is the best next step in
the management of this patient?

Choices:
1. Low molecular weight heparin for deep venous thrombosis prevention
2. Calcium channel blocker to treat hypertension
3. Observation
4. Intravenous mannitol to reduce cerebral edema

Photo:Contributed by Scott Dulebohn, MD


Answer: 1 - Low molecular weight heparin for deep venous thrombosis prevention
Explanations:
This patient complaining of left-sided hemiparesis and sensory loss on the left side of his
body with a CT scan showing hypodensity in the right middle cerebral artery territory likely
had a right middle cerebral artery ischemic stroke.
Patients who present with a stroke less than 4.5 hours after symptom onset should be given
fibrinolytic therapy. Patients who present after 4.5 hours of symptom onset should not be
given fibrinolytic therapy and should be managed with aspirin and permissive hypertension.
A blood pressure up to 220/120 mmHg is permitted in patients who did not receive
fibrinolytic therapy.
A swallow screening test should be done in all patients with a stroke before administering
oral medications or food to avoid aspiration.
All patients with ischemic stroke should receive subcutaneous heparin or low molecular
weight heparin for deep vein thrombosis prophylaxis. Patients with hemorrhagic stroke
should be given mechanical deep vein thrombosis prophylaxis, instead of heparin.
Pulmonary embolism from deep vein thrombosis is the most common cause of early death
in patients with acute stroke.

Go to the next page if you knew the correct answer, or click the link image(s) below to further
research the concepts in this question (if desired).

Research Concepts:
Middle Cerebral Artery Stroke

We update eBooks quarterly and Apps daily based on user feedback. Please tap flag to
report any questions that need improvement.
Question 437: What is the total Glasgow coma scale score for an adult patient who opens
their eyes when asked, uses inappropriate words, and withdraws in response to pain?

Choices:
1. 8
2. 9
3. 10
4. 11

Photo:Contributed by Tammy Toney-Butler, AS, RN, CEN, TCRN, CPEN


Answer: 3 - 10
Explanations:
The total Glasgow coma scale score is 10 out of 15.
He receives three points for opening his eyes when asked.
He receives three points for speaking with inappropriate words.
He receives four points for withdrawing to pain. If the patient opened his eyes to a verbal
request, that is a movement following a command and this is 6 points.

Go to the next page if you knew the correct answer, or click the link image(s) below to further
research the concepts in this question (if desired).

Research Concepts:
Glasgow Coma Scale

We update eBooks quarterly and Apps daily based on user feedback. Please tap flag to
report any questions that need improvement.
Question 438: A 65-year-old male presents to the emergency department with multiple
injuries following a road traffic accident. His cervical spine is immobilized, and high flow
oxygen is started. His breathing is noisy and labored, and oxygen saturation is 90% on 15 L O2.
His heart rate is 112 bpm, and blood pressure is 102/60 mmHg. On examination, the patient's
eyes are closed, and he is not making any verbal communications. With supraorbital pressure, he
flexes at the elbow with arms brought close to his body and flexed wrists. The same stimulus
causes him to open his eyes and moan. Pupils are equal and reactive to light. What is the next
best step in management?

Choices:
1. Perform a focused assessment with sonography in trauma (FAST)
2. Transfuse crossmatched packed red blood cells
3. Rapid sequence induction and intubation
4. Start levetiracetam as seizure prophylaxis

Photo:Image courtesy S Bhimji MD


Answer: 3 - Rapid sequence induction and intubation
Explanations:
The patient has a Glasgow coma score of 7 with eyes 2, verbal 2, and motor 3. With this
level of consciousness, his airway is compromised, demonstrated by his labored breathing.
His oxygen saturations are 90% despite 15 L of high flow oxygen. Therefore, it is best
practice to intubate and ventilate to stabilize his airway and breathing. Once stable and safe,
the patient can be transferred for CT evaluation.
The adduction of the arms with flexion at the elbows and wrists is abnormal posturing.
Specifically, decorticate posturing. This gives a motor score of 3 on the Glasgow coma
scale (GCS).
Decorticate posturing implies injury to the corticospinal tracts resulting in disinhibition of
red nucleus reflexes causing flexion posturing of the upper limbs and extension posturing of
the lower limbs.
With a patient in a coma and a GCS of 8 or less, it is necessary to intubate and ventilate,
thus protecting their airway and breathing. Obtaining a FAST scan to assess for abdominal
trauma, initiating blood transfusion for presumed hemorrhagic shock, and initiating seizure
prophylaxis are all important steps in this patient's management. However, they are all
secondary to securing the airway.

Go to the next page if you knew the correct answer, or click the link image(s) below to further
research the concepts in this question (if desired).

Research Concepts:
Decerebrate And Decorticate Posturing

We update eBooks quarterly and Apps daily based on user feedback. Please tap flag to
report any questions that need improvement.
Question 439: A 16-year-old boy is admitted to the intensive care unit due to suspected
sepsis. A radial artery catheter is inserted for continuous blood pressure monitoring. The nurse
reports that the arterial waveform seems underdamped. Which of the following best describes the
expected changes in blood pressure readings due to this issue?

Choices:
1. Systolic blood pressure would be unchanged, diastolic blood pressure lower
2. Systolic blood pressure higher and diastolic blood pressure would be lower
3. Systolic blood pressure lower and diastolic blood pressure would be higher
4. Systolic blood pressure would be higher and diastolic would be unchanged

Photo:Contributed by Divij Pasrija


Answer: 2 - Systolic blood pressure higher and diastolic blood pressure would be lower
Explanations:
Dampening of the arterial blood pressure waveform implies loss or gain of energy in any
part of the circuit used for transduction. This starts from the arterial catheter to the snap tab
device and tubing and the transducer.
Underdampening implies a gain of energy, and in this case, systolic blood pressure would
be higher, whereas diastolic would be slightly lower. Overdampening implies a loss in
energy, which results in a lower reading of systolic blood pressure peak, and the diastolic
blood pressure is higher. Dampening of the arterial pressure waveform can be fixed by
recalibrating/zeroing the circuit, with a heparinized saline flush in case there is a small
occlusion, or it might require re-inserting a new catheter in the artery.
A gain in energy or excessive whip, which is exaggerated high systolic pressure, can also
manifest on arterial line waveform if the tip of the arterial catheter keeps hitting the vessel
wall with each transmitted pulsation.
To prevent dampening in the circuit, the arterial catheter must be well secured. The tubing
should be non-compliant; there should be a constant flow of a heparinized solution through
the tubing.

Go to the next page if you knew the correct answer, or click the link image(s) below to further
research the concepts in this question (if desired).

Research Concepts:
Arterial Lines

We update eBooks quarterly and Apps daily based on user feedback. Please tap flag to
report any questions that need improvement.
Question 440: A 32-year-old male presents to the ED with right lower quadrant abdominal
pain of acute onset and persistent nausea and vomiting. He has no significant medical, surgical,
or family history and does not take home medications. During the physical exam, he is
significantly uncomfortable with severe rebound tenderness over the right lower quadrant. His
vital signs are within normal limits except for a low-grade fever, and a temperature of 39 degrees
Celsius. His laboratory results revealed leukocytosis with normal hemoglobin and platelet
counts. A CMP and Lipase were within normal ranges. A CT scan of the abdomen with contrast
revealed signs consistent with acute appendicitis. A surgeon evaluated the patient and while
planning for emergent surgery ordered a routine EKG. A consult was placed to internal medicine
for evaluation of the rhythm seen below. What is the following rhythm?

Choices:
1. Respiratory sinus arrhythmia
2. Atrial fibrillation
3. Sinus rhythm
4. Third degree AV block

Photo:Contributed by Michael Soos


Answer: 1 - Respiratory sinus arrhythmia
Explanations:
Sinus arrhythmia is characterized by an irregular rate with R-R interval variation greater
than 0.12 seconds. Additionally, P waves are typically monoform and in a pattern consistent
with atrial activation originating from the sinus node.
Currently, sinus arrhythmia is thought to be a normal physiologic variation of sinus rhythm
found in young healthy individuals; recent studies have revealed a reduction in its
prevalence in the elderly. Typically this rhythm does not require additional evaluation or
workup by a Cardiologist.
Respiratory sinus arrhythmia is a common finding. There are 3 types of sinus arrhythmia,
the most common of which is respiratory sinus arrhythmia.
In nonrespiratory sinus arrhythmia, electrocardiograms will appear similar to the respiratory
type. The two differ in that nonrespiratory sinus arrhythmia is not associated with the
respiratory cycle.

Go to the next page if you knew the correct answer, or click the link image(s) below to further
research the concepts in this question (if desired).

Research Concepts:
Sinus Arrhythmia

We update eBooks quarterly and Apps daily based on user feedback. Please tap flag to
report any questions that need improvement.
Question 441: A 65-year-old male, suffering from diabetes mellitus type 2 and
dyslipidemia, presented to the emergency department with complaints of heavy retrosternal chest
pain, diaphoresis, and nausea. His EKG was remarkable for ST elevations in leads V1, V2, and
V3. The patient was taken to the coronary cath lab in 20 minutes with successful
revascularization of the left anterior descending artery. The patient was admitted to the intensive
care unit after the successful PCI. Two hours later, the patient developed this rhythm (see
below). However, he was asymptomatic with a blood pressure of 115/75 mmHg, a heart rate of
92/min, a respiratory rate of 16/min, and SPO2 of 97%. What is the next best step in the
management of this rhythm?

Choices:
1. Watchful waiting
2. Start amiodarone
3. Cardioversion
4. Activate the cath lab

Photo:Contributed by Abdulrahman S Museedi, MD


Answer: 1 - Watchful waiting
Explanations:
Accelerated idioventricular rhythm is a very common phenomenon observed on EKG after
revascularization.
Accelerated idioventricular rhythm is a benign rhythm and the risk to develop ventricular
tachycardia is very low.
No treatment is required for accelerated idioventricular rhythm other than watching.
The heart rate is the key to differentiate the accelerated idioventricular rhythm from
ventricular tachycardia (accelerated idioventricular rhythm 100 and ventricular tachycardia
> 100).

Go to the next page if you knew the correct answer, or click the link image(s) below to further
research the concepts in this question (if desired).

Research Concepts:
Acute ST Elevation Myocardial Infarction

We update eBooks quarterly and Apps daily based on user feedback. Please tap flag to
report any questions that need improvement.
Question 442: When can Physician Orders for Life-Sustaining Treatment (POLST) or
Medical Orders for Scope of Treatment (MOST) forms be completed?

Choices:
1. Only when an attorney is present for validation
2. Only when the patient is deemed competent
3. When the patient or surrogate can understand the implications doing or not doing a particular
treatment
4. When all of the next of kin agree on the course of treatment
Answer: 3 - When the patient or surrogate can understand the implications doing or not
doing a particular treatment

Explanations:
Advance directives are instructions given by a patient expressing their wishes about their
medical care, including living wills, POLST/POST/MOLST/MOST forms, and Durable
Power of Attorney (DPOA) for healthcare. For a living will or DPOA to be valid, the
patient must have decision-making capacity (determined by a physician) to sign it, i.e., they
can understand the risks, benefits, and implications of doing or not doing a particular
treatment; however, the patient does not have to be deemed competent (a legal decision
made by a judge).
A Living Will is a legal document, and the patient must have the decision-making capacity
to complete it. An attorney may be used but is not required in most, if not all, states. The
POLST, etc. forms may contain the same directives that are in a Living Will, but they are in
the form of medical orders that are actionable by healthcare professionals. The patient OR
the surrogate must have the decision-making capacity to complete the POLST or MOST
with the healthcare provider. In short, the patient must complete a Living Will; whereas, the
patient and/or the surrogate may complete a POLST form.
The most desirable time for the execution of an advance directive is when the patient has
decision-making capacity while in a stable rather than an urgent or emergent condition.
However, if the patient has not completed an advance directive and the patient's medical
condition warrants, the family/next-of-kin/surrogate/guardian may have to make the
medical decisions and can complete a POLST form.
Oral advance directives require only that the patient has made an oral statement regarding
their wishes. Validity is strengthened if the patient was clear and specific in their wishes and
if the decision was repeated to multiple people over time.

Go to the next page if you knew the correct answer, or click the link image(s) below to further
research the concepts in this question (if desired).

Research Concepts:
Advance Directives

We update eBooks quarterly and Apps daily based on user feedback. Please tap flag to
report any questions that need improvement.
Question 443: A 72-year-old male is undergoing percutaneous coronary intervention. Due
to concerns for hemodynamic instability, a radial arterial line is placed. During the procedure, the
cardiologist is able to measure the pressure in the ascending aorta using the angiography
catheter. Compared with aortic pressure, radial artery pressure exhibits which of the following?

Choices:
1. Higher mean pressure
2. Wider pulse pressure
3. Decreased resonance
4. Earlier dicrotic notch
Answer: 2 - Wider pulse pressure
Explanations:
As one moves away from the aorta, there is an increasing number of branch points in the
arterial tree. This results in an increase in pressure wave reflections and greater resonance.
The BP distal to the aorta will have a higher SBP and lower DBP due to increased
resonance. As a result, there is a wider pulse pressure.
The distance from the aorta affects the SBP and DBP but does not affect the mean arterial
pressure, so this will be the same at the aorta and the radial artery.
Moving further away from the heart causes a wider pulse pressure and greater delay in the
dicrotic notch.

Go to the next page if you knew the correct answer, or click the link image(s) below to further
research the concepts in this question (if desired).

Research Concepts:
Arterial Pressure Monitoring

We update eBooks quarterly and Apps daily based on user feedback. Please tap flag to
report any questions that need improvement.
Question 444: A 65-year-old female with a history of paroxysmal atrial fibrillation and
hypertension presents with an episode of 6 hours duration. The patient denies chest pain or
shortness of breath. Medications include lisinopril and aspirin. Her blood pressure is 105/75
mmHg. ECG shows atrial fibrillation with a ventricular response rate of 150 without acute
changes. Select appropriate management.

Choices:
1. Intravenous metoprolol followed by oral metoprolol
2. Urgent cardioversion
3. Heparinization and then cardioversion
4. Warfarin
Answer: 1 - Intravenous metoprolol followed by oral metoprolol
Explanations:
The first step in treatment should be to reduce the ventricular rate with a beta blocker.
Emergent cardioversion is only needed if the patient is not stable.
As the fibrillation has only been present for 6 hours, anticoagulation before elective
cardioversion is not needed if it is done immediately.
Anticoagulation should be continued after cardioversion for at least 4 weeks.

Go to the next page if you knew the correct answer, or click the link image(s) below to further
research the concepts in this question (if desired).

Research Concepts:
Atrial Fibrillation

We update eBooks quarterly and Apps daily based on user feedback. Please tap flag to
report any questions that need improvement.
Question 445: A 16-year-old asthmatic from India who has been stable on inhaled
bronchodilators and corticosteroids has been having frequent exacerbations since last year. His
provider added a few more drugs to his prescription and advised a close follow up which he
failed to have as they moved to the US afterward. Now he has presented to the emergency
department in an agitated state complaining of palpitations, chest tightness, abdominal pain, and
feeling of the sinking of the heart. He is extremely short of breath and had one episode of
vomiting on his way to the hospital. On examination, there is a restless young male with severe
dyspnea, tachycardia, tachypnea, and hypotension. His electrocardiogram reveals
supraventricular tachycardia. While taking his blood samples he starts yelling that he can see a
wolf howling in front of him. Which of the following is the best treatment plan for him?

Choices:
1. Isotonic saline (20 mL/kg), ondansetron, cardiac cardioversion, and involve ICU
2. Hemodialysis, endotracheal intubation, gastric lavage, and cardiac defibrillation
3. Isotonic saline (20 mL/kg), benzodiazepine, haloperidol, and diphenhydramine
4. Just IV hydration is needed
Answer: 1 - Isotonic saline (20 mL/kg), ondansetron, cardiac cardioversion, and involve ICU
Explanations:
Around the world, theophylline (1,3-dimethylxanthine) is primarily used as a bronchodilator
for patients with asthma and chronic obstructive pulmonary disease (COPD). However, in
the United States, asthma and COPD are mainly treated with other agents, and theophylline
is largely used to treat bradycardia and apnea in premature newborns. Theophylline causes
endogenous release catecholamines through indirect stimulation of beta-1 and beta-2
receptors, which at therapeutic levels cause desired bronchodilation. Unfortunately,
theophylline has a narrow therapeutic window, and even levels slightly above this
therapeutic window can have many adverse effects in the setting of acute and chronic
toxicity. Serum levels between 5-15 mcg/mL of theophylline are considered safe
Theophylline has an extremely narrow therapeutic window. Theophylline toxicity occurs
when serum theophylline levels surpass the levels in the therapeutic range. This can occur
by intentional overdose or unintentionally when metabolism and/or clearance of
theophylline is altered due to certain physiological stressors.
Theophylline has 2 primary mechanisms. One mechanism is that theophylline blocks
adenosine receptors, which has both therapeutic and toxic effects such as bronchodilation,
tachycardia, cardiac arrhythmias, seizures, and cerebral vasoconstriction. At larger doses,
theophylline inhibits phosphodiesterase causing increased cyclic adenosine monophosphate
resulting in increased levels of adrenergic activation and catecholamine release. In
theophylline toxicity, epinephrine levels can be 4- to 8-times higher than normal, and
norepinephrine concentrations can be 4- to 10-times higher than normal. Increased
catecholamine concentrations have a variety of adverse effects such as cardiac arrhythmias,
metabolic acidosis, hyperglycemia, and hypokalemia.
Most patients with theophylline toxicity are successfully managed with supportive care.
Airway, breathing, circulation, and hemodynamic monitoring are essential to the care of
patients with theophylline toxicity. Intubation with ventilator support may be required for
airway protection.

Go to the next page if you knew the correct answer, or click the link image(s) below to further
research the concepts in this question (if desired).

Research Concepts:
Theophylline Toxicity

We update eBooks quarterly and Apps daily based on user feedback. Please tap flag to
report any questions that need improvement.
Question 446: A 43-year-old male is found outdoors, several hours following a traumatic
deep venous thrombosis after his left lower extremity was compressed by machinery. Following
removal of the machinery, he suffers a subsequent pulmonary embolism, is stabilized in transit to
the hospital, and transported to the intensive care unit (ICU). He arrives with the following vital
signs: pulse 56/minute, blood pressure 110/67 mmHg, respirations 18/minute, temperature 31 C
(87.8 F), and pulse oximetry 89%. Upon completion of arterial blood gas processing, it is
determined that his partial pressure of oxygen (PO2) values are much higher than anticipated.
Which of the following disease states is most likely to be documented as part of this patient’s
history?

Choices:
1. Remote renal cell carcinoma status post nephrectomy
2. Iron deficiency anemia
3. Polycythemia vera
4. Sickle cell anemia
Answer: 3 - Polycythemia vera
Explanations:
Many algorithms related to blood gas correction do not consider the underlying hematocrit
(HCT) of the patient. Patients with higher HCT will have a more robust increase of oxygen
partial pressure upon warming from 31 to 37 degrees as there is more oxyhemoglobin to
release O2 molecules as P50 (hemoglobin-oxygen affinity indicator) increases.
Polycythemia vera and other viscous states also predispose patients to thrombosis.
Polycythemia vera is a state of erythrocytosis and will demonstrate a proportionally larger
increase of partial pressure of oxygen (PO2) upon warming compared to a person with a
normal hematocrit. This effect is magnified compared to anemic states.
Sickle cell anemia and malignancy can also increase the likelihood of thrombosis. However,
sickle cell anemia patients and patients lacking the erythropoietic potential of a kidney are
more likely to suffer from low hematocrit.

Go to the next page if you knew the correct answer, or click the link image(s) below to further
research the concepts in this question (if desired).

Research Concepts:
Blood Gas Temperature Correction

We update eBooks quarterly and Apps daily based on user feedback. Please tap flag to
report any questions that need improvement.
Question 447: A 45-year-old female is transferred to the floor from the neurosurgery
intensive care unit after having a craniotomy for clipping of a ruptured cerebral aneurysm 5 days
ago. At transfer, she was awake, alert, able to follow commands. She moves all extremities
equally and has no sensory deficit. This patient should be monitored closely for which of the
following complications?

Choices:
1. Secondary hemorrhage
2. Cerebral edema
3. Cerebral infarction
4. Meningitis
Answer: 3 - Cerebral infarction
Explanations:
Delayed ischemia secondary to vasospasm can happen in up to 1/3 of patients with
subarachnoid hemorrhage.
Vasospasm can lead to ischemic brain injury (delayed ischemia) as a result of the restricted
blood flow caused by vessel constriction.
Vasospasm usually occurs 5-10 days after subarachnoid hemorrhage. In a setting of acute
onset neurologic deficits in a patient with subarachnoid hemorrhage, transcranial Doppler,
or cerebral angiography can be used to diagnose vasospasm.
Calcium channel blockers like nimodipine or nicardipine are often used to prevent
vasospasm.

Go to the next page if you knew the correct answer, or click the link image(s) below to further
research the concepts in this question (if desired).

Research Concepts:
Cerebral Aneurysm

We update eBooks quarterly and Apps daily based on user feedback. Please tap flag to
report any questions that need improvement.
Question 448: A 35-year-old lady is brought to the emergency department complaining of
shortness of breath and swelling of legs for the past two weeks. She also complains of occasional
dysphagia while taking solid food and has also noticed hoarseness of her voice. A detailed
history reveals that she is twenty-four weeks pregnant. Her blood pressure is 90/60 mmHg, pulse
102/min and temperature is 36.9 C (98.42 F). ECG was done which showed atrial fibrillation and
a biphasic P wave in leads V1 and V2. A chest x-ray was done which shows straightening of the
left heart border. Which one of the following is the most likely finding on physical examination
of this patient?

Choices:
1. Pan-systolic murmur
2. Crescendo-decrescendo murmur
3. Diastolic murmur with an opening snap
4. Early, blowing diastolic murmur
Answer: 3 - Diastolic murmur with an opening snap
Explanations:
The clinical scenario is consistent with the diagnosis of mitral stenosis. Pregnancy is a risk
factor for the development of mitral stenosis as it is associated with a 50% increase in the
plasma volume.
Mitral stenosis presents with some unique features like hoarseness and dysphagia due to the
enlarged left atrium pressing on the laryngeal nerve and esophagus respectively.
On examination, an opening snap is heard followed by a diastolic rumble.
Left atrial hypertrophy shows up as a biphasic P wave in leads V1 and V2.

Go to the next page if you knew the correct answer, or click the link image(s) below to further
research the concepts in this question (if desired).

Research Concepts:
Mitral Stenosis

We update eBooks quarterly and Apps daily based on user feedback. Please tap flag to
report any questions that need improvement.
Question 449: A male is examined by a provider for heart problems. The provider reclines
the patient and holds his arm up vertically. He then notices and feels a tapping impulse
transmitted through the bulk of the muscles. Based on these findings, what condition does this
patient have?

Choices:
1. Aortic stenosis
2. Aortic regurgitation
3. Thoracic outlet syndrome
4. Raynaud syndrome
Answer: 2 - Aortic regurgitation
Explanations:
A water hammer pulse is bounding and very forceful. A water hammer pulse is often seen
in aortic regurgitation because of the increased stroke volume of the left ventricle. The wide
pulse pressure often results in a large amount of blood being pumped out during each
heartbeat.
In patients with asymptomatic left ventricular dysfunction, more than 25% develop
symptoms in less than one year. Cardiac function usually worsens more rapidly after the
appearance of symptoms. Mortality may be more than 10% per year.
Symptoms can include shortness of breath, chest pain, and palpitation.
Other disorders with water hammer pulse include patent ductus arteriosus (PDA),
pregnancy, fever, and anemia.

Go to the next page if you knew the correct answer, or click the link image(s) below to further
research the concepts in this question (if desired).

Research Concepts:
Aortic Regurgitation

We update eBooks quarterly and Apps daily based on user feedback. Please tap flag to
report any questions that need improvement.
Question 450: A 65-year-old patient had been intubated and managed in ventilatory
support following acute exacerbation of chronic obstructive pulmonary disease (COPD)
complicated with severe pneumonia. The patient had already failed two extubation trials
previously. Now the patient has good self respiratory efforts with minimal ventilatory support of
positive end-expiratory pressure (PEEP) of 8 and fraction of inspired oxygen (FiO2) 0.50. He is
hemodynamically stable and has also passed the spontaneous breathing trial (SBT). What is the
recommended next step in the management of the patient?

Choices:
1. Extubate the patient
2. Tracheostomy
3. Administration of methylprednisolone just prior to extubation
4. Cuff leak test
Answer: 4 - Cuff leak test
Explanations:
The patients who have been intubated for greater than 6 days, had failed previous
extubation, and with traumatic intubation are at high risk for post-extubation stridor and
failed extubation. These patients should ideally undergo a cuff leak test prior to extubation.
The cuff leak test is performed by the expired tidal volume following the deflation of the
tube cuff. The leak is the difference in the tidal volume of the patient with and without the
cuff.
A significant leak of air above 110 ml ensures a sufficient cross-sectional area of the
airway, thereby minimizing the occurrence of post-extubation stridor in the patient.
The patient with a low leak in a cuff leak test should only be considered for undergoing
tracheostomy. The methylprednisone is given 4 hours prior to the extubation in patients
with a high risk of post-extubation stridor as an adjunct to a positive cuff leak test.

Go to the next page if you knew the correct answer, or click the link image(s) below to further
research the concepts in this question (if desired).

Research Concepts:
Extubation

We update eBooks quarterly and Apps daily based on user feedback. Please tap flag to
report any questions that need improvement.
Question 451: A 46-years-old male patient brought to the hospital after sustaining a serious
burn from an accidental fire at his home. 30 % of his body is affected. He has been shifted to the
intensive care unit. 3 days after, he develops cough, fever, 101 degrees Fahrenheit, chest
auscultation reveals crackles on the bilateral lung field. A broad-spectrum beta-lactam antibiotic
is planned to be used. Along with that, an aminoglycoside antibiotic is planned, which has no
synergistic effect with beta-lactam drugs. What is the peak concentration of this antibiotic that is
preferred for the treatment of this hospital-acquired infection?

Choices:
1. 10 to 20 mcg/ml
2. 50 to 60 mcg/ml
3. 20 to 30 mcg/ml
4. 40 to 50 mcg/ml
Answer: 4 - 40 to 50 mcg/ml
Explanations:
This patient developed hospital-acquired pneumonia. Aminoglycoside is added in this
condition to cover gram-negative anaerobes. The target serum concentration for traditional
dosing for amikacin is a peak of 20 to 30 mcg/ml and a trough of less than 8 mcg/ml (1 to 4
mcg/ml).
Higher peak concentrations of 40 to 50 mcg/ml are recommended for serious infections
such as hospital-acquired pneumonia with multi-drug resistant organisms.
These higher peaks can be achieved with extended-interval dosing. Infusion time is over an
hour.
The dose for extended-interval dosing is 15 mg/kg. The frequency of subsequent doses
depends on the creatinine clearance.

Go to the next page if you knew the correct answer, or click the link image(s) below to further
research the concepts in this question (if desired).

Research Concepts:
Amikacin

We update eBooks quarterly and Apps daily based on user feedback. Please tap flag to
report any questions that need improvement.
Question 452: A 35-year-old man with HIV presents with hemoptysis, dyspnea, and
fatigue. The client has not been compliant with his antiretroviral therapy. Scattered crackles are
heard bilaterally on chest auscultation. His CD4 count is 130 cells/mm3. A CT scan of the chest
shows ground-glass opacities in both lungs. Which of the following is the first-line treatment for
this client's suspected condition?

Choices:
1. Trimethoprim-sulfamethoxazole
2. Azithromycin-trimethoprim
3. Penicillin
4. Ceftriaxone-azithromycin
Answer: 1 - Trimethoprim-sulfamethoxazole
Explanations:
This client has developed pneumocystis jiroveci pneumonia (PCP). Clients presenting with
PCP may show signs of fever, cough, dyspnea, and, in severe cases, respiratory failure.
Pneumocystis is thought to be transmitted from person to person via an airborne route.
The first-line treatment for both HIV-infected and uninfected patients is 21 days of
trimethoprim-sulfamethoxazole (TMP-SMX).
For mild to moderate disease, give TMP 15 to 20 mg/kg/day and SMX 75 to 100 mg/kg/day
orally in 3 divided doses or TMP-SMX double strength (DS) 2 tablets 3 times per day.
Typically, clients at risk are those with any underlying disease state that alters host
immunity such as those with cancer, HIV, transplant recipients, or those being treated with
immunosuppressive medications.

Go to the next page if you knew the correct answer, or click the link image(s) below to further
research the concepts in this question (if desired).

Research Concepts:
Prevention Of Opportunistic Infections In HIV/AIDS

We update eBooks quarterly and Apps daily based on user feedback. Please tap flag to
report any questions that need improvement.
Question 453: Pneumonectomy is considered a high-risk procedure associated with
significant morbidity and mortality. After a pneumonectomy, which of these complications is the
most common cause of death?

Choices:
1. Infection
2. Anastomotic dehiscence
3. Non-cardiogenic pulmonary edema
4. Arrhythmia
Answer: 3 - Non-cardiogenic pulmonary edema
Explanations:
Non-cardiogenic pulmonary edema is infrequent but one of the most serious complications
after pneumonectomy. It usually develops in 4% of patients undergoing major resections of
lung parenchyma.
Pathophysiology of postpneumonectomy pulmonary edema is incompletely understood.
One of the proposed mechanisms is increased filtration gradient across pulmonary
capillaries together with increased permeability.
It presents soon after surgery. The chest x-ray may show fluffy infiltrates and may be
confused with acute respiratory distress syndrome.
Treatment consists of restriction of fluid intake, diuretics, and respiratory support to
maintain adequate oxygenation.

Go to the next page if you knew the correct answer, or click the link image(s) below to further
research the concepts in this question (if desired).

Research Concepts:
Pneumonectomy

We update eBooks quarterly and Apps daily based on user feedback. Please tap flag to
report any questions that need improvement.
Question 454: A 17-year-old male is brought in by ambulance to the emergency
department. He was found unconscious at a dance nightclub. Paramedics report he has a
Glasgow Coma Scale of 10. Vital signs include a temperature of 36.3 degrees Celsius, blood
pressure of 110/80 mmHg, heart rate of 48 beats per minute, respiratory rate of 6 breaths per
minute, and oxygen saturation of 95% on room air. Fingerstick glucose was 97 mg/dl. The
patient is intubated and abruptly awakens 2 hours later. He extubates himself and demands to go
home. Which of the following drugs is most likely to be given to this patient?

Choices:
1. Naloxone
2. Atropine
3. Pralidoxime
4. Diazepam
Answer: 2 - Atropine
Explanations:
The patient in this clinical scenario has most likely developed gamma-hydroxybutyrate
(GHB) overdose. The hallmark of gamma-hydroxybutyrate (GHB) overdose is an abrupt
onset of coma followed later by an abrupt resolution of symptoms.
Signs and symptoms of GHB toxicity include CNS depression, respiratory depression, and
bradycardia. Mild hypothermia can also be seen.
There is no antidote for GHB toxicity. The mainstay of treatment is airway protection and
monitoring. Management should also include cardiorespiratory monitoring, pulse oximetry,
and capnography if available. Patients may develop severe respiratory depression or apnea,
and therefore immediate evaluation of the airway is paramount. In milder cases,
supplemental oxygen with or without a nasopharyngeal airway is sufficient until the patient
awakens. In more severe cases, endotracheal intubation may be necessary.
GHB-intoxicated patients usually do not require any sedation while mechanically ventilated
and will precipitously awaken and potentially extubate themselves or require immediate
extubation. Severe bradycardia can be treated with atropine, and hypotension is often
sufficiently managed with intravenous (IV) fluids. If there is any concern for opioid co-
ingestion and toxicity, IV naloxone should be strongly considered. Patients can often be
safely discharged home once they are awake, symptom-free, and all other co-intoxications
or injuries are ruled out.

Go to the next page if you knew the correct answer, or click the link image(s) below to further
research the concepts in this question (if desired).

Research Concepts:
Gamma-Hydroxybutyrate Toxicity

We update eBooks quarterly and Apps daily based on user feedback. Please tap flag to
report any questions that need improvement.
Question 455: A 29-year-old male with a history of asthma and prior intubation presents
with acute severe episodes of shortness of breath. The exam shows a silent chest. An arterial
blood gas is done showing a PaO2 of 50, SpO2 of 80%, and a PaCO2 of 68 with a pH of 7.05.
The patient is immediately intubated and admitted to the ICU with the following ventilator
settings: tidal volume 400ml, respiratory rate 16, FiO2 60%, and positive end-expiratory pressure
(peep) 12. Repeat arterial blood gas (ABG) one hour after intubation shows PaO2 of 73, SpO2
94%, and a PaCO2 of 60 with a pH of 7.13. What is the next best step to take on managing this
patient's ventilator?

Choices:
1. Increase the respiratory rate
2. Decrease the FiO2
3. Decrease the tidal volume
4. Increase the inspiratory time
Answer: 1 - Increase the respiratory rate
Explanations:
The current patient is presenting with acute asthma exacerbation and severe hypercapnic
and hypoxic respiratory failure. The appropriate management was done by intubating the
patient as he is currently experiencing respiratory collapse with severe acidosis. After
intubation, the patient still has severe hypercapnia and acidosis, with now resolved
hypoxemia. To improve hypercapnia and respiratory acidosis, the best option is to increase
minute ventilation. (MV) = RR x TV.
Increasing Respiratory rate or tidal volume would increase minute ventilation and decrease
PCO2, improving hypercapnia and acidosis. Our ability to increase Tidal volume will be
limited by respiratory mechanics, and increasing this parameter would increase plateau
pressures.
Increasing Respiratory rate will also achieve decreasing CO2 and improving acidosis. Our
ability to increase RR is limited by the expiratory time. Increasing RR, too, will reduce the
expiratory time. If the expiratory time is made too short, we risk the patient experiencing
auto-PEEP and increasing plateau pressures.
Changing FiO2 will only affect oxygenation and no ventilation. Changing inspiratory times
can help us use higher RR while preventing auto-PEEP. DECREASING (not increasing)
inspiratory time will let us improve RR while permitting longer expiratory times and that
way, preventing auto-PEEP.

Go to the next page if you knew the correct answer, or click the link image(s) below to further
research the concepts in this question (if desired).

Research Concepts:
Ventilator Management

We update eBooks quarterly and Apps daily based on user feedback. Please tap flag to
report any questions that need improvement.
Question 456: A 45-year-old man is brought to the emergency with lethargy, confusion,
abdominal distension, and hematemesis. He migrated from India 2 years ago. He has a 7-year
history of hepatitis C infection. His abdominal distention improves with furosemide.
Esophagogastroduodenoscopy reveals medium esophageal varices on the distal esophagus.
Which of the following is the next best step in the management of this patient?

Choices:
1. Lisinopril
2. Nadolol
3. Transjugular intrahepatic portosystemic shunt (TIPS)
4. Ciprofloxacin
Answer: 2 - Nadolol
Explanations:
Patients with a medium to large esophageal varices should receive primary prophylaxis to
prevent bleeding, which causes hematemesis and is also cause of hepatic encephalopathy.
Primary prophylaxis of varices is achieved by nonselective beta-blockers or endoscopic
variceal ligation.
The choice of beta-blocker or ligation depends on patient preference and size of varices.
Ligation is preferred for large varices. Nonelective beta-blocker reduce portal pressure by
blocking the adrenergic vasodilatory response of mesenteric arterioles, which result in
unopposed alpha-adrenergic vasoconstriction and thus reduced portal blood flow.
ACE inhibitors have no role in the management of cirrhosis. They are the mainstay of
hypertension, heart failure, and chronic kidney diseases.
TIPS is performed when a patient does not respond to diuretics and has recurrent
esophageal varices. TIPS causes hepatic encephalopathy in 35% of the patients.

Go to the next page if you knew the correct answer, or click the link image(s) below to further
research the concepts in this question (if desired).

Research Concepts:
Portal-Systemic Encephalopathy

We update eBooks quarterly and Apps daily based on user feedback. Please tap flag to
report any questions that need improvement.
Question 457: A patient suddenly collapsed in the ICU. The patient had just had open-heart
surgery two days ago. The nurse had just put the temperature probe in his mouth when he
arrested. Blood pressure is unrecordable and carotids are not palpable. CPR is started. While the
anesthesiologist is attempting to intubate the patient, it is decided to administer epinephrine.
What is the correct dose of the drug in this scenario?

Choices:
1. 1 mg IV push every 3 - 5 minutes
2. 0.1 mg IV push every 3 - 5 minutes
3. 0.1 mg/kg IV push every 3 - 5 minutes
4. 10 mg in 10 cc normal saline down the endotracheal tube
Answer: 1 - 1 mg IV push every 3 - 5 minutes
Explanations:
The standard dose of epinephrine in cardiac arrest is 1 mg every 3-5 minutes.
Higher doses are not recommended and may even be harmful.
Addition of vasopressin to epinephrine does not offer an advantage.
Most experts recommend administering epinephrine ASAP after the cardiac arrest so that
the heart becomes responsive to the defibrillator.

Go to the next page if you knew the correct answer, or click the link image(s) below to further
research the concepts in this question (if desired).

Research Concepts:
Epinephrine

We update eBooks quarterly and Apps daily based on user feedback. Please tap flag to
report any questions that need improvement.
Question 458: A 65-year-old male patient is brought to the emergency department by his
family after an episode of loss of consciousness. The patient did not experience any prodromal
features, generalized body movements, involuntary loss of urine, or tongue bite during the
episode. He mentions that he had an upper respiratory tract infection a week ago that resolved
spontaneously. The patient has a history of hypertension and gastroesophageal reflux disease, for
which he takes amlodipine and famotidine. Vitals show a blood pressure of 122/72 mm Hg,
which drops to 104/68 mmHg on inspiration, pulse of 64/min, respiratory rate of 15/min, and
temperature of 98.6 F (37 C). Physical examination reveals distended neck veins and distant
heart sounds. The lungs are clear to auscultation bilaterally. What is the most likely diagnosis?

Choices:
1. Cardiac tamponade
2. Constrictive pericarditis
3. Pleural effusion
4. Pulmonary embolism
Answer: 1 - Cardiac tamponade
Explanations:
The most likely diagnosis is cardiac tamponade. The patient’s history of an upper
respiratory tract infection, current presentation, distant heart sounds, and distended neck
veins on examination indicate the diagnosis.
Pulsus paradoxus is frequently seen in cardiac tamponade. Pulsus paradoxus is defined as a
drop in blood pressure by more than 10 mmHg on inspiration.
In cardiac tamponade, there is decreased ventricular compliance, and the increased venous
return during inspiration results in the bowing of intraventricular septum towards the left,
decreasing stroke volume.
It can be seen in constrictive pericarditis, pulmonary embolism, and bilateral pleural
effusion but this patient is more likely to have developed cardiac tamponade.

Go to the next page if you knew the correct answer, or click the link image(s) below to further
research the concepts in this question (if desired).

Research Concepts:
Pulsus Paradoxus

We update eBooks quarterly and Apps daily based on user feedback. Please tap flag to
report any questions that need improvement.
Question 459: A 55-year-old man is admitted to the intensive care unit with gram-negative
sepsis. His clinical condition deteriorates over the next few days. He ultimately develops
refractory hypotension and multi-organ failure and dies despite aggressive treatment. Autopsy
reveals bilateral enlarged and hemorrhagic suprarenal structures. Histopathological examination
shows hyperplasia along with congestion and extensive hemorrhage. Which of the following
factors is most likely responsible for the development of hemorrhage in this structure?

Choices:
1. Anatomical location
2. Vascular arrangement
3. Drug toxicity
4. Age-related degeneration
Answer: 2 - Vascular arrangement
Explanations:
This patient has died secondary to septic shock. The organ described in the autopsy is the
adrenal gland. The adrenals are paired endocrine organs located in the retroperitoneum
above the kidneys. They have two anatomically and functionally distinct parts each with its
unique embryological origin, the outer cortex and the inner medulla. The autopsy findings
are consistent with the development of adrenal hemorrhage.
Adrenal hemorrhage is an unusual and often underdiagnosed disease characterized by
bleeding into the adrenals. Septicemia and critical illness are important causes. The adrenal
gland is one of the most well-perfused organs of the body. It has a rich and extensive
arterial supply. Usually, 3 sets of arteries, the superior, middle, and inferior adrenal artery
which are generally derived from the inferior phrenic, the abdominal aorta and the renal
arteries respectively, contribute to the arterial supply. There is a rich subcapsular plexus
which drains into medullary sinusoids, the outflow through venules is limited. This pattern
of vascular supply has been described as a 'vascular dam.' Furthermore, the outflow is via a
single adrenal vein. This unique vascular arrangement can lead to adrenal congestion and
subsequent hemorrhagic transformation.
In times of stress, there is trophic stimulation of adrenals due to ACTH release from the
anterior pituitary. This leads to hyperplasia of the gland and increased vascularity. This
increased vascular congestion, together with vascular predisposition, contributes to adrenal
hemorrhage.
The adrenals are responsible for producing steroid hormones and catecholamines. Their
anatomic location and aging do not contribute significantly to hemorrhage. Drug toxicity
also is unlikely to contribute to the autopsy findings.

Go to the next page if you knew the correct answer, or click the link image(s) below to further
research the concepts in this question (if desired).

Research Concepts:
Adrenal Hemorrhage

We update eBooks quarterly and Apps daily based on user feedback. Please tap flag to
report any questions that need improvement.
Question 460: A patient was admitted after a fall where she hit her upper back and neck.
On examination, there are no cranial nerve deficits. The motor examination shows 4/5 in elbow
extension and wrist extension of both upper extremities and 3/5 on both lower extremities. There
was decreased sensation below the clavicle and the last two digits of both hands. She was
diagnosed with a spinal cord injury. Several weeks later, she is receiving therapy when she
develops a severe headache, diaphoresis, and hypertension. What is the best treatment for this
condition?

Choices:
1. Nitroprusside
2. Remove the offending stimulus
3. Opioid analgesia
4. Restrict fluids
Answer: 2 - Remove the offending stimulus
Explanations:
Patients with SCI above T6 are at risk for autonomic dysreflexia.
A minor noxious stimulus causes a sympathetic discharge causing severe hypertension.
The patient should be placed in an upright position.
Medications used to treat this are rapid short-acting nitrates (nitroglycerin or nifedipine).

Go to the next page if you knew the correct answer, or click the link image(s) below to further
research the concepts in this question (if desired).

Research Concepts:
Autonomic Dysreflexia

We update eBooks quarterly and Apps daily based on user feedback. Please tap flag to
report any questions that need improvement.
Question 461: A 16-year-old patient sustained a severe head injury following a motor
vehicle collision. His intracranial pressure (ICP) has been monitored by the placement of an
external ventricular drain (EVD) placement. Despite keeping the patient intubated and sedated in
mechanical ventilation, his ICP is persistently above 20 mm Hg. His serum electrolytes were
normal, and serum osmolarity was 350 mosmol/kg. The treating clinician then plans to start
medical therapy for managing his refractory cerebral edema. Which of the following is the most
rational approach in managing the patient?

Choices:
1. Hypertonic saline
2. Mannitol
3. Urea
4. Glycerol
Answer: 1 - Hypertonic saline
Explanations:
The use of hypertonic saline in the management of intracranial hypertension has shown to
be of rapid onset, sustained as well as long-lasting effects with collateral improvement in
cerebral perfusion as well.
3% hypertonic saline with a loading dose of 5 ml/kg and a maintenance dose of 2 ml/kg
every six hourly has shown to be highly efficacious as well as safe in managing refractory
intracranial hypertension.
The occurrence of side effects such as central pontine myelinolysis and acute tubular
necrosis with hypertonic saline is minimal in patients with serum osmolality of above 320
mosmol/kg and normal serum sodium values.
Urea and glycerol have low efficacy in managing cerebral edema. Mannitol use leads to a
high occurrence of rebound cerebral edema, renal failure, and electrolyte imbalance. Both
hypertonic saline, as well as mannitol have comparable all-cause mortality rates.

Go to the next page if you knew the correct answer, or click the link image(s) below to further
research the concepts in this question (if desired).

Research Concepts:
Intracranial Hypertension

We update eBooks quarterly and Apps daily based on user feedback. Please tap flag to
report any questions that need improvement.
Question 462: A 65-year-old female is admitted to the hospital for a non-ST-segment
elevation myocardial infarction. During the initial assessment by the night shift, she appears pale
and diaphoretic. She reports shortness of breath and dizziness. A bedside cardiac monitor reveals
monomorphic wide-complex ventricular tachycardia (VT). Before further evaluation, the patient
becomes unresponsive and becomes pulseless. The cardiac monitor continues to show VT.
Cardiopulmonary resuscitation (CPR) and subsequent biphasic defibrillation with 200 joules is
given. After 2 minutes of CPR, the patient remains pulseless, and the rhythm now shows
ventricular fibrillation (VF). She is defibrillated again with 300 joules, and 1 mg of epinephrine
is given intravenously with continued CPR. After 2 minutes, the patient remains in VF. Which of
the following is the next best step in the management of this patient?

Choices:
1. Continue CPR, defibrillate with 360 joules and give 150 mg of amiodarone intravenously
2. Continue CPR, defibrillate with 300 joules and give 300 mg of amiodarone intravenously
3. Continue CPR, defibrillate with 360 joules and give 1 mg/kg of norepinephrine intravenously
4. Continue CPR, defibrillate with 200 joules and give 1 mg of epinephrine intravenously
Answer: 2 - Continue CPR, defibrillate with 300 joules and give 300 mg of amiodarone
intravenously

Explanations:
During cardiopulmonary resuscitation for VF or VT, each successive defibrillation should
be of equal or greater value until the maximum dose is achieved.
After the first unsuccessful defibrillation, CPR should be continued with a second
defibrillation and 1 mg of epinephrine bolus intravenously. The CPR should be continued
for 2 minutes after the epinephrine and the shock has been delivered.
If the patient remains in VT or VF at the next rhythm and pulse check, CPR should be
continued with another dose of defibrillation. At this time acceptable antiarrhythmic agents
include 1 mg of epinephrine or 300 mg of amiodarone bolus.
Lidocaine is no longer preferred for cardiopulmonary resuscitation of VT or VF.

Go to the next page if you knew the correct answer, or click the link image(s) below to further
research the concepts in this question (if desired).

Research Concepts:
Defibrillation

We update eBooks quarterly and Apps daily based on user feedback. Please tap flag to
report any questions that need improvement.
Question 463: A 65-year-old woman presents to the hospital with two days of fever,
headache, and sore throat. She was seen at an urgent care center the day before and was given
azithromycin without relief. Her vital signs reveal blood pressure 88/50 mmHg, temperature 103
F (39.4 C), pulse 115 beats/min, and SpO2 97% on room air. She appears lethargic. Her pharynx
is normal and she is swallowing and speaking normally. Lungs are clear to auscultation. She has
one cold sore on her lip. The abdomen is soft and non-tender. What is the next best step in the
management of this patient?

Choices:
1. IV ceftriaxone, vancomycin, and acyclovir
2. CT head
3. Lumbar puncture
4. Acetaminophen
Answer: 1 - IV ceftriaxone, vancomycin, and acyclovir
Explanations:
This patient most likely has meningitis.
The definitive test of choice is a lumbar puncture to send CSF for necessary analysis.
If the patient is hemodynamically unstable, it is appropriate to start treatment immediately
before lumbar puncture is performed.
Cold sores may indicate that the patient may have a herpes simplex virus. Herpes simplex
virus can cause aseptic meningitis, which should be treated with acyclovir.

Go to the next page if you knew the correct answer, or click the link image(s) below to further
research the concepts in this question (if desired).

Research Concepts:
Aseptic Meningitis

We update eBooks quarterly and Apps daily based on user feedback. Please tap flag to
report any questions that need improvement.
Question 464: An 18-year-old male presents with a long history of easy bruising and
recurrent nosebleeds, for which he has required multiple admissions. He has a family history of
nosebleeds. Now he has been complaining of a headache, gait difficulty with weakness,
paresthesias, confusion, and nausea. Examination reveals that he has numerous telangiectasias
over his face and lips. MRI reveals that he has a ring-enhancing lesion in the brain. What is the
cause of the brain lesion?

Choices:
1. Bleeding in the brain
2. AVMs in the lung
3. Fragile blood-brain barrier
4. Use of IV drugs
Answer: 2 - AVMs in the lung
Explanations:
Patients with hereditary hemorrhagic telangiectasia develop arteriovenous malformations of
the liver, lungs, brain, and spine.
The pulmonary shunts commonly associated with this disorder put patients at significant
risk for the development of brain abscesses. When there are AVMs in the lung, the normal
filtering ability of the pulmonary circulation is lost and septic emboli go to the brain.
These patients also experience paradoxical cerebral emboli, strokes, and subarachnoid
hemorrhage, due to an increased risk of intracranial aneurysms.
Brain abscess will present as a ring-enhancing lesion with symptoms of weakness, seizures,
headache and even paraplegia.

Go to the next page if you knew the correct answer, or click the link image(s) below to further
research the concepts in this question (if desired).

Research Concepts:
Osler-Weber-Rendu Disease

We update eBooks quarterly and Apps daily based on user feedback. Please tap flag to
report any questions that need improvement.
Question 465: A 69-year-old man is brought to the emergency department with chest pain,
diaphoresis, and dyspnea. An acute ST-elevation myocardial infarction is diagnosed. He had no
prior cardiovascular symptoms. Past history is significant for hypertension and diabetes mellitus.
On the fifth day of the hospitalization, the patient develops shock with a blood pressure of 60/40
mg and a pulse of 118/ min. Left ventricular rupture is suspected, and echocardiography is being
done for evaluation. Which of the following is the best determinant of mortality in this patient?

Choices:
1. Early diagnosis and surgical intervention
2. The time period from acute myocardial infarction and the onset of symptoms
3. The severity of clinical symptoms
4. Etiology of the rupture
Answer: 1 - Early diagnosis and surgical intervention
Explanations:
Type and size of the injury, rapidity of transfer to the hospital, patient's hemodynamic
stability, and the time till intervention play an important role in in-hospital mortality.
Early detection of symptoms and signs is the key to decreasing mortality, especially in the
acute critical condition of myocardial rupture.
Early diagnosis and immediate surgical intervention are critical as the mortality is extremely
high without intervention.
The severity of symptoms, onset of symptoms, or etiology does not affect mortality. It
depends on the time of initiation of the intervention.

Go to the next page if you knew the correct answer, or click the link image(s) below to further
research the concepts in this question (if desired).

Research Concepts:
Left Ventricular Rupture

We update eBooks quarterly and Apps daily based on user feedback. Please tap flag to
report any questions that need improvement.
Question 466: A 16-year-old boy with newly diagnosed Wolff-Parkinson-White syndrome
is admitted to the hospital for laparoscopic appendectomy. He is not currently taking any
medications and is otherwise healthy. Upon insufflation of the abdomen, he develops a wide
complex tachycardia with a heart rate of 170/min. His blood pressure is 110/70 mmHg. Which of
the following is the next best step in the management of this patient?

Choices:
1. Adenosine
2. Procainamide
3. Esmolol
4. Verapamil
Answer: 2 - Procainamide
Explanations:
Wolff-Parkinson-White (WPW) is a syndrome in which the heart contains an anomalous
bundle of conducting tissue called an accessory pathway (AP), which bypasses the normal
AV nodal conduction system.
A tachyarrhythmia resulting from forming a circuit between the AV conducting system and
the AP is called an atrioventricular reciprocating tachycardia (AVRT). There are two types
of AVRTs: orthodromic and antidromic.
In the setting of WPW with wide complex tachycardia, agents such as procainamide are
preferred since they will slow conduction through the accessory pathway. Administration of
AV node blocking agents in antidromic AVRT can increase conduction through the AP and
potentially lead to ventricular fibrillation.
Orthodromic AVRT consists of antegrade conduction through the normal AV conducting
system and retrograde conduction through the AP. Consequently, the tachycardia will have
a narrow QRS and can be treated with most agents that slow conduction through the AV
node (e.g., adenosine, verapamil, esmolol).

Go to the next page if you knew the correct answer, or click the link image(s) below to further
research the concepts in this question (if desired).

Research Concepts:
Wolff Parkinson White Syndrome

We update eBooks quarterly and Apps daily based on user feedback. Please tap flag to
report any questions that need improvement.
Question 467: A 20-year-old man is brought to the emergency department with a history of
being found falling suddenly unconscious on the roadside. On examination, his face and tongue
are swollen. He is drowsy, and his pulse rate is 112/ minute, and his blood pressure is 82/56
mmHg. His blood investigations reveal elevated levels of angiotensin II. Which of the following
enzymes present in the patient's body might have caused elevated levels of this chemical?

Choices:
1. Tryptase
2. Chymase
3. Trehalase
4. Dihydropeptidase
Answer: 2 - Chymase
Explanations:
The presence of facial swelling and hypotension suggests the diagnosis of an anaphylactic
reaction.
Elevated levels of angiotensin II may be seen following anaphylaxis.
Mast cell degranulation releases chymase.
Chymase facilitates the conversion of angiotensin I to angiotensin II.

Go to the next page if you knew the correct answer, or click the link image(s) below to further
research the concepts in this question (if desired).

Research Concepts:
Anaphylaxis

We update eBooks quarterly and Apps daily based on user feedback. Please tap flag to
report any questions that need improvement.
Question 468: An elderly homeless man presents with altered mental status, seizures, and
vomiting. Exam reveals multiple dental caries, and focal neurological deficit, with no fever, head
trauma, or neck stiffness. What should the work up include next?

Choices:
1. Lumbar puncture
2. Non-contrast head CT
3. Contrast-enhanced head CT
4. Placement of ICP monitor
Answer: 3 - Contrast-enhanced head CT
Explanations:
Brain abscess is one consideration and presents sometimes without fever, elevated WBC,
and signs of meningeal irritation.
Brain abscesses develop by either contiguous spread or hematogenous spread. Common
sites of infection include sinuses, mouth/teeth, and ears. Contrast-enhanced CT and MRI
reveal a ring-enhancing lesion usually at the gray-white interface with surrounding edema.
Treatment involves identification of organism through relief of mass effect with aspiration
vs open excision followed by IV antibiotics.
Brain abscess today is more common in the immunocompromised, including HIV patients.

Go to the next page if you knew the correct answer, or click the link image(s) below to further
research the concepts in this question (if desired).

Research Concepts:
Brain Abscess

We update eBooks quarterly and Apps daily based on user feedback. Please tap flag to
report any questions that need improvement.
Question 469: A 92-year-old man is brought to the emergency department at 2:35 a.m.
from his nursing home with pronounced confusion. He has dilated pupils, poorly responsive to
light. He was recently prescribed a new medication to treat vomiting. His blood pressure is
134/78 mmHg, and his temperature is 102.9 F (39.4 C). What is the best initial step in the
management of this patient?

Choices:
1. Administer doxylamine
2. Draw blood cultures
3. Begin constant cardiac monitoring
4. Order a chest MRI
Answer: 3 - Begin constant cardiac monitoring
Explanations:
Doxylamine may cause arrhythmias in toxic doses.
Doxylamine may cause respiratory failure in toxic doses.
In toxic doses, doxylamine may cause arrhythmias, respiratory failure, or hyperthermia that
may lead to cardiac dysfunction. Doxylamine toxicity is more common in the young and
elderly populations.
Doxylamine may cause hyperthermia in toxic doses, which could result in heat-induced
cardiac dysfunction.

Go to the next page if you knew the correct answer, or click the link image(s) below to further
research the concepts in this question (if desired).

Research Concepts:
Doxylamine

We update eBooks quarterly and Apps daily based on user feedback. Please tap flag to
report any questions that need improvement.
Question 470: A 66-year-old male patient presents in septic shock and does not respond to
appropriate fluid resuscitation. The patient requires a central venous line (CVL) placement.
However, the patient is taking warfarin for atrial fibrillation and his INR is highly deranged i.e.
7.6. Which of the following measures need to be taken immediately?

Choices:
1. Administer vitamin K through the IV, continue IV hydration through the peripheral lines,
place a central line in the femoral vein as this is the most compressible site
2. Administer PO vitamin K, order FFP to be given in the ICU, continue IV hydration, call the
intensivist and let them know that they need to call surgery to place the line once INR is
corrected
3. Administer IV vitamin K and IV FFP. Consider peripheral vasopressor support while
administering vitamin K and FFP. Place a central line in the femoral or internal jugular vein
using ultrasound to help visualize vessels.
4. Administer IV vitamin K and IV FFP. Consider peripheral vasopressor support while
administering vitamin K and FFP. Place a subclavian line slowly and controlled with the patient
in the Trendelenburg position to best dilate the subclavian vein making it easier to cannulate in
the first attempt.
Answer: 3 - Administer IV vitamin K and IV FFP. Consider peripheral vasopressor support
while administering vitamin K and FFP. Place a central line in the femoral or internal jugular
vein using ultrasound to help visualize vessels.

Explanations:
A compressible site is preferred, however vitamin K alone will not reverse fast enough. This
patient requires vasopressor support as fluids alone are not enough for resuscitation.
This patient needs access and pressor support as he did not respond to fluid resuscitation.
The best way is to rapidly reverse the coagulopathy, discuss options with family, and start
peripheral pressor support while correcting coagulopathy. Then place a line in a
compressible site with ultrasound to limit the number of attempts needed
A patient who is hypotensive but has not received any fluid via a peripheral line should be
resuscitated first before deciding to place a CVL and starting them on vasopressors.
The subclavian approach is contraindicated. There are no absolute indications for a central
venous catheter (CVC). Generally, patients requiring vasoactive medications will need one
due to the potential for vein sclerosis and ischemia through peripheral IVs. Patients who
require multiple medications and continued resuscitation, have inadequate peripheral access,
are critically ill, or require multiple phlebosclerotic medications also candidates for central
line placement.

Go to the next page if you knew the correct answer, or click the link image(s) below to further
research the concepts in this question (if desired).

Research Concepts:
Central Line

We update eBooks quarterly and Apps daily based on user feedback. Please tap flag to
report any questions that need improvement.
Question 471: A 67-year-old female is receiving mechanical ventilation in the intensive
care unit for severe acute respiratory distress syndrome (ARDS) secondary to a motor vehicle
accident. The ventilator mode is volume assist control, and the tidal volume is set at 500 mL.
Which of the following is a significant disadvantage of volume assist control for patients with
ARDS?

Choices:
1. Inability to deliver a constant tidal volume
2. Increased risk for elevated plateau pressures and pulmonary barotrauma
3. Lack of adequate alarms
4. Inability to provide proper oxygenation
Answer: 2 - Increased risk for elevated plateau pressures and pulmonary barotrauma
Explanations:
Volume assist control is a volume-cycled mode of ventilation. It delivers a fixed amount of
tidal volume on every breath provided by the ventilator.
Since the ventilator will always deliver the same amount of tidal volume, the peak and
plateau pressures generated by the system will depend on lung compliance.
Stiffer lungs, such as those in patients with ARDS, will generate higher peak and plateau
pressures to deliver the same amount of tidal volume when compared to healthy lungs.
Increased plateau pressure, higher than 35 cmH2O, has been associated with an increased
risk of pulmonary barotrauma.

Go to the next page if you knew the correct answer, or click the link image(s) below to further
research the concepts in this question (if desired).

Research Concepts:
Barotrauma And Mechanical Ventilation

We update eBooks quarterly and Apps daily based on user feedback. Please tap flag to
report any questions that need improvement.
Question 472: A 40-year-old man was involved in a motor vehicle accident. The patient
was an unrestrained driver when he lost control of his car and hit a tree head-on. Airbags
deployed. The patient was awake when EMS first arrived, and he is brought to the hospital. The
ATLS algorithm is followed, and the patient is stabilized. Non-contrast CT of the maxillofacial
bones reveals a LeForte 2 fracture with the involvement of the right maxillary sinus. Soon
afterward, there is an acute change in the patient's clinical status. The patient opens eyes to pain,
is making incomprehensible sounds, and has a decorticate posture. Which of the following is the
next best step in the management of this patient?

Choices:
1. Nasotracheal intubation
2. Nonrebreather mask
3. Non-contrast head CT
4. Tracheostomy
Answer: 4 - Tracheostomy
Explanations:
This patient has a GCS of 7/15 and needs to have their airway secured.
Given the patient's unstable facial fractures, a tracheostomy is a good choice in securing the
airway.
This patient should not undergo nasotracheal intubation given the extent of his facial
injuries.
Though this patient may eventually need further imaging, it is more important to secure his
airway at this time.

Go to the next page if you knew the correct answer, or click the link image(s) below to further
research the concepts in this question (if desired).

Research Concepts:
Maxillary Sinus Fracture

We update eBooks quarterly and Apps daily based on user feedback. Please tap flag to
report any questions that need improvement.
Question 473: A 65-year-old man is admitted to the intensive care unit after coronary
artery bypass surgery. The respiratory therapist has set the ventilator to synchronized intermittent
mandatory ventilation (SIMV) mode, rate of 12, a fraction of inspired oxygen (FiO2) of 50% and
tidal volume of 550 milliliters. The patient's respiratory rate reads 22 breaths per minute. Which
of the following can be expected with this ventilator mode?

Choices:
1. Increased work of breathing
2. Decreased work of breathing
3. Tidal volume of 550 with spontaneous breaths
4. Evidence of breath stacking
Answer: 1 - Increased work of breathing
Explanations:
Synchronized intermittent mandatory ventilation (SIMV) has been shown to increase work
of breathing in ventilated patients.
The addition of pressure support may decrease the work of breathing.
The ventilator will synchronize the delivery of mandatory and spontaneous breaths in SIMV
to avoid breath stacking.
Volumes generated during spontaneous breaths in SIMV mode are 100% patient effort
driven. The tidal volumes will vary by breath.

Go to the next page if you knew the correct answer, or click the link image(s) below to further
research the concepts in this question (if desired).

Research Concepts:
Synchronized Intermittent Mandatory Ventilation

We update eBooks quarterly and Apps daily based on user feedback. Please tap flag to
report any questions that need improvement.
Question 474: A 67-year-old man with a past medical history of chronic hepatitis C,
diabetes mellitus, and hypertension presents to the emergency department with hematemesis and
altered mental status. On examination, he is somnolent, has asterixis, ascites, and palmar
telangiectasias. His hematemesis is uncontrollable with standard treatment and supportive
therapy. Which of the following is the strongest contraindication to the use of molecular
adsorbent recirculating system (MARS) in this patient?

Choices:
1. Uncontrolled hemorrhage
2. Hepatic encephalopathy
3. Chronic hepatitis C
4. Uncontrolled sepsis
Answer: 1 - Uncontrolled hemorrhage
Explanations:
Uncontrolled haemorrhage prohibits the use of MARS and is considered as a relative
contraindication.
MARS has a very good safety profile and is tolerated well by the patients.
Thrombocytopenia has been documented during the therapy, but it is mild and pose no
significant threat to the patient. The risks associated with MARS are the same as the ones
with conventional haemodialysis and requires the use of anticoagulants to prevent
coagulation activation during the procedure.
MARS is relatively contraindicated if there is increased risk of coagulopathy (example:
disseminated intravascular coagulation) i.e. when platelets are below 50,000/microlitre of
blood or when the INR is greater than 2.3.
Uncontrolled sepsis also prohibits the use of MARS but is not present in this patient.

Go to the next page if you knew the correct answer, or click the link image(s) below to further
research the concepts in this question (if desired).

Research Concepts:
Molecular Absorbent Recirculating System

We update eBooks quarterly and Apps daily based on user feedback. Please tap flag to
report any questions that need improvement.
Question 475: A 26-year-old man presents with a 2-month history of generalized fatigue,
night sweats, and 3 kg unintentional weight loss. Following further investigation, he is diagnosed
with stage IV Hodgkin lymphoma. The patient is started on a chemotherapy regimen that
includes a cytotoxic drug belonging to a family of glycopeptide antibiotics that disrupts DNA
synthesis. Which of the following tests is most important to perform regularly in this patient
whilst he is taking this drug?

Choices:
1. Lumbar puncture
2. Echocardiogram
3. Chest x-ray
4. Abdominal ultrasound
Answer: 3 - Chest x-ray
Explanations:
Bleomycin is an antibiotic and antineoplastic agent.
Bleomycin's most concerning adverse effect is pulmonary fibrosis.
It is advisable to perform chest x-rays at regular intervals, therefore, while a patient is on
therapy with bleomycin.
Less severe reactions to bleomycin include skin pigmentation changes, itching, hypogeusia,
rash, nausea, vomiting, and weight loss.

Go to the next page if you knew the correct answer, or click the link image(s) below to further
research the concepts in this question (if desired).

Research Concepts:
Bleomycin

We update eBooks quarterly and Apps daily based on user feedback. Please tap flag to
report any questions that need improvement.
Question 476: A late preterm infant is delivered at 34 weeks of gestational age via
emergency cesarean section due to worsening preeclampsia in the mother. The mother received
only one dose of betamethasone two hours prior to delivery. The infant emerged with a weak
spontaneous cry and required positive pressure ventilation in the delivery room. Six hours later,
his respiratory distress worsened, necessitating endotracheal intubation and placement of an
umbilical arterial catheter. An umbilical venous catheter was attempted but was unsuccessful.
Parenteral nutrition was infused through the umbilical arterial catheter. Over the next few hours,
his respiratory status stabilized with improvement in blood gases, but the blood glucose levels
start decreasing, requiring an increase in the glucose infusion rate to 15 mg/kg/min. At this point,
what would be the best course of management to address the underlying cause of the problem?

Choices:
1. Perform glucagon stimulation test
2. Increase the glucose infusion rate (GIR) further to maintain euglycemia
3. Perform lumbar puncture to rule out meningitis
4. Consider alternate access for parenteral nutrition
Answer: 4 - Consider alternate access for parenteral nutrition
Explanations:
Refractory hypoglycemia is a rare complication of umbilical arterial catheters when
dextrose is infusing through them.
When the catheter tip is in a high position ending near the celiac axis or mesenteric arteries,
the dextrose can stimulate beta cells of the pancreas, secreting insulin leading to
hyperinsulinemic hypoglycemia.
The condition is reversible after discontinuation of the dextrose-containing fluids.
The umbilical artery is not usually used for infusion of maintenance glucose/electrolyte
solutions or medications.

Go to the next page if you knew the correct answer, or click the link image(s) below to further
research the concepts in this question (if desired).

Research Concepts:
Umbilical Artery Catheterization

We update eBooks quarterly and Apps daily based on user feedback. Please tap flag to
report any questions that need improvement.
Question 477: An on-call physician receives a call that he is soon to receive an ICU
transfer from an outside facility. The charge nurse informs the physician that the patient is being
transferred with anemia, thrombocytopenia, prolonged aPTT, and INR, and schistocytes noted on
a peripheral blood smear. Before the patient arrives, the physician develops an action plan for the
patient while considering the additional medical problems the patient is likely to have. What is
the most likely additional disease process occurring in this patient?

Choices:
1. Placental abruption
2. Aortic aneurysm
3. Malignancy
4. Sepsis
Answer: 4 - Sepsis
Explanations:
Sepsis is the most common cause of disseminated intravascular coagulation (DIC).
DIC occurs in 30% to 50% of cases of severe sepsis.
Trauma, malignancy, aortic aneurysms, and obstetrical complications are causes of DIC,
though they are less common.
DIC has classically been associated with gram-negative sepsis.

Go to the next page if you knew the correct answer, or click the link image(s) below to further
research the concepts in this question (if desired).

Research Concepts:
Disseminated Intravascular Coagulation

We update eBooks quarterly and Apps daily based on user feedback. Please tap flag to
report any questions that need improvement.
Question 478: A 50-year-old male is being evaluated in the intensive care unit. He suffered
from right cerebellar hemorrhage, four weeks back, which was complicated by aspiration
pneumonia. He was admitted and intubated due to respiratory distress. An elective tracheostomy
was done due to prolonged anticipated mechanical ventilation. He is seen today in the ICU and is
doing well, his chest X-ray has improved, and he was placed on spontaneous mode this morning.
His current medications include clopidogrel, enalapril, omeprazole, enoxaparin, and piperacillin-
tazobactam. He developed minor bleeding from the tracheostomy during this morning’s
suctioning and pulmonary toilet. During the round, he suddenly deteriorates, and his blood
pressure is shown to be 80/50 mmHg and pulse of 150 beats per minute. His spO2 drops to 75%,
and he becomes tachypneic. His tracheostomy suctioning demonstrates fresh blood that is seen to
spurt. What immediate measures should be taken given the likely diagnosis?

Choices:
1. Overinflate tracheostomy tube
2. Administer protamine sulfate
3. Place the patient in the supine position
4. Administer tissue plasminogen activator
Answer: 1 - Overinflate tracheostomy tube
Explanations:
This patient has been on prolonged ventilatory support and has had an elective
tracheostomy. The patient is improving on intravenous antibiotics and develops massive
pulmonary hemorrhage. Sudden massive hemorrhage, along with spurting of blood,
indicates bleeding from an arterial site. The likely diagnosis, in this case, is the formation of
a trachea-innominate artery fistula (TIAF).
TIAF is caused by erosion of anterior tracheal wall by a tracheostomy cuff of the tip and
subsequent catastrophic bleed. The best temporary measure is to over inflate the
tracheostomy cuff. Overinflation of cuff will lead to pressure over the innominate artery and
may stop the bleeding.
If bleeding does not stop by this maneuver, digital pressure over the artery can be used to
buy time. Diagnostic procedures such as bronchoscopy and angiography can help to
diagnose the condition.
Surgical management or endoscopic coiling is the definitive management. But temporary
stopping bleed should be of topmost priority. Placing patient supine or reversal of
anticoagulation will not stop arterial bleeding.

Go to the next page if you knew the correct answer, or click the link image(s) below to further
research the concepts in this question (if desired).

Research Concepts:
Tracheo Innominate Artery Fistula

We update eBooks quarterly and Apps daily based on user feedback. Please tap flag to
report any questions that need improvement.
Question 479: A 55-year-old man who underwent an uneventful two-vessel coronary artery
bypass continues to have minor bleeding during the closure. The patient is given whole blood
rapidly to maintain volume. Soon after, he develops hyperthermia, hypotension, and diffuse
bleeding from the incision and suture lines. What is the underlying pathophysiology of this
reaction?

Choices:
1. Immune complex-mediated reaction
2. Antibody-mediated reaction
3. Mast cell-mediated reaction
4. T-cell mediated reaction
Answer: 2 - Antibody-mediated reaction
Explanations:
Type II hypersensitive reactions are the antibody-mediated destruction of cells.
This type of reaction is best seen with blood-transfusion reactions, where the host antibodies
react with foreign antigens on the incompatible transfused blood cells and mediate the
destruction of these cells.
In this patient, diffuse bleeding and renal failure developed right after the blood transfusion
indicating acute hemolytic reaction. Acute hemolytic reactions are fatal and develop in
cardiac patients.
These reactions can be prevented by a proper crossmatching between the recipient's and the
donor's blood. The cross-matching reveals the presence of the antibodies in the donor or
recipient sera that can cause these reactions.

Go to the next page if you knew the correct answer, or click the link image(s) below to further
research the concepts in this question (if desired).

Research Concepts:
Transfusion Reactions

We update eBooks quarterly and Apps daily based on user feedback. Please tap flag to
report any questions that need improvement.
Question 480: A 55-years-old man undergoes an emergency laparotomy for intestinal
obstruction. The patient is shifted to the intensive care unit, where he is mechanically ventilated
with 20 cmH2O of positive end-expiratory pressure (PEEP). On the third day of ICU admission,
the patient suddenly deteriorates. His blood pressure falls from 120/75 mmHg to 50/30 mmHg,
and the oxygen saturation drops to 70%. Which of the following is the most likely cause of the
patient's deterioration?

Choices:
1. Tension pneumothorax
2. Hyperventilation
3. Pulmonary embolism
4. Acute myocardial infarction
Answer: 1 - Tension pneumothorax
Explanations:
PEEP is produced by applying positive pressure to the mechanical ventilator's exhalation
valve after the expiratory phase. It is often used to increase arterial oxygenation when FiO2
exceeds 0.50 to reduce O2 toxicity.
There are, however, several potential hazards associated with the use of PEEP. These
include decreased cardiac output, pulmonary barotrauma (i.e., tension pneumothorax),
increased extravascular lung water, and redistribution of pulmonary blood flow.
Barotrauma, such as pneumothorax, pneumomediastinum, and subcutaneous emphysema,
occurs due to the overdistention of alveoli by PEEP. Pulmonary barotrauma should be
suspected when there is an abrupt deterioration of arterial oxygenation and cardiovascular
function during mechanical ventilation with PEEP.
If barotrauma is suspected, a chest x-ray film should be obtained, and if tension
pneumothorax is present, a chest tube should be placed in the involved chest cavity.

Go to the next page if you knew the correct answer, or click the link image(s) below to further
research the concepts in this question (if desired).

Research Concepts:
Positive End-Expiratory Pressure

We update eBooks quarterly and Apps daily based on user feedback. Please tap flag to
report any questions that need improvement.
Question 481: A 45-year-old woman presents to the provider complaining of high fever,
left eye chemosis, and headache for the past three days. Past medical history is significant for a
history of diabetes mellitus for the past ten years. Vital signs reveal a blood pressure of 120/80
mmHg, a pulse rate of 79 beats per minute, a respiratory rate of 14/min., and a temperature of 39
degrees Celsius. On physical examination, she has bilateral proptosis, and palsy of the sixth
cranial nerve on the right side. Which of the following antibiotics should be included in the
initial treatment of her underlying condition?

Choices:
1. Vancomycin and cefotaxime
2. Cefotaxime and metronidazole
3. Vancomycin and metronidazole
4. Vancomycin, cefotaxime, and metronidazole
Answer: 4 - Vancomycin, cefotaxime, and metronidazole
Explanations:
An infected cavernous sinus thrombosis is a medical emergency. Even with early antibiotic
therapy, the mortality rate is approximately 20 percent.
There are numerous potential sources of infection; therefore, the initial therapy must be
broad-spectrum until definitive determination of the causative pathogen.
Recent research shows that anticoagulation is beneficial in one study, but thrombolytics
have not yet been studied.
The dangerous area of the face consists of the triangular area from the corners of the mouth
to the nasal bridge, including the lower part of the nose and maxilla. Venous drainage from
this area drains directly into the cavernous sinus. Therefore, any infection involving the
dangerous area of the face is a potential cause of cavernous sinus thrombosis.

Go to the next page if you knew the correct answer, or click the link image(s) below to further
research the concepts in this question (if desired).

Research Concepts:
Cavernous Sinus Thrombosis

We update eBooks quarterly and Apps daily based on user feedback. Please tap flag to
report any questions that need improvement.
Question 482: A 17-year-old male involved in a motor vehicle collision was found alert
and stable at the scene but complaining of chest pain and shortness of breath. He was not
wearing a seat belt and was suddenly thrown onto the dashboard. His airway is patent, and
pulmonary exam reveals diminished breath sounds on the left. There are no signs of penetrating
injury, subcutaneous emphysema, or flail chest. His blood pressure is 110/60 mmHg, pulse 100
beats per minute, and oxygen saturation 92%. A portable chest x-ray shows opacification of the
left chest with a midline shift to the right. What is the next step in the management of this
patient?

Choices:
1. Chest ultrasound
2. Chest CT
3. Rib radiographs
4. Chest tube
Answer: 4 - Chest tube
Explanations:
Chest x-ray is often the first imaging modality in patients with chest trauma. However,
ultrasound is more specific for evaluation of pleural effusions and hemothorax. Ultrasound
is usually readily available, rapid, repeatable, and reliable in experienced operators. In this
case, a chest x-ray showing tension hemothorax was performed first. The next appropriate
step is to stabilize the patient with a chest tube, rather than additional imaging. Lung
ultrasound can be used to identify small pleural effusions greater than 20 mL in cases where
the chest x-ray findings are equivocal.
Chest CT would be the gold standard to identify all possible thoracic injuries. In this
patient, who is displaying radiologic evidence of tension hemothorax, and signs of
impending decompensation, a chest thoracostomy tube would be the next best step. Further
imaging may be obtained once the patient becomes more stable.
A CT scan is the gold standard imaging modality to evaluate intrathoracic injuries.
However, in an acute trauma setting, a plain radiograph is usually adequate to initiate
treatment. A rib series may be appropriate for a more stable, ambulatory patient with a low
energy mechanism of injury.
Once a traumatic pleural effusion sufficient to opacify an entire hemithorax is identified,
chest tube placement should not be delayed for further imaging.

Go to the next page if you knew the correct answer, or click the link image(s) below to further
research the concepts in this question (if desired).

Research Concepts:
Hemothorax

We update eBooks quarterly and Apps daily based on user feedback. Please tap flag to
report any questions that need improvement.
Question 483: A 17-year-old boy with a history of Lennox-Gastaut syndrome, chronic
pain, and depression is rushed to the emergency department after being found unconscious in his
bed at home. Vital signs show temperature 98.4 F, blood pressure 110/88 mmHg, heart rate
58/min, respiratory rate 4/min, and SpO2 94% on room air. On exam, he is unable to open his
eyes or follow commands but does withdraw to pain. His father is still recovering from the flu,
and the patient the previous night complained of fatigue, but otherwise, the patient has had no
known sick contacts. His current medications include buprenorphine, fluoxetine, and
lamotrigine. He was recently started on cannabidiol (CBD) for treatment for his seizures, as well
as starting the ketogenic diet a few weeks ago. What is the most likely cause of this patient's
clinical presentation?

Choices:
1. Medication interaction
2. Nutritional deficiencies
3. Influenza
4. Suicide attempt
Answer: 1 - Medication interaction
Explanations:
Buprenorphine and cannabidiol (CBD) should not be used together due to potentials serious
side effects, including respiratory depression, coma, and death.
This patient has a respiratory rate of 4 and is likely suffering from a potentially fatal drug
interaction.
Currently, cannabidiol (CBD) has only been approved for children ages two and older who
suffer from the seizure disorders Lennox-Gastaut syndrome and Dravet syndrome.
The ketogenic diet has been proven to be beneficial for patients with seizure disorders.
While the patient may have nutritional deficiencies, this is not the cause of the respiratory
depression. The patient was complaining of fatigue, which can be a sign of influenza.
However, it is unlikely that the patient's respiratory rate is depressed because of an infection
by the flu. The patient suffers from depression, but besides this, no other clues are
suggesting a suicidal attempt. It is more likely that the patient is suffering from an almost
fatal drug interaction between buprenorphine and CBD.

Go to the next page if you knew the correct answer, or click the link image(s) below to further
research the concepts in this question (if desired).

Research Concepts:
Cannabidiol (CBD)

We update eBooks quarterly and Apps daily based on user feedback. Please tap flag to
report any questions that need improvement.
Question 484: A 55-year-old male is brought to the emergency department following a
high-energy explosion that set fire to a building. The patient is brought in by the emergency
medical technicians in a semi-conscious state and is violently coughing. A primary survey
reveals a lean, old man in obvious respiratory distress. His vital signs show a heart rate of 100
beats/minute, a blood pressure of 110/70 mmHg, a respiratory rate of 16/minute, and an oxygen
saturation of 90% in room air. Which of the following most accurately describes the blast phase
injury in this patient?

Choices:
1. Primary blast injury
2. Secondary blast injury
3. Tertiary blast injury
4. Quaternary blast injury
Answer: 4 - Quaternary blast injury
Explanations:
Quaternary blast injuries include all blast-induced injuries and conditions that are not
caused by primary, secondary, and tertiary blast effects.
These injuries can be caused by exposure to resulting fire, fumes, radiation, biological
agents, smoke, dust, toxins, environmental exposure, and the psychological impact of the
event.
Exacerbation of underlying medical conditions such as COPD, asthma, unstable angina,
anxiety, depression is also a part of quaternary blast injuries and has a major impact on
mortality and morbidity.
In an intentional explosion, other substances can be added to cause more injury, illness, or
to induce more fear and panic in the community.

Go to the next page if you knew the correct answer, or click the link image(s) below to further
research the concepts in this question (if desired).

Research Concepts:
Blast Injuries

We update eBooks quarterly and Apps daily based on user feedback. Please tap flag to
report any questions that need improvement.
Question 485: A 66-year-old Hispanic male is brought to the emergency department after
being found unconscious at his home. His past medical history is significant for uncontrolled
diabetes mellitus type 2 and hypertension. He is currently non-compliant with his medications
per the family members at the bedside. Blood glucose was found to be 300 mg/dL. IV fluids,
potassium, and insulin were initiated as per the DKA protocol. Four hours later, the patient's
blood pressure dropped precipitously to 92/54 mmHg, and his heart rate became elevated at 110
bpm. It was then discovered that the patient had a 5x6 cm sacral ulcer with moderate erythema
surrounding the base and moderate warmth on palpation. The ulcer was cleaned and dressed
appropriately. Blood cultures were sent. He was then transferred to the intensive care unit (ICU)
and started on empiric IV antibiotics for sepsis. 48 to 72 hours after admission into the ICU, the
provider orders a serum protein marker that helps distinguish if the antibiotics are working
appropriately. Which of the following increases this protein directly?

Choices:
1. Lipopolysaccharide (LPS)
2. Tumor necrosis factor (TNF)
3. Interleukin-1 (IL-1)
4. Interferon-gamma (IFN-Y)
Answer: 1 - Lipopolysaccharide (LPS)
Explanations:
Procalcitonin (PCT) is a serum biomarker that is upregulated following a bacterial infection.
Current guidelines recommend that PCT levels should be ordered 48-72 hours following the
initiation of antibiotics to ensure that the correct regimen was initiated. If the antibiotics are
effective, the levels should drop accordingly. Lipopolysaccharide (LPS) is a lipid and
polysaccharide molecule found in the outer membrane of gram-negative bacteria that
increases procalcitonin directly along with inducing a robust immune response.
TNF, also known as cachectin, is a pro-inflammatory cytokine that also increases
procalcitonin but does so indirectly.
Interleukin 1 is a cytokine that is an endogenous pyrogen. It increases procalcitonin
indirectly.
Interferon gamma is the major macrophage-activating cytokine with antiviral properties. It
decreases procalcitonin.

Go to the next page if you knew the correct answer, or click the link image(s) below to further
research the concepts in this question (if desired).

Research Concepts:
Procalcitonin

We update eBooks quarterly and Apps daily based on user feedback. Please tap flag to
report any questions that need improvement.
Question 486: A 60-year-old female with a history of hypertension, diabetes, atrial
fibrillation presents to the emergency department with altered mental status and new-onset right-
sided weakness that started an hour prior to arrival. On presentation, blood pressure was 200/140
mmHg. Examination revealed an NIH stroke scale of 25. Neurological examination was
significant for global aphasia, right-sided hemiparesis, and sensory loss. Further laboratory
investigations showed a normal complete blood count, serum sodium of 130 mEq/L. Initial
computed tomography head without contrast showed a dense middle cerebral artery on the left
with the loss of grey-white matter differentiation in the left frontoparietal cortex and sulcal
effacement. Which of the following interventions will prevent secondary brainstem hemorrhages
from occurring?

Choices:
1. Early involvement of neurosurgery for possible decompressive hemicraniectomy
2. Correction of serum sodium
3. Aggressive blood pressure control
4. Discontinuing anticoagulation
Answer: 1 - Early involvement of neurosurgery for possible decompressive hemicraniectomy
Explanations:
Secondary brainstem hemorrhages or Duret hemorrhages happen in the clinical setting of
transtentorial herniation of any cause. The patient described likely has a malignant left
middle cerebral artery infarct. Such patients are at risk of brain edema, which can progress
to cause transtentorial herniation and thereby secondary brainstem hemorrhages. Early
involvement of neurosurgery for a possible decompressive hemicraniectomy can prevent
herniation and secondary brainstem hemorrhage to occur.
In malignant middle cerebral artery infarcts, brain edema occurs between day 1 to day 5.
These patients require serial computed tomography head.
Kinking of posterior cerebral artery against tentorium cerebelli can result in occipital lobe
infarcts, which can further increase intracranial pressure and risk of herniation.
Aggressive blood pressure control and discontinuation of anticoagulation should be done to
prevent the hemorrhagic conversion of the infarct. While serum sodium should be corrected
and maintained between above 140 mmol/L but it alone may not prevent herniation.

Go to the next page if you knew the correct answer, or click the link image(s) below to further
research the concepts in this question (if desired).

Research Concepts:
Duret Hemorrhages

We update eBooks quarterly and Apps daily based on user feedback. Please tap flag to
report any questions that need improvement.
Question 487: A 36-year-old patient presents to the emergency department with severe
epigastric pain and vomiting. He is diagnosed with severe alcoholic pancreatitis and
subsequently admitted to the intensive care unit. He requires vasopressors for hemodynamic
instability and mechanical ventilation. He is kept nil-per-os (NPO), and total parenteral nutrition
supplementation (TPN) is initiated. After 1 week of TPN supplementation, there is no
improvement in pancreatitis; therefore, a feeding jejunostomy is placed. Twenty-four hours after
initiation of jejunal tube feeds, he is noted to be hypotensive and has high peak pressure changes
on the ventilator. An arterial blood gas analysis is suggestive of metabolic alkalosis. Which of
the following is associated with the pathophysiology of this patient’s change in condition?

Choices:
1. Increased glycogenolysis
2. Decreased gluconeogenesis
3. Increased secretion from alpha pancreatic cells
4. Extracellular shift of electrolytes like potassium and phosphate
Answer: 2 - Decreased gluconeogenesis
Explanations:
Refeeding syndrome is associated with a rapid increase in insulin.
Insulin acts to reduce blood glucose. It does this by preventing glycogenolysis (the
breakdown of glycogen into glucose), preventing gluconeogenesis (formation of new
glucose molecules), and increasing glycogenesis (conversion of glucose into glycogen).
Insulin drives electrolytes like potassium and phosphate intracellularly. Insulin is sometimes
used to treat hyperkalemia as it facilitates the diffusion of potassium intracellularly.
The alpha cells of the pancreas secrete glucagon, beta cells secrete insulin, and the delta
cells secrete somatostatin. Glucagon promotes the breakdown of glycogen and increases
blood glucose. The actions of insulin are discussed previously. Somatostatin is an inhibitory
hormone, inhibiting multiple neuroendocrine and gastrointestinal hormones.

Go to the next page if you knew the correct answer, or click the link image(s) below to further
research the concepts in this question (if desired).

Research Concepts:
Feeding Jejunostomy Tube

We update eBooks quarterly and Apps daily based on user feedback. Please tap flag to
report any questions that need improvement.
Question 488: A 62-year-old patient with alcohol use disorder is admitted with liver
failure. A workup reveals that he is developing severe cognitive deficits and a flapping tremor. It
is decided to treat him with an antibiotic. Which of the following should be given?

Choices:
1. Amoxicillin
2. Erythromycin
3. Neomycin
4. Trimethoprim/sulfamethoxazole
Answer: 3 - Neomycin
Explanations:
Hepatic encephalopathy treatment, in part, is aimed at manipulating the intestinal milieu.
Neomycin is an aminoglycoside that is used orally to reduce bowel flora.
It can be used before bowel surgery or to reduce ammonia production in patients with
hepatic encephalopathy.
Neomycin is not given parenterally as it is ototoxic and nephrotoxic.

Go to the next page if you knew the correct answer, or click the link image(s) below to further
research the concepts in this question (if desired).

Research Concepts:
Acute Liver Failure

We update eBooks quarterly and Apps daily based on user feedback. Please tap flag to
report any questions that need improvement.
Question 489: A middle-aged male patient presents to the clinic with a Wagner stage 3
diabetic foot ulcer and has a history of impaired wound healing. The clinician decides to proceed
with hyperbaric oxygen therapy. The clinician monitors the patient as he "dives" into the
hyperbaric oxygen therapy chamber. During decompression, the patient complains of chest pain
and shortness of breath. What is the most likely underlying pathology for this complaint?

Choices:
1. The patient is experiencing acid reflux from laying on his back
2. The patient is experiencing normal side effects of hyperbaric oxygen therapy and increased
atmospheric pressure
3. The patient is experiencing a pneumothorax
4. The patient is experiencing barotrauma to his ears
Answer: 3 - The patient is experiencing a pneumothorax
Explanations:
Pneumothorax is a rare but potentially fatal risk of HBO. Pneumothorax is commonly called
a collapsed lung.
Barotrauma to the ear is a more common but much less morbid side effect of HBOT. It is
commonly referred to as a "squeeze".
History of air-trapping lung lesion, spontaneous pneumothorax, or mechanical ventilation
put one at increased risk for pneumothorax during HBOT.
Pneumothorax can progress to tension pneumothorax and decreased venous return to the
heart. Treatment usually includes the insertion of a chest tube.

Go to the next page if you knew the correct answer, or click the link image(s) below to further
research the concepts in this question (if desired).

Research Concepts:
Impaired Wound Healing

We update eBooks quarterly and Apps daily based on user feedback. Please tap flag to
report any questions that need improvement.
Question 490: A 60-year-old man, with a history of hypertension, obesity, and
dyslipidemia, presents with exertional dyspnea and pitting ankle edema. Which of the following
therapies would best treat the findings expected on his chest x-ray?

Choices:
1. Continuous positive airway pressure (CPAP)
2. High flow supplemental oxygen
3. Vasodilators
4. Opioids
Answer: 1 - Continuous positive airway pressure (CPAP)
Explanations:
Congestive heart failure is associated with interstitial and alveolar edema. The determining
factor as to whether fluid is filtered out of pulmonary vasculature and into the alveoli is a
balance between forces in both the alveoli and the capillaries themselves. Oncotic pressures
are determined by protein and other solutes in the alveoli, as well as the capillary
hydrostatic pressure, exert forces that will force fluid into the alveoli. On the contrary,
oncotic pressures in the pulmonary capillaries and hydrostatic pressure in the alveoli will
force fluid back into the pulmonary capillaries.
Continuous positive airway pressure (CPAP) increases the pressure within the alveoli,
forcing fluid back into pulmonary capillaries. This is the only management strategy or
medication listed that actively pushes fluid out of alveoli.
Supplemental oxygen is provided through the application of CPAP, which acts to both
improve the patient's pulse oximetry and increase the alveolar hydrostatic pressure. This
assumes all other factors like hemoglobin content are normal.
Additional management considerations include vasodilators such as nitroglycerin and
opioids. However, these interventions take time to work. Therefore, the most rapid
intervention in the case of congestive heart failure that will do the most good to improve
ventilation of the patient is CPAP.

Go to the next page if you knew the correct answer, or click the link image(s) below to further
research the concepts in this question (if desired).

Research Concepts:
Continuous Positive Airway Pressure

We update eBooks quarterly and Apps daily based on user feedback. Please tap flag to
report any questions that need improvement.
Question 491: A 51-year-old male is admitted to the hospital for his next chemotherapy
dose for a recently diagnosed non-Hodgkin lymphoma. The patient has no other significant past
medical history. On his 4th day of hospitalization, the patient suddenly develops difficulty
breathing, muscle pain, and a significant worsening of his lower limb edema without any fevers
or chills. The patient had a blood pressure of 155/94 mmHg 2 hours ago. When the nurse inflates
the blood pressure cuff to check the blood pressure, the patient develops muscle spasms in the
same upper limb. He is in mild respiratory distress, and crackles can be auscultated throughout
the lung fields. The patient has a respiratory rate of 24 breaths/min. Neck veins seem to be
distended. Heart rate is 112 beats/min with a regular rhythm. BIlateral 2+ edema in the lower
limb is also noted. Lab reports include serum creatinine of 3.4 mg/dl, leukocyte count of 12,000
cells/mm3, Hgb levels of 9 g/dl, platelet count of 20,000 cells/mm3, sodium levels of 131
mEq/L, potassium levels of 6.3 mEq/L, bicarbonate levels of 15 mEq/L, blood urea of 77 mg/dl,
calcium levels of 6.8 mg/dl, phosphorus levels of 8.6 mg/dl, uric acid levels of 19 mg/dl and
albumin levels of 4 g/dl. Urinalysis reports trace blood and 2+ protein levels. Urine microscopy
revealed two red blood cells/HPF, 4-10 WBC/HPF, uric acid crystals, and granular casts. In
addition to hemodialysis, which of the following is the next most appropriate next step in
management?

Choices:
1. A drug which is a competitive inhibitor of xanthine oxidase
2. A drug which is a recombinant uricase catalyzing uric acid to allantoin
3. A drug that binds and stabilizes tubulin to inhibit microtubule polymerization, impairing
neutrophil chemotaxis and degranulation.
4. A drug that Inhibits reabsorption of uric acid in the distal convoluted tubule
Answer: 2 - A drug which is a recombinant uricase catalyzing uric acid to allantoin
Explanations:
In a patient with hyperuricemia due to tumor lysis syndrome, the most appropriate way to
lower the uric acid levels rapidly is the use of rasburicase, which is a drug that is a
recombinant uricase catalyzing uric acid to allantoin which is more water-soluble than uric
acid.
Because rasburicase also leads to the formation of hydrogen peroxide, treated patients are at
risk for methemoglobinemia and hemolytic anemia. Therefore, patients should be checked
for G6PD deficiency.
Allopurinol and febuxostat work by limiting uric acid formation. Neither of these drugs
metabolizes previously generated uric acid. Thus these drugs can be used only for the
prevention of uric acid formation which is a little too late in this case.
Conditions of accelerated cell breakdown or turnover such as rhabdomyolysis, hemolysis,
and tumor lysis can also be a purine source and thus, increase uric acid production. Uric
acid is filtered in the glomerulus and is mostly reabsorbed in the proximal convoluted
tubule. The distal convoluted tubule is mostly impermeable to uric acid.

Go to the next page if you knew the correct answer, or click the link image(s) below to further
research the concepts in this question (if desired).

Research Concepts:
Hyperuricemia

We update eBooks quarterly and Apps daily based on user feedback. Please tap flag to
report any questions that need improvement.
Question 492: A patient is brought to the emergency room because of behavior problems.
His friend says that he has a history of using illicit drugs. The individual appears to be
hallucinating, euphoric, and extremely agitated. He has reddish skin, slurred speech, and a very
unsteady gait. An ocular exam reveals dilated pupils with nystagmus. Which of the following
drugs was most likely ingested?

Choices:
1. Cocaine
2. Phencyclidine
3. Amphetamine
4. Marijuana
Answer: 2 - Phencyclidine
Explanations:
Phencyclidine also is known as angel dust and is a potent hallucinogen. The drug works as
an N-methyl-D-aspartate receptor antagonist and is a dangerous chemical.
The behavioral effects resemble schizophrenia, and most individuals are euphoric.
Other findings of phencyclidine ingestion include rage, reddish skin, dilated pupils,
nystagmus, and excitation. . PCP causes mydriasis in 6% of patients and miosis in 2%.
The overdose is managed by supportive care and use of sedatives and antipsychotics.

Go to the next page if you knew the correct answer, or click the link image(s) below to further
research the concepts in this question (if desired).

Research Concepts:
Phencyclidine Toxicity

We update eBooks quarterly and Apps daily based on user feedback. Please tap flag to
report any questions that need improvement.
Question 493: A 29-year-old woman at 35 weeks gestation is brought in by ambulance to
the emergency department after being involved in a motor vehicle collision. She endorses pain
only to the left thigh. A pelvic exam reveals only a deformity of the left femur. The patient's vital
signs reveal a temperature of 37.1 C, a heart rate of 105 bpm, blood pressure of 90/55 mmHg, the
respirations of 18/min, and oxygen saturation of 99%. The abdominal is non-tender, and the
gravid uterus is approximately 35 weeks distended. An ultrasound of the uterus reveals posterior
placenta, fetal heart rate of 143 bpm, and no evidence of subchorionic hematoma. X-ray of the
left lower extremity reveals a mid-shaft fracture of the left femur. After Intravenous fluid
administration, the patient's heart rate is 89 bpm, and blood pressure is 110/75 mmHg.
Orthopedic surgery is consulted and recommends immediate surgery of the left femur. While in
the operation room for an open reduction and internal fixation of the femur, the patient's heart
rate spikes to 121 bpm, and blood pressure is 85/49 mmHg. A bolus of intravenous fluid does
improve the patient's hemodynamics. What is the next step in the management of the patient's
condition?

Choices:
1. Thoracotomy
2. Reposition the patient
3. Vasopressor administration
4. Consult obstetrics and gynecology
Answer: 2 - Reposition the patient
Explanations:
The pregnant patient who was previously hemodynamically stable prior to the surgical
intervention is hemodynamically unstable during the procedure. While a number of
etiologies can be potential culprits to this patient's condition, such as fat embolism,
pulmonary embolism, hypovolemic shock, or hemorrhagic shock, the most simple
explanation is that the patient has inferior vena cava (IVC) syndrome because her inferior
cava is being obstructed by the compression of the gravid uterus while she is supine for the
surgery. Therefore, the most non-invasive intervention should be tried first, which is fluid
resuscitation, and if that does not help, then the patient should be repositioned so that the
uterus is away from the IVC.
IVC compression is very common in pregnant women and the incidence increases with
increased gestational age. Pregnant women should be counseled on laying down and
sleeping in the left lateral decubitus position to avoid compression of the IVC.
The IVC courses underneath the gravid uterus and repositioning the patient should relieve
the obstruction, therefore increasing venous blood return to the heart and improving this
patient's hemodynamics.
A thoracotomy is indicated only for penetrating trauma leading to cardiac arrest and would
do more harm to the patient than benefit.

Go to the next page if you knew the correct answer, or click the link image(s) below to further
research the concepts in this question (if desired).

Research Concepts:
Inferior Vena Cava Syndrome

We update eBooks quarterly and Apps daily based on user feedback. Please tap flag to
report any questions that need improvement.
Question 494: A 43-year-old man is brought to the emergency department (ED) for altered
mental status. The patient was found in his house by his neighbor. Upon arrival at the scene,
EMS report the patient was somnolent and looked yellow and did not respond to 0.4 mg of
naloxone. Upon arrival to the ED, the patient has a heart rate of 108/min, blood pressure 93/54
mmHg, respiratory rate 28/min, oxygen saturation 94% on the nasal cannula, and temperature
101.4 F. The patient is noted to have a distended abdomen and upon palpation of the right upper
quadrant, winces in pain, and grunts in discomfort. 30 ccs/kg of IV fluid bolus is given and initial
labs reveal white blood cell count of 18000/microL, AST 89 IU/L, and ALT 119 IU/L. The
patient is taken to the operating room for emergent surgery. Which of the following intubation
medications is most likely to worsen this patient's condition?

Choices:
1. Ketamine
2. Fentanyl
3. Midazolam
4. Rocuronium
Answer: 2 - Fentanyl
Explanations:
The patient is febrile, tachycardic, hypoxic, and has a WBC count of 18000/microL. The
patient is in septic shock. In septic shock, the patient has decreased systemic vascular
resistance, which causes hypotension. For tracheal intubation, induction and sedation
medications should be chosen accordingly. Medications that can cause further hypotension,
such as fentanyl, should be avoided.
Fentanyl is a synthetic opioid that has a rapid onset of action and is rapidly metabolized. It
causes profound analgesia and is best used for sedation after tracheal intubation in patients
with intracranial injuries as it can help reduce intracranial pressure. Conversely, it can cause
a worsening of systemic hypotension in patients with septic shock.
Fentanyl can also cause respiratory depression, and in this patient with hypoxia and
respiratory distress, it is important to avoid fentanyl.
Ketamine (choice 1) can cause laryngospasm but is not contraindicated in patients with
hypotension. Midazolam (choice 3) is a benzodiazepine that can also cause hypotension and
respiratory depression but is easily titratable and has less central nervous system depression
than fentanyl. Rocuronium (choice 4) is an induction agent that is not contraindicated in
patients in septic shock.

Go to the next page if you knew the correct answer, or click the link image(s) below to further
research the concepts in this question (if desired).

Research Concepts:
Tracheal Intubation Medications

We update eBooks quarterly and Apps daily based on user feedback. Please tap flag to
report any questions that need improvement.
Question 495: A 75-year-old African American man presents to the emergency department
for evaluation of shortness of breath. He has a past medical history of asthma, congestive heart
failure with an ejection fraction of 25%, hypertension, and coronary artery disease. He describes
a 2-week history of shortness of breath, which has progressively gotten worse. The patient is
unable to complete full sentences without getting short of breath. He denies any associated chest
pain but states he has been having racing of the heart for the past one week. The patient takes
aspirin, lisinopril, metoprolol, spironolactone, atorvastatin, albuterol inhaler as needed for
wheezing, and he has been compliant with all his medications. On physical exam, the patient has
bibasilar crackles, irregularly irregular heartbeat, and 2+ bilateral lower extremity edema. Vital
signs show blood pressure 140/90 mmHg, heart rate 140/min, and SpO2 95% on room air. EKG
reveals irregularly irregular rhythm, with variable P-R interval and variable P-wave
morphologies. Renal function tests, electrolytes, D-dimer are within normal limits. Serum
magnesium is 2.3 mEq/L and potassium 4.2 mEq/L. In the emergency department, a trial of
intravenous magnesium is given, which does not help with his heart rate. Which of the following
is the next best step in the management of this patient?

Choices:
1. Albuterol/ipratropium nebulizations
2. Repeat intravenous magnesium
3. Verapamil
4. Increase the dose of metoprolol
Answer: 4 - Increase the dose of metoprolol
Explanations:
The patient in the above clinical vignette presented with possible worsening heart failure
caused by multifocal atrial tachycardia (MAT). Increasing the dose of metoprolol to target a
heart rate of 110/min would be the ideal next step of management.
Metoprolol is a selective beta-blocker that suppresses atrial ectopy and also decreases AV
nodal conduction.
It is preferred over calcium channel blockers for heart rate control especially in patients
with heart failure with reduced ejection fraction.
At this time, breathing treatment is not indicated. Patient electrolytes are within normal
limits. One dose of magnesium did not help with the heart rate. Studies have shown
magnesium replacement in patients with or without hypomagnesemia has improved MAT.
Verapamil has been shown to be efficacious in controlling heart rate in patients with
multifocal atrial tachycardia by decreasing AV nodal conduction. In this above clinical
vignette, the patient has reduced ejection fraction in which case the better option would be
to increase dose of metoprolol. Calcium channel blockers are generally avoided in patients
with reduced ejection fraction.

Go to the next page if you knew the correct answer, or click the link image(s) below to further
research the concepts in this question (if desired).

Research Concepts:
Multifocal Atrial Tachycardia

We update eBooks quarterly and Apps daily based on user feedback. Please tap flag to
report any questions that need improvement.
Question 496: A 42-year-old male presents to the emergency department with altered
mental status. The family reports that the patient had recently gone through a divorce and was
found with a bottle of pink-colored pills next to him. His past medical history is significant for
hypertension, allergic rhinitis, and insomnia. His home prescription medications include
lisinopril. His blood pressure is 125/87 mmHg, heart rate is 186 beats/min, respiratory rate is 37
breaths/min, and oxygen saturation (SpO2) is 96% on room air. Physical exam reveals
respiratory distress and dry mucous membranes. Electrocardiogram (ECG) demonstrates a QTc
interval of 525 milliseconds. Ingestion took place approximately 85 minutes ago. He starts
seizing on the gurney, and his SpO2 falls to 79%. What is the best step in the management of this
patient?

Choices:
1. Lorazepam bolus and ICU admission for emergent hemodialysis
2. Lorazepam bolus and emergent intubation for airway protection
3. Diazepam bolus and albuterol nebulizer
4. Intravenous naloxone
Answer: 2 - Lorazepam bolus and emergent intubation for airway protection
Explanations:
Benzodiazepines, such as lorazepam, can be used for symptomatic management of seizures
secondary to diphenhydramine toxicity.
Given that this patient’s SpO2 is declining, continued respiratory distress can be fatal.
Emergent intubation is warranted in this situation to protect the patient’s airway and to
provide respiratory support.
Naloxone is used for opioid reversal and thus, would be ineffective in reversing the effects
of diphenhydramine.

Go to the next page if you knew the correct answer, or click the link image(s) below to further
research the concepts in this question (if desired).

Research Concepts:
Diphenhydramine Toxicity

We update eBooks quarterly and Apps daily based on user feedback. Please tap flag to
report any questions that need improvement.
Question 497: A 43-year-old man is brought to the emergency department after a referral
for the management of diabetic ketoacidosis. He has a history of type 1 diabetes, being on
regular insulin since childhood. His major complaints are high-grade fever and severe headache
for the past 5 days. He was prescribed amoxicillin for suspected sinusitis. His girlfriend reports
that he has had decreased food intake in the past 3-4 days, and he had also skipped his regular
medicines due to extreme lethargy. He also reports severe nausea and vomiting, along with mild
breathing difficulty for the past 8 hours. Examination reveals an unconscious patient in a gasping
state. He is immediately intubated and artificially ventilated. Further examination reveals
proptosis, reddish discoloration, blackish eschar, and swelling of his right eye. His right pupil is
fixed and dilated. His bedside blood glucose is 510 mg/dL, and ketones are strongly positive.
The base excess is -14 mE/L. Treatment for diabetic ketoacidosis is initiated. He is
hemodynamically stable and saturating 98% on room air. A T2 weighted MRI with Fat Sat
sequence reveals soft tissue densities in the right sinuses and orbit, and gadolinium contrast
evaluation confirms thrombosis in the right cavernous sinus. Sporangia and rhizoids were
visualized with lactophenol blue staining of the tissues obtained from debridement. Which of the
following is the next best step in the management of this patient?

Choices:
1. Liposomal amphotericin B
2. Amphotericin B
3. Posaconazole
4. Voriconazole
Answer: 1 - Liposomal amphotericin B
Explanations:
This young patient in diabetic ketoacidosis with evidence of sinus/ orbit and cavernous
sinus involvement merits a high consideration for rhino-orbital cerebral mucormycosis. The
suspicion or diagnosis of rhino-orbital cerebral mucormycosis is a medical as well as
surgical emergency since it carries high morbidity & mortality even with appropriate
treatment. IV liposomal amphotericin is initiated immediately at a dose of 5 mg per Kg
body weight per day, with many clinicians going up to a dose of 10 mg per Kg body
weight/day. Surgical debridement is planned immediately.
Extensive surgical debridement is to be planned at the earliest. This will help to arrive at an
early tissue diagnosis via frozen section specimens. Tissue PCR techniques also will help in
reaching a diagnosis. Surgical debridement is also expected to reduce the fungal load.
Extensive exploration and debridement of sinuses along with orbital decompression or
exenteration is usually done. Local installation of amphotericin is often done in debrided
cavities. The intracranial disease is also debrided as much as feasible. The role of surgical
exploration and debridement in intracranial involvement is a bit controversial since many
experts believe that mortality is inevitable. However, there are case reports of survival in
early intracranial involvement.
Liposomal amphoterin B has better tissue penetration and minimal renal toxicities
compared to parenteral amphotericin B.
IV posaconazole or isavuconazole is used only when the patient is not tolerating
amphotericin or is not responding to amphotericin. They are used as a step-down oral
continuation therapy agent, only after many weeks of IV amphotericin therapy has resulted
in clinical improvement in the given patient.

Go to the next page if you knew the correct answer, or click the link image(s) below to further
research the concepts in this question (if desired).

Research Concepts:
Rhino-orbital Cerebral Mucormycosis

We update eBooks quarterly and Apps daily based on user feedback. Please tap flag to
report any questions that need improvement.
Question 498: A 54-year-old woman presents after noticing red spots develop on both her
legs just above the ankles. Her past medical history includes beta-thalassemia major that is
managed with recurrent blood transfusions. She has no allergies. On examination, she is afebrile,
her respiratory rate is 10/minute, heart rate 78 beats per minute, and blood pressure 134/80
mmHg. Lab results show normal WBC and platelet counts, BUN 30 mg/dL (7-20 mg/dL) and
creatinine 2.0 mg/dL (0.8-1.3 mg/dL ). C4 is low, and C3 is normal. Autoimmune studies show
negative ANA and positive RF and cryoglobulins. Further screening reveals positive HCV
antibody and HCV RNA high viral load. Given the likely diagnosis, which of the following
patient factors indicate a likely poor prognosis?

Choices:
1. Age under 65-years
2. Renal impairment
3. Low C4 complement
4. Skin lesions above ankle
Answer: 2 - Renal impairment
Explanations:
In the absence of liver cirrhosis, renal impairment is an important prognostic factor in HCV
related mixed cryoglobulinemic vasculitis.
Kidney involvement can progress rapidly to renal failure and death.
Renal involvement is a poor prognostic factor in non-infectious related mixed
cryoglobulinemic vasculitis as well.
Poor prognostic factors in the absence of liver failure include proteinuria greater than 1
g/day, serum creatinine greater than 150 micromol/L, cardiomyopathy, gastrointestinal, and
CNS involvement. Age more than 65 years is a poor prognostic factor in non-infectious
related mixed cryoglobulinemic vasculitis. Palpable purpura in the dependent areas like legs
is a common presentation in this disease and not a prognostic factor. C4 can be low due to
immune complex-mediated complement activation and does not dictate prognosis.

Go to the next page if you knew the correct answer, or click the link image(s) below to further
research the concepts in this question (if desired).

Research Concepts:
Cryoglobulinemic Vasculitis

We update eBooks quarterly and Apps daily based on user feedback. Please tap flag to
report any questions that need improvement.
Question 499: A 16-year-old man is brought to the emergency department due to fever,
headache, photophobia, and vomiting for the past two days. His medical history is significant for
hemolytic uremic syndrome diagnosed two weeks ago. The temperature is 39.2 C (102.6 F),
blood pressure is 120/75 mmHg, the pulse is 100/min, and the respiratory rate is 17/min.
Funduscopy was unremarkable. His neck is stiff. White blood cell count is 16,500/mm with
neutrophilic predominance. CT scan of the head is unremarkable. Blood and lumbar puncture
results are suggestive of bacterial meningitis, but cultures are still pending. Which of the
following is responsible for increasing the risk of invasive meningitis?

Choices:
1. Eculizumab
2. Vitamin A supplementation
3. Over the counter antipyretics
4. Meningococcal vaccination
Answer: 1 - Eculizumab
Explanations:
Eculizumab is a monoclonal antibody that binds complement protein C5 leading to terminal
complement cascade inhibition.
The late complement system plays the main role in the host defense against invasive
bacterial meningitis. (1400- to a 10,000-fold increase in meningococcal disease risk)
Terminal complement deficiency is associated with recurrent invasive bacterial infections
(in up to 50% of patients) with a recurrence rate of 100-150 times higher than that in the
general population.
Meningococcal vaccine is a cellular fraction vaccine that is generally safe and doesn't
increase the risk of bacterial meningitis.

Go to the next page if you knew the correct answer, or click the link image(s) below to further
research the concepts in this question (if desired).

Research Concepts:
Meningitis

We update eBooks quarterly and Apps daily based on user feedback. Please tap flag to
report any questions that need improvement.
Question 500: A 16-year-old boy is brought to the emergency department with complaints
of an inability to move his both upper and lower limbs after halfway through a soccer match. He
reports no fever, pain, and change in sensation in his limbs. He went to camping 1 month back
does not recall any insect bite. He does not recall any recent illness and denies the use of any
medication currently. He had a similar episode of muscle weakness a few months back, which
resolved spontaneously. His vital signs are within normal limits. Cardiovascular, respiratory, and
gastrointestinal examinations reveal no abnormality. On neurological examination, muscle
strength is decreased, and knee and ankle reflexes are depressed. His touch, vibration, and
proprioception sensations are intact. Laboratory work reveals sodium 140 mEq/L, potassium 2.2
mEq/L, serum urea nitrogen 16 mEq/L, creatinine 0.7 mg/dL, chloride 106 mEq/L, bicarbonate
24 mEq/L, and creatine kinase 1110 U/L. EKG shows depressed ST-segment and tall T waves.
What is the best initial therapy for this patient?

Choices:
1. Intravenous glucocorticoids
2. Plasma exchange
3. Oral potassium supplementation
4. Equine serum antitoxin
Answer: 3 - Oral potassium supplementation
Explanations:
Sudden onset of flaccid paralysis in a young male patient with past medical of a similar
episode with hypotonia and depressed deep tendon reflexes without changes in the sensation
in the setting of hypokalemia strongly leads to suspicion of the hypokalemic periodic
paralysis.
Oral potassium solution is the initial management approach for the patient presenting with
an attack of muscle weakness in hypokalemic periodic paralysis.
Intravenous glucocorticoids, equine serum antitoxin, plasma exchange are ineffective in
hypokalemic periodic paralysis. Administration of the oral potassium usually alleviates the
symptoms of the hypokalemic periodic paralysis.
Absence of fever, no insect bite, or any recent illness, intact sensation on examination, are
helpful to rule out the possible causes of flaccid paralysis like acute viral myelitis, Guillian-
Barre syndrome, or tick paralysis. The occurrence of the symptoms several hours after
strenuous exercise, playing in a soccer match in this patient, is more suspicious for
hypokalemic periodic paralysis. Prolonged rest after strenuous exercise is an identified
triggering factor for flaccid paralysis in hypokalemic periodic paralysis.

Go to the next page if you knew the correct answer, or click the link image(s) below to further
research the concepts in this question (if desired).

Research Concepts:
Hypokalemic Periodic Paralysis

We update eBooks quarterly and Apps daily based on user feedback. Please tap flag to
report any questions that need improvement.
Section 6
Question 501: An 86-year-old man with a past medical history of follicular cell lymphoma
status post-chemotherapy presents to the emergency department for the fourth time this month
with abdominal pain, bloating, and an inability to have a bowel movement for 6 days and pass
gas for 2 days. A CT scan of the abdomen is done and is consistent with small bowel obstruction
secondary to a large mesenteric mass. The patient is consistently nauseous and vomiting, and he
is unable to tolerate anything orally. Which of the following is the most appropriate indication
for a percutaneous endoscopic gastrostomy (PEG) tube placement in this patient?

Choices:
1. Mechanical reopening of the obstruction
2. Increasing the caloric intake
3. Stimulating a bowel movement
4. Decompression of the stomach
Answer: 4 - Decompression of the stomach
Explanations:
A PEG tube provides enteral feeds in a patient with malnutrition. However, enteral feeds do
not aid in relieving his distended stomach and symptoms secondary to his bowel
obstruction.
PEG tubes can also be used to decompress the stomach due to direct percutaneous access to
the stomach.
PEG tube can be used to decompress the stomach in a patient with a gastric obstruction
causing a distended stomach.
Percutaneous endoscopic gastrostomy (PEG) tubes serve as the favorable route of feeding
and nutritional support in patients with a functional gastrointestinal (GI) system who require
long-term enteral nutrition, usually beyond 4 weeks.

Go to the next page if you knew the correct answer, or click the link image(s) below to further
research the concepts in this question (if desired).

Research Concepts:
Percutaneous Endoscopic Gastrostomy Tube

We update eBooks quarterly and Apps daily based on user feedback. Please tap flag to
report any questions that need improvement.
Question 502: A 63-year-old male presents with severe shortness of breath and chest pain
on inspiration with bluish discoloration of the fingers. On exam, his blood pressure is 100/70
mmHg, pulse 105/min, respiratory rate 26/min, and SpO2 is 81% on room air and 83% on
supplemental oxygen. On auscultation, S1 and S2 are normal with no added sounds. Chest
auscultation reveals fine inspiratory crepitations. The differentials include cardiogenic edema and
acute respiratory distress syndrome. Which of the following differentiates acute respiratory
distress syndrome from cardiogenic pulmonary edema?

Choices:
1. ST-segment changes on electrocardiogram
2. Bilateral lung infiltrates on chest x-ray
3. Pulmonary capillary wedge pressure (PCWP) less than 18 mm Hg
4. Hypoxemia
Answer: 3 - Pulmonary capillary wedge pressure (PCWP) less than 18 mm Hg
Explanations:
Acute respiratory distress syndrome (ARDS) can be differentiated from cardiogenic
pulmonary edema by pulmonary capillary wedge pressure (PCWP), which is measured by
an invasive technique (Swan Ganz catheter). Cardiogenic edema has PCWP greater than 18
mm Hg, but ARDS has PCWP of less than 18 mmHg. Cardiogenic pulmonary edema
increases the pressure in the left atrium and hence an increased PCWP.
Cardiogenic pulmonary edema can also be differentiated from ARDS further by B-type
natriuretic peptide (BNP)/pro BNP levels. It tends to increase significantly in cardiogenic
pulmonary edema as it is released by the walls of the atria and ventricles of the heart.
Radiological investigations like chest x-ray and echocardiography can also be used for
differentiation between cardiogenic pulmonary edema and ARDS. A chest x-ray will show
cardiomegaly, vascular redistribution, and pleural effusion in cases of cardiogenic
pulmonary edema, while these findings are less likely in ARDS. Echo can also be used to
differentiate the two as it can detect any valvular defects of the heart, ejection fraction, and
wall dysfunction.
Tachycardia, hypoxemia, and lung infiltrations are the common findings in both cardiogenic
edema and ARDS. Still, hypoxemia of cardiogenic cause responds quickly to high flow
supplemental oxygen, while hypoxemia of ARDS can be refractory to oxygen therapy.

Go to the next page if you knew the correct answer, or click the link image(s) below to further
research the concepts in this question (if desired).

Research Concepts:
Cardiogenic Pulmonary Edema

We update eBooks quarterly and Apps daily based on user feedback. Please tap flag to
report any questions that need improvement.
Question 503: A 68-year-old male with a past medical history of congestive heart failure
with recurrent pleural effusions undergoes an indwelling pleural catheter insertion. Three weeks
later, the patient presents with fever, difficulty breathing, and cough. The patient says he is
unable to drain the pleural catheter. Computed tomography chest reveals large right-sided pleural
effusion, and the pleural catheter was in place. Diagnostic thoracentesis shows empyema with
loculated effusion. With therapeutic thoracentesis, the patient's lung fails to expand. What is the
best next step?

Choices:
1. Repeat thoracentesis
2. Remove the pleural catheter and replace the new one
3. TPA through the pleural catheter
4. Decortication
Answer: 4 - Decortication
Explanations:
It is characterized by the lung's inability to expand and fill the thoracic cavity due to a
restricting fibrous visceral pleural peel. The resulting chronic pleural space is fluid-filled,
and the persistence of the fluid is solely due to hydrostatic equilibrium. As the patient's lung
is not expanding secondary to the trapped lung, repeated thoracentesis is not helpful.
As the patient has empyema, pleural catheters should be removed, but as the patient has
extensive, complicated effusion with trapped lung, decortication is the treatment of choice.
Decortication is an option for lung re-expansion if symptoms persist 6 months after
empyema resolution.
TPA through the pleural catheters can be tried to see if catheter drains. Still, as the patient
has empyema with trapped lung, it is recommended to remove the existing pleural catheter,
followed by decortication.
Repeated pleural catheter insertion, especially in the setting of empyema, is not
recommended.

Go to the next page if you knew the correct answer, or click the link image(s) below to further
research the concepts in this question (if desired).

Research Concepts:
Trapped Lung

We update eBooks quarterly and Apps daily based on user feedback. Please tap flag to
report any questions that need improvement.
Question 504: A 68-year-old man presents with severe respiratory distress. He has a past
medical history of chronic obstructive pulmonary disease (COPD), hypertension, and
hyperlipidemia. His medications include metoprolol, inhaled fluticasone/salmeterol, and
atorvastatin. His vital signs show oxygen saturation 87% on room air, respiratory rate 32 per
minute, heart rate 106 bpm, blood pressure 110/72 mmHg, and temperature 101.4 F. A chest x-
ray shows consolidation in the right lung base. Arterial blood gases show pH 7.30 (7.35-7.45),
PaO2 56 mmHg (75-100), and PaCO2 48 mmHg (35-45). The patient is started on empiric
antibiotics, blood cultures are performed, and he is intubated. After 12 hours, he is noted to have
increased intracranial pressure. Which of the following findings would also be expected?

Choices:
1. Hyponatremia
2. Hypernatremia
3. Hyperkalemia
4. QT prolongation
Answer: 4 - QT prolongation
Explanations:
This patient likely was induced with sevoflurane, which is a halogenated inhalational
anesthetic approved for induction and maintenance of general anesthesia used in adults and
pediatric patients for inpatient and outpatient surgery.
Sevoflurane is a volatile anesthetic. This drug provides hypnosis, amnesia, analgesia,
akinesia, and autonomic blockade during surgical and procedural interventions.
Sevoflurane can cause QT prolongation. It can rarely cause increased intracranial pressure.
Succinylcholine can cause hyperkalemia.

Go to the next page if you knew the correct answer, or click the link image(s) below to further
research the concepts in this question (if desired).

Research Concepts:
Sevoflurane

We update eBooks quarterly and Apps daily based on user feedback. Please tap flag to
report any questions that need improvement.
Question 505: A 16-year-old girl is brought to the emergency department with dizziness,
weakness, lack of appetite, and lethargy for three weeks. She further states that lately, she has
been craving salt, which is unusual for her. She has a history of asthma. Her mother has type 1
diabetes and Hashimoto thyroiditis. On examination, her blood pressure in the lying position is
105/65 mmHg with a pulse rate of 95/min. On standing, she is noted to have a blood pressure of
85/55 mmHg with a pulse rate of 118/min. Her temperature is 99.8 F. An initial set of
investigations is shown below.
Patient value Reference range
Sodium 126 mEq/L 134-144 mEq/L
Potassium 5.9 mEq/L 3.6-5.0 mEq/L
Chloride 113 mEq/L 98-107 mEq/L
Bicarbonate 21 mEq/L 21-28 mEq/L
BUN 43 mEq/L 5-20 mg/dL
Creatinine 1.9 mg/dL 0.8-1.4 mg/dL
Glucose 50 mg/dL 80–140 mg/dL
Urine sodium 62 mEq/L >20 mEq/L
She is given appropriate hormonal therapy to treat the underlying pathology. Which of the
following is the most appropriate fluid therapy for this patient?

Choices:
1. 5% dextrose in water
2. 5% albumin
3. 0.9% saline
4. Lactated ringer
Answer: 3 - 0.9% saline
Explanations:
Hyponatremia is defined as a serum sodium concentration of less than 135 mEq/L but can
vary to some extent depending upon the set values of varied laboratories. Hyponatremia is a
common electrolyte abnormality caused by an excess of total body water in comparison to
that of the total body sodium content.
The etiology of hyponatremia can be classified based upon the volume status of the
extracellular fluid. As mentioned earlier, sodium is the major solute of extracellular fluid
(ECF). Based upon the volume of ECF, a patient can be classified into hypovolemic,
euvolemic, or hypervolemic. Physiological stimuli that cause vasopressin release in adjunct
with increased fluid intake can cause hyponatremia. Hypothyroidism and adrenal
insufficiency may contribute to an increased release of vasopressin. Physiological stimuli
for vasopressin release include loss of intravascular volume (hypovolemic hyponatremia)
and the loss of effective intravascular volume (hypervolemic hyponatremia).
Symptoms depend upon the degree and chronicity of hyponatremia. Patients with mild-to-
moderate hyponatremia (greater than 120 mEq/L) or a gradual decrease in sodium (greater
than 48 hours) have minimal symptoms. Patients with severe hyponatremia (less than 120
mEq/L) or rapid decrease in sodium levels have multiple varied symptoms. Symptoms can
range from anorexia, nausea and vomiting, fatigue, headache, and muscle cramps to altered
mental status, agitation, seizures, and even coma.
This patient's orthostatic hypotension, electrolyte abnormalities, and arranged renal profile
indicate adrenal insufficiency, likely Addison disease. In addition to hydrocortisone and
fludrocortisone administration, the patient requires normal saline to increase total body
volume. Both volume depletion and hormone replacement improve blood pressure and
electrolytes.

Go to the next page if you knew the correct answer, or click the link image(s) below to further
research the concepts in this question (if desired).

Research Concepts:
Hyponatremia

We update eBooks quarterly and Apps daily based on user feedback. Please tap flag to
report any questions that need improvement.
Question 506: A 65-year-old male presents to the emergency department with diarrhea and
muscle weakness. The patient states that for the past 5 days, he has been having difficulty
walking around the house. His past medical history is significant for gastroesophageal reflux
disease, diabetes mellitus, dyslipidemia, and hypothyroidism. The patient states he does not
know what medication he takes but that they are all "over-the-counter." The physical exam is
unremarkable. However, lab tests reveal an ionized calcium level of 5.2 mg/dL, magnesium level
of 16 mEq/L, and phosphate 4.2 mg/dL. The patient's vital signs are abnormal. However, he is
hemodynamically stable. Which of the following abnormalities is likely to be found in vital
signs?

Choices:
1. Tachycardia
2. Hypotension
3. Tachypnea
4. Bradycardia
Answer: 2 - Hypotension
Explanations:
Magnesium toxicity that is uncorrected can eventually cause SA/AV node block and
eventually cardiac arrest. However, this patient's vital signs are noted to be abnormal but
otherwise stable and magnesium toxicity does not cause bradycardia or tachycardia.
The toxic effects of magnesium are inherently linked to the levels (mEq/liters) found in the
serum. As the levels of magnesium rise, different symptoms start to manifest and the fatality
of those symptoms is proportional to the levels of magnesium found.
Starting at 5-10 mEq/L, patients will begin to develop ECG changes (prolonged PR interval,
widened QRS). At 10 mEq/L, there will be a loss of deep tendon reflexes and muscle
weakness. At 15 mEq/L, signs of abnormal conductivity surface as SA/AV node block.
Additionally, patients begin to experience respiratory paralysis. At 20 mEq/L or higher, the
patient is likely to experience cardiac arrest.
One of the most common vital sign findings in magnesium toxicity is hypotension. This is
due to the vasodilatory effect of magnesium and so patients who have elevated levels of
magnesium can develop severe hypotension.

Go to the next page if you knew the correct answer, or click the link image(s) below to further
research the concepts in this question (if desired).

Research Concepts:
Magnesium Toxicity

We update eBooks quarterly and Apps daily based on user feedback. Please tap flag to
report any questions that need improvement.
Question 507: A 65-year-old male comes in with abdominal distension, anorexia,
insomnia, and generalized weakness. He has a history of alcohol use disorder. Two days back, he
met his primary care provider who started lactulose, multivitamins, furosemide, decreased his
daily protein intake, and performed a small volume paracentesis. His son says that for the past 6
hours, the patient has been drowsy and non-responsive. Physical examination reveals trembling
of his hands when held at 90 degrees with his arms fully extended. Which of the following could
have been the most likely reason for this patient's sudden deterioration from his baseline?

Choices:
1. Procedure performed to relieve his abdominal distension
2. Reduction of protein from his diet
3. Lactulose
4. Anti-hypertensive medication initiation
Answer: 4 - Anti-hypertensive medication initiation
Explanations:
This patient with alcohol use disorder presents with hepatic encephalopathy (HE). Triggers
of HE include renal failure, gastrointestinal bleeding (e.g., esophageal varices),
constipation, infection, medication non-compliance, excessive dietary protein intake,
dehydration (e.g., fluid restriction, diuretics, diarrhea, vomiting, excessive paracentesis),
electrolyte imbalance, consumption of alcohol, or consumption of certain sedatives,
analgesics or diuretics all in the setting of chronic liver disease.
This patient was started on furosemide, which was the trigger. In some cases, hepatic
encephalopathy may occur following the creation of a transjugular intrahepatic
portosystemic shunt (TIPS).
Under normal conditions, ammonia is produced by bacteria in the gastrointestinal tract (e.g.,
a breakdown product of amines, amino acids, purines, and urea) followed by metabolism
and clearance by the liver. In the case of cirrhosis or advanced liver dysfunction, however,
there is either a decrease in the number of functioning hepatocytes, portosystemic shunting,
or both, resulting in decreased ammonia clearance and hyperammonemia.
Antibiotics (e.g., rifaximin, neomycin/paromomycin/metronidazole, or vancomycin) are
often given empirically due to the frequency of infection as an underlying cause. Additional
treatment measures include lactulose/lactitol (a non-absorbable osmotic laxative that also
helps convert ammonia to non-absorbable ammonium in the gastrointestinal tract), LOLA
(L-ornithine and L-aspartate preparation - increases the use of ammonia in the urea cycle to
produce urea), zinc (to correct underlying deficiency common in cirrhotic patients) either
alone or in combination with each other or antibiotics.

Go to the next page if you knew the correct answer, or click the link image(s) below to further
research the concepts in this question (if desired).

Research Concepts:
Hepatic Encephalopathy

We update eBooks quarterly and Apps daily based on user feedback. Please tap flag to
report any questions that need improvement.
Question 508: A 65-year-old African American woman is being evaluated in the intensive
care unit. She presented to the emergency department with an acute left lower quadrant
abdominal pain 16 days ago. She has a history of recurrent diverticulitis and underwent an urgent
segmental resection with primary anastomosis. The patient started receiving parenteral nutrition
with glucose and lipid infusion rates of 7 mg/kg/min and 0.1 g/kg/hour. On postoperative day 13,
the patient expresses concern over a new right upper quadrant pain. Vital signs show blood
pressure 130/95 mmHg, temperature 96.8 F, and respiratory rate 12/min. AST and ALT are 80
IU/L and 95 IU/L, respectively. Which of the following is the next best step in the management
of this patient?

Choices:
1. Decreasing glucose infusion rate
2. Discontinue peripheral parenteral nutrition
3. Discontinue total parenteral nutrition
4. Perform a right upper quadrant ultrasound
Answer: 1 - Decreasing glucose infusion rate
Explanations:
Increased glucose in the blood induces hepatic lipogenesis. Increased glucose also triggers
an increase in the insulin level leading to even more lipogenesis.
Increased hepatic lipogenesis can lead to eventual steatosis of the liver.
This can be managed by decreasing dextrose dosage to under 5 g/kg day and substituting
30% of dextrose energy with lipids.
Substituting 30% of dextrose energy with lipids can decrease the amount of insulin in the
blood.

Go to the next page if you knew the correct answer, or click the link image(s) below to further
research the concepts in this question (if desired).

Research Concepts:
Total Parenteral Nutrition

We update eBooks quarterly and Apps daily based on user feedback. Please tap flag to
report any questions that need improvement.
Question 509: A 75-year-old American Association Anesthesiology class-III (ASA-III)
man with a history of coronary artery disease, chronic renal disease, and obstructive sleep apnea
is taken to the operating room for inguinal hernia repair. He had been seen by a cardiologist and
cleared for surgery. General anesthesia via laryngeal mask airway (LMA) is considered with
maintenance using desflurane. Vital signs are normal prior to pre-op, and induction and
placement of the LMA are uneventful. Ten minutes later, during surgical prep, the patient's vital
signs show HR 115/min, BP 148/96 mmHg, RR 14/min, SpO2 99%, EtCO2 45 mmHg, and T
98.9 F (37.1 C). The end-tidal desflurane value reads 8.5, and the only other medication
administered at this point was cefazolin. What is the most likely cause for the patient's clinical
condition?

Choices:
1. Rapid titration of desflurane
2. Malignant hyperthermia
3. Insufficient depth of anesthetic
4. Allergic reaction to perioperative antibiotic administered
Answer: 1 - Rapid titration of desflurane
Explanations:
Desflurane, especially when rapidly titrated, will stimulate the sympathetic nervous system.
A higher flow rate on the anesthesia machine can lead to an increase in the fraction of
inspired volatile, which will yield more rapid titration of anesthetic. This rapid change in
end-tidal volatile, rather than a slow titration, can increase HR and BP transiently.
In this patient with a history of CAD, sympathetic stimulation should be avoided to prevent
further ischemia.
While desflurane is a malignant hyperthermia triggering agent, it is unlikely given the
patient's modest elevation in EtCO2, which is more likely explained by his history of OSA.
The elevation in HR and BP are non-specific, and it would be much more likely to
malignant hyperthermia with a much more rapid elevation in EtCO2. The non-specific
nature of isolated tachycardia and hypertension are much more commonly associated with
rapid titration of desflurane as opposed to MH in this scenario. Inadequate depth of
anesthesia certainly presents as hypertension and tachycardia. However, it is much less
likely in this scenario where the patient has no surgical stimulation yet as the prep is
ongoing, and it has been 10 minutes since the stimulation of placing the LMA occurred. In
addition, the patient's MAC is > 1, which is more than adequate given the lack of
stimulation at this time. It is because of the rapid titration of the volatile agent desflurane
that likely led to tachycardia and hypertension. Allergic or anaphylactic reactions are
common with steroidal non-depolarizing neuromuscular blocking drugs as well as
perioperative antibiotics. However, it is much more commonly associated with hypotension,
tachycardia, and increased peak airway pressures. The etiology of this patient's tachycardia
and hypertension are much more likely caused by sympathetic stimulation from desflurane.
The mechanism by which the sympathetic stimulation occurs is not fully understood, but
rapid titration tends to increase the incidence of stimulation.

Go to the next page if you knew the correct answer, or click the link image(s) below to further
research the concepts in this question (if desired).

Research Concepts:
Desflurane

We update eBooks quarterly and Apps daily based on user feedback. Please tap flag to
report any questions that need improvement.
Question 510: A 16-years-old male presents with sudden onset of severe headache, low-
grade fever, and diplopia. A few days ago he underwent surgical drainage for a furuncle on his
midface. On examination, there is a sick looking young male with periorbital edema, chemosis,
lateral gaze palsy, and ptosis on the right side. On fundoscopy, there is bilateral disc edema.
Lumbar puncture reveals elevated cerebrospinal fluid (CSF) opening pressure. What is the most
appropriate initial management option?

Choices:
1. Broad-spectrum antibiotics
2. Low molecular weight heparin
3. Corticosteroids
4. Immediate neurosurgical intervention
Answer: 1 - Broad-spectrum antibiotics
Explanations:
This is the case of cavernous sinus thrombosis secondary to staphylococcus aureus
infection. The patient has developed nerve palsies and raised intracranial pressure.
It is advised to start broad-spectrum antibiotics initially. Low molecular weight heparin and
neurosurgical intervention have controversial roles.
When there is raised intracranial pressure, it is prudent to keep the head elevated to 30
degrees and neutrally positioned to minimize venous outflow resistance and improve
cerebral spinal fluid displacement from the intracranial to the spinal compartment.
Hypoxia and hypercapnia can increase intracranial pressure (ICP). Controlling ICP through
optimal respiratory management is crucial. It is essential to control ventilation to maintain a
normal PaCO2 and maintain adequate oxygenation without increasing the positive end-
expiratory pressure (PEEP).

Go to the next page if you knew the correct answer, or click the link image(s) below to further
research the concepts in this question (if desired).

Research Concepts:
Intracranial Hypertension

We update eBooks quarterly and Apps daily based on user feedback. Please tap flag to
report any questions that need improvement.
Question 511: A 16-year-old boy is being evaluated in the intensive care unit on the fourth
day of admission due to pneumonia. He has been requiring multiple sedatives and analgesics
while on a pressure-assist ventilator. Due to ongoing agitation, the pharmacist warns an
intensivist about the maximum dose of sedatives and opioid use. A plan is made to change the
ventilator mode to neurally-adjusted ventilatory assist (NAVA). Following the switch, arterial
blood gas analysis shows pH 7.42, pCO2 39 mmHg, and pO2 150 mmHg. He is currently on
NAVA of 2 cmH2O/microV and Edi max is 8 microV. Which of the following is the next best
step in the management of this patient?

Choices:
1. Decrease NAVA by 0.5 cmH2O/microV
2. Extubate patient now
3. Increase NAVA level by 3 cmH2O/microV
4. Change back to the pressure-assist ventilator
Answer: 1 - Decrease NAVA by 0.5 cmH2O/microV
Explanations:
As NAVA improves asynchrony during ventilation, it may be helpful in decreasing
agitation and lower the need for sedative medications.
Arterial or capillary blood gas is an important tool to monitor ventilator requirements.
If blood gas is acceptable, NAVA level can be weaned by 0.5-1 cmH2O/microV.
It is recommended to consider extubation once the NAVA level reaches 1cmH2O/microV.

Go to the next page if you knew the correct answer, or click the link image(s) below to further
research the concepts in this question (if desired).

Research Concepts:
Neurally Adjusted Ventilatory Assist (NAVA)

We update eBooks quarterly and Apps daily based on user feedback. Please tap flag to
report any questions that need improvement.
Question 512: A 65-year-old woman comes to the emergency department because of acute
onset shortness of breath and coughing up blood for the past 5 hours. She has hypertension and
type 2 diabetes mellitus. She also has a history of developing thrombocytopenia after using an
anticoagulant for deep vein thrombosis. On examination, she appears anxious, her pulse is 112
beats/min, and respirations are 24 breaths/min, blood pressure 110/60 mmHg. She has normal
breath sounds with no wheezing on auscultation of the chest. Her chest x-ray shows no
abnormalities, but her ventilation-perfusion scan shows a small defect in the right middle lobe.
The drug most likely to be used for the patient acts on which of the following substances?

Choices:
1. Glycoprotein IIb/IIIa
2. Factor VII
3. Antithrombin
4. Factor Xa
Answer: 4 - Factor Xa
Explanations:
Novel oral anticoagulants (NAOCs) like rivaroxaban are direct factor Xa inhibitors and can
be used as anticoagulants for acute pulmonary embolism and also continued for prevention
of recurrent venous thromboembolic events.
This is an appropriate choice of anticoagulant in this patient with a history of heparin-
induced thrombocytopenia where heparin is contraindicated.
NOACs are becoming a safe and effective choice of oral anticoagulant in the management
of hemodynamically stable patients with acute pulmonary embolism without shock or
hypotension. NOACs like rivaroxaban offer a convenient and cost-effective single-drug
therapeutic approach. The ease of not requiring frequent clinical appointments for lab
monitoring and route of administration makes management more convenient and enables
earlier discharge and outpatient treatment in low-risk patients.
Warfarin is not the drug of choice for acute pulmonary embolism because the onset of
action is typically 24 - 72 hours, and peak therapeutic effect is only seen 5 to 7 days after
initiation. Heparin acts by indirectly inactivating thrombin and activated factor X (factor
Xa) through binding with antithrombin to enhance its activity and is the initial treatment of
choice for most patients with acute pulmonary embolism but given the history of heparin-
induced thrombocytopenia in this patient, this drug is contraindicated. Antiplatelet drugs
like abciximab act as glycoprotein IIb/IIIa inhibitor preventing platelet aggregation and are
used during percutaneous coronary intervention.

Go to the next page if you knew the correct answer, or click the link image(s) below to further
research the concepts in this question (if desired).

Research Concepts:
Rivaroxaban

We update eBooks quarterly and Apps daily based on user feedback. Please tap flag to
report any questions that need improvement.
Question 513: A 35-year-old female with a past medical history of asthma presented to the
emergency department complaining of severe shortness of breath. Patient states since the past
week she wasn't feeling well. Seven days ago she first started to have a runny nose and dry
cough progressively the symptoms have worsened. She started having shortness of breath which
progressively got to the point she was not able to talk 2 words without getting short of breath.
She denied any fevers, chills but states she has racing of the heart. She denied any sick contacts.
The patient uses an albuterol inhaler as needed for asthma but did not help with her symptoms
this time. In the emergency department, her blood pressure was 140/90, heart rate of 140, pulse
ox showed 95% on room air. On physical exam, the patient was tachypneic with a bilateral
expiratory wheeze, prolonged expiratory phase, tachycardic with an irregular rhythm. A chest x-
ray did not show any significant cardio-thoracic process. EKG showed finding suggestive of
multifocal atrial tachycardia. The patient was started on breathing treatments with Lev-albuterol
nebulization treatments, IV glucocorticoids were initiated. After a few minutes of treatment the
patient's respiratory rate is 23, a heart rate of 135, pulse-ox showed 95% on room air appears to
be in moderate discomfort due to her breathing but she denied any tiredness. An arterial blood
gas sample was obtained which showed respiratory alkalosis. what is the next step of
management?

Choices:
1. Place the patient on Non-invasive positive pressure ventilator support
2. Start patient of albuterol-ipratropium nebulization treatments
3. Start intravenous diltiazem
4. Start intravenous magnesium
Answer: 4 - Start intravenous magnesium
Explanations:
Non-invasive positive pressure ventilation is not indicated yet. The patient only received
initial treatment for asthma exacerbation and needs some time to optimize treatment for her
asthma exacerbation. The patient has moderate discomfort due to her breathing but did not
appear tired. Her Arterial blood sample showed respiratory alkalosis. The concern should
arise in this patient if her PCO2 normalizes or if it gets higher than 45 which indicates
respiratory muscle fatigue which can lead to respiratory failure. To prevent respiratory
muscle fatigue non-invasive positive pressure ventilation can be used but is premature in
this case.
In the above patient inhaled albuterol and ipratropium can be started but it increases the
patient heart rate even higher. The patient is already on levalbuterol inhalation at this time
albuterol/ipratropium inhalation is not indicated.
Heart rate control with intravenous diltiazem is not indicated at this time. Asthma
exacerbation is indicated before heart rate control.
Intravenous magnesium is indicated at this time for the treatment of asthma exacerbation.
Studies have shown Magnesium replacement to help to reduce atrial and ventricular ectopic
activity.

Go to the next page if you knew the correct answer, or click the link image(s) below to further
research the concepts in this question (if desired).

Research Concepts:
Multifocal Atrial Tachycardia

We update eBooks quarterly and Apps daily based on user feedback. Please tap flag to
report any questions that need improvement.
Question 514: A 19-year-old previously healthy camp counselor was admitted to the
hospital with two days history of headache and low-grade fever followed by unresponsiveness
and generalized seizure. He underwent a lumbar puncture, which showed an increase in cell
count and proteins, but it was negative for gram stain and bacterial culture. He was treated with
acyclovir, vancomycin, and ceftriaxone. In spite of therapy and supportive management, he died
on the second day in the intensive care unit. Autopsy revealed suppurative meningoencephalitis
and necrotic tissue, which showed trophozoites of protozoa. What is the most likely route for this
patient to have acquired this form of meningoencephalitis?

Choices:
1. Diving or swimming in contaminated water
2. Intravenous drug abuse
3. Using human excrement as vegetable fertilizer
4. Eating raw fish or seafood
Answer: 1 - Diving or swimming in contaminated water
Explanations:
This is a case of amebic meningoencephalitis. Contaminated water can cause infection with
Acanthamoeba or Naegleria which may develop into meningoencephalitis.
The organism is thought to acquire access to the CNS through the cribriform plate.
Human excrement also known as night soil can be a source of parasitic worm infections,
such as ascariasis because the helminth eggs are in feces and can thus be transmitted from
one infected person to another person (fecal-oral transmission of disease).
Major types of food poisoning that can result from eating raw or undercooked fish and
shellfish include Salmonella and Vibrio vulnificus.

Go to the next page if you knew the correct answer, or click the link image(s) below to further
research the concepts in this question (if desired).

Research Concepts:
Amebic Meningoencephalitis

We update eBooks quarterly and Apps daily based on user feedback. Please tap flag to
report any questions that need improvement.
Question 515: A 17-year-old male driver is brought in by emergency medical services after
a rollover motor vehicle collision. He is hemodynamically stable with a Glasgow coma scale of
15 and denies loss of consciousness. He has a significant crush injury to his right leg, and his lab
work is notable for acute kidney injury, hyperkalemia, hypocalcemia, and hyperphosphatemia.
Which of the following is the most potent treatment for his hyperphosphatemia?

Choices:
1. Calcium acetate
2. Sevelamer
3. Aluminum hydroxide
4. Continuous dialysis
Answer: 4 - Continuous dialysis
Explanations:
While aluminum hydroxide is a potent hypophosphatemic, it is not as effective as
continuous dialysis.
Aluminum hydroxide should be used with caution in the setting of acute kidney injury.
Intermittent dialysis is not as potent at phosphate removal as continuous dialysis.
The gold standard for phosphate clearance is continuous hemodialysis.

Go to the next page if you knew the correct answer, or click the link image(s) below to further
research the concepts in this question (if desired).

Research Concepts:
Aluminum Hydroxide

We update eBooks quarterly and Apps daily based on user feedback. Please tap flag to
report any questions that need improvement.
Question 516: Which diagnostic test should be ordered in the case of a 45-year-old male
who works as a painter and is found obtunded at work? The head CT, urine toxicology, and
ethanol are all negative. Lab values reveal Na 136 mEq/L, K 5.4 mEq/L, Cl 111 mEq/L, HCO3
11 mEq/L, BUN 16 mg/dL, Creatinine 1.4 mg/dL, Glucose 90 mg/dL, ABG: PO2 88, PCO2 25,
and pH 7.18.

Choices:
1. Urine ketones
2. Serum sodium
3. Echocardiogram
4. Plasma osmolality
Answer: 4 - Plasma osmolality
Explanations:
The patient has a compensated high anion gap metabolic acidosis and with signs of
intoxication.
The next step is to measure the plasma osmolality and compare it to the estimated
osmolality which is 287 (2Na + (BUN/2.8) + (glucose/18) + (blood ethanol/4.6)) to
determine the osmolar gap (measured osmolality-calculated osmolality=osmolar gap). This,
in normal cases, is less than 10. A higher gap is due to low molecular weight molecules
such as ethylene glycol, methanol, acetone, or ketones that are not accounted for in the
equation.
Methanol is used in paint thinners and is a likely culprit in this case.
Ketoacidosis is a cause of anion gap metabolic acidosis, but given the normal blood glucose
in this case, is unlikely.

Go to the next page if you knew the correct answer, or click the link image(s) below to further
research the concepts in this question (if desired).

Research Concepts:
Methanol Toxicity

We update eBooks quarterly and Apps daily based on user feedback. Please tap flag to
report any questions that need improvement.
Question 517: A 68-year-old woman with a past medical history of alcohol use disorder
and hypertension is admitted to the hospital with a diagnosis of grade 3 subarachnoid
hemorrhage (Hunt and Hess scale). Few hours following the admission, the patient complains of
shortness of breath. A chest x-ray is notable for pulmonary edema. EKG shows for ST-segment
elevation in the anterolateral leads. There is no reciprocal ST depression. Troponin T is mildly
elevated. An echocardiogram is notable for akinesis of the left ventricle apical segments. Which
of the following is the most likely cause of the patient's current presentation?

Choices:
1. Irritation of pericardium
2. Coronary artery disease
3. Catecholamine surge
4. Alcohol use
Answer: 3 - Catecholamine surge
Explanations:
This scenario describes a case of Takotsubo cardiomyopathy following subarachnoid
hemorrhage (SAH).
Takotsubo cardiomyopathy, also known as Gebrochenes-Herz syndrome, transient apical
ballooning syndrome, apical ballooning cardiomyopathy, stress-induced cardiomyopathy,
stress cardiomyopathy, or broken-heart syndrome, is a form of non-ischemic
cardiomyopathy. It is characterized by transient regional systolic dysfunction of the left
ventricle mimicking acute myocardial infarction but with only minimal release of cardiac
enzymes.
The cause of Takotsubo cardiomyopathy is due to a catecholamine surge following
aneurysm rupture in this patient.
The most widely accepted criteria are the Mayo Clinic diagnostic criteria for the
identification of stress cardiomyopathy. From the following key features, all are required to
meet the diagnosis: 1. Transient hypokinesis, akinesis, or dyskinesis in the left ventricular
mid segments with or without apical involvement; regional wall motion abnormalities that
extend beyond. 2. A single epicardial vascular distribution; and frequently, but not always, a
stressful trigger. 3. The absence of obstructive coronary disease or angiographic evidence of
acute plaque rupture. 4. New EKG abnormalities (ST-segment elevation and/or T-wave
inversion) or modest elevation in cardiac troponin. 5. The absence of pheochromocytoma
and myocarditis.

Go to the next page if you knew the correct answer, or click the link image(s) below to further
research the concepts in this question (if desired).

Research Concepts:
Subarachnoid Hemorrhage

We update eBooks quarterly and Apps daily based on user feedback. Please tap flag to
report any questions that need improvement.
Question 518: A 68-year-old woman is brought to the emergency department by an
ambulance after a motor vehicle collision. She is unconscious, and a neurological examination is
not possible. Her past medical history is significant for hypertension, coronary artery disease,
atrial fibrillation on warfarin, and congestive heart failure. CT scan confirms the presence of a
subdural hematoma. Which of the following is the next best step in the management of this
patient?

Choices:
1. Prothrombin complex concentrate
2. Levetiracetam
3. Mannitol
4. Intravenous fluids
Answer: 1 - Prothrombin complex concentrate
Explanations:
Warfarin greatly worsens the bleeding associated with a head injury.
One of the first priorities is to reverse the effects of the patient's anticoagulation rapidly.
The best medication to immediately start to reverse the warfarin is prothrombin complex
concentrate.
Fresh frozen plasma can also be given, but prothrombin complex concentrate is preferred.

Go to the next page if you knew the correct answer, or click the link image(s) below to further
research the concepts in this question (if desired).

Research Concepts:
Geriatric Head Injury

We update eBooks quarterly and Apps daily based on user feedback. Please tap flag to
report any questions that need improvement.
Question 519: A 65-year-old male with end-stage renal disease (ESRD) on hemodialysis
complains of right-sided neck pain during his routine outpatient treatment. On examination, he is
afebrile with BP 145/90 mmHg, HR 92 bpm. A right internal jugular tunneled hemodialysis
catheter is noted. The exit site has purulent drainage and erythema that extends 4 cm cephalad
along the subcutaneous tunnel. The catheter cuff is not exposed. On palpation, there is induration
and tenderness over the subcutaneous tunnel. A blood sample drawn from the hemodialysis
circuit shows a leucocyte count of 13,000/microL with 80% neutrophils and a normal platelet
count. Which of the following is the most appropriate management strategy for this patient?

Choices:
1. Culture exit site discharge and do not remove the catheter
2. Culture exit site discharge and remove the catheter
3. Culture exit site discharge and exchange catheter over the guidewire
4. Culture blood and remove the catheter only if evidence of bacteremia
Answer: 2 - Culture exit site discharge and remove the catheter
Explanations:
The patient has evidence of both exit-site infection and tunnel infection of the tunneled
hemodialysis catheter. According to the Infectious diseases society of America (IDSA),
tunnel infection is defined as tenderness, hyperemia, or induration that extends >2 cm from
the exit site and along the subcutaneous tunnel. It may or may not be associated with
bacteremia. If there is purulent drainage, it should be collected and sent for gram staining
and culture.
The catheter should always be removed in the setting of tunnel infection.
Exit site infections can be treated with topical antibiotics in the absence of systemic signs of
infection or sepsis. However, tunnel infections require parenteral antibiotics and catheter
removal due to the higher risk of bacteremia.
Catheter exchange over a wire should be done. After the removal of the catheter, a new
catheter must be inserted at an alternate site. Blood cultures should be drawn before starting
empiric broad-spectrum antibiotics to cover both gram-positive and gram-negative
organisms. Based on culture and sensitivity results, the antibiotic regimen can be titrated. If
a catheter-related bloodstream infection (CRBSI) is also present, then the duration of
treatment will be determined by the clinical presentation, complications, microorganism
isolated, and vascular access options of the patient.

Go to the next page if you knew the correct answer, or click the link image(s) below to further
research the concepts in this question (if desired).

Research Concepts:
Central Line Associated Blood Stream Infections

We update eBooks quarterly and Apps daily based on user feedback. Please tap flag to
report any questions that need improvement.
Question 520: A 65-year-old woman has deep vein thrombosis extending from the thigh to
the common femoral vein. Overnight catheter-directed therapy with tissue plasminogen activator
is prescribed. No heparin is administered. The fibrinogen level is monitored. What is the next
step in management if bleeding occurs at the groin puncture site?

Choices:
1. Administer fresh frozen plasma
2. Administer vitamin K
3. Administer aminocaproic acid
4. Administer protamine
Answer: 3 - Administer aminocaproic acid
Explanations:
Aminocaproic acid is a derivative of lysine and an inhibitor of certain proteolytic enzymes.
Aminocaproic acid can treat excessive fibrinolysis.
Side effects of aminocaproic acid include nausea, fever, and vomiting.
Aminocaproic acid can also cause inflammation of the liver and appendix.

Go to the next page if you knew the correct answer, or click the link image(s) below to further
research the concepts in this question (if desired).

Research Concepts:
Thrombolytic Therapy

We update eBooks quarterly and Apps daily based on user feedback. Please tap flag to
report any questions that need improvement.
Question 521: A 52-year-old man presents to the clinic with a nonhealing ulcer on his left
foot for the past two weeks. He has a history of alcoholic cirrhosis, for which he received a liver
transplant two years ago. Physical examination shows a cold maculopapular lesion with an
ulcerated center or a violaceous nodular lesion. A swab from the ulcer does not show any
bacterial growth after 72 hours. Three days later, the patient develops fever and a headache. A
lumbar puncture analysis performed is shown below.
Patient findings Normal range
Fibrin webs
Appearance Clear
visible
Protein 0.7 g/L 0.1-0.4 g/L
Glucose 37 mg/dL 40-70 mg/dL
White cell
70 lymphocytes 0-10 cells
count
Gram stain Negative Negative
Which of the following tests is most likely to confirm the diagnosis in this patient?

Choices:
1. KOH preparation of the ulcer swab
2. Blood cultures
3. Serum polysaccharide antigen
4. MRI
Answer: 3 - Serum polysaccharide antigen
Explanations:
The clinical scenario describes an immunocompromised patient with possible cyptococcus
infection.
Cryptococcosis is the third most common invasive fungal infection in organ transplant
recipients, after candidiasis and aspergillosis.
Cutaneous manifestations can be protean, may mimic an abscess, present as cellulitis that
later ulcerates, blisters, and shows necrosis, or mimic panniculitis. The polysaccharide
capsule antigen is a large molecule, and assays have been developed to detect the antigen in
serum and other body fluids.
If cryptococcal polysaccharide antigen (CA) is present in cerebrospinal fluid (CSF), then
there is a good chance that the yeast is present as well. The cryptococcal polysaccharide
antigen assay is nearly 100% sensitive and 96% to 99.5% specific in serum and 96% to
100% sensitive and 93.5% to 99.8% specific in CSF.

Go to the next page if you knew the correct answer, or click the link image(s) below to further
research the concepts in this question (if desired).

Research Concepts:
Cutaneous Cryptococcus

We update eBooks quarterly and Apps daily based on user feedback. Please tap flag to
report any questions that need improvement.
Question 522: A 45-year-old man with adult respiratory distress syndrome secondary to
acute pancreatitis is admitted to the intensive care unit for septic shock. The patient is on
maximal doses of norepinephrine and vasopressin and thus was started on angiotensin II
following which his blood pressure improved, and norepinephrine and vasopressin requirements
have come down. The patient is on day two of angiotensin II therapy. Each time the angiotensin
II is weaned, the patient's blood pressure drops. According to the hospital policy, this agent
cannot be used after two days. Which of the following is the next best step in the management of
this patient?

Choices:
1. Continue using angiotensin II
2. Stop the angiotensin II
3. Consult the pharmaceutical manufacturer for advice
4. Palliative care consult
Answer: 1 - Continue using angiotensin II
Explanations:
Angiotensin II originally studied for only two days however certain patients in the ATHOS-
3 trial were very dependent on this agent and continue to be on that for seven days.
The maximum dose of this agent for 7 days Is 80 nanograms per kg per minute.
Any use beyond seven days, this agent would fall under the experimental realm.
It potentially may be unethical to stop this agent on a dependent patient in 2 days as
originally designed in ATHOS 3 trial; thus, a continuation of this agent for 7 days was
allowed.

Go to the next page if you knew the correct answer, or click the link image(s) below to further
research the concepts in this question (if desired).

Research Concepts:
Angiotensin II

We update eBooks quarterly and Apps daily based on user feedback. Please tap flag to
report any questions that need improvement.
Question 523: A 66-year-old woman isis admitted to the intensive care unit after
undergoing liver transplantation. She remains on a mechanical ventilator postoperatively. She
lives alone and has a history of alcohol use disorder and chronic liver disease. Her body mass
index is 17 kg/m2. Postoperatively, the patient has a hemoglobin of 10 g/dL, leukocyte count
6000/microL, platelet count 60000/microL, BUN 8 mg/dL, and international normalized ratio
(INR) of 4.0. Which of the following is the most suitable for prophylaxis against peptic
ulceration in this patient?

Choices:
1. Sucralfate
2. Histamine-2-receptor antagonist
3. Proton pump inhibitor
4. Aluminum hydroxide
Answer: 2 - Histamine-2-receptor antagonist
Explanations:
Ventilator bundles typically consist of a constellation of various preventative strategies to
reduce the risk of ventilator-associated complications. There is no ideal ventilator bundle.
They may differ across hospitals, regions, and countries. However, the primary preventative
strategies are mostly similar across different institutions.
Mechanically ventilated patients are at risk for gastric ulceration due to their predisposition
to aerobic Gram-negative bacterial (AGNB) colonization of the stomach. Therefore, peptic
ulcer prophylaxis may be administered as part of a ventilator bundle of strategies aimed at
preventing ventilator-associated complications.
Upper gastrointestinal (GI) bleeding is a known complication of gastric ulceration. In the
assessment of mechanically ventilated patients, it is important to identify risk factors for
clinically significant upper GI bleeding (including vitamin K deficiency, coagulopathy or
bleeding diatheses) to guide the appropriate choice of agent for ulcer prophylaxis.
In this scenario, the patient is malnourished (as evidenced by her body mass index of 17
kg/m2) and has a history of alcohol use disorder. It is likely she may have a deficiency of
vitamin K. Due to her history of chronic liver disease, she may likely have a clotting factor
deficiency as a result of impairment in synthetic hepatic function. This is correlated by her
high international normalized ratio (INR) of 4. She also has thrombocytopenia. These
aspects indicate coagulopathy. Sucralfate is often preferred for use in mechanically
ventilated patients as it protects against gastric ulceration without increasing the gastric pH.
Although histamine-2-receptor antagonists (H2-RA) have the effect of increasing gastric
pH, which is unfavorable because an increased gastric pH promotes colonization with
aerobic Gram-negative bacteria, mechanically ventilated patients with risk factors for severe
upper GI bleeding, including those with coagulopathy, should receive prophylaxis with an
H2-RA rather than sucralfate.

Go to the next page if you knew the correct answer, or click the link image(s) below to further
research the concepts in this question (if desired).

Research Concepts:
Ventilator Complications

We update eBooks quarterly and Apps daily based on user feedback. Please tap flag to
report any questions that need improvement.
Question 524: A 16-year-old female is admitted to the ICU for ventilator management after
developing respiratory failure secondary to benzodiazepine use to control agitation. She
presented to the emergency department yesterday after an intentional overdose of citalopram.
Which laboratory test confirms the diagnosis?

Choices:
1. Serum lactate level
2. Urine drug screen
3. Serum ammonia level
4. No specific test, this is a diagnosis of exclusion
Answer: 4 - No specific test, this is a diagnosis of exclusion
Explanations:
Serotonin toxicity is a clinical diagnosis initially defined by the Sternbach criteria.
Due to the nonspecific nature of the Sternbach criteria, the Hunt serotonin toxicity criteria
were developed.
Besides signs and symptoms, a history suggestive of serotonin exposure or overdose is
required to diagnose serotonin toxicity.
Diagnosis of serotonin toxicity is not only clinical but also a diagnosis of exclusion,
therefore, even though diagnosis does not rely on any specific laboratory value, thorough
testing to rule out other causes of altered mental status is essential.

Go to the next page if you knew the correct answer, or click the link image(s) below to further
research the concepts in this question (if desired).

Research Concepts:
Selective Serotonin Reuptake Inhibitor Toxicity

We update eBooks quarterly and Apps daily based on user feedback. Please tap flag to
report any questions that need improvement.
Question 525: A 36-year-old male is brought to the emergency department with
hypotension with a blood pressure of 90/60 mmHg, tachycardia with a heart rate of 122/min, and
tachypnea with a respiratory rate of 30/min. Pulse oximetry reveals a peripheral capillary oxygen
saturation of 100% percent on room air. The patient's skin appears cherry red, and he appears to
have soot in his nose and mouth area. His pupils are dilated, and he appears intoxicated. Initial
investigations showed sodium levels of 146 mEq/L, potassium levels of 5.5 mEq/L, bicarbonate
levels of 12 mEq/L, blood urea nitrogen 8 mg/dL, creatinine 1.96 mg/dL, and anion gap 32.6.
His total leucocyte count was 12.9 mg/dL, hemoglobin 13.2 mg/dL, and a platelet count of
106000/mircoliter. Arterial blood gases included a pH of 6.67, carbon dioxide 84 mmHg, PO2 of
157 mmHg, and bicarbonate levels at 9.7 mEq/L. Liver function tests revealed aspartate
transaminase levels of 151 U/L. His creatinine kinase is 2095 U/L, and ammonia is 23 U/L. The
plasma lactate level was 16.0 mmol/L. Which of the following blood report of the patient have
the highest sensitivity in diagnosing the patient?

Choices:
1. A raised anion gap metabolic acidosis
2. Plasma lactate levels of greater than 8mmol/L
3. Raised liver function tests
4. Raised total leucocyte count
Answer: 2 - Plasma lactate levels of greater than 8mmol/L
Explanations:
Cyanide poisoning presents with bradycardia, hypertension, tachypnea, cherry-red skin, and
soot around the face. Pulse oximetry can be high and falsely reassuring. Cyanide can block
aerobic metabolism, causing oxygen delivered to cells to go unused. Lactic acidosis can
result from the shift from aerobic to anaerobic metabolism.
Metabolic acidosis with a reduced arteriovenous difference suggests the diagnosis.
Plasma lactate levels greater than eight mmol/L is 94% sensitive and 70% specific for
significant cyanide toxicity.
Aggressive therapy with oxygen is needed. Cyanide antidotes include hydroxocobalamin.
Cyanocobalamin combines with cyanide and allows the kidneys to excrete it. Raised total
leukocyte count can also be in infections. Liver function tests are increased in multiple
diseases of the liver which is not specific.

Go to the next page if you knew the correct answer, or click the link image(s) below to further
research the concepts in this question (if desired).

Research Concepts:
Cyanide Toxicity

We update eBooks quarterly and Apps daily based on user feedback. Please tap flag to
report any questions that need improvement.
Question 526: A 65-year-old male with an unknown past medical history is brought to the
emergency department with shortness of breath and lethargy. The patient is found to be confused
and hypoxic, with an O2 saturation of 65%. The basic metabolic panel includes sodium 140
mEq/L, potassium 5.8 mEq/L, chloride 100 mEq/L, bicarbonate 19 mEq/L, blood urea nitrogen
65 mg/dL, creatinine 3.2 mg/dL, and blood glucose 132 mg/dL. The patient weighs 100
kilograms. The clinician decides to perform emergency intubation. What induction and paralytic
medications should be used?

Choices:
1. Rocuronium 100 mg IV followed by etomidate 30 mg IV
2. Propofol 50 mg IV followed by rocuronium 100 mg IV
3. Etomidate 30 mg IV followed by succinylcholine 100 mg IV
4. Etomidate 30 mg IV followed by rocuronium 100 mg IV
Answer: 4 - Etomidate 30 mg IV followed by rocuronium 100 mg IV
Explanations:
This patient presents with acute hypoxic respiratory failure requiring rapid sequence
intubation. The patient has an anion gap metabolic acidosis as well as an acute kidney
injury, causing hyperkalemia.
The proper dose of etomidate is 0.1-0.6 mg/kg, and the proper dose of rocuronium is 0.6-1.2
mg/kg, etomidate 30 mg IV, followed by rocuronium 100 mg IV, is the proper dosing and
medications for this patient.
Paralytic agents should always be given after induction agents to avoid paralyzing a
conscious patient.
Due to the hyperkalemia, succinylcholine is contraindicated. The proper dosing of propofol
as an induction agent is 1.5-2.5 mg/kg; therefore, 50 mg would be an inadequate dosage.

Go to the next page if you knew the correct answer, or click the link image(s) below to further
research the concepts in this question (if desired).

Research Concepts:
Tracheal Rapid Sequence Intubation

We update eBooks quarterly and Apps daily based on user feedback. Please tap flag to
report any questions that need improvement.
Question 527: A previously healthy 34-year-old male is in the intensive care unit due to a
traumatic brain injury. On physical examination, he is unarousable to painful stimuli, pupils are
unresponsive to light bilaterally, and gag reflex testing elicits no response. Diagnostic testing is
completed to confirm brain death. Most recent laboratory data include a white blood cell count of
8300/mm3 and a hemoglobin of 12.2 g/dL. He is taken to the operating room for donation of
select organs. Which of the following is the best next step for a successful organ donation?

Choices:
1. Inject organs with sirolimus to reduce immunogenicity
2. Cool organs to 4 C (39.3 F) to reduce warm ischemia damage
3. Store organs at 37 C (98.6 F) until transplanted into the host
4. Store organs at 0 C (32 F)
Answer: 2 - Cool organs to 4 C (39.3 F) to reduce warm ischemia damage
Explanations:
The most common type of injury to organs after procurement is warm ischemia damage.
This includes a rush of inflammatory markers and free radicals, which can adversely affect
the transplantation process.
In an effort to reduce warm ischemia injury, organs are rapidly cooled to 4 C (39.3 F). It is
important to note that cellular processes are not halted completely but rather delayed.
Another method to reduce warm ischemia damage is flushing with an appropriate
preservation solution. This leads to a decreased immune reaction and the formation of
oxygen-free radicals upon reperfusion.
Warm ischemia times currently tolerated are less than 30 minutes. Storge at a temperature
below 4 C can lead to protein denaturation.

Go to the next page if you knew the correct answer, or click the link image(s) below to further
research the concepts in this question (if desired).

Research Concepts:
Tissue and Organ Donation

We update eBooks quarterly and Apps daily based on user feedback. Please tap flag to
report any questions that need improvement.
Question 528: A 32-year-old patient with post-traumatic diffuse brain edema was being
monitored clinically by the placement of the intraventricular intracranial pressure (ICP)
monitoring system. Suddenly his attending nurse sees a gradual rise in the P2 wave going above
the P1 and P3 waves as visualized in the ICP waveform analysis. There is also a fall in his motor
score from M5 (localizing) to M3 (flexing.) Which of the following is the appropriate immediate
next step in the management of the patient?

Choices:
1. Prepare for decompressive hemicraniectomy
2. Take the patient for an emergency repeat CT scan
3. Paralyze and sedate the patient
4. Lower down the external ventricular drain (EVD) below the level of the tragus
Answer: 4 - Lower down the external ventricular drain (EVD) below the level of the tragus
Explanations:
The clinical scenario, as depicted in the ICP waveform analysis, is typical of rising ICP
pressure, and therefore, there is an urgent need for intervention to reduce the same.
The intraventricular ICP monitoring system in addition to being the gold standard for
measuring the same, allows for the immediate release of CSF, thereby counteracting any
rise in ICP.
So the best immediate plan of management would be to allow for egress of CSF by
lowering the transducer and the EVD bag below the level of the tragus (external auditory
canal).
After combating this sudden rise in ICP only, we can ask for other interventions like
repeating CT scans and preparing for damage control surgery.

Go to the next page if you knew the correct answer, or click the link image(s) below to further
research the concepts in this question (if desired).

Research Concepts:
Intracranial Pressure Monitoring

We update eBooks quarterly and Apps daily based on user feedback. Please tap flag to
report any questions that need improvement.
Question 529: A 55-year-old man with a history of hypertension and diabetes presents to
the emergency department after 1 day of fever and rigors. The patient's vital signs are a
temperature of 102.4 F, a heart rate of 130/min, respiratory rate 25/min, blood pressure 90/50
mmHg, and SpO2 90% on room air. The patient's physical exam reveals an ill-appearing,
diaphoretic, and dyspneic male with signs of poor peripheral perfusion. The patient is presumed
to be in septic shock, and supplemental oxygen is administered, a peripheral IV is secured,
antibiotics are given, and blood drawn for lab tests. An arterial blood gas is performed, while the
patient is on room air, with the following results pH of 7.3, paCO2 of 30 mmHg, paO2 of 60
mmHg, HCO3 of 12 mEq/L and SaO2 of 89%. Which of the following is the most accurate
interpretation of these results?

Choices:
1. Primary respiratory acidosis with compensation
2. Primary metabolic acidosis with compensation
3. Primary respiratory acidosis without compensation
4. Primary metabolic acidosis without compensation
Answer: 2 - Primary metabolic acidosis with compensation
Explanations:
Using 7.40 as a cutoff point, the pH is not in the normal range, in which case it is 7.40, so
acidosis is present.
The PaCO2 is decreased, indicating a respiratory alkalosis, and the HCO3 is decreased,
indicating a metabolic acidosis.
The value consistent with the pH is the HCO3. Therefore, this is a primary metabolic
acidosis.
The acid-base that is inconsistent with the pH is the paCO2, as it is decreased, indicating a
respiratory alkalosis. Therefore, there is compensation in response to the primary disorder.
However, it is ineffective as the pH is still low despite the compensatory response.

Go to the next page if you knew the correct answer, or click the link image(s) below to further
research the concepts in this question (if desired).

Research Concepts:
Arterial Blood Gas

We update eBooks quarterly and Apps daily based on user feedback. Please tap flag to
report any questions that need improvement.
Question 530: A 66-year-old man is brought to the emergency department after a motor
vehicle accident. He complains of severe left flank abdominal pain. His vital signs reveal blood
pressure 90/60 mmHg, heart rate 110/min, and respiratory rate 22/min. He has a past medical
history of hypertension, colon cancer diagnosed five months ago, and pulmonary embolism four
months ago. He is currently on losartan 50 mg daily and rivaroxaban 15 mg/twice daily. He is
compliant with his medication and took it five hours ago before the car accident. The surgeon on
call reviews the CT scan and decides to take the patient to the operating room immediately for
exploratory laparotomy. What is the most appropriate management strategy to reverse
anticoagulation in this patient?

Choices:
1. Vitamin K
2. Idarucizumab
3. Prothrombin complex concentrate
4. Andexanet-alfa
Answer: 4 - Andexanet-alfa
Explanations:
The FDA approved andexanet-alfa as a specific agent for emergency reversal of patients on
rivaroxaban and apixaban anticoagulation therapy. The dose of andexanet-alfa varies
depending on the milligrams of rivaroxaban or apixaban that the patient is receiving daily
and the time since the last dose.
In this case, the previous dose of the patient was 5 hours ago, and he is taking 15 mg of
rivaroxaban. By fabricant recommendations, the andexanet dose will be 800 mg infusion.
Andexanet-alfa is a recombinant factor Xa, and the onset of action is evident two minutes
after starting the infusion.
For vitamin K peak of its pharmacological action is around 12-14 hours after
administration. As this case is spleen trauma requiring emergent damage control surgery,
vitamin K will not reverse the anticoagulation of the patient in the needed time.
Additionally, vitamin K is widely used as a specific reversal agent for patients on warfarin
with higher INR that does not have significant bleeding. The FDA approved idarucizumab
as a specific antidote for dabigatran. This therapy would work in this case. Prothrombin
complex concentrate was the first line of treatment for patients on DOACs that required
reversal of anticoagulation for many years. However, per recommendations of ASRA
guidelines, this first option should be andexanet-alfa as it is the specific antidote.

Go to the next page if you knew the correct answer, or click the link image(s) below to further
research the concepts in this question (if desired).

Research Concepts:
Perioperative Anticoagulation Management

We update eBooks quarterly and Apps daily based on user feedback. Please tap flag to
report any questions that need improvement.
Question 531: A 67-year-old male with a recent diagnosis of stage 3 non-small cell
carcinoma presents to the emergency department with left leg swelling for the past 3 days.
Venous duplex ultrasound is performed and the patient is found to have a left lower limb deep
venous thrombosis. What is the next step regarding the treatment of this DVT?

Choices:
1. Low molecular weight heparin
2. Rivaroxaban
3. Intravenous unfractionated heparin
4. IVC filter
Answer: 1 - Low molecular weight heparin
Explanations:
Patients presenting with VTE and malignancy are ideally treated with lower molecular
weight heparin for immediate anticoagulation.
Lower extremity DVT is the most common type of DVT in patients presenting with
malignancy. Patients with no contraindications for LMW and no renal insufficiency are
started on LMW heparin for immediate anticoagulation.
Direct oral anticoagulants are also preferred by some experts, rivaroxaban or apixaban, as
an alternative.
There is not enough data present for the use of DOACs in a setting where immediate
anticoagulation is required. However, some experts use it. UFH can be used in patients with
renal insufficiency.

Go to the next page if you knew the correct answer, or click the link image(s) below to further
research the concepts in this question (if desired).

Research Concepts:
Hypercoagulability

We update eBooks quarterly and Apps daily based on user feedback. Please tap flag to
report any questions that need improvement.
Question 532: A 40-year-old female farmer with a history of hyperlipidemia for the past
five years presents to the hospital with a chronic cough for the last three months not suppressed
by over-the-counter-cough suppressants. She has not smoked in her life, but her husband has
been smoking for the past 25 years. She finds it difficult to work on her farm as she feels tired
and becomes dyspneic on mild exertion. She has an increase in weight gain and bilateral pedal
edema. On examination, there are no abnormal lung sounds. There is bilateral 2+ pedal edema,
elevated JVP, and positive hepatojugular reflux. On cardiac auscultation, S3 is heard on the right
lower sternal border. On echocardiography, there is mild tricuspid regurgitation and right
ventricular hypertrophy. The left side of the heart shows normal findings. Cardiac catheterization
reveals pressures of 28 mmHg in the pulmonary artery and 10 mmHg pulmonary capillary wedge
pressure (PCWP). She is started on appropriate medications. What is the most likely mechanism
of action of the preferred treatment?

Choices:
1. Blockade of cellular calcium channels
2. Blocking G-protein coupled cell surface receptors
3. Blockade of cGMP signaling pathway
4. Blockade of Phosphodiesterase-5 enzyme
Answer: 2 - Blocking G-protein coupled cell surface receptors
Explanations:
For a patient with NYHA class, 3 or 4 and diagnostic findings implying pulmonary
hypertension is initiated on diuretics, and refractory cases are started on endothelin receptor
blockers (ERB). Bosentan is a non-selective blocker of both endothelin A (ETA) and
endothelin B (ETB) receptors. Sitaxentan blocks only ETA and is a selective blocker.
Bosentan blocks endothelin receptors of which ETA is of greater affluence in its activity
compared to ETB. Both are G-protein coupled cell surface receptors found on various parts
of the body. ETA and ETB, both are present in the lungs, where ETA is predominantly on
vascular smooth muscle cells. ET-1 is seen to be elevated in significant levels among
females with pulmonary artery hypertension. The blockade reduces ET-1 activity.
In the pathogenesis of pulmonary hypertension, endothelial dysfunction is a reason for the
release of excessive ET-1 in circulation that interacts with the vasodilatory function by
nitric oxide (NO), furthermore, it activates the increased release of Angiotensin-II, tilting
the balance towards vasoconstriction. The blockade restabilizes this balance.
Calcium channel blockers block the channels located on the surface of the cell surface. This
leads to a decline in the intracellular calcium levels due to reducing the movement of
calcium from extra to intracellular spaces. This acts to cause vasodilation and reduced
cardiac contractility. It is often used as a first-line drug for systemic hypertension. cGMP
pathway is utilized by nitric oxide and atrial natriuretic peptide, both of which are not to be
blocked for the treatment of pulmonary artery hypertension. Phosphodiesterase-5 inhibitors
are good alternatives to ERBs. Females tend to have elevated ET-1 levels as a significant
association of pulmonary hypertension. Recent studies show that females started on ERBs
have better outcomes, while males are better when initiated on PDE-5 inhibitors.

Go to the next page if you knew the correct answer, or click the link image(s) below to further
research the concepts in this question (if desired).

Research Concepts:
Bosentan

We update eBooks quarterly and Apps daily based on user feedback. Please tap flag to
report any questions that need improvement.
Question 533: A 65-year-old man presents due to the weeping of his legs. He has a past
medical history of obesity, non-alcoholic steatohepatitis, and hypertension. His vital signs show
oxygen saturation 98% on room air, respiratory rate 26 per minute, heart rate 115 bpm, blood
pressure 80/60 mmHg, and temperature 98 F. On abdominal ultrasound, there is mild to
moderate ascites with evidence of liver cirrhosis. An echocardiogram shows a hyperdynamic
heart, concentric left ventricular thickening, grade 1 diastolic dysfunction, and an ejection
fraction of 60-65%. Laboratory investigations rule out infectious and renal pathologies. Which of
the following findings would be expected given the most likely cause of this patient's
hypotension?

Choices:
1. Increased procalcitonin
2. Decreased stroke volume
3. Low left ventricular end-diastolic pressure
4. Low systemic vascular resistance
Answer: 4 - Low systemic vascular resistance
Explanations:
High output cardiac failure is characterized by an increase in cardiac output with a low
systemic vascular resistance; however, even in this high output state, it is still insufficient
for the body's demand, leading to clinical heart failure.
Low systemic vascular resistance can be due to several different mechanisms depending on
the underlying etiology.
To compensate, the stroke volume and heart rate rise to increase cardiac output.
If high output cardiac failure continues to progress, it can lead to cardiomyopathy, either
secondary to direct cardiac effects of the underlying disease or by tachycardia-mediated
cardiomyopathy.

Go to the next page if you knew the correct answer, or click the link image(s) below to further
research the concepts in this question (if desired).

Research Concepts:
High-Output Cardiac Failure

We update eBooks quarterly and Apps daily based on user feedback. Please tap flag to
report any questions that need improvement.
Question 534: A 16-year-old female, who had been diagnosed to have chronic otitis media
in her right ear for the past one year, presents to the outpatient department with the complaint of
right-sided headache for the past five days. She had two episodes of vomiting today. On
examination, there is no neurological deficit. She undergoes a magnetic resonance imaging of the
brain, which shows an area of restricted diffusion in the right temporoparietal region (T1
hypointense and T2 hyperintense) having perilesional edema and a midline shift of 3 mm. The
periphery of the lesion is not enhancing on contrast administration. If she undergoes a surgical
biopsy of this lesion, what could be the probable histological finding?

Choices:
1. Perivascular infiltrates
2. Reticular matrix
3. Neovascularity
4. Collagen capsule
Answer: 2 - Reticular matrix
Explanations:
The patient is having the right temporoparietal brain abscess in the late cerebritis stage (4-9
days).
Reticular matrix (collagen precursor) is characteristic of this stage.
The imaging findings (restricted diffusion with no peripheral contrast enhancement) show a
lack of capsule formation.
Perivascular infiltrates suggest early cerebritis (days 1-3), neovascularity suggests early
capsule formation (days 10-13), and collagen capsule suggests late capsule stage (>14
days).

Go to the next page if you knew the correct answer, or click the link image(s) below to further
research the concepts in this question (if desired).

Research Concepts:
Brain Abscess

We update eBooks quarterly and Apps daily based on user feedback. Please tap flag to
report any questions that need improvement.
Question 535: An 85-year-old woman with congestive heart failure has been admitted to
the hospital. She lives independently but as of last few weeks, the family reports inability to
bathe herself, needing help with continence, skin break down, several falls, not communicating
much, keeping to herself, and confusion. Her frailty score is calculated to be high. For which of
the following dverse outcomes during inpatient hospitalization does she have the highest risk?

Choices:
1. Weight gain
2. Delirium
3. Depression
4. Hypertension
Answer: 2 - Delirium
Explanations:
Frailty is a risk factor for several adverse outcomes during hospital stays, including
functional decline, falls, pressure ulcers, and delirium.
Frailty is also associated with a higher risk of mortality during and after a hospital stay,
admissions to nursing homes.
It places a heavy and increasing burden on health and aged care systems.
Frailty is the result of cumulative cellular damage from diverse etiologies over the life of the
individual.

Go to the next page if you knew the correct answer, or click the link image(s) below to further
research the concepts in this question (if desired).

Research Concepts:
Clinical Frailty Scale

We update eBooks quarterly and Apps daily based on user feedback. Please tap flag to
report any questions that need improvement.
Question 536: A 65-year old woman is brought to the healthcare provider. She was taking
warfarin for deep vein thrombosis when she developed neurological complaints. CT brain shows
right basal ganglia hematoma. She is admitted to the intensive care unit where her INR is 3. The
patient is given vitamin K. She has a history of a previous blood transfusion reaction. What other
treatment will be ideal to reverse the coagulopathy?

Choices:
1. Fresh frozen plasma (FFP)
2. Prothrombin complex concentrates (PPCs)
3. Platelet concentrates
4. Recombinant activated factor VII (rFVIIa)
Answer: 2 - Prothrombin complex concentrates (PPCs)
Explanations:
Prothrombin complex concentrates (PPCs) have fewer complications than fresh frozen
plasma (FFP).
PCCs do not require cross-matching and can be reconstituted and administered rapidly.
PCCs contains clotting factors II, VII, IX, and X and can rapidly normalize INR.
rFVIIa does not replace all clotting factors.

Go to the next page if you knew the correct answer, or click the link image(s) below to further
research the concepts in this question (if desired).

Research Concepts:
Hemorrhagic Stroke

We update eBooks quarterly and Apps daily based on user feedback. Please tap flag to
report any questions that need improvement.
Question 537: A 42-year-old man comes to the clinic complaining of fatigue. He has a
history of hypertension, hyperlipidemia, and chronic pancreatitis. He denies fever, dizziness, but
reports abdominal pain, vomiting blood twice in the last week, and dark stool. Medications are
hydrochlorothiazide and atorvastatin. On physical exam, the patient is afebrile, blood pressure is
110/75 mmHg, pulse rate is 110/min, and respiratory rate is 20/min. No blood is visible on rectal
exam, but fecal occult blood is positive. Upper gastrointestinal bleeding is suspected, and the
patient is sent for upper gastrointestinal endoscopy. On endoscopy, gastric varices are visible in
the absence of esophageal varices. What is the most common cause of this endoscopic
presentation?

Choices:
1. Cirrhosis of liver
2. Splenic vein thrombosis
3. Portal vein thrombosis
4. Pancreatitis
Answer: 2 - Splenic vein thrombosis
Explanations:
Patients with splenic vein thrombosis commonly present with gastric varices in the absence
of esophageal varices.
They can also have gastric varices that are greater than esophageal varices.
Patients with splenic vein thrombosis can be asymptomatic or can present with a variety of
symptoms.
These symptoms include abdominal pain, variceal bleeding, splenomegaly, and
thrombocytopenia.

Go to the next page if you knew the correct answer, or click the link image(s) below to further
research the concepts in this question (if desired).

Research Concepts:
Splanchnic Venous Thrombosis

We update eBooks quarterly and Apps daily based on user feedback. Please tap flag to
report any questions that need improvement.
Question 538: A 47-year-old male automobile mechanic is hospitalized after severe lead
exposure. He is in the ICU intubated for airway protection and receiving IV chelation therapy.
He has not made significant urine output for the past six hours. On evaluation, the patient is
intubated and sedated. His skin and mucous membranes are pale and dry. He has a heart rate of
104/min but otherwise has normal vital signs. Which of the following is the next best step to help
eliminate lead in this patient?

Choices:
1. Stop IV chelation therapy immediately
2. Administer IV crystalloid fluid
3. Begin renal replacement therapy
4. Administer IV loop diuretic
Answer: 2 - Administer IV crystalloid fluid
Explanations:
Lead is primarily eliminated through the urine.
Most patients who present with acute lead poisoning or encephalopathy will be dehydrated
from poor intake and nausea, vomiting, or diarrhea.
Adequate IV fluid resuscitation is the first-line treatment to promote sufficient urine output
and consequently lead to elimination from the body.
IV loop diuretics or renal replacement therapy may be considered as further supportive
measures in patients who have been adequately resuscitated. Chelation therapy should not
be stopped at least until the patient's symptoms improve.

Go to the next page if you knew the correct answer, or click the link image(s) below to further
research the concepts in this question (if desired).

Research Concepts:
Lead Encephalopathy

We update eBooks quarterly and Apps daily based on user feedback. Please tap flag to
report any questions that need improvement.
Question 539: An 80-year old man with a history of hypertension on metoprolol presents to
the emergency department via emergency medical services after being in a motor vehicle
accident where he was the unrestrained driver who struck a tree head-on. He is currently awake
with mild confusion and a Glasgow coma scale of 14 (baseline is 15) with a traumatic contusion
to the anterior chest wall and upper abdomen from the steering wheel. He complains of severe
abdominal and left shoulder pain. Vital signs reveal pulse 80/minute, respiratory rate 22/minute,
blood pressure 100/70 mmHg, and pulse oximetry 92% on room air. A focused assessment with
sonography for trauma is performed, which is positive for fluid in the left upper quadrant. A
chest x-ray is negative. He undergoes computed tomography of the head, neck, chest, abdomen,
and pelvis, which reveals a splenic laceration with small hemoperitoneum but no active
extravasation. Which of the following is the next best step in the management of this patient?

Choices:
1. Normal saline intravenous at maintenance rate
2. STAT one-liter intravenous bolus of normal saline followed by blood transfusion
3. STAT two-liter intravenous bolus of normal saline
4. Emergent intubation and resuscitation with intravenous fluid and blood
Answer: 2 - STAT one-liter intravenous bolus of normal saline followed by blood
transfusion

Explanations:
There are many anatomic and physiologic changes associated with normal aging, which
need to be understood to best diagnose and treat geriatric trauma patients. As we age, all our
organ systems deteriorate with time and lose their underlying ability to function as they
once optimally had at a younger age. This leads to significant considerations that must be
undertaken when taking care of the geriatric trauma patient. Many elderly patients also
present with a multitude of medical comorbidities that adversely affect their physiologic
response and may alter their clinical presentation. Polypharmacy is also very common in the
elderly and has a variety of effects on the cardiovascular system in this patient population in
which beta-blockade, calcium-channel blockers, and cardiac glycosides are common, which
lead to negative inotropic, dromotropic and chronotropic effects. Under normal
circumstances, this is the desired effect of the medication; however, with an acute insult,
these medications prevent the host from amounting to a normal physiologic response to
compensate and maintain homeostasis.
Shock, defined as global tissue hypoperfusion, is often clinically defined as a systolic blood
pressure less than 90 mmHg. This definition is often incomplete for many patients,
especially those who are geriatric trauma patients over the age of 65. Current literature
supports a systolic blood pressure of 110 mmHg to be a better benchmark, which should be
used for this purpose to identify occult shock. Furthermore, certain indices have been
utilized to better detect occult shock as opposed to using one discrete variable such as heart
rate, blood pressure, or urine output as they are all insensitive. Utilization of the shock index
has been found to be a more sensitive indicator. It is calculated as the quotient of the
systolic blood pressure by the heart rate (shock index = heart rate/systolic blood pressure).
A shock index of 0.5-0.7 is considered normal, but when it exceeds 0.7, the patient is in
shock. Further iterations of this index have also been proposed, which are felt to be more
sensitive such as the respiratory adjusted shock index or RASI. RASI is calculated by
multiplying the shock index by (respiratory rate/10). When the RASI is greater than 1.3, it
suggests that the patient is in occult shock. This patient's shock index is 0.8, and RASI 1.76
is which both indicate that the patient, even though has what would be deemed normal vital
signs, is in occult shock. As previously mentioned, shock can be insidious in the elderly
geriatric trauma population and should be readily sought.
A general rule of thumb is that a patient who presents with an injury and hypotension
presumably has traumatic hemorrhagic shock until proven otherwise. Adult patients can
only hemorrhage into five spaces enough volume to cause hemorrhagic shock to be present,
and each of the following is examined during the primary and secondary resuscitation:
chest, abdomen and pelvis, retroperitoneum, long bones, and field or floor. If this triage is
negative, then additional etiologies of shock must be considered. This patient presents with
left upper quadrant free fluid on ultrasound as well as computed tomography evidence of a
splenic injury. Left shoulder pain occurs from diaphragmatic irritation by blood, and this is
known as Kerr’s sign.
Current advanced trauma life support teaching is that an attempt should be made to stop all
hemorrhage as able. For cases in which the hemorrhage is considered non-compressible
such as, in the chest, abdomen, or retroperitoneum, radiologic or surgical approaches are
often warranted. Patients who are in hemorrhagic shock should be resuscitated with as little
crystalloid as possible, preferably normal saline or Ringer’s lactate up to 1 liter, and then
blood products or whole blood should be utilized as better outcomes have been obtained.
Permissive hypotension, a concept where resuscitation is withheld or limited to maintain a
radial pulse and mentation and not try to achieve physiologically normal systolic blood
pressures, is often practiced in the young trauma patient. The thought is that by keeping
pressures low, the body won’t “pop the clot,” leading to worsening of the hemorrhage. This
practice however should not be utilized in patients with brain injuries or in the geriatric
population as the outcomes have been found to be more dismal. Trauma patients who are in
respiratory failure and are in hemorrhagic shock are at risk for hemodynamic collapse if not
first resuscitated and then intubated as the induction medications and positive pressure
ventilation lead to vasodilation with hypotension and increased intrathoracic pressure with a
decrease in preload respectively. Consideration to change the standard airway, breathing,
circulation (ABC) approach for the patient who presents with hemorrhagic shock to
circulation, airway, and breathing (CAB) would be appropriate to prevent cardiovascular
collapse.

Go to the next page if you knew the correct answer, or click the link image(s) below to further
research the concepts in this question (if desired).

Research Concepts:
Geriatric Trauma

We update eBooks quarterly and Apps daily based on user feedback. Please tap flag to
report any questions that need improvement.
Question 540: A 66-year-old man is admitted to the hospital for acute pancreatitis. He is
prescribed subcutaneous heparin for DVT prophylaxis. 5 days later, his platelet count drops from
a baseline of 184000/microL at presentation to 63000/microL. The team sends a platelet factor 4
(PF4) ELISA and serotonin release assay (SRA), discontinues heparin, and starts the patient on
argatroban infusion. The SRA eventually comes back positive. The patient does not develop any
thrombosis, his platelet counts recover, he is ready for discharge, and argatroban is stopped.
Which of the following is the most appropriate anticoagulation regimen on discharge for this
patient?

Choices:
1. Enoxaparin for 30 days
2. Fondaparinux for 90 days
3. Rivaroxaban for 30 days
4. Warfarin for 90 days
Answer: 3 - Rivaroxaban for 30 days
Explanations:
Given that the patient has isolated HIT without any thromboembolic complications, he will
require 30 days of anticoagulation on discharge.
Rivaroxaban is the most appropriate response.
Rivaroxaban is one of the recommended agents for HIT, and the treatment duration of 30
days is appropriate.
Enoxaparin is not an appropriate choice, as it is a form of heparin and will interact with PF4
and IgG. Fondaparinux is an appropriate choice; however, the treatment duration of 90 days
is too long for isolated HIT. Warfarin is an appropriate choice; however, the treatment
duration of 90 days is too long for isolated HIT. In addition, if warfarin were to be initiated,
it would require bridging (with argatroban or fondaparinux) for at least 5 days and until 2
therapeutic INRs are obtained.

Go to the next page if you knew the correct answer, or click the link image(s) below to further
research the concepts in this question (if desired).

Research Concepts:
Heparin Induced Thrombocytopenia

We update eBooks quarterly and Apps daily based on user feedback. Please tap flag to
report any questions that need improvement.
Question 541: A 36-year-old male patient presents for the evaluation of weakness. His
symptoms started about a month ago when he noted weakness of the right hand. He has difficulty
grabbing small objects and problems with lifting the right hand above his head. Two weeks
before the consultation, he noted that his right leg was dragging. About a week ago, his live-in
partner noticed some behavioral and memory changes; he was described to be more irritable,
impulsive, and can not remember simple words for everyday objects. He had a positive human
immunodeficiency virus (HIV) screen six months ago. On examination, his vital signs are within
normal limits. His pupils are isocoric, briskly reactive to light, with a visual field cut on the right
temporal and left nasal fields. There is peripheral facial palsy on the right side. The motor exam
shows 3/5 strength on the right upper and lower extremity, 4/5 on the left upper extremity, and
5/5 on the left lower extremity. There is a 20% deficit to light touch, pain, and temperature on the
right. Babinski sign is positive bilaterally. Head MRI reveals T1 hypointense white matter
lesions in the parietooccipital region. Which of the following is the most likely cause of this
patient's symptoms?

Choices:
1. Herpes simplex virus (HSV)
2. CNS lymphoma
3. John Cunningham (JC) virus
4. Human immunodeficiency virus (HIV)
Answer: 3 - John Cunningham (JC) virus
Explanations:
Progressive multifocal leukoencephalopathy (PML) is caused by the JC virus that is normal
under control of the immune system. Immunosuppressed states lead to the activation of the
virus.
PML is a rapidly progressive neurological disorder that is fatal. The diagnosis requires a
combination of clinical, imaging, and microbiological evidence.
The disorder can present with a wide range of symptoms that are often similar to what
observes in a patient with multiple sclerosis. Visual symptoms occur in one-third of patients
and are often the initial presentation of the infection.
MRI is more sensitive in picking up white matter changes. Usual locations include the
parieto-occipital region, usually asymmetric, but may also involve the infratentorial region.

Go to the next page if you knew the correct answer, or click the link image(s) below to further
research the concepts in this question (if desired).

Research Concepts:
Progressive Multifocal Leukoencephalopathy

We update eBooks quarterly and Apps daily based on user feedback. Please tap flag to
report any questions that need improvement.
Question 542: A 44-year-old man with cerebral palsy and intellectual disability who
resides in a group home is brought to the emergency department (ED) with concerns of poor oral
intake, weight loss of 10 pounds, and low-grade fevers recorded intermittently for the past two
weeks. In the ED, the patient is found to be febrile with a temperature of 101.5 F, respiratory rate
22/min, heart rate 105/min, and blood pressure 100/70 mmHg. On examination, he has a stage 3
sacral pressure ulcer with some discharge. Labs show a white blood cell count of 27000/microL
with neutrophilia and left shift. He is given a bolus of IV fluids, and two sets of blood cultures
are drawn. He is given empiric antibiotics with IV piperacillin-tazobactam and IV vancomycin.
The next day, the microbiology lab notified the primary team that the patient is growing
Enterococcus faecium in 4 out of 4 culture bottles with sensitivities still pending. The same
evening the patient develops worsening hypoxia and is put on 5 L/min oxygen via nasal cannula.
Which of the following is the next best step in the management of this patient?

Choices:
1. Switch piperacillin-tazobactam to daptomycin
2. MRI of the sacrum
3. Echocardiogram
4. CT angiogram of the chest
Answer: 3 - Echocardiogram
Explanations:
Given the poor appetite, low-grade fevers, weight loss, tachycardia, and Enterococcus
bacteremia, there should be a concern for infective endocarditis and obtain a transthoracic
echocardiogram to look for a vegetation.
Also, in the light of new hypoxia, we are concerned about septic embolization from native
valve vegetation.
The sacral pressure ulcer seems to be an obvious source of E. faecium infection. The patient
also resides in a group home which puts him at risk for getting an infection with resistant
species like E. faecium.
Although MRI to look for sacral osteomyelitis would help determine the duration of
antibiotics, it would not be the next step in managing this patient. Although E. faecium was
shown to be 80% vancomycin-resistant and 90% ampicillin-resistant, we ordinarily wait for
final sensitivity data. In the case of endocarditis or osteomyelitis, the duration of antibiotics
is 6 to 8 weeks. Hence stopping antibiotics and switching to another agent is not warranted.
Given that the patient is found to have acute hypoxia, a pulmonary embolism is a good
differential. However, in this case, we are leaning towards a septic embolus from a
vegetation on the heart valves - likely the tricuspid valve. Hence we would choose an
echocardiogram to be the next step in the management of this patient.

Go to the next page if you knew the correct answer, or click the link image(s) below to further
research the concepts in this question (if desired).

Research Concepts:
Enterococcus Infections

We update eBooks quarterly and Apps daily based on user feedback. Please tap flag to
report any questions that need improvement.
Question 543: A 45-year-old patient with past history of type 2 diabetes mellitus presents
following a fall two days prior. She is now experiencing severe pain and tenderness over the
upper arm. On physical exam, her pain seems out of proportion to the overlying erythema and
extends proximally. Laboratory studies reveal leukocytosis. Tissue aspiration PCR is positive for
streptococcal pyrogenic exotoxin. What initial treatment for this condition, in addition to surgical
debridement and antibiotics, has been shown to reduce mortality?

Choices:
1. Leaving wound open after surgical debridement
2. Hyperbaric oxygen therapy
3. Reconstruction of the wound with a skin flap
4. Topical oxygen therapy
Answer: 2 - Hyperbaric oxygen therapy
Explanations:
In cases of necrotizing fasciitis, amputation may be required, but it is preferable to make
that decision after multiple surgical debridements, antibiotic therapy, and hyperbaric oxygen
therapy have been used first.
Hyperbaric oxygen therapy is FDA approved to treat necrotizing fasciitis. It has been shown
to reduce the significant inflammation caused by the condition. It also maintains perfusion
of the affected tissues and can exert an antimicrobial effect.
While reconstruction of the resulting wound following debridement is often performed in
cases of necrotizing fasciitis, this is a late procedure after the infection has been treated.
Initially, the wound needs to remain open for frequent irrigation, debridement, and packing
with antibiotic dressings.
Topical oxygen therapy, though theorized for decades, has not been shown to be effective as
there is an inability to create a great enough diffusion gradient for oxygen to move across
the skin.

Go to the next page if you knew the correct answer, or click the link image(s) below to further
research the concepts in this question (if desired).

Research Concepts:
Necrotizing Fasciitis

We update eBooks quarterly and Apps daily based on user feedback. Please tap flag to
report any questions that need improvement.
Question 544: A 32-year-old woman presents to the emergency department with confusion,
vomiting, and abdominal pain. The patient is ill-appearing, restless, and confused. Her husband
reports a 2-week history of abdominal pain, followed by poor oral intake, nausea, and vomiting.
On physical exam, her blood pressure is 106/70 mmHg, her pulse is 145/min, her temperature is
36.2 °C (97.1 °F), and her respiratory rate is 28/min. Scleral icterus and jaundice are noted with
moderate right upper quadrant tenderness. Laboratory testing revealed total bilirubin 5.6 mg/dL,
aspartate aminotransferase (AST) level of 3428 U/L, alanine aminotransferase (ALT) level of
7498 U/L, alkaline phosphate level of 310 U/L, international normalized ratio (INR) of 6.8, and
an arterial pH of 6.92. Her serum acetaminophen (paracetamol) level is 0 mcg/mL (normal range
10 to 30 mcg/mL). What is the next best step in the management of this patient?

Choices:
1. Start N-acetylcysteine infusion
2. Start ledipasvir/sofosbuvir treatment
3. Start pegylated interferon-alfa plus oral ribavirin treatment
4. Emergent transfer to a liver transplant center
Answer: 4 - Emergent transfer to a liver transplant center
Explanations:
Acute hepatitis can progress to acute liver failure. Signs and symptoms of acute liver failure
include hyperbilirubinemia, hepatic encephalopathy, and coagulopathy. Coagulopathy,
defined as INR greater than 1.5, indicates severely impaired liver function and is one
criterion used when considering the need for a liver transplant.
Patients with acute liver failure can have multiorgan failure. They will need fluid
resuscitation and monitoring for hypoglycemia, hypokalemia, hypomagnesemia,
hypophosphatemia, bleeding, and encephalopathy.
Patients with acute hepatitis that process to acute liver failure are best treated in liver
transplant centers. Transfers should be arranged as soon as possible when patients are
medically stable for emergent transfer.
Acute hepatitis can progress to acute liver failure. In the United Kingdom, the United
States, and Australia, drug-induced liver failure from acetaminophen (paracetamol) causes
39-50% of cases of acute liver failure. The other predominant cause is acute viral hepatitis.

Go to the next page if you knew the correct answer, or click the link image(s) below to further
research the concepts in this question (if desired).

Research Concepts:
Acute Hepatitis

We update eBooks quarterly and Apps daily based on user feedback. Please tap flag to
report any questions that need improvement.
Question 545: A 17-year-old boy is brought to the hospital after accidentally ingesting the
tablets used in cleaning jewelry in the electroplating industry one hour ago. Since the ingestion,
the patient has vomited thrice and has become agitated and irritable. Gastric lavage is performed,
and the aspirate is sent for analysis. His past medical history is significant for major depressive
disorder managed on sertraline. On examination, his Glasgow coma score is 12, heart rate is
126/min, respiratory rate is 34/min, oxygen saturation is 94% on room air, and blood pressure is
104/66 mmHg. Pupils are bilaterally reactive to light. Cherry-red discoloration of the lips and
mucous membranes is observed. Blood tests show hemoglobin of 18.2 g/dL, platelet count of
293,000/microliter, sodium of 144 mmol/L, potassium 3.9 mmol/L, magnesium of 2.6 mg/dL,
alanine aminotransferase level of 54 IU/L, and creatinine of 1.8 mg/dL. Arterial blood gas
analysis shows a severe metabolic acidosis. Intravenous fluids are started, and high flow oxygen
therapy is initiated. What is the most appropriate treatment for this patient?

Choices:
1. Sodium thiosulfate
2. Hydroxycobalamin
3. Cyanocobalamin
4. Nitrites
Answer: 2 - Hydroxycobalamin
Explanations:
This patient took tablets used in the jewelry cleaning industry. This coupled with his
presentation of altered mental status, confusion, agitation, irritability, tachypnea, metabolic
acidosis, nausea, vomiting, and cherry red mucus membranes leads to the diagnosis of
cyanide poisoning.
Labs that are pertinent to the initial evaluation in a patient with cyanide poisoning are
complete blood count, electrolytes, urinalysis, urine toxicology screen, arterial blood gas,
carboxyhemoglobin level (if in a fire), chest x-ray and EKG. Plasma lactate also may be
obtained, and a level of greater than 8 mmol/L is 94% sensitive and 70% specific for
significant cyanide toxicity in smoke inhalation victims. Narrowing of the venous-arterial
oxygen gradient. Cyanide inhibits cellular oxidative phosphorylation resulting in a marked
decrease in peripheral tissue oxygen extraction from the blood. This results in elevated
central venous oxygenation.
Hydroxycobalamin is the antidote of choice for acute cyanide poisoning, especially if the
patient has coexisting carbon monoxide poisoning. Other antidotes impair oxygen-carrying
capacity and worsen cellular hypoxia and acidosis. The standard dose is 5 grams, given
intravenously over 15 minutes. Beware that this antidote turns urine dark red; this is not due
to myoglobinuria.
Sodium thiosulfate is the second-line treatment for cyanide toxicity, and nitrites may be
used to induce methemoglobinemia if both hydroxycobalamin and sodium thiosulfate are
not available or when they are contraindicated .

Go to the next page if you knew the correct answer, or click the link image(s) below to further
research the concepts in this question (if desired).

Research Concepts:
Cyanide Toxicity

We update eBooks quarterly and Apps daily based on user feedback. Please tap flag to
report any questions that need improvement.
Question 546: A 16-year-old male presents to the emergency room with symptoms
consistent of an overdose. When asked what he had been exposed to, he says he does not
remember exactly but that he had picked up eye drops from a friend's house and ingested them in
a suicide attempt. He has symptoms of tachycardia, flushing, delirium, and upon examination,
mydriasis. What is the most appropriate medication given for this overdose?

Choices:
1. Reversible cholinesterase inhibitor
2. Selective serotonin reuptake inhibitor
3. Beta blocker
4. Antipsychotics
Answer: 1 - Reversible cholinesterase inhibitor
Explanations:
Physostigmine is a reversible cholinesterase inhibitor used to treat atropine poisoning or
overdose. It is a tertiary amine that crosses the blood-brain barrier, unlike any other
cholinesterase inhibitors, making it the drug of choice. The antimuscarinic toxidrome results
from blockade of the neurotransmitter acetylcholine at central and peripheral muscarinic
receptors. Physostigmine is a carbamate that acts by reversibly inhibiting
acetylcholinesterase.
Selective serotonin reuptake inhibitors are most commonly used to treat depression and
anxiety disorders. Although this patient has tachycardia and flushing, his symptoms of
mydriasis and delirium are not explained by depression or anxiety disorders. Mydriasis and
delirium are seen in atropine poisoning.
Beta-blockers are used to treat tachycardia but are not the treatment of choice for atropine
poisoning. Tachycardia is a symptom of atropine poisoning.
Antipsychotics are used to treat symptoms of psychosis, bipolar disorder, and at times,
delirium. This would not be a suitable treatment for atropine poisoning.

Go to the next page if you knew the correct answer, or click the link image(s) below to further
research the concepts in this question (if desired).

Research Concepts:
Physostigmine

We update eBooks quarterly and Apps daily based on user feedback. Please tap flag to
report any questions that need improvement.
Question 547: A 65-year-old man presents to the emergency department with left lower
quadrant abdominal pain of sudden onset. CT shows minimal spots of free air around the
sigmoid colon. The patient has stable vital signs. Physical examination shows a nontender
abdomen, no masses or guarding, and normal bowel sounds, with no peritoneal signs. What is
the most likely diagnosis?

Choices:
1. Diverticulitis with micro-perforation
2. Perforated appendicitis
3. Perforated peptic ulcer
4. Meckel diverticulum
Answer: 1 - Diverticulitis with micro-perforation
Explanations:
The most likely diagnosis in this age group with these findings is acute diverticulitis.
Most patients with acute diverticulitis do not require surgery.
A high fiber diet may decrease the incidence of diverticulitis.
Most acute episodes of acute diverticulitis are self-limiting.

Go to the next page if you knew the correct answer, or click the link image(s) below to further
research the concepts in this question (if desired).

Research Concepts:
Bowel Perforation

We update eBooks quarterly and Apps daily based on user feedback. Please tap flag to
report any questions that need improvement.
Question 548: A 45-year-old woman with no significant past medical history is brought to
the emergency department with complaints of fever and severe, new-onset persistent headache
for 2-3 days. She reports being vaccinated with the ChAdOx1 nCoV-19 vaccine 15 days ago.
Vital signs at admission show blood pressure of 118/70 mmHg, heart rate 65/min, and
temperature 98.3°F. Physical examination is unremarkable. Initial blood work demonstrates
hemoglobin 11.9 g/dL and platelet count 78000/microL. Liver and renal function are
unremarkable. D-Dimer is 10000 ng/mL (reference range 250 ng/mL) at admission. POCT
SARS-CoV-2 is negative. Which of the following is the next best step in the management of this
patient?

Choices:
1. Electroencephalogram (EEG)
2. CT cerebral venography
3. X-ray of the skull
4. Ibuprofen and discharge
Answer: 2 - CT cerebral venography
Explanations:
The patient in the above clinical vignette most likely has vaccine-induced immune
thrombotic thrombocytopenia (VITT) given her recent history of single-dose vaccination
against COVID-19 with ChAdOx1 nCoV-19 vaccine 15 days ago, thrombocytopenia, and
severe new-onset headache indicating possible cerebral sinus venous thrombosis (CSVT).
The next best step in the management of this patient is to confirm the presence of CSVT
with a CT cerebral venography.
Diagnosis of CSVT is an integral component to confirm the diagnosis of VITT, and early
diagnosis of VITT is important given its therapeutic implications.
X-ray of the skull and EEG would not evaluate for the presence of CSVT, and offering
supportive management would be futile in this patient.

Go to the next page if you knew the correct answer, or click the link image(s) below to further
research the concepts in this question (if desired).

Research Concepts:
Coronavirus (COVID-19) Vaccine-Induced Immune Thrombotic
Thrombocytopenia (VITT)

We update eBooks quarterly and Apps daily based on user feedback. Please tap flag to
report any questions that need improvement.
Question 549: A 65-year-old woman is admitted to the hospital with triple vessel disease
and is scheduled for urgent CABG. Her medication allergy list states a history of
thrombocytopenia with heparin, which occurred 6 months ago. Which antithrombotic strategy is
most appropriate during CABG in this patient?

Choices:
1. Bivalirudin
2. Fondaparinux
3. Obtain an SRA and if negative, use heparin
4. Heparin
Answer: 1 - Bivalirudin
Explanations:
Bivalirudin is the most appropriate choice for use during CABG in patients with a history of
HIT.
In patients with a remote history of HIT who require CABG, it is appropriate to check a
platelet serotonin-release assay (SRA), and if negative, heparin may be used only during
CABG, at the lowest dose and shortest duration needed.
Given that the patient requires urgent CABG, sending an SRA and waiting for the results
would not be appropriate. Therefore, bivalirudin is the best option.
Fondaparinux is not approved for use during CABG. Even in patients with a remote history
of HIT, heparin products may not be used unless an SRA is checked and is negative. Even if
the SRA is negative, heparin may be used only during CABG, at the lowest dose and
shortest duration needed.

Go to the next page if you knew the correct answer, or click the link image(s) below to further
research the concepts in this question (if desired).

Research Concepts:
Heparin Induced Thrombocytopenia

We update eBooks quarterly and Apps daily based on user feedback. Please tap flag to
report any questions that need improvement.
Question 550: A 37-year-old man with a past medical history of chronic pain syndrome in
the lower extremities on a pain regimen, hyperlipidemia on atorvastatin, hypertension on
lisinopril, and prior IV drug use is brought to the hospital after he was found at home confused 1
hour ago. The roommate reports that his mental status has significantly changed from 2 days ago
and had a few episodes of vomiting this morning. The physical exam is notable for diffuse
jaundice, right upper quadrant tenderness, and not being oriented to self. Initial labs reveal BUN
40 mg/dL, creatinine 2.5 mg/dL, bicarbonate 19 mEq/L, AST 15,000 IU/L, ALT 12,000 IU/L,
INR 1.5, and the toxicity panel is still pending. An analgesic with which of the following
mechanisms of action is most likely responsible for the patient's presentation?

Choices:
1. Dopamine antagonism
2. Increasing serotonin effects in the CNS
3. Inhibition of the hypothalamic heat-regulating center
4. Inhibition of the mu-receptors
Answer: 2 - Increasing serotonin effects in the CNS
Explanations:
Acetaminophen is widely used in combination with hydrocodone to treat pain.
Liver cirrhosis increases the risk of developing acetaminophen toxicity.
Analgesia effects of the acetaminophen are obtained via activation of the serotonergic
inhibitory pathways in the CNS.
Antipyretic properties are obtained by inhibition of the hypothalamic heat-regulating center.

Go to the next page if you knew the correct answer, or click the link image(s) below to further
research the concepts in this question (if desired).

Research Concepts:
Hydrocodone and Acetaminophen

We update eBooks quarterly and Apps daily based on user feedback. Please tap flag to
report any questions that need improvement.
Question 551: A 50-year-old man with no significant past medical history undergoes a
decompressive craniectomy for an acute subdural hematoma following a traumatic head injury
24 hours ago. He is now on the neurosurgical intensive care unit and remains intubated and
ventilated. His urine output is 800 mL over the last 2 hours, despite appropriate intravenous fluid
administration. Which of the following additional findings is most consistent with the patient's
most likely diagnosis?

Choices:
1. Urine specific gravity >1.005
2. Serum Na+ 135 mmol/L
3. Urine osmolality 300 mOsm/kg
4. Serum osmolality 295 mmol/kg
Answer: 3 - Urine osmolality 300 mOsm/kg
Explanations:
Neurogenic diabetes insipidus (DI) is associated with traumatic brain injury (incidence up to
35%), subarachnoid hemorrhage, and pituitary surgery.
Abnormally low urine osmolality (300 mOsm/kg) indicates the presence of diabetes
insipidus.
Development of DI following nonpituitary surgery is often associated with severe cerebral
edema. DI may be transient or permanent. It occurs as a result of the failure of ADH release
from the hypothalamic-pituitary axis. This leads to the production of large volumes of dilute
urine (as the body is unable to concentrate the urine) and inappropriate loss of water.
The patient becomes clinically dehydrated and hypernatraemic. Typically, patients may
have a urine output of more than 6 L/day.

Go to the next page if you knew the correct answer, or click the link image(s) below to further
research the concepts in this question (if desired).

Research Concepts:
Diabetes Insipidus

We update eBooks quarterly and Apps daily based on user feedback. Please tap flag to
report any questions that need improvement.
Question 552: An 85-year-old female is brought to the emergency department with severe
respiratory distress. On arrival, she is confused and agitated. Vital signs include blood pressure
100/65 mmHg, pulse 105/minute, respiratory rate 23/minute, oxygen saturation 70%, and
temperature 38.1 C (100.5 F). The patient is immediately intubated. The respiratory therapist can
see a color change on the end-tidal carbon dioxide monitor with condensation on the inside of the
endotracheal tube. Breath sounds are equal bilaterally. What is the next best step in the
management of this patient?

Choices:
1. Obtain a CT scan
2. Continue to monitor the patient
3. Obtain a chest x-ray
4. Listen for bilateral breath sounds twice per hour for 4 hours
Answer: 3 - Obtain a chest x-ray
Explanations:
A chest x-ray is necessary to confirm the placement of the endotracheal tube after an
intubation procedure.
Bilateral breath sounds heard on examination is an indication that the endotracheal tube is
placed correctly, but confirming with a chest x-ray is still necessary.
Monitoring the patient is not sufficient to ensure that the endotracheal tube is in place.
A CT of the chest is unnecessary as a chest x-ray is adequate to confirm placement.

Go to the next page if you knew the correct answer, or click the link image(s) below to further
research the concepts in this question (if desired).

Research Concepts:
Tracheal Rapid Sequence Intubation

We update eBooks quarterly and Apps daily based on user feedback. Please tap flag to
report any questions that need improvement.
Question 553: A 56-year-old woman is admitted to the ICU with a diagnosis of viral
pneumonia with respiratory failure. She required ventilatory support for 6 days and was
extubated on the 7th day as the pneumonia was resolving. On the 8th day, she develops a fever of
101°F. Chest radiograph shows significant clearance of pneumonia. Her vital signs reveal a heart
rate of 133/min, respiratory rate of 32/min, and blood pressure of 90/60 mmHg. General
examination reveals doubtful redness around the central line site. Which of the following is the
next best step in the management of this patient?

Choices:
1. Antibiotic lock
2. Remove the central venous catheter
3. CT chest
4. Acetaminophen
Answer: 2 - Remove the central venous catheter
Explanations:
The patient seems to have a catheter-related bloodstream infection (CRBSI). CRBSI is
defined as the presence of bacteremia arising from the intravenous catheter.
Prompt removal of the catheter is indicated when CRBSI is suspected. Two sets of blood
cultures should be obtained, one from the central venous line and the other from a
peripheral site. Empirical antibiotic therapy should be started immediately.
CRBSI is the commonest cause of nosocomial bacteremia.
Common pathogens causing CRBSI are Staphylococcus aureus, Pseudomonas aerogenosa,
coagulase-negative staphylococci, Escherichia coli, Klebsiella pneumoniae, and
Acinetobacter baumanii.

Go to the next page if you knew the correct answer, or click the link image(s) below to further
research the concepts in this question (if desired).

Research Concepts:
Fever In the Intensive Care Patient

We update eBooks quarterly and Apps daily based on user feedback. Please tap flag to
report any questions that need improvement.
Question 554: A 90-year-old-man from a nursing home is evaluated after multiple episodes
of ventricular tachycardia. The patient had an implantable cardioverter-defibrillator placed 1 year
back. Interrogation shows multiple shocks provided for frequent ventricular tachycardia in the
last 1 month. The patient received an amiodarone infusion after the recent episode of ventricular
tachycardia. He has nonischemic dilated cardiomyopathy, and echocardiography done 1 month
back showed left ventricular ejection fraction 20% (the same as 6 months back). The patient also
has a history of alcoholism, dementia, chronic kidney disease, diabetes mellitus, dyslipidemia,
hypothyroidism, and multiple ischemic infarcts. He has been admitted multiple times in the last 1
year for exacerbation of heart failure. On examination, he is bed-bound and not oriented to time,
place, or person. He is afebrile, with a heart rate of 86/min, blood pressure 92/50 mmHg, and
oxygen saturation of 92% on 3 L/min of oxygen via nasal cannula. The patient has elevated
jugular venous pressure. Lung examination reveals crackles on bilateral lung bases. S3 is heard
on auscultation. There is bilateral 2+ pitting edema up to the knees. Laboratory evaluation shows
serum sodium 125 mEq/L and serum creatinine 3.2 mg/dL. The patient does not tolerate beta-
blockers due to soft blood pressure and angiotensin-converting enzyme inhibitors or angiotensin
receptor blockers due to chronic kidney disease. Which of the following is the next best step in
the management of this patient?

Choices:
1. Implantation of left ventricular assist device
2. Digoxin
3. Discussion with next of kin regarding goals of care and advanced directives
4. Dopamine infusion
Answer: 3 - Discussion with next of kin regarding goals of care and advanced directives
Explanations:
Elderly patients should have discussions about end of life and goals of care early before the
health crises, preferably in ambulatory settings.
Patients with a terminal illness such as end-stage heart disease, end-stage lung disease, end-
stage kidney disease, malignancy requiring a third-line therapy, etc., have predicted a life
span of less than a year.
Such patients for whom the answer to the question - "will you be surprised if the patient
died in next 1 year?" is "no," should have a discussion about goals of care and advanced
directives. Studies have shown that early initiation of such discussion leads to a better
quality of life and satisfaction of family members.
Arrhythmias due to end-stage heart failure are difficult to control even with maximal
antiarrhythmic agents. The addition of digoxin may not provide significant benefit,
improvement in quality or quantity of life, so goals of care should be discussed prior to
adding any chronic medication. Intravenous inotropic agents are used as a bridge to the left
ventricular assist device. This patient is not a candidate for any major surgery due to
multiple co-morbidities. The patient is not a good candidate for major surgeries. Before
proceeding with any invasive intervention, a discussion should be held to know about the
patient's wishes and goals.

Go to the next page if you knew the correct answer, or click the link image(s) below to further
research the concepts in this question (if desired).

Research Concepts:
Geriatric Care Special Needs Assessment

We update eBooks quarterly and Apps daily based on user feedback. Please tap flag to
report any questions that need improvement.
Question 555: A 48-year-old woman presents to the emergency department with one day of
nausea and emesis and an inability to pass stool or flatus. Her heart rate is 121/min, blood
pressure 100/60 mmHg, respiratory rate 18/min, and SpO2 98% on room air. Her past medical
and surgical history is significant only for a Roux-en-Y gastric bypass for obesity three years
ago. Two large-bore IVs are inserted for resuscitation and a nasogastric tube is placed. Which of
the following is the next best step in the management of this patient?

Choices:
1. Exploratory laparotomy
2. CT abdomen with IV and oral contrast
3. Abdominal x-ray
4. Inpatient observation
Answer: 1 - Exploratory laparotomy
Explanations:
This patient most likely has an internal hernia secondary to her Roux-en-Y bypass with a
gastrojejunostomy.
Typical imaging modalities may be unable to detect a small amount of bowel trapped and,
therefore cannot be relied upon.
This patient should be taken directly to the OR to examine whether they have any bowel
trapped in an internal hernia and to have it reduced if possible or resected if it is infarcted.
Given that just a small amount of bowel can become trapped and create a closed-loop
obstruction a lack of significant findings on abdominal x-ray does not rule out strangulated
bowel. Admitting the patient for observation would merely increase the likelihood that
incarcerated bowel would become infarcted.

Go to the next page if you knew the correct answer, or click the link image(s) below to further
research the concepts in this question (if desired).

Research Concepts:
Gastrojejunostomy

We update eBooks quarterly and Apps daily based on user feedback. Please tap flag to
report any questions that need improvement.
Question 556: A 67-year-old man is recovering from a large intraperitoneal bleed in the
intensive care unit after receiving multiple hits from a baseball bat during a mugging. The patient
was initially somnolent and difficult to arouse. His initial blood pressure reading was 60/45
mmHg. The patient received a 1 L bolus infusion of 0.9% saline. Soon afterward, his blood
pressure rose to 75/55 mmHg. The patient is now somewhat arousable and able to speak in short
phrases. What is the next most appropriate management strategy for this patient?

Choices:
1. An additional 2 L bolus infusion of 0.9% saline to reach systolic pressures between 80-90
mmHg
2. Incremental 100 -200 mL infusions of 0.9% saline to reach systolic pressures between 80-90
mmHg
3. Incremental 500 mL infusions of 0.9% saline to reach systolic pressures between 90-100
mmHg
4. An additional 1 L bolus infusion of 0.9% saline and incremental 500 mL infusions to reach
systolic pressure between 90-100 mmHg
Answer: 2 - Incremental 100 -200 mL infusions of 0.9% saline to reach systolic pressures
between 80-90 mmHg

Explanations:
Smaller 100-200 mL infusions of fluid towards acceptable hypotensive systolic levels have
shown to have better outcomes on patient survivability.
Blunt trauma should be managed with systolic pressures in the 80 -90 mmHg range until the
source of the bleeding is repaired.
The Advanced Trauma Life Support (ATLS) recommends bolus infusions no greater than
1.5 L in order to rescue a patient from imminent danger. Afterward, smaller infusions may
be used to titrate the patient's blood pressure toward appropriate permissive hypotensive
levels.
A 2 L bolus infusion may lead to an increase in bleeding due to a reflex vasodilatory effect,
compared to smaller infusions towards acceptable hypotensive levels.

Go to the next page if you knew the correct answer, or click the link image(s) below to further
research the concepts in this question (if desired).

Research Concepts:
Permissive Hypotension

We update eBooks quarterly and Apps daily based on user feedback. Please tap flag to
report any questions that need improvement.
Question 557: A culture is taken from a draining purulent ulcer in the groin of a critically
ill patient in the intensive care unit who has been admitted for diabetic ketoacidosis. The patient
had initially been improving but then developed groin pain and fever. What antibiotic regimen
should the patient be placed on to best treat the underlying pathology of his groin pain?

Choices:
1. Piperacillin-tazobactam
2. Bactrim
3. Metronidazole & ceftriaxone
4. Gentamicin & metronidazole
Answer: 1 - Piperacillin-tazobactam
Explanations:
Piperacillin-tazobactam, or Zosyn, is the treatment of choice for Fournier's gangrene as it
provides antibiotic coverage against the aerobic and anaerobic bacteria that are at the root of
Fournier's gangrene.
Bactrim would be the appropriate answer to simple cellulitis; however, in Fournier's
gangrene, a strong antibiotic with more coverage is required.
Metronidazole & a 3rd generation cephalosporin is part of the previously used triple
antibiotic therapy that was previously used to treat Fournier's Gangrene.
Triple antibiotic therapy included a 3rd generation cephalosporin or an aminoglycoside in
addition to penicillin & metronidazole.

Go to the next page if you knew the correct answer, or click the link image(s) below to further
research the concepts in this question (if desired).

Research Concepts:
Fournier Gangrene

We update eBooks quarterly and Apps daily based on user feedback. Please tap flag to
report any questions that need improvement.
Question 558: A 45-year-old man presents to the hospital with acute abdominal pain,
which started a couple of hours ago. He appears diaphoretic, uncomfortable, and prefers to lie
still. While examining, he has an episode of non-bilious vomiting and tries to contract the
abdominal wall muscles upon palpation voluntarily. There is no ascites. His vital signs reveal
pulse 110 beats/min, respiratory rate 20 breaths/min, blood pressure 142/86 mmHg, and
temperature 38.3 C. He reports experiencing an episode of epigastric abdominal pain, nausea,
and vomiting over a month ago which self-resolved in 1-2 days. He did not seek medical care at
that time. On careful history, he shares struggling with alcohol use disorder, consuming 6-10
drinks of whiskey every day. Liver functions, amylase, and lipase are elevated while renal
function shows elevated BUN with normal creatinine. What is the best next step in the
management of this patient?

Choices:
1. Reassure and discharge with oral ibuprofen
2. IV fluids and observe inpatient
3. CT of the abdomen and pelvis
4. Urgent upper endoscopy
Answer: 3 - CT of the abdomen and pelvis
Explanations:
This patient has hallmark signs and symptoms of peritonitis. The patient usually is in
tremendous pain, vague, associated with nausea and vomiting. They usually prefer to lie
still.
Given his history of prior epigastric abdominal pain, nausea, and vomiting in the
background of chronic alcohol use, it appears to be an acute episode of self-resolving
pancreatitis. A pseudocyst is a common complication seen in patients with alcoholic
pancreatitis.
Spontaneous regression is usually seen, but in a few cases, complications such as
perforation have been reported. Management will include immediate diagnosis, stabilizing
the patient, and urgent surgical consultation.
Peritonitis requires inpatient management, surgical intervention if needed. To send the
patient home will be unethical. Although IV fluids and stabilizing the patient is part of the
plan, urgent accurate diagnosis of the location and size of perforation is important. Upper
endoscopy has no role in the management of peritonitis.

Go to the next page if you knew the correct answer, or click the link image(s) below to further
research the concepts in this question (if desired).

Research Concepts:
Pancreatic Pseudocyst

We update eBooks quarterly and Apps daily based on user feedback. Please tap flag to
report any questions that need improvement.
Question 559: A 33-year-old woman with a BMI of 35 kg/m2 is brought to the emergency
department for evaluation of shortness of breath of one-day duration. The patient is 33 weeks
pregnant and has a sedentary lifestyle. Her vital signs on evaluation show blood pressure 150/94
mmHg, heart rate 110/min, respiratory rate 22/min, and temperature 100 F. The pulse oximetry is
91% on room air. On physical examination, she is in mild respiratory distress and able to
complete full sentences. The chest is clear on examination, heart rate is regular, the abdomen is
soft, non-tender, fundal height is 33 cm, and she has 1+ pedal pitting edema. Blood counts, renal
function, liver function, electrolytes, and troponin are unremarkable. D-dimer is 1100 ng/ml.
EKG shows sinus tachycardia without any significant ST segment or T wave changes. The chest
x-ray is unremarkable. Ultrasound doppler for lower extremity is negative. Lung V/Q scan shows
a high probability of pulmonary embolism. Which of the following is the next best step in the
management of this patient?

Choices:
1. Therapeutic low molecular weight heparin
2. Unfractionated heparin (UFH) infusion
3. Thrombectomy
4. Apixaban
Answer: 1 - Therapeutic low molecular weight heparin
Explanations:
Treatment of choice of venous thromboembolism in pregnancy is low molecular weight
heparin (LMWH). It’s more efficacious and has a better safety profile than intravenous
unfractionated heparin.
However, LMWH should be discontinued at least 24 hours prior to delivery, and
postpartum treatment should be started as soon as hemostasis is achieved.
Anticoagulation therapy should be continued for at least 6 weeks postpartum. The goal of
the total anticoagulation period is 3-6 months.
A subset of pregnant patients who have a higher risk of recurrent PE (acute PE within the
last month) cannot be left without anticoagulation for 24-36 hours. These patients may
benefit from intravenous unfractionated heparin (UFH) within 24 hours of delivery, which
can be discontinued 4-6 hours prior to delivery. It is also preferred in patients with an
elevated risk of bleeding or persistent hypotension. The rationale is UFH has a shorter half-
life, and a near-complete reversal is possible with protamine. Thrombolytic therapy raises
the risk of maternal hemorrhage and should only be reserved for life-threatening acute
pulmonary embolism. The safety profile of novel oral anticoagulants (NOACs) has not been
studied well for pregnancy.

Go to the next page if you knew the correct answer, or click the link image(s) below to further
research the concepts in this question (if desired).

Research Concepts:
Lung Ventilation Perfusion Scan (VQ Scan)

We update eBooks quarterly and Apps daily based on user feedback. Please tap flag to
report any questions that need improvement.
Question 560: A 50-year-old woman is undergoing right knee arthroscopy. The procedure
goes as planned, and a postoperative adductor canal block is being given. The area is prepped
and draped, and due to the patient's body habitus, visualization proves to be difficult.
Bupivacaine is administered, and the patient immediately becomes severely hypotensive and
appears to have a wide-complex tachyarrhythmia on the monitor. Which of the following is the
best initial step in the management of this patient?

Choices:
1. Initiate immediate intravenous lipid emulsion (ILE) therapy
2. Perform gastric lavage
3. Start advanced cardiac life support
4. Arrange for hemodialysis
Answer: 3 - Start advanced cardiac life support
Explanations:
ACLS guidelines are the most appropriate initial step to follow in hemodynamically
unstable patients.
intravenous lipid emulsion (ILE) therapy is the standard treatment for local anesthetic
systemic toxicity (LAST) according to the ASRA professional society.
ILE may be an adjunct therapy to ACLS for cardiac arrest due to local anesthesia.
It has been emerging in emergency rooms and critical care units as potential rescue therapy
for many other acute toxicities and poisonings. Drug classes that have been investigated
include tricyclic antidepressants, calcium channel blockers, beta-blockers, antipsychotics,
insecticides, and organophosphates.

Go to the next page if you knew the correct answer, or click the link image(s) below to further
research the concepts in this question (if desired).

Research Concepts:
Lipid Emulsion Therapy

We update eBooks quarterly and Apps daily based on user feedback. Please tap flag to
report any questions that need improvement.
Question 561: A 75-year old woman is being evaluated on post-operative day one
following laparoscopic cholecystectomy. Morning labs indicate an increase in her serum
creatinine from 0.8 mg/dL to 1.5 mg/dL. Which of the following best approximates the renal
mass lost for this much increase in serum creatinine in this patient?

Choices:
1. 10%
2. 20%
3. 50%
4. 70%
Answer: 3 - 50%
Explanations:
Perioperative acute kidney injury (AKI) is a serious yet under-recognized problem in
patients who have recently undergone surgery. Due to increasing age and number of
comorbidities, perioperative AKI is increasing in incidence and has significant morbidity
and mortality.
Serum BUN and creatinine levels increase late in the process of acute kidney injury (AKI)
development, with increases in serum creatinine not being recognized until approximately
50% of the renal mass is lost.
Innovations in laboratory analysis of AKI has determined the existence of AKI biomarkers
that can be detected before renal mass is lost. These biomarkers are surrogates of early
stress of the renal tubules. They are mobilized secondary to cardiopulmonary bypass and
surgical stimulation and reflect underlying mechanisms of renal injury since different
biomarkers are released in response to specific insults. Of note, insults due to G1 cell cycle
arrest are associated with TIMP-2 and IGFBP-7 biomarkers and are predictive of major
adverse kidney events, including death, persistent renal dysfunction, and need for dialysis.
Early identification of high-risk patients is made using these biomarkers, with early
application of KDIGO bundles. In patients for whom bundled care is initiated early, there
are lower rates of moderate and severe AKI when compared to patients with AKI that are
treated with standard therapy.

Go to the next page if you knew the correct answer, or click the link image(s) below to further
research the concepts in this question (if desired).

Research Concepts:
Perioperative Acute Kidney Injury

We update eBooks quarterly and Apps daily based on user feedback. Please tap flag to
report any questions that need improvement.
Question 562: A 70-year-old male patient presents to the emergency department (ED) with
tetany and seizures. His EKG shows a prolonged QT interval. His laboratory tests are WBC:
6700 cells/mm^3, Hb: 14.8 mg/dl, and platelet: 167000 cells/microliter. Intravenous calcium
gluconate is initiated. What is the next preferred management?

Choices:
1. Cardiac catheterization
2. Initiate aspirin
3. Check magnesium level
4. Reassure and discharge home
Answer: 3 - Check magnesium level
Explanations:
Hypomagnesemia can cause hypocalcemia, and hypocalcemia is difficult to correct without
first normalizing magnesium.
Replete magnesium level if magnesium concentration is less than 0.8 mEq/L.
Hypocalcemia typically presents with tetany, seizures, and prolonged QT interval.
Continuous observation after administering IV calcium gluconate every 4 to 6 hours is
essential and the patient should not be discharged home.

Go to the next page if you knew the correct answer, or click the link image(s) below to further
research the concepts in this question (if desired).

Research Concepts:
Calcium Gluconate

We update eBooks quarterly and Apps daily based on user feedback. Please tap flag to
report any questions that need improvement.
Question 563: A 62-year-old male is brought to the emergency department following a
witnessed seizure in the grocery store. He is currently alert, and his orientation is improving.
Finger-stick blood glucose upon arrival was 112 mg/dL, with no dextrose administered en route.
Upon questioning, the clinician discovers this has never happened before, and he has no other
significant medical or family history. He undergoes a CT of the head that demonstrates a frontal
lobe parenchymal mass with scattered coarse and gyriform calcifications. After it is determined
there isn't any enhancement on MRI, what is the next test that will aid the most in making the
diagnosis?

Choices:
1. Magnetic resonance spectroscopy
2. Tumor genetic testing
3. Tumor histologic markers
4. Tumor cytology and architecture
Answer: 2 - Tumor genetic testing
Explanations:
A nonenhancing, parenchymal, seizure producing central nervous system tumor is most
likely a low-grade glioma in a patient without a history of cancer. Of these, a frontal lobe
mass with coarse calcifications is most likely an oligodendroglioma. The best way to
determine this from other gliomas is through genetic marker testing.
Oligodendrogliomas are defined by an isocitrate dehydrogenase (IDH) mutation and a
1p/19q codeletion. This combination is absent in the other gliomas such as astrocytoma and
glioblastoma.
Genetic testing plays a prognostic role in diagnosing these tumors, as those that have a
mutated IDH and 1p/19q codeletion respond better to chemotherapy, and are less aggressive
than those with wild-type IDH and lacking the codeletion.
Oligodendrogliomas often share many histological markers with the other glial tumors, such
as glial fibrillary acidic protein (GFAP) found in astrocytomas. Their cytology and cell
architecture is not definitive in diagnosing these tumors.

Go to the next page if you knew the correct answer, or click the link image(s) below to further
research the concepts in this question (if desired).

Research Concepts:
Oligodendroglioma

We update eBooks quarterly and Apps daily based on user feedback. Please tap flag to
report any questions that need improvement.
Question 564: A 45-year-old male who works as a painter and is found obtunded at work.
He has a negative head CT, negative urine toxicology, negative ethanol and the following lab
values: Na 138 mEq/L, K 5.2 mEq/L, Cl 101 mEq/L, HCO3 12 mEq/L, BUN 15 mg/dL,
creatinine 1.1 mg/dL, glucose 96 mg/dL, ABG: PO2 89, PCO2 26, pH 7.19. Which diagnostic
test should be ordered?

Choices:
1. Ketones
2. Urinary potassium
3. Drug screen
4. Plasma osmolality
Answer: 4 - Plasma osmolality
Explanations:
The patient has a compensated (PCO2 should be 1.5(HCO3)+8 for compensation which is
the case) anion gap (AG=32=Na-(Cl+HCO3)) metabolic acidosis and with signs of
intoxication.
The next step is to measure the plasma osmolality and compare it to the estimated
osmolality which is 287 (2Na+(BUN/2.8)+(glucose/18)+(blood ethanol/4.6)) to determine
the osmolar gap (measured osmolality-calculated osmolality=osmolar gap) which, in normal
cases, is less than 10 so a higher gap is due to low molecular weight molecules such as
ethylene glycol, methanol, acetone, and ketones that are not accounted for in the equation.
Methanol is used in paint thinners and is a likely culprit in this case.
Ketoacidosis is a cause of anion gap metabolic acidosis, but given the normal blood glucose
in this case, it is unlikely.

Go to the next page if you knew the correct answer, or click the link image(s) below to further
research the concepts in this question (if desired).

Research Concepts:
Methanol Toxicity

We update eBooks quarterly and Apps daily based on user feedback. Please tap flag to
report any questions that need improvement.
Question 565: A 17-year-old male presents to the emergency department with a history of
being involved in a motor vehicle collision. He currently has no external injury or visible signs.
He mentions difficulty and pain with swallowing and changes in his voice. What is the best time
to perform a CT scan to evaluate his voice changes?

Choices:
1. Within 24 hours
2. After one week of vocal rest
3. After two weeks of vocal rest
4. Never
Answer: 1 - Within 24 hours
Explanations:
It is recommended to perform CT within 24 hours of trauma as a laryngeal fracture is
frequently underdiagnosed.
Triad of odynophagia, dysphagia, and dysphonia should always raise a high degree of
suspicion.
CT scan is the most important diagnostic tool.
CT can differentiate between displaced and undisplaced laryngeal fractures and can make a
difference in management. Early diagnosis and treatment will significantly help prevent
complications.

Go to the next page if you knew the correct answer, or click the link image(s) below to further
research the concepts in this question (if desired).

Research Concepts:
Laryngeal Fracture

We update eBooks quarterly and Apps daily based on user feedback. Please tap flag to
report any questions that need improvement.
Question 566: A 56-year-old man with a past medical history significant for Graves disease
status post thyroidectomy, type 2 diabetes mellitus, chronic kidney disease stage III, and
hypertension is admitted with complaints of confusion, bilateral pedal edema, and abdominal
distention. On physical examination, the patient is found to have bilateral pitting edema, severe
ascites, and icterus. He is drowsy and lethargic. Labs reveal platelet count of 70,000/microL, PT
27 seconds, INR of 2.1, AST 389 IU/L, ALT 206 IU/L, total bilirubin 6.2 mg/dL, gamma
globulin 6.8 mg/dL, and albumin 2.3 mg/dL. Ammonia levels are elevated at 62 mmol/L.
Ultrasound of the abdomen shows a nodular liver with a strong suspicion of cirrhosis. Both the
serum antinuclear antibody (ANA) and anti-smooth muscle antibody (ASMA) come back
positive. The patient is diagnosed to have autoimmune hepatitis. The patient is started on
lactulose, which shows only a slight improvement in his ammonia levels and mental status in 72
hours. He is given IV furosemide and has minimal diuresis and continues to have findings of
severe ascites and significant pedal edema. Which of the following is the next best step in the
management of this patient?

Choices:
1. High-dose prednisone and azathioprine
2. Liver transplantation
3. Continue current treatment and monitor response
4. Obtain liver biopsy
Answer: 2 - Liver transplantation
Explanations:
The mainstay of treatment is around and has been the use of corticosteroids alone or in
combination with azathioprine. The majority of patients respond to initial therapy and go
into remission; however, once the treatment is stopped, the majority of patients relapse as
well.
However, patients who have pre-existing comorbid conditions, and those who have end-
stage decompensated liver disease are usually referred for liver transplantation and are not
started on medications. This patient has significant co-morbid conditions and also appears
to have end-stage decompensated liver disease, as evidenced by his severe ascites, hepatic
encephalopathy, bilateral lower extremity pedal edema, and lab and imaging abnormalities.
He also has not responded very well to the treatment in 72 hours. This patient should be
referred for liver transplantation.
The initiation of steroids and azathioprine is not indicated at this time. He also has not
responded very well to the treatment in 72 hours and is unlikely to respond to the
continuation of the same treatment. A liver biopsy can be done to find out the severity of
disease, but the patient is unstable to undergo an invasive procedure like a liver biopsy at
this time. As evidenced by his severe ascites, hepatic encephalopathy, bilateral lower
extremity pedal edema, lab values, and ultrasound of the abdomen showing a strong
suspicion for cirrhosis, he appears to have end-stage decompensated liver disease already
and liver biopsy would not be very helpful at this time in aiding the diagnosis. Therefore he
should be referred for liver transplantation.

Go to the next page if you knew the correct answer, or click the link image(s) below to further
research the concepts in this question (if desired).

Research Concepts:
Hepatitis

We update eBooks quarterly and Apps daily based on user feedback. Please tap flag to
report any questions that need improvement.
Question 567: A 75-year-old man presents to the hospital with complaints of generalized
fatigue and shortness of breath for the past two weeks. The symptoms started gradually and have
become worse with time. A detailed medical history reveals that he has been diagnosed with
diabetes mellitus about thirty years ago and is on metformin. His vital signs show blood pressure:
85/65 mmHg, pulse rate: 112 beats per minute, temperature: 39 C (102.2 F), respiratory rate: 28
breaths per minute, and oxygen saturation: 89% at room air. A physical examination was carried
out, which reveals a decrease of 20 mmHg of blood pressure on inspiration. Furthermore,
abdominal examination reveals hepatomegaly and peripheral edema. Echocardiography was
carried out, which shows an echolucent free space between the visceral and parietal pericardium.
What physical finding, in addition to these, is most likely to be found in this case?

Choices:
1. Biot respiration
2. Cheyne–Stokes respiration
3. Kussmaul sign
4. Crescendo-decrescendo murmur
Answer: 3 - Kussmaul sign
Explanations:
The clinical scenario is consistent with the diagnosis of effusive-constrictive pericarditis.
Constrictive pericarditis is a disease involving scarring and loss of elasticity of the
pericardium surrounding the heart, leading to impaired filling. Effusive-constrictive
pericarditis (ECP) is a syndrome involving both constriction of the visceral pericardium and
an effusion causing a tamponade-like effect on the heart.
It presents with symptoms of volume overload such as shortness of breath, ascites, and
peripheral edema. Tachycardia occurs reflexively due to decreased cardiac output.
There are many findings on physical examination. One of which is Kussmaul sign, a rise in
right atrial pressure, and eventually the jugular venous pressure during inspiration.
Other physical findings include dyspnea, hepatomegaly, an added heart sound, and pulsus
paradoxus, which refers to a drop in blood pressure of greater than 10mmHg during
inspiration.

Go to the next page if you knew the correct answer, or click the link image(s) below to further
research the concepts in this question (if desired).

Research Concepts:
Constrictive-Effusive Pericarditis

We update eBooks quarterly and Apps daily based on user feedback. Please tap flag to
report any questions that need improvement.
Question 568: Which of the following is the correct statement about duplex scanning for
thrombosis of the inferior vena cava (IVC)?

Choices:
1. The proximity of the IVC to the abdominal wall posteriorly makes the procedure technically
easy
2. Respiratory variation and compressibility of the IVC are not as useful as in duplex scanning of
the femoral system
3. The large caliber of the IVC makes duplex scanning here more reliable than at the femoral
level
4. Only the IVC and iliac systems are visualized with duplex in the abdomen
Answer: 2 - Respiratory variation and compressibility of the IVC are not as useful as in
duplex scanning of the femoral system

Explanations:
The cardiac cycle has more effect than respiratory variation in the IVC above the renal
veins.
The IVC is not compressible as the femoral system is.
Visualization of dilated collaterals may be of value in diagnosis of IVC thrombosis.
The disadvantages of duplex in this situation include anatomic limitations and operator
dependence.

Go to the next page if you knew the correct answer, or click the link image(s) below to further
research the concepts in this question (if desired).

Research Concepts:
Inferior Vena Caval Thrombosis

We update eBooks quarterly and Apps daily based on user feedback. Please tap flag to
report any questions that need improvement.
Question 569: A 32-year-old man presents to the clinic with a 5-month history of
arthralgia, asthenia, decreased libido, cramps, nocturia, dysuria, post-void dribbling, and foamy
urine. He has a history of chronic exposition to paraquat and glyphosate for 15 years in his
agricultural work in EL Salvador, Central America before migrating to the United States. He
takes no medications. An initial set of investigations is shown below.
Patient value Reference range
Serum sodium 128 mEq/L 134-144 mEq/L
Serum potassium 2.7 mEq/L 3.6-5.0 mEq/L
Serum chloride 92 mEq/L 98-107 mEq/L
Serum
1.1 mg/dL 1.8-3.0 mg/dL
magnesium
Serum creatinine 2.9 mg/dL 0.6-1.2 mg/dL
Serum BUN 98 mg/dL 5-20 mg/dL
Urine protein 0.3 g/24 hours 0.15 g/24 hours
Which of the following additional lab findings is most likely to be seen in this patient?

Choices:
1. Urine sediment showing dysmorphic erythrocytes
2. Reduced electrolytes in urine and blood
3. Increased osmolality in blood
4. Elevated fractional excretion of sodium and magnesium
Answer: 4 - Elevated fractional excretion of sodium and magnesium
Explanations:
Electrolyte loss in urine, primarily magnesium, phosphorus, sodium, and potassium—
beginning in the early stages of the disease, signifies primarily tubular damage as the initial
site of renal damage, which once more points to chronic tubulointerstitial nephropathy. The
proximal tubule reabsorbs 60% of electrolytes, which is why this must be the segment most
involved.
This electrolyte loss explains the electrolyte polyuria and low concentrations of some
electrolytes in the blood, as well as the symptoms of cramping and fainting.
Elevated fractional excretion of electrolytes is found in CINAC with a low concentration of
electrolytes in the blood.
Dysmorphic erythrocytes are not described in CINAC. The osmolality in blood in CINAC
decreases due to increased fractional excretion of electrolytes in urine.

Go to the next page if you knew the correct answer, or click the link image(s) below to further
research the concepts in this question (if desired).

Research Concepts:
Chronic Interstitial Nephritis in Agricultural Communities (CINAC)

We update eBooks quarterly and Apps daily based on user feedback. Please tap flag to
report any questions that need improvement.
Question 570: A 28-year-old male patient presents to the emergency department of tertiary
care hospital with complaints of shortness of breath. Upon further inquiry, the patient states that
he works in an industry that is involved in manufacturing generators that utilize a gas that
consists of three atoms of oxygen (O) linked in a cyclic structure. On physical examination, his
vital measurements indicate that he has a blood pressure of 120/80 mmHg, a heart rate of 94
beats per minute, and a breathing rate of 31 breaths per minute. After the initial assessment and
evaluation, a chest radiograph has been ordered. Which of the following other tests should be
considered in the initial emergency evaluation of this patient?

Choices:
1. Electrocardiogram
2. Complete blood count
3. Arterial blood gases
4. Metabolic panel with an anion gap
Answer: 3 - Arterial blood gases
Explanations:
The history of working in a generator manufacturing factory that utilizes a gas that consists
of three atoms of oxygen (O) linked in a cyclic structure along with respiratory complaints
are suggestive of ozone toxicity.
Arterial blood gas and chest radiography should be performed in such patients as exposure
to other pulmonary irritants are possible.
Ozone can lead to the formation of other toxic substances such as formaldehyde when it
reacts with other chemicals from a multitude of products.
Evaluation of ozone toxicity is similar to the evaluation of any pulmonary irritant. Oxygen
saturation monitoring should be implemented and also bedside spirometry. An
electrocardiogram should be performed as well in patients with or at risk for the underlying
cardiac disease.

Go to the next page if you knew the correct answer, or click the link image(s) below to further
research the concepts in this question (if desired).

Research Concepts:
Ozone Toxicity

We update eBooks quarterly and Apps daily based on user feedback. Please tap flag to
report any questions that need improvement.
Question 571: A 17-year-old male presents with severe headaches. He emigrated from
Mexico 5 months ago. He noted headaches about a month ago, on the frontal region, initially
graded 3/10, and gradually progressed to 7/10. This is more prominent in the mornings and
accompanied by nausea. He also claims that for the past week, he has been having vivid visual
hallucinations of animals like goats and insects in his room, which he can describe in great detail.
On examination, BP 140/90 mmHg, HR 54 beats/min, RR 18 breaths/min, T 99.6 F. His pupils
are isocoric, briskly reactive to light, there are no visual field cuts, and fundoscopy reveals
bilateral papilledema. There is no facial asymmetry or dysarthria. The motor and sensory
examinations are normal. There is positive nuchal rigidity, Brudzinski, and bilateral Babinski.
The head CT was normal. The chest x-ray shows pleural apical thickening.The CSF examination
reveals: an opening pressure of 23 cm/H2O, WBC 345/mm^3, 85% lymphocytes, glucose 23
mg/dL, and protein 90 mg/dL. Which of the following etiologic agents is most likely?

Choices:
1. Mycobacterium tuberculosis
2. Cryptococcus neoformans
3. Herpes simplex
4. Rickettsia rickettsia
Answer: 1 - Mycobacterium tuberculosis
Explanations:
Tuberculous meningitis (TBM) is a manifestation of extrapulmonary tuberculosis caused by
the seeding of the meninges with the bacilli of Mycobacterium tuberculosis (MTB).
Tuberculous meningitis presents 1% of all cases of extra-pulmonary TB. In the developed
world where there is a lower prevalence of TB in the population, estimates are that TBM
accounts for 6% of all causes of meningitis.
In locations with a higher prevalence of MTB in the population, estimates are that TBM
accounts for up to one third to one half of all bacterial meningitis.
Tuberculous meningitis assessment is by obtaining cerebrospinal fluid (CSF) for analysis.
Typically, the CSF reveals low glucose, elevated protein, and modestly elevated WBC
count with a lymphocytic predominance. The CSF analysis most closely resembles the CSF
analysis of viral meningitis.

Go to the next page if you knew the correct answer, or click the link image(s) below to further
research the concepts in this question (if desired).

Research Concepts:
Tuberculous Meningitis

We update eBooks quarterly and Apps daily based on user feedback. Please tap flag to
report any questions that need improvement.
Question 572: A 65-year-old man presents with a complaint of left-sided chest pain
radiating to the left arm for the last three hours. His past medical history is significant for
diabetes and hyperlipidemia. Initial vitals showed a pulse of 110/min, blood pressure of 80/40
mmHg, respiratory rate of 16/min, and oxygen saturation of 96% on room air. Physical
examination is unremarkable. The electrocardiogram shows ST elevation in the lead II and aVF
with ST depression in V1-V3. The patient is diagnosed with inferior wall myocardial infarction.
After initial stabilization, the patient is rushed to the catheterization lab. A large thrombus is
noted in the proximal right coronary artery on the coronary angiogram. What is the most
appropriate initial management of hypotension in this patient?

Choices:
1. Epinephrine
2. Dopamine
3. Intravenous fluids
4. Dobutamine
Answer: 3 - Intravenous fluids
Explanations:
Right ventricular infarction complicates up to 40% of inferior ST-elevation myocardial
infarction.
Intravenous fluids are essential to improve the right ventricle preload in the right ventricular
infarct.
Early revascularization is vital to improve the right ventricle infarct.
Norepinephrine and dobutamine should be considered for right ventricular failure when
cardiogenic shock persists after intravenous fluids but not for first-line management.

Go to the next page if you knew the correct answer, or click the link image(s) below to further
research the concepts in this question (if desired).

Research Concepts:
Coronary Artery Thrombus

We update eBooks quarterly and Apps daily based on user feedback. Please tap flag to
report any questions that need improvement.
Question 573: Patient's with mitral stenosis commonly have elevated pressures of which of
the following?

Choices:
1. Left ventricle
2. Pulmonary veins
3. Aorta
4. Peripheral arteries
Answer: 2 - Pulmonary veins
Explanations:
Pulmonary venous pressure is elevated in mitral valve stenosis.
In patients with mitral stenosis, the blood from the left atrium is unable to enter the left
ventricle.
Over time, this results in an enlarged left atrium and the backward pressure results in
elevations in pulmonary venous pressure.
The Left atrium can be massively enlarged and push on the trachea and lead to a chronic
cough.

Go to the next page if you knew the correct answer, or click the link image(s) below to further
research the concepts in this question (if desired).

Research Concepts:
Mitral Stenosis

We update eBooks quarterly and Apps daily based on user feedback. Please tap flag to
report any questions that need improvement.
Question 574: A 65-year-old patient on hospice whose family member pages the nurse stat
to have a healthcare team member evaluate distressing symptoms after administration of
glycopyrronium. Which of the following underlying diseases is least likely to have predisposed
this patient to an adverse effect of antisialagogue?

Choices:
1. Unstable ischemic heart disease
2. Narrow-angle glaucoma
3. End-stage renal disease on dialysis
4. Chronic stable atrial fibrillation
Answer: 4 - Chronic stable atrial fibrillation
Explanations:
Cardiovascular effects (e.g., atrial fibrillation, tachycardia) may occur after administration.
Caution must be exercised in patients with a history of long QT syndrome or whose QTc is
prolonged at the screening.
Patients with narrow-angle glaucoma are at increased risk of adverse effects including
halos, headache, severe eye pain, decreased vision, and nausea/vomiting.
Patients with severe renal impairment including end-stage renal disease on dialysis are at a
highly increased risk of severe systemic exposure to glycopyrrolate.
This agent should be used with caution in patients with unstable ischemic heart disease, left
ventricular failure, history of myocardial infarction, and arrhythmia (excluding chronic
stable atrial fibrillation). Chronic stable atrial fibrillation confers the lowest risk of the
choices to precipitate adverse effects.

Go to the next page if you knew the correct answer, or click the link image(s) below to further
research the concepts in this question (if desired).

Research Concepts:
Antisialagogues

We update eBooks quarterly and Apps daily based on user feedback. Please tap flag to
report any questions that need improvement.
Question 575: A 35-year old man is brought to the emergency department after an
intentional drug overdose. He is unconscious with GCS of 9. His friends report that he had been
feeling depressed for the past few weeks. On examination, the patient has a blood pressure of
90/60 mm Hg, rapid, weak pulse, respiratory rate of 7/min, and dilated pupils. Further
investigation reveals that the patient has taken a large amount of sedative-hypnotic drug that acts
on GABA receptors. Which of the following is the mainstay of treatment for this patient?

Choices:
1. Supportive management focused on blood pressure and respiratory status
2. Administration of naloxone
3. Hyperbaric oxygen therapy
4. Administration of atropine
Answer: 1 - Supportive management focused on blood pressure and respiratory status
Explanations:
Barbiturate overdose results in cold, clammy skin, shallow respirations, dilated pupils, weak
and rapid pulse, sedation, coma, and even death.
Treatment should focus on supporting blood pressure and respiratory status.
The presentation for barbiturate toxicity is somewhat nonspecific with overlap with many
other drugs, including alcohol, benzodiazepines, and anticonvulsants.
It is important to perform toxicological studies to determine the concentration of drugs in
the body, as well as confirm that a barbiturate was taken. Patients may have taken other
drugs in conjunction with the barbiturate, and this may affect treatment.

Go to the next page if you knew the correct answer, or click the link image(s) below to further
research the concepts in this question (if desired).

Research Concepts:
Barbiturate Toxicity

We update eBooks quarterly and Apps daily based on user feedback. Please tap flag to
report any questions that need improvement.
Question 576: A 65-year-old male is in the ICU after sustaining an extensive myocardial
infarction one week ago. He is making a slow recovery. He complains of a new-onset severe
right upper quadrant abdominal pain. He's had very little by mouth during his recovery. His
white cell count is 18,000 and total bilirubin is 3. The abdominal exam shows moderate right
upper quadrant pain with fullness in that area. The ultrasound shows a distended gallbladder with
inflammatory fluid and a thickened gallbladder wall but no gallstones. What is the best treatment
for this patient?

Choices:
1. Laparoscopic cholecystectomy
2. Open cholecystectomy
3. Close observation
4. Percutaneous cholecystostomy tube
Answer: 4 - Percutaneous cholecystostomy tube
Explanations:
The best definitive treatment for a stable patient would be laparoscopic cholecystectomy.
Percutaneous cholecystostomy tube will decompress the acute distended gallbladder in an
unstable patient who is a poor surgical risk.
Often times the acute gallbladder will culture Klebsiella, Strep, Staph, or E. coli.
Unstable patients who are poor surgical candidates may recover without incident when a
percutaneous cholecystostomy tube is utilized and may not require a future
cholecystectomy, especially in the setting of acalculous cholecystitis.

Go to the next page if you knew the correct answer, or click the link image(s) below to further
research the concepts in this question (if desired).

Research Concepts:
Acalculous Cholecystitis

We update eBooks quarterly and Apps daily based on user feedback. Please tap flag to
report any questions that need improvement.
Question 577: A 40-year-old African American female is brought in by her husband,
complaining that she has been drinking a lot of water recently. She has been urinating numerous
times in a day. For two days, she has become drowsy and not been herself as per her husband.
She suffered a traumatic brain injury after falling from the back of a truck 4 months ago and has
had some cognitive deficits since. She also has a history of sarcoidosis, diabetes mellitus, and
hypertension. Her medications include hydrochlorothiazide, metformin, and
hydroxychloroquine. She has not had a flare-up of sarcoidosis recently. Her blood pressure is
140/92 mmHg. Lab work is significant only for serum sodium 108 mEq/L. She is admitted for
the management of hyponatremia. She is administered 3% hypertonic saline at 50 mL/h in the
ICU. Serum sodium is 117 mEq/L 4 hours later. What is the next best step in the management of
this patient?

Choices:
1. Infuse sterile water through the intravenous line
2. Infuse dextrose 5% water
3. Continue hypertonic saline but at a decreased rate of 25 mL/h
4. Administer tolvaptan
Answer: 2 - Infuse dextrose 5% water
Explanations:
The rapid rise in serum sodium in a chronic hyponatremic patient can cause central pontine
myelinolysis. Central pontine myelinolysis is caused by dehydration of the brain tissue and
separation of the myelin sheath from the axons due to higher osmolality in the serum when
compared to the neuron sheaths.
The correct answer here is to infuse D5 water, which is an isotonic solution to decrease
serum sodium level since the serum sodium level in this patient has increased too quickly
by 9 mEq/L in a span of four hours.
D5 water has no sodium in it, thereby diluting and decreasing the sodium level in the blood.
If the serum sodium is increasing too fast, it should be slowed down or reversed, to decrease
the rate of correction. Sodium should not increase by more than 10-12 mEq/L in 24 hours.
Central pontine myelinolysis results in neurological deficits like altered mental status,
difficulty with the ability to speak and swallow, weakness of body parts, and decrease in
sensation, etc. Infusion of sterile water can cause hemolysis due to the hypotonicity of the
fluid. Decreasing hypertonic saline will still make the sodium level go up. Administering
tolvaptan, which is a vasopressin antagonist, will also make the sodium level go up, which
is not desired in this case.

Go to the next page if you knew the correct answer, or click the link image(s) below to further
research the concepts in this question (if desired).

Research Concepts:
Primary Polydipsia

We update eBooks quarterly and Apps daily based on user feedback. Please tap flag to
report any questions that need improvement.
Question 578: A 23-year-old man presents to the emergency department with severe right
upper abdominal pain and abdominal swelling. Laboratory investigations reveal AST 341 U/L,
ALT 455 U/L, and lactic acid 4.1 mg/dL. The obliteration of the hepatic vein is evident in the
abdominal ultrasound. The patient was recently discharged from the hospital following an
episode of minimal change disease. Which of the following is the most likely responsible for this
unusual site of thrombosis?

Choices:
1. Factor V Leiden mutation
2. Protein C deficiency
3. Protein S deficiency
4. Antithrombin III deficiency
Answer: 4 - Antithrombin III deficiency
Explanations:
This patient is likely suffering from thrombosis of the hepatic vein following the acquired
deficiency of antithrombin III due to renal loss.
Antithrombin III binds to heparin on endothelial cells and forms a complex with thrombin
(thrombin-antithrombin (TAT) complex) thus inhibiting coagulation. Its deficiency may
present as early age thrombosis (less than 50 years old) and carries the highest risk for
thrombotic events among the inherited thrombophilias.
ATIII deficiency can occur as a consequence of reduced synthesis (liver damage) or
increased loss (nephrotic syndrome, enteropathy, DIC, sepsis, burn, trauma,
microangiopathy, and cardiopulmonary bypass surgery).
The prothrombin gene mutation, factor V Leiden mutation, deficiencies in protein S, protein
C, and antithrombin account for most of the remaining cases.

Go to the next page if you knew the correct answer, or click the link image(s) below to further
research the concepts in this question (if desired).

Research Concepts:
Hypercoagulability

We update eBooks quarterly and Apps daily based on user feedback. Please tap flag to
report any questions that need improvement.
Question 579: A patient is involved in a motor vehicle accident at 100 mph. The patient
was a restrained driver and is clutching his chest secondary to chest pain. There are muffled heart
sounds, jugular venous distension, pulsus paradoxus, and the impression of the steering wheel on
his chest. Blood pressure is 85/55 mmHg. What should be done?

Choices:
1. CT of the chest
2. Pericardiocentesis or a a wide subxiphoid pericardial window
3. Chest x-ray
4. Cardiac enzymes and EKG
Answer: 2 - Pericardiocentesis or a a wide subxiphoid pericardial window
Explanations:
The patient has pericardial tamponade. There is no time for the confirmative study of
transesophageal echocardiography.
Beck's triad is jugular venous distension, hypotension, and muffled heart tones.
Depending on the skill level of the physician start with pericardiocentesis or if possible
evacuation of blood and clots by a wide subxiphoid pericardial window. This is followed by
a very quick median sternotomy when repair of the associated cardiac injuries is necessary.
Emergency department thoracotomy can be considered if the patient develops cardiac arrest.
Cardiac tamponade can be identified during the FAST exam.

Go to the next page if you knew the correct answer, or click the link image(s) below to further
research the concepts in this question (if desired).

Research Concepts:
Cardiac Tamponade

We update eBooks quarterly and Apps daily based on user feedback. Please tap flag to
report any questions that need improvement.
Question 580: A 17-year-old male patient presents to the emergency department with a 3-
hour history of depersonalization, blurry vision, slurred speech, an ataxic gait, and
hallucinations. The patient was at a party shortly before his symptoms began. His blood pressure
is 170/92 mmHg, pulse 120/min, and temperature 99.9 F (37.7 C). On physical examination, the
patient is aggressive, combative, and poorly cooperative. His skin is diffusely erythematous,
pupils are 8mm bilaterally, and bilateral nystagmus is present. Urine toxicology confirms the
diagnosis. The patient is most likely to also experience which of the following symptoms?

Choices:
1. Homicidal impulses
2. Increased appetite
3. Nosebleed
4. Decreased respiratory rate
Answer: 1 - Homicidal impulses
Explanations:
Phencyclidine (PCP) is a dissociative anesthetic that is a commonly used recreational drug.
PCP is a crystalline powder that can be ingested orally, injected intravenously, inhaled, or
smoked.
PCP is a non-competitive antagonist to the NMDA receptor, which causes analgesia,
anesthesia, cognitive defects, and psychosis. PCP blocks the uptake of dopamine and
norepinephrine, leading to sympathomimetic effects such as hypertension, tachycardia,
bronchodilation, and agitation.
Patients intoxicated with PCP are at increased risk for suicidal or homicidal behaviors and
impulses.
Aggressive behavior, loss of boundaries, and combativeness are associated with PCP
intoxication.

Go to the next page if you knew the correct answer, or click the link image(s) below to further
research the concepts in this question (if desired).

Research Concepts:
Phencyclidine Toxicity

We update eBooks quarterly and Apps daily based on user feedback. Please tap flag to
report any questions that need improvement.
Question 581: A 65-year-old male patient received epidural anesthesia for total knee
arthroplasty with no complications during the procedure. The next day he started complaining of
bilateral leg weakness that progressed to paralysis with no associated pain. What is the cause of
the most likely diagnosis in this patient?

Choices:
1. Accidental dural puncture with an epidural needle
2. Nerve injury
3. Epidural space infection
4. Blood accumulation and ischemic compression of the spinal cord
Answer: 4 - Blood accumulation and ischemic compression of the spinal cord
Explanations:
Accidental puncture of the dura mater with the epidural needle or wet tap presents with a
headache. Bilateral leg weakness is more consistent with the physical compression of the
spinal cord that can be explained by hematoma formation.
Nerve root injury can happen when providing epidural anesthesia. However, the
presentation is usually unilateral and symptoms manifest in the muscular and dermatomal
region that corresponds to the injured nerve root. The bilateral complete paralysis that the
patient presents in this case, can be better explained by a lumbar epidural hematoma.
The epidural abscess formation typically takes more than 24 hours and is accompanied by
signs of infection. An epidural hematoma can present accurately as evidenced in this
patient.
An epidural hematoma can be formed by bleeding inside the epidural space. Ischemic
compression of the spinal cord can cause permanent neurologic damage. Epidural
hematoma is the most likely explanation for this patient considering the acute presentation,
paralysis and lack of accompanying infection signs.

Go to the next page if you knew the correct answer, or click the link image(s) below to further
research the concepts in this question (if desired).

Research Concepts:
Epidural

We update eBooks quarterly and Apps daily based on user feedback. Please tap flag to
report any questions that need improvement.
Question 582: A 25-year-old woman presents with a severe bilateral headache. The
symptoms started two days ago as a mild, unilateral headache, which she tried treating with
paracetamol, but the symptoms slowly progressed and spread to include both sides of her head.
She has no past medical history and is not prone to headaches or migraines. She takes a
combined oral contraceptive pill. An urgent CT head with contrast is ordered, which
demonstrates a cavernous sinus thrombosis. What cranial nerve palsy is most commonly
associated with this condition?

Choices:
1. 3rd nerve palsy
2. 4th nerve palsy
3. 5th nerve palsy
4. 6th nerve palsy
Answer: 4 - 6th nerve palsy
Explanations:
Cavernous sinus thrombosis can cause several cranial nerve palsies, most including palsies
in nerves 3 to 6. The most common, however, is a 6th nerve palsy.
Patients typically present with fever, headache (50% to 90%), periorbital swelling and pain,
and visual changes (photophobia, diplopia, loss of vision).
In general, antimicrobial and antithrombotic therapies are primary considerations as
thrombophlebitis can cause severe infection and death.
Anticoagulation with unfractionated heparin (UFH) or low molecular weight heparin
(LMWH) for several weeks to several months is recommended.

Go to the next page if you knew the correct answer, or click the link image(s) below to further
research the concepts in this question (if desired).

Research Concepts:
Cavernous Sinus Thrombosis

We update eBooks quarterly and Apps daily based on user feedback. Please tap flag to
report any questions that need improvement.
Question 583: A 79-year-old man is hospitalized in ICU for severe pneumonia. In addition
to drug therapy, he is given non-invasive ventilation cycles with a face mask. At the beginning of
the ventilation, the patient is very cooperative. After a few hours, there is a marked improvement
in the respiratory picture with a good increase in oxygenation and reduced dyspnea. However,
despite the improvement, the patient begins to be impatient, shakes, and removes the facial mask
repeatedly. In a short time, he completely loses his temper, threatens the staff, and removes the
venous line because he wants to leave the place that no longer recognizes to be the hospital
where he was initially hospitalized. His heart rate is 57/min. Detailed evaluation reveals no
obvious cause of the patient's behavior. His wife does, however, report a history of excessive
alocohol use. Which of the following is the next best step in the management of this patient?

Choices:
1. Dexmedetomidine
2. Midazolam
3. Haloperidol
4. Rivastigmine
Answer: 2 - Midazolam
Explanations:
Although benzodiazepines can have a delirogenic effect, they are exclusively useful in
patients with alcohol or sedative withdrawal, or for delirium resulting from seizures.
Short-acting benzodiazepines such as midazolam, and lorazepam, must be preferred.
Apart from managing delirium due to alcohol withdrawal, there is no evidence to support
the use of benzodiazepines in the treatment of other types of delirium.
Although rivastigmine, donepezil, and physostigmine have been proposed for ICU delirium
management, evidence of effectiveness is currently very scarce. Dexmedetomidine is useful
in adults under mechanical ventilation when hyperactive delirium can obstacle the weaning,
but it is not recommended for this aim. Moreover, bradycardia in a serious side effect.

Go to the next page if you knew the correct answer, or click the link image(s) below to further
research the concepts in this question (if desired).

Research Concepts:
ICU Delirium

We update eBooks quarterly and Apps daily based on user feedback. Please tap flag to
report any questions that need improvement.
Question 584: A 24-year-old man with no significant past medical history is admitted to
the ICU with severe acute respiratory distress syndrome (ARDS) secondary to influenza. His
ABG shows Pa02 of 75 mmHg while on 100% FIO2. He is intubated and sedated with fentanyl
and propofol. He is on continuous cisatracurium infusion, and his tidal volume is 6 ccs/kg ideal
body weight. He is on inhaled nitric oxide at 40 PPM. He is on airway pressure release
ventilation mode of mechanical ventilation. Today is day 5 of his intubation. He has been prone
for 2 days with no improvement in his PaO2. Which of the following treatment modalities is
most likely to provide the greatest mortality benefit for this patient?

Choices:
1. Use of paralytics
2. Inhaled nitric oxide
3. Airway pressure release ventilation
4. 6 ccs/kg ideal body weight of tidal volume (lung-protective mechanical ventilation strategy)
Answer: 4 - 6 ccs/kg ideal body weight of tidal volume (lung-protective mechanical
ventilation strategy)

Explanations:
Lung-protective mechanical ventilation strategy ( 6cc/kg IBW) as published in 2000 in
NEJM by ARDS-NET group is the only strategy amongst the listed modality which has
shown mortality benefit.
The mechanism of action of lung-protective ventilation (6 ccs/kg IBW tidal volume) is by
preventing further injury/ barotrauma to the already inflamed alveoli in ARDS.
You may not see improvement in oxygenation however you still use lung-protective
mechanical ventilation.
The use of paralytics improves ventilator synchrony; however, in recent trials, it has shown
to offer no mortality benefit. Inhaled nitric oxide improves oxygenation in ARDS patients
by pulmonary vasodilation. However, it has not shown any mortality benefits in clinical
trials. Airway pressure release ventilation is a rescue mode of mechanical ventilation, which
has also not shown mortality benefits.

Go to the next page if you knew the correct answer, or click the link image(s) below to further
research the concepts in this question (if desired).

Research Concepts:
Acute Respiratory Distress Syndrome

We update eBooks quarterly and Apps daily based on user feedback. Please tap flag to
report any questions that need improvement.
Question 585: A 31-year-old woman presents to the hospital for worsening shortness of
breath of 2 weeks duration. She works as a hairdresser and is pretty much on her feet all day. She
has no significant past medical history and had cholecystectomy a year ago. She reports some
mild parasternal chest pain that is nonpleuritic and nonradiating. Her only medication is an over
the counter multivitamin. On examination, her vital signs are normal except for a heart rate of
110/min and oxygen saturation 89%. She also reports some mild left lower extremity pain with
swelling and mild tenderness on palpation around the calf. Homan sign is positive. An EKG
shows sinus tachycardia. D-dimer is elevated. Lower extremity doppler shows a nonocclusive
thrombus involving the left saphenous vein. A stat chest CT angiogram confirms the presence of
pulmonary emboli. A heparin infusion is started and the patient is admitted. The patient's
oxygenation improves within 24 hours on the floor on supplemental oxygen, and she is
transitioned to oral anticoagulation. What is the most appropriate duration of anticoagulation
therapy for this patient?

Choices:
1. 3 months
2. 6 months
3. Indefinite
4. 6 to 12 months
Answer: 3 - Indefinite
Explanations:
Indefinite anticoagulation therapy is indicated in a patient with a first episode of
unprovoked DVT, and unprovoked symptomatic pulmonary embolism.
The risk of bleeding should be considered when deciding on indefinite anticoagulation
treatment. Patients with a low risk of bleed should be anticoagualted indefinitely. For those
with a higher risk of bleeding, indefinite anticoagulation is usually not adviced.
The estimated lifetime risk for recurrent unprovoked thromboembolism is reduced when
anticoagulation therapy is extended.
Anticoagulation therapy for 3 months is recommended for patients with a provoked
thromboembolic event if the inciting event is eliminated.

Go to the next page if you knew the correct answer, or click the link image(s) below to further
research the concepts in this question (if desired).

Research Concepts:
Anticoagulation

We update eBooks quarterly and Apps daily based on user feedback. Please tap flag to
report any questions that need improvement.
Question 586: A 16-year-old male patient presents to the hospital with the difficulty of
breathing. His condition started a week ago with fever and cough. It was noticed that he is
irritable and with bluish discoloration of his lips. His vital signs show a temperature of 40 C (104
F), pulse rate 110/min, respiratory rate 60 breaths/min, and oxygen saturation of 93%. He is
lethargic. Lung examination shows symmetrical chest expansion, with deep intercostal
retractions and crackles in both lung fields. Arterial blood gas examination shows pH 7.2, PCO2
60 mmHg, HCO3 24 mmol/L, PaO2 60 mmHg, and O2 saturation 90% on room air. The patient
is intubated and is hooked to a mechanical ventilator with the following set-up on assist control
mode FiO2 80%, tidal volume 195 ml (6 ml/kg), PEEP of 8 cmH2O. Chest x-ray shows diffuse
bilateral opacities and infiltrates. PaO2/FiO2 ratio is 200 mmHg. Which of the following phases
of his condition shows the repair process takes place by the restoration of epithelial and
endothelial barriers and reestablishing the epithelial integrity with the absorption of the intra-
alveolar fluid that enhances functional recovery?

Choices:
1. Exudative phase
2. Transudative phase
3. Proliferative phase
4. Fibrotic phase
Answer: 3 - Proliferative phase
Explanations:
Acute respiratory distress syndrome (ARDS) is a common condition that is caused by
pulmonary and extra-pulmonary pathologies, and it carries high rates of mortality and
morbidity.
The definition is non-cardiogenic pulmonary edema that occurs because of an inflammatory
process that increases capillary membranes permeability leading to impairment in
oxygenation and gas exchange presenting with hypoxemia and bilateral pulmonary
infiltrates.
There are three distinct phases in the development of ARDS: exudative, proliferative, and
fibrotic phase.
The first exudative phase that occurs over the first 7 to 10 days occurred after lung exposure
to injury with subsequent activation of an inflammatory cascade leading to the accumulation
of protein-rich fluid and hemorrhage secondary to alveolar endothelial and epithelial
barriers damage. In the second proliferative phase, the repair process takes place by the
restoration of epithelial and endothelial barriers and reestablishing the epithelial integrity
with the absorption of the intra-alveolar fluid that enhances functional recovery. The fibrotic
phase, which does not occur in all patients, is associated with fibrous tissue formation and
linked to increased mortality and prolonged duration of mechanical ventilation.

Go to the next page if you knew the correct answer, or click the link image(s) below to further
research the concepts in this question (if desired).

Research Concepts:
Respiratory Distress Syndrome

We update eBooks quarterly and Apps daily based on user feedback. Please tap flag to
report any questions that need improvement.
Question 587: A 30-year-old man is brought to the emergency department after a road
traffic accident. He has multiple bruises over the anterior chest and abdominal wall. His blood
pressure is 80/40 mmHg, and his pulse rate is 110/min. The trachea is midline. Pulmonary
capillary wedge pressure (PCWP) is 12 mmHg. After rapid administration of 1 L crystalloid, the
PCWP is 22 mmHg, and the blood pressure is 85/45 mmHg, with a pulse rate of 124/min. Which
of the following is most likely responsible for the patient's persistently low blood pressure?

Choices:
1. Myocardial contusion
2. Pneumothorax
3. Hypovolaemic shock
4. Neurogenic shock
Answer: 1 - Myocardial contusion
Explanations:
Hypotension and tachycardia following trauma are consistent with shock. The most
common cause of shock in these settings is hypovolaemia. The PCWP is low in
hypovolaemia. Following a fluid challenge, the PCWP increased with no corresponding
change in blood pressure. These findings exclude the diagnosis of shock caused by
hypovolaemia.
Elevated CVP/PCWP with persistent hypotension after fluid bolus should suggest an
alternative diagnosis. Myocardial contusion should be suspected in this patient, which can
be confirmed with elevated cardiac enzymes and ECG changes.
The clinical findings are not suggestive of pneumothorax in this patient.
Pulmonary embolism is associated with high pulmonary artery pressure but normal PCWP.

Go to the next page if you knew the correct answer, or click the link image(s) below to further
research the concepts in this question (if desired).

Research Concepts:
Pulmonary Capillary Wedge Pressure

We update eBooks quarterly and Apps daily based on user feedback. Please tap flag to
report any questions that need improvement.
Question 588: A 17-year-old patient presented to an emergency department following a
sudden onset of severe headache. CT head revealed the presence of an acute subarachnoid
hemorrhage within the interhemispheric cisterns with evolving hydrocephalus. The treating
physician placed an external ventricular drain for managing acute hydrocephalus. However, ten
minutes later, the patient deteriorated, and both his pupils were fixed and dilated. What is the
most likely cause of this sudden deterioration in the patient?

Choices:
1. Blockage of the external ventricular drain
2. Rebleeding
3. Vasospasm
4. Pneumocephalus
Answer: 2 - Rebleeding
Explanations:
The major cause of mortality in patients with ruptured aneurysmal bleed is rebleeding of the
aneurysm.
The mortality in cases of re-rupture of an aneurysm is as high as 90%.
The placement of external ventricular drain and egress of cerebral spinal fluid can reduce
the tamponade effect from the hematoma onto the aneurysmal dome. This can paradoxically
lead to the re-rupture of the aneurysm.
Vasospasm leads to new clinical deficits but seldom leads to acute malignant intracranial
hypertension. The onset of pneumocephalus in cases of acute hydrocephalus is minimal.

Go to the next page if you knew the correct answer, or click the link image(s) below to further
research the concepts in this question (if desired).

Research Concepts:
Cerebral Aneurysm

We update eBooks quarterly and Apps daily based on user feedback. Please tap flag to
report any questions that need improvement.
Question 589: A 67-year-old man is brought to the emergency department by EMS after
the patient was noted to have multiple hematemesis episodes. He was intubated en route to the
ED for airway protection. Based on the history obtained from the patient’s wife, the patient has a
history of alcohol use disorder and reports drinking one bottle of whiskey daily for the past 30
years. On examination, the patient appears disheveled and is sedated and intubated. His heart rate
is 120/min, blood pressure 70/50 mmHg, and respiratory rate is 28/min. Physical examination is
suggestive of spider angiomata on the chest wall, scleral icterus, and abdominal distention. An
initial set of investigations is shown below.
Patient value Reference range
Hemoglobin 6.8 g/dL 13.2-17.5 g/dL
Platelets 90000/microL 150000-400000/microL
Total bilirubin 3.3 mg/dL 0.3-1.0 mg/dL
AST 240 IU/L 35 IU/L
ALT 180 IU/L 10-35 IU/L
ALP 183 IU/L 44-147 IU/L
After initial resuscitation, the patient undergoes an esophagogastroduodenoscopy (EGD)
examination that shows actively bleeding GOV1 varices not amenable to endoscopy intervention
despite multiple attempts, and the performing endoscopist recommends urgent TIPS, which is
only available in a hospital a few hours away. Treatment with multiple pressors is initiated due to
hemorrhagic shock, and the pressor requirement is being escalated to maintain a MAP of 65
mmHg. Which of the following is the next best step in the management of this patient?

Choices:
1. Open surgical repair
2. Palliative care
3. Liver transplantation
4. Balloon tamponade with Linton-Nachlas tube
Answer: 4 - Balloon tamponade with Linton-Nachlas tube
Explanations:
This patient presented with acute gastric variceal hemorrhage secondary to active bleeding
from underlying GOV1 varices.
This patient is in hemorrhagic shock secondary to gastric variceal bleeding based on the
requirement of multiple pressors.
Considering that the patient failed endoscopy intervention, he should be referred for urgent
TIPS placement. However, considering that the current hospital does not have
interventional radiology support and the nearest hospital that can perform TIPS is a few
hours away, the patient would immensely benefit from tamponade with Linton-Nachlas tube
placement as a bridge to definite therapy.
A surgical evaluation must be considered as a last resort.

Go to the next page if you knew the correct answer, or click the link image(s) below to further
research the concepts in this question (if desired).

Research Concepts:
Gastric Varices

We update eBooks quarterly and Apps daily based on user feedback. Please tap flag to
report any questions that need improvement.
Question 590: A 17-year-old male presents to the emergency department after ingesting
slang nut in a suicide attempt. His vital signs include a blood pressure of 128/88 mmHg, pulse of
110/min, a temperature of 99.8 F, and a respiratory rate of 14/min. Treatment for this ingestion
predominantly consists of which of the following?

Choices:
1. Gastric lavage and physostigmine
2. Exchange transfusion and methylene blue
3. Hemodialysis and atropine
4. Airway management and benzodiazepines
Answer: 4 - Airway management and benzodiazepines
Explanations:
Slang nut is the Southeast Asian plant that contains strychnos nux vomica, or strychnine
toxin.
Strychnine toxicity results in inhibition of the inhibitory neurotransmitter, glycine, causing
uncontrolled muscle spasms.
If a patient presents early before any clinical signs of ingestion, activated charcoal can be
administered.
Treatment is with benzodiazepines, decreasing stimulation, intubation, and neuromuscular
blockade if spasms are uncontrolled, patients develop hyperthermia or significant
rhabdomyolysis, or airway compromise occurs.

Go to the next page if you knew the correct answer, or click the link image(s) below to further
research the concepts in this question (if desired).

Research Concepts:
Strychnine Toxicity

We update eBooks quarterly and Apps daily based on user feedback. Please tap flag to
report any questions that need improvement.
Question 591: A 68-year-old female presents to the emergency department with a change
in her mental status. Her past medical history is significant for drug use disorder and chronic
obstructive pulmonary disease (COPD). Her family reports daily alcohol intake. On physical
examination, her vital signs show a heart rate of 110 beats/minute, blood pressure of 160/90
mmHg, and oxygen saturation of 83%. She smells of alcohol and is disoriented. She is breathing
without difficulty, but you notice some vomitus on the edge of her mouth. Her electrocardiogram
shows ST depressions in leads V3-V5. Which of the following is an indication for the use of
airway management?

Choices:
1. Patient's heart rate of 100 beats/minute
2. Patient's oxygen saturation of 83%
3. Patient's disorientation
4. ST depression in leads V3-V5
Answer: 2 - Patient's oxygen saturation of 83%
Explanations:
Indications for the use of airway management are failure to oxygenate, failure to ventilate,
and failure to maintain a patent airway. Her oxygen saturation indicates the failure of
oxygenation.
The modality of airway management primarily depends on the cause and severity of the
patient's condition but is also subject to factors such as environment and clinician skill.
The patient is noted to be breathing without difficulty, so she is maintaining a patent airway,
and disorientation per se is not an indication.
The patient is tachycardic and has ST depressions in the lateral leads (V3-V5), but this
would not, by itself, be a reason for airway management/intubation. These clinical issues do
not fit the previously stated indications for the use of airway management.

Go to the next page if you knew the correct answer, or click the link image(s) below to further
research the concepts in this question (if desired).

Research Concepts:
Airway Assessment

We update eBooks quarterly and Apps daily based on user feedback. Please tap flag to
report any questions that need improvement.
Question 592: A 65-year-old male is hospitalized with persistent atrial fibrillation. He is
started on enoxaparin 30 mg IV followed by 1 mg/kg SQ every 12 hourly to minimize the risk of
ischemic infarction. How should the level of anticoagulation be followed in the patient so as to
minimize the risk of abnormal bleeding?

Choices:
1. Partial thromboplastin time
2. Prothrombin time
3. No monitoring is necessary
4. Bleeding time
Answer: 3 - No monitoring is necessary
Explanations:
Enoxaparin is derived from the depolymerization of unfractionated heparin.
It, however, retains UFH’s ability to activate antithrombin and thereby is effective for
anticoagulation therapy.
Anti-Xa activity level is recommended by the College of American Pathologists for
monitoring the effect of LMWH.
Bleeding time is used to assess platelet function. Prothrombin time is widely used to
monitor warfarin.

Go to the next page if you knew the correct answer, or click the link image(s) below to further
research the concepts in this question (if desired).

Research Concepts:
Enoxaparin

We update eBooks quarterly and Apps daily based on user feedback. Please tap flag to
report any questions that need improvement.
Question 593: A 50-year-old woman from the South-West of the United States presents to
the hospital with fever, headache, and malaise for the last three days. She also complains of
intense pain and swelling on her inner thigh. Her vital signs show a heart rate of 110 beats per
minute, blood pressure of 110/70 mmHg, and temperature of 101 F (38.3 C). Clinical
examination also shows warm and non-fluctuant masses involving her inguinal lymph nodes.
She reveals that she works for a spy agency and that she suspicious animal remains around her
house in the preceding weeks, most likely dead rats. Which of the following is the best initial
therapy for this patient?

Choices:
1. Gentamicin
2. Chloramphenicol
3. Amoxicillin
4. Ceftriaxone
Answer: 1 - Gentamicin
Explanations:
Aminoglycosides such as streptomycin or gentamicin are considered first-line treatment for
seven to ten days in bubonic plague.
More than 80 to 95% of Y. pestis infections usually present with suppurative adenitis,
which is known as the bubonic plague.
Symptoms include sudden fever, chills, headache, and malaise. Within a day or so, a bubo
develops, starting as intense pain and swelling in the regional lymph node area, commonly
inguinal, which is followed by the involvement of axillary or cervical nodes.
Chloramphenicol is preferred in case of meningitis.

Go to the next page if you knew the correct answer, or click the link image(s) below to further
research the concepts in this question (if desired).

Research Concepts:
Comprehensive Review Of Bioterrorism

We update eBooks quarterly and Apps daily based on user feedback. Please tap flag to
report any questions that need improvement.
Question 594: A 65-year-old female presents to the emergency department with a chief
complaint of altered mental status. The patient was found unresponsive. Her heart rate is
110/min; the blood pressure is 90/60 mmHg, respiratory rate 12 breaths per minute, SpO2 98%
on room air, finger-stick glucose is 125mg/dL. Initial resuscitative and evaluation measures are
appropriately implemented. The patient's home-health attendant states that the patient has been
refusing oral intake except for toast and drinks tea exclusively. During the evaluation, the patient
suffers a tonic-clonic seizure that is successfully aborted with 4 mg of intravenous lorazepam.
The basic metabolic panel shows Na 113 mmol/L. The first-line treatment for this condition is
administered, and the patient is eventually admitted to the medical ICU. Several days later, the
patient again seizes and is found to have decreased reflexes. Repeat serum sodium is within
normal limits. What is the likely iatrogenic cause of the patient's deteriorating condition?

Choices:
1. Bacterial meningitis
2. Hypoglycemia
3. Central pontine myelinolysis
4. Subarachnoid hemorrhage
Answer: 3 - Central pontine myelinolysis
Explanations:
Overcorrection of serum sodium, particularly when chronically low can lead to the
devastating condition of central pontine myelinolysis.
The "tea and toast" diet is a common etiology of hyponatremia, other potential causes
include beer potomania, cerebral salt wasting, and SIADH.
Serum sodium should be corrected at a rate not faster than 6-12 mEq in the first 24 hours
and no more than 18 mEq in 48 hours.
3% Sodium Chloride is indicated in this patient given acute neurologic sequelae of severe
hyponatremia.

Go to the next page if you knew the correct answer, or click the link image(s) below to further
research the concepts in this question (if desired).

Research Concepts:
Crystalloid Fluids

We update eBooks quarterly and Apps daily based on user feedback. Please tap flag to
report any questions that need improvement.
Question 595: A 26-year-old patient is suspected of having overdosed on heroin that was
suspected. He required a total of 4 mg of naloxone. An initial 2 mg was given by emergency
medical responders and 2 mg in the emergency department. The clinician is called to the bedside
by the emergency department nurse as the patient seems to be having increased work of
breathing, and his oxygen saturation is 87%. His pulse is 110/min, and his respiratory rate is
25/min. What is the next best step in the management of this patient?

Choices:
1. Administer an additional dose of 4 mg IV naloxone
2. Place the patient on supplemental oxygen
3. Urgent intubation and ventilation
4. Obtain a CT-pulmonary angiogram of the chest to evaluate for pulmonary embolism
Answer: 2 - Place the patient on supplemental oxygen
Explanations:
Negative pressure pulmonary edema is a known complication of naloxone administration,
occurring in up to 3.6% of patients who receive naloxone.
The mechanism for developing pulmonary edema in the context of opioid reversal with
naloxone is multifactorial.
Mechanisms proposed include catecholamine surge, pulmonary capillary leak, and fleeting
airway obstruction by a large inspiration against a closed glottis.
Despite the possible complication of negative pressure pulmonary edema, this should not
deter the use of naloxone in case of overdose.

Go to the next page if you knew the correct answer, or click the link image(s) below to further
research the concepts in this question (if desired).

Research Concepts:
Naloxone

We update eBooks quarterly and Apps daily based on user feedback. Please tap flag to
report any questions that need improvement.
Question 596: A 65-year-old undergoes an antrectomy with truncal vagotomy. Four days
later, he is found to have mediastinal emphysema, fever, and mild dyspnea. A chest radiograph
reveals pleural fluid in the left hemithorax and bilateral fluffy air-space disease. What is the most
likely diagnosis?

Choices:
1. Congestive heart failure
2. Pneumonia
3. Pulmonary embolism
4. Esophageal tear
Answer: 4 - Esophageal tear
Explanations:
When surgery is performed on the esophagus, there is always the chance of an esophageal
tear.
After a truncal vagotomy, the esophagus is occasionally torn.
Tearing of the esophagus can present with rapid acute respiratory distress syndrome and
subcutaneous emphysema.
The diagnostic test of choice for an esophageal rupture is a contrast esophagram, beginning
with water-soluble contrast and following, if no tear is evident, with small amounts of
barium.

Go to the next page if you knew the correct answer, or click the link image(s) below to further
research the concepts in this question (if desired).

Research Concepts:
Esophageal Perforation And Tears

We update eBooks quarterly and Apps daily based on user feedback. Please tap flag to
report any questions that need improvement.
Question 597: A 17-year-old man on treatment for frequent seizures is brought to the
emergency department by his friend after two episodes of myoclonic jerking in the legs, each
lasting about 6 minutes. According to his friend, the patient can predict the onset of his seizures
and would stare into the distance with a lip-smacking movement for about 1 minute. Sometimes
he would start jerking both arms for about 30 seconds, but this time it lasted 6 minutes and
happened once again on their way to the emergency department. He did not lose consciousness
or urinate on himself. Blood pressure is 100/60 mm Hg, and the pulse rate is 70/min. On
neurological exam, a weakness in the lower right limb, with a power of +1, is observed.
Laboratory reports are all within normal limits. The blood valproate level is within normal limits.
One hour after the initial evaluation, the patient has no recurrence of seizure activity has a power
of 4+ in all the limbs. The pupils are bilaterally equal and reactive. Neurology consult
recommended a new drug to be added to the patient's regimen to control his condition better.
What is the side effect classically associated with this medication?

Choices:
1. Pancreatitis
2. Liver failure
3. Gingival hyperplasia
4. Steven Johnson syndrome
Answer: 4 - Steven Johnson syndrome
Explanations:
Lamotrigine is the treatment of choice in a case of partial seizures associated with
secondary generalization. Lamotrigine acts by blocking voltage-gated sodium channels.
Lamotrigine is used in patients who do not tolerate, do not respond to, or have
contraindications to valproate. Apart from partial seizures, it is also specifically used for
Lennaux Gestaut syndrome. Lamotrigine is also used in treating bipolar disorder.
The adverse reaction associated with lamotrigine use is Stevens-Johnson syndrome,
especially when used with carbamazepine. Stevens-Johnson syndrome presents with a
generalized rash, widespread bullae, and mucosal involvement. These bullae rapidly
progress to denude the surface epithelium, potentially leading to severe dehydration and
sepsis, among other complications. Treatment involves immediate cessation of the drug and
supportive care. Lamotrigine is also associated with toxic epidermal necrolysis and DRESS
(Drug rash with eosinophilia and systemic symptoms) syndrome.
Lamotrigine can occasionally cause liver problems, but this is uncommon. DRESS (Drug
rash with eosinophilia and systemic symptoms) syndrome is a noteworthy side effect of
both phenytoin and carbamazepine, while gingival hyperplasia is specific to phenytoin.
Pancreatitis is a rare side effect with valproate, not lamotrigine.

Go to the next page if you knew the correct answer, or click the link image(s) below to further
research the concepts in this question (if desired).

Research Concepts:
Partial Epilepsy

We update eBooks quarterly and Apps daily based on user feedback. Please tap flag to
report any questions that need improvement.
Question 598: An 18-year-old man is brought to the hospital after a high-speed car
accident. He is mildly confused and has a GCS of 14/15, his heart rate is 100/min, and his blood
pressure is 110/70 mmHg. Admission labs have been sent and are pending. He has large bruises
around each eye and is complaining of significant left flank pain. During the primary survey, he
begins to have multiple episodes of emesis. Two large-bore IV lines are placed. Which of the
following is the next best step in the management of this patient?

Choices:
1. Place a nasogastric tube
2. Turn the patient on his side to minimize aspiration risk and continue the primary survey
3. Turn the patient on his side and administer supportive care such as ondansetron
4. Proceed emergently to the operating room
Answer: 2 - Turn the patient on his side to minimize aspiration risk and continue the primary
survey

Explanations:
Turning the patient on his side to avoid aspiration and continuing the survey is correct for
this patient. As he is a trauma patient, it is imperative to complete the primary and
secondary surveys to ensure no major injury is missed.
If the emesis continues and becomes intractable, it should raise suspicion for possible
increased intracranial pressure, especially in this patient who was injured in a high-speed
accident and has bilateral periorbital ecchymoses.
However, symptoms associated with increased intracranial pressure, such as blown pupils,
are evaluated in the primary survey, which should be performed in a systematic manner.
The bilateral ecchymoses around each eye, also known as raccoon eye sign, should raise
suspicion for a basilar skull fracture. Placement of a nasogastric tube under these conditions
is contraindicated as it may worsen the fracture, or the tube may even slip inside the skull
cavity. Turning the patient on his side to minimize aspiration risk is the correct maneuver,
but providing supportive care should wait until the primary and secondary surveys are
completed in order to ensure significant injuries are not missed. Given the information
provided in the question stem, it is unclear whether this patient needs to go to the operating
room. Completing the primary and secondary surveys are imperative to determine the best
option for this patient.

Go to the next page if you knew the correct answer, or click the link image(s) below to further
research the concepts in this question (if desired).

Research Concepts:
Nasogastric Tube

We update eBooks quarterly and Apps daily based on user feedback. Please tap flag to
report any questions that need improvement.
Question 599: A large group of politicians were at a party and suddenly become ill. Several
died having suffered from convulsions. Those that present to the hospital have cyanosis,
headache, hyperventilation, hypotension, lightheadedness, nausea, vomiting, and metabolic
acidosis. Which of the following should be administered first?

Choices:
1. 100% oxygen
2. Sodium thiosulfate
3. Sodium nitrate
4. Amyl nitrite
Answer: 1 - 100% oxygen
Explanations:
Initial care for patients with cyanide exposure is airway control, ventilation, and 100%
oxygen.
Crystalloids and vasopressors are used for hypotension.
Sodium bicarbonate is titrated according to arterial blood gas.
Administer hydroxocobalamin or sodium thiosulfate and sodium nitrite if the diagnosis is
strongly suspected. Do not wait for laboratory confirmation.

Go to the next page if you knew the correct answer, or click the link image(s) below to further
research the concepts in this question (if desired).

Research Concepts:
Cyanide Toxicity

We update eBooks quarterly and Apps daily based on user feedback. Please tap flag to
report any questions that need improvement.
Question 600: A 65-year-old farmer is brought to the emergency department of a tertiary
care hospital with complaints of chest pain six hours after being bitten by an insect while he was
walking in his fields barefoot. Upon physical examination, he is diaphoretic and clutching his
chest, and two-bite marks with surrounding erythema are seen at the site of the bite.
Cardiovascular examination reveals a regular rhythm and rate along with bilaterally strong and
symmetric pulses. Respiratory examination reveals that the lungs are clear bilaterally. An ECG
done shows nonspecific ST-T changes in multiple leads. Which of the following is the next best
step in the management of this patient?

Choices:
1. Send troponins
2. Give IV fluids
3. Attach oxygen
4. Irrigate the wound
Answer: 1 - Send troponins
Explanations:
The sign and symptoms in this patient are suggestive of Centipede envenomation.
Centipede venom is a pharmacologically diverse and potent substance. Venom can include
bioactive proteins, peptides, and other small molecules. These can have myotoxic,
cardiotoxic, and neurotoxic effects.
Local wound care is the primary management of uncomplicated centipede bites. Monitoring
for systemic symptoms is recommended. If the patient complains of specific symptoms such
as chest pain, a workup for myocardial ischemia, including troponins and EKG, should be
obtained.
Treatment of minor centipede bites includes irrigating the site to reduce the risk of infection,
applying ice packs also helps as the cold elevates the pain threshold impedes nerve
conduction, and vasoconstricts vessels to prevent tissue edema.
Some patients report pain improvement with submersion of the extremity in hot water, as it
is thought to denature any heat-labile toxins in the venom. However, some patients have
also reported increased pain with hot water exposure.

Go to the next page if you knew the correct answer, or click the link image(s) below to further
research the concepts in this question (if desired).

Research Concepts:
Centipede Envenomation

We update eBooks quarterly and Apps daily based on user feedback. Please tap flag to
report any questions that need improvement.
Section 7
Question 601: A 22-year-old woman is brought to the hospital with altered mental status,
hallucinations and severe rigidity. She had an intrathecal pump implanted 12 months ago for the
management of her underlying movement disorder. Her mother states that she has not been able
to follow up with her appointments recently. Which of the following is the most likely cause of
her presenting symptoms?

Choices:
1. Drinking alcohol during the administration of the intrathecal medication
2. Abrupt withdrawal of the intrathecal medication
3. Overdose of the intrathecal medication
4. Taking erythromycin while being on the intrathecal administration of medication
Answer: 2 - Abrupt withdrawal of the intrathecal medication
Explanations:
Abrupt withdrawal of intrathecal baclofen may occur in patients using the drug over 2
months and may result in a hypermetabolic state with hyperpyrexia, impaired mental status,
muscle rigidity and severe rebound spasticity which may advance to rhabdomyolysis and
multi-system failure.
The FDA requires a black box warning for baclofen. Abrupt withdrawal of intrathecal
baclofen may occur in patients using the drug over 2 months and may result in a
hypermetabolic state with hyperpyrexia, impaired mental status, muscle rigidity, and severe
rebound spasticity which may advance to rhabdomyolysis and multi-organ system failure.
The symptoms improve upon recommencing of baclofen by a similar dose as before the
interruption. Withdrawal most commonly occurs because of a delivery system problem. The
patient and caregiver must understand the importance of monitoring the pump to prevent
withdrawal events.
Abrupt discontinuation of oral baclofen therapy might cause seizures and hallucinations.
Gradual dose reduction is recommended to prevent withdrawal symptoms.
Drinking alcohol in a patient on the administration of intrathecal baclofen will not cause a
hypermetabolic state. Overdose of the intrathecal baclofen will not cause a hypermetabolic
state. Taking erythromycin while being on the intrathecal administration of baclofen will
not cause a hypermetabolic state.

Go to the next page if you knew the correct answer, or click the link image(s) below to further
research the concepts in this question (if desired).

Research Concepts:
Baclofen

We update eBooks quarterly and Apps daily based on user feedback. Please tap flag to
report any questions that need improvement.
Question 602: A 65-year-old male presents to the emergency department in respiratory
distress. He has obvious tongue swelling, increased work of breathing, and drooling. Vital signs
show tachycardia, respiratory rate of 30/minute, and oxygen saturation of 70%. He has no known
medical problems and takes no medications. What is the next best step in the management of this
patient?

Choices:
1. Intubation
2. Epinephrine
3. Corticosteroids
4. Continuous positive airway pressure
Answer: 1 - Intubation
Explanations:
This patient is suffering from hereditary angioedema resulting in airway compromise. In the
absence of ACE inhibitors or allergen contacts, this patient has a much higher probability of
suffering from a C1 esterase inhibitor defect.
Patients in respiratory distress with low oxygen saturation and oral swelling should receive
a definitive airway sooner rather than later due to obtaining an airway becoming more
difficult as swelling worsens.
In cases of airway swelling, epinephrine should be attempted as it is more common to have
anaphylaxis than hereditary angioedema.
Once attempts at epinephrine and other anaphylactic medications have been unsuccessful,
and the diagnosis of hereditary angioedema (HAE) is suspected, specific therapies should
be attempted. Specific therapy for C1 esterase inhibitor deficiency with laryngeal edema
involves replacing the deficient enzyme. The easiest way is to give human plasma, which
contains C1 esterase inhibitor (C1INH ). The initial dose of human plasma is two units.
C1INH concentrate, which is derived from human plasma, may be used. The initial dose is
1500 units.

Go to the next page if you knew the correct answer, or click the link image(s) below to further
research the concepts in this question (if desired).

Research Concepts:
C1 Esterase Inhibitor Deficiency

We update eBooks quarterly and Apps daily based on user feedback. Please tap flag to
report any questions that need improvement.
Question 603: A 58-year-old male with a past medical history of prostate cancer, status
post prostate resection 5 years ago, as well as long-standing osteoarthritis and cigarette smoking
which is 3 weeks status post left knee replacement presents to the emergency department with a
complaint of 3 days of progressive pleuritic chest discomfort, shortness of breath, and a mild
cough with blood-tinged sputum. His left lower extremity has been persistently swollen since the
surgery. His vital signs include a heart rate of 104 beats per minute, blood pressure of 138/84
mm Hg, pulse oximetry reading of 97% on room air, and an oral temperature of 37.4 C (99.32
F). Which of the following is most accurate regarding his diagnosis?

Choices:
1. The left upper lobe of his lung is the most likely region of the lung to be affected by this
condition.
2. Air bronchograms on a CT of the chest are highly suggestive of this diagnosis.
3. An area of radiographic oligemia (increased lucency) on a chest x-ray makes this diagnosis
less likely.
4. A peripheral wedge-shaped hyperdensity seen on a chest x-ray in the appropriate clinical
setting is a specific finding for this diagnosis.
Answer: 4 - A peripheral wedge-shaped hyperdensity seen on a chest x-ray in the appropriate
clinical setting is a specific finding for this diagnosis.

Explanations:
Pulmonary embolism is the most common cause of pulmonary infarction and, though there
are numerous risk factors and signs of pulmonary embolism, clinical signs and symptoms of
deep vein thrombosis, heart rate over 100 beats per minute, immobilization or surgery
within the previous 4 weeks, history of previous venous thromboembolism, hemoptysis,
malignancy, exogenous hormone use, and age over 50 are frequently cited.
Hampton’s hump (wedge-shaped consolidation at the lung periphery), Westermark’s sign
(radiographic oligemia or increased lucency), and Fleischer sign (prominent pulmonary
artery) are specific chest x-ray findings for pulmonary infarction.
Though chest x-rays can show specific findings that are highly suggestive of pulmonary
infarction, these findings have poor overall sensitivity, making CT angiogram of the chest
the preferred method for diagnosis of pulmonary embolism with or without infarction. CT
angiogram findings associated with pulmonary infarction include a feeding vessel or "vessel
sign," central lucency, and semicircular shape. Additionally, the finding of air
bronchograms on CT makes the presence of a pulmonary infarction less likely.
Unilateral infarct occurs most common with pulmonary infarction, with the lower lobes
having a strong predilection over the upper lobes, and the right lower lobe is the most
frequently affected.

Go to the next page if you knew the correct answer, or click the link image(s) below to further
research the concepts in this question (if desired).

Research Concepts:
Pulmonary Infarction

We update eBooks quarterly and Apps daily based on user feedback. Please tap flag to
report any questions that need improvement.
Question 604: A 51-year-old man presents to the hospital with myalgia and dark-colored
urine after a crush injury. Vital signs show a heart rate of 100/min and blood pressure 130/100
mmHg. The physical examination is unremarkable except for a laceration over the right leg.
Initial labs reveal a creatine phosphokinase level of 6000 IU/L (normal 30-223 IU/L). His
troponin level is 0.1 ng/mL ( normal 0.00 to 0.04 ng/mL). EKG shows T wave inversion in lead
1. What is the best initial step in the management of this patient?

Choices:
1. Intravenous normal saline
2. Aspirin and clopidogrel
3. Echocardiogram
4. Hemodialysis
Answer: 1 - Intravenous normal saline
Explanations:
The diagnosis of rhabdomyolysis is almost certain when creatine phosphokinase levels
exceed 5000 IU/L.
Rhabdomyolysis is suspected in levels above 1000 IU/L. It is the diagnosis in this patient
because there was a crush injury and also dark urine along with muscle pain.
Treatment of rhabdomyolysis is fluid resuscitation with isotonic saline.
Myocardial infarction would not cause discoloration in urine, and also, an ECG would show
more specific findings. Sometimes rhabdomyolysis causes a mild elevation in troponins, but
that does not indicate myocardial infarction in the absence of cardiac symptoms.

Go to the next page if you knew the correct answer, or click the link image(s) below to further
research the concepts in this question (if desired).

Research Concepts:
Creatine Phosphokinase

We update eBooks quarterly and Apps daily based on user feedback. Please tap flag to
report any questions that need improvement.
Question 605: A 29-year-old woman is brought to the emergency department after having
an episode of a seizure lasting for more than 5 minutes. She is intubated for airway protection,
and intravenous (IV) phenytoin is given. Her family reports that she has been complaining of
severe headaches for the last few days before the onset of a seizure. On examination, she remains
sedated, unresponsive, and withdraws to noxious stimuli more prominently on the left side.
Contrast CT scan detects a ring-enhancing lesion vasogenic edema in the left paracentral gyrus.
Further workup indicates halo-shaped pulmonary infiltrates. Blood investigations show
neutropenia. She also has glioblastoma (GBM) multiforme, for which she takes high-dose
prednisone. Which of the following is the most likely etiology of the focal lesion seen on CT
scan in this patient?

Choices:
1. Aspergillosis
2. Cryptococcus
3. Candidiasis
4. Coccidioides
Answer: 1 - Aspergillosis
Explanations:
Aspergillus is a fungus found throughout the world that can cause infection in primarily
immunocompromised hosts and individuals with the underlying pulmonary disease.
Invasive aspergillosis is common only in the immunocompromised population, composed
of patients with AIDS, neutropenic patients, those on long-term corticosteroids, and
recipients of transplants on anti-rejection medications.
The most common initial symptoms include dyspnea, increased sputum production, chills,
headache, and arthralgias. As the condition quickly progresses, fever, toxicity, and weight
loss manifest. On examination, there may be sinus tenderness, nasal discharge, rales,
dermatologic changes, or meningeal signs of the central nervous system.
Chest radiographs may show parenchymal opacities of pulmonary aspergilloma (fungus
ball). CT imaging of the lungs will show characteristic nodules with surrounding
attenuation (“halo sign”), aspergilloma (fungal ball in a pre-existing lung cavity),
cavitations, or fibrosis. CT may also be useful to evaluate for sinus involvement such as
masses, opacification, or destruction of sinus walls if this manifestation of aspergillosis is
suspected.

Go to the next page if you knew the correct answer, or click the link image(s) below to further
research the concepts in this question (if desired).

Research Concepts:
Aspergillosis

We update eBooks quarterly and Apps daily based on user feedback. Please tap flag to
report any questions that need improvement.
Question 606: A 53-year-old male industrial worker is brought to the hospital in an acute
confusional state. The patient's attendant states that this company is a pharmaceutical company
where this patient was handling a bag that contains carbon tetrachloride. He was not wearing a
protective face mask. On examination, the patient appears confused. Vital signs show
temperature 39°C, blood pressure 130/86 mmHg, pulse 100/min, and respiratory rate 15/min. On
laboratory investigation, serum ALT and AST are elevated. Serum creatinine is 1.8 mg/dL.
Intravenous cimetidine is given to the patient. Which of the following best explains the
underlying mechanism of action of this medication beneficial for this patient?

Choices:
1. Stimulation of CYP450 enzyme
2. Inhibition of CYP450 enzyme
3. Inhibition of glutathione reductase
4. Stimulation of glutathione peroxidase
Answer: 2 - Inhibition of CYP450 enzyme
Explanations:
Carbon tetrachloride (CCL4) toxicity causes multiple organ dysfunction by lipid
peroxidation and some other mechanisms. This lipid peroxidation occurs by free radical
formation from CCL4 by the help of CYP450 enzymes. This enzyme can be inhibited by
intravenous cimetidine.
Cimetidine acts as a CYP450 enzyme inhibitor. When this enzyme is inhibited by
cimetidine, free radical formation is decreased from CCL4. So, complications will be less.
Intravenous glucose also acts like as intravenous cimetidine to downregulate CYP450 to
reduce CCL4 degradation.
If an appropriate measure is not taken in a patient with CCL4 intoxication, life-threatening
complications may develop. CO2 induced hyperventilation to exhale toxin, enzyme
inhibition by cimetidine, or glucose to decrease the free radical formation are commonly
available options. Electrolyte imbalance and metabolic acidosis should also be treated if
present.

Go to the next page if you knew the correct answer, or click the link image(s) below to further
research the concepts in this question (if desired).

Research Concepts:
Carbon Tetrachloride Toxicity

We update eBooks quarterly and Apps daily based on user feedback. Please tap flag to
report any questions that need improvement.
Question 607: A car pulls up to the emergency department entrance, and several teenagers
rush in, asking for help. The staff pulls a somnolent 16-year-old girl from the back seat and
brings her into the resuscitation room. The patient sent a text message to her friends, stating that
she was depressed about a bad breakup and had taken an overdose to "end the pain." Her
companions don’t know much about her health history, but one mentions that she has “Tetralogy
or something like that” and had surgery as a baby. She believes that she only takes one
medication, ”a heart pill.” Her EKG reveals bradycardia and a first-degree heart block. Also, her
blood glucose is 368 mg/dL. Her cardiac problem quickly escalates to a 2nd and then a 3rd-
degree heart block. Having astutely decided what the likely offending agent is, what are the
probable treatment options?

Choices:
1. Glucagon, vasopressors, cardiac pacing
2. Sodium bicarbonate, intralipid, urine alkalinization
3. High dose insulin, intralipid, IV calcium, atropine, vasopressors, methylene blue, ECMO
4. Sodium bicarbonate, benzodiazepines, ECMO
Answer: 3 - High dose insulin, intralipid, IV calcium, atropine, vasopressors, methylene
blue, ECMO

Explanations:
This patient has Tetralogy of Fallot and was placed on calcium channel blockers (CCBs)
due to potential rhythm disturbances after the surgery. She overdosed on this medication
and already exhibited toxicity when she was found.
CCBs interfere with insulin secretion which leads to hyperglycemia. Hemodynamic effects
are bradycardia, hypotension, and eventual cardiovascular collapse.
This often fatal ingestion requires prompt, aggressive, and multimodal treatment, much of
which can have minimal effect in massive intakes. Vasopressors and ECMO are often
lifesaving in these extreme cases.
Glucagon, vasopressors, and cardiac pacing would be most appropriate for a beta-blocker
overdose. Beta-blockers also interfere with gluconeogenesis, usually leading to
hypoglycemia. This patient has hyperglycemia. Sodium bicarbonate, intralipid, urine
alkalinization would be most appropriate for an overdose of tricyclic antidepressants
(TCAs). TCAs typically cause tachycardia followed by QRS prolongation and right axis
deviation. Sodium bicarbonate, benzodiazepines, and ECMO would be most appropriate for
an overdose of antimalarial drugs.

Go to the next page if you knew the correct answer, or click the link image(s) below to further
research the concepts in this question (if desired).

Research Concepts:
Deadly Single Dose Agents

We update eBooks quarterly and Apps daily based on user feedback. Please tap flag to
report any questions that need improvement.
Question 608: A 32-year-old woman is brought to the hospital because of her inability to
pass urine for the last 24 hours. Immediate catheterization is done. After taking proper history
and past medical records, she has been found as a patient of chronic kidney disease. She also has
fatigue, general weakness. Serum electrolytes show a potassium level of 6.4 mEq/L. An oral
medication with no systemic absorption is prescribed. While treating with this medication, the
patient develops respiratory distress, which becomes more severe while lying on bed and
improves while sitting or standing. She also develops swelling in the lower limb. Cardiac
auscultation reveals S3 gallop. The medication given for hyperkalemia is stopped. Which of the
following is most likely responsible for the patient's new symptoms?

Choices:
1. Hyperkalemia
2. Hypermagnesemia
3. Sodium load
4. Hypercalcemia
Answer: 3 - Sodium load
Explanations:
Sodium polystyrene sulfonate is an oral drug that can be used for hyperkalemia.
Besides lowering the potassium level, it may cause sodium loading.
Patients with preexisting fluid overload (respiratory distress, orthopnea, edema, S3) may not
be able to tolerate this sodium load.
In this condition, SPS should be used with great caution, and electrolytes should be
monitored regularly.

Go to the next page if you knew the correct answer, or click the link image(s) below to further
research the concepts in this question (if desired).

Research Concepts:
Sodium Polystyrene Sulfonate

We update eBooks quarterly and Apps daily based on user feedback. Please tap flag to
report any questions that need improvement.
Question 609: A 74-year-old male patient is recovering in the surgical ICU after a motor
vehicle accident. The patient has been treated for multiple extremity fractures and consistently
notes pain level at 8/10. He is currently undergoing a trial of pressure support ventilation. The
current ventilator setting is as follows: pressure support mode 12 cmH2O, CPAP 5 cmH2O, and
45 % FiO2. The patient presents with the following monitored parameters 10 min into the PSV
mode: VTe 550 ml, spontaneous RR 28 bpm, and BP 160/95 mmHg. The patient presents to be
very anxious. Which of the following is the next best step in the management of this patient?

Choices:
1. Sedate the patient with propofol.
2. Return the patient to a mechanically controlled rate.
3. Give fentanyl for analgesia and monitor the respiratory status.
4. Verbally reassure the patient.
Answer: 3 - Give fentanyl for analgesia and monitor the respiratory status.
Explanations:
When a patient experiences pain, other monitored data such as heart rate and blood pressure
deviate from acceptable ranges.
Maintaining a patient comfort level while keeping the respiratory drive in mind is an
important part of a spontaneous breathing trial.
Pain and anxiety should be ruled out before discontinuing SBT.
Sedating this patient with propofol will halt the weaning process. The goal is to allow the
patient to stay alert and maintain spontaneous respirations. Propofol should only be
adminsited when medically appropriate.

Go to the next page if you knew the correct answer, or click the link image(s) below to further
research the concepts in this question (if desired).

Research Concepts:
Pressure Support

We update eBooks quarterly and Apps daily based on user feedback. Please tap flag to
report any questions that need improvement.
Question 610: A 68-year-old man is admitted to the hospital with complaints of fever and
chills for 1 week. His past medical history is significant for type 2 diabetes mellitus,
hypertension, chronic urinary retention with chronic indwelling Foley catheter, and obesity.
Upon presentation, the patient's heart rate is 106/min, blood pressure 72/56 mmHg, temperature
102 F, and SpO2 92% on room air. His systemic examination reveals a drowsy person, arousable
to touch and to follow commands. Other systems are unremarkable. Initial lab tests revealed a
WBC count of 18000/microL and elevated CRP. Urinary analysis reveals pyuria, hematuria,
many bacteria, with positive leukocyte esterase and positive nitrites. The patient is given broad-
spectrum antibiotics, norepinephrine, and admitted to the intensive care unit. Which of the
following findings is most likely to be appreciated on right heart catheterization in this patient?

Choices:
1. Elevated pulmonary capillary wedge pressure
2. Normal wedge pressure
3. Elevated right atrial pressure
4. Elevated pulmonary artery pressure
Answer: 2 - Normal wedge pressure
Explanations:
This patient most likely has septic shock secondary to a urinary tract infection (UTI).
The chronic urinary retention and indwelling Foley catheter are common risk factors for a
UTI. The urinary analysis also reveals signs of infection.
In patients with septic shock, the wedge pressure will be normal as the left ventricular
function is uncompromised.
In patients with septic shock, the right atrial pressure or CVP can be used to assess the need
for fluid resuscitation and responsiveness to treatment.

Go to the next page if you knew the correct answer, or click the link image(s) below to further
research the concepts in this question (if desired).

Research Concepts:
Pulmonary Capillary Wedge Pressure

We update eBooks quarterly and Apps daily based on user feedback. Please tap flag to
report any questions that need improvement.
Question 611: An 87-year-old woman is placed on ventilator support after a cardiac arrest
and subsequent successful resuscitation. The patient's family has gathered to discuss palliative
extubation. The primary provider reviews the patient's condition and options. The patient does
not have an advanced directive or a medical power of attorney. During the discussion, several
family members begin arguing about the decision. Which of the following is the most
appropriate tool to aid with de-escalation?

Choices:
1. Emphasizing the importance of focusing on what the patient would want
2. Restating the provider's medical opinion to guide decision making
3. Stating that the closest living relative to the patient will make the decision
4. Leaving the situation until a decision is made
Answer: 1 - Emphasizing the importance of focusing on what the patient would want
Explanations:
Providers have a responsibility to help mediate and advocate for their patients, and shared
decision-making with mediation is the ideal goal of discussing difficult medical decisions
with family members.
Often emphasizing that this decision should be made with the patient's wishes in mind can
help the patient's family members work together towards a common goal.
While providing medical information is important, restating or trying to enforce a provider's
preference is not appropriate in this situation.
A patient's closest living relative may potentially be used if there are difficulties in agreeing
with a decision. However, this option is usually used only after attempting to settle
differences in opinion to arrive at a shared decision. While it may be appropriate to give
family members space to help their decision, it is not appropriate to be absent from the
decision-making process in this situation.

Go to the next page if you knew the correct answer, or click the link image(s) below to further
research the concepts in this question (if desired).

Research Concepts:
Refusal of Care

We update eBooks quarterly and Apps daily based on user feedback. Please tap flag to
report any questions that need improvement.
Question 612: A 34-year-old man with a past medical history of hypertension and diabetes
is admitted to ICU for acute respiratory distress. He states that he had a cough, nausea, and
vomiting last week. Vital signs show blood pressure 164/86 mmHg, pulse 88/min, respiratory
rate 25/min, and SpO2 90% on room air. Labs show WBC count within normal limits, decreased
lymphocytes, elevated ESR, elevated CRP, elevated LDH, and elevated ferritin. The patient is
treated for possible coronavirus pneumonia. Overnight, the patient becomes confused and
hypoxic with SpO2 55% and is intubated. Vent settings are tidal volume 370 mL, PEEP 15,
respiratory rate 28/min, and FiO2 80%. Three days later, his creatinine increases from initial 1.0
mg/dL on admission to 6.9 mg/dL with potassium 6.0 mEq/L. A hemodialysis catheter is
attempted in the right internal jugular vein. During the procedure, the patient starts to desaturate
to 82% on 100% FiO2 on the vent. The procedure is aborted. Current vital signs show blood
pressure 87/69 mmHg and pulse 147/min. The trachea appears to deviate, the jugular veins are
distended, and he has hyperresonant sounds on the right side of the chest. What is the next best
step in the management of this patient?

Choices:
1. Place the patient in the prone position
2. Needle decompression
3. Decrease PEEP
4. Start vasopressors
Answer: 2 - Needle decompression
Explanations:
The patient's hypoxia is due to tension pneumothorax, most likely secondary to central
venous catheter placement in the internal jugular.
Pneumothorax is more commonly caused by central venous catheter placements in the
internal jugular or subclavian as compared to barotrauma.
Needle compression is the best management. Tension pneumothorax, which is a life-
threatening emergency. Needle compression must be done immediately. Otherwise, the
patient can end up in cardiac arrest or death.
Putting the patient in the prone position would be a treatment for acute respiratory distress
syndrome. Low PEEP is required for tension pneumothorax. However, tension
pneumothorax is a life-threatening emergency that must be treated immediately. High PEEP
will worsen the pneumothorax. The drop in blood pressure is due to the tension
pneumothorax. When the pressure in the pleural cavity increases, it causes a mechanical
obstruction by compressing on the superior vena cava and inferior vena cava, which
decreases venous return and ultimately decreases cardiac output. This causes a drop in
blood pressure. Vasopressors may keep blood pressure up, but the underlying problem
should be resolved.

Go to the next page if you knew the correct answer, or click the link image(s) below to further
research the concepts in this question (if desired).

Research Concepts:
Tension Pneumothorax

We update eBooks quarterly and Apps daily based on user feedback. Please tap flag to
report any questions that need improvement.
Question 613: A 39-year-old woman was brought in by paramedics with a gunshot wound
in the abdomen. The patient was intubated and had an exploratory laparotomy that showed a
small bowel injury with intraperitoneal hemorrhage, which was repaired and contained.
Postoperatively, the patient had a central line placement for total parenteral nutrition
administration. Which of the following laboratory abnormality is associated with the patient's
nutrition source?

Choices:
1. Hyperglycemia
2. Hypocalcemia
3. Hypokalemia
4. Hyponatremia
Answer: 4 - Hyponatremia
Explanations:
Total parenteral nutrition (TPN) is mainly associated with hyponatremia.
Other complications are hyperglycemia and low calcium or potassium.
TPN requires a dedicated central line. One of the most common complications of the central
line is an infection of the catheter, which can cause sepsis.
Long-term TPN also has been linked to nonalcoholic fatty liver.

Go to the next page if you knew the correct answer, or click the link image(s) below to further
research the concepts in this question (if desired).

Research Concepts:
Total Parenteral Nutrition

We update eBooks quarterly and Apps daily based on user feedback. Please tap flag to
report any questions that need improvement.
Question 614: A 25-year-old man presents to the emergency department with complaints of
anorexia, fatigue, and progressive dyspnea with decreased exercise capacity. Three days back, he
developed a fever and cough with mucopurulent expectoration. He denies a prior history of
cardiac or pulmonary disease. On admission, he is in acute pulmonary edema with respiratory
failure and acidosis. Chest x-ray reveals cardiomegaly with heterogenous diffuse infiltrates
sparing lung bases and upper lobes. He is intubated, artificially ventilated, and prescribed IV
antibiotics and diuretics. Serum BNP level is elevated. Blood and sputum cultures are negative.
He recovers with treatment and is extubated on the 5th day. Echocardiography reveals thickening
and trabeculations of the posterior and lateral wall of the left ventricle. The left ventricle is
dilated with global hypokinesia and moderately reduced ejection fraction. What is the most likely
cause of the patient's symptoms?

Choices:
1. Lower respiratory tract infection
2. Heart failure
3. Coronary artery disease
4. Interstitial lung disease
Answer: 2 - Heart failure
Explanations:
LV noncompaction is considered congenital cardiomyopathy caused due to intrauterine
arrest of myocardial development.
Although the left ventricle is commonly affected, the involvement of both ventricles
involvement occurs in 22% - 38% of patients.
The most common presentation is left ventricular dysfunction, which can progress to heart
failure.
Treatment is largely nonspecific, mainly dependent on clinical manifestations.

Go to the next page if you knew the correct answer, or click the link image(s) below to further
research the concepts in this question (if desired).

Research Concepts:
Left Ventricular Non-compaction Cardiomyopathy

We update eBooks quarterly and Apps daily based on user feedback. Please tap flag to
report any questions that need improvement.
Question 615: A 76-year-old woman with a history of chronic obstructive pulmonary
disease (COPD) on home oxygen at 1 L/min presents to the emergency room with dyspnea,
chills, and dry cough. On physical examination, she is afebrile and tachypneic, and her oxygen
saturation is 95% on 3 L of oxygen. Her pulmonary examination is within normal limits. The
chest radiograph reveals bilateral lower lobe consolidation. The patient is admitted and
diagnosed with community-acquired pneumonia and a COPD exacerbation. On the morning
following admission, she is found to be minimally arousable. The on-call provider notes that her
oxygen saturation is 98% and that she is receiving oxygen by nasal cannula at a rate of 4 L/min.
Which of the following should be done next?

Choices:
1. Increase nasal cannula oxygen flow from 4 L to 8 L
2. Place the patient on a non-rebreather oxygen mask
3. Transfer patient to ICU and attempt non-invasive positive pressure ventilation and draw
arterial blood gas
4. Intubate immediately
Answer: 3 - Transfer patient to ICU and attempt non-invasive positive pressure ventilation
and draw arterial blood gas

Explanations:
Target O2 saturation in patients with COPD should be between 88%-92%, as higher
saturations may cause oxygen-induced hypercapnia.
The pathophysiology of COPD is associated with airway outflow obstruction.
Ideal ventilator settings in a patient with COPD are assist-control or synchronized
intermittent mandatory ventilation (SIMV). Low inspiratory-expiratory (I:E) ratio, thus
allowing more time for expiration to avoid auto-peep.
High flow oxygen is relatively contraindicated in COPD exacerbation. The increased
oxygen levels lead to hypercapnia by inhibiting hypoxic vasoconstriction and increasing
alveolar dead space.

Go to the next page if you knew the correct answer, or click the link image(s) below to further
research the concepts in this question (if desired).

Research Concepts:
Oxygen Toxicity

We update eBooks quarterly and Apps daily based on user feedback. Please tap flag to
report any questions that need improvement.
Question 616: A 23-year-old man is found unconscious by the riverside. CPR is done by
the rescue service, and the patient is shifted to the emergency department. At that time of
presentation in the emergency department, his blood pressure is 60/10 mmHg, and his pulse rate
is 30/min. His EKG shows progressive type 2 atrioventricular block. Urgent external cardiac
pacing is planned to save his life. Which of the following is the most appropriate location for the
placement of negative leads in this patient?

Choices:
1. Left parasternal region
2. Right parasternal region
3. Left subscapular area
4. Right subscapular area
Answer: 1 - Left parasternal region
Explanations:
Multiple variations of pacer pad placement have been shown to be equally effective. Pacing
pads are positioned on the patient's chest, either in the anterolateral position of
anteroposterior one.
The negative electrode is usually placed in the left parasternal region.
The negative electrode can also be positioned at the cardiac apex.
The positive electrode can be placed in the left subscapular region or right parasternal
region.

Go to the next page if you knew the correct answer, or click the link image(s) below to further
research the concepts in this question (if desired).

Research Concepts:
External Pacemaker

We update eBooks quarterly and Apps daily based on user feedback. Please tap flag to
report any questions that need improvement.
Question 617: A 16-year-old girl is brought to the hospital with complaints of facial
rigidity, muscle spasms, a painful abdomen, and headaches for one week. Recently, she has
started feeling palpitations, loss of bladder control, and fecal incontinence. On examination, her
blood pressure and heart rate fluctuate three times in thirty minutes. She has a bandage on her
right heel, which her mother says is from a piercing wound she got from a rusty nail. Her ECG
reveals supraventricular tachycardia. She is given benzodiazepines for rigidity. Which of the
following additional therapies is most likely to improve the patient's symptoms?

Choices:
1. IV propofol
2. IV dantrolene
3. IV magnesium
4. IV calcium gluconate
Answer: 3 - IV magnesium
Explanations:
The clinical features of tetanus include lockjaw, grimace facial expression (risus
sardonicus), generalized muscle spasms associated with severe pains, drooling, uncontrolled
urination and defecation, and back arching spasm (opisthotonus) that may cause respiratory
distress.
With the loss of autonomic control, patients can present with labile blood pressure and heart
rate, diaphoresis, bradyarrhythmias, tachyarrhythmias, and cardiac arrest.
Benzodiazepines are considered a cornerstone therapy for tetanus manifestations, and
diazepam is the most frequently studied and utilized drug in this regard. It not only reduces
anxiety but also causes sedation and relaxes muscles, thereby preventing lethal respiratory
complications.
Intravenous magnesium has been shown to prevent muscle spasms. Clinicians must also
provide supportive care, especially for patients with autonomic instability (labile blood
pressure, hyperpyrexia, hypothermia). Magnesium is often used in combination with
benzodiazepines to manage these complications.

Go to the next page if you knew the correct answer, or click the link image(s) below to further
research the concepts in this question (if desired).

Research Concepts:
Tetanus

We update eBooks quarterly and Apps daily based on user feedback. Please tap flag to
report any questions that need improvement.
Question 618: A patient initially notices double vision after eating supper. Shortly
thereafter, she starts to experience weakness in arms and legs, difficulty with swallowing,
extreme trouble focusing her vision, and some minor breathing difficulties. 911 is called and she
is brought to the ER. The clinician notices bilateral drooping of the eyelids and dilated fixed
pupils. Her respiratory status has become more fragile. Which of the following tests is most
likely to be helpful in making the diagnosis?

Choices:
1. Erythrocyte sedimentation rates
2. Bioassays
3. Cerebrospinal fluid analysis
4. WBC counts
Answer: 2 - Bioassays
Explanations:
Botulism may be confirmed with a mouse neutralization bioassay.
This assay identifies the toxin.
Routine laboratory testing is generally not helpful in the diagnosis of botulism.
CSF analysis may show slight elevations of certain proteins.

Go to the next page if you knew the correct answer, or click the link image(s) below to further
research the concepts in this question (if desired).

Research Concepts:
Botulism

We update eBooks quarterly and Apps daily based on user feedback. Please tap flag to
report any questions that need improvement.
Question 619: A 27-year old male patient was involved in a motor vehicle accident. He is
alert at the scene but unable to move his legs. He is taken to the emergency department with a
Glasgow coma scale (GCS) of 14. A quick survey reveals a pelvic fracture and urethral injury. A
total body CT scan does not show any other injury. He has an external fixator placed, and the
urologist has been called regarding a suprapubic catheter. Seven hours later, the patient’s mental
status changes, and his GCS drops to 8. He is intubated. What should be ordered to search for his
sudden changes in mental status?

Choices:
1. Pulmonary angiography
2. Echocardiogram
3. Diagnostic peritoneal lavage
4. CT chest
Answer: 2 - Echocardiogram
Explanations:
In patients who suffer fat embolism syndrome, the fat particles travel in the veins, chiefly to
the lungs.
In about 20% of patients, there is a patent foramen ovale, which can allow the fat emboli to
cross the pulmonary circulation into the systemic circulation.
In a patient with fat embolism syndrome who develops acute central nervous system
changes, one should get an echocardiogram.
CT scan will show marked edema and white matter changes. The pulmonary angiogram is
indicated in pulmonary embolism. Diagnostic peritoneal lavage is indicated in patients with
blunt trauma abdomen with suspected hemoperitoneum which is less likely in the patient
owing to his normal initial CT scans.

Go to the next page if you knew the correct answer, or click the link image(s) below to further
research the concepts in this question (if desired).

Research Concepts:
Fat Embolism

We update eBooks quarterly and Apps daily based on user feedback. Please tap flag to
report any questions that need improvement.
Question 620: A 16-year-old Hispanic male with a past medical history of status
epilepticus and asthma is rushed to Emergency Room with suspected drug overdose after
girlfriend came home and found a suicide note and an empty bottle of his prescription pills next
to the unconscious body. When asked about any change in behavior leading up to the suicide
attempt, the girlfriend said in the past couple of weeks she noticed his speech was slurred, he
would stumble frequently, lose balance on level terrain, and complained about feeling sleepy.
Vitals: Blood pressure 109/65, Heart Rate 45, Temperature 98.6 F, Respiration 6 breaths/minute.
Physical exam: No needle track marks or visible wounds on the body. Cardio S1 S2 no murmurs,
lung exam exhibited shallow breaths bilateral, and the abdominal exam was nonsignificant. The
pupils are of normal size and reactive to light. Intubation was initiated and the patient started on
intravenous fluids. Based on the girlfriend's interview and the patient's physical exam, which
suspected drug of abuse could be found unchanged in the patient's urine?

Choices:
1. Phenobarbital
2. Amobarbital
3. Thiopental
4. Butalbital
Answer: 1 - Phenobarbital
Explanations:
Phenobarbital also has a very long half-life and can be found up to 15 days later in the
urine. Most barbiturates are fast acting, and their presence in the urine indicates that the
patient likely ingested the drug sometime within the past 3 days.
Amobarbital is an intermediate-acting barbiturate and is not typically used for status
epilepticus.
Thiopental is a short-acting barbiturate and is not typically used for status epilepticus.
Butalbital is commonly used in combination with other medications for the treatment of
headaches.

Go to the next page if you knew the correct answer, or click the link image(s) below to further
research the concepts in this question (if desired).

Research Concepts:
Phenobarbital

We update eBooks quarterly and Apps daily based on user feedback. Please tap flag to
report any questions that need improvement.
Question 621: A 56-year-old man taking anticoagulants presents to the emergency
department with black tarry stools, a pulse of 130/minute and blood pressure of 60/30 mmHg.
Nurses are unable to establish an intravenous peripheral catheter. An interosseous needle is
placed in the right medial proximal tibia, and O positive blood is being transfused. After 30
minutes, there is no change in the vital signs of the patient, and the flow rate through the IO is
very slow. What is the next best step to manage the low flow rates?

Choices:
1. Apply a pressure bag to the blood
2. Choose a new IO site
3. Remove the blood tubing and advance the IO needle
4. Abandon the site and start an ultrasound-guided central venous catheter
Answer: 1 - Apply a pressure bag to the blood
Explanations:
A pressure bag is needed to ensure adequate flow rates on all IO infusions. Always rapidly
flush 10 ccs of saline in the adult patient and 2 ccs to 5 cc saline in neonates and children
before injection and infusion.
Aspiration of marrow confirms the correct placement of the IO needle.
The sternal IO site has been documented to have the highest flow rates.
Two IO needles can be placed, but not in the same extremity.

Go to the next page if you knew the correct answer, or click the link image(s) below to further
research the concepts in this question (if desired).

Research Concepts:
Intraosseous Vascular Access

We update eBooks quarterly and Apps daily based on user feedback. Please tap flag to
report any questions that need improvement.
Question 622: A 35-year-old male patient was admitted to the emergency department (ED)
due to the recent onset of respiratory distress. The vital signs on admission were blood pressure:
98/60 mmHg, pulse rate: 100 beats per minute, respiratory rate: 25 breath per minute, and
temperature: 38.8 C. The chest x-ray revealed a large air-fluid level lesion in the right upper lobe.
Following initial medical therapy, the preferred surgical management was selected. What is the
preferred airway management during surgery?

Choices:
1. Direct laryngoscopy following double-lumen tube (DLT) insertion
2. Utilization 39 french double-lumen tube
3. Advance the DLT with the tip of the bronchial concave curve facing posteriorly
4. Positioning the DLT with rigid bronchoscopy
Answer: 2 - Utilization 39 french double-lumen tube
Explanations:
The Robertshaw double-lumen tubes have one lumen in the trachea and one lumen in the
mainstem bronchus. They have D-shaped lumens. R-sided tubes have an endobronchial cuff
that is donut-shaped, which allows separate access to the right upper lobe.
Fiberoptic bronchoscopy is the gold standard for confirmation of the correct placement of a
DLT. Auscultation alone is unreliable as an indicator of proper DLT placement.
To initiate one-lung ventilation, the bronchial cuff is inflated, the lung to be isolated is
clamped off at the corresponding connector, the connector is then opened to the atmosphere
to allow lung collapse. Lung collapse is most rapid if initiated at end-expiration.
For blind insertion of a DLT, first, perform direct laryngoscopy and visualize the vocal
cords. Visualization of the vocal cords is important as these tubes are large and placement
more challenging than placing an SLT. Once the vocal cords are visualized, gently advance
the DLT with the tip of the bronchial concave curve facing anteriorly through the vocal
cords until the bronchial cuff passes through the cords. The DLT can also be positioned
using fiberoptic bronchoscopy. Using a fiberoptic bronchoscope through the bronchial
lumen and guiding the DLT over fiber-optic scope increases the accuracy of placement.

Go to the next page if you knew the correct answer, or click the link image(s) below to further
research the concepts in this question (if desired).

Research Concepts:
Double Lumen Endobronchial Tubes

We update eBooks quarterly and Apps daily based on user feedback. Please tap flag to
report any questions that need improvement.
Question 623: A 23-year-old female patient presents to the emergency department after her
roommate found her lying on their living room floor unconscious. Her blood pressure is 100/70
mmHg, pulse 60/min, respirations 6/min, and temperature 98.0 F (36.7 C). The patient is unable
to cooperate with a physical examination. Her pupils are contracted bilaterally. A workup shows
the patient took an overdose of a medication that increases the duration of chloride channel
opening. What is the most appropriate next step in the management of this patient?

Choices:
1. Flumazenil
2. Alkalinize the urine
3. Activated charcoal
4. Naloxone
Answer: 3 - Activated charcoal
Explanations:
Barbiturates are a class of sedative-hypnotic drugs. They are commonly used as
antiepileptics (phenobarbital) and for the induction of general anesthesia (thiopental). Some
states administer barbiturates for provider-assisted suicide/euthanasia and use them for
capital punishment by lethal injection.
Barbituates potentiate GABA-A receptors and increase the duration of chloride channel
opening. Even though barbiturates mediate their effects via GABA, flumazenil is not
effective in the treatment of barbiturate toxicity.
Barbiturates have no antidotes, and the treatment is supportive. Activated charcoal is
beneficial in the management of a barbiturate overdose. However, alkalizing the urine has
not been shown to be effective.
Treatment of barbiturate toxicity remains supportive as there is no specific antidote or
overdose. The first step in treatment, as with any overdose, is assessing the patient’s airway,
breathing, and circulation. With significant sedation and respiratory depression, intubation
and mechanical ventilation may become necessary. Early treatment with activated charcoal
may be useful and can be given via a nasogastric tube.

Go to the next page if you knew the correct answer, or click the link image(s) below to further
research the concepts in this question (if desired).

Research Concepts:
Barbiturate Toxicity

We update eBooks quarterly and Apps daily based on user feedback. Please tap flag to
report any questions that need improvement.
Question 624: A 59-year-old man with a past medical history of heart failure with reduced
ejection fraction, chronic obstructive pulmonary disease, ischemic coronary heart disease, and
essential hypertension is seen in the intensive-care unit for respiratory distress with acute
respiratory distress syndrome (ARDS). The patient's clinical condition has been progressively
deteriorating. He is intubated and sedated. Vital signs include blood pressure 90/46 mmHg, heart
rate 118/min, and temperature 37.5 C (99.5 F). The patient's hypoxia has been refractory to
multiple ventilator settings. Current ventilator settings include FiO2 100%, tidal volume 460 mL,
PEEP 8 cmH2O, plateau pressure 30 cmH2O, and rate 28 breaths/minute. His oxygen saturation
is measured at 78% and drops to 75% on a subsequent reading 2 hours later. Which of the
following is the next best step in the management of this patient's hypoxia?

Choices:
1. Decrease PEEP, increase plateau pressure
2. Establish extracorporeal membranous oxygenation (ECMO)
3. Increase respiratory rate
4. Increase PEEP, decrease plateau pressure
Answer: 2 - Establish extracorporeal membranous oxygenation (ECMO)
Explanations:
For cases of refractory hypoxemia and hypoxia in ARDS, ECMO may be the only option to
improve oxygenation. This modality is considered a last resort.
Recently, extracorporeal membrane oxygenation (ECMO) has been advocated as salvage
therapy in refractory hypoxemic ARDS.
Despite advances in critical care, ARDS still has high morbidity and mortality. Even those
who survive can have a poorer quality of life.
Besides the restriction of fluids in high-risk patients, close monitoring for hypoxia by the
team is vital. The earlier the hypoxia is identified, the better the outcome.

Go to the next page if you knew the correct answer, or click the link image(s) below to further
research the concepts in this question (if desired).

Research Concepts:
Acute Respiratory Distress Syndrome

We update eBooks quarterly and Apps daily based on user feedback. Please tap flag to
report any questions that need improvement.
Question 625: A 34-year-old woman is admitted to labor and delivery for active labor. She
has no significant past medical history and only takes a prenatal vitamin. The patient undergoes
successful epidural catheterization on the second attempt, and neuraxial analgesia is provided
with significant pain relief. Following normal spontaneous vaginal delivery of a healthy female
infant, the patient is able to urinate and ambulate, and her catheter is removed on post-partum
day 1. She is discharged home the next day, but returns on post-partum day 5 with thoracolumbar
back pain, bilateral lower extremity numbness, and gait difficulty. What is the most likely cause
of the patient's new symptoms?

Choices:
1. Fibrocartilaginous embolism with spinal cord infarction
2. Aortic dissection with occlusion of the radicular arteries
3. Spinal epidural hematoma
4. Large central disc herniation with cauda equina syndrome
Answer: 3 - Spinal epidural hematoma
Explanations:
Spinal epidural hematoma is a rare complication of epidural catheterization for neuraxial
analgesia. It is more likely to occur in patients with a history of coagulopathy or
thrombocytopenia. However, in rare cases, it can occur in a patient without identifiable risk
factors.
Bleeding from spinal epidural hematoma typically occurs from a venous source following
an inadvertent dural puncture, and symptoms occur over the course of days following the
procedure.
MRI is the standard diagnostic exam for spinal epidural hematoma.
Treatment of spinal epidural hematoma involves urgent decompressive laminectomy with
the evacuation of blood products, particularly in the setting of new neurologic deficits.

Go to the next page if you knew the correct answer, or click the link image(s) below to further
research the concepts in this question (if desired).

Research Concepts:
Spinal Cord Compression

We update eBooks quarterly and Apps daily based on user feedback. Please tap flag to
report any questions that need improvement.
Question 626: A 36-year-old gentleman underwent right donor hepatectomy. His
comorbidity includes G6PD deficiency. The procedure was uneventful, and he was transferred to
ICU for monitoring. On a postoperative day 6, he has intractable hiccups, his temperature is
101.5 F, heart rate of 110/min, blood pressures of 90/60 mm Hg. Laboratory investigations
reveal a total count of 38,390 cells/cumm. What is the next investigation of choice?

Choices:
1. X-ray erect abdomen
2. MRI abdomen
3. CT abdomen
4. Ultrasound abdomen
Answer: 4 - Ultrasound abdomen
Explanations:
Ultrasound is the gold standard investigation in the detection of a right-sided subphrenic
abscess.
Ultrasound can also be used in the percutaneous aspiration of the collection and insertion of
a drain.
Ultrasound may sometimes show debris and gas, which will help to identify if it is
contaminated.
CT scan of the abdomen may be used to detect left subdiaphragmatic collections. It helps in
quantification and source of the collection. Contrast enhancement will suggest an abscess as
air and fluid collection in the abdomen is common post-surgery.

Go to the next page if you knew the correct answer, or click the link image(s) below to further
research the concepts in this question (if desired).

Research Concepts:
Subphrenic Abscess

We update eBooks quarterly and Apps daily based on user feedback. Please tap flag to
report any questions that need improvement.
Question 627: A 61-year-old male with a history of diabetes mellitus type II, coronary
artery disease, and chronic kidney disease presented to the emergency department with chest
pain. A coronary angiogram is ordered, which is essentially unremarkable. The following day the
patient experienced tachycardia, diarrhea, and increased sweating. This change in status was
reported to the on-call clinician, who ordered the thyroid profile of the patient. Thyroid profile
shows elevated T3 and T4 and reduced TSH levels. The provider explains the patient is
exhibiting an uncommon thyrotoxic phenomenon. Which of the following was responsible for
the patient's condition?

Choices:
1. History of thyroid malignancy
2. Exposed to iodine-containing contrast medium
3. History of diabetes
4. History of chronic kidney disease
Answer: 2 - Exposed to iodine-containing contrast medium
Explanations:
The Jod Basedow effect is hyperthyroidism following administration of iodine either in diet
or in contrast medium. The phenomenon is iodine-induced hyperthyroidism. It can occur
when a patient with an endemic goiter is relocated to an abundant iodine region.
This should not be confused with the Wolff-Chaikoff effect, which exhibits a reduction in
thyroid hormone secondary to large iodine ingestion, an autoregulatory phenomenon.
The Jod Basedow effect can be seen with small increases of iodine in a diet previously
starved of iodine.
Jod-Basedow, in addition to being seen with iodine administration in the diet or iodinated
intravenous contrast, can also be seen after amiodarone or prolonged use of iodinated
antiseptic solutions. History of CKD is a predisposing factor; however, it is not the direct
cause of the patient's symptoms.

Go to the next page if you knew the correct answer, or click the link image(s) below to further
research the concepts in this question (if desired).

Research Concepts:
Jod Basedow Syndrome

We update eBooks quarterly and Apps daily based on user feedback. Please tap flag to
report any questions that need improvement.
Question 628: A 90-year-old, 40 kg woman presents to the emergency department with
severe lower back, bilateral hip, and knee pain. She declines opioids but makes multiple requests
for topical lidocaine patches. An hour after placing a total of 4 patches of 5% topical lidocaine
patches in the painful areas, she becomes progressively somnolent and hypotensive, requiring
vasopressor support, and proceeds to cardiac arrest. Unfortunately, 20% intravenous lipid
emulsion is not available, though the pharmacy did recently acquire a novel lipid formation of
mixed chain fatty acids for parenteral nutrition therapy. What is the next best step in the
management of this patient?

Choices:
1. Initiate advanced cardiovascular life support (ACLS)
2. Administer propofol as an intravenous lipid emulsion (ILE) alternative
3. Prepare for extracorporeal membrane oxygenation (ECMO)
4. Administer the new lipid formula
Answer: 1 - Initiate advanced cardiovascular life support (ACLS)
Explanations:
ACLS is the appropriate initial management for local anesthetic systemic toxicity (LAST)-
induced cardiac arrest.
The 10% lipid emulsion used in propofol is never an acceptable intravenous lipid emulsion
(ILE) alternative.
ECMO may require additional personnel and setup, which should not delay ACLS and lipid
emulsion therapy.
100% long-chain fatty acid formulations of lipid emulsions are recommended at this time.

Go to the next page if you knew the correct answer, or click the link image(s) below to further
research the concepts in this question (if desired).

Research Concepts:
Lipid Emulsion Therapy

We update eBooks quarterly and Apps daily based on user feedback. Please tap flag to
report any questions that need improvement.
Question 629: A 41-year-old man presents to the emergency department with a painful red
and purple rash which started 3 days ago and has been getting worse since onset. The patient
states that the rash started just as he was recovering from flu-like symptoms, which he had been
having for 2 weeks. The patient has several medications that he has been prescribed and is
currently taking. Which of the following medications is most likely responsible for the patient's
symptoms?\

Choices:
1. Metformin
2. Albuterol
3. Nevirapine
4. Aluminum hydroxide
Answer: 3 - Nevirapine
Explanations:
Stevens-Johnson syndrome is a rare and unpredictable reaction that can be caused by drugs
such as nevirapine, anti-gout medications (e.g., allopurinol), anticonvulsants,
antipsychotics, acetaminophen, and ibuprofen.
Risk factors for contracting Stevens-Johnson syndrome include an HIV infection, a
weakened immune system, and a family history of Stevens-Johnson syndrome.
When Stevens-Johnson syndrome onsets secondary to taking nevirapine, it may first present
with prodromal, flu-like symptoms such as fever, sore throat, fatigue, and cough.
After the consumption of nevirapine, the prodromal symptoms are followed by a painful red
or purple rash that blisters and spreads. Recovery time for this syndrome can take weeks to
months.

Go to the next page if you knew the correct answer, or click the link image(s) below to further
research the concepts in this question (if desired).

Research Concepts:
Nevirapine

We update eBooks quarterly and Apps daily based on user feedback. Please tap flag to
report any questions that need improvement.
Question 630: A 33-year-old woman has recently recovered from Guillain-Barre
syndrome, which began 13 days ago. She was intubated for five days and now has inspiratory
stridor, which is mild. On the fifth day post-extubation, she failed a swallow study as she was
aspirating thin liquids. She has been on a nectar-thick diet since then. Her voice is normal. Her
breathing becomes much more audible after climbing two flights of stairs, with frank biphasic
stridor but no subjective discomfort. She can eat and drink on a bedside swallow examination
everything except for plain water, where she coughs after swallowing. On flexible laryngoscopy,
her true vocal folds are in a median position with a glottic airway estimated at 3mm. What is the
next step in management?

Choices:
1. Maintain strict NPO and place a gastrostomy tube.
2. Urgent tracheostomy due to profound airway narrowing.
3. Careful observation and serial flexible laryngoscopy.
4. Posterior cordotomy to increase airway but minimize aspiration risk.
Answer: 3 - Careful observation and serial flexible laryngoscopy.
Explanations:
This patient has bilateral vocal fold paralysis. Guillain-Barre syndrome can rarely present as
isolated bilateral vocal fold paralysis without concurrent ascending muscle weakness.
As she has recovered and is largely asymptomatic, simple diet modification and close
observation are all that is warranted, as full recovery is expected (though it may be a
prolonged recovery course).
Permanent measures to augment the airway such as cordotomy or tracheostomy are
premature in this largely asymptomatic patient, as she has a very good chance of
spontaneous recovery.
Patients with bilateral vocal fold paralysis tend to have the vocal folds in a median position,
yielding a near-normal voice but potential stridor. This can be inspiratory, expiratory, or
biphasic. So long as they are comfortable breathing, expectant management is all that is
needed. If they are symptomatic, a reversible procedure such as a tracheostomy is preferred.

Go to the next page if you knew the correct answer, or click the link image(s) below to further
research the concepts in this question (if desired).

Research Concepts:
Airway Glottic Insufficiency

We update eBooks quarterly and Apps daily based on user feedback. Please tap flag to
report any questions that need improvement.
Question 631: A 59-year-old woman presents with left-sided weakness. The symptoms
started 30 mins ago when she was having lunch in a restaurant. The patient has a past medical
history of hypertension and type 2 diabetes mellitus. On examination, the patient is awake, alert,
able to follow commands, and is oriented to time, person and place. Cranial nerve examination
reveals preferential gaze to the right and dysarthria. The patient is unable to recognize left-sided
stimulus on double simultaneous stimulation. The motor examination shows 3/5 strength on the
left upper and left lower limb. There is a 20% sensory deficit on the left to pain and light touch.
Babinski sign is positive on the left. A head CT is performed. Which of the following do you
expect to see on this scan?

Choices:
1. Encephalomalacic changes in the right periventricular region
2. A small area of hyperdensity in the pons
3. Insular ribbon sign with obscuration of the lentiform nucleus
4. Hyperdensity at the Sylvian fissure and base of the skull
Answer: 3 - Insular ribbon sign with obscuration of the lentiform nucleus
Explanations:
Ischemic brain tissue stops working in seconds and suffers necrosis in as soon as 5 minutes
after complete lack of oxygen and glucose supply, prompting a myriad of potential
symptoms and disability.
Cerebral ischemia can be focal or generalized and cause varying clinical pictures.
While generalized cerebral ischemia is generally related to syncope and pre-syncope, focal
cerebral ischemia usually causes a stroke or transient ischemic attacks.
As a rule of thumb, acute cerebral ischemia is never painful.

Go to the next page if you knew the correct answer, or click the link image(s) below to further
research the concepts in this question (if desired).

Research Concepts:
Cerebral Ischemia

We update eBooks quarterly and Apps daily based on user feedback. Please tap flag to
report any questions that need improvement.
Question 632: A 55-year-old male patient with no significant past medical history aside
from excessive smoking presents with complaints of right lower extremity swelling and pain.
Imaging demonstrates a massive iliofemoral venous thrombosis. She undergoes catheter-directed
thrombolysis with tissue plasminogen activator (tPA), and the patient is being monitored closely
in the ICU. After several hours fibrinogen is checked and is found to be less than 100mg/dL. If
the infusion is not stopped, the patient is most at risk for which of the following complications?

Choices:
1. Myocardial rupture
2. Limb loss
3. Hypotension
4. Bleeding complications
Answer: 4 - Bleeding complications
Explanations:
The major and most feared complication of thrombolytic therapy is bleeding complications,
most notably intracranial hemorrhage. Other complications include systemic hemorrhage,
immunologic complications, hypotension, and myocardial rupture.
Fibrinogen levels need to be monitored closely in patients undergoing thrombolysis to
ensure they are above 150mg/dL. When levels drop below 150mg/dL, this poses a
significant bleeding risk. Treatment includes reducing the infusion rate and stopping the
infusion rate completely once below 100mg/dL. Platelets and hemoglobin levels should also
be monitored closely as well.
Hypotension is a known risk with "reperfusion"; however, generally, this can be controlled
with infusions and will ultimately resolve as the infusion is stopped and patient resuscitated
adequately.
Myocardial rupture is a known complication when tPA is used to treat patients who suffer
from a myocardial infarction but is less commonly associated with catheter-directed
thrombolysis for lower extremity deep vein thrombosis (DVT).

Go to the next page if you knew the correct answer, or click the link image(s) below to further
research the concepts in this question (if desired).

Research Concepts:
Phlegmasia Alba And Cerulea Dolens

We update eBooks quarterly and Apps daily based on user feedback. Please tap flag to
report any questions that need improvement.
Question 633: A 32-year-old man is brought to the emergency department after his car
engine explodes in a motor vehicle collision. The total body burn area is estimated to be around
18%. On presentation, the patient is in pain and somewhat disoriented. The Glasgow Coma Scale
(GCS) score is 14/15. Nasoendoscopy shows swollen upper respiratory tract and slightly
edematous vocal cords. Bronchoscopy shows mild erythema throughout and a normal lower
airway. After this type of injury, which long-term complication is most likely to develop in this
patient?

Choices:
1. Pneumonia
2. Atelectasis
3. Bronchiectasis
4. Pulmonary edema
Answer: 3 - Bronchiectasis
Explanations:
Long-term complications from smoke inhalation injury are much less common than short-
term complications. They include subglottic stenosis, bronchiectasis, bronchiolitis
obliterans, interstitial fibrosis, and reactive airways dysfunction syndrome (RADS).
Short-term complications are seen in more severe injuries within 4 to 5 days, and the most
common complication is pneumonia. Acute respiratory distress syndrome and pulmonary
edema are also seen in the short term.
These patients will often demonstrate changes in pulmonary function testing and may
require ventilatory support.
Complication rates are higher in patients with a history of an underlying lung disease, such
as chronic obstructive pulmonary disease (COPD) or asthma.

Go to the next page if you knew the correct answer, or click the link image(s) below to further
research the concepts in this question (if desired).

Research Concepts:
Inhalation Injury

We update eBooks quarterly and Apps daily based on user feedback. Please tap flag to
report any questions that need improvement.
Question 634: A 68-year-old man presents to the clinic from a nursing home with
worsening dyspnea for three months. He was diagnosed with subacute pulmonary embolism
three months ago and has been on anticoagulation therapy with minimal symptom improvement.
CT pulmonary angiography at that time showed extensive distal emboli. His past medical history
is significant for hypertension, diabetes mellitus, and cerebrovascular accident with residual left-
sided weakness. Vital signs are a blood pressure of 146/88 mmHg, pulse 88/minute, and SpO2
94% on room air. Electrocardiogram (ECG) and chest x-ray are unremarkable. An
echocardiogram reveals a mean pulmonary artery pressure of 35 mmHg. What is the best initial
treatment for this patient?

Choices:
1. Pulmonary endarterectomy
2. Bosentan and sildenafil
3. Epoprostenol
4. Riociguat
Answer: 4 - Riociguat
Explanations:
Medical treatment of chronic thromboembolic pulmonary hypertension (CTEPH) is based
primarily on PAH-targeted therapy, including endothelin receptor antagonists,
phosphodiesterase type-5 inhibitors, and prostanoids.
Riociguat, a guanylate cyclase stimulator, is used primarily in patients ineligible for surgery
or persistent disease after pulseless electrical activity (PEA).
Treprostinil and macitentan have also been extensively studied and proven to be beneficial
in these patients.
Pulmonary endarterectomy is the best treatment to improve hemodynamics and exercise
capacity in operable patients. This patient with limited functional capacity and distal emboli
is not considered operable. Bosentan and sildenafil are alternatives to riociguat, but
riociguat is preferred in mild to moderate disease. IV epoprostenol therapy is only
considered in patients who are severely ill.

Go to the next page if you knew the correct answer, or click the link image(s) below to further
research the concepts in this question (if desired).

Research Concepts:
Chronic Thrombo-Embolic Pulmonary Hypertension

We update eBooks quarterly and Apps daily based on user feedback. Please tap flag to
report any questions that need improvement.
Question 635: A 38-year-old man presents to the emergency department with
lightheadedness and dizziness. His physical examination shows a blood pressure of 78/40
mmHg. He is given 2 liters of Ringer's lactate (RL) and is admitted for dehydration. All of his
other labs are within normal limits. At night, a rapid response is called as the patient is confused.
He is shifted to the intensive care unit and started on dexmedetomidine for a possible diagnosis
of delirium tremens (DTs). His condition gets worse, and he is sedated with propofol and is
intubated. An orogastric tube is also inserted, and the patient is started on tube feeds. For the next
two days, the patient remains in DTs. Overnight, he has a seizure, and his oxygen requirements
go up from 30% to 60%. On chest x-ray, he has bilateral basal haziness. His hemoglobin drops
from 10 g/dL to 6.8 g/dL. His creatinine is elevated from 1 mg/dL to 1.8 mg/dL. His CK levels
are 6000 U/L. Blood glucose level is 200 mg/dL. ABG shows a pH of 7.32, pCO2 of 48 mmHg,
bicarbonate of 24 mEq/L, and pO2 of 80 mmHg. What is the next best step in the management of
this patient?

Choices:
1. Continuous electroencephalogram
2. Check echocardiogram
3. Check the electrolytes panel
4. Check beta-hydroxybutyrate level
Answer: 3 - Check the electrolytes panel
Explanations:
The patient in the scenario has hypophosphatemia. He is at a very high risk of developing
electrolyte disturbances due to alcohol use disorder and poor nutritional status.
The effects of hypophosphatemia are broad and impact nearly every system. Effects
primarily are due to intracellular depletion; however, chronic effects can be seen in the bone
structures. Prolonged hypophosphatemia leads to osteopenia, osteoporosis, rickets, or
osteomalacia due to decreased bone mineralization. The central nervous system may
manifest with metabolic encephalopathy as a result of ATP depletion and may include
altered mental state, irritability, paresthesias, numbness, seizures, or coma.
Cardiac function is impacted by ATP depletion. In addition to possible systolic heart failure,
the myocytes become less stable, and arrhythmias are possible. The decreased
diaphragmatic function impacts pulmonary function with subsequent hypoventilation.
Ventilator dependent patients have been shown to have longer hospital courses and worse
outcomes when hypophosphatemia is present.
Gastrointestinal dysfunction occurs as a result of ATP deficiency, also with dysphagia or
ileus possible. Generalized muscular weakness can occur.

Go to the next page if you knew the correct answer, or click the link image(s) below to further
research the concepts in this question (if desired).

Research Concepts:
Hypophosphatemia

We update eBooks quarterly and Apps daily based on user feedback. Please tap flag to
report any questions that need improvement.
Question 636: A 60-year-old female was found in a semi-comatose condition and brought
to the emergency department. On physical exam, her height was 66 inches (167.6 cm), and her
weight was 123 lb (55.8 kg) (BMI=19.9 kg/m2). Her blood pressure was 80/40 mmHg; her pulse
rate was 150 beats/min with a temperature of 99 F (37 C). She was disoriented and lethargic with
normal deep tendon reflexes. Her home medications include aspirin and ketoconazole prescribed
for onychomycosis of her toenails 6 months ago.Laboratory test results demonstrate: hemoglobin
9 g/dl; hematocrit 25%; WBC count 9840 cells/mm3; platelet count 297,000 cells/mm3;
creatinine 1.09 mg/dl sodium 128 meq/l; potassium 5.2 mEq/L; chloride 93 mEq/L; and
bicarbonate 18 mEq/L. Chest radiography and CT brain didn’t reveal any abnormalities. Blood
and urine cultures were sent. The patient was started on intravenous fluid resuscitation with
broad-spectrum antibiotics. Which of the following is true regarding this medical condition?

Choices:
1. Ketoconazole is a precipitating factor
2. The patient can present with hypocalcemia
3. Hyperglycemia is a common manifestation
4. Lymphopenia is a common manifestation
Answer: 1 - Ketoconazole is a precipitating factor
Explanations:
The patient has an adrenal crisis precipitated by ketoconazole use. Manifestations include
hypotension, tachycardia, hyponatremia, hyperkalemia with confusion. Ketoconazole
interfere with steroidogenesis and clinicians should be aware that it can precipitate adrenal
crises
Confusion and lethargy are common manifestations of adrenal crises. The altered mental
status can be exacerbated in the setting of hyponatremia and hypoglycemia due to the
inhibition of gluconeogenesis. Slow correction of hyponatremia is required to avoid
precipitation of osmotic demyelination syndrome
Other medications that can precipitate adrenal crises are anti-adrenal medications, mitotane,
metyrapone, anticancer medications, immune checkpoint inhibitors, tyrosine kinase
inhibitors(sunitinib, imatinib), fluconazole, etomidate, rifampicin, cyproterone acetate,
diuretics, and megestrol acetate.
In adrenal crises, the patient usually presents with hypercalcemia secondary to dehydration,
lymphocytosis, and eosinophilia.

Go to the next page if you knew the correct answer, or click the link image(s) below to further
research the concepts in this question (if desired).

Research Concepts:
Adrenal Crisis

We update eBooks quarterly and Apps daily based on user feedback. Please tap flag to
report any questions that need improvement.
Question 637: A 70-year-old female with a prior history of diabetes and hypertension, who
is also a former smoker, is being evaluated for dyspnea. The patient underwent an
echocardiogram that revealed normal left ventricular ejection fraction. The patient was referred
for pulmonary artery catheterization as echocardiogram revealed moderately elevated pulmonary
artery pressures. At baseline, she has a heart rate of 78 beats per minute with a blood pressure of
148/70 mmHg. While performing right heart catheterization, the patient suddenly becomes
dyspneic and complains of chest pain. Her heart rate is now 115 beats per minute. The patient is
able to speak and tell you he is uncomfortable. The nurse checks the fluid-filled transducer and
finds a large air column that had not been completely evacuated. What is the next best step?

Choices:
1. Remove the pulmonary artery catheter immediately
2. Flush the transducer to evacuate the air from it
3. Place the patient in Trendelenburg position and administer oxygen
4. Intubate the patient
Answer: 3 - Place the patient in Trendelenburg position and administer oxygen
Explanations:
Removing the pulmonary catheter is not necessary as the suspected issue is air embolism
from the air column in the fluid system.
Flushing the air column is a necessary step before inserting the catheter into the vein and
also before proceeding again. However, as the patient has already experienced an air
embolism episode, this would not be the next best step.
Placing the patient in the Trendelenburg position and administering high flow oxygen
should be the next best step as it helps improve reabsorption of the injected air into the
systemic circulation.
The patient is still alert and oriented even though he is experiencing symptoms. If the
respiratory distress worsens and the patient is unable to protect her airways, intubation
should be considered. But intubation would not be the immediate management.

Go to the next page if you knew the correct answer, or click the link image(s) below to further
research the concepts in this question (if desired).

Research Concepts:
Right Heart Cardiac Catheterization

We update eBooks quarterly and Apps daily based on user feedback. Please tap flag to
report any questions that need improvement.
Question 638: A 28-year-old, previously healthy male presents with a three-day history of
fever, headache, nausea, and vomiting. There are nuchal rigidity and fever, but otherwise, the
exam and CT of the head are normal. Lumbar puncture shows mild elevation of protein,
lymphocytic pleocytosis, and normal glucose. Select the most likely diagnosis.

Choices:
1. Bacterial meningitis
2. Herpes simplex encephalitis
3. Herpes simplex meningitis
4. Enterovirus meningitis
Answer: 4 - Enterovirus meningitis
Explanations:
The clinical picture is that of viral meningitis although polymerase chain reactions and
cultures should be done.
Eighty-five percent of viral meningitis is caused by enteroviruses, usually Coxsackie and
echovirus.
In developed countries, herpes simplex is the next most common pathogen in adolescents
and adults.
Treatment with fluids and antipyretics is usually sufficient.

Go to the next page if you knew the correct answer, or click the link image(s) below to further
research the concepts in this question (if desired).

Research Concepts:
Viral Meningitis

We update eBooks quarterly and Apps daily based on user feedback. Please tap flag to
report any questions that need improvement.
Question 639: A 45-year-old male is evaluated in intensive care. He was admitted 7 days
back into with road traffic acceded and extensive injuries. He was intubated due to deteriorating
GCS. His condition has been worsening over the past several hours. His vitals reveal a blood
pressure of 80/40 mm Hg, a pulse of 135 beats per minute, spO2 of 95 %. He is on ventilatory
support with FiO2 of 0.5. His investigations reveal a total leukocyte count of 21,000 per microL,
hemoglobin of 10 gm/dl, and PaO2 95 %. A sample is obtained from his central line and is
subjected to blood gas analysis and reveals an oxygen saturation of 82 %. What is the likely
cause of this patient’s deterioration?

Choices:
1. Myocardial depression
2. Hypovolemia
3. Pulmonary embolism
4. Sepsis
Answer: 4 - Sepsis
Explanations:
This patient is in critical care and has been deteriorating with hypotension and tachycardia.
The laboratory investigations reveal a high WBC count and a normal arterial oxygen
saturation. The oxygen saturation from the blood obtained from the central line represents
central venous oxygen saturation. This is normally 65-70 %.
This patient’s oxygen extraction is only 15 %, representing a reduction in oxygen extraction
in the peripheral tissues. This can occur in individuals with blood redistribution, such as in
sepsis.
Sepsis leads to the micro-capillary obstruction, which leads to the shunting of arterial blood
across the capillary bed and thus leads to an increase in venous oxygen concentration.
Venous oxygen concentration can be used to guide therapy in these individuals.
Venous oxygen saturation can also be measure continuously. Mixed venous oxygen
saturation decreases in conditions such as heart failure, hypovolemia, and pulmonary
embolism. Increased extraction or decreased oxygenation of blood will lead to reduced
oxygen saturation in the venous system.

Go to the next page if you knew the correct answer, or click the link image(s) below to further
research the concepts in this question (if desired).

Research Concepts:
Venous Oxygen Saturation

We update eBooks quarterly and Apps daily based on user feedback. Please tap flag to
report any questions that need improvement.
Question 640: A 17-year-old male patient was admitted to the emergency department
following an abrupt onset striking pattern of chest pain 30 minutes prior to the admission. His
vitals were blood pressure: 68/40 mmHg, pulse rate: 110 beats per min, respiratory rate: 22 /min,
and normal temperature. He is 2.1 m (7 ft.) in height and 77 kg (170 lbs.). Following intravenous
fluid resuscitation with 1000 cc Ringer's lactate, his vitals were blood pressure: 69/50 mmHg,
pulse rate: 108 bpm, respiratory rate: 20/min. His ECG and troponin levels were unremarkable,
while his portable chest x-ray was significant for a wide mediastinum. What is the next preferred
management?

Choices:
1. Obtaining CT-angiography
2. Obtaining MR-angiography
3. Transfer to the operating room
4. Request transesophageal echocardiography in the emergency department
Answer: 3 - Transfer to the operating room
Explanations:
The above suggests a diagnosis of Marfan syndrome and dissection of the aorta. Patients
with a history of Marfan syndrome have a high risk of aortic complications that occur at a
young age. As the patient is unstable, he should be taken to the operating room where
transesophageal echocardiography (TEE) can be done. This is the modality of choice for
diagnosis.
TEE assesses both the aorta and the valves for pathology.
Currently, the preferred modality for the diagnosis of aortic dissection is intravenous
contrast CT-scan.
Although the sensitivity of chest CT-scan in obtaining the diagnosis of aortic dissection is
almost 100%, it is limited to those with stable hemodynamics.

Go to the next page if you knew the correct answer, or click the link image(s) below to further
research the concepts in this question (if desired).

Research Concepts:
Aortic Dissection

We update eBooks quarterly and Apps daily based on user feedback. Please tap flag to
report any questions that need improvement.
Question 641: An 82-year-old man with a history of hypertension, hyperlipidemia, and
chronic obstructive pulmonary disease is brought to the emergency department from his nursing
home after a medical assistant noticed a change in the patient's mental status. The nursing home
states that the patient is typically alert and oriented. Upon examination, the patient is obtunded
and is only aroused by painful stimulation. Accessory muscles of respiration are visibly
recruited. Vital signs upon arrival show blood pressure 86/48 mmHg, heart rate 118/min,
temperature 102.3 F, and respiratory rate 36/min. Chest radiography in the emergency
department reveals bilateral opacities in the lung fields and a normal cardiac silhouette. The
patient is intubated for airway protection. Which of the following modes of mechanical
ventilation is likely to provide the highest benefit to this patient?

Choices:
1. Tidal volume 6 mL/kg, plateau pressure > 45 cmH2O
2. Tidal volume 6 mL/kg, plateau pressure 30 cmH2O
3. Tidal volume 12 mL/kg, plateau pressure 30 cmH2O
4. Tidal volume 12 mL/kg, plateau pressure > 45 cmH2O
Answer: 2 - Tidal volume 6 mL/kg, plateau pressure 30 cmH2O
Explanations:
Studies support the use of low tidal volume and plateau pressure less than 30 cmH2O for
the management of patients with ARDS over other settings.
Patients are mechanically ventilated, guarded against fluid overload with diuretics, and
given nutritional support until evidence of improvement is observed.
The mode in which a patient is ventilated has an effect on lung recovery. Evidence suggests
that some ventilatory strategies can exacerbate alveolar damage and perpetuate lung injury
in the context of ARDS. Care is placed in preventing volutrauma (exposure to large tidal
volumes), barotrauma (exposure to high plateau pressures), and atelectrauma (exposure to
atelectasis)
A lung-protective ventilatory strategy is advocated to reduce lung injury. The NIH-NHLBI
ARDS Clinical Network Mechanical Ventilation Protocol (ARDSnet) sets the following
goals: Tidal volume (V) from 4 to 8 mL/kg of ideal body weight (IBW), respiratory rate
(RR) up to 35/min, SpO2 88% to 95%, plateau pressure (P) less than 30 cmH2O, pH goal
7.30 to 7.45, and inspiratory-to-expiratory time ratio less than 1. To maintain oxygenation,
ARDSnet recognizes the benefit of PEEP. The protocol allows for a low or a high PEEP
strategy relative to FiO2. Either strategy tolerates a PEEP of up to 24 cm HO in patients
requiring 100% FiO2. The inspiratory-to-expiratory time ratio goal may need to be
sacrificed and an inverse inspiratory-to-expiratory time ratio strategy instituted to improve
oxygenation in a certain clinical situation.

Go to the next page if you knew the correct answer, or click the link image(s) below to further
research the concepts in this question (if desired).

Research Concepts:
Acute Respiratory Distress Syndrome

We update eBooks quarterly and Apps daily based on user feedback. Please tap flag to
report any questions that need improvement.
Question 642: A 45-years-old man presents to the clinic for evaluation. He has noted more
fatigue, headaches, shortness of breath, and swelling in the legs over the past few weeks. He was
diagnosed with systemic sclerosis 1 year ago when he had Raynaud syndrome with digital ulcers,
thickening of the skin in the upper and lower extremities, joint pain and joint swelling in the
hands, heartburn, and dysphagia. He takes hydroxychloroquine, omeprazole, and amlodipine. His
Raynaud syndrome, heartburn, and joint pain improved, although the skin thickening and
tightening persist. His past medical history is otherwise unremarkable. Vital signs show blood
pressure 198/108 mmHg, pulse 102/min, and temperature 98 F. On exam, he has 2+ pitting
edema in the lower extremities. The skin of the entire bilateral upper and lower extremities and
the face is thick and tight. He does not have any active digital ulcerations. He is unable to make a
fist in both hands but denies any pain. The range of motion of bilateral wrists is limited with
palpable crepitus on the dorsal wrists with the range of motion. Cardiac and pulmonary exams
are normal. Labs reveal hemoglobin 9.1 g/dL and platelets 101000/microL. His CBC was normal
2 months ago. Which of the following is the most appropriate next step in the management of
this patient?

Choices:
1. Anti-centromere, anti-SCL-70 and anti-RNA-polymerase-III antibodies
2. C-reactive protein
3. Serum creatinine and urinalysis
4. Iron and total iron-binding capacity panel
Answer: 3 - Serum creatinine and urinalysis
Explanations:
Clinically, this patient has diffuse systemic sclerosis (DcSSc). Severe elevation in blood
pressure and new-onset anemia and thrombocytopenia suggest scleroderma renal crisis and
renal function with serum creatinine and urinalysis shall be performed immediately in this
patient.
Risk factors for scleroderma renal crisis include diffuse systemic sclerosis, African-
American race, pericardial effusion, tendon friction rubs, new-onset anemia, anti–RNA
polymerase III antibody and use of high dose (or chronic low-dose) corticosteroids.
Patients with a serum creatinine of more than 3.0 mg/dL have poor outcomes. Other factors
indicating poor outcome include male sex and older age of onset.
Immediate treatment with high-dose ACE-inhibitors is indicated and can improve outcomes
significantly.

Go to the next page if you knew the correct answer, or click the link image(s) below to further
research the concepts in this question (if desired).

Research Concepts:
Systemic Sclerosis

We update eBooks quarterly and Apps daily based on user feedback. Please tap flag to
report any questions that need improvement.
Question 643: An 82-year-old man is being evaluated in the intensive care unit. He was
admitted following an emergency aortic aneurysm repair 5 days ago. Despite his vital signs
being stable throughout the day, he has become increasingly confused and appears to be having
visual hallucinations. He has pulled out his nasogastric tube and an arterial line and is trying to
get out of bed, despite reassurance. Which of the following is the next best step in the
management of this patient?

Choices:
1. Intramuscular haloperidol
2. Intravenous lorazepam
3. Oxygen via a non-rebreathe mask
4. Physical restraints
Answer: 1 - Intramuscular haloperidol
Explanations:
Delirium in critically ill patients is a common occurrence. Delirium is associated with
increased mortality, infection rates, and increased length of hospital stay. The cause is
usually multifactorial.
The Society of Critical Care Medicine and the American Psychiatric Association
recommends haloperidol as the first-line drug of choice or olanzapine if haloperidol is not
tolerated.
Risk factors for delirium include old age, alcoholism, smoking, visual/ hearing impairment,
sepsis/pyrexia, sleep disturbance, immobilization, and medication.
Benzodiazepines should be avoided in delirium as they may worsen symptoms and be
ineffective.

Go to the next page if you knew the correct answer, or click the link image(s) below to further
research the concepts in this question (if desired).

Research Concepts:
Postoperative Delirium

We update eBooks quarterly and Apps daily based on user feedback. Please tap flag to
report any questions that need improvement.
Question 644: A 66-year-old male patient is diagnosed with non-STEMI and is treated with
low molecular weight heparin. The initial lab test is as follows: WBC:8000/µl, HCT:41%,
Plt:180,000/mm3.The daily physical examination on the fifth day of admission was significant
for swelling and tenderness in the passive movements of his left leg. The lab tests on the day 8
are as follows; WBC:9000/µl, HCT:39%, Plt:80,000/mm3, Total serum bilirubin: 2.5
mg/dl,AST: 250 U/L,ALT: 100 U/L,Urea: 80,and Creatinine: 2 mg/dl. What is the preferred
management?

Choices:
1. Stop heparin therapy and platelet transfusion
2. Emergent hemodialysis and platelet transfusion
3. Stop heparin therapy and the addition of argatroban
4. Start argatroban treatment and taper the heparin therapy within 5 following days
Answer: 3 - Stop heparin therapy and the addition of argatroban
Explanations:
Argatroban is an FDA approved agent for prophylaxis and treatment of thrombosis in
patients with HIT (heparin-induced thrombocytopenia and HITTS (heparin-induced
thrombocytopenia and thrombosis syndrome).
Argatroban doesn’t need dose adjustment in elderly patients, in patients with impaired renal
function or based on gender.
Argatroban is metabolized primarily by hepatic cytochrome P450 enzymes and excreted
mainly via feces. Argatroban should be used with caution in patients with hepatic
dysfunction and a lower dose should be used.
The treatment of choice in heparin-induced thrombocytopenia is stopping heparin and begin
an alternative anticoagulant. Stopping heparin without the addition of another anticoagulant
is not adequate to prevent thrombosis in this setting. Alternative anticoagulants are
primarily thrombin inhibitors.

Go to the next page if you knew the correct answer, or click the link image(s) below to further
research the concepts in this question (if desired).

Research Concepts:
Argatroban

We update eBooks quarterly and Apps daily based on user feedback. Please tap flag to
report any questions that need improvement.
Question 645: A 65-year-old female with alcohol use disorder is transported by paramedics
to the emergency department with severe agitation. On examination, she is confused and
irritable. The pulse rate is 110 bpm, blood pressure is 170/100 mm Hg and no fever. She does not
have any neurological deficits. She is admitted to the psychiatry ward and treated with thiamine,
intravenous fluids, and benzodiazepines. Her serum sodium is found to be 112 mEq/L and is
corrected with 3 percent saline over 24 hours, and the repeat sodium value is 136 mEq/L. Her
condition deteriorated, and she became non-responsive with agonal breathing. She could not be
revived after cardiopulmonary resuscitation. What is the most likely risk factor that led to the
unfortunate demise of this patient?

Choices:
1. Serum sodium 112 mEq/L for less than 6 hours before admission
2. Serum sodium 112 mEq/L for 6-12 hours before admission
3. Serum sodium 112 mEq/L for 12-24 hours before admission
4. Serum sodium 112 mEq/L for more than 48 hours before admission
Answer: 4 - Serum sodium 112 mEq/L for more than 48 hours before admission
Explanations:
Central pontine myelinolysis (CPM) is an acute demyelinating condition that primarily
affects the pons, although some cases have concomitant extra pontine myelinolysis.
It commonly occurs in malnourished patients or patients with alcohol use disorder who
undergo rapid correction of chronic hyponatremia.
The clinical picture includes seizures, dysphagia, dysarthria, pseudobulbar palsy, behavioral
abnormalities, hyperreflexia, quadriplegia, and coma.
Serum sodium of less than 120 mEq/L for more than 48 hours is a risk factor for CPM.

Go to the next page if you knew the correct answer, or click the link image(s) below to further
research the concepts in this question (if desired).

Research Concepts:
Central Pontine Myelinolysis

We update eBooks quarterly and Apps daily based on user feedback. Please tap flag to
report any questions that need improvement.
Question 646: A 32-year-old woman is intubated for the last 5 days in an ICU setting after
being hit by a car in a motor vehicle collision. She develops a high-grade fever, and vital signs
show a blood pressure of 90/60 mmHg, a heart rate of 110 beats per minute, and a temperature of
102oF. Cultures are taken, and within 48 hours of incubation, Gram-positive cocci are found in
clusters. The tests for catalase and coagulase are both positive. She is started on vancomycin, but
her condition keeps on worsening despite the treatment. Which of the following is the most
appropriate next step in the management of this patient?

Choices:
1. Stop vancomycin and start daptomycin
2. Stop vancomycin and start piperacillin/tazobactam
3. Stop vancomycin and start linezolid
4. Stop vancomycin and start rifampin
Answer: 3 - Stop vancomycin and start linezolid
Explanations:
The findings in this patient suggest infection with a methicillin-resistant Staphylococcus
aureus (MRSA).
MRSA is a leading cause of hospital-acquired and ventilator-associated pneumonia.
Hospital-acquired pneumonia (HAP), or nosocomial pneumonia, is characterized as
pneumonia developing 48 hours or more after hospital admission, indicating that it was not
incubating at the time of admission. Ventilator-associated pneumonia (VAP) is defined as
pneumonia developing 48 hours or more after implementation of endotracheal intubation
and mechanical ventilation and was not present before intubation.
Linezolid is the drug of choice for pneumonia caused by MRSA that has failed vancomycin
treatment.
Daptomycin is not a choice because, despite covering MRSA, it is deactivated by
surfactants in the lungs. MRSA is not affected by piperacillin/tazobactam. Rifampin is
effective against MRSA, but it should not be used alone.

Go to the next page if you knew the correct answer, or click the link image(s) below to further
research the concepts in this question (if desired).

Research Concepts:
Methicillin Resistant Staphylococcus Aureus

We update eBooks quarterly and Apps daily based on user feedback. Please tap flag to
report any questions that need improvement.
Question 647: A 16-year-old male presents with spiking fevers, diaphoresis, and
progressive dyspnea for the past week. He was shot in the right chest and had a chest tube placed
a month ago. He was discharged home after a five-day admission. X-ray, ultrasound, and CT
scan of the chest reveal pleural effusion. Which of the following is the most likely cause of this
patient's condition?

Choices:
1. Chest tube placement
2. Pneumonia
3. Trauma
4. Atelectasis
Answer: 3 - Trauma
Explanations:
Around 20% of patients with pneumonia will develop a parapneumonic effusion that can
lead to empyema. About 70% of patients with empyema have parapneumonic effusion. The
other 30% are related to trauma, post-thoracic surgery, esophageal rupture, cervical
infection, or, in small numbers, a primary empyema (not associated with previous
pneumonia or intervention).
If the blood is not adequately evacuated from the chest cavity, it can clot and get infected,
resulting in empyema.
An infected hemothorax usually presents as fever after a few weeks. Chest x-ray and CT
scan are diagnostic.
Delayed presentation means an open thoracotomy, but the surgeon may attempt video-
assisted thoracoscopic surgery to clean the chest.

Go to the next page if you knew the correct answer, or click the link image(s) below to further
research the concepts in this question (if desired).

Research Concepts:
Empyema

We update eBooks quarterly and Apps daily based on user feedback. Please tap flag to
report any questions that need improvement.
Question 648: A 52-year-old man is scheduled for a Whipple procedure for pancreatic
carcinoma. An ultrasound-guided right internal jugular cannulation is attempted, but
unfortunately, the carotid is punctured twice. Another proceduralist decides to try cannulation of
the left internal jugular vein. Which of the following complications is most likely to occur in this
patient now, compared to a right-sided attempt?

Choices:
1. Brachial plexus injury
2. Hemopneumothorax
3. Chylothorax
4. Infection
Answer: 3 - Chylothorax
Explanations:
The thoracic duct usually starts from the level of the twelfth thoracic vertebrae (T12). It
extends to the root of the neck. It drains into the venous system at or near the junction of the
left internal jugular vein and the left subclavian veins at the commencement of the
brachiocephalic vein. It can be injured when placing a central line in the left internal jugular
vein.
Thoracic duct injury is a very rare complication of central line placement, but it is a unique
injury occurring during left-sided placement. The thoracic duct collects lymph in the body
except for the right thorax, upper limbs, head, and neck. It transports fats from the intestinal
lymphatics to the venous system. It also transports protein and T-lymphocytes back to the
venous circulation.
Injury to the thoracic duct causes chylothorax. Chylothorax can be diagnosed by pleural
fluid analysis. The pleural fluid will reveal high-fat content, specifically triglycerides, and
high T-lymphocyte count in chylothorax.
Central line placement is associated with a lot of complications including infection,
pneumothorax, and brachial plexus injury. However, the rates of these complications are
similar between the left and right sides. The carotid puncture may be more common on the
left side as there is more overlap of the carotid artery and internal jugular vein on the left
side as compared to the right side.

Go to the next page if you knew the correct answer, or click the link image(s) below to further
research the concepts in this question (if desired).

Research Concepts:
Internal Jugular Vein Central Venous Access

We update eBooks quarterly and Apps daily based on user feedback. Please tap flag to
report any questions that need improvement.
Question 649: An incident involving a hazardous chemical in an industrial facility is
reported by the local authorities. A 40-year-old male who was present at the facility is brought in
by his co-worker minutes after the incident. The patient complains of irritation of the eyes,
throat, and skin. His vital signs are within normal limits. The patient is not in acute distress, and
he can speak in full sentences. The hospital you work at does not have a toxicology service,
intensive care unit, or a specialized HAZMAT team available. What is the next step in the
management of this patient?

Choices:
1. Put personal protective equipment on the patient and emergency department staff. Transfer the
patient to the closest facility with a toxicology service, a HAZMAT team, and an intensive care
unit available
2. Isolate the patient in a negative pressure room in the emergency room and consult a poison
control
3. Establish intravenous access, apply oxygen by non-rebreather, and start fluid resuscitation
4. Remove the patient’s clothing, perform copious water irrigation, place on a cardiac monitor,
and apply oxygen
Answer: 4 - Remove the patient’s clothing, perform copious water irrigation, place on a
cardiac monitor, and apply oxygen

Explanations:
Personal protective equipment is used to minimize exposure of medical staff and first
responders. Transferring the patient to another facility without previous decontamination
entails an Emergency Medical Treatment and Active Labor Act (EMTALA) violation.
Contaminated victims should be placed in a negative pressure room where the
decontamination process is performed and where basic life support can be provided.
Intravenous access and fluid resuscitation are part of medical care after exposure to
hazardous chemicals. However, these measures should not delay decontamination.
Decontamination is the priority in HAZMAT victims. This is most effectively achieved
with the removal of clothes and copious water irrigation. After decontamination is done, the
patient can be transferred to a higher-level facility.

Go to the next page if you knew the correct answer, or click the link image(s) below to further
research the concepts in this question (if desired).

Research Concepts:
Toxic Exposure Hazardous Materials

We update eBooks quarterly and Apps daily based on user feedback. Please tap flag to
report any questions that need improvement.
Question 650: A 67-year-old man presents to the hospital with a chief complaint of chest
pain. The patient is found to have severe aortic valve insufficiency. He is taken emergently to the
operating room, where he undergoes replacement of his aortic valve. He is found to have a
severely reduced ejection fraction and is unable to come off cardiopulmonary bypass. His chest
remains open for 24 hours until he can be placed on a veno-arterial ECMO for cardiac support,
and he undergoes a left-sided heart pump system placement. The patient undergoes a cardiac
transesophageal echocardiogram and is found to have an ejection fraction of 12%. Vital signs
show blood pressure of 94/64 mmHg, heart rate of 64/min, and oxygen saturation of 98%. Which
of the following is the primary strategy for weaning this patient from VA ECMO?

Choices:
1. Norepinephrine infusion
2. Resolution of the underlying cause of cardiogenic shock
3. Decrease oxygenation
4. Removal of the left-sided heart pump system
Answer: 2 - Resolution of the underlying cause of cardiogenic shock
Explanations:
ECMO support cannot be withdrawn until the source of the initial problem is resolved. VA
ECMO support is performed to aid in arterial support of the body for propulsion when there
is cardiogenic shock. Regarding this patient who has a hard time weaning from
cardiopulmonary bypass, they can be placed on VA ECMO until the source of the
cardiogenic shock can be resolved.
Weaning from ECMO cannot be performed on this patient at this time due to his continued
low ejection fraction. Serial transesophageal echoes can be completed to look for signs of
improvement in heart function daily or even more frequently. Weaning from VA ECMO
can usually occur when the ejection fraction has improved to 25%-30%.
When placing a patient on VA ECMO, a left-sided heart pump system can be placed to help
offload the left ventricle. VA ECMO can cause the left ventricle to form a thrombus due to
congestion of blood in the left ventricle. The left-sided heart pump system can be placed
into the left ventricle to help propel blood forward to help offload the work of the left
ventricle. When weaning VA ECMO, the pump can be left in place to aid the left side of the
heart during weaning.
When weaning VA ECMO, the patient will need an EF greater than 25% to 30%. Therefore
he is not ready to be weaned with heavy pressor support. O2 saturations are important with
regards to weaning VA ECMO, but with this patient's low EF, this takes precedence when
weaning trials are going to be attempted. The left-sided heart pump system does not need to
be removed prior to VA ECMO weaning attempts.

Go to the next page if you knew the correct answer, or click the link image(s) below to further
research the concepts in this question (if desired).

Research Concepts:
Extracorporeal Membrane Oxygenation Weaning

We update eBooks quarterly and Apps daily based on user feedback. Please tap flag to
report any questions that need improvement.
Question 651: A 65-year-old immigrant from India presents to establish primary care. He
had tuberculosis at the age of 50 and received several years of treatment for that. He brings a
"declared cured" report card from a tuberculosis control program from India. He reports having
an effusion of the right side for which he underwent several thoracenteses. He states that "they
never found any bacteria in that." He also reports that he has some pleural thicking around his
lung and was told it would persist for a long time. CT scan of this chest showed right pleural
thicking with mild effusion without any parenchymal changes. CT guided thoracentesis shows a
milky white grossly exudative fluid, which is lymphocyte-rich (90%) fluid. The fluid is negative
for acid-fast bacilli (AFB) stain and polymerase chain reaction (PCR) assay to detect
Mycobacterium tuberculosis (TB). He is also negative for interferon-gamma release assay
(IGRA). Which of the following is the next best step in the management of this patient?

Choices:
1. Start treatment with second-line treatment for TB due to exudative and lymphocytic pleural
fluid
2. Order a tuberculin skin test to look for reaction and if positive, treat with 4 drugs first-line
therapy
3. Send pleural fluid for triglycerides and cholesterol levels to rule out pseudo-chylous effusion
4. Thoracic surgery consultation for decortication
Answer: 3 - Send pleural fluid for triglycerides and cholesterol levels to rule out pseudo-
chylous effusion

Explanations:
The patient is presently asymptomatic and likely to have trapped lung with chronic effusion.
The patient has been adequately treated for tuberculosis in the past and has negative TB-
PCR assay with negative TB-gold test making active tuberculosis unlikely. Long-standing
exudative fluid can accumulate cholesterol from membranes of lysed cells leading to
pseudo-chylothorax. True chylothorax, which is caused by the accumulation of chyle, is
rich in triglycerides and deficient in cholesterol.
The triglycerides will be more than 120 mg/dl, and the cholesterol will be less than 100
mg/dl in true chylous effusion. In pseudo-chylous effusion, triglycerides are less than 100
mg/dl, and cholesterol is more than 200 mg/dl.
The presence of chylomicrons detected through electrophoresis will confirm the true
chylothorax, and the presence of cholesterol crystals confirms the diagnosis of pseudo-
chylous effusion.
As the patient is from a country with endemic tuberculosis with BCG vaccinations, the
tuberculin test would have very high false-positive rates. As the patient is asymptomatic,
neither the antituberculosis treatment nor the surgical decortication is needed. However, the
patient needs to closely follow up due to the high risk of relapse.

Go to the next page if you knew the correct answer, or click the link image(s) below to further
research the concepts in this question (if desired).

Research Concepts:
Chylothorax

We update eBooks quarterly and Apps daily based on user feedback. Please tap flag to
report any questions that need improvement.
Question 652: A 76-year-old man is brought to the emergency department for an altered
mental status for the last two days. His past medical history is significant for heart failure with
reduced ejection fraction, type 2 diabetes mellitus, hypertension, coronary artery disease status-
post coronary artery bypass graft (CABG) 2 years ago, and chronic obstructive pulmonary
disease (COPD). On examination, he appears fatigued but is easily arousable. He is not oriented
to time, place, or person. Systemic examination reveals prominent pulsations in his neck,
bilateral basal inspiratory crackles, and bilateral pitting pedal edema. His extremities are cold to
touch, but he is moving all his limbs. No focal deficits are noted. A computed tomography (CT)
scan of the brain is unremarkable. Vital signs reveal a pulse of 106 beats per minute, blood
pressure of 70/52 mmHg, and pulse oximetry of 88% on room air. A chest x-ray reveals bilateral
pulmonary congestion. An electrocardiogram (EKG) shows normal sinus rhythm with an old left
bundle branch block. The patient is started on norepinephrine and transferred to the coronary
care unit for further management. A procedure to check his left-sided filling pressures is being
considered. Which of the following complications is most likely to occur as a consequence of
this procedure in this patient?

Choices:
1. Infection
2. Pulmonary artery rupture
3. Complete heart block
4. Pulmonary hypertension
Answer: 3 - Complete heart block
Explanations:
This patient is in cardiogenic shock, as evidenced by the hypotension, need of pressors, and
pulmonary edema.
The anticipated procedure is a right heart catheterization to measure pulmonary capillary
wedge pressure, which is a surrogate marker of left-sided filling pressures.
A relative contraindication is the presence of left bundle branch block. If this is the case,
due to the risk of the right bundle branch block during catheter insertion, external or
transvenous pacer should be placed at the moment of the procedure to prevent complete
heart block.
Though infection and pulmonary artery rupture are possible complications, they are less
likely in this patient if appropriate precautions and protocols are followed.

Go to the next page if you knew the correct answer, or click the link image(s) below to further
research the concepts in this question (if desired).

Research Concepts:
Pulmonary Capillary Wedge Pressure

We update eBooks quarterly and Apps daily based on user feedback. Please tap flag to
report any questions that need improvement.
Question 653: A 65-year-old female patient with a significant history of hypertension
presented to the clinic for evaluation of dyspnea upon exertion. She has no other complaints. She
is compliant with her lisinopril 10 mg and furosemide 20 mg every day. The blood pressure at
the visit was noted to be 120/80 mmHg. Physical examination revealed a diastolic murmur at the
apex of the heart. Breath sounds were clear on both lungs. An echocardiogram revealed an LVEF
of 55-60% by visual estimation. Left atrial enlargement and severe mitral stenosis were noted.
What is the next best step in the management of the patient?

Choices:
1. Increase the dose of lisinopril
2. Refer for percutaneous mitral balloon valvuloplasty
3. Increase the dose of furosemide
4. Perform a chest X-ray
Answer: 2 - Refer for percutaneous mitral balloon valvuloplasty
Explanations:
Left atrial enlargement (LAE) is most commonly due to conditions caused by pressure
overload, volume overload, or both. Mitral valve stenosis is one of the several underlying
causes of LAE.
Patients with LAE might present with a variety of symptoms, including palpitations,
dyspnea, syncope, peripheral edema, fatigue, and weight gain.
No known medical therapy is available to reverse the left atrial remodeling.
The focus of care and medical therapy in patients with LAE is the identification and
treatment of underlying pathologies.

Go to the next page if you knew the correct answer, or click the link image(s) below to further
research the concepts in this question (if desired).

Research Concepts:
Left Atrial Enlargement

We update eBooks quarterly and Apps daily based on user feedback. Please tap flag to
report any questions that need improvement.
Question 654: A 29-year-old woman presents to the emergency department for presyncope
and persistent hypotension. She reports a family history of pulmonary embolism secondary to
protein C deficiency. Acute pulmonary embolism is noted on CT pulmonary angiography.
Anticoagulation with IV heparin and a vitamin K antagonist is initiated. If protein C deficiency is
confirmed in this patient, what is the preferred duration of anticoagulant therapy?

Choices:
1. 4-6 weeks
2. 6-12 weeks
3. 3-6 months
4. Lifelong
Answer: 4 - Lifelong
Explanations:
In patients with an acute pulmonary embolism (PE), anticoagulation therapy is generally
continued for three to six months if there is an identifiable provoking, transient, or
reversible risk factor (recent trauma, orthopedic procedure, surgery, prolonged
immobilization).
In patients with inherited thrombophilias or persistently elevated risk for PE, lifelong
anticoagulation is preferred given their high risk for recurrence.
Patients with protein C or S deficiency may require lifelong anticoagulation.
Placement of an inferior vena cava filter should be considered if anticoagulants are
contraindicated, or a pulmonary embolism occurs in a patient who is being anticoagulated.

Go to the next page if you knew the correct answer, or click the link image(s) below to further
research the concepts in this question (if desired).

Research Concepts:
Acute Pulmonary Embolism

We update eBooks quarterly and Apps daily based on user feedback. Please tap flag to
report any questions that need improvement.
Question 655: A 72-year-old man is admitted to the hospital with urosepsis. He has a past
medical history of hypertension. He is receiving intravenous fluoroquinolones. Blood tests show
creatinine 1.8 mg/dl (0.8-1.3). On the fourth day of admission, his creatinine is 1.3 mg/dl, and he
is clinically improving. He develops a mild maculopapular rash on both lower extremities on the
fifth day of hospitalization, and his creatinine goes up to 2 mg/dl and to 2.6 mg/dl the next day.
There is no new onset of fever, dysuria, or cough. A kidney biopsy is performed and, pending the
result, his antibiotic is held. His creatinine started trending down after three days of withholding
the antibiotic. What is the pathophysiological mechanism of this patient's disease?

Choices:
1. Delayed hypersensitivity reaction
2. Ischemic renal injury
3. Glomerulitis and tubulitis associated with the infection
4. Podocyte injury
Answer: 1 - Delayed hypersensitivity reaction
Explanations:
This scenario is suggestive of drug-induced acute interstitial nephritis (AIN). New onset of
renal failure associated with the rash.
The classic triad of low-grade fever, rash, and eosinophilia may not be present in all
patients.
Drug-induced AIN is an idiosyncratic delayed hypersensitivity reaction.
It is not related to the dose or previous exposure of the offending agent.

Go to the next page if you knew the correct answer, or click the link image(s) below to further
research the concepts in this question (if desired).

Research Concepts:
Interstitial Nephritis

We update eBooks quarterly and Apps daily based on user feedback. Please tap flag to
report any questions that need improvement.
Question 656: A 65-year-old male patient presents to the emergency department with a
one-day history of fever, sharp substernal chest pain, and difficulty breathing. He states that the
chest pain is worse when lying down and improves when leaning forward. He recently had a
myocardial infarction five weeks ago and is concerned about a reoccurrence. He has a past
medical history of diabetes and hypertension. His blood pressure is 138/82 mmHg, pulse 88/min,
respirations 18/min, and temperature 101.0 F (38.3 C). The patient has a pericardial friction rub
on examination. His EKG shows diffuse ST elevation concave upward with PR depression. What
is the most appropriate next step in management?

Choices:
1. Administer naproxen
2. Administer aspirin, morphine, and nitrate
3. Urgent cardiac catheterization
4. Discharge and recommend follow up in 3 weeks
Answer: 1 - Administer naproxen
Explanations:
This case is likely due to Dressler syndrome, also known as post-myocardial infarction
syndrome, which is a benign, autoimmune pleuritis or pericarditis that can occur weeks
after the event.
Dressler syndrome (DS) is an immune-related inflammatory response after this patient's
myocardial infarction five weeks prior. Therefore medications that would be appropriate to
treat DS would typically include NSAIDs, such as naproxen.
Dressler syndrome is a form of secondary pericarditis. Pericarditis is a condition in which
the pericardium (a fibroelastic sac surrounding the heart and comprised of a parietal and
visceral layer which is separated by a potential space) becomes inflamed. Under normal
circumstances, the pericardial cavity holds 15 to 50mL of pericardial fluid (an ultrafiltrate
of plasma).
The exact cause of DS is unknown, though it is thought to be immune-mediated.
Antimyocardial antibodies have been shown to be elevated in the blood of patients with DS,
but it is unclear whether these antibodies are the cause or occur as a result of the syndrome.
These antimyocardial antibodies are thought to target antigens that have become exposed
through damage to the pericardium.

Go to the next page if you knew the correct answer, or click the link image(s) below to further
research the concepts in this question (if desired).

Research Concepts:
Dressler Syndrome

We update eBooks quarterly and Apps daily based on user feedback. Please tap flag to
report any questions that need improvement.
Question 657: A 60-year-old man is admitted with severe cardiogenic shock. A pulmonary
artery catheter is placed to gauge the volume status of the patient. After 48 hours, the patient
develops a fever (101°F). Blood tests show a leucocyte count of 16000 per microliter (4500 to
11000) with 80% neutrophils and normal platelet count. An infection of the pulmonary artery
catheter is suspected. What is the most appropriate next step in the management of this patient?

Choices:
1. Remove the catheter
2. Remove the catheter and introducer
3. Remove the catheter and introducer, culture the catheter tip
4. Remove the catheter and introducer, and culture the introducer tip
Answer: 4 - Remove the catheter and introducer, and culture the introducer tip
Explanations:
When a pulmonary artery catheter infection is suspected, the highest yield is to culture the
introducer rather than the catheter itself.
For infected central venous catheters, it is necessary to culture the tip and not the
subcutaneous segment.
When a catheter-related infection is suspected, and there is catheter exit site exudate, swab
the drainage to obtain samples for gram stain and culture.
A diagnosis of a catheter-related infection requires one of the following, the same organism
is cultured from a peripheral vein and the catheter tip, the same organism is cultured from a
peripheral vein and the catheter hub, the same organism is cultured from samples from two
catheter lumens, or one sample is quantitatively three times greater than the other.

Go to the next page if you knew the correct answer, or click the link image(s) below to further
research the concepts in this question (if desired).

Research Concepts:
Central Line Associated Blood Stream Infections

We update eBooks quarterly and Apps daily based on user feedback. Please tap flag to
report any questions that need improvement.
Question 658: A 16-year-old female patient restrained passenger with lap and shoulder belt
is involved in a rear-end accident with another car. The patient now arrives in the trauma bay and
is emergently evaluated. Her vital signs are blood pressure: 108/70 mmHg, pulse rate: 90 beats
per minute, respiratory rate: 19 breath per minute, and temperature: 99.6 F. She is
hemodynamically stable, denies any loss of consciousness, denies neck pain, and only complains
of mild pain in her lower abdomen. On clinical exam, the patient is alert and oriented, and breath
sounds are equal bilaterally, the abdomen is tender with voluntary guarding without peritoneal
signs. She maintains movement in her lower extremities with normal sensation, anal sphincter
tone is intact, and denies any bowel or bladder incontinence. She only takes birth control pills.
The patient has a chest and pelvic x-ray, which are both normal, and undergoes focused
abdominal sonography for trauma evaluation, which is unable to view her bladder. Blood is
noted at her urethral meatus, and a retrograde urethrogram is performed, which shows an
extraperitoneal bladder rupture. What would be the next step management at this time?

Choices:
1. Obtain CT of her brain and cervical spine as well as chest, abdomen, and pelvis with
intravenous contrast
2. Obtain CT of her abdomen and pelvis with intravenous contrast with delays and if no intra-
peritoneal injury, place an intra-urinary bladder catheter and consult a surgeon
3. Proceed emergently to the operating room for repair of the extra-peritoneal bladder rupture
4. Place an emergent supra-pubic urinary catheter to drain the bladder and prevent renal injury
Answer: 2 - Obtain CT of her abdomen and pelvis with intravenous contrast with delays and
if no intra-peritoneal injury, place an intra-urinary bladder catheter and consult a surgeon

Explanations:
A common injury pattern seen in motor vehicle crashes in patients who wear lap and
shoulder restraints improperly is mesenteric and hollow viscus injuries. Due to the nature of
the bladder's intra-pelvic position, it is often not frequently injured. However, patients who
have a full bladder who are subject to this mechanism of injury are at high risk for bladder
rupture. Furthermore, young children have not developed enough that their intra-abdominal
organs have migrated to be protected by the lower thoracic cage or pelvis and thus remain
more susceptible to blunt force injuries.
Obtaining a computed tomography (CT) of the abdomen and pelvis would be a reasonable
diagnostic study to look for other occult injuries to the intra-abdominal organs. Computed
tomography with delays is adequate for examining the remainder of the urogenital system
and to determine the extent of urogenital injuries further. It also allows for the definitive
classification of a bladder injury being either intra or extra-peritoneal in nature. This is
important as both are treated differently.
If no other intra-abdominal injury is found that requires operative therapy, an extra-
peritoneal bladder rupture is managed with intra-urinary bladder catheter drainage with
eventual contrast cystogram and removal under a urologist's care. Suppose an intra-
peritoneal injury is found that warrants operative exploration, directed repair of an extra-
peritoneal bladder rupture can be carried out. Suprapubic urinary catheters have fallen out
of favor as primary treatment but are sometimes used in major urogenital trauma when
directed intra-urinary bladder catheter drainage is not possible.
In pediatric trauma patients, all attempts are made to limit potentially unnecessary ionizing
radiation exposure. Low energy imaging protocols are specially designed and utilized in this
patient group. In patients who are alert and oriented without concern for the loss of
consciousness or altered sensorium, computed tomography of the brain may be withheld.
By PECARN (Pediatric Emergency Care Applied Research Network) algorithms, this
patient would have a less than 0.05% risk of a clinically significant traumatic brain injury.
Also, patients who have a normal neurologic exam are not intoxicated, have no midline
cervical tenderness, neurologic deficit, or distracting injury qualify for clinical cervical
spine clearance based on the National Emergency X-Radiography Utilization Study
(NEXUS criteria). This prevents the real but small risk of ionizing radiation to a pediatric
patient with the possible future development of a malignancy.

Go to the next page if you knew the correct answer, or click the link image(s) below to further
research the concepts in this question (if desired).

Research Concepts:
Blunt Force Trauma
We update eBooks quarterly and Apps daily based on user feedback. Please tap flag to
report any questions that need improvement.
Question 659: A 32-year-old male is admitted to the ICU with the diagnosis of septic
shock, intubated, sedated, and with a central venous catheter. Closer inspection of the patient
reveals a right calf that is tense and a right foot that is cool and pulseless. Further proximal
inspection of the right lower extremity reveals two bandaged puncture sites to the proximal
medial tibia 2 cm medial to the tibial tuberosity. What is the next best step in the management of
this patient?

Choices:
1. Venous ultrasonography of the right lower extremity
2. Arterial ultrasound of the affected extremity
3. Orthopedic consult
4. Obtain compartment pressures on the distal right lower extremity
Answer: 4 - Obtain compartment pressures on the distal right lower extremity
Explanations:
One complication of Intraosseous (IO) needles is compartment syndrome. This can result
from penetration through the posterior cortex of the bone causing infusion of medication
and fluids into the muscular compartments of the lower extremity.
Two IO needle punctures in the same bone can cause extravasation of medications through
the previous puncture site into the tissues.
Necrosis of tissue can occur with extravasation of medication and fluids.
Every IO needle needs to be stabilized to help ensure it is not dislodged.

Go to the next page if you knew the correct answer, or click the link image(s) below to further
research the concepts in this question (if desired).

Research Concepts:
Intraosseous Vascular Access

We update eBooks quarterly and Apps daily based on user feedback. Please tap flag to
report any questions that need improvement.
Question 660: A 65-year-old woman presents to the hospital complaining of productive
cough, fever with rigors and chills, and pleuritic type chest pain for five days. She has been
suffering from diabetes mellitus and hypertension for ten years. On examination, her blood
pressure is 160/90 mmHg, the pulse is 91/min regular, and she has a high-grade fever. On chest
auscultation, there are coarse crackles in the right lower chest with absent breath sounds in the
base. A chest radiograph shows right-sided infiltrates and basal opacity with lateral upward
sloping of a meniscus-shaped contour. A pleural tap is done which shows frank pus. What is the
most appropriate next step?

Choices:
1. Chest drain insertion
2. Therapeutic pleurocentesis
3. Pleurodesis
4. Bronchoscopy
Answer: 1 - Chest drain insertion
Explanations:
This patient appears to be suffering from pneumococcal pneumonia along with empyema
which is a grave complication of the disease.
Frank pus in the pleural tap is one of the indications of passing a chest drain as soon as
possible.
The mainstay of treatment for patients with S. pneumonia is antibiotic therapy and
supportive care including mechanical ventilation if necessary. Treatment of CAP varies
based on the area of practice and severity of the disease.
Patients with low-risk CAP are typically treated as an outpatient with macrolide
monotherapy. Respiratory fluoroquinolones are used in outpatient patients at higher risk.
Non-ICU inpatients are treated with a dual therapy of a b-lactam plus a macrolide or
respiratory fluoroquinolone monotherapy.

Go to the next page if you knew the correct answer, or click the link image(s) below to further
research the concepts in this question (if desired).

Research Concepts:
Streptococcus Pneumoniae

We update eBooks quarterly and Apps daily based on user feedback. Please tap flag to
report any questions that need improvement.
Question 661: A 51-year-old man develops adult respiratory distress syndrome (ARDS)
after being admitted for a pelvic fracture. On the third ICU day, he is sedated, intubated, and
ventilated with PEEP (positive end-expiratory pressure) of 12 cmH2O and a FiO2 of 0.6 (60%).
Suddenly, his pulse increases from 100/min to 140/min, systolic blood pressure drops from 120
to 90 mmHg, central venous pressure increases from 25 to 45 cmH2O, and jugular veins are
distended. Which of the following is the most likely cause of the patient's sudden worsening?

Choices:
1. Shifting of the endotracheal tube into the right main bronchus
2. Pulmonary thromboembolism
3. Myocardial infarction
4. Tension pneumothorax
Answer: 2 - Pulmonary thromboembolism
Explanations:
Pulmonary embolism (PE) occurs when there is a disruption to the flow of blood in the
pulmonary artery or its branches by a thrombus that originated somewhere else.
PE usually occurs when a part of this thrombus breaks off and enters the pulmonary
circulation. Very rarely, PE can occur from the embolization of other materials into the
pulmonary circulation such as air or fat following a trauma; in this case, pelvic fracture.
The most common symptoms of PE include dyspnea, pleuritic chest pain, cough,
hemoptysis, presyncope, or syncope.
Blood clot travels through the blood to any remote site such as lungs, causing pulmonary
embolism, which causes acute respiratory distress, which is life-threatening.

Go to the next page if you knew the correct answer, or click the link image(s) below to further
research the concepts in this question (if desired).

Research Concepts:
Acute Pulmonary Embolism

We update eBooks quarterly and Apps daily based on user feedback. Please tap flag to
report any questions that need improvement.
Question 662: A 60-year-old female was found in a semi-comatose condition and brought
to the emergency department. On physical exam, her height was 66 inches (167.6 cm), and her
weight was 123 lb (55.8 kg) (BMI=19.9 kg/m2). Her blood pressure was 80/40 mm; her pulse
rate was 50 beats/min with a temperature of 32 C (89.6F). She was disoriented and lethargic.
There was no palpable thyroid enlargement, but there was a scar on her neck. Laboratory test
results demonstrate: hemoglobin 9 g/dl; hematocrit 25%; WBC count 9840 cells/mm3; platelet
count 297,000 cells/mm3; creatinine 1.09 mg/dl sodium 126 meq/l; potassium 3.8 mEq/L;
chloride 93 mEq/L; and bicarbonate 24.5 mEq/L. Chest radiography and CT brain didn’t reveal
any abnormalities. Blood and urine cultures were sent. The patient was started on intravenous
fluid resuscitation with broad-spectrum antibiotics. Which of the following is most likely to be
seen in the ECG of this patient?

Choices:
1. ST depression and short QT interval
2. Peaked T wave and short QT interval
3. Low voltage ECG and long QT interval
4. Low voltage ECG and peaked T wave
Answer: 3 - Low voltage ECG and long QT interval
Explanations:
The patient has myxedema coma with hypotension, bradycardia, and hyponatremia. Given
the scar in her neck on physical exam, the most likely precipitating factor is untreated
hypothyroidism acquired after thyroidectomy. Complete heart block, bradycardia, flattened
T waves, low voltage, and bundle branch blocks are common ECG findings in these
patients.
Low voltage on EKG can be representative of pericardial effusion due to the accumulation
of fluid rich in mucopolysaccharides.
Fatal arrhythmias are important to recognize in myxedema coma. The patients can develop
QT interval prolongations leading to “Torsades De Pointes” which resolves with treatment
of the myxedema.
Myxedema causes decreased myocardial contractility and reduced cardiac output which
leads to hypotension.

Go to the next page if you knew the correct answer, or click the link image(s) below to further
research the concepts in this question (if desired).

Research Concepts:
Myxedema

We update eBooks quarterly and Apps daily based on user feedback. Please tap flag to
report any questions that need improvement.
Question 663: A 40-year-old woman is admitted to the intensive care unit following a high-
speed motor vehicle collision, in which she sustained head, chest, and lower limb injuries. She is
intubated and placed on a ventilator. On the fourth day of admission, she suddenly desaturates to
80% on FiO2 100%. The endotracheal tube is in position and suctioning is done, but she further
deteriorates. Her pulse is 110/min, blood pressure 100/70 mmHg, and oxygen saturation 75% on
100% FiO2. ECG shows sinus tachycardia. She is not receiving low molecular weight heparin.
Which of the following is the most appropriate next step?

Choices:
1. Portable chest x-ray
2. D-dimer
3. Bedside echocardiogram
4. CT pulmonary angiogram
Answer: 3 - Bedside echocardiogram
Explanations:
The most likely diagnosis is pulmonary embolism. Classification of pulmonary
thromboembolism (PTE) can be based on physiological grounds; a differentiation between
massive, submassive, and non-massive PE has been proposed. Massive PE (5%) can cause
hemodynamic instability (as in this patient) and precipitate cardiopulmonary arrest.
Echocardiography is helpful in assessing cardiac function at the bedside. In massive PE,
thrombus can occasionally be seen proximally in the main pulmonary artery.
Echocardiography should be reserved for unstable patients with suspected massive or
submassive PE who cannot be safely transported to the radiology department. Computerized
tomographic pulmonary angiography (CTPA) is the radiological investigation of choice in
stable patients with suspected non-massive PE.
ECG, chest x-ray, and D-dimer have insufficient sensitivity or specificity, even when used
in association with a clinical assessment to exclude PE. The classic S1Q3T3 pattern on
ECG is a nonspecific sign of acute right heart strain and occurs in 20% of patients with PE.

Go to the next page if you knew the correct answer, or click the link image(s) below to further
research the concepts in this question (if desired).

Research Concepts:
Acute Pulmonary Embolism

We update eBooks quarterly and Apps daily based on user feedback. Please tap flag to
report any questions that need improvement.
Question 664: A patient suffers a bullet wound to the abdomen and undergoes emergency
surgery. After surgery, there is redness around the wound which begins to spread quickly. The
area becomes swollen, discolored, and warm to the touch with the superficial appearance of
blisters and ulcers. The patient develops tachycardia and is now febrile. Which of the following
is the best management choice for this condition?

Choices:
1. It is a superficial soft tissue infection that usually resolves with IV antibiotics
2. It is a deep soft tissue infection, best treated with hyperbaric oxygen therapy and IV antibiotics
3. It is a deep soft tissue infection best treated by local surgical exploration, debridement, and IV
antibiotics
4. It is best treated by wide surgical excision and debridement along with broad-spectrum
antibiotics
Answer: 4 - It is best treated by wide surgical excision and debridement along with broad-
spectrum antibiotics

Explanations:
This disorder is sometimes referred to as wound gangrene, Meleney ulcer, suppurative
fasciitis, or hospital gangrene, but it is most widely known as necrotizing fasciitis or
Fournier's gangrene.
Necrotizing fasciitis can occur after trauma or around foreign bodies in surgical wounds.
The point of bacterial inoculation may be relatively small and may be relatively distant from
the affected area.
Necrotizing fasciitis is a dangerous and rapidly spreading, inflammatory, infectious
condition that is often located in the deep fascia, with secondary involvement of the
subcutaneous tissues. Subcutaneous air may be seen on x-rays.
To reduce mortality and morbidity, the condition must be emergently treated with
aggressive surgical debridement, hemodynamic support, and broad-spectrum antibiotics.
Repeated surgeries to remove additional necrotic material may be necessary.

Go to the next page if you knew the correct answer, or click the link image(s) below to further
research the concepts in this question (if desired).

Research Concepts:
Necrotizing Fasciitis

We update eBooks quarterly and Apps daily based on user feedback. Please tap flag to
report any questions that need improvement.
Question 665: A 60-year-old man is brought to the hospital by emergency medical services
after choking at a restaurant. His wife indicates that he was a chronic smoker, has a desk job, and
does not exercise routinely. The neurological examination shows dysphagia, Horner's syndrome,
loss of pain and temperature for the left side of the face, nystagmus, and a loss of the left corneal
reflex. What is the most likely diagnosis?

Choices:
1. Inferior medial pontine syndrome
2. Lateral medullary syndrome
3. Medial medullary syndrome
4. Lateral pontine syndrome
Answer: 2 - Lateral medullary syndrome
Explanations:
Sudden onset in symptoms is typical for ischemic events. An ischemic stroke is supported
by the patient's hypertension, overweight status, and sedentary lifestyle. This can be
confirmed with a non-contrast CT. Non-contrast is utilized as a first imaging modality to
ensure that the brain is not subjected to the contrast agent if hemorrhage is observed.
All of the symptoms can originate from damage to the lateral medulla, hence the diagnosis
of lateral medullary syndrome (Wallenberg Syndrome).
Dysphagia and hoarseness can occur due to damage to the nucleus ambiguus in the medulla.
Loss of pain and temperature occurs with damage to the spinal trigeminal nucleus. Damage
to the inferior vestibular nucleus will cause nystagmus. Damage to the sympathetic fibers
will cause Horner's syndrome.
The facial nerve is the motor output for the corneal reflex. The facial nerve exits the
brainstem at the pontomedullary junction. Damage to the posterior inferior cerebellar artery
can affect the nerve at this point, causing a loss of the corneal reflex.

Go to the next page if you knew the correct answer, or click the link image(s) below to further
research the concepts in this question (if desired).

Research Concepts:
Corneal Reflex

We update eBooks quarterly and Apps daily based on user feedback. Please tap flag to
report any questions that need improvement.
Question 666: A 16-year-old patient was rushed to the emergency department following a
motor vehicle collision. The patient was complaining of a moderate headache. His Glasgow
coma scale was 15/15. The patient had no sensory or motor deficits. His brain computed
tomographic (CT) scan revealed a punctate anterior left temporal contusion of 1 cm x 1.5 cm.
The patient was admitted to the intensive care unit for observation and was managed
symptomatically. His blood panel studies at admission were within the normal ranges. His repeat
brain CT showed significant progression in the size of the contusion to 3.5 cm x 2.4 cm. He had
mild expressive aphasia as well. What is the most likely explanation for the adverse event seen in
the patient?

Choices:
1. Vitamin K deficiency
2. Idiopathic thrombocytopenic purpura
3. Protein C and S deficiency
4. Release of tissue factor
Answer: 4 - Release of tissue factor
Explanations:
The contused brain is one of the major sources of the release of the tissue factor, a
component of the coagulation cascade, following traumatic brain injury.
The release of the tissue factor leads to the activation of the coagulation cascade, thereby
predisposing the risk of consumptive coagulopathy.
Trauma-induced coagulopathy is one of the significant factors determining prognostic
outcome in patients with a head injury. Its incidence has been reported as high as 97% of
the patients with traumatic brain injury.
The patient with protein C and S deficiency will have a history of bleeding diathesis in the
past.

Go to the next page if you knew the correct answer, or click the link image(s) below to further
research the concepts in this question (if desired).

Research Concepts:
Cerebral Contusion

We update eBooks quarterly and Apps daily based on user feedback. Please tap flag to
report any questions that need improvement.
Question 667: A 65-year-old man with a history of hypertension, tobacco use, and stage III
chronic kidney disease suffers an out-of-hospital cardiac arrest. He is intubated at the scene and
transported to the nearest emergency department. After being successfully resuscitated, he is
admitted to the intensive care unit for therapeutic hypothermia. His core temperature is
maintained at 33 C (91.4 F) for 24 hours, after which he undergoes passive rewarming. His
temperature increases from 33 C (91.4 F) to 36 C (96.8 F) within 2 hours. Which of the
following clinical findings is most likely to be seen in this patient?

Choices:
1. Triphasic waves on the EEG
2. Peaked T-waves on ECG
3. Elevated peak airway pressures
4. Hyperbilirubinemia on morning labs
Answer: 2 - Peaked T-waves on ECG
Explanations:
During therapeutic hypothermia, potassium shifts intracellularly. During the rewarming
phase, this process is reversed. This can result in hyperkalemia, particularly if the
rewarming process is accelerated. Hyperkalemia could be consistent with the EKG changes
described.
In patients undergoing therapeutic hypothermia, the rewarming phase is critical. Patients
can experience increased cerebral edema, shivering, and electrolyte abnormalities,
especially when rewarming occurs too quickly. In this example, the patient's body
temperature increased by 3 degrees over a 2 hour period, which considerably exceeds the
0.25-0.5 degree/hour rate, which is recommended during rewarming.
Hyperkalemia typically produces peaked T waves early in the course, which is especially
notable in the precordial leads. If left untreated, this can progress to PR prolongation and
widening of the QRS complex. Fatal arrhythmias can result.
EEG can be very useful for evaluating neurological function in the post-arrest patient.
Triphasic waves are a common finding in patients with metabolic encephalopathies,
including hepatic encephalopathy. They are not characteristic of patients with anoxic brain
injury.

Go to the next page if you knew the correct answer, or click the link image(s) below to further
research the concepts in this question (if desired).

Research Concepts:
Anoxic Encephalopathy

We update eBooks quarterly and Apps daily based on user feedback. Please tap flag to
report any questions that need improvement.
Question 668: A 56-year-old male was admitted with acute hemorrhagic stroke. His past
medical history is significant for hypertension, diabetes mellitus, atrial fibrillation, and a history
of anaphylaxis to penicillin. The patient is found to have significant dysphagia, and the decision
to place a feeding tube is made. Which of the following should be administered prior to the
procedure?

Choices:
1. Clindamycin 900 mg 3 hours prior to the procedure
2. Clindamycin 900 mg 1 hour prior to the procedure
3. Cefazolin 2 gm 3 hours prior to the procedure
4. Cefazolin 2 gm 1 hour prior to the procedure
Answer: 2 - Clindamycin 900 mg 1 hour prior to the procedure
Explanations:
Antibiotic prophylaxis is required prior to the placement of a percutaneous gastrostomy
tube. Given the patient's severe allergy to penicillin, clindamycin is preferred.
Antibiotics should be administered 1 hour prior to the procedure.
Antibiotics help prevent the development of postprocedural infections.
Given the patient's severe penicillin allergy, cefazolin is not recommended. Most patients
with minor reactions to penicillin tolerate cefazolin.

Go to the next page if you knew the correct answer, or click the link image(s) below to further
research the concepts in this question (if desired).

Research Concepts:
Percutaneous Gastrostomy And Jejunostomy

We update eBooks quarterly and Apps daily based on user feedback. Please tap flag to
report any questions that need improvement.
Question 669: A 26-year-old man who underwent surgery for a craniopharyngioma a week
ago presents to the clinic with symptoms of increased thirst and urination. He reports going to
the bathroom 8 times last night and an increase in the urine volume. Laboratory evaluation shows
a serum sodium level of 155 mEq/L and a urine osmolarity of 125 mOsm/L. Which of the
following is the next best step in the management of this patient?

Choices:
1. IV Ringer's lactate
2. Vasopressin nasal spray
3. Repeat surgery
4. Low sodium diet
Answer: 2 - Vasopressin nasal spray
Explanations:
The patient in this scenario has developed diabetes insipidus, which is a common
complication after pituitary surgery in patients with craniopharyngioma.
The signs of diabetes insipidus (DI) are increase thirst and urination. Biochemically patients
will have a low urine osmolality and an elevated plasma sodium level.
DI following surgery is usually temporary, but in patients with craniopharyngioma, DI can
be permanent.
Vasopressin supplementation is indicated to normalize sodium and relieve the patient's
symptoms.

Go to the next page if you knew the correct answer, or click the link image(s) below to further
research the concepts in this question (if desired).

Research Concepts:
Craniopharyngioma

We update eBooks quarterly and Apps daily based on user feedback. Please tap flag to
report any questions that need improvement.
Question 670: A young male with tetanus is in the intensive care unit on a ventilator
because of difficulty with breathing. While on the ventilator he continues to develop generalized
spasms and trismus. He already is on a midazolam drip, but the spasms persist. What other agent
may be of benefit in this patient?

Choices:
1. Magnesium
2. Calcium
3. Gabapentin
4. Ethylenediaminetetraacetic acid
Answer: 1 - Magnesium
Explanations:
Magnesium infusion often is used to break up tetanus spasms in patients who fail to respond
to benzodiazepines.
Magnesium also helps stabilize autonomic instability.
A drip of 2 to 3 grams, intravenously administered, every 12 hours may help.
It is important to monitor magnesium levels to avoid hypermagnesemia.

Go to the next page if you knew the correct answer, or click the link image(s) below to further
research the concepts in this question (if desired).

Research Concepts:
Tetanus

We update eBooks quarterly and Apps daily based on user feedback. Please tap flag to
report any questions that need improvement.
Question 671: A 65-year-old male with a history of hypertension, diabetes mellitus, and
congestive heart failure is hospitalized for pneumonia and a day later develops acute dyspnea and
chest pain. He is tachycardic and tachypneic. His lungs are clear to auscultation and there is no
peripheral edema. Which diagnostic test should be ordered?

Choices:
1. CT pulmonary angiogram
2. D-dimer level
3. Echocardiogram
4. Ventilation/perfusion scan
Answer: 1 - CT pulmonary angiogram
Explanations:
CT pulmonary angiogram should be obtained to diagnose pulmonary embolus.
V/Q scans are more challenging to interpret in patients with pneumonia, give probabilistic
rather than definitive answers, and if negative provide no alternative diagnoses. They are
now largely restricted to those who have a contraindication to CT pulmonary angiography.
D-dimer levels may be elevated in this patient without pulmonary embolus.
CT pulmonary angiography is recommended as the procedure of choice to diagnose or
exclude acute pulmonary embolism even in pregnant women.

Go to the next page if you knew the correct answer, or click the link image(s) below to further
research the concepts in this question (if desired).

Research Concepts:
Acute Pulmonary Embolism

We update eBooks quarterly and Apps daily based on user feedback. Please tap flag to
report any questions that need improvement.
Question 672: A 66-year-old male is brought to the emergency department with complaints
of difficulty breathing. He also complains of increased sputum production and change in the
color of the sputum. The patient has a past medical history of COPD, chronic heart failure,
diabetes mellitus, sleep-disordered breathing, smoking, and alcohol use disorder. Physical
examination is notable for cyanosis, crackles in the right middle lung fields, bilateral wheezing,
and peripheral edema (2+) of the extremities. Vitals reveal a temperature of 99.8 F, respiratory
rate of 22/min, blood pressure of 90/60 mmHg, and pulse rate of 108/min. Which of the
following is a contraindication to the use of non-invasive ventilation with average volume-
assured pressure support modality in this patient?

Choices:
1. Sleep-disordered breathing
2. Acute exacerbation of COPD
3. Hemodynamic compromise
4. History of smoking
Answer: 3 - Hemodynamic compromise
Explanations:
Average volume-assured pressure support (AVAPS) is a relatively newer modality of non-
invasive ventilation that integrates the characteristics of both volume and pressure-
controlled non-invasive ventilation.
AVAPS is contraindicated in patients with cardiac arrest/hemodynamic
compromise/unstable arrhythmia.
AVAPS can cause significant hypotension in patients with cardiac dysfunction due to
increased intrathoracic pressure and decreased preload.
AVAPS has been successful in the treatment of COPD-associated acute hypercapnic
respiratory failure. Significant improvement was seen in arterial blood gases, pH,
respiratory rate, and the Glasgow Coma Scale. Nonetheless, it is important to note that the
patients with a higher APACHE II score, calculated during ICU admission, had a higher
risk of treatment failure.

Go to the next page if you knew the correct answer, or click the link image(s) below to further
research the concepts in this question (if desired).

Research Concepts:
Average Volume-Assured Pressure Support

We update eBooks quarterly and Apps daily based on user feedback. Please tap flag to
report any questions that need improvement.
Question 673: A 50-year-old male with past medical history significant for hypertension,
diabetes mellitus type 2, and colon cancer resected 2 months ago presents to the emergency
department with complaints of right upper quadrant pain, nausea, vomiting, fatigue, malaise, and
yellow discoloration of the skin for one week. The patient appears lethargic and is slurring his
speech. Physical examination shows yellow sclera, yellow skin, and right upper quadrant
tenderness without peritoneal or Murphy's sign. He is also found to have a flapping tremor.
Laboratory work showed marked elevation of liver enzymes and bilirubin, elevated international
normalized ratio (INR) of 2.3, and thrombocytopenia. Computed tomography scan of the
abdomen and pelvis with intravenous contrast shows hepatic congestion, minimal flow in the
hepatic veins, a collapsed inferior vena cava, and splenomegaly, without any other liver
parenchymal, renal, pancreatic, or intestinal abnormality. Apart from definitive treatment, what
additional steps should be taken?

Choices:
1. Magnetic resonance imaging of the liver
2. Ursodeoxycholic acid
3. Emergent laparotomy
4. Evaluation for a liver transplant
Answer: 4 - Evaluation for a liver transplant
Explanations:
This patient has acute liver failure secondary to Budd-Chiari syndrome.
Acute onset of symptoms and a history of colon cancer, along with results of the imaging
studies, help establish the diagnosis.
A patient with Budd-Chiari syndrome who develops acute liver failure should be referred
for evaluation for a liver transplant.
Magnetic resonance imaging of the liver is unlikely to provide any further information.
Ursodeoxycholic acid has no role in the treatment of acute liver failure or Budd-Chiari
syndrome.

Go to the next page if you knew the correct answer, or click the link image(s) below to further
research the concepts in this question (if desired).

Research Concepts:
Budd Chiari Syndrome

We update eBooks quarterly and Apps daily based on user feedback. Please tap flag to
report any questions that need improvement.
Question 674: A 57-year-old female patient presents to the emergency department with
high-grade fever, arthralgias, and an itchy skin rash for three days. She has also experienced
chest pain for the past 1 day, worse with deep breathing. She has a history of rheumatoid arthritis
diagnosed one year ago, which has been under good control with adalimumab (4 months) and
methotrexate (1 year). Other significant past medical history includes a history of hypertension
for which she is on hydrochlorothiazide. On examination, she has a high-grade fever, heart rate is
100/minute, blood pressure is 134/88 mmHg. She appears in distress because of pain. The skin
exam reveals diffuse maculopapular rash on her lower extremities. The chest exam reveals
bibasal decreased breath sounds. The musculoskeletal exam is significant for tenderness and
swelling with effusion in bilateral knees, and few proximal interphalangeal joints in the hands.
Laboratory evaluation reveals complete blood count with hemoglobin 10.0 mg/dL, platelet count
105,000, and WBC 3000. The erythrocyte sedimentation rate is 78 mm/hr. The chest Xray
reveals bilateral pleural effusions. Further serological workup is pursued, which reveals a
positive anti-nuclear antibody, anti-dsDNA antibody, rheumatoid factor, and anti-CCP antibody.
The rest of the autoimmune profile is normal. Serological work-up one year ago at the time of
her rheumatoid arthritis diagnosis was significant for positive anti-CCP and rheumatoid factor,
and a negative anti-nuclear antibody. Which of the following is the most likely diagnosis?

Choices:
1. New-onset systemic lupus erythematous
2. Drug-induced lupus secondary to hydrochlorothiazide
3. Drug-induced lupus secondary to adalimumab
4. New extra-articular manifestation of rheumatoid arthritis
Answer: 3 - Drug-induced lupus secondary to adalimumab
Explanations:
New-onset fever, arthralgia, rash, serositis, and pancytopenia in a patient who was recently
started on an anti-TNF agent should raise a strong suspicion of drug-induced lupus.
Anti-TNF agents can cause positive ANA and positive anti-dsDNA antibodies. Anti-histone
antibodies may not always be present.
Management is the withdrawal of the anti-TNF agent (and avoiding all anti-TNF agents in
the future). NSAIDs or a short course of corticosteroids may be considered if symptoms do
not resolve within a few weeks of discontinuing the anti-TNF agent. It may take several
months for the serologies to turn negative.
Hydrochlorothiazide induced lupus is usually associated with a positive anti-histone
antibody and not an anti-dsDNA antibody. New-onset lupus is possible, but the timeline and
typical serologies/symptoms are more characteristic of anti-TNF induced lupus. While rash,
arthralgias, serositis, and pancytopenia can be seen as extra-articular manifestations of
rheumatoid arthritis, fevers are unusual secondary to rheumatoid arthritis and the serological
workup now is consistent with new-onset drug-induced lupus secondary to an anti-TNF
agent in this patient with rheumatoid arthritis.

Go to the next page if you knew the correct answer, or click the link image(s) below to further
research the concepts in this question (if desired).

Research Concepts:
Tumor Necrosis Factor Inhibitors

We update eBooks quarterly and Apps daily based on user feedback. Please tap flag to
report any questions that need improvement.
Question 675: An 85-year-old male with a past medical history of advanced Alzheimer
dementia and hypertension is admitted to the intensive care unit with acute hypoxic respiratory
failure and septic shock. He is intubated and placed on mechanical ventilation. Broad-spectrum
IV antibiotics are initiated. A chest radiograph reveals a large right pleural effusion. Follow up
computed tomography of the chest shows a dense right lower lobe consolidation with air
bronchograms and a large effusion. Thoracentesis is consistent with the exudative neutrophilic
type of effusion. The patient undergoes tube thoracostomy. Due to poor drainage by
hospitalization day 3, a repeat CT scan reveals loculation in the pleural space. Which of the
following is the next best step in the management of this patient?

Choices:
1. Tissue plasminogen activator (TPA) alone
2. TPA + deoxyribonuclease
3. Normal saline irrigation
4. No further intervention
Answer: 2 - TPA + deoxyribonuclease
Explanations:
The use of adjunctive intrapleural medications can be beneficial in patients with inadequate
chest tube drainage.
The MIST-2 Trial showed combination therapy of fibrinolytic and mucolytics (vs. double
placebo, fibrinolytic + placebo, or mucolytics + placebo) improved fluid drainage in
patients with pleural infection, reduced the frequency of surgical referral, and the duration
of the hospital stay.
Of note, there was no proven mortality benefit noted with this intervention.
Adverse effects between all groups were not significant.

Go to the next page if you knew the correct answer, or click the link image(s) below to further
research the concepts in this question (if desired).

Research Concepts:
Thoracic Empyema

We update eBooks quarterly and Apps daily based on user feedback. Please tap flag to
report any questions that need improvement.
Question 676: A 26-year-old woman presents to the clinic for discussion regarding her
HIV care. Two weeks ago, she had an emergency department visit for shortness of breath, cough,
and an unintentional weight loss of 10 lbs in the preceding 1 month. During the encounter, a CT
chest demonstrated right upper lobe cavitary disease and hila lymph node enlargement. Sputum
samples were obtained during the visit. She was then prescribed amoxicillin-clavulanic acid and
doxycycline for 5 days and asked to follow up as an outpatient. Currently, she continues to
complain of nonproductive cough, lack of appetite, low-grade fever, and chills. Her sputum AFB
stain reports show Mycobacterium tuberculosis (TB). She is an immigrant from Nigeria and
contracted HIV working as a sex worker but has never taken HAART (highly active
antiretroviral therapy). Her CD4 count was 34/microL during the ED visit. Which of the
following management strategies is most likely to minimize the risk of TB-related immune
reconstitution inflammatory syndrome (IRIS) in this patient?

Choices:
1. Start antitubercular regimen now, then prednisone, and then HAART 4 weeks later
2. Start antitubercular regimen now, then HAART after completion of TB treatment
3. Start antitubercular therapy and HAART now
4. Start HAART only, defer antitubercular regimen till CD4 counts improved to >100 /microL
Answer: 1 - Start antitubercular regimen now, then prednisone, and then HAART 4 weeks
later

Explanations:
The patient has a simultaneous diagnosis of HIV and TB. It is important to treat both.
However, the risk of IRIS is significantly high, with TB in HIV patients compared to non-
HIV patients.
In patients with CD4 counts of 50/microL, early initiation of HAART (i.e., 1-4 weeks) after
initiation of anti-TB treatment has been found to have improved mortality outcomes with an
increased risk of nonfatal TB-IRIS.
Due to this mortality benefit, early initiation of HAART is recommended along with the use
of prophylactic steroids to prevent IRIS symptoms.
Waiting for the antitubercular regimen to be completed before starting HAART would delay
antiretroviral therapy by at least 6 months and is not recommended. Starting antitubercular
medication and HAART at the same time is also not recommended due to the risk of TB-
IRIS. HAART should be initiated ideally after 8-12 weeks of TB medications (delayed ART
initiation) or after 1-4 weeks (early ART initiation) if they have CD4 counts 50/microL.
Starting HAART only and not antitubercular medication is not the correct response as it
does not treat the opportunistic infections associated with HIV. Treatment of latent or active
infections in HIV should be done either prior to or during HAART therapy not after.
Additionally, it is not recommended to wait till we see CD4 count improvement to initiate
treatment against opportunistic infections.

Go to the next page if you knew the correct answer, or click the link image(s) below to further
research the concepts in this question (if desired).

Research Concepts:
Immune Reconstitution Inflammatory Syndrome

We update eBooks quarterly and Apps daily based on user feedback. Please tap flag to
report any questions that need improvement.
Question 677: A 30-year-old woman with a past medical history of anxiety and alcohol use
disorder is brought to the emergency department by ambulance after a motor vehicle accident.
The patient is unconscious and breathing at seven breaths per minute. The airway is secured, and
fluid resuscitation is initiated. The patient is given chlordiazepoxide for possible alcohol
withdrawal. After a few hours, her husband arrives and informs the medical team that she has
recently been started on chlordiazepoxide by her primary care team. She has therefore received
an accidental overdose of this medication. What is the mechanism of action of the most
appropriate drug to reverse this?

Choices:
1. Competitive inhibition of the alpha-beta subunit of GABA binding site
2. Competitive antagonist of beta-1-adrenergic receptors
3. Carbonic anhydrase inhibitor
4. Inhibition of conversion of angiotensin I to angiotensin II
Answer: 1 - Competitive inhibition of the alpha-beta subunit of GABA binding site
Explanations:
Flumazenil acts by competitive inhibition of GABA receptors. It is sometimes used for a
partial or complete reversal of the sedative property of benzodiazepines, such as
chlordiazepoxide.
After the administration of flumazenil, clinicians should monitor the patient for the risk of
seizure associated with this medication, especially in patients with chronic benzodiazepine
use.
In the case of chlordiazepoxide toxicity or overdose, the patient may present with
confusion, diminished reflexes, comatose, or somnolent.
Competitive antagonist of beta-1-adrenergic receptors is the mechanism of action of beta-
blockers such as propranolol. Inhibition of conversion of angiotensin I to angiotensin is the
mechanism of action of ACE inhibitors, such as captopril, which reduces blood pressure.
Therefore, it is not helpful in this patient.

Go to the next page if you knew the correct answer, or click the link image(s) below to further
research the concepts in this question (if desired).

Research Concepts:
Chlordiazepoxide

We update eBooks quarterly and Apps daily based on user feedback. Please tap flag to
report any questions that need improvement.
Question 678: A patient in the intensive care unit has a mixed venous oxygen saturation of
55%. His hemodynamics reveal a cardiac index of less than 1.7, a blood pressure of 100/64 mm
Hg, and a heart rate of 110 beats per minute. His Pa02 is 90 on an FIO2 of 50%. His hematocrit
is 31%. Which of the following is the most appropriate next step in this patient's management?

Choices:
1. Blood transfusion
2. Increase FIO2
3. Inotropic support
4. Vasopressor administration
Answer: 3 - Inotropic support
Explanations:
It is important to know that mixed venous oxygen is affected by anemia, oxygen, cardiac
output, and metabolic activity.
In the above case, the cardiac index is low. Therefore, inotropic support is required.
It is helpful to distinguish low cardiac index from failure versus vasodilation seen in
distributive or septic shock.
Other useful modalities to recognize acute heart failure, including pulmonary edema or
enlarged cardiac silhouette on x-ray, decreased ejection fraction on ultrasound.

Go to the next page if you knew the correct answer, or click the link image(s) below to further
research the concepts in this question (if desired).

Research Concepts:
Cardiogenic Shock

We update eBooks quarterly and Apps daily based on user feedback. Please tap flag to
report any questions that need improvement.
Question 679: A 28-year-old man is brought to the hospital by his family for altered mental
status. They report a history of depression with psychotic features and recent suicidal thoughts.
He was found with an empty bottle of risperidone, which appears to have been filled less than a
week prior. On examination, he is altered and ill-appearing. His vitals are notable for
tachycardia, tachypnea, and a temperature of 40 C. The presence of which of the following
features best distinguishes this disorder from serotonin syndrome?

Choices:
1. Anhidrotic hyperthermia
2. Clonus
3. Rigidity
4. Hyperpnea
Answer: 3 - Rigidity
Explanations:
Neuroleptic malignant syndrome (NMS) is a syndrome of altered mental status, autonomic
instability, hyperthermia, and “lead pipe” rigidity associated with the use of certain
medications. NMS is most commonly secondary to dopaminergic antagonists but, more
rarely, can occur due to withdrawal from dopaminergic medications.
Haloperidol and fluphenazine, both first-generation neuroleptic agents, are the most
commonly associated with NMS. However, NMS can also occur with second-generation or
"atypical" neuroleptic agents such as risperidone. Risk factors for the development of NMS
include high doses of neuroleptic medications, recent or rapid escalation in dosing, a switch
from one agent to another, and parenteral administration of the agent causing the syndrome.
The treatment of neuroleptic malignant syndrome is largely supportive and includes
maintenance of euvolemia and normovolemia as well as prevention of complications, which
include deep venous thrombosis, rhabdomyolysis, renal failure, and cardiac dysrhythmias.
In cases of severe muscle rigidity not improved with supportive measures, dantrolene and
bromocriptine can be used.
The distinguishing feature of neuroleptic malignant syndrome is the classic "lead pipe"
rigidity. This can help distinguish NMS from other conditions that present with altered
mental status, including sepsis, serotonin syndrome, sympathomimetic toxicity,
anticholinergic syndrome, salicylate overdose, and thyrotoxicosis.

Go to the next page if you knew the correct answer, or click the link image(s) below to further
research the concepts in this question (if desired).

Research Concepts:
Neuroleptic Agent Toxicity

We update eBooks quarterly and Apps daily based on user feedback. Please tap flag to
report any questions that need improvement.
Question 680: A 26-year-old man presents with painful bullous skin lesions and dyspnea
after exposure to a chemical warfare agent. On examination, there are severe ocular burns,
redness, and bullae filled with yellow fluid on the face, axilla, and groin. There is pulmonary
edema. Other victims recall the smell of horseradish just before developing symptoms. What
metabolite can be detected in the urine up to 2 weeks after exposure to this agent?

Choices:
1. Glutathione
2. Thiodiglycol
3. Soman
4. Phosphite
Answer: 2 - Thiodiglycol
Explanations:
Patients that have been exposed to sulfur mustard usually describe the smell of onions or
garlic at the scene of exposure or they have a recent interaction with old artillery shells.
Thiodiglycol is a metabolite of sulfur mustard that can be detected in the urine.
Thiodiglycol has low utility in clinical settings due to delay in obtaining results and it is
often rapidly enzymatically transformed.
Glutathione is depleted in significant sulfur mustard systemic exposure, resulting in a
decreased ability of the cells to protect against oxidative stress. It is not excreted in the
urine.

Go to the next page if you knew the correct answer, or click the link image(s) below to further
research the concepts in this question (if desired).

Research Concepts:
Blister Agents

We update eBooks quarterly and Apps daily based on user feedback. Please tap flag to
report any questions that need improvement.
Question 681: A 23-year-old college student is admitted to the ICU for fever, headache,
vomiting, and neck stiffness. Examination shows blood pressure 110/70 mmHg, a pulse of 120
beats per minute, a respiratory rate of 22/min, and a temperature of 101 degrees Fahrenheit.
There is a diffuse purpuric eruption on his trunk and extremities. He is diagnosed as a case of
meningococcal meningococcemia, and appropriate therapy is instituted. On the third post-
admission day, he develops new abdominal pain and hypotension. His investigations show a
sodium level of 130 mmol/l, a potassium level of 6.1 mmol/l, and a bicarbonate level of 15
mEq/l. Which of the following describes the most likely finding to be seen on his abdominal CT
scan?

Choices:
1. Hyperdense adrenal enlargement without contrast enhancement
2. Hypodense adrenals with calcifications
3. Unilateral adrenal enlargement with a fatty component
4. Bilateral adrenal masses with contrast enhancement
Answer: 1 - Hyperdense adrenal enlargement without contrast enhancement
Explanations:
This patient has been diagnosed with disseminated meningococcal infection. A gram-
negative diplococcus is an important cause of meningitis, especially in individuals living in
dormitories. Neisseria meningitides can cause disseminated infection and can present with a
purpuric rash. An important complication that occurs is bilateral adrenal hemorrhage,
termed Waterhouse Frederichson syndrome.
Waterhouse Frederichson syndrome can present with diffuse abdominal pain and the
development of hypotension in a patient with meningococcemia. Other signs of adrenal
insufficiency may be present. Investigations may reveal a fall of hemoglobin/hematocrit and
the classic hyponatremia, hyperkalemia, and hypoglycemia associated with adrenal
insufficiency. Low serum cortisol and elevated ACTH further aids in the diagnosis.
A CT scan is important in the diagnosis of adrenal hemorrhage. It shows heterogeneous
round to ovoid lesions without enhancement. There are often peri-adrenal inflammatory
changes and possibly fat stranding. The adrenals appear hyperdense but without any
contrast enhancement. The hematoma may also extend into the perinephric space. Over
time, the hematoma appears less dense on subsequent CT scans and may develop
calcifications. This explains the CT findings of reduced density of the adrenals and
calcifications, which are seen in chronic adrenal hematomas.
Overall, adrenal hemorrhage is associated with a 15% mortality rate, which varies according
to the severity of the bleed. This increases to a 55 to 60% mortality rate in patients with
Waterhouse-Friderichsen syndrome (adrenal hemorrhage can be due to meningococcal
sepsis). Treatment would be dependent on the severity, but in general, corticosteroid
replacement should be used for adrenal insufficiency. Adrenal enlargement with a
significant fatty component is seen in angiomyolipomas. Adrenal lesions with contrast
enhancement are seen in cases of neoplastic deposits.

Go to the next page if you knew the correct answer, or click the link image(s) below to further
research the concepts in this question (if desired).

Research Concepts:
Adrenal Hemorrhage

We update eBooks quarterly and Apps daily based on user feedback. Please tap flag to
report any questions that need improvement.
Question 682: A 16-year-old male born in Central America patient presents to the hospital
due to a new-onset seizure. He has never had a seizure before. The patient's brother states that
the patient almost exclusively eats pork. Based on history and symptoms, an MRI is ordered that
reveals the presence of larvae. He is commenced on dexamethasone and another specific drug for
the underlying diagnosis. What is the mechanism of action of the drug in the discussion?

Choices:
1. Inhibits parasitic actin filament formation
2. Disrupts parasitic microtubule polymerization
3. Generates reactive oxygen species to damage parasite DNA
4. Decreases production of inflammatory mediators and enzymes
Answer: 2 - Disrupts parasitic microtubule polymerization
Explanations:
This is a case of neurocysticercosis, and the drug in the discussion is albendazole. The
mechanism of action of albendazole involves the binding of its active metabolite to the b-
tubulin subunit of microtubules. This stops the polymerization of microtubules within the
parasite and stops its motility, leading to its death.
Neurocysticercosis is more common in patients who eat pork products, especially
undercooked or raw pork products. A new unprovoked seizure in a Latino patient should
almost always have neurocysticercosis ruled out due to the high incidence of pork
consumption in this population. Albendazole, steroids, and anticonvulsants are usually the
mainstay of treatment.
Albendazole inhibits microtubular function and can also inhibit parasitic glucose utilization,
which depletes its ATP and also can lead to its death.
Steroids decrease inflammation by decreasing the inflammatory response. No current drug
inhibits parasitic actin filament formation, and albendazole does not regenerate reactive
oxygen species to damage DNA.

Go to the next page if you knew the correct answer, or click the link image(s) below to further
research the concepts in this question (if desired).

Research Concepts:
Albendazole

We update eBooks quarterly and Apps daily based on user feedback. Please tap flag to
report any questions that need improvement.
Question 683: A 35-year-old man is brought to the emergency department with complaints
of headache, blurring of vision, and weakness of the right side of his body. He has been
experiencing daily headaches for the past 6 days. The headaches are worse in the morning and
are aggravated by coughing and bending forwards. He complains that his right arm and leg are
numb and weak. He has also been experiencing intermittent fevers for the past 2 weeks. He was
diagnosed with HIV 10 years ago but did not seek treatment. His examination reveals a blood
pressure of 130/70 mmHg, pulse of 110 bpm, a temperature of 100 F (37.7 C), and a respiratory
rate of 22/min. His neurological examination reveals a weakness of 3/5 in the right upper and
lower arm with brisk reflexes. All sensations are diminished on the right side. Investigations
reveal a WBC of 3,000/microL, hemoglobin 10 gm/dl, platelets 120,000/microL, CD4+ count of
10/microL, serum ALT 65 IU/l, serum AST 40 IU/l, serum LDH 700 IU/l, CRP 75 mg/l, and
serum creatinine 1.0 mg/dl. CSF analysis reveals a cell count of 75 cells/microL, a protein of 110
mg/dl and glucose of 45 mg/dl. A Giemsa stain reveals the presence of crescent-shaped
organisms with blue cytoplasm and red staining nucleus. What is the appropriate management of
this patient?

Choices:
1. Amphotericin B
2. Pyrimethamine/sulfadiazine
3. Acyclovir
4. Ampicillin
Answer: 2 - Pyrimethamine/sulfadiazine
Explanations:
This individual is an HIV positive patient with advanced immunosuppression, CD4+ count
less than 50 cells/micro. He has presented with signs of raised intracranial pressure and
focal neurology. CSF examination confirms the presence of tachyzoites of Toxoplasma
gondii.
Cerebral toxoplasmosis is seen in advanced HIV infection with reactivation of latent
Toxoplasma infection or rarely because of primary infection. The management is with drugs
that demonstrate ready penetration into the CSF. Pyrimethamine and sulfadiazine are the
treatment of choice due to the ability to cross the blood-brain barrier.
The usual dose of pyrimethamine is 50-75 mg daily, and sulfadiazine is 1000-1500 mg daily
for 4-6 weeks. The addition of leucovorin prevents the development of hematological side
effects associated with pyrimethamine use. Adjunctive glucocorticoids may be added to
reduce inflammatory reaction.
Treatment duration is 4-6 weeks. Antiretroviral therapy must be instituted to improve CD4+
count. Amphotericin B is used in cases of fungal CNS infections, and acyclovir is used for
viral encephalitis. Isolation of listeria would warrant the addition of ampicillin.

Go to the next page if you knew the correct answer, or click the link image(s) below to further
research the concepts in this question (if desired).

Research Concepts:
HIV-1 Associated Toxoplasmosis

We update eBooks quarterly and Apps daily based on user feedback. Please tap flag to
report any questions that need improvement.
Question 684: A 90-year-old gentleman is referred for medical decision-making capacity to
consent to a percutaneous endoscopic gastrostomy (PEG) tube for feeding following a stroke.
Which of the following statements is correct regarding the evaluation of this patient's capacity to
consent to this procedure?

Choices:
1. A cognitive screen, such as the mini mental state examination (MMSE), will suffice
2. A full neuropsychological evaluation is always required
3. The individual is assumed to have capacity unless proven otherwise
4. The individual’s ability to write a will is also determined at the same time as the capacity
assesment
Answer: 3 - The individual is assumed to have capacity unless proven otherwise
Explanations:
Although cognitive impairment and psychiatric disturbances can impair one’s capacity to
make decisions, people are presumed to have capacity unless this comes into question.
A capacity assessment always involves an interview with the patient and any collaterals, as
well as an assessment of functional abilities. A screen would not be sufficient.
A full neuropsychological assessment is not always required, although some evaluation of
cognition is usually performed.
One type of capacity does not equal another type. Medical decision-making does not apply
to writing a will, which is civil capacity.

Go to the next page if you knew the correct answer, or click the link image(s) below to further
research the concepts in this question (if desired).

Research Concepts:
Neuropsychological Assessment

We update eBooks quarterly and Apps daily based on user feedback. Please tap flag to
report any questions that need improvement.
Question 685: A 43-year-old man with a past medical history of hypertension recently
returned from Cuba. He presented to the emergency department with abdominal pain that is
localized to his right lower quadrant with associated watery diarrhea. He states that he has fevers
intermittently that have been controlled with acetaminophen. The patient states that these
symptoms began less than 12 hours ago. The patient has a past surgical history of
cholecystectomy. What is the best initial step in the management of this patient?

Choices:
1. CT abdomen
2. Blood cultures
3. Administer gentamicin and doxycycline
4. IV fluids, bowel rest, and fever control
Answer: 4 - IV fluids, bowel rest, and fever control
Explanations:
The question stem highlights important aspects of the diagnosis of pseudoappendicitis
secondary to Yersinia enterocolitica infection. First, the stem highlights in the patient's
history, travel abroad to Cuba, which may expose the patient to the consumption of meat
from pigs and cattle, which may be infected with Y. enterocolitica.
Secondly, most patients infected with Y. enterocolitica never develop symptoms, but
advanced signs of infection include symptoms such as diarrhea, fever, chills. The patient
presents with signs of symptomatic pseudoappendicitis likely secondary to Y.
enterocolitica. The patient's right lower quadrant pain is also likely secondary to mesenteric
adenitis due to bacterial collection in the terminal ileum. As most cases of Y. enterocolitica
are self-limiting, presentation of advanced infection and symptoms warrants treatment. The
question stem asks what the initial best step in management is.
Elucidating and confirming the patient's diagnosis with other lab tests and imaging can wait,
but symptomatic management takes priority. Therefore, the best initial step in management
is controlling symptoms with IV fluids, bowel rest, and fever control.
The first step in management is controlling symptoms. After the patient's symptoms are
controlled, then can other lab tests and imaging modalities be used to make the diagnosis,
control of symptoms should not be delayed to make a definitive diagnosis. Obtaining blood
cultures would be important if the patient demonstrated signs and symptoms of septic
shock. However, the question stem does not indicate any signs of shock. The patient does
demonstrate symptoms of fever, diarrhea, and abdominal pain that require management and
control. Therefore the first step in management is control of symptoms. Pseudoappendicits
secondary to Y. enterocolitica infection is often a self-limiting condition, and antibiotics are
rarely required. However, in cases that do not respond to symptomatic therapy, and
especially in patients who are immunocompromised, antibiotics should be initiated, which
include doxycycline and an aminoglycoside such as gentamicin. The question stem does not
indicate an immunocompromised state of the patient. The question asks what the best initial
step of management is, which is control of symptoms with IV fluids, bowel rest, and fever
control. Control of symptoms should precede antibiotic initiation.

Go to the next page if you knew the correct answer, or click the link image(s) below to further
research the concepts in this question (if desired).

Research Concepts:
Pseudoappendicitis

We update eBooks quarterly and Apps daily based on user feedback. Please tap flag to
report any questions that need improvement.
Question 686: A 72-year-old female was admitted with a newly diagnosed cerebrovascular
accident and is evaluated for swallowing function. It is recommended to keep her NPO and place
a feeding tube. The patient is unable to provide a surgical history, but her spouse states that she
had a weight loss surgery, and part of her stomach was removed. What is the next best step?

Choices:
1. Further imaging to delineate the anatomy
2. Proceed with percutaneous endoscopic gastrostomy tube placement
3. Proceed with open gastrostomy tube placement
4. Discharge the patient with a nasoenteric feeding tube
Answer: 3 - Proceed with open gastrostomy tube placement
Explanations:
Though additional imaging may be helpful, it is safest to assume that the patient had a
bariatric procedure that altered the anatomy, such as a gastric sleeve or roux-en-y gastric
bypass.
The safest procedure for the patient is an open procedure, either gastrostomy or
jejunostomy.
This allows the surgeon to evaluate the anatomy during the procedure and make the best
decision for tube placement.
The patient will need long term feeding access, and a nasoenteric feeding tube is not the
most appropriate.

Go to the next page if you knew the correct answer, or click the link image(s) below to further
research the concepts in this question (if desired).

Research Concepts:
Feeding Tube

We update eBooks quarterly and Apps daily based on user feedback. Please tap flag to
report any questions that need improvement.
Question 687: A 21-year-old man with a past medical history of metastatic testicular
cancer with extensive metastasis to bone, brain, and spine is brought to the emergency
department with altered mental status. Bedside physical exam is limited due to his mental state
but reveals that he is unresponsive to a verbal or painful stimulus. He has no nuchal rigidity and
has normal and reactive pupils. The patient is found to having tongue lacerations due to self-
inflicted bites. A CT head is suggestive of multiple metastatic lesions. The patient appears
obtunded and intermittently continues to have violent shaking episodes. His mother is at the
bedside and is tearful given her son's lack of improvement. She wants him to be made
comfortable. Which of the following is the next best step in the management of this patient?

Choices:
1. Olanzapine
2. Soft restraints
3. IV midazolam
4. MRI brain
Answer: 3 - IV midazolam
Explanations:
The patient presents with an emergent need for sedation for the alleviation of symptoms.
The patient has generalized tonic-clonic seizures, thus administration of IV benzodiazepines
such as midazolam would control seizure activity and achieve palliative sedation.
Neurosurgey should also be consulted. However, the patient presents with seizure-like
activity and thus needs treatment for it immediately.
The etiology of this patient's seizures is likely due to his intracranial metastasis. Thus
administering antipsychotics is not the correct answer as it is reserved for patients with
catatonia and underlying psychosis presenting with altered mental status. Likewise placing
the patient on restraints would not be a part of his treatment as he is having seizures and not
merely presenting with agitations. While getting an MRI of the brain for further evaluation
of the mass is a good decision, it should not delay the administration of medication that
would achieve symptoms control and relief to the patient. Palliative sedation can be offered
before or while he is being evaluated by neurosurgery.

Go to the next page if you knew the correct answer, or click the link image(s) below to further
research the concepts in this question (if desired).

Research Concepts:
Palliative Sedation In Patients With Terminal Illness

We update eBooks quarterly and Apps daily based on user feedback. Please tap flag to
report any questions that need improvement.
Question 688: A 31-years-old man presents to the hospital with a chief complaint of
productive cough. He also reports nausea and malaise. His vital signs show a temperature of 38.6
C and respiratory rate of 32/min. He has a history of one-time cocaine use with his friends 4 days
before. He also has a history of gingivitis a week ago and had not resolved. What is the most
likely risk factor responsible for the patient's condition?

Choices:
1. Esophageal dysmotility
2. Cocaine use
3. Immunosuppression
4. Dental infection
Answer: 4 - Dental infection
Explanations:
Productive cough and fever increase the likelihood of lung abscess as one of the differential
diagnoses.
Gingivitis is a dental infection that may predispose to lung abscess infection.
Lung abscesses are more common in immunocompromised hosts (HIV-AIDS, post-
transplantation, or those receiving prolonged immune suppressive therapy). Lung abscesses
are also common in those patients with high risk for aspiration: seizures, bulbar
dysfunction, and cognitive impairment. Lung abscess is in the group of lung infections such
as lung gangrene and necrotizing pneumonia which is characterized by multiple abscesses
The patient has no risk factors for immunosuppression.

Go to the next page if you knew the correct answer, or click the link image(s) below to further
research the concepts in this question (if desired).

Research Concepts:
Lung Abscess

We update eBooks quarterly and Apps daily based on user feedback. Please tap flag to
report any questions that need improvement.
Question 689: A 68-year-old man is brought to the emergency department with
hematemesis. He has a heart rate of 116/min blood pressure 98/63 mmHg. He has a history of
type 2 diabetes, hypertension, and alcohol use disorder. He recently recovered from a traumatic
fracture of his femur. His hemoglobin is found to be 9 g/dL. Urgent endoscopy reveals a
circumferential blackened appearance to the distal third of the esophagus. Which of the
following is the most likely cause of the patient's condition?

Choices:
1. Portal hypertension
2. Esophageal malignancy
3. Esophageal necrosis
4. Esophageal stricture
Answer: 3 - Esophageal necrosis
Explanations:
Chronic vascular disease and acute ischemia can lead to the development of acute
oesophageal necrosis.
The distal third of the esophagus has the poorest blood supply in relation to the other two-
thirds. This makes it the most common region to develop necrosis.
Traumatic femoral fracture often results in significant blood loss creating an acute
ischaemic state that can lead to the development of acute oesophageal necrosis.
Acute oesophageal necrosis is the only answer that would give circumferential blackening
of the esophagus in an acutely unwell patient.

Go to the next page if you knew the correct answer, or click the link image(s) below to further
research the concepts in this question (if desired).

Research Concepts:
Esophageal Necrosis

We update eBooks quarterly and Apps daily based on user feedback. Please tap flag to
report any questions that need improvement.
Question 690: A 70-year-old man with a history of hypertension and 45 pack-year smoking
is brought to the emergency department with acute onset chest pain. The pain is described as
"sharp," and it radiates to his lower back. On physical examination, he is looking uncomfortable
and in significant pain. His blood pressure is 202/90 mmHg in both arms; the pulse is 118/min,
respiratory rate 26/min, and temperature 37.2 C (99 F). The lungs are clear to auscultation. The
heart examination is unremarkable for murmur or gallop. An electrocardiogram shows sinus
tachycardia without ST segments elevation or depression. A chest x-ray shows a normal aortic
silhouette. CT of the chest/abdomen/pelvic with intravenous contrast shows acute aortic
dissection extending from proximal descending aorta distal to the left subclavian and extending
distally to the right renal artery. Which of the following is the next best step in the management
of this patient?

Choices:
1. Intravenous hydralazine
2. Intravenous nifedipine
3. Intravenous sodium nitroprusside
4. Intravenous labetalol
Answer: 4 - Intravenous labetalol
Explanations:
In acute aortic dissection, the initial treatment goal is to control heart rate and blood
pressure. Beta-blockers, such as intravenous labetalol or esmolol, are the preferred first-line
medication.
The medical therapy goal is to lower the heart rate to a goal of 60/min and systolic blood
pressure of 120 mmHg.
Beta-blockers are the preferred agent owing to their negative chronotropic and lowering
blood pressure effects.
Other blood pressure medications do not reliably lower the heart rate and are at risk of
increasing aortic wall shear stress.

Go to the next page if you knew the correct answer, or click the link image(s) below to further
research the concepts in this question (if desired).

Research Concepts:
Systolic Hypertension

We update eBooks quarterly and Apps daily based on user feedback. Please tap flag to
report any questions that need improvement.
Question 691: A 55-year-old woman presents to the hospital for 1 day if experiencing
recurrent episodes of palpitations that led to shortness of breath and lightheaded. She has a past
medical history of atrial myxoma resection 20 years ago, nonobstructive coronary artery disease,
type 2 diabetes, hypertension, and hyperlipidemia. She also has a history of atrial flutter
refractory to optimal drug therapy with the maximum dose of beta-blockers, rhythm control with
dronedarone. Her vital signs are within normal limits. EKG shows atrial flutter with a variable
conduction block. Due to the inefficacy of non-invasive intervention, a plan was made to
cardiovert the patient. Which of the following is the most appropriate relation of paced cycle
length compared to the tachycardia cycle length for entraining this patient's rhythm?

Choices:
1. 10 to 20 ms shorter
2. 20 to 30 ms shorter
3. Equal
4. 10 to 20 ms longer
Answer: 1 - 10 to 20 ms shorter
Explanations:
Waldo formulated three criteria for entrainment. These are fixed fusion with constant pacing
rate, progressive fusion at faster pacing, and resumption of tachycardia with captured not
fused beat on termination of pacing.
Through entrainment, Waldo et al. differentiated between atrial flutter type I and II. Atrial
flutter type I can be transiently entrained and interrupted by rapid atrial pacing since it is
caused by reentry and has an excitable gap. Type I atrial flutter can show concealed
entrainment with a zone of slow conduction. In contrast, type II atrial flutter has not been
entrained, and the mechanism remains speculative.
Entrainment with fusion proves that reentry is the cause of the investigated arrhythmia. The
PPI-TCL duration gives information on whether the place of stimulation is inside our
outside the reentry circuit. PPI response pattern characterizes different electrophysiologic
properties of the tissues (excitable gap, excited tissue, refractory tissue).
To entrain the tachyarrhythmia, the paced cycle length needs to be 10 to 20 ms shorter than
the tachycardia cycle length (TCL).

Go to the next page if you knew the correct answer, or click the link image(s) below to further
research the concepts in this question (if desired).

Research Concepts:
Reentry Arrhythmia

We update eBooks quarterly and Apps daily based on user feedback. Please tap flag to
report any questions that need improvement.
Question 692: A 65-year-old male with a past medical history of atrial fibrillation on
warfarin and metoprolol, gastroesophageal disease, and type 2 diabetes on metformin presents to
the emergency department after the onset of word slurring and left facial droop 3 hours ago. He
is still unable to articulate a complete sentence. After a comprehensive neurologic exam and
imaging, an area of decreased perfusion is found in the territory supplied by the middle cerebral
artery, and tPA is administered. The patient's symptoms partially improve. Several hours after
treatment, the patient develops worsening confusion and arm weakness. Repeat computed
tomography scan shows a new large area of increased attenuation in the lateral parietal lobe.
What is the appropriate next step?

Choices:
1. Repeat tPA
2. Computed tomography scan with IV contrast
3. Neurosurgical intervention
4. Magnetic resonance angiogram
Answer: 3 - Neurosurgical intervention
Explanations:
Post-tPA intracranial hemorrhage should be considered if the patient develops a worsening
neurologic exam, new headache, change in the level of consciousness, increased weakness,
or sudden change in blood pressure. Risk factors include older age, greater stroke severity,
hyperglycemia/diabetes, hypertension, atrial fibrillation, heart disease, and anticoagulant
use.
This pathology usually happens within 36 hours after thrombolytic infusion. Only about
half of the patients with this condition are diagnosed 5–10 hours after the administration of
tPA.
The first step taken should be to immediately stop the infusion of tPA if it is still running.
Computed tomography scan of the brain is the first imaging step in the evaluation of
possible stroke or when there is suspicion of conversion to hemorrhagic after administration
of thrombolytic agents.
If hemorrhage is found (seen on computed tomography scan as the previous ischemic
infarct with superimposed hyperintense cerebral hemorrhage), the next steps would be to
collect baseline labs and reverse the anticoagulation using cryoprecipitate or other products.
In addition, neurosurgery should be called for potential surgical treatment.

Go to the next page if you knew the correct answer, or click the link image(s) below to further
research the concepts in this question (if desired).

Research Concepts:
Cerebral Ischemia

We update eBooks quarterly and Apps daily based on user feedback. Please tap flag to
report any questions that need improvement.
Question 693: Which of the following patients in cardiogenic shock would benefit the most
from an intra-aortic balloon pump?

Choices:
1. A patient with a history of cardiogenic shock who has new Q waves and pulmonary edema
2. A trauma patient with hypotension, distended neck veins, and muffled heart tones
3. A patient with left mainstem artery disease and poor response to IV dopamine
4. A patient with an inferoposterior myocardial infarction and new onset of a cooing, systolic
murmur
Answer: 4 - A patient with an inferoposterior myocardial infarction and new onset of a
cooing, systolic murmur

Explanations:
Intra-aortic balloon pump is a device that increases myocardial oxygen perfusion through
the dilation of coronaries during diastole and also augments cardiac output.
A balloon pump is most useful to stabilize a patient with a surgically correctable lesion that
is causing heart failure, such as a ruptured papillary muscle with acute mitral regurgitation.
Patients with a new myocardial infarction, in addition to severe, preexisting cardiac disease,
have a poor prognosis.
The balloon pump is not indicated for pericardial tamponade, which may occur in a trauma
patient.

Go to the next page if you knew the correct answer, or click the link image(s) below to further
research the concepts in this question (if desired).

Research Concepts:
Intra-Aortic Balloon Pump

We update eBooks quarterly and Apps daily based on user feedback. Please tap flag to
report any questions that need improvement.
Question 694: A patient develops meningitis after head trauma. The patient expires and the
autopsy reveals softening of brain tissue and diffuse edema. The patient most likely had what
type of necrosis?

Choices:
1. Coagulation necrosis
2. Liquefactive necrosis
3. Wedge infarct
4. Fat necrosis
Answer: 2 - Liquefactive necrosis
Explanations:
Liquefactive necrosis is transformation of solid tissue into fluid.
Liquefactive necrosis is typical of bacterial or fungal infections.
The necrosis is due to a release of hydrolytic enzymes.
Liquefactive necrosis often results in abscess formation.

Go to the next page if you knew the correct answer, or click the link image(s) below to further
research the concepts in this question (if desired).

Research Concepts:
Cell Liquefactive Necrosis

We update eBooks quarterly and Apps daily based on user feedback. Please tap flag to
report any questions that need improvement.
Question 695: A 65-year-old female undergoes a hip replacement after a fall. Two days
later, she develops sudden shortness of breath and tachycardia. On examination, she is
tachypneic with a heart rate of 120 beats per minute. Her blood pressure is 120/57 mm Hg. A
blood gas reveals a pH of 7.38, CO2 of 35, O2 of 72 and bicarbonate of 23. A duplex of the
extremities reveals a large clot in the iliofemoral veins. For a patient with these findings, what is
the most frequent finding on an x-ray?

Choices:
1. Normal
2. Hampton hump: a cone-shaped area of opacification
3. Westermark sign: a dilated pulmonary artery with distal oligemia
4. Pleural effusion
Answer: 1 - Normal
Explanations:
It is virtually impossible to diagnose a pulmonary embolism based on a chest x-ray. The
most common presentation of a patient with a pulmonary embolism will be a normal chest
x-ray.
When the x-ray is normal and suspicion is still high, the next step is to obtain a spiral CT. If
the CT is normal with a normal x-ray, one can exclude the diagnosis of pulmonary
embolism.
When a pulmonary embolus occurs, it may occlude one or a few branches of the pulmonary
artery. It may present on an x-ray as a cone-shaped area of opacification, representing an
area of atelectasis or infraction. This sign is seen in only a few patients.
Occasionally there is a large pulmonary embolus that blocks the pulmonary artery. The
proximal artery then appears dilated with distal collapse. This is seen as an area of
decreased vascular markings on the chest x-ray. The chest x-ray must be compared to a
previous normal x-ray to decipher these fine changes. Again, this sign is seen in only a few
patients with pulmonary embolism.

Go to the next page if you knew the correct answer, or click the link image(s) below to further
research the concepts in this question (if desired).

Research Concepts:
Acute Pulmonary Embolism

We update eBooks quarterly and Apps daily based on user feedback. Please tap flag to
report any questions that need improvement.
Question 696: A 50-year-old man is brought into the emergency department. The patient
had flu-like symptoms in the past few days. On physical examination, the patient is tachycardic,
febrile, and tachypneic. The patient's past medical history is unremarkable. While admitted at the
hospital, the patient was placed in a room with the air conditioner set at 55 degrees Fahrenheit.
The next day the patient is found hypothermic with a new irregular irregular heart rhythm.
Which conduction pathway in the heart is most likely affected?

Choices:
1. Sinoatrial node
2. Atrioventricular node
3. Coronary sinus
4. Bundle of His
Answer: 1 - Sinoatrial node
Explanations:
Atrial fibrillation may develop from low body temperature and can successfully be treated
by warming the patient to normal body temperature. Mild hypothermia may have a
presentation of shivering and intact mentation. Act quickly to rewarm before arrhythmias,
or other complications develop. Remove wet clothing, dry the patient, and raise the
temperature in the room. Give warmed oral liquids containing glucose if no
contraindications exist such as vomiting. Warmed oxygen and intravenous fluids may be
needed. Atrial fibrillation will show an absence of p waves which are derived from the SA
node.
Mild hypothermia is classified as a core temperature of 89.6 degrees F to 95 degrees F or 32
degrees C to approx 35 degrees C. Moderate hypothermia is 82.4 degrees F to 89.6 degrees
F or 28 degrees C to 32 degrees C, and severe hypothermia is 82.4 degrees F or 28 degrees
C.
Atrial fibrillation and ventricular fibrillation are common in hypothermic patients. The risk
for ventricular fibrillation rises as hypothermia worsens. Under 28 degrees C or 82 degrees
F, it is essential to prepare for ventricular fibrillation (vfib). Vfib is not treated the same way
in the presence of hypothermia. Rewarming must take place for shocks to be effective. One
shock and one round of drugs are administered. If no effect, rewarm the patient to a core
temperature of >86 degrees F or 30 degrees C before more attempts are made.
Even the slightest movement can cause vfib in a profound hypothermic patient.

Go to the next page if you knew the correct answer, or click the link image(s) below to further
research the concepts in this question (if desired).

Research Concepts:
Hypothermia

We update eBooks quarterly and Apps daily based on user feedback. Please tap flag to
report any questions that need improvement.
Question 697: A 47-year-old male is brought to the emergency department after being
found unconscious in the snow. His blood pressure is 90/55 mmHg, the pulse is 56 beats per
minute, respiratory rate is 13 breaths per minute, and a temperature of 31 C. He is semi-
conscious and arousable to sternal pressure. The paramedic is only able to get interosseous
cannulation in the left tibia after attempting for IV access multiple times. Central venous line
placement is planned. Which of the following is the best possible site of central venous line
placement in this patient?

Choices:
1. Brachiocephalic vein
2. Internal iliac vein
3. Femoral vein
4. Internal jugular vein
Answer: 3 - Femoral vein
Explanations:
Hypothermia can cause slowed impulses conduction through potassium channels resulting
in prolonged ECG intervals. Patients with moderate to severe hypothermia will likely
exhibit bradycardia and are at increased risk for ventricular arrhythmias. Therefore, care
should be taken to not stimulate the heart due to the increased irritability of the
myocardium.
If central access is required in the setting of hypothermia, a femoral central venous line
(CVL) is preferred due to less irritation of myocardium and a lower risk of dysrhythmia.
The ideal central venous access is through the femoral vein, which leads to external iliac
then to the common iliac and finally ending at the inferior vena cava.
Internal jugular and brachiocephalic veins both lead to the superior vena cava. Both are not
ideal sites for venous access compared to femoral vein cannulation due to higher
myocardial irritability and dysrhythmia. Internal iliac vein empties into the common iliac
vein without directly connecting to the femoral vein.

Go to the next page if you knew the correct answer, or click the link image(s) below to further
research the concepts in this question (if desired).

Research Concepts:
Hypothermia

We update eBooks quarterly and Apps daily based on user feedback. Please tap flag to
report any questions that need improvement.
Question 698: A 46-year-old male with ARDS from aspiration pneumonia is admitted to
the intensive care unit. He has been intubated and sedated for the last several days with
increasing oxygen requirement. The patient has been sedated with propofol infusion and fentanyl
infusion, however, the patient has been over-breathing the ventilator. It is decided to start the
patient on a cisatracurium drip. The patient received 10 mg of bolus followed by infusion of
cisatracurium at 4 mcg/kg/min. He had RASS goal of - 4 and train of four was 2, and the patient
was completely synchronous with the ventilator. Vital signs were Blood pressure 100/60 mmHg,
heart rate 66 beats per minute, Temp 98.4 F at the time of initiation of the medication. 12 hours
later, the attending clinician noticed a significant increase in the drip rate of cisatracurium from 4
mcg/kg/min to 10 mcg/kg/min. The patency of the intravenous line and IV tubing was assessed,
and no issue was identified. The only change in patient's clinical condition was the fact that he
had been persistently febrile overnight with Tmax of 101.4 F. What could be a possible
explanation for this increased requirement of the medication?

Choices:
1. Tachyphylaxis
2. Fever
3. Worsening ARDS
4. Malfunctioning intravenous line
Answer: 2 - Fever
Explanations:
Cisatracurium is eliminated by Hoffman's elimination which is a process independent of
liver and kidney.
Factors that affect Hoffman elimination process include temperature and pH changes.
Patient had significant fever thus increasing the elimination process of the medication and
needing more medication to achieve the same degree of paralysis
Tachyphylaxis to such an extent is not usually seen this early with this medication.
Worsening ARDS has no bearing on increasing requirements of paralytic dose. It was
ascertained that the intravenous lines were working fine so there is no reason to doubt that
statement.

Go to the next page if you knew the correct answer, or click the link image(s) below to further
research the concepts in this question (if desired).

Research Concepts:
Cisatracurium

We update eBooks quarterly and Apps daily based on user feedback. Please tap flag to
report any questions that need improvement.
Question 699: A 65-year old female diagnosed with a lung abscess two weeks ago. The
patient was started on intravenous (IV) piperacillin and tazobactam, but her symptoms did not
improve and she had a fever, productive cough, and shortness of breath. Her past medical history
is significant for congestive heart failure (CHF) with reduced ejection fraction with an automated
implantable cardioverter-defibrillator (AICD) and left-sided hemiplegia. She has been bed-bound
for the past year. Her vitals show a blood pressure of 100/90 mmHg, a temperature of 103
Fahrenheit, a pulse rate of 110 beats per minute, and a respiratory rate of 21 breaths per minute.
A chest x-ray revealed a cavity lesion in the posterior segment of the right middle lobe.
Computed tomography (CT) of the chest showed a lung abscess with a size of around 8 cm. The
size of the abscess has increased as compared to the previous CT scan. Which of the following is
the next step in the management of the patient's current condition?

Choices:
1. Continue with the antibiotic coverage
2. Surgical resection
3. Endoscopic or percutaneous drainage
4. Bronchoalveolar lavage
Answer: 3 - Endoscopic or percutaneous drainage
Explanations:
Depending on the patient's immune system, a lung abscess can be indolent or symptomatic.
Surgical intervention is considered in patients who fail to respond to medical therapy. A
segmentectomy or lobectomy of the lung is done.
Patients who are poor surgical candidates percutaneous and endoscopic drainage can be
considered.
Lung abscesses are common in patients with high risk for aspiration, who have seizures,
bulbar dysfunction, and cognitive impairment.

Go to the next page if you knew the correct answer, or click the link image(s) below to further
research the concepts in this question (if desired).

Research Concepts:
Lung Abscess

We update eBooks quarterly and Apps daily based on user feedback. Please tap flag to
report any questions that need improvement.
Question 700: A 68-year-old woman presents with sudden onset difficulty walking. The
symptoms started about 6 hours ago upon awakening. She has a past medical history of type 2
diabetes mellitus, which is well controlled. Her vital signs show oxygen saturation 98% on room
air, respiratory rate 17 per minute, heart rate 87 beats per minute, blood pressure 170/95 mmHg,
and temperature 98 F. On examination, she is, alert, able to follow commands, and is orientated
to time, place, and person. A CT scan of the head reveals a hypodense area in the right inferior
cerebellum. Which of the following neurological exam findings should be investigated in this
patient?

Choices:
1. Dysdiadochokinesia
2. Dysphasia
3. Peripheral facial palsy
4. Hemiparesis
Answer: 1 - Dysdiadochokinesia
Explanations:
The patient has a lesion in the cerebellum. Cerebellar lesions can present with instability
and imbalance in the absence of weakness or sensory deficits.
The absence of cognitive deficits points to a lesion below the cerebrum.
The posterior inferior cerebellar artery supplies the inferior cerebellum. It is the most likely
artery affected in this patient.
In the acute presentation of neurological deficits, in a vulnerable population, a plain cranial
CT followed by a computed tomography angiography (CTA) would be helpful to direct
stroke treatment.

Go to the next page if you knew the correct answer, or click the link image(s) below to further
research the concepts in this question (if desired).

Research Concepts:
Cerebellar Dysfunction

We update eBooks quarterly and Apps daily based on user feedback. Please tap flag to
report any questions that need improvement.
Section 8
Question 701: A 70-year-old man with a history of prior myocardial infarction with the
placement of drug-eluting stents and diabetes presents to the emergency department with
crushing chest pressure starting 2 hours ago. His electrocardiogram is notable for ST elevations
in leads I, aVL, and reciprocal depressions in leads II, III, aVF. The patient is emergently taken
for a left heart catheterization. He is noted to have an embolus in the proximal left anterior
descending artery. His prior stent in the right coronary artery is patent. An aspiration
thrombectomy is performed to remove the embolus. He is then referred for a transthoracic
echocardiogram, which reveals a patent foramen ovale, and a 40 mm mass on the aortic valve on
the left coronary cusp. What is the next best step in the management of this patient?

Choices:
1. Send the patient for emergent cardiac surgery to fix the patent foramen ovale (PFO)
2. Obtain transesophageal echocardiogram to evaluate the cardiac mass better
3. Implant an internal cardiac defibrillator for risk of sudden cardiac death
4. Reassurance
Answer: 2 - Obtain transesophageal echocardiogram to evaluate the cardiac mass better
Explanations:
Transesophageal echocardiogram (TEE) is the correct answer because it provides better
spatial resolution than a TTE and helps narrow the differential diagnosis of an aortic valve
mass, which includes valve thrombus, endocarditis, and papillary fibroelastoma (PFE).
On TEE, PFE should have a distinct appearance of a mass on a stalk with a shimmering
border.
Before surgery, you need to be sure of the diagnosis, which requires further imaging
evaluation.
PFO is a distractor. While a venous side thrombus can traverse potentially to the left atrium
via a PFO, the probability is lower with a 40 mm mass sitting much closer to the coronaries
in the aortic root. While PFEs can increase the risk of sudden cardiac death, there is no data
to support the routine use of ICD implantation. The patient became symptomatic from PFE
and does not need further evaluation and intervention.

Go to the next page if you knew the correct answer, or click the link image(s) below to further
research the concepts in this question (if desired).

Research Concepts:
Papillary Fibroelastoma

We update eBooks quarterly and Apps daily based on user feedback. Please tap flag to
report any questions that need improvement.
Question 702: A 29-year-man with a history of polysubstance abuse and IV drug use is
brought to the hospital by ambulance after being found unresponsive in a back alley. On his last
admission, he was found to have methicillin-sensitive staphylococcus aureus bacteremia without
complication. He completed antibiotic therapy two months ago. Vital signs reveal temperature 95
F, blood pressure 90/50 mmHg, pulse 98/min, respiratory rate 4/min, and SpO2 86% on ambient
air. Physical exam shows a comatose well-developed male with poor oral hygiene. Toxicology is
positive for benzodiazepines, opiates, marijuana, and alcohol. He has mildly elevated
transaminases. He is given naloxone and flumazenil with minimal response. He is subsequently
intubated and admitted to the intensive care unit. After two days of admission, he develops a
fever. Urinalysis and physical examination are unrevealing. The chest radiograph shows the
correct positioning of the endotracheal tube and a new infiltrate in the left lower lung. Which of
the following factors is most strongly associated with an infection due to a multi-drug resistant
organism in this patient?

Choices:
1. Hospitalization within the last 90 days
2. Poor oral hygiene
3. Positive sepsis criteria on admission
4. Prior IV antibiotic use within the last 90 days
Answer: 4 - Prior IV antibiotic use within the last 90 days
Explanations:
This patient has developed ventilator-associated pneumonia (VAP). VAP presents after 48
hours of intubation and mechanical ventilation.
The most common pathogens seen in VAP are Staphylococcus aureus, Pseudomonas,
Candida species, Klebsiella oxytoca and pneumoniae, Streptococcus species, and
Enterobacter species. Multidrug-resistant organisms (MRDO) are commonly seen in VAP.
Risk factors for VAP with MDRO are: prior IV antibiotic use within the last 90 days, need
for ventilatory support, septic shock at the time of VAP, acute respiratory distress syndrome
preceding VAP, greater than five days of hospitalization before VAP onset and need for
acute renal replacement therapy.
This patient received IV antibiotics for his S. aureus bacteremia infection two months ago
and is the strong risk factor for developing MDRO infection.

Go to the next page if you knew the correct answer, or click the link image(s) below to further
research the concepts in this question (if desired).

Research Concepts:
Nosocomial Infections

We update eBooks quarterly and Apps daily based on user feedback. Please tap flag to
report any questions that need improvement.
Question 703: A 45-year-old man with a history of irritable bowel syndrome who has
undergone appendectomy and cholecystectomy in the past is admitted to the emergency
department with abdominal pain. The patient states that the pain has been ongoing over the last
month. The pain is poorly localized and has been relieved by his home opioid pain medications
until three days ago when the medications “just don’t seem to cut it.” It feels different from his
previous flares of irritable bowel syndrome, for which morphine has helped to resolve. He has
been taking oxycodone 10 mg twice daily for the pain. A review of the state drug prescription
database shows that he was previously taking hydrocodone 10 mg until approximately one
month ago when he was prescribed oxycodone. The patient’s vital signs reveal blood pressure:
128/76 mmHg, temperature: 37 C, and pulse rate: 82 beats/min. Physical examination reveals a
well-appearing male in no acute distress and a mildly-tender abdomen. A CT scan does not show
any acute intra-abdominal pathology. His bloodwork is unremarkable. A review of his medical
record reveals multiple negative CT scans over the past 3 months. What is the next preferred step
in management?

Choices:
1. Administer IV morphine
2. Confront the patient regarding drug-seeking behavior
3. Place a nasogastric tube and consult surgery
4. Symptomatic control with non-narcotic therapy
Answer: 4 - Symptomatic control with non-narcotic therapy
Explanations:
This patient with chronic abdominal pain despite multiple surgeries an unremarkable
workup in the ED, including normal vital signs, normal laboratory findings, and negative
imaging, has a diagnosis most consistent with narcotic bowel syndrome (NBS).
Although this patient has a history of irritable bowel syndrome, these two disease entities
can occur together. The symptoms of NBS can easily be erroneously attributed to IBS, as
both are funcitonal abdominal pain disorders. This patient who states that the pain is
different in quality from his previous IBS flares should prompt clinicians to consider an
alternative diagnosis.
NBS is a hyperalgesic syndrome caused by opioids. Thus, the mainstay of treatment is
narcotic cessation. This can be done through the use of a multi-disciplinary approach, with
gradual tapering of opioids. It is important to first counsel the patient that opioids are likely
worsening his overall condition.
The administration of opioid analgesics will likely worsen his condition, and may cause him
to require an escalation of opioid therapy in the future to temporarily relieve the pain.
Although a multi-disciplinary approach, including psychotherapy and/or cognitive
behavioral therapy, will likely be used for the definitive management of this patient's
condition, the patient must first be counseled on the condition before this can begin. It is
easy to attribute this patient's behavior to drug-seeking behavior. However, the fact that the
patient has undergone multiple surgical procedures and multiple ED workups should
prompt one to believe that the patient has been genuinely seeking treatment, rather than just
medications.

Go to the next page if you knew the correct answer, or click the link image(s) below to further
research the concepts in this question (if desired).

Research Concepts:
Narcotic Bowel Syndrome

We update eBooks quarterly and Apps daily based on user feedback. Please tap flag to
report any questions that need improvement.
Question 704: A 65-year-old woman presents to the clinic with swelling over the bilateral
lower limbs and dyspnea for seven months. She says that the swelling initially started around the
ankle and progressed to the knee. She also started experiencing shortness of breath around the
same time, which has progressed over the last few months. Now she can only walk 100 m before
she starts panting. She has no history of alcohol consumption, smoking, or illicit drug use. Blood
pressure is 100/90 mmHg, the pulse rate is 90/min, and the jugular venous pulse (JVP) is 10 cm.
Examination reveals pitting edema over the bilateral lower limb. The cardiovascular exam
reveals a diastolic murmur over the right sternal border. S2 is split, and S1 is normal. A 2D-
Echocardiogram reveals normal left-sided cardiac function but does reveal a pulmonary and
tricuspid regurgitation. Laboratory investigations reveal prothrombin time (PT) 10 seconds,
activated partial thromboplastin time (aPTT) is 30 seconds; the D-dimer study is normal.
Pulmonary arterial pressure is 25 mmHg, and capillary wedge pressure is 10 mmHg. There is
mild tenderness over the right subcostal region, but there is no organomegaly observed. What is
the treatment of choice for the primary cause of this patient's symptoms?

Choices:
1. Enoxaparin
2. Bosentan
3. Right ventricular assist device
4. Levosimendan
Answer: 2 - Bosentan
Explanations:
This patient with bilateral pitting edema till the knee, dyspnea for seven months, a wide
split S2 and a tricuspid regurgitation on 2D echocardiogram is a case of primary pulmonary
hypertension.
Right heart failure in this patient is secondary to back pressure from primary pulmonary
hypertension, causing an abnormal function at both tricuspid and pulmonary valve. With the
backpressure, the right atrium becomes dilated. Subsequently, it causes a reduction in the
venous return, further leading to a raised JVP and edema in the organs upstream from the
right heart. Bosentan is an endothelin receptor antagonist and aids in pulmonary
vasodilation.
Chronic right ventricular failure (RVF) secondary to primary pulmonary hypertension can
be treated by causing pulmonary vasculature to dilate. This is done by endothelin receptor
antagonists (ERAs) and phosphodiesterase inhibitors (PDE5-inhibitors). They are
pulmonary vasodilators that reduce PAP, reduce pulmonary venous resistance (PVR), and
improve cardiac output in patients with RVF. ERAs block the endothelin-A and endothelin-
B receptors in endothelial and vascular smooth muscle cells, reducing the vasoconstrictive,
proliferative, and proinflammatory effects of endothelin. The use of ERAs in the ICU is
limited by their longer half-life and hepatotoxicity (bosentan). PDE5-inhibitors block
degradation of cGMP.
Enoxaparin is low molecular weight heparin used that could be used in cases of right heart
failure secondary to a massive pulmonary embolus. However, a normal D-dimer study and
the chronic nature of this patient's complaints make this diagnosis less likely. Levosimendan
is used to increase left ventricular (LV) output. It is an inotrope. However, this patient has a
normal left ventricular function as per 2D-Echocardiogram, and her right heart failure is not
secondary to left heart failure. This treatment would not be effective. A right ventricular
assist device may be an option for the patient with isolated RVF awaiting transplant.
However, a transplant without addressing pulmonary hypertension would lead to a
recurrence in right heart failure post-transplant.

Go to the next page if you knew the correct answer, or click the link image(s) below to further
research the concepts in this question (if desired).

Research Concepts:
Right Heart Failure

We update eBooks quarterly and Apps daily based on user feedback. Please tap flag to
report any questions that need improvement.
Question 705: A 65-year-old man presents to the emergency department with dyspnea and
palpitations. He has a past medical history significant for congestive heart failure and moderately
severe asthma. He is allergic to diltiazem. He drinks alcohol occasionally and denies smoking.
His vital signs show blood pressure 118/80 mmHg and heart rate 150/min with an irregular
rhythm. EKG shows absent P waves along with an irregular QRS complex. Which of the
following is the next step in the management of this patient?

Choices:
1. Quinidine
2. Amiodarone
3. Digoxin
4. Metoprolol
Answer: 3 - Digoxin
Explanations:
The initial goal in this patient is rate control. Beta-blockers and diltiazem are first-line
therapies. Metoprolol should be used with caution in patients with asthma. Digoxin is
indicated in the treatment of heart failure with concomitant atrial fibrillation.
Cardiac glycosides such as digoxin work by inhibiting the sodium-potassium ATPase (Na-
K ATPase) pump.
Digoxin has a narrow therapeutic index ranging between 0.5 and 0.9 ng/mL. Digoxin
toxicity manifests when the serum drug level reaches more than 2.0 ng/mL.
Digoxin overdose can happen due to myriad of reasons, including iatrogenic causes such as
prescribing higher doses of the drug to patients, noncompliance by the patient, accidental
overdosing by taking more pills as prescribed, consumption of plants containing cardiac
glycosides, chronic kidney disease, and hypokalemia can also predispose patients to digoxin
excess.

Go to the next page if you knew the correct answer, or click the link image(s) below to further
research the concepts in this question (if desired).

Research Concepts:
Digoxin

We update eBooks quarterly and Apps daily based on user feedback. Please tap flag to
report any questions that need improvement.
Question 706: A 65-year-old patient presents with numerous complaints. He claims that
over the past few months, he has no appetite, intermittent nausea, weakness, headache, muscle
cramps, and confusion. He denies any trauma and has no medical issues except for being a
smoker. The physical exam is unremarkable. Blood work reveals sodium of 120 mEq/L, serum
osmolality is 230, and high urine osmolality. The urine sodium is more than 40 mEq/L. The
blood urea nitrogen is low, but the potassium levels are normal. CT scan reveals the presence of
cerebral edema. What should the initial treatment be in this patient?

Choices:
1. Demeclocycline
2. Fluid restriction
3. Low dose lithium
4. Intravenous hypertonic saline
Answer: 4 - Intravenous hypertonic saline
Explanations:
The treatment of the syndrome of inappropriate antidiuretic hormone secretion (SIADH)
depends on whether the patient is symptomatic, the degree of hyponatremia, and whether
the condition is acute or chronic.
As this patient has sodium of 120 mEq/L, hypertonic saline is indicated.
Correcting hyponatremia very quickly can lead to central pontine myelinolysis.
In patients where the sodium is not as low, the first-line treatment is the restriction of fluid
intake. The use of lithium or ADH antagonists is not recommended.

Go to the next page if you knew the correct answer, or click the link image(s) below to further
research the concepts in this question (if desired).

Research Concepts:
Syndrome of Inappropriate Antidiuretic Hormone Secretion

We update eBooks quarterly and Apps daily based on user feedback. Please tap flag to
report any questions that need improvement.
Question 707: A 17-year-old male patient is 48 hours into treatment for septic shock. He is
currently ventilated with the following settings: FiO2 is 0.6, peak inspiratory pressure/positive
end-expiratory pressure ratio is 28:8, and the rate is 18 breaths per minute. He is receiving a
combination of epinephrine (0.1 mcg/kg per min) and norepinephrine (0.1 mcg/kg per min) for
circulatory support, and dexmedetomidine for sedation. His vital signs include a temperature of
38.3 C (100.9 F), heart rate of 110 beats per minute, blood pressure of 98/65 mmHg, and central
venous pressure (CVP) of 10 mmHg. Peripheral perfusion is characterized by 1+ to 2+ pulses
and 2 to 3 second capillary refill. Urine output is 0.4 mL/kg per hour and has not changed since a
bolus of normal saline increased his central venous pressure from 8 to 10 mmHg. His acidosis
has resolved. The clinician is about to start a milrinone infusion. Which of the following is most
accurate regarding this patient’s management?

Choices:
1. The low urine output most likely is related to inadequate intravascular volume
2. Decreasing his norepinephrine dose is more likely to improve peripheral perfusion than
decreasing the epinephrine infusion rate
3. Heart rate and contractility will be affected to about the same degree with a decrease in either
the epinephrine or norepinephrine
4. The low urine output most likely will improve with antibiotics
Answer: 2 - Decreasing his norepinephrine dose is more likely to improve peripheral
perfusion than decreasing the epinephrine infusion rate

Explanations:
Although the initial response to distributive shock is a hyperdynamic state with elevated
heart rate to maintain cardiac output despite low stroke volume, the ongoing capillary leak
from inflammation leads to inadequate end-organ perfusion with resultant altered mental
status, oliguria, and lactic acidosis. Initial steps to re-establish end-organ perfusion in shock
should include adequate fluid resuscitation followed by the administration of vasoactive
medications. However, practitioners should be careful not to over-resuscitate patients. A
central venous pressure goal of 10, in the absence of heart failure, indicates the adequate
establishment of intravascular volume.
Norepinephrine is an endogenous catecholamine that is derived directly from dopamine,
which is, in turn, derived from the amino acids, phenylalanine, or tyrosine. The
cardiovascular effects of norepinephrine are mediated primarily through alpha-1 receptors
in the peripheral vasculature, leading to vasoconstriction. As a sympathomimetic
medication, it is an excellent first choice in volume-refractory sepsis states, i.e., severe
sepsis or septic shock, characterized by a capillary leak. However, high doses of
norepinephrine (> 0.1 mcg/kg/min) can lead to elevated systemic vascular resistance that
can compromise end-organ perfusion. In this scenario, the high doses of norepinephrine
may be worsening the glomerular filtration rate, leading to oliguria despite the adequate
intravascular volume.
Epinephrine is an endogenous catecholamine derived from the methylation of
norepinephrine. Its cardiovascular effects are mediated through beta-1, beta-2, and alpha-1
receptors. When administered as a sympathomimetic drug, it leads to elevations in heart rate
(chronotropy) and contractility (inotropy).
Milrinone is a selective inhibitor of phosphodiesterase-3. Phosphodiesterase-3 is an enzyme
that degrades cAMP and cGMP. With inhibition of this enzyme, there is a reduction in the
degradation of cAMP, and therefore its effects are potentiated. The effects include an
increase in inotropy, vasodilation, and minimal chronotropy. Milrinone seems to improve
diastolic function, possibly by having a direct effect on the myocardium via enhancement of
sarcomere uptake of calcium ions, that is, lusitropy.

Go to the next page if you knew the correct answer, or click the link image(s) below to further
research the concepts in this question (if desired).

Research Concepts:
Septic Shock
We update eBooks quarterly and Apps daily based on user feedback. Please tap flag to
report any questions that need improvement.
Question 708: A 67-year-old man with is being treated with proton pump inhibitors due to
gastroesophageal reflux disease has weakness and nausea. Laboratory tests show a chronic
kidney disease. The clinical evaluation demonstrates only a loss of deep tendon reflexes and
apart from weakness and nausea the patient is asymptomatic. Which of the following is most
likely to be seen on serum electrolyte profile of this patient?

Choices:
1. Mild Hypermagnesemia
2. Moderate Hypermagnesemia
3. Mild Hyponatremia
4. Moderate Hyponatremia
Answer: 1 - Mild Hypermagnesemia
Explanations:
In individuals with chronic kidney failure, some conditions including proton pump
inhibitors, malnourishment, and alcohol use disorder can increase the risk of
Hypermagnesemia.
The earliest clinical sign of mild hypermagnesemia is the loss of deep tendon reflexes.
The most frequent symptoms of mild hypermagnesemia include weakness, nausea,
dizziness, and confusion.
Moderate hypermagnesemia is characterized by the worsening of the confusional state and
sleepiness, bladder paralysis, flushing, headache, paralytic ileus as well as a slight reduction
in blood pressure and blurred vision caused by diminished accommodation and
convergence.

Go to the next page if you knew the correct answer, or click the link image(s) below to further
research the concepts in this question (if desired).

Research Concepts:
Hypermagnesemia

We update eBooks quarterly and Apps daily based on user feedback. Please tap flag to
report any questions that need improvement.
Question 709: A 65-year-old male patient underwent a microsurgical clipping following a
ruptured anterior communicating artery aneurysm. The patient, on his third post-operative day,
became slightly drowsy, along with the worsening of the motor power in his bilateral lower
limbs. What is the most likely cause of the deterioration in this patient?

Choices:
1. Meningitis
2. Rebleed
3. Vasospasm
4. Seizure
Answer: 3 - Vasospasm
Explanations:
There is a high risk of vasospasm in subarachnoid hemorrhage following aneurysmal bleed,
owing to the release of bilirubin.
In cases of anterior communicating artery aneurysmal (ACOM) bleed, there is an onset of
vasospasm within the territories of the distal anterior cerebral arteries (DACA).
The distal anterior cerebral arteries supply paracentral lobule that controls power in the
bilateral lower limbs. Therefore, vasospasm in this region leads to weakness of bilateral
lower limbs.
Rebleeding, despite microsurgical clipping, is rare. Since the patient did not have a high-
grade fever, chills, and neck rigidity, the possibility of meningitis is also low. The patient
also did not have any seizurogenic activities.

Go to the next page if you knew the correct answer, or click the link image(s) below to further
research the concepts in this question (if desired).

Research Concepts:
Anterior Cerebral Artery Stroke

We update eBooks quarterly and Apps daily based on user feedback. Please tap flag to
report any questions that need improvement.
Question 710: A 20-year-old man presents with hoarseness and drooling after another
player elbowed him in the anterior neck while playing basketball. His heart rate is 90 beats per
minute, blood pressure 120/80 mmHg, respiratory rate 27 breaths per minute, and he is saturating
93% on room air. Which of the following is true regarding the management of this patient?

Choices:
1. The treatment of choice is humidified oxygen
2. Early orotracheal intubation should be avoided
3. A cricothyroidotomy kit should be at the bedside when intubation is attempted
4. Awake intubation is contraindicated
Answer: 3 - A cricothyroidotomy kit should be at the bedside when intubation is attempted
Explanations:
Following blunt laryngotracheal trauma, early control of the airway is advisable as edema
may rapidly progress to airway compromise.
This patient presents with signs and symptoms concerning for impending airway loss given
the mechanism of his injury. This patient's airway should be secured early via intubation,
but equipment to perform a cricothyroidotomy should be readily available in case the
patient loses his airway during the attempted intubation.
Tracheal trauma may present with no symptoms, or with symptoms including hoarseness.
The mildest, usually minimally symptomatic, cases may be treated with supplemental
oxygen and observation.
Awake intubation is preferred over rapid sequence intubation when intubation and
ventilation are predicted to be difficult. In this patient with potential impending airway loss,
awake intubation would be ideal. Regardless of the type of intubation technique planned,
equipment necessary for a surgical airway, such as a cricothyroidotomy, should be readily
available in case intubation fails due to loss of patency of the airway.

Go to the next page if you knew the correct answer, or click the link image(s) below to further
research the concepts in this question (if desired).

Research Concepts:
Tracheal Trauma

We update eBooks quarterly and Apps daily based on user feedback. Please tap flag to
report any questions that need improvement.
Question 711: A 50-year-old man is brought to the hospital in a state of altered mentation
at home by his close relatives. He has a history of HIV for the past 8 years and has been on
regular antiretroviral therapy. He also has a history of hypertension and diabetes, well-controlled
with his regular medications. An urgent MRI reveals bilateral diffuse cortical changes with
patchy brain stem involvement. A toxicology screen is negative. CSF cytology is within normal
limits. CSF for HSV PCR comes back positive for HSV-1. What is the best initial therapy for
this patient?

Choices:
1. Ganciclovir
2. Foscarnet
3. Cidofovir
4. Acyclovir
Answer: 4 - Acyclovir
Explanations:
This immunocompromised person is diagnosed to have herpes simplex encephalitis on the
basis of positive HSV-1 PCR of CSF. HSV-1 is the most common sporadic and fatal
encephalitis worldwide in adults. He needs to be immediately initiated on IV acyclovir
which is the first-line agent in herpes simplex encephalitis.
Acyclovir prevents viral multiplication. Herpes simplex encephalitis has got significant
morbidity & mortality despite antiviral treatment. The mortality and morbidity are much
higher in immunocompromised. The notable side effects of acyclovir include
thrombophlebitis and crystal-induced nephropathy; the latter is prevented by adequate
hydration.
Atypical or subtle clinical presentations are common in immunocompromised patients
which could delay the diagnosis and treatment. Diffusion-weighted MRI with flair
sequences can usually detect the classic temporal lobe/limbic system changes in herpes
simplex encephalitis (HSE). However, a predominantly extra-temporal presentation with
more diffuse involvement of the brain can occur in immunocompromised. CSF pleocytosis
may not occur in immunocompromised.
IV ganciclovir can be considered if IV acyclovir preparation is not available. Acyclovir
resistance is higher in immunocompromised patients than immunocompetent. Foscarnet or
cidofovir is considered in such cases.

Go to the next page if you knew the correct answer, or click the link image(s) below to further
research the concepts in this question (if desired).

Research Concepts:
Herpes Simplex Encephalitis

We update eBooks quarterly and Apps daily based on user feedback. Please tap flag to
report any questions that need improvement.
Question 712: A 65-year-old female presents with fever, chills, and confusion. Her
respiratory rate and heart rate are elevated and her blood pressure is 90/60. Urine output is
diminished. Which treatment will most likely increase systemic vascular resistance and be the
best immediate treatment option in this patient?

Choices:
1. Epinephrine
2. Dopamine
3. Intravenous fluid at keep vein open
4. Dobutamine
Answer: 2 - Dopamine
Explanations:
Dopamine and IV fluid boluses increase preload, systemic vascular resistance, and
circulating blood volume in patients with shock.
Dopamine at high doses increases vasoconstriction and increases blood pressure.
The effects of high doses of dopamine between 2 to 10 micrograms/kg/min on
vasoconstriction are the result of its actions on the alpha- and beta-adrenergic receptors in
the peripheral vasculature
Lower doses of dopamine have opposite effects, including increasing excretion by the
kidneys and increasing heart rate.

Go to the next page if you knew the correct answer, or click the link image(s) below to further
research the concepts in this question (if desired).

Research Concepts:
Dopamine

We update eBooks quarterly and Apps daily based on user feedback. Please tap flag to
report any questions that need improvement.
Question 713: A 45-year-old male is presented to the ER after a road traffic accident. On
arrival to ER, he has a pulse of 134/min, blood pressure of 90/60 mmHg, and a respiratory rate of
28/min. On examination, his extremities are cold. Urethral catheterization is done in the ER. IV
fluids are started. What is the best indicator of adequate capillary perfusion in response to
treatment in this patient?

Choices:
1. Urine output
2. Blood pressure
3. Skin temperature
4. Pulse
Answer: 1 - Urine output
Explanations:
The best indicator that resuscitation is going well is urine output. However, there are also
other laboratory parameters that one can use to assess the adequacy of resuscitation.
Both base deficit and lactate levels can be used to follow resuscitation. The base deficit
reflects the intensity of shock and oxygen delivery.
Sublingual capnometry is a new tool used to assess organ perfusion. The device consists of
a C02 sensor that detects levels of C02.
Lactate levels are a reliable marker for hypoperfusion and indicate anaerobic metabolism. A
failure to clear lactate levels within 24 hours after shock and resuscitation is a predictor of
increased mortality. Blood pressure and pulse are not reliable indicators of capillary
perfusion.

Go to the next page if you knew the correct answer, or click the link image(s) below to further
research the concepts in this question (if desired).

Research Concepts:
Hemorrhagic Shock

We update eBooks quarterly and Apps daily based on user feedback. Please tap flag to
report any questions that need improvement.
Question 714: A 75-year-old female patient presents from a nursing home after pulling out
her feeding tube, which was placed approximately 6 months ago. Her medical history is
significant for osteoporosis, unstable angina, and advanced dementia. Her vital signs are stable.
What is the first procedure that should be attempted to give the patient a new feeding tube?

Choices:
1. Percutaneous endoscopic gastrostomy tube placement
2. Fluoroscopic gastrostomy tube placement
3. Bedside insertion of a new gastric tube
4. Open gastrostomy tube placement on the operating room
Answer: 3 - Bedside insertion of a new gastric tube
Explanations:
When a patient has had a feeding tube for a long period of time, usually more than a few
months, the track is well established a new tube with an inflatable balloon is the first thing
that should be attempted.
If successful with bedside replacement of the feeding tube, many providers will obtain an x-
ray where contrast is placed into the feeding tube to confirm placement into the stomach.
There are many additional techniques that can be used to help successfully replace a feeding
tube at the bedside.
If unsuccessful bedside replacement, the patient will likely need to have a percutaneous
endoscopic or fluoroscopically placed feeding tube.

Go to the next page if you knew the correct answer, or click the link image(s) below to further
research the concepts in this question (if desired).

Research Concepts:
Feeding Tube

We update eBooks quarterly and Apps daily based on user feedback. Please tap flag to
report any questions that need improvement.
Question 715: A 53-year-old male with a history of lung cancer presents to the emergency
department with severe respiratory distress. Endotracheal intubation is performed. After
successful intubation, the capnography waveform is reviewed. Which of the following values
would indicate the correct endotracheal tube placement?

Choices:
1. 25 mmHg
2. 37 mmHg
3. 55 mmHg
4. 48 mmHg
Answer: 2 - 37 mmHg
Explanations:
The normal range of capnography is 35 mmHg to 45 mmHg. If the value falls into this
range, post-intubation correlates with a properly positioned tube.
If the capnography value were zero, this would indicate that the endotracheal tube was not
in the airway. Most likely, this would indicate esophageal intubation, and repeat
endotracheal intubation would need to be performed.
Capnography assesses ventilation, which is the movement of air in and out of the lungs. It
does not assess oxygenation.
If a capnography reading is above 45 mmHg, the endotracheal tube is in the correct
position. However, the patient will need more ventilation, and there will be changes in the
ventilator machine.

Go to the next page if you knew the correct answer, or click the link image(s) below to further
research the concepts in this question (if desired).

Research Concepts:
Airway Management

We update eBooks quarterly and Apps daily based on user feedback. Please tap flag to
report any questions that need improvement.
Question 716: A 23-year-old man presents to the emergency department with high-grade
fever, confusion, headache, and photophobia. He has recently come back from a trip to Southeast
Asia. His symptoms were mild for the past three days but have flared up in the past couple of
hours. He has had HIV for the last 5 years. On examination, the patient is visibly uncomfortable
due to the bright lights. His thighs flex every time his head is lifted. His right eye is in the "down
and out" position. A lumbar puncture is performed. Which of the following findings is most
likely to be seen in this patient's cerebrospinal fluid (CSF)?

Choices:
1. A glucose level of 20 mg/dL
2. Neutrophil count of 1000 cells/microL
3. A glucose level of 120 mg/dL
4. A protein level of 20 mg/dL
Answer: 1 - A glucose level of 20 mg/dL
Explanations:
This patient most likely has tuberculous meningitis. (TBM), as evidenced by his symptoms
of meningitis, possible exposure to tuberculosis (TB), and a history of HIV. TBM often
presents with cranial nerve palsies as well.
Tuberculous meningitis assessment is by obtaining cerebrospinal fluid (CSF) for analysis.
Typically, the CSF reveals low glucose, elevated protein, and modestly elevated WBC
count with a lymphocytic predominance. The CSF analysis most closely resembles the CSF
analysis of viral meningitis.
First-line antituberculous treatments have excellent CSF penetration. Treatment for TBM
consists of two months of daily isoniazid (INH), rifampin (RIF), pyrazinamide (PZD), and
either streptomycin (SM) or ethambutol (EMB). This regimen is then followed by 7 to 10
months of INH and RIF.
Confirming the diagnosis of TB is a challenging diagnostic dilemma; this is especially true
in resource-poor areas. Definitive diagnosis results from the identification of
mycobacterium tuberculosis (MTB) in the CSF. Standard Ziehl-Neelsen acid-fast bacilli
(AFB) identification smears from CSF are highly unreliable. A neutrophil count of 1000
cells/microL is seen in bacterial meningitis.

Go to the next page if you knew the correct answer, or click the link image(s) below to further
research the concepts in this question (if desired).

Research Concepts:
Tuberculous Meningitis

We update eBooks quarterly and Apps daily based on user feedback. Please tap flag to
report any questions that need improvement.
Question 717: A 16-year-old female is brought to the emergency department with the
complaint of generalized paralysis. History taken from the friend reveals that she had some
seafood in the evening. Otherwise, her past medical history is insignificant. Vital signs are pulse
rate 95/min, blood pressure 135/85 mmHg, respirations 27/min, and temperature 98.6 F (37 °C).
Physical examination reveals a generalized decreased muscle tone. What is the next step in the
management of this patient?

Choices:
1. Sodium bicarbonate
2. Intravenous immunoglobulin
3. Respiratory support and supportive care
4. Calcium gluconate
Answer: 3 - Respiratory support and supportive care
Explanations:
Tetrodotoxin is a neurotoxin that is most commonly found in marine animals. The toxin
blocks sodium channels leading to gastrointestinal, neurological, and cardiac symptoms in
poisoned patients. There is currently no known antidote.
A monoclonal antibody targeted against TTX (Anti-TTX) is available. No studies on
efficacy have been published. Since there is no known antidote for tetrodotoxin, the
management is interprofessional. All symptomatic patients must be monitored closely in the
ICU and resuscitated according to the trauma ABCDE protocol. Specialists should be
involved depending on the organ involved.
Respiratory support and supportive care is the mainstay of treatment for TTX poisoning.
There is no specific antidote. TTX is excreted through the urine. Hemodialysis may be
required for patients with renal pathology.
There is no specific treatment for TTX poisoning. For suspected tetrodotoxin poisoning,
patients should be observed in the intensive care unit (ICU) for 24 hours due to some
patients having a delayed onset of symptoms of up to 20 hours.

Go to the next page if you knew the correct answer, or click the link image(s) below to further
research the concepts in this question (if desired).

Research Concepts:
Tetrodotoxin Toxicity

We update eBooks quarterly and Apps daily based on user feedback. Please tap flag to
report any questions that need improvement.
Question 718: A 32-year old male commercial marine diver is evaluating a sailing vessel
for repairs at 4-8 meters below sea level. He has no past medical history and takes no
medications. He drinks alcohol socially and does not smoke tobacco products, vape, or use illicit
drugs. His SCUBA equipment becomes entangled, and his regulator hose is damaged. He
coughs, loses his regulator, and his face mask starts to fill with water. He rapidly ascends to the
surface without stopping or exhaling. At the surface, he is unconscious, seizing, and lifeless.
Advanced cardiac life support is initiated, and he is immediately transported to the local hospital.
He regains consciousness, is confused, dizzy, and dysarthric. Chest x-ray reveals
pneumomediastinum and bibasilar infiltrates, but no pneumothorax. Electrocardiogram reveals
sinus tachycardia without ischemia. An intravenous crystalloid solution is infusing, and
supplemental oxygen at 100% non-rebreather is in place. Which of the following is the next best
step in the management of this patient?

Choices:
1. Broad-spectrum antibiotic therapy
2. Hyperbaric oxygen recompression therapy
3. Computed tomography (CT) scan of the head
4. Electroencephalogram (EEG) and intravenous levetiracetam
Answer: 2 - Hyperbaric oxygen recompression therapy
Explanations:
Air-gas emboli are formed when pneumomediastinum air is forced into the capillaries,
pulmonary veins, and central circulation. These bubbles collect in the heart or brain and
obstruct blood flow.
Initial presentation of cerebral arterial gas embolism is the loss of consciousness, seizure,
and/or cardiac arrest due to lack of blood flow in the large vessels or from air gas emboli in
the brainstem. Air-gas emboli can also cause headaches, confusion, numbness, tingling,
paresthesia, fatigue, hemiplegia, paralysis, aphasia, or any stroke symptom.
The damaged epithelial and endothelial cells release inflammatory mediators. This damage
to the cells leads to leaky vessels that cause swelling and edema. In the brain, this manifests
as cerebral edema.
The need for recompression and hyperbaric oxygen therapy needs to efficiently and
accurately assessed for the patient. Prompt treatment within the first two hours provides the
most benefit and resolution of symptoms. Delays in recompression and hyperbaric oxygen
therapy of more than 6 hours are associated with worse outcomes.

Go to the next page if you knew the correct answer, or click the link image(s) below to further
research the concepts in this question (if desired).

Research Concepts:
Pulmonary Barotrauma

We update eBooks quarterly and Apps daily based on user feedback. Please tap flag to
report any questions that need improvement.
Question 719: A 27-year-old woman with a history of depression, insomnia, anemia, and
fibromyalgia is brought to the emergency department after reported ingestion of an unknown
toxin about 45 minutes before arrival. She texted her family to say she was going to kill herself
by taking a bottle of one of her medications. Her family called 911, and she was promptly taken
to the emergency department. On arrival, her blood pressure is 150/85 mmHg, temperature 37.0
C (98.6 F), and her pulse is 102/min. She is awake but uncooperative with providing a history of
what she ingested. She is quiet with poor eye contact, but she is speaking clearly and appears to
be protecting her airway. Chart review of a recent outpatient visit provides a medication list,
including acetaminophen, amitriptyline, ferrous sulfate, paroxetine, and tramadol. For which of
the following medications of hers would the administration of activated charcoal be ineffective in
reducing absorption?

Choices:
1. Acetaminophen
2. Amitriptyline
3. Ferrous sulfate
4. Paroxetine
Answer: 3 - Ferrous sulfate
Explanations:
Activated charcoal best adsorbs toxins in their nonionized forms. Polar, water-soluble
molecules are less likely to be adsorbed. Due to the pharmacodynamics of activated
charcoal, it best absorbs nonpolar, poorly water-soluble organic toxins.
Most ingested toxins will have decreased systemic absorption in the presence of activated
charcoal, including acetaminophen, aspirin, barbiturates, tricyclic antidepressants,
theophylline, phenytoin, and a majority of inorganic and organic materials.
Activated charcoal does not effectively adsorb alcohols, metals such as iron and lithium,
electrolytes such as magnesium, potassium, or sodium, and acids or alkalis due to the
polarity of these substances.
In this case, iron supplementation (ferrous sulfate) would not be bound by activated
charcoal as it is an ionized toxin. There would be no benefit to activated charcoal
administration in the setting of an iron overdose.

Go to the next page if you knew the correct answer, or click the link image(s) below to further
research the concepts in this question (if desired).

Research Concepts:
Activated Charcoal

We update eBooks quarterly and Apps daily based on user feedback. Please tap flag to
report any questions that need improvement.
Question 720: A 22-year-old man with a history of cystic fibrosis-associated liver disease
presents with worsening abdominal distension and jaundice. On physical examination, the
patient's temperature is noted to be 37.7 °C, heart rate of 88/min and blood pressure
100/60mmHg. The patient is noted to have scleral icterus, palmar erythema, and moderate
ascites. Laboratory results show hemoglobin 10.0 g/dL (13-17 g/dL), platelet count 69 x 10^9/L
(150-400 x 10^9/L), serum creatinine 2.4 mg/dL (0.8-1.3 mg/dL), sodium 133 mmol/L (135-145
mmol/L), total bilirubin 4.2 mg/dl (2-20 µmol/L), albumin 2.4 g/dL (35-50 g/L), AST 110 U/L
(5-30 U/L), ALT 100U/L (5-30 U/L) ,ALP 289 U/L (50-100 U/L) and INR of 2.1. An ultrasound
examination of the abdomen shows significant ascites and nodular liver consistent with cirrhosis.
A CT scan of the abdomen is negative for biliary ductal dilatation or biliary obstruction. What is
the most appropriate next step in the management of this patient?

Choices:
1. Arrange an urgent magnetic resonance cholangiopancreatography (MRCP) examination
2. Schedule the patient for an outpatient endoscopic retrograde cholangiopancreatography
(ERCP) examination
3. Transfer the patient to ICU for close supportive management
4. Refer the patient to a transplant center for liver transplantation
Answer: 4 - Refer the patient to a transplant center for liver transplantation
Explanations:
Liver transplantation confers a substantial survival advantage in cystic fibrosis liver disease
(CFLD) patients but the clinical criteria and timing of liver transplantation in CFLD are not
well established.
Debray et al recommend liver transplantation should be considered if CFLD patients
demonstrate progressive hepatic failure characterized by progressive hypoalbuminemia and
coagulopathy, worsening jaundice and ascites, variceal bleeding not controlled by
conventional means, hepatopulmonary and portopulmonary syndromes, severe malnutrition
despite intensive nutritional provision, worsening pulmonary function (FEV1/FVC 50%)
and declining quality of life linked to liver disease.
This patient would benefit from a referral to a transplant center considering he has
progressive hepatic dysfunction characterized by a high MELD score (30).
Combined lung and liver transplantation (CLLT) should be considered for patients with
advanced pulmonary and liver disease.

Go to the next page if you knew the correct answer, or click the link image(s) below to further
research the concepts in this question (if desired).

Research Concepts:
Cystic Fibrosis And Liver Disease

We update eBooks quarterly and Apps daily based on user feedback. Please tap flag to
report any questions that need improvement.
Question 721: A 40-year-old male presents to the clinic with worsening shortness of breath
(stage IV NYHA). He has been admitted twice in the last one year for ST-segment elevation
myocardial infarction. During both the admissions, work-up with coronary artery catheterization
showed normal coronary arteries, blood alcohol levels > 0.2 g/dL, and urine toxicology positive
for cocaine. An echocardiogram done in the last admission, three months ago, showed left
ventricular ejection fraction (LVEF) 15%. The rest of the laboratory work-up showed creatinine
0.9 mg/dL, hemoglobin 10 g/dL, hematocrit 30%, platelets 150,000/mircoliter. INR 1.2.
Treatment was initiated with lisinopril and spironolactone on discharge. On evaluation in the
clinic today, he mentions he is compliant with his medications and wants to explore other
treatment options. An echocardiogram is repeated, which continues to show LVEF of 17%. He
continues to endorse the intermittent use of alcohol and recreational drugs. Which of the
following is a contraindication to LVAD placement in the patient?

Choices:
1. Creatinine 0.9 mg/dL
2. Hemoglobin 10 g/dL
3. INR 1.2
4. Blood alcohol level 0.2 and positive urine toxicology for cocaine
Answer: 4 - Blood alcohol level 0.2 and positive urine toxicology for cocaine
Explanations:
End-stage renal disease (glomerular filtration rate 30 or creatinine clearance 30) is a
contraindication to LVAD placement.
Active severe bleeding, chronic thrombocytopenia, and refusal to blood transfusions is a
contraindication to LVAD placement.
Severe liver disease (bilirubin 2.5 or international normalized ratio > 2.0 with cirrhosis or
portal hypertension) is a contraindication to LVAD placement.
Psychosocial elements such as evidence of ongoing alcohol, smoking or drug use or
dependency, inability to adhere to medical regimen, active mental illness, and poor social
support are contraindications to LVAD placement.

Go to the next page if you knew the correct answer, or click the link image(s) below to further
research the concepts in this question (if desired).

Research Concepts:
Left Ventricular Assist Devices

We update eBooks quarterly and Apps daily based on user feedback. Please tap flag to
report any questions that need improvement.
Question 722: A 44-year-old man is evaluated for dry cough and progressive dyspnea for
the last six days. Dyspnea is limiting his ability to exercise. He is also reporting fevers, myalgia,
and headache. His temperature is 100.4 F (38 C), O2 saturation is 92% on 3 liters via nasal
cannula, heart rate is 122/min, and respiratory rate is 26/min. The chest radiograph reveals
bilateral interstitial infiltrates. On physical exam, he has bilateral crackles all over the lung field.
A high-resolution CT scan of the chest shows extensive ground-glass changes bilaterally. One
hour after the initial presentation, his breathing gets worse, and O2 saturation drops. The patient
gets intubated and placed on mechanical ventilation. What is the best next diagnostic test?

Choices:
1. Test for influenza A and B
2. Bronchoalveolar lavage
3. Bronchoscopy
4. Schedule for lung biopsy
Answer: 1 - Test for influenza A and B
Explanations:
Acute interstitial pneumonia (AIP) is an acute, rapidly progressive idiopathic pulmonary
disease that often leads to fulminant respiratory failure and acute respiratory distress
syndrome (ARDS). Symptoms usually start as viral-like prodrome followed by shortness of
breath with cough, and fever which then progress rapidly to acute respiratory distress. The
physical exam is non-specific with hypoxia, tachypnea, and bilateral diffuse crackles. Many
of these patients are severely hypoxemic and require mechanical ventilation. It is an
idiopathic disease with unknown etiology, and diagnosis is only confirmed after excluding
other causes of ARDS. Influenza screening is important, as well as serology for atypical
organisms and fungus.
Excluding cardiac causes of pulmonary edema is essential. Echocardiography is needed to
rule out underlying cardiomyopathy or valve dysfunction.
Microbiologic workup should include blood and sputum culture. Legionella is an atypical
organism that can be detected on a urine antigen test.
Bronchoscopy with bronchoalveolar lavage (BAL) is needed to exclude diffuse alveolar
hemorrhage (DAH), eosinophilia, or malignant infiltrates but is not the initial step. If all the
previous workup fails to achieve an alternative diagnosis, lung biopsy might be indicated.

Go to the next page if you knew the correct answer, or click the link image(s) below to further
research the concepts in this question (if desired).

Research Concepts:
Acute Interstitial Pneumonia

We update eBooks quarterly and Apps daily based on user feedback. Please tap flag to
report any questions that need improvement.
Question 723: A 35-year old female is brought to the emergency department due to an
altered level of consciousness after an intentional drug overdose. She was diagnosed with
multiple sclerosis 5 years ago and was recently prescribed a medication that acts on central alpha
2 receptors to treat her symptoms of muscle spasticity. The physical examination is significant
for a pulse rate of 50/min, a blood pressure of 90/50 mm Hg, respiratory rate of 10/min, and
diminished deep tendon reflexes. Despite initial resuscitation, her vitals fail to improve.
Administration of which of the following is the next best step?

Choices:
1. Intralipid
2. Sodium bicarbonate
3. Naloxone
4. Norepinephrine
Answer: 4 - Norepinephrine
Explanations:
Alpha-agonist toxicity may occur accidentally or intentionally. The primary mechanism of
the toxicity of alpha agonists is through central alpha-2 agonism.
Stimulation of central alpha-2 receptors causes decreased secretion of catecholamines
through a negative feedback mechanism.
Norepinephrine is indicated for bradycardia and hypotension secondary to central alpha-2
agonist toxicity that does not respond to stimulation and intravenous fluids.
Norepinephrine provides direct replacement of the inhibited catecholamines in this setting.

Go to the next page if you knew the correct answer, or click the link image(s) below to further
research the concepts in this question (if desired).

Research Concepts:
Alpha Receptor Agonist Toxicity

We update eBooks quarterly and Apps daily based on user feedback. Please tap flag to
report any questions that need improvement.
Question 724: A 75-year-old man with chronic atrial fibrillation presents to the clinic for a
follow-up. He has been taking digoxin 0.25 mg per day for 10 years. He recently had a diagnosis
of peptic ulcer disease with positive H. pylori and was prescribed clarithromycin and
omeprazole. He has had increased nausea, anorexia, fatigue, and exercise intolerance. Which of
the following is the most likely cause of his complaints?

Choices:
1. Inhibition of liver function by omeprazole
2. Decreased protein-binding of digoxin
3. Decreased inactivation of digoxin by enteric bacteria
4. Decreased fluid intake
Answer: 3 - Decreased inactivation of digoxin by enteric bacteria
Explanations:
In 10% to 15% of patients receiving digoxin, a significant amount of the drug can get
inactivated in the gastrointestinal tract by enteric bacteria, primarily Eubacterium lentum.
Classic drug-drug interaction occurs with the macrolides, which sterilize the gastrointestinal
tract of this bacterium and cause increased digoxin absorption.
Of the macrolides, clarithromycin is the most cited offending medication and is often part of
the tripe cocktail for H. pylori.
Malnutrition can lead to decreased protein and albumin levels, but digoxin is not highly
protein-bound. Other drugs that can potentially interact with digoxin include quinidine,
verapamil, diltiazem, amiodarone, and spironolactone. Omeprazole does not interact with
digoxin to cause toxicity. Providers prescribing medications to patients taking digoxin
should be aware of the possible drug interactions that exist and have an understanding of the
pharmacokinetics of digoxin. Fluid intake is important with patients on digoxin, but often
with chronic heart failure, fluid restriction is needed.

Go to the next page if you knew the correct answer, or click the link image(s) below to further
research the concepts in this question (if desired).

Research Concepts:
Cardioactive Steroid Toxicity

We update eBooks quarterly and Apps daily based on user feedback. Please tap flag to
report any questions that need improvement.
Question 725: A 65-year-old man with a past medical history of hypertension and diabetes
mellitus presents to the emergency department with an inability to move either of his lower
extremities and loss of pain and temperature sensation below his waist. His symptoms came on
suddenly approximately three hours ago. A full neurologic exam shows 0/5 power in both lower
extremities, but his proprioceptive and fine touch sensations to be normal. However, after an
hour in the emergency department, he begins to complain of suprapubic discomfort, and an
overly distended bladder on ultrasound is discovered. Which of the following additional
complications is most likely to occur in this patient?

Choices:
1. Tachycardia
2. Ankle clonus
3. Hoffman sign
4. Hypotension
Answer: 4 - Hypotension
Explanations:
This patient’s clinical presentation is characteristic of anterior cord syndrome, caused by
ischemia or infarction of the anterior spinal artery supplying the anterior two-thirds of the
spinal cord.
The lateral horns of the spinal cord span from T1 to L2, and can be affected in anterior cord
syndrome depending on the location of the lesion.
The lateral horns house the neuronal cell bodies of the sympathetic nervous system.
Damage to this region of the spinal cord can lead to autonomic dysfunction.
This patient has an overdistended bladder, which indicates that he is experiencing
autonomic dysfunction. Another common manifestation of autonomic dysfunction in
anterior cord syndrome is hypotension.

Go to the next page if you knew the correct answer, or click the link image(s) below to further
research the concepts in this question (if desired).

Research Concepts:
Anterior Cord Syndrome

We update eBooks quarterly and Apps daily based on user feedback. Please tap flag to
report any questions that need improvement.
Question 726: A 45-year-old man with a history of alcohol use disorder is brought to the
hospital for severe epigastric pain and nausea. He has had eight similar admissions in the past
two years. Vital signs show a blood pressure of 102/64 mmHg, pulse 94/min, respiratory rate
20/min, and SpO2 95% on room air. Initial labs show hemoglobin 11.3 g/dL, WBC count
17000/microL, platelets 71000/microL, blood glucose 284 mg/dL, serum lipase 290 IU/L, serum
AST 475 IU/L, serum calcium 8.1 mg/dL, PO2 80 mmHg, base deficit 3 mEq/L, serum
creatinine 1.1 mg/dL, serum BUN 16 mg/dL, and serum LDH 640 IU/L. The patient is given IV
normal saline and admitted to the hospital. Ranson criteria are being used to assess the patient's
mortality. Which of the following is the minimum time required to reassess the patient's labs to
complete the criteria?

Choices:
1. 12 hours
2. 24 hours
3. 48 hours
4. 72 hours
Answer: 3 - 48 hours
Explanations:
Ranson criteria are used to predict the severity and mortality of acute pancreatitis. Five
parameters are assessed on admission, and the other six are assessed at 48 hours post-
admission. One point is given for each positive parameter for a maximum score of 11. The
modified criteria have a max score of 10. Five parameters assessed on admission and the
other 5 at the 48-hour mark.
The criteria with 11 parameters are used to assess the severity of alcoholic pancreatitis. The
5 parameters on admission are age older than 55 years, WBC count greater than 16,000
cells/mm^3, blood glucose greater than 200 mg/dL (11 mmol/L), serum AST greater than
250 IU/L, and serum LDH greater than 350 IU/L.
At 48 hours, the remaining 6 parameters are: serum calcium less than 8.0 mg/dL (less than
2.0 mmol/L), hematocrit fall greater than 10%, PaO2 less than 60 mmHg, BUN increased
by 5 or more mg/dL (1.8 or more mmol/L) despite intravenous (IV) fluid hydration, base
deficit greater than 4 mEq/L, and sequestration of fluids greater than 6 L.
Score Interpretation: 0 to 2 points: mortality 0% to 3%, 3 to 4 points: 15%, 5 to 6 points:
40%, 7 to 11: nearly 100%.

Go to the next page if you knew the correct answer, or click the link image(s) below to further
research the concepts in this question (if desired).

Research Concepts:
Ranson Criteria

We update eBooks quarterly and Apps daily based on user feedback. Please tap flag to
report any questions that need improvement.
Question 727: A 51-year-old patient with alcohol use disorder is seen in the ER with
complaints of a fever, chills, night sweats and anorexia. He says he has had productive thick
sputum for the past 2 days and has a persistent cough. He describes his sputum as red and
viscous. He has a WBC of 18 and a left shift. The rest of the exam is unremarkable. Chest x-ray
reveals a possible consolidation. What organism is most likely causing this infection?

Choices:
1. Mycobacterium tuberculosis
2. Staphylococcus aureus
3. Klebsiella pneumoniae
4. Aspergillosis
Answer: 3 - Klebsiella pneumoniae
Explanations:
Patients with Klebsiella typically present with an acute onset of high fever, chills, and flu-
like symptoms.
Most patients with Klebsiella will develop a productive cough with abundant, thick,
tenacious, and blood-tinged sputum sometimes called currant jelly sputum.
In the U.S., 66% of patients have alcohol use disorder. In some parts of Asia it causes up to
40% of community acquired pneumonia in the elderly.
Mortality is from 50 to 100%.

Go to the next page if you knew the correct answer, or click the link image(s) below to further
research the concepts in this question (if desired).

Research Concepts:
Bacterial Pneumonia

We update eBooks quarterly and Apps daily based on user feedback. Please tap flag to
report any questions that need improvement.
Question 728: A 65-year-old man with a past medical history of heart failure with reduced
ejection fraction (HFrEF) and morbid obesity is brought to the emergency department after a
head trauma after a fall at home. The patient is poorly responsive. Vital signs show blood
pressure 110/60 mmHg, pulse 110/min, respiratory rate 8/min, temperature 98.6 F, and SpO2
94% on room air. The patient is admitted to the ICU. Transthoracic echocardiography is planned
to assess the patient's fluid status. However, the patient is found to have poor cardiac windows,
and cardiac function and volume status could not be assessed. Which of the following is the next
best step in the management of this patient?

Choices:
1. Transesophageal echocardiography
2. Endotracheal intubation
3. Fluid resuscitation
4. Vasopressors
Answer: 2 - Endotracheal intubation
Explanations:
The patient should be intubated due to poor mental status with respiratory depression to
help ventilation.
Transesophageal echocardiography is absolutely contraindicated in a patient with
respiratory depression.
The airway should be secured before making any attempts for TEE.
The decision between IVF and vasopressors can be made once the patient has a secure
airway.

Go to the next page if you knew the correct answer, or click the link image(s) below to further
research the concepts in this question (if desired).

Research Concepts:
Intraoperative Echocardiography

We update eBooks quarterly and Apps daily based on user feedback. Please tap flag to
report any questions that need improvement.
Question 729: A 72-year-old man is admitted to the emergency department (ED) with a
complaint of tearing chest pain that started about an hour ago and has progressively increased in
severity since that time. He states he has never experienced pain like this before. He rates his
pain as a 9 on a scale of 0-10. He has a past medical history of well-controlled hypertension,
atherosclerosis, and renal failure for which he undergoes dialysis. His BMI is 43. Imaging
obtained in the ED confirms a diagnosis of aortic dissection, and the patient is prepared for
transfer to the operating room for emergent repair. The anesthetist completes his pre-operative
assessment and designates this patient as ASA-PS VE. Which aspect of this patient’s history
most likely classifies him as ASA-PS VE?

Choices:
1. Diagnosis of acute aortic dissection
2. History of renal failure and dialysis
3. Well-controlled hypertension
4. BMI of 43
Answer: 1 - Diagnosis of acute aortic dissection
Explanations:
Acute aortic dissection is a constant threat to life and an emergency. The patient is not
expected to live without surgical intervention.
This diagnosis would designate this patient as ASA-PS V, while the emergency presentation
justifies the "E" added to the classification.
This is the most significant active problem this patient is experiencing and is the top priority
when assessing this patient.
History of renal failure would cause this patient to be classified incorrectly as ASA-PS III.
The acute aortic dissection is the most significant active problem, therefore the patient
would be more correctly classified according to the presence of aortic dissection. Well-
controlled hypertension alone would designate this patient as ASA-PS II. The presence of
an emergent aortic dissection takes precedence when assessing a patient and assigning the
proper ASA-PS classification. BMI of 43 is considered morbid obesity, which alone would
classify a patient as ASA-III. Although this patient is morbidly obese, the aortic dissection
is emergent and takes precedence as the most significant active problem. It would be more
correct to classify the patient as ASA VE.

Go to the next page if you knew the correct answer, or click the link image(s) below to further
research the concepts in this question (if desired).

Research Concepts:
American Society of Anesthesiologists Staging

We update eBooks quarterly and Apps daily based on user feedback. Please tap flag to
report any questions that need improvement.
Question 730: A 26-year-old female developed seizures in the recovery room following her
wisdom teeth extraction under general anesthesia. She was febrile and was hemodynamically
unstable. She developed tachycardia and showed minimal respiratory efforts. She was intubated
and given was lorazepam. Paramedics transferred her to the emergency room from the surgical
center. Examination showed blood pressure of 155/90 mm Hg, heart rate of 135, and temperature
of 42.2 degrees Celsius. She was unresponsive to pain. Her pupils were 3 mm equal and reactive
to light. There were increased muscle tone and hyperreflexia. Her ECG showed sinus tachycardia
with peaked T-waves. Which is the next plan in managing the patient?

Choices:
1. Paralyze and ventilate
2. Creatinine Kinase
3. Dantrolene sodium
4. Escalate her antiepileptic medication
Answer: 3 - Dantrolene sodium
Explanations:
The patient most likely has malignant hyperthermia, which is a hypermetabolic response to
some potent volatile anesthetic gases.
In susceptible individuals, with a mutation in the Ryanodine receptor 1 (RYR-1) gene, some
medications induce the excessive release of stored calcium ions within muscle cells. The
resulting increase in calcium concentrations within the cells leads to unopposed muscle
contractions. This leads to hyperthermia and metabolic acidosis.
Dantrolene depresses excitation-contraction coupling in skeletal muscle by acting as a
receptor antagonist to the ryanodine receptor and decreasing free intracellular calcium
concentration.
Hypothermia and antiepileptics are only adjunctive treatment in malignant hyperthermia.

Go to the next page if you knew the correct answer, or click the link image(s) below to further
research the concepts in this question (if desired).

Research Concepts:
Malignant Hyperthermia

We update eBooks quarterly and Apps daily based on user feedback. Please tap flag to
report any questions that need improvement.
Question 731: A 60-year-old man with an unknown past medical history is brought to the
emergency department after being found unconscious on a park bench. On examination, the
patient is lethargic but arousable to sternal rub. Vital signs reveal a blood pressure of 100/60
mmHg, pulse rate of 60 per minute, respiratory rate of 24 per minute, and temperature of 33 C
(91.4 F). Physical exam shows a middle-aged gentleman with moist clothing that smells of
alcohol. There are no signs of trauma, and the rest of his physical exam is unremarkable. His wet
clothing is removed, and he is started on warm IV fluids and placed in a warming blanket. After
three hours, the patient's temperature remains 33 C (91.4 F). Which of the following is the best
next step in the management of this patient?

Choices:
1. Pleural irrigation
2. Venoarterial extracorporeal membrane oxygenation
3. Antibiotics and glucocorticoids
4. Cardiopulmonary bypass
Answer: 3 - Antibiotics and glucocorticoids
Explanations:
Mild hypothermia is defined by an estimated core temperature of 32 to 35 C (90 to 95 F).
Moderate hypothermia is an estimated core temperature of 28 to 32 C (82 to 90 F). Severe
hypothermia is an estimated core temperature of 28 C (82 F). Passive rewarming is the
modality of choice for mild hypothermia. It may also be used as an adjunct for a patient
undergoing aggressive rewarming for moderate to severe hypothermia.
Active external rewarming is indicated in moderate to severe hypothermia and for a patient
with mild hypothermia who is unstable, lacks physiological reserves, or failed initial
treatment. Active internal rewarming is indicated in severe hypothermia or a patient with
moderate hypothermia who failed to respond to less aggressive techniques. Venoarterial
extracorporeal membrane oxygenation is used in extreme cases (cardiac arrest, frozen
limbs) or if all other measures before have failed.
A patient that does not adequately respond to rewarming or has symptoms that do not match
the severity of their hypothermia should have further testing to rule out other possible
causes of hypothermia such as infection, hypothyroidism, hypoglycemia, etc. Empirically
treating with antibiotics and glucocorticoid may provide more benefits than harm as
infection and adrenal insufficiency are possible etiologies of hypothermia that do not
respond to adequate rewarming.
The patient in this question stem has mild hypothermia that is not responding to adequate
treatment. Pleural irrigation is an active internal rewarming technique reserved for patients
with moderate or severe hypothermia who failed to respond to less aggressive techniques.
Venoarterial extracorporeal membrane oxygenation and cardiopulmonary bypass are highly
invasive, so they are reserved for extreme cases or if all other measures before have failed.

Go to the next page if you knew the correct answer, or click the link image(s) below to further
research the concepts in this question (if desired).

Research Concepts:
Hypothermia

We update eBooks quarterly and Apps daily based on user feedback. Please tap flag to
report any questions that need improvement.
Question 732: A construction worker fell 20 feet (6 m) and suffered a head injury. In the
emergency department, he is unresponsive and posturing with his forearms flexed close to his
body and legs rigidly extended with internal rotation and plantar flexion. What is the Glasgow
coma scale motor score allocated for such a response seen in the patient?

Choices:
1. Two
2. Three
3. Four
4. Five
Answer: 2 - Three
Explanations:
Decorticate posturing involves the arms flexed close to the body with internal rotation and
the legs stiffly extended.
Decorticate posturing indicates an injury above the red nucleus in the midbrain. The red
nucleus governs the flexion of the upper limbs and extension of the lower limbs.
The decorticate posturing is given a score of three while assessing motor response.
The decerebrate posturing has a motor score of two. The motor score of one indicates no
response to painful stimuli in the patient.

Go to the next page if you knew the correct answer, or click the link image(s) below to further
research the concepts in this question (if desired).

Research Concepts:
Decorticate Posturing

We update eBooks quarterly and Apps daily based on user feedback. Please tap flag to
report any questions that need improvement.
Question 733: A 16-year-old male patient is brought to the hospital by his parents with
complaints of persistent cough for one week and excessive use of his blue inhaler. Previously, he
had been well on fluticasone/salmeterol inhaler and montelukast daily. His parents are concerned
that he may not have taken his medication because of a domestic quarrel. On examination, his
respiratory rate is 32/min, peak expiratory flow rate (PEFR) is 235 L/min, (predicted is 590
L/min), and oxygen saturation is 89%. He is started on oxygen therapy, salbutamol and
ipratropium bromide, nebulizers, and IV hydrocortisone. Later in the evening, he is found to
have pO2 of 7.6 kPa, pCO2 of 5.9 kPa, pH of 7.36, respiratory rate of 15/min, and PEFR of 230
L/min. What is the most appropriate next step in the management of this patient?

Choices:
1. IV aminophylline
2. Consult ICU team
3. IV magnesium
4. Back to back salbutamol nebulization
Answer: 2 - Consult ICU team
Explanations:
If a patient has received 3 doses of an inhaled bronchodilator and shows no response, the
factors that should be used to determine admission are the severity of airflow obstruction,
duration of asthma, response to medications, adequacy of home support, any mental illness.
The most appropriate next step, in this case, would be to involve the ICU team. Despite
management, his PEFR has declined further and he is making a less respiratory effort now.
There is evidence of hypoxemia with hypercapnia on arterial blood gases which should be
taken as an indication to include ICU physicians.
While back to back nebulizations, IV magnesium and IV aminophylline are all needed,
calling the ICU team is of utmost importance. Mortality is higher when the ICU team is
involved late, such as in peri-arrest situations.

Go to the next page if you knew the correct answer, or click the link image(s) below to further
research the concepts in this question (if desired).

Research Concepts:
Asthma

We update eBooks quarterly and Apps daily based on user feedback. Please tap flag to
report any questions that need improvement.
Question 734: A 32-year-old right-handed lady with a past medical history of systemic
lupus erythematosus (SLE) presents to the emergency department with the symptoms of nausea,
dizziness, headache, confusion, vertigo, dysarthria, and bilateral central vision loss with the onset
of symptoms two hours ago. On examination, the temperature is 37 C, pulse rate is 90/min, blood
pressure is 165/73 mmHg, and the respiratory rate is 36/min breaths per minute. The patient is
alert but confused and complaining of a headache. Her blood glucose level is 93 mg/dL, and the
National Institute of Health stroke scale is 7. CT scan of the head without contrast is negative for
an acute bleed, and subsequent CT angiogram of head and neck shows a large vessel occlusion
(LVO) at the basilar tip. Which of the following steps is recommended at this time?

Choices:
1. MRI scan of head and neck to characterize the infarct
2. CT perfusion scan to measure the extent of the infarct
3. Mechanical thrombectomy
4. Antiplatelet medications
Answer: 3 - Mechanical thrombectomy
Explanations:
MRI scan of head and neck is not indicated at this time, and the patient should immediately
receive tissue plasminogen activator (tPA) such as alteplase and transferred to an
angiography suite for mechanical thrombectomy (MT).
At this point, there is no need to delay the treatment with unnecessary testing, and the
patient should immediately receive alteplase.
Mechanical thrombectomy (MT) is recommended in this case based on the recent DAWN
and DEFUSE-3 trials for anterior circulation.
Everyone should be considered for alteplase administration as per NINDS trial results
published in 1993, which showed significant mortality and outcomes benefits of alteplase
administration if the patient has arrived in the emergency department (ED) within the three
hours window and has no contraindication to alteplase. The patient should first receive
alteplase as she is within the tPA window. Although after tPA administration, the patient
should be moved to an angiography suite for MT as the symptom onset is within 24 hours,
and there is an LVO at the basilar tip, which would require MT.

Go to the next page if you knew the correct answer, or click the link image(s) below to further
research the concepts in this question (if desired).

Research Concepts:
Basilar Artery Infarct

We update eBooks quarterly and Apps daily based on user feedback. Please tap flag to
report any questions that need improvement.
Question 735: A patient diagnosed is diagnosed with diabetes insipidus. Appropriate fluids
are started. Which of the following medications is given?

Choices:
1. Glipizide 10 mg intravenously, daily
2. Glipizide 10 mg by mouth, daily
3. Vasopressin 10 units intravenously, daily
4. Vasopressin 5-10 units intramuscularly or subcutaneously, every 8 to 12 hours
Answer: 4 - Vasopressin 5-10 units intramuscularly or subcutaneously, every 8 to 12 hours
Explanations:
Vasopressin is an antidiuretic hormone replacement drug. It may be given intramuscularly,
subcutaneously, and nasally to treat diabetes insipidus, a disorder of the pituitary gland.
Adult dosing is 5 to 10 units intramuscularly or subcutaneously every 8 to 12 hours in the
acute management of diabetes insipidus. Pediatric dosing is 2.5 to 10 units intramuscularly,
subcutaneously, or intranasally every 8 to 12 hours. The dose is titrated based on serum
sodium and osmolarity, urine output, and overall fluid balance in both adults and pediatric
clients.
Normally, the hypothalamus produces antidiuretic hormone, which accumulates in the
posterior pituitary.
Glipizide is a therapy for diabetes mellitus, not diabetes insipidus.
Vasopressin is not given intravenously.

Go to the next page if you knew the correct answer, or click the link image(s) below to further
research the concepts in this question (if desired).

Research Concepts:
Diabetes Insipidus

We update eBooks quarterly and Apps daily based on user feedback. Please tap flag to
report any questions that need improvement.
Question 736: A 17-year-old man is brought to the emergency department by his friends
with bizarre behavior. They mention that the patient has been saying that he has been selected by
the “divine authorities to fight the evil in this world.” The patient has stopped attending classes
and stays in his room. He has not been eating or sleeping in the past week. They mention that he
has not been using drugs. The patient's interview supports this history. Vitals are within normal
limits. On examination, the patient appears disorganized, disheveled, has a pressured speech, and
is pacing across the room. Laboratory investigations reveal no abnormalities. He is admitted after
a diagnosis is made and started on appropriate treatment. Over the next week, the patient
becomes calmer. Eleven days after admission, he complains of a sudden increase in sweating and
stiffness of his muscles. Vital signs are temperature 102 F (38.8 C), blood pressure 162/91 mm
Hg, pulse 126/min, and respiratory rate of 24/min. All medications are stopped and intravenous
fluids are started. What medication should be given next if the patient does not improve with
these interventions?

Choices:
1. Cyproheptadine
2. Benztropine
3. Diphenhydramine
4. Dantrolene
Answer: 4 - Dantrolene
Explanations:
The patient initially had features of mania and was most likely treated with antipsychotics.
The patient has most likely developed the neuroleptic malignant syndrome.
Discontinuing the offending agent is paramount, followed by supportive therapy. This
includes aggressive cooling and correction of volume deficits and any electrolyte
imbalances.
Dantrolene is a muscle relaxant and acts on the ryanodine receptors. It is specific for the
neuroleptic malignant syndrome.
Cyproheptadine is used for serotonin syndrome. Benztropine and diphenhydramine are
useful for antipsychotic-induced extrapyramidal symptoms.

Go to the next page if you knew the correct answer, or click the link image(s) below to further
research the concepts in this question (if desired).

Research Concepts:
Neuroleptic Malignant Syndrome

We update eBooks quarterly and Apps daily based on user feedback. Please tap flag to
report any questions that need improvement.
Question 737: A 36-year-old man is brought by paramedics after being hit by a car. After
securing the airway and breathing, two large-bore intravenous lines are attached, and the patient
is started on normal saline. Vital signs after 3 liters of normal saline include respiratory rate 18
per minute, heart rate 110 beats per minute, and blood pressure 80/55 mmHg. Examination
shows a Glasgow coma scale of 9 with cold extremities and a weak tachycardic pulse. A FAST
scan shows intraabdominal fluid. What is the most appropriate next step in the management of
this patient's hypovolemia?

Choices:
1. Give packed red blood cells instead of normal saline
2. Start the patient on colloids
3. Continue IV normal saline
4. Perform emergency laparotomy
Answer: 4 - Perform emergency laparotomy
Explanations:
This patient is suffering from acute hypovolemia.
Acute blood loss results in peripheral vasoconstriction causing cold extremities.
After an acute trauma, patients who do not respond to normal saline bolus are called non-
responders.
Emergency laparotomy is necessary for non-responders, so the bleeding source may be
identified and repaired.

Go to the next page if you knew the correct answer, or click the link image(s) below to further
research the concepts in this question (if desired).

Research Concepts:
Hypovolemic Shock

We update eBooks quarterly and Apps daily based on user feedback. Please tap flag to
report any questions that need improvement.
Question 738: A 32-year-old woman with a past medical history of factor V Leiden
presents to the emergency department with complaints of sudden-onset weakness in her right
arm and leg for the past 3 hours. Glasgow coma scale score is 15/15, and the NIH stroke scale is
9. The patient has recently started taking combination oral contraceptive pills. Vitals show a
temperature of 38.4 C, blood pressure 130/80 mmHg, heart rate 98/min, and respiratory rate
17/min. Non-contrast CT imaging of the brain shows no hemorrhage. Initial labs are shown
below.
Patient value Reference range
Hemoglobin 6.9 g/dL 11.5-16 g/dL
4100-
WBC count 8800/microL
10900/microL
150000-
Platelet count 38000/microL
400000/microL
Creatinine 1.7 mg/dL 0.8-1.4 mg/dL
Total bilirubin 5.8 mg/dL 0.3-1.0 mg/dL
LDH 520 IU/L 90-230 IU/L
Sodium 139 mEq/L 134-144 mEq/L
Potassium 4.8 mEq/L 3.6-5.0 mEq/L
Glucose 142 mg/dL 65-100 mg/dL
Which of the following is the next best step in the management of this patient?

Choices:
1. Intravenous alteplase
2. Aspirin
3. Plasma exchange therapy
4. Nicardipine
Answer: 3 - Plasma exchange therapy
Explanations:
This scenario describes a case of thrombotic thrombocytopenic purpura (TTP).
TTP is a type of microangiopathic hemolytic anemia that classically has been characterized
by the pentad of fever, thrombocytopenia, hemolytic anemia, renal dysfunction, and
neurologic dysfunction. TTP can be either congenital or acquired. Acquired TTP is more
common than the congenital type and is caused by autoantibodies targeting ADAMTS13.
Antiplatelet drugs, immunosuppressive agents, HIV, estrogen-containing birth control, and
pregnancy are the most commonly listed triggers for ADAMTS13 autoantibody formation,
causing acquired TTP. Factor V Leiden has been implicated as a risk factor, although rare,
for TTP.
Although renal failure is part of the classic presentation, in practice, it is rare to have renal
dysfunction. The CNS is the most commonly affected end-organ, with manifestations that
can include a headache, focal neurologic deficits (ischemic stroke and TIA), seizures,
confusion, and vertigo.
The mainstay of treatment in TTP is plasma exchange with high dose steroids (1 mg/kg of
prednisone).

Go to the next page if you knew the correct answer, or click the link image(s) below to further
research the concepts in this question (if desired).

Research Concepts:
Ischemic Stroke

We update eBooks quarterly and Apps daily based on user feedback. Please tap flag to
report any questions that need improvement.
Question 739: A 16-year-old female presents to the emergency department following a
front impact motor vehicle collision. She is extremely short of breath and cannot talk, and seems
to be gasping for air. Physical examination reveals bruising in a linear pattern across the low
neck and chest. There is also a crunching sensation felt on palpation of the neck and the chest
around the sternal notch. What should be the next step in management?

Choices:
1. Needle decompression of possible pneumothorax
2. Portable chest x-ray
3. Awake fiberoptic intubation
4. Awake tracheostomy
Answer: 3 - Awake fiberoptic intubation
Explanations:
The patient is gasping for air and is unable to talk, indicating the possibility of laryngeal
injury. Impending airway loss due to suspected laryngeal injury must be managed with
direct visualization of the airway. Tracheostomy may be required and should be prepared
for, but awake fiberoptic intubation is a safe option to both assess the airway and proceed to
secure it.
Signs of laryngeal injury include hoarseness, stridor, subcutaneous air, seatbelt sign, or
other signs of an anterior neck injury.
Airway management of laryngeal injury without direct visualization of the airway above
and below the vocal cords may lead to the false passage of the endotracheal tube.
This patient may have pneumothorax as well, but the impending airway obstruction must be
managed first. A chest x-ray may be helpful to diagnose a pneumothorax, but should not
delay emergent airway management.

Go to the next page if you knew the correct answer, or click the link image(s) below to further
research the concepts in this question (if desired).

Research Concepts:
Laryngeal Injury

We update eBooks quarterly and Apps daily based on user feedback. Please tap flag to
report any questions that need improvement.
Question 740: A previously healthy 52-year-old man presents to the hospital with
substernal chest pain, severe worsening dyspnea with associated dry cough, abdominal pain, and
low-grade fever. He reports he recently traveled to North Africa but denies sick contacts. A chest
radiograph reveals diffuse bilateral opacities concerning for pulmonary edema. Abdominal
ultrasound reveals hepatosplenomegaly. Labs reveal unexplained pancytopenia, and arterial
blood gas shows profound hypoxemia. EKG reveals an acute ST-elevation myocardial infarction.
He undergoes cardiac catheterization, which shows multi-vessel coronary artery disease
requiring emergent surgical revascularization. During the procedure, a lung biopsy is performed,
which reveals foamy macrophages with a peribronchial thickening. A diagnosis of lipoid
pneumonia is made, and the patient improved with antibiotics, corticosteroids, and whole-lung
lavage. Which of the following is the most likely etiology of this patient's underlying condition?

Choices:
1. Accumulation of glucocerebroside in alveolar macrophages
2. Sphingomyelinase deficiency
3. Dysregulated immune response causing persistent stimulation of phagocytic cells
4. Acquired clonal stem cell disorder
Answer: 2 - Sphingomyelinase deficiency
Explanations:
Niemann-Pick disease is a rare, autosomal recessive, congenital lipid storage disorder
caused by a deficiency of the enzyme sphingomyelinase. Lack of this enzyme inhibits the
breakdown of sphingomyelin, which is a fat found in cell membranes. When the lung is
involved, endogenous lipoid pneumonia can be a presenting feature of this disease.
There are multiple variants of this disease. Patients with Niemann-Pick Type A are severe
with central nervous system involvement leading to death in early childhood. Niemann-Pick
type B is a less severe form without CNS involvement, and patients often survive into
adulthood. Interstitial lung disease is one of the major causes of morbidity and mortality due
to the accumulation of lipid-laden phagocytes throughout the lung parenchyma.
Characteristic histological findings in Niemann-Pick disease include foamy phagocytes
(lipid-laden macrophages) caused by intracellular deposition of sphingomyelin in various
tissues throughout the body. Bone marrow, liver, or lung biopsies often make the diagnosis.
Symptoms are dependent upon the organs that accumulate these toxic lipids. Common
clinical manifestations of Niemann-Pick disease include hyperlipidemia,
hepatosplenomegaly, thrombocytopenia, and interstitial lung disease.

Go to the next page if you knew the correct answer, or click the link image(s) below to further
research the concepts in this question (if desired).

Research Concepts:
Lipoid Pneumonia

We update eBooks quarterly and Apps daily based on user feedback. Please tap flag to
report any questions that need improvement.
Question 741: A 65-year-old male patient with a history of chronic obstructive pulmonary
disease (COPD) controlled on inhaled corticosteroids presents to the hospital complaining of
sudden onset dyspnea and left-sided pleuritis type chest pain. A chest radiograph is performed
which shows a 1.5 cm large area of air on the left side without lung markings. On aspiration, it
reduces to 0.75 cm. What is the most appropriate next step in management?

Choices:
1. Admit overnight for oxygen supplementation
2. Discharge with follow up after 24 hours
3. Admit for chest drain insertion
4. Repeat aspiration
Answer: 1 - Admit overnight for oxygen supplementation
Explanations:
A pneumothorax is defined as a collection of air outside the lung but within the pleural
cavity. It occurs when air accumulates between the parietal and visceral pleurae inside the
chest. The air accumulation can apply pressure on the lung and make it collapse. The degree
of collapse determines the clinical presentation of pneumothorax.
There are two types of pneumothorax: traumatic and atraumatic. The two subtypes of
atraumatic pneumothorax are primary and secondary. A primary spontaneous pneumothorax
(PSP) occurs automatically without a known eliciting event, while a secondary spontaneous
pneumothorax (SSP) occurs subsequent to an underlying pulmonary disease.
In secondary spontaneous pneumothorax, if size/depth of pneumothorax is less than 1cm
and no dyspnea then the patient is admitted, high flow oxygen is given and observation is
done for 24 hours.
If size/ depth is between 1-2cm, needle aspiration is done, then the residual size of
pneumothorax is seen, if the depth after the needle aspiration is less than 1cm management
is done with oxygen inhalation and observation and in case of more than 2cm, tube
thoracostomy is done. In case of depth more than 2cm or breathlessness, tube thoracostomy
is done.

Go to the next page if you knew the correct answer, or click the link image(s) below to further
research the concepts in this question (if desired).

Research Concepts:
Pneumothorax

We update eBooks quarterly and Apps daily based on user feedback. Please tap flag to
report any questions that need improvement.
Question 742: A 69-year-old man with a past medical history of hypertension, diabetes,
hyperlipidemia, and an abdominal aortic aneurysm is found to be unconscious. The patient was
in the hospital waiting room, visiting his family when he suddenly felt lightheaded and collapsed.
In the hospital, the rapid response is called. His vital signs show a blood pressure of 80/60
mmHg, heart rate of 140/min, and SpO2 95% on room air. The patient appears to be pale,
minimally responsive, tachycardic, having labored breathing, and bilateral abdominal
ecchymosis. Large wide bore intravenous access is obtained subsequently, and IV fluid
resuscitation is started. He is also intubated. Complete blood count and basic metabolic panels
are sent. What is the next best step in the management of this patient?

Choices:
1. CT abdomen/pelvis
2. Abdominal ultrasound
3. Close monitoring in the intensive care unit
4. Emergency laparotomy
Answer: 4 - Emergency laparotomy
Explanations:
The likely diagnosis given the patient's presentation is acute abdominal aortic aneurysmal
rupture.
Since the patient's hemodynamical instability, he would need urgent open surgical repair.
Delay in surgery carries high mortality in these patients.
CT of the abdomen and pelvis are not indicated for this patient as he is hemodynamically
unstable. Abdominal ultrasound is not indicated since the patient is thermodynamically
unstable. Bedside fast ultrasound can be done in patients with no known history of an
abdominal aneurysm but if clinical suspicion for aortic rupture is high. It is not appropriate
to transfer to ICU at this time as the patient needs emergent surgery.

Go to the next page if you knew the correct answer, or click the link image(s) below to further
research the concepts in this question (if desired).

Research Concepts:
Abdominal Aortic Aneurysm Rupture

We update eBooks quarterly and Apps daily based on user feedback. Please tap flag to
report any questions that need improvement.
Question 743: A 45-year-old male patient is brought to the hospital after a car accident.
The patient complains of severe back pain on arrival. A foley catheter is inserted and dark red
urine can bee seen in the tubing. He is sent for a CT scan with contrast to assess the status of his
kidneys. Eight hours later, the patient is anuric. A urinalysis is done which reveals the fractional
excretion of sodium to be 3% and the urinary sodium concentration to be 50 mEq/l. The
creatinine and BUN are elevated. What additional finding will be seen in the urine of this
patient?

Choices:
1. Hyaline casts
2. Muddy brown casts
3. Waxy casts
4. White blood cell casts
Answer: 2 - Muddy brown casts
Explanations:
This patient is suffering from acute tubular necrosis. Muddy casts in the urine are typical of
acute tubular necrosis. He was in a hypovolemic state as well as exposed to nephrotoxic
contrast during the CT scan.
The urinalysis of acute tubular necrosis shows muddy brown casts or renal tubular epithelial
cells secondary to the sloughing of tubular cells into the lumen due to ischemia or toxic
injury. The urinary sodium excretion determines that the kidney is sodium avid in
hypovolemic states (prerenal) where kidneys try to conserve sodium. Also, they lose sodium
due to tubular injury with values more than 40 to 50 mEq/L indicating acute tubular
necrosis and less than 20 mEq/L suggestive of prerenal disease.
Fractional excretion of sodium is a good test to differentiate between acute tubular necrosis
and prerenal disease with a value of less than 1% favoring prerenal disease and more than
2% supporting acute tubular necrosis. However, these values are not always accurate as in
chronic prerenal states such as congestive heart failure and cirrhosis in which there is an
overlap between both (ATN and prerenal AKI) the value is less than 1%.
Numerous biomarkers have evolved to detect AKI/acute tubular necrosis early as compared
to serum creatinine. These biomarkers include serum cystatin C to be an early and reliable
marker of renal injury as compared to serum creatinine which is often witnessed 48 to 72
hours after the initial insult. In prerenal disease, the urinalysis microscopy is normal or may
contain hyaline casts. White blood cell casts are seen in glomerulonephritis.

Go to the next page if you knew the correct answer, or click the link image(s) below to further
research the concepts in this question (if desired).

Research Concepts:
Acute Renal Tubular Necrosis

We update eBooks quarterly and Apps daily based on user feedback. Please tap flag to
report any questions that need improvement.
Question 744: A 28-year-old male presents with nausea and headache. He started
complaining of headaches about a week ago. Yesterday, the headaches are now accompanied by
nausea and double vision. He is positive for HIV but refused treatment. Examination shows BP
140/90 mmHg, HR 64 beats/min, respiratory rate, and temperature are normal. Cranial nerve
examination shows isocoric pupils, briskly reactive to light, bilateral papilledema on fundoscopy,
and bilateral lateral rectus palsy. The motor and sensory examinations are normal. Babinski is
positive bilaterally. The CT scan is normal. A lumbar puncture is done. Which of the following
pathogens is most likely?

Choices:
1. Gram-negative diplococci
2. Gram-positive diplococci
3. Mucicarmine positive encapsulated budding yeast cells
4. Strongly acid-fast tubercles
Answer: 3 - Mucicarmine positive encapsulated budding yeast cells
Explanations:
Cryptococcus is the most likely CNS fungal infection.
Patients who are HIV positive may have minimal or nonspecific symptoms at presentation.
The patient commonly presents with neurological symptoms such as a headache, altered
mental status, and other signs and symptoms include lethargy along with fever, stiff neck
(both associated with an aggressive inflammatory response), nausea, and vomiting.
CSF usually presents low glucose and high protein levels. White cell count can be normal or
higher than 20 microL and have a lymphocyte predominance. Nevertheless, CSF can be
normal and have positive results on India ink stain and antigen testing, especially in HIV-
positive patients who do not have an adequate inflammatory response.

Go to the next page if you knew the correct answer, or click the link image(s) below to further
research the concepts in this question (if desired).

Research Concepts:
Cryptococcal Meningitis

We update eBooks quarterly and Apps daily based on user feedback. Please tap flag to
report any questions that need improvement.
Question 745: A 24-year-old man with a history of active Crohn disease and
antiphospholipid syndrome undergoes a multi-visceral transplant, which is complicated by
bleeding and requiring intravenous prothrombotic therapies. Due to the recent procedure, the
transplant team felt it would be best to hold some of the patient’s home medicines. In the
immediate post-operative period, he does well and tube feeds are started. On postoperative day
two, the patient develops excruciating abdominal pain with associated nausea. Vital signs show
temperature 99.8 F, pulse rate 131/min, blood pressure 153/97 mmHg, and respiratory rate
22/min. His abdomen is soft without any guarding. Incisions appear unremarkable. Labs are
obtained, which show a worsening leukocytosis to 23,000/microL, lactic acid of 7.1 mmol/L, and
a creatinine level of 0.9 mg/dL. Which of the following is the next best step in the management
of this patient?

Choices:
1. Fluid bolus
2. CT mesenteric angiography
3. Mesenteric duplex
4. Hold tube feeds and re-evaluate patient in a few hours
Answer: 2 - CT mesenteric angiography
Explanations:
The patient is most likely suffering from post-operative mesenteric ischemia, secondary to
his hypercoagulable state, and unable to be safely placed on anticoagulation. His laboratory
values show an elevated WBC count and evidence of anaerobic metabolic stress.
Acute mesenteric ischemia in the setting of intestinal transplant is a surgical emergency and
requires immediate diagnosis and return to the operating room for management.
CT scan is the best imaging modality for mesenteric ischemia as it is readily available and a
rapid test to image the mesenteric vessels.
Mesenteric duplex is a modality that can diagnose mesenteric ischemia but is not as rapid
and is user dependant. It is highly specific but does have lower sensitivity than CT due to
overlying bowel gas disrupting the images.

Go to the next page if you knew the correct answer, or click the link image(s) below to further
research the concepts in this question (if desired).

Research Concepts:
Intestinal And Multivisceral Transplantation

We update eBooks quarterly and Apps daily based on user feedback. Please tap flag to
report any questions that need improvement.
Question 746: A 16-year-old male brought to the emergency department by law-
enforcement for “bizarre behavior.” The patient is extremely agitated and uncooperative
requiring physical restraint by four police officers. The patient has a heart rate of 135 bpm, blood
pressure 177/82 mm of Hg, respiratory rate 26 breaths/minute, oxygen saturation 92% on room
air. Fingerstick blood sugar 110 mg/dL. The patient is given 4 mg IM lorazepam for sedation.
The patient is warm to the touch. Eye exam shows miotic pupils and rotary nystagmus. Police
state that witnesses report that the patient was smoking “angel dust” before exhibiting his
“bizarre behavior.” Which of the following is false pertaining to the substance that the patient
has been using?

Choices:
1. Dopamine reuptake is blocked
2. Acetylcholine receptor stimulation is increased
3. Sigma receptor stimulation is decreased
4. Gama-aminobutyric acid (GABA) receptor stimulation is increased
Answer: 3 - Sigma receptor stimulation is decreased
Explanations:
Sigma receptor stimulation is increased by phencyclidine causing lethargy and coma.
Phencyclidine inhibits reuptake of dopamine and norepinephrine leading to
sympathomimetic effects such as hypertension, tachycardia, bronchodilation, and agitation.
Phencyclidine binds to gamma-aminobutyric acid (GABA) receptors causing sedation.
Phencyclidine binds to acetylcholine receptors to cause muscarinic and nicotinic effects.

Go to the next page if you knew the correct answer, or click the link image(s) below to further
research the concepts in this question (if desired).

Research Concepts:
Phencyclidine Toxicity

We update eBooks quarterly and Apps daily based on user feedback. Please tap flag to
report any questions that need improvement.
Question 747: A 32-year-old man is brought to the emergency department by paramedics
after being found unconscious. He was found with a needle in his arm in an alleyway. He was
breathing slowly, and he was unresponsive with pinpoint pupils. On the scene, his heart rate was
55 beats/minute, blood pressure 127/84 mmHg, a respiratory rate of about four breaths/minute,
and his temperature was 98.5 degrees F (36.9 C). Before administering naloxone, the paramedic
noted that with the patient's eyelids open, moving his head side to side caused his eyes to move
in the opposite direction, like a doll's eyes. This doll's eyes reflex involves several cranial nerves
and pathways within the brainstem. One well-known route within this reflex connects cranial
nerve III and VI. What disease is known to cause a lesion in this pathway?

Choices:
1. Huntington's chorea
2. Muscle eye brain disease
3. Claude syndrome
4. Multiple sclerosis
Answer: 4 - Multiple sclerosis
Explanations:
Multiple sclerosis is a demyelinating disorder that can cause pathology in the cranial nerves
to the eye.
Multiple sclerosis can cause internuclear ophthalmoplegia (INO) by causing demyelination
of the medial longitudinal fasciculus.
The medial longitudinal fasciculus among other things connects cranial nerve III and VI.
The medial longitudinal fasciculus is critical for the connection of cranial nerves in the
doll's eyes reflex (vestibulo-ocular reflex).

Go to the next page if you knew the correct answer, or click the link image(s) below to further
research the concepts in this question (if desired).

Research Concepts:
Doll's Eyes

We update eBooks quarterly and Apps daily based on user feedback. Please tap flag to
report any questions that need improvement.
Question 748: A 67-year-old man is being evaluated on postoperative day 1 following a
right hemicolectomy. The nurse reports the patient has had low-normal blood pressures with a
mean arterial pressure of 65-68 mmHg for approximately 1 hour. Emergent labs reveal no
leukocytosis, but his lactate level is 2.1 mmol/L, and his serum creatinine has increased from 0.8
mg/dL to 1.9 mg/dL. On examination, his mucous membranes are moist, and his pain is well-
controlled. He denies bowel movement or flatus as of yet but has no complaints otherwise. He
has been receiving 0.9% sodium chloride solution via IV at 125 mL/h. Which of the following is
the next best step in the management of this patient?

Choices:
1. 1 L bolus of crystalloid solution over 60 minutes
2. Initiation of pressor support
3. Initiation of hydroxyethyl starch-containing solution
4. Initiate fenoldopam
Answer: 1 - 1 L bolus of crystalloid solution over 60 minutes
Explanations:
Studies have shown that the development of lactic acidosis or hypotension intraoperatively
are late indicators of reduced renal perfusion.
As such, cardiac output and renal blood flow should be maintained with aggressive fluid
resuscitation, and when fluids alone are not sufficient, inotropes are necessary.
It is most appropriate to start with crystalloid solutions.
Pressor support is indicated if fluid resuscitation is ineffective, or if hemodynamic
instability develops. The use of hydroxyethyl starches, when compared to colloids, has been
linked to increased rates of AKI and increased need for RRT in some investigations, while it
has not been shown to increase AKI in other studies. Fenoldopam is a selective dopamine-1
agonist that has been shown to reduce the need for RRT. However, this has only been
consistently demonstrated in patients having undergone cardiac surgery, and its use is
limited by its propensity to cause systemic hypotension.

Go to the next page if you knew the correct answer, or click the link image(s) below to further
research the concepts in this question (if desired).

Research Concepts:
Perioperative Acute Kidney Injury

We update eBooks quarterly and Apps daily based on user feedback. Please tap flag to
report any questions that need improvement.
Question 749: A 58-year-old man is brought to the hospital after a witnessed unresponsive
spell while seated at a dinner theater. There was no prodrome of headache or chest pain.
Witnesses report that he slumped to the floor and had brief generalized motor movements lasting
perhaps ten seconds. He was alert and oriented a few seconds later. He has a history of
hypertension but no other significant medical history. He was incontinent during the event, but
there was no tongue-biting. He states he had some wine with dinner but was not clinically
intoxicated. Physical examination is unremarkable. Which of the following best identifies the
type of event?

Choices:
1. Provoked seizure
2. Unprovoked seizure
3. Convulsive concussion
4. Convulsive syncope
Answer: 4 - Convulsive syncope
Explanations:
The differential diagnosis of brief dissociative spells is complicated. The lack of any
postictal confusional state perhaps is the strongest factor weighing against a seizure and
suggesting syncope.
All historical factors should be considered in the evaluation. Brief symmetric motor
movements commonly occur with syncope and are given the term convulsive syncope.
Tongue-biting, when present tends, involves the anterior tongue with syncope and the
lateral tongue with seizures.
Convulsive syncope was studied in blood banks. Patients typically had a prodrome of
yawning, sweating, slight diaphoresis, and then a typical faint. Brief motor movements were
noted in many patients.

Go to the next page if you knew the correct answer, or click the link image(s) below to further
research the concepts in this question (if desired).

Research Concepts:
Seizure

We update eBooks quarterly and Apps daily based on user feedback. Please tap flag to
report any questions that need improvement.
Question 750: A 61-year-old patient with stage IIIB squamous cell carcinoma presents to
the emergency department with confusion and a disoriented state. Laboratory investigations
revealed that he has a calcium level of 18.5 mg/dL. In addition to that, his serum parathyroid
hormone levels are seven pg/mL (normal = 10-55 pg/mL ), but the parathyroid-like hormone is
positive. Treatment with large volumes of intravenous fluids and furosemide was done, but it
only lowered the calcium to 17.0mg/dL. Which of the following is the most appropriate
treatment for this patient?

Choices:
1. Continue fluids and furosemide
2. Start calcitonin
3. Start pamidronate
4. Start calcitonin and pamidronate
Answer: 4 - Start calcitonin and pamidronate
Explanations:
Paraneoplastic syndrome is one of several causes of hypercalcemia.
Squamous cell carcinoma of the lung is the most likely cause of parathyroid hormone-
related peptide.
Calcitonin can drop the calcium acutely but tachyphylaxis develops.
Intravenous pamidronate, a bisphosphonate, takes 1 to 2 days for effect; therefore, for acute
reduction, calcitonin is given along with pamidronate.

Go to the next page if you knew the correct answer, or click the link image(s) below to further
research the concepts in this question (if desired).

Research Concepts:
Paraneoplastic Syndromes

We update eBooks quarterly and Apps daily based on user feedback. Please tap flag to
report any questions that need improvement.
Question 751: A fourth-year medical student is about to start his sub-internship in the
general surgery service. He decides to get involved in the surgery boot camp that his medical
school offers. The first day they are taught how to perform central venous catheters to get
prepared for his critical care rotations as a resident. What would be the best method to teach this
student?

Choices:
1. Verbal repetition of the procedure following a checklist with one simulated case at the end of
the day
2. Repetition of the procedure in several different models with verbal feedback at the end of the
camp
3. Repetition of the procedure in one model several times with feedback at the end of the day
4. Repetition of the procedure with a partner with constant feedback of the technique as well as
feedback at the end of the day by the trainer
Answer: 4 - Repetition of the procedure with a partner with constant feedback of the
technique as well as feedback at the end of the day by the trainer

Explanations:
In the skills assessment and learning, the more feedback a trainee gets, the better for him.
The more repetitions a trainee has, the more self-consciousness is developed.
When learning new skills, the best way to improve and creating self-consciousness of
details is by comparing yourself to another trainee. It helps develop skills faster than doing a
self-evaluation.
Reinforcement of the same technique under the same conditions and using a checklist to
establish a pattern to follow, this will make every repetition similar to the previous one,
improving the technique.
Verbal repetition or written tests have been less effective in teaching any skill, especially in
the critical care setting.

Go to the next page if you knew the correct answer, or click the link image(s) below to further
research the concepts in this question (if desired).

Research Concepts:
Simulation Training and Skill Assessment in Critical Care

We update eBooks quarterly and Apps daily based on user feedback. Please tap flag to
report any questions that need improvement.
Question 752: A 65-year-old female immigrant from Pakistan presents with the chief
complaints of severe shortness of breath, productive cough, and inability to carry out activities of
daily living. She has a history of biomass fuel exposure for twenty years in her hometown. On
examination, a restless female with severe respiratory distress, blood pressure 110/70 mmHg,
pulse 112/min regular, respiratory rate 26/min, and oxygen saturation of 88% on room air. On
auscultation, her chest is largely silent with decreased air entry bilaterally. Her arterial blood
gases suggest a type II respiratory failure. Which of the following should be considered besides
supportive care?

Choices:
1. Elective endotracheal intubation
2. High flow oxygen therapy
3. Theophylline
4. Bilevel positive airway pressure
Answer: 4 - Bilevel positive airway pressure
Explanations:
In the case of chronic obstructive pulmonary disease (COPD) with type II respiratory
failure, it is recommended to put the patient on bilevel positive airway pressure. The non-
pharmacological approach includes vaccination (influenza and pneumococcal), smoking
cessation, and pulmonary rehabilitation. Pulmonary rehabilitation is indicated in all stages
of COPD. Bronchodilators are the primary pharmacologic treatment used. Short-acting
bronchodilators provide immediate relief, and long-acting bronchodilators are used for
maintenance in patients with advanced disease. A long-acting beta2 agonist alone or in
combination with an inhaled corticosteroid has shown to reduce exacerbation. A long-acting
antimuscarinic agent has also been shown to improve dyspnea and reduce exacerbations. A
combination of two different classes of long-acting bronchodilators can be used in patients
with moderate-to-severe COPD when monotherapy is insufficient. Continuous oxygen
therapy is indicated for hypoxemic patients with COPD when arterial oxygen saturation is
equal to, or less than 88% or partial pressure of oxygen is equal to or less than 55 mmHg.
Based on current estimates, COPD will be the third leading cause of death worldwide by
2030. In the United States, it is estimated that more than 12 million people are affected by
COPD and is more prevalent in women than men. The death rate for COPD has declined in
men from 1999 to 2010, but it is steady among women.
Typical symptoms of COPD include a chronic, productive cough, shortness of breath, and
wheezing. Acute exacerbation of COPD is usually defined as at least two of the following
three conditions: worsening of shortness of breath, change in purulence, or quantity of
sputum. Physical findings include barrel-shaped chest, diminished breath sounds, and
hyperresonance on percussion. In advanced disease with cor pulmonale, jugular venous
distention, accentuated second heart sound, hepatomegaly, and pedal edema can be seen.
Lung auscultation may show a prolonged expiratory phase with or without wheezing.
Respiratory symptoms are often relieved with pursed-lip breathing and tripod position.
Pulmonary function test (PFT) is recommended to diagnose COPD. Airflow limitation is
diagnosed when forced expiratory volume in one second (FEV1) to forced vital capacity
ratio is less than 70%. According to Global Initiative for Chronic Obstructive Lung Disease
classification system, the severity of obstructive defect is mild, moderate, severe, or very
severe when FEV 1 percent predicted is greater than 80%, 50% to 79%, 30% to 49%, and
less than 30% respectively. Diffusion capacity for carbon monoxide is reduced in COPD. A
chest radiograph often shows hyperinflation and flattening of the diaphragms, especially in
advanced disease. High-resolution CT of the chest may demonstrate emphysema. An
echocardiogram is used to assess secondary pulmonary hypertension which is due to
hypoxemic vasoconstriction of the pulmonary blood vessels. A walking or nocturnal
desaturation study identifies patients who would benefit from oxygen therapy with activities
or during sleep. Testing for alpha-1 antitrypsin level should be considered in age younger
than 40 years and non-smokers if a significant obstructive defect is found on PFTs.

Go to the next page if you knew the correct answer, or click the link image(s) below to further
research the concepts in this question (if desired).
Research Concepts:
Chronic Obstructive Pulmonary Disease

We update eBooks quarterly and Apps daily based on user feedback. Please tap flag to
report any questions that need improvement.
Question 753: A 23-year-old female who is 33 weeks pregnant presents to the emergency
department after a motor vehicle collision. The patient is supine on a backboard with a cervical
collar in place. Given the patient's mechanism of injury, there is concern for a possible cervical
spine injury. On arrival, the patient is hypotensive with a blood pressure of 80/60 mmHg and a
heart rate of 95 beats per minute. The evaluating clinician is concerned about the aortocaval
compression syndrome and knows the patient needs to be re-positioned immediately to help treat
the hypotension. What is the most appropriate next step of management?

Choices:
1. Isolated elevation of the left hip
2. Manual displacement of the uterus to the left of the midline
3. Tilt the patient's bed into a reverse Trendelenburg position
4. Tilt the backboard 45 degrees to the right
Answer: 2 - Manual displacement of the uterus to the left of the midline
Explanations:
Since there is a concern for a cervical spine injury, manual displacement of the uterus to the
left side of the patient's abdomen. This will decrease potential compression of the inferior
vena cava by the uterus and is one of the accepted initial treatments when aortocaval
compression is suspected. This is done by placing the provider's hand on the right side of
the abdomen, lateral to the gravid uterus, and shifting the uterus to the left, and thus off of
the vena cava. This maneuver is simple, requires no expertise, and is not harmful to the
uterus or fetus.
Once other causes of hypotension are treated and/or ruled out, then the physical movement
of the uterus off of the spine (and inferior vena cava) is necessary.
When trauma is not involved, simply placing the patient in the left lateral position may be
done. However, when trauma is present, immobilization of the spine must simultaneously
occur, and different maneuvers are indicated.
Isolated elevation of the right hip alone may alleviate the compression. According to the
advanced trauma life support guidelines, tilting the backboard 15 to 30 degrees to the left
(right side upwards) is an additional option. This may be done either manually or with
elevators, including premade elevators or towel rolls placed under the board. This option is
sometimes difficult as the weight of the gravid abdomen gravitationally pulls the patient to
the left side, potentially compromising spinal immobilization. Though difficult, in-line
immobilization is a priority.

Go to the next page if you knew the correct answer, or click the link image(s) below to further
research the concepts in this question (if desired).

Research Concepts:
Aortocaval Compression Syndrome

We update eBooks quarterly and Apps daily based on user feedback. Please tap flag to
report any questions that need improvement.
Question 754: A 16-year old is seen in the hematology clinic with complaints of fatigue.
Examination reveals mild jaundice and splenomegaly. Blood work reveals a white blood cell
(WBC) count of 2.9, hematocrit of 22, and hemoglobin of 8. The mean corpuscular hemoglobin
concentration (MCHC) is markedly increased. Which of the following is the most threatening
long-term complication for a patient treated for this condition?

Choices:
1. Overwhelming post-splenectomy sepsis (OPSS)
2. Infection of the Incision site
3. Vulnerability to infection
4. Reaction to medications
Answer: 1 - Overwhelming post-splenectomy sepsis (OPSS)
Explanations:
Overwhelming post-splenectomy sepsis (OPSS) is a life-threatening complication for those
undergoing splenectomy.
Infection of the incision site is always a concern for those undergoing any surgery.
Antibiotic therapy until the age of sixteen (possibly longer) Is a standard treatment because
of the increased vulnerability to infection following splenectomy. A two-year course of
penicillin or long-term treatment with ampicillin is often recommended by physicians for
patients undergoing a splenectomy.
Vaccination therapy is the standard of care prior to surgery, including a booster dose of
pneumococcal vaccine five to 10 years after undergoing a splenectomy.

Go to the next page if you knew the correct answer, or click the link image(s) below to further
research the concepts in this question (if desired).

Research Concepts:
Hereditary Spherocytosis

We update eBooks quarterly and Apps daily based on user feedback. Please tap flag to
report any questions that need improvement.
Question 755: A 76-year-old woman with a history of hypertension, hyperlipidemia, and a
40 pack-year smoking history presents to the hospital with fluctuating left-sided weakness and
dysarthria. On examination, the patient was noted to have a horizontal gaze palsy but a preserved
vertical gaze. Her National Institute of Health Stroke Scale fluctuated between 4 and 7. CT brain
imaging did not show any acute pathology, and vessel imaging with a CT head and neck
angiography was negative for a large vessel occlusion and a patient basilar artery, but with
extensive atherosclerotic disease in multiple vascular territories. She was normal 4 hours before
the presentation. Blood pressure ranges were between systolic of 140-160 mmHg and diastolic of
90-100 mmHg. Other vital signs are within normal limits. Which of the following is the next best
step in the management of this patient?

Choices:
1. Intravenous tissue plasminogen activator, if no contraindications
2. Aspirin
3. Diagnostic angiography to evaluate the atherosclerotic disease with an intention to treat the
intracranial stenosis with angioplasty +/- stenting
4. Heparin infusion and warfarin
Answer: 1 - Intravenous tissue plasminogen activator, if no contraindications
Explanations:
The patients presented with a pontine warning syndrome with fluctuating clinical features
and is within the therapeutic window for the use of a thrombolytic agent.
Thrombolysis is indicated in patients presenting within a 4.5-hour time frame with an
ischemic stroke presentation with no clear contraindications.
This stroke presentation is likely due to atherosclerotic disease likely large artery
atherosclerotic disease or small vessel disease based on the extensive vascular risk factors
and the CT angiography showing extensive atherosclerotic disease. Irrespective of the
etiology of the stroke, thrombolysis has been shown to be beneficial if used within a
reasonable time frame.
If the patient presented outside of the therapeutic window of 4.5 hours from the clinical
presentation, aspirin would have been a reasonable choice.

Go to the next page if you knew the correct answer, or click the link image(s) below to further
research the concepts in this question (if desired).

Research Concepts:
Pontine Infarction

We update eBooks quarterly and Apps daily based on user feedback. Please tap flag to
report any questions that need improvement.
Question 756: A 61-year-old woman is placed on veno-arterial (VA) ECMO due to low
ejection fraction after a long clamp time on her on-pump CABG. The patient is taken to the
intensive care unit after placement of the VA ECMO catheters. Over the course of her ICU stay,
her EF shows signs of recovery. When attempting to wean the VA ECMO circuit, which of the
following best identifies the minimum ejection fraction and flow rate that would allow weaning
the patient from VA ECMO?

Choices:
1. Ejection fraction 10%; flow rate 3 L/min
2. Ejection fraction 15%, flow rate 4 L/min
3. Ejection fraction 30%, flow rate 4 L/min
4. Ejection fraction 25%, flow rate 1.5 L/min
Answer: 4 - Ejection fraction 25%, flow rate 1.5 L/min
Explanations:
When weaning a patient from VA ECMO, the cause of the cardiogenic shock must be
resolved prior to weaning. Attempts at weaning before restoring the ejection fraction should
not be attempted until the heart can function without the aid of maximum pressor support.
When weaning from VA ECMO, transesophageal echocardiography is essential in looking
at the function of the heart. Echo is often performed on patients to look for progress for
resolution of the cardiogenic shock. VA ECMO should not be weaned until the ejection
fraction has recovered to a minimum of 25%. This number has shown that the heart is
strong enough to function for weaning trials.
When decreasing the flow rates on the VA ECMO circuit, a more accurate echo of the
heart's ejection fraction can be obtained. The flow rate should be reduced as low as possible,
with the minimum flow rate being 1.5 L/min as this allows to see the heart working with
little support. It is recommended not to go with lower flow rates as this can cause
thrombosis to the ECMO circuit.
Weaning from VA ECMO should not be attempted until resolution of the cardiogenic
shock, ejection fractions less than 25% are not recommended for weaning until the heart is
stronger. Flow rates greater than 1.5 L/min give the heart more support and can falsely alert
the physician to know the true function of the heart.

Go to the next page if you knew the correct answer, or click the link image(s) below to further
research the concepts in this question (if desired).

Research Concepts:
Extracorporeal Membrane Oxygenation Weaning

We update eBooks quarterly and Apps daily based on user feedback. Please tap flag to
report any questions that need improvement.
Question 757: A 45-year-old male presents with complaints of progressive shortness of
breath. He has had the symptoms for the past year. His shortness of breath was gradual in onset
but is progressive. The patient now feels breathless even on mild exertion. He has been finding it
difficult to perform his daily activities. He also reports occasional episodes of dizziness and loss
of consciousness. There is no history of cough or fever. His examination reveals a blood pressure
of 130/50 mmHg, a pulse of 100 beats per minute, a respiratory rate of 27 per minute, and a
temperature of 98 F. His cardiac auscultation reveals the presence of an early diastolic grade 4/6
murmur best heard over the left 3rd intercostal space. The murmur increases on expiration. His
respiratory, gastrointestinal, and neurological examinations are unremarkable. His investigations
revealed a WBC count of 5,000 per microL, hemoglobin of 12 gm/dl, platelet count of 200,000
per microL, serum ALT 35 IU/l, serum AST 35 IU/l, and serum creatinine of 1.1 mg/l.
Serologies are performed, which are anti- HCV negative, anti-Hbs positive, HbsAg negative.
Anti-Hbc IgM negative, RPR negative, TPHA positive, ANA negative, and RA factor negative.
An X-ray reveals the presence of mediastinal widening and linear calcifications of the ascending
aorta. What is the most likely etiology for this patient’s shortness of breath?

Choices:
1. Mitral valve stenosis
2. Pericardial effusion
3. Aortic inflammation, regurgitation, and insufficiency
4. Valvular calcification
Answer: 3 - Aortic inflammation, regurgitation, and insufficiency
Explanations:
This patient presents with symptoms of progressive dyspnea. His history of dizziness and
loss of consciousness suggests a cardiovascular cause. The presence of an early diastolic
murmur suggests the presence of aortic regurgitation. The serology investigation reveals
positivity for treponema pallidum (TPHA) and his x-ray suggests aortic dilation. Linear
calcifications of the ascending aorta is closely associated with the most likely diagnosis
which is cardiovascular syphilis.
Cardiovascular syphilis is a form of tertiary syphilis characterized by aortitis, aortic valve
regurgitation, and aortic dilatation eventually resulting in heart failure. The usual
presentation is 15-30 years after the primary infection. Morphologically, the aorta appears
dilated with patchy areas of fibrosis and necrosis referred to as "tree bark aorta".
Histologically, it is characterized by a chronic perivascular inflammatory,
lymphoplasmacytic infiltrate. There is obliterative endarteritis of the vasa vasorum and
subsequent necrosis.
There is co-existent aortic insufficiency, which may lead to left ventricular failure.
Cardiovascular syphilis may cause coronary ostial lesions but does not typically cause
atherosclerosis. Valvular calcification and stenosis may cause shortness of breath, such as in
aortic and mitral stenosis.

Go to the next page if you knew the correct answer, or click the link image(s) below to further
research the concepts in this question (if desired).

Research Concepts:
Syphilis

We update eBooks quarterly and Apps daily based on user feedback. Please tap flag to
report any questions that need improvement.
Question 758: A 50-year-old woman with a past medical history of diabetes presents to the
office with left lower extremity cellulitis. She is prescribed sulfamethoxazole-trimethoprim and
follows up 48 hours later, reporting new-onset fever and persistent erythema. She is admitted to
the hospital and started on vancomycin and piperacillin/tazobactam. The next morning, her vital
signs are stable, but there are bullae on her leg with no regression of erythema. What is the next
best step in the management of this patient?

Choices:
1. Change vancomycin to linezolid
2. Add levofloxacin for improved pseudomonal coverage
3. Consult infectious disease
4. Obtain a stat CT scan of the leg
Answer: 4 - Obtain a stat CT scan of the leg
Explanations:
Necrotizing fasciitis is a rare infection of the fascia that leads to necrosis of the
subcutaneous tissue. It is characterized by fevers, erythema, edema, pain out of proportion
to the exam, and crepitus.
If necrotizing fasciitis is suspected and the patient is unstable, crepitus is noted, and/or
symptoms have worsened, surgery should be called immediately.
In a stable patient without crepitus or rapid progression of erythema, edema, etc. a CT scan
is often the imaging modality used to assess for gas.
If you're suspicious of necrotizing fascitis, surgical consultation is recommended over an
infectious disease consultation. Changing from vancomycin to linezolid is not necessary.
Double coverage for pseudomonas is not typically recommended.

Go to the next page if you knew the correct answer, or click the link image(s) below to further
research the concepts in this question (if desired).

Research Concepts:
Cellulitis

We update eBooks quarterly and Apps daily based on user feedback. Please tap flag to
report any questions that need improvement.
Question 759: A 34 years old patient presented at the ED for weakness. On examination,
pupils were isochoric and equally reactive to light, there was impaired lateral and medial gaze,
the was able to close his eyes tightly, there was bilateral facial droop, the patient was unable to
stick out his tongue, and the speech was severely dysarthric. The manual muscle testing shows
3/5 on the right upper and lower extremity, 4/5 on the left upper and lower extremity, sensory
exam shows deficit more on the left than the right. Babinski was positive bilaterally. Where is
this patient's lesion?

Choices:
1. Cerebellar vermis
2. Right parietotemporal region
3. Bilateral occipital lobe
4. Anterior pontine region
Answer: 4 - Anterior pontine region
Explanations:
Occlusion in the vertebrobasilar system can result in locked-in syndrome. As the basilar
artery gets occluded, it will affect cranial nerve nuclei and tracts in the pons, causing
bilateral facial palsy, as well as problems with eye movement due to the involvement of the
medial longitudinal fasciculus and paramedian pontine reticular formation.
The involvement of the corticospinal tracts in this level will cause weakness of both sides of
the body to varying degrees, and quadriplegia as an endpoint.
The involvement of the spinothalamic tracts causes sensory impairment on both sides.
Lock-in syndrome is the full syndrome in basilar artery occlusion, causing quadriplegic and
cranial nerve deficits. The sparing of the ascending reticular formation in the pons causes
the patient to be awake.

Go to the next page if you knew the correct answer, or click the link image(s) below to further
research the concepts in this question (if desired).

Research Concepts:
Locked-in Syndrome

We update eBooks quarterly and Apps daily based on user feedback. Please tap flag to
report any questions that need improvement.
Question 760: A 3-year old girl is admitted to the recovery after undergoing an open
reduction and fixation of her midshaft radius and ulna fractures. On exam, she is responding to
voice, her blood pressure 140/85 mmHg, heart rate 98 bpm, respiratory rate 12 cycles per min,
and saturation 92% on 5L of O2. Her past medical exam is significant for tonsillar hypertrophy.
She is put on ventilation support. Which of the following is an absolute contraindication for her
ventilation support?

Choices:
1. Epistaxis
2. Pneumothorax
3. Bullous lung disease
4. Inability to protect their airway
Answer: 4 - Inability to protect their airway
Explanations:
Continuous positive airway pressure therapy (CPAP) is a form of noninvasive positive-
pressure ventilation.
It is routinely used in individuals with obstructive sleep apnea and chronic obstructive
pulmonary disease.
Epistaxis, pneumothorax, bullous lung disease, and apnea are contraindications to CPAP.
This patient has obstructive sleep apnea from tonsillar hypertrophy. She will need CPAP in
the recovery room as she is used to chronic hypercapnia and hypoxia. The inability to
protect the airway is an absolute contraindication for CPAP. It may lead to aspiration of
gastric contents.

Go to the next page if you knew the correct answer, or click the link image(s) below to further
research the concepts in this question (if desired).

Research Concepts:
Continuous Positive Airway Pressure

We update eBooks quarterly and Apps daily based on user feedback. Please tap flag to
report any questions that need improvement.
Question 761: A 16-year-old boy who was recently diagnosed as HIV-positive presents
with a 6-hour history of fever, headache, and blurred vision. A head CT scan shows no
intracranial abnormality. A lumbar puncture is performed, and CSF testing reveals encapsulated
organisms. Besides amphotericin, which drug should be given to this patient?

Choices:
1. Trimethoprim
2. Ketoconazole
3. Flucytosine
4. Nystatin
Answer: 3 - Flucytosine
Explanations:
Amphotericin B is an essential part of induction treatment for cryptococcal meningitis.
The most effective combination for the induction therapy is flucytosine and amphotericin B.
Unfortunately, flucytosine is frequently unavailable because of its cost.
The infectious diseases society of America (IDSA) guidelines do not include the use of
ketoconazole, nystatin, or trimethoprim for the treatment of cryptococcal meningitis.

Go to the next page if you knew the correct answer, or click the link image(s) below to further
research the concepts in this question (if desired).

Research Concepts:
Cryptococcal Meningitis

We update eBooks quarterly and Apps daily based on user feedback. Please tap flag to
report any questions that need improvement.
Question 762: A 75-year-old Caucasian man presents to the emergency department with
one episode of a small amount of hemoptysis. He has also had a dry cough and decreased urine
output for the last 2 days. He denies fever, chills, or dysuria. He denies frank hematuria. On
examination, his respiratory rate is 24/min, heart rate 88/min, and blood pressure 130/78 mmHg.
Labs reveal serum creatinine 2.4 mg/dL and BUN of 26 mg/dL. His urinalysis is positive, with
1+ protein and moderate blood. His chest x-ray is suggestive of bilateral fluffy infiltrates. He
never had kidney disease in the past, and his most recent serum creatinine during his visit was 1
mg/dL 3 months back. He denies the recent use of NSAIDs other medications. A renal biopsy is
planned. Soon after admission, he develops progressive respiratory discomfort, requiring
mechanical ventilatory support. What is the best initial therapy for this patient?

Choices:
1. IV methylprednisone
2. Oral prednisone
3. Plasmapheresis
4. Oral azathioprine
Answer: 1 - IV methylprednisone
Explanations:
The clinical vignette is suggestive of anti-GBM disease. Timely treatment is the key to
preserving renal function. The recommended guideline is to start pulse dose steroids as soon
as anti-GBM disease is suspected.
It is not advisable to wait for the biopsy result to confirm anti-GBM disease.
Pulse dose methylprednisone 500- 1000 mg IV for 3-5 days is recommended.
Once the diagnosis is confirmed plasmapheresis can be initiated.

Go to the next page if you knew the correct answer, or click the link image(s) below to further
research the concepts in this question (if desired).

Research Concepts:
Rapidly Progressive Glomerulonephritis

We update eBooks quarterly and Apps daily based on user feedback. Please tap flag to
report any questions that need improvement.
Question 763: What is the pathology behind cardiac tamponade?
Choices:
1. Accumulation of 250 ml of fluid
2. Rate of fluid accumulation
3. Afterload
4. Increased venous return
Answer: 2 - Rate of fluid accumulation
Explanations:
The rate of accumulation of fluid is more important than the volume in the pathogenesis of
cardiac tamponade.
As little as 150 ml of fluid can cause clinically significant symptoms while as much as a
liter can slowly accumulate if the pericardium is distensible.
In acute cases, even 100 ml of fluid accumulation can cause symptoms but in chronic cases,
the pericardial sac can tolerate as much as 1,000 ml.
The other factor that also affects development of tamponade is the compliance of the
pericardium. If the pericardium is pliable, it may initially stretch and prevent symptoms.

Go to the next page if you knew the correct answer, or click the link image(s) below to further
research the concepts in this question (if desired).

Research Concepts:
Cardiac Tamponade

We update eBooks quarterly and Apps daily based on user feedback. Please tap flag to
report any questions that need improvement.
Question 764: A 45-year-old man presents to the emergency department with a complaint
of breathlessness and dry cough. This is his third admission this year with similar complaints. On
arrival, his vitals are blood pressure 100/70 mmHg, pulse 90/min, respiratory rate 18/min and a
temperature of 99 F. His spO2 is 86%. Laboratory evaluation reveals an LDH level of 750 IU/L,
CD4 count 140/microL and the chest x-ray shows bilateral interstitial infiltrates. What is the best
initial therapy for this patient?

Choices:
1. Trimethoprim/ sulfamethoxazole
2. Zidovudine (AZT)
3. Pentamidine isethionate
4. Ceftriaxone/azithromycin
Answer: 1 - Trimethoprim/ sulfamethoxazole
Explanations:
Trimethoprim/sulfamethoxazole is the first-line therapy for Pneumocystis jiroveci
pneumonia (PCP). This drug is an anti-infective/amebicide/antiprotozoal. The organism
responsible for PCP is the P jiroveci protozoon.
PCP is normally present in the lungs of humans and various animals. However, it becomes
an aggressive pathogen in the immunocompromised patient.
Pentamidine is among the drugs of choice for treating pneumocystis carinii and used for
patients with sulfa allergies.
PCP does not respond to antifungal treatment. Second-line drugs include dapsone,
pentamidine, or atovaquone. In HIV patients, the treatment duration is for 21 days.

Go to the next page if you knew the correct answer, or click the link image(s) below to further
research the concepts in this question (if desired).

Research Concepts:
Pneumocystis Jirovecii Pneumonia

We update eBooks quarterly and Apps daily based on user feedback. Please tap flag to
report any questions that need improvement.
Question 765: A 65-year-old man with a history of hypertension, coronary artery disease,
ischemic cardiomyopathy, and pacemaker implantation secondary to complete heart block
presents to the emergency department with worsening shortness of breath. He also reports
orthopnea and lower extremity edema but no chest pain. His home medications include aspirin,
atorvastatin, metoprolol, and lisinopril. He reports compliance with medications and a low salt
diet. Vital signs show blood pressure 125/80 mmHg, heart rate 110/min, and SpO2 97% on 2
L/min oxygen. Examination shows diffuse crackles bilaterally, and +2 pitting edema but is
otherwise unremarkable. EKG shows an atrial sensed ventricular paced rhythm at a rate of
110/min. A chest X-ray shows pulmonary edema. Labs are remarkable for normal CBC, normal
kidney function, negative troponin, and BNP of 2200 pg/mL. Which of the following is the most
appropriate next step in evaluating the cause of his acute decompensated heart failure?

Choices:
1. Coronary angiogram
2. Right heart catheterization
3. Transesophageal echocardiogram
4. Pacemaker interrogation
Answer: 4 - Pacemaker interrogation
Explanations:
Patients with chronic heart failure may have acute decompensation due to a variety of
reasons, including non-compliance with diet or medications, an ischemic event, arrhythmia,
or any other factor that increases cardiac demand (anemia, thyrotoxicosis, etc.). In this case,
the patient has a tachycardic rhythm, most likely pacemaker-mediated tachycardia, which
resulted in the worsening of his heart failure symptoms.
The reason why tachycardia can result in acute decompensation of heart failure symptoms is
two-fold. First, tachycardia increases the oxygen demand of cardiac muscle, which may
cause a supply-demand mismatch. Second, tachycardia causes a decrease in diastolic filling
time, which can cause decreased preload, which lowers the cardiac output.
Patients who develop pacemaker-mediated tachycardia and stay in that rhythm are prone to
develop a variety of symptoms. Classically, they present with palpitations or dizziness.
However, in cases where the tachycardia lasts for a long time, in a patient with underlying
heart failure, this may cause acute decompensation of the heart failure symptoms.
In cases of acute decompensated heart failure in a patient with a permanent pacemaker,
careful examination of the EKG, and subsequently, device interrogation should be one of
the first steps in the evaluation.

Go to the next page if you knew the correct answer, or click the link image(s) below to further
research the concepts in this question (if desired).

Research Concepts:
Pacemaker Mediated Tachycardia

We update eBooks quarterly and Apps daily based on user feedback. Please tap flag to
report any questions that need improvement.
Question 766: A 17-year-old male was brought to an emergency department after he was
struck by a car. On arrival, his blood pressure was 112/74 mmHg, heart rate is 114/min,
respiratory rate is 14 breaths/min, and oxygen saturation is 90% at room air. On physical
examination, the patient had bruises and abrasion all over the body with a laceration of the
abdomen. His Glasgow coma scale was 15/15. Examination of the left ear revealed purplish
colored tympanic membrane. X-rays of upper and lower limbs demonstrated fractures of the left
radius and the left femur. CT scan of the abdomen showed irregular linear areas of
hypoattenuation. The patient was immediately shifted to the operating room. He had several
surgeries and was improving but still required morphine for pain control. One week after the
surgery, he develops a fever and altered mental status. His Glasgow coma scale becomes 8/15.
What is the next best step in the management of this patient?

Choices:
1. CT of head and lumbar puncture to rule out meningitis
2. Order a CT scan to rule out a pulmonary embolism
3. Check blood cultures and start antibiotics for sepsis
4. Chest x-ray to evaluate for atelectasis secondary to hypoventilation
Answer: 1 - CT of head and lumbar puncture to rule out meningitis
Explanations:
A basilar skull fracture allows communication between the nasopharynx or middle ear and
the meningeal space. Fractures that involve the petrous ridge of the temporal bone will
cause blood to pool behind the tympanic membrane causing it to appear purple. This usually
appears within hours of injury and maybe the earliest clinical finding.
Altered mental status and fever with known basilar skull fracture require evaluation of the
CSF. CSF leak is not easy to diagnose and the fluid should be sent for analysis of beta
transferrin.
The majority of CSF leaks resolve spontaneously within 5-10 days but some can persist for
months. Meningitis may occur in less than 5% of patients but the risk increases with the
duration of the CSF leak.
Pulmonary emboli are rare in children but a fat embolus could occur. Hypoventilation and
atelectasis would have more respiratory symptoms.

Go to the next page if you knew the correct answer, or click the link image(s) below to further
research the concepts in this question (if desired).

Research Concepts:
Basilar Skull Fractures

We update eBooks quarterly and Apps daily based on user feedback. Please tap flag to
report any questions that need improvement.
Question 767: A 65-year-old male patient presents to the emergency department due to
lower gastrointestinal bleeding after attempting suicide by ingesting an unknown amount of his
prescribed medications 6 hours ago. He takes metoprolol and rivaroxaban for atrial fibrillation
and thyroxine for hyperthyroidism. On examination, the patient looks drowsy, and his pulse is
98/min, respiratory rate is 16 breaths/min, and blood pressure is 100/70 mm hg. His hemoglobin
level is 9.0 g/l; platelet count is 135,000/mcL, PT is 10 seconds, aPTT 38 seconds, and INR 3.3.
His capillary blood glucose level is 80 mg/dL. After hospital admission, he is managed with IV
fluids and oxygen, but he still complains of shortness of breath and lightheadedness and has
ongoing lower gastrointestinal bleeding. Which of the following blood components is the most
appropriate choice to control the bleeding in this patient?

Choices:
1. Platelet concentrate
2. Packed red blood cells
3. 4-factor prothrombin complex concentrate
4. Idarucizumab
Answer: 3 - 4-factor prothrombin complex concentrate
Explanations:
Patients with life-threatening/major bleeding should be managed in an intensive care setting
with appropriate hemodynamic support, an antifibrinolytic agent like tranexamic acid, oral
activated charcoal when appropriate, and drug discontinuation. Reversal of anticoagulation
is generally thought to be desirable in a patient with serious or life-threatening bleeding who
remains actively anticoagulated.
Patients with refractory bleeding even after massive transfusion or with history of taking
direct factor Xa inhibitor like rivaroxaban need an inactivated 4-factor prothrombin
complex concentrate comprising of factor II, VII, IV, X or the newer FDA approved
reversal agents such as Andexanet Alfa which is a recombinant coagulation factor Xa.
There is insufficient evidence to strongly favor either 4-factor PCC or andexanet alfa over
the other. If both are unavailable, then a 3-factor PCC can be used in supplementation with
FFP to supply factor VII.
Platelet concentrate transfusion is not used to reverse the anticoagulant effect of DOACs in
a patient with a normal platelet count like this patient. But, thrombocytopenic patients with
bleeding should be treated for the underlying cause of thrombocytopenia and receive
platelet transfusions if it is severe and bleeding is major or life-threatening. Packed red
blood cell is not required in this patient, but RBC transfusion may be required, in case of
severe anemia depending on the hemoglobin level, rate of bleeding, and amount of blood
loss. Idarucizumab is a humanized monoclonal antibody used in patients treated with
dabigatran when a reversal is needed for emergency surgery or procedures, or in the event
of life-threatening uncontrolled bleeding.

Go to the next page if you knew the correct answer, or click the link image(s) below to further
research the concepts in this question (if desired).

Research Concepts:
Rivaroxaban

We update eBooks quarterly and Apps daily based on user feedback. Please tap flag to
report any questions that need improvement.
Question 768: A 70-year-old, 85 kg man with a history of diabetes and hypertension is
admitted to the emergency department with a history of high-grade fever, burning micturition,
and altered sensorium. His blood pressure is 70/40 mmHg, pulse 102/min, respiratory rate
24/min. Despite receiving 1.5 L of 0.9% saline over the last hour, his blood pressure has only
increased minimally from 70/40 mmHg to 80/50 mmHg, and he has passed 10 mL of urine. He
has received appropriate antibiotics. Which of the following is the next best step in the
management of this patient?

Choices:
1. A further 500 mL of crystalloid
2. 250 mL boluses of 6% hydroxyethyl starch
3. Intravenous hydrocortisone 100 mg
4. Noradrenaline infusion
Answer: 1 - A further 500 mL of crystalloid
Explanations:
The latest Surviving Sepsis Guidelines recommends the use of crystalloids, such as 0.9%
saline, as the initial fluid for resuscitation for people with severe sepsis. They suggest that
patients with sepsis-induced hypoperfusion with suspicion of hypovolaemia can be
challenged with as much as 30 mL/kg of crystalloid greater amounts may be needed in
some patients.
Fluid administration should be continued as long as there is a hemodynamic improvement
based on either dynamic (change in pulse pressure or stroke volume variation) or static
(heart rate, arterial pressure) variables.
Evidence is growing against the use of starches, with recent studies showing an association
with renal impairment and the need for renal replacement therapy.
The guidelines also discourage intravenous corticosteroid therapy in patients where fluid
resuscitation and vasopressors can restore adequate blood pressure. Noradrenaline is the
first choice for vasopressor therapy for septic shock refractory to fluid administration, and
adrenaline may be added to maintain adequate blood pressure.

Go to the next page if you knew the correct answer, or click the link image(s) below to further
research the concepts in this question (if desired).

Research Concepts:
Urosepsis

We update eBooks quarterly and Apps daily based on user feedback. Please tap flag to
report any questions that need improvement.
Question 769: A 65-year-old male was foraging for mushrooms in November in the forest.
He had no training in mushroom identification. He thought he picked a white to beige
mushroom, with a cap, gills, and annulus that he had eaten previously. It tasted normal in a salad
uncooked. Eight hours after eating the mushroom, he developed vomiting, and 12 hours after
eating, he had crampy abdominal pain and profuse watery diarrhea. In the emergency
department, his initial vital signs include a heart rate of 135 bpm, blood pressure 95/65 mmHg,
respiratory rate 20 breaths/min, temperature 98.6 F at 24 hours post-ingestion. On the abdominal
exam, the bowel sounds are normal, and there are no peritoneal signs. An intravenous line is
started, and a fluid bolus of ringer lactate is given. Labs show he had elevated blood urea
nitrogen (35 mg/dl) and creatinine ( 1.4 mg/dl) consistent with dehydration but normal liver
transaminases. The next day despite IV fluids, he had rising transaminases (AST and ALT >
2000 IU). After two days of supportive care, he was placed on the liver transplant list for an
urgent transplant, and on day four, he had a liver transplant. What is the histopathologic
manifestation of liver injury that would be found on his native liver on pathology?

Choices:
1. Centrilobular necrosis
2. Periportal necrosis
3. Veno-occlusive disease
4. Hepatic steatosis
Answer: 1 - Centrilobular necrosis
Explanations:
The amatoxin of Amanita phalloides is implicated in this hepatic injury. It takes about 24
hours before any signs or laboratory indicators of liver injury begin to appear. The first
stage does not begin until six to 12 hours after ingestion. There are no signs of a problem
for at least 6 hours. After this silent phase, it is followed by the onset of nausea, abdominal
cramps, profuse watery diarrhea, and signs of dehydration. A physical exam may reveal dry
mucosal membranes and tachycardia, and given sufficient dehydration, hypotension.
After the GI phase, the second stage appears where the patient seems to cover transiently,
and GI symptoms resolve, but ongoing liver damage continues. This stage may last two to
three days and is characterized by rising liver function transaminase, bilirubin, the
development of coagulopathy, and eventually hepatic encephalopathy. In the third stage,
both liver and renal function become compromised. Hepato-renal syndrome and hepatic
encephalopathy may occur rapidly after laboratory signs of liver injury, and death can occur
in three to seven days.
The mechanism of action of amatoxin is by inhibiting RNA polymerase, causing disruption
of transcription of mRNA. As a result, hepatocytes cannot synthesize key protein-coding
genes, leading to the disintegration of nucleoli and pathologically centrilobular hepatic
necrosis.
The veno-occlusive disease occurs with some toxic plants and herbal teas associated with
pyrrolizidine alkaloids. Periportal necrosis is more common in viral liver disease, especially
chronic hepatitis. Hepatic steatosis rarely leads to fulminant liver disease but is highly
prevalent in a high-fat diet, alcohol misuse, and metabolic syndrome.

Go to the next page if you knew the correct answer, or click the link image(s) below to further
research the concepts in this question (if desired).

Research Concepts:
Amatoxin Mushroom Toxicity

We update eBooks quarterly and Apps daily based on user feedback. Please tap flag to
report any questions that need improvement.
Question 770: A 65-year-old male is brought to the emergency department with complaints
of difficulty in breathing. His medical history includes chronic obstructive pulmonary disease
(COPD), diabetes mellitus, hypertension, sleep-disordered breathing, smoking, and alcohol use.
An arterial blood gas (ABG) performed at the time of presentation revealed a pH of 7.25, PaCO2
of 70 mmHg (FiO2 28%), PaO2 of 87 mmHg, and bicarbonate of 42 mmol/L. The patient is
admitted to the ICU and is placed on non-invasive ventilation. The usage of AVAPS (Average
volume-assured pressure support) modality is associated with which of the following advantages
in comparison to the BiPAP (Bilevel positive airway pressure) modality in this patient?

Choices:
1. Low risk of aspiration
2. Rapid clinical improvement
3. Low risk of air leaks
4. Decreased rate of treatment-failure
Answer: 2 - Rapid clinical improvement
Explanations:
AVAPS has been successful in the treatment of COPD-associated acute hypercapnic
respiratory failure.
Although both AVAPS and BiPAP are reliable in the treatment of acute hypercapnic
respiratory failure, AVAPS is associated with a speedy improvement of the patient
clinically. The patients also experience greater comfort and satisfaction comparatively, and
therefore, improved treatment-compliance.
Rather than having one fixed IPAP setting, the AVAPS mode can set a range of values for
the IPAP, a maximum and a minimum IPAP. The pressure-support is no longer fixed as the
IPAP changes by itself within the set range. The ventilator does this based on the targeted
tidal volume, a pre-set value. It uses a feedback loop to either increase or decreases the
inspiratory pressure from breath to breath to ensure the pre-set tidal volume is delivered.
Remember that both the therapies (AVAPS and BiPAP) are effectual, and there was no
difference in the duration of hospital stay or the time-period that the patient was on non-
invasive ventilation. Similar complications and contraindications exist for the varying
modes of NIPPV, including AVAPS and BiPAP.

Go to the next page if you knew the correct answer, or click the link image(s) below to further
research the concepts in this question (if desired).

Research Concepts:
Average Volume-Assured Pressure Support

We update eBooks quarterly and Apps daily based on user feedback. Please tap flag to
report any questions that need improvement.
Question 771: A 58-year-old man is about to undergo a lower extremity orthopedic
surgery. He has an epidural indwelling catheter to manage his pain intraoperatively. It is also
planned to leave in his epidural catheter after surgery and connect him to a patient-controlled
analgesia machine to manage his pain postoperatively. During the procedure, the patient
suddenly reports blurry vision, numbness and tingling around his mouth and a sense of
impending doom. What is the next best step in the evaluation of this patient?

Choices:
1. Order a stat potassium level
2. Echocardiogram
3. Electrocardiogram
4. Head CT
Answer: 3 - Electrocardiogram
Explanations:
Local anesthetic systemic toxicity is a complication of any sodium channel blocker that is
being injected.
Local anesthetic systemic toxicity occurs when the injection is placed into an artery or vein,
or so much has been placed in the local tissue that toxic doses have been absorbed into the
cardiovascular system.
Symptoms of local anesthetic systemic toxicity are tinnitus, blurred vision, dizziness,
tongue parathesias, circumoral numbness, nervousness, agitation, restlessness, and muscle
twitching. High concentrations can lead to prolonged PR intervals and widened QRS
complexes, and sinus/brady arrest.
When local anesthetic systemic toxicity presents in a patient, an electrocardiogram needs to
be performed immediately to monitor the effects on the heart and help decide if treatment
with Intralipid is necessary.

Go to the next page if you knew the correct answer, or click the link image(s) below to further
research the concepts in this question (if desired).

Research Concepts:
Patient Controlled Analgesia

We update eBooks quarterly and Apps daily based on user feedback. Please tap flag to
report any questions that need improvement.
Question 772: A 56-year-old man, post office worker, presents to the emergency
department with difficulty breathing and a dry cough with fatigue that has been worsening for
the last six days. His son called 911 today after he fell on the floor. He has a medical history of
diabetes and hyperlipidemia, for which he takes metformin and simvastatin. He never smoked
cigarettes or used illicit drugs. In the emergency department, he becomes severely hypoxemic
and requires immediate mechanical ventilation. He gets admitted to the intensive care unit. Work
up is negative for infectious, toxic, or autoimmune etiologies. X-rays reveal an interstitial pattern
bilaterally. HRCT shows ground-glass opacification. Lung biopsy reveals diffuse alveolar
damage with edema in the interstitium and alveolus. What is the definitive management option in
such a disorder?

Choices:
1. Rotating antibiotics
2. Glucocorticoids
3. Supportive care
4. Posaconazole
Answer: 3 - Supportive care
Explanations:
Acute Interstitial Pneumonia (AIP) is an acute, rapidly progressive idiopathic pulmonary
disease that often leads to fulminant respiratory failure and acute respiratory distress
syndrome (ARDS). Symptoms usually start as viral-like prodrome followed by shortness of
breath with cough, and fever, and progress rapidly to acute respiratory distress.
There is no proven treatment in acute interstitial pneumonia. Management is mainly based
on supportive care. Adequate oxygenation often cannot be achieved without invasive
mechanical ventilation.
As in acute respiratory distress syndrome, arterial blood gases can identify the severity of
respiratory distress based on Berlin Criteria (PaO2/FiO2), which may guide the treatment.
There is no proven treatment in acute interstitial pneumonia. Management is mainly based
on supportive care. Adequate oxygenation often cannot be achieved without invasive
mechanical ventilation.

Go to the next page if you knew the correct answer, or click the link image(s) below to further
research the concepts in this question (if desired).

Research Concepts:
Acute Interstitial Pneumonia

We update eBooks quarterly and Apps daily based on user feedback. Please tap flag to
report any questions that need improvement.
Question 773: A 65-year-old farmer was admitted to the hospital following exposure to
nitrogen dioxide after entering a silo and inhaling a reddish-brown cloud. The patient was
admitted for observation. After 24 hours, he got progressively worsening dyspnea, new bilateral
infiltrates on x-ray, and required supplemental oxygenation in the form of bilevel positive airway
pressure (BiPAP). Why was this patient's dyspnea and hypoxemia delayed?

Choices:
1. Nitrogen dioxide is highly water-soluble and has delayed effects on the lungs
2. Nitrogen dioxide is highly water-soluble and has an immediate effect on the lungs
3. Nitrogen dioxide has a low water-solubility and therefore can have delayed effects on the
lungs
4. The farmer had a concomitant exposure that delayed the effects of nitrogen dioxide
Answer: 3 - Nitrogen dioxide has a low water-solubility and therefore can have delayed
effects on the lungs

Explanations:
Nitrogen dioxide has a low water-solubility with little to no immediate warning signs.
A patient may have prolonged exposure without signs or symptoms.
When someone is exposed to nitrogen dioxide, they should be observed for delayed
pulmonary effects.
The lower the water-solubility, the greater the effect on the lower airways.

Go to the next page if you knew the correct answer, or click the link image(s) below to further
research the concepts in this question (if desired).

Research Concepts:
Nitrogen Dioxide Toxicity

We update eBooks quarterly and Apps daily based on user feedback. Please tap flag to
report any questions that need improvement.
Question 774: A 45-year-old man has been placed on venovenous (VV) extracorporeal
membrane oxygenation using femoral and internal jugular veins for cannulation for severe
refractory hypoxemia secondary to acute respiratory distress syndrome. His extracorporeal
membrane oxygenation flow is at 3.5L/min, and cardiac output is 6 L/min. Arterial blood gas
showed pH7.38, PaCO2 45 mmHg, PaO2 55 mmHg, HCO3 22 mEq/L. The FiO2 at the
membrane is 100%. His current ventilator settings include RR 20/min, tidal volume of 300 ml,
PEEP of 5, and FiO2 of 40%. The oxygen saturation of the blood entering the oxygenator is
90%, and the oxygen saturation leaving the oxygenator is 100%, and the oxygen saturation in the
venous blood in the superior vena cava is 65%, and the patient’s current arterial oxygen
saturation is 85%. How can his hypoxemia be improved?

Choices:
1. Increase the extracorporeal membrane oxygenation blood flow to 6L/min
2. This is the oxygenator problem and must be changed
3. Increase the ventilator settings to a PEEP of 10 and increase the FiO2 to 80%
4. Get an X-ray to look for malposition of cannulas
Answer: 4 - Get an X-ray to look for malposition of cannulas
Explanations:
This is a clear-cut scenario of recirculation during venovenous extracorporeal membrane
oxygenation, and it occurs when well-oxygenated blood returning from the extracorporeal
membrane oxygenation circuit gets sucked in by the venous drainage cannula again instead
of going through the lungs.
The classic signs of recirculation are low patient’s oxygen saturation and high pre-
oxygenator saturations and can be identified by the proximity of the drainage and the
reinfusion cannulae on the X-ray.
Increasing the pump flow might improve the oxygen saturations but will eventually increase
recirculation and causing further hypoxemia.
The complication of recirculation is very exclusive to venovenous extracorporeal membrane
oxygenation and can be minimized by increasing the distance between the drainage and
reinfusion cannulae.

Go to the next page if you knew the correct answer, or click the link image(s) below to further
research the concepts in this question (if desired).

Research Concepts:
Refractory Hypoxemia and Venovenous ECMO

We update eBooks quarterly and Apps daily based on user feedback. Please tap flag to
report any questions that need improvement.
Question 775: A 70-year-old man is brought to the hospital with confusion. He is
windowed and stays alone at home, but is able to perform day to day activities quite
independently. His kids visit him once a week or so. He had an intermittent low-grade fever for
the past week, for which his primary care provider had prescribed acetaminophen tablets. His
neighbor noticed subtle behavioral changes since the past week. Grandchildren who visited him
today found him irrelevant, abusive and aggressive. They also noticed that he has had passed
urine and stools in the living room. CT brain is unremarkable. A urine toxicology screen is
negative. CSF study shows RBCs 150 microL, WBCs 300/mmicroL (90% lymphocytes). The
protein is 136 mg/dL and glucose 50 mg/dL (corresponding blood glucose is 85 mg/dL).
Remaining CSF reports are awaited. Which of the following evaluations is the gold standard to
confirm the diagnosis in this patient?

Choices:
1. Brain biopsy
2. MRI brain with diffusion-weighted images and FLAIR sequences
3. Clinical symptoms and signs
4. CSF polymerase chain reaction (PCR)
Answer: 4 - CSF polymerase chain reaction (PCR)
Explanations:
The most sensitive and specific way to diagnose HSV encephalitis is to do an HSV PCR in
CSF. HSV PCR has got a sensitivity of 98% and a specificity of 94-100%. HSV PCR could
be false negative in the initial 72 hours and needs to be repeated after 3-7 days if the
suspicion is high. RBCs in the CSF can also give a false negative test.
Herpes simplex virus (HSV) can be type 1 or type 2. Herpes simplex virus type 1 (HSV-1)
is the most common cause of life-threatening sporadic encephalitis in the world. It causes
encephalitis in adults and children beyond the neonatal age. Neonatal herpes encephalitis
could be either due to type 1 or type 2, the latter being more common.
Herpes simplex encephalitis is associated with significant morbidity and mortality in
children and adults. A high index of suspicion, rapid diagnostic workups, and early
diagnosis with early initiation of acyclovir can further improve morbidity and mortality.
Clinical signs and symptoms are highly non-specific. Immunocompromised or elderly can
have atypical or subtle presentations. Cognitive, behavioral and personality changes could
be easily misdiagnosed as a psychiatric disorder. Classic radiologic features that have been
described include asymmetric involvement of the limbic system, abnormalities in the
inferior medial temporal lobes and inferolateral frontal lobes, and the insular cortices.
Diffusion-weighted MRI with flair sequences is 90% sensitive in detecting changes in HSE
(herpes simplex encephalitis). CT can detect changes only in half to two-thirds of patients.
Brain biopsy is very rarely performed nowadays, and considerations include undiagnosed
brain pathology where the patient is further deteriorating, or not responding to antiviral
treatment in spite of a positive HSV PCR.

Go to the next page if you knew the correct answer, or click the link image(s) below to further
research the concepts in this question (if desired).

Research Concepts:
Herpes Simplex Encephalitis

We update eBooks quarterly and Apps daily based on user feedback. Please tap flag to
report any questions that need improvement.
Question 776: A 65-year-old woman with a history of diabetes mellitus presents to the
emergency room with complaints of dysuria, nausea, vomiting and abdominal pain. Her vitals
are blood pressure 85/45 mmHg, heart rate 110/minute, temperature 101.5 F (36.6 C). After 3
liters of intravenous ringer lactate, there is no improvement in blood pressure. She is started on
norepinephrine infusion. The next day her creatinine is noted to be 2.1 mg/dL from baseline of
0.9 mg/dL. A urinalysis is sent and microscopic slide prepared. What is the most likely findings
to be seen in the urine sediment?

Choices:
1. Bland sediment, few renal tubular cells
2. Coarse, granular casts, renal tubular epithelial cells
3. Dysmorphic red blood cells
4. White blood cells, white blood cell casts
Answer: 2 - Coarse, granular casts, renal tubular epithelial cells
Explanations:
The patient is presenting with septic shock due to urosepsis, blood pressure is low and has
signs of infection. Renal injury associated with septic shock is acute tubular necrosis.
In acute tubular necrosis tubular cells are injured so reabsorption can not take place.
Hence the fractional excretion of sodium >1%, sediment shows renal tubular epithelial cells
along with casts, coarse, granular casts, also known as "muddy brown" casts.
Dysmorphic red blood cells and casts are seen in glomerulonephritis (example lupus
nephritis, anticytoplasmic antineutrophilic antibody (ANCA) vasculitis. White blood cells
and casts are seen more in acute interstitial nephritis. Bland sediment with few renal tubular
epithelial cells are seen in prerenal azotemia. Other findings are fractional excretion of
sodium 1%, BUN/ Cr ratio >20. These can be seen in volume depletion due to
gastrointestinal losses

Go to the next page if you knew the correct answer, or click the link image(s) below to further
research the concepts in this question (if desired).

Research Concepts:
Renal Failure

We update eBooks quarterly and Apps daily based on user feedback. Please tap flag to
report any questions that need improvement.
Question 777: A 65-year-old man is brought to the emergency department by his son for
chest pain and difficulty breathing for the past 8 days. He mentions that he developed headaches,
watery diarrhea, and abdominal pain 2 days ago. He recently returned from a cruise. The patient
smokes a pack of cigarettes for the past 36 years. Vital signs show blood pressure of 98/62
mmHg, pulse of 59/min, respiratory rate of 24/min, and temperature of 101 F (38.3 C). He
appears confused. Examination reveals lung crackles. Laboratory investigations reveal a sodium
level of 129 mEq/L, blood urea nitrogen level of 34 mg/dL, and elevated liver enzymes. Presence
of which of the following features in this patient most strongly relates to disease severity?

Choices:
1. Blood urea nitrogen level of 34 mg/dL
2. Sodium level of 129 mEq/L
3. Respiratory rate of 24/min
4. Pulse of 59/min
Answer: 1 - Blood urea nitrogen level of 34 mg/dL
Explanations:
This patient most likely has Legionnaire's disease. The most important clinical infection
caused by Legionella pneumophila in both children and adults is pneumonia.
In patients of severe disease, expect respiratory failure, bilateral pneumonia, pulmonary
infiltrates, and the presence of at least two of the following: diastolic blood pressure lower
than 60 mm Hg; respiratory rate greater than 30/min; and blood urea nitrogen greater than
or equal to 30 mg/dL.
The morbidity is highest in very young and very old patients, those who have chronic lung
disease, are immunosuppressed, patients with diabetes mellitus, and have delayed
antimicrobial therapy.
Bradycardia and hyponatremia are features of the disease and are not linked to disease
severity.

Go to the next page if you knew the correct answer, or click the link image(s) below to further
research the concepts in this question (if desired).

Research Concepts:
Legionnaires' Disease

We update eBooks quarterly and Apps daily based on user feedback. Please tap flag to
report any questions that need improvement.
Question 778: This 45-year old Indian lady has been ventilated for severe acute respiratory
distress syndrome secondary to COVID-19 pneumonia. Her oxygen requirement has gone up in
the past few minutes. Air entry was equal on both sides, and a bedside chest ultrasound revealed
the presence of pleural slides bilaterally, with bibasal consolidation and copious B lines. The
echocardiography was normal. Her current ventilator settings are show volume control
ventilation FiO2 0.7, tidal volume 350 cc, respiratory rate 32 per minute, PEEP 12 cms H2O.
Her predicted body weight (PBW) is 60 kg. Her latest ABG revealed PaO2 56 mm Hg, PaCO2
68 mm Hg, pH 7.273, HCO3 38 mEQ/L. The peak pressure is 38 cms H20, and the plateau
pressure is 30 cms. What should be the next best step in the management of this patient?

Choices:
1. Increase FiO2 and PEEP
2. Recruitment maneuvers
3. Proning
4. Diuresis
Answer: 3 - Proning
Explanations:
This lady has severe ARDS with a P/F ratio of 80 is very likely to get benefit from prone
position ventilation. Prone position ventilation is not only known to improve hypoxemia in
ARDS patients but also known to minimize lung injury by increasing homogeneity of
ventilation. The latter aspect has been shown in animal studies.
Prone position ventilation has been shown to improve the mortality in ARDS patients, as
shown in a 2010 metanalysis, and also at the landmark PROSEVA trial on severe ARDS
patients published in 2013. The reduced mortality is believed to be partly contributed by
decreased risk for ventilatory induced lung injury since prone position ventilation improves
the homogeneity of ventilation.
Prone position ventilation improves oxygenation by improving the end-expiratory lung
volume and by rectifying the ventilation-perfusion mismatch. Also, it reduces the mass
effect of the heart and mediastinum on the lower lobes and improves regional ventilation.
Prone position ventilation is relatively avoided in hemodynamically unstable patients and
patients with unstable spinal cord or abdominal injuries. Increased incidence of pressure
ulcers, increased endotracheal tube obstruction, and Intercostal tube displacements are
reported with prone position ventilation. ECMO is an alternate option to be considered
when there is a strong contraindication/inability to perform prone positioning.

Go to the next page if you knew the correct answer, or click the link image(s) below to further
research the concepts in this question (if desired).

Research Concepts:
Ventilator-Induced Lung Injury (VILI)

We update eBooks quarterly and Apps daily based on user feedback. Please tap flag to
report any questions that need improvement.
Question 779: A 66-year-old female presents to the emergency department in severe
respiratory distress. She has been having trouble breathing for the last two hours. She has had
chronic obstructive pulmonary disease (COPD) for eight years and is on home oxygen therapy.
Sometimes, she experiences acute exacerbations, but she describes her current symptoms as
being severe. Her respiratory rate is 42/min, heart rate is 98, BP is 118/70 mmHg, and her temp
is 98 F. Pulmonary examination shows decreased breath sound on both sides; the right side is
higher. She was resuscitated immediately, intubated, and mechanically ventilated. Her bedside x-
ray shows airspace disease suggestive of collapsed alveoli. What is the best way to monitor the
progress of her lung disease and allow for the optimization of mechanical ventilation settings in
the intensive care unit?

Choices:
1. Continuous pulse-oximetry and monitoring of her percentage oxygen saturation
2. Continuous monitoring of arterial blood pressure to guide judicious fluid administration and
minimize pulmonary edema
3. Continuous monitoring of respiratory compliance using continuous flow technique and
examining pressure-volume curve measurements
4. Daily bed-side x-ray and assessing evolution of the airspace disease
Answer: 3 - Continuous monitoring of respiratory compliance using continuous flow
technique and examining pressure-volume curve measurements

Explanations:
The patient is intubated as a result of acute respiratory distress syndrome (ARDS) and it is
vital to monitor the progress of her lung disease.
Respiratory compliance is a measure of the change in volume relative to the change in
pressure at any given time. The ventilator can measure pulmonary compliance continuously.
This measurement permits analysis of the static mechanical properties of the respiratory
system.
The static pressure-volume curve properties are impaired in ARDS.
Continuous flow technique is a simple and reliable tool to measure respiratory compliance
and helps in monitoring lung injury in patients with ARDS in ICU ventilator settings.

Go to the next page if you knew the correct answer, or click the link image(s) below to further
research the concepts in this question (if desired).

Research Concepts:
Pulmonary Compliance

We update eBooks quarterly and Apps daily based on user feedback. Please tap flag to
report any questions that need improvement.
Question 780: A 74-year-old male has been admitted to the hospital with malignant
hypertension. His medical history is remarkable for congestive heart failure,
hypercholesterolemia, alcoholic cirrhosis, and hypertension. He is started on an infusion of
sodium nitroprusside at a rate of 0.5 mcg/kg/minute that is titrated up to 2 mcg/kg/minute over a
period of 24 hours. The next day, he develops persistent bradycardia and confusion. O2
saturation is 89% on room air. Venous Blood gas demonstrates a pH of 7.05, pO2 of 102 mm
Hg, and pCO2 of 20 mm Hg, and bicarbonate of 8 mmol/L. What is the most appropriate
treatment for this patient?

Choices:
1. Dialysis
2. Thiosulfate infusion
3. Activated charcoal
4. Hyperbaric oxygen
Answer: 2 - Thiosulfate infusion
Explanations:
Nitroprusside is an effective intravenous antihypertensive medication that has a rapid onset
of action. It is approved for use in acute decompensated heart failure and acute
hypertension.
Prolonged use of nitroprusside can result in excess cyanide production. Cyanide toxicity
results in lactic acidosis and altered mental status and can develop in a matter of hours
following administration. Close attention must be paid to acidemia and dropping serum
bicarbonate levels (although these parameters are not always reliable indicators of toxicity
in early stages).
The cyanide toxicity can be treated with infusion of sodium thiosulfate, which detoxifies
cyanide into thiocyanate by acting as a sulfur donor.
Aside from cyanide toxicity, other important side effects of sodium nitroprusside are
profound hypotension with resultant end-organ ischemia, increased intracranial pressure,
methemoglobinemia, and thiocyanate toxicity.

Go to the next page if you knew the correct answer, or click the link image(s) below to further
research the concepts in this question (if desired).

Research Concepts:
Sodium Nitroprusside

We update eBooks quarterly and Apps daily based on user feedback. Please tap flag to
report any questions that need improvement.
Question 781: A 67-year-old man with a history of chronic obstructive pulmonary disease
(COPD) was transferred to the medical floor from the intensive care unit (ICU). He was admitted
to ICU three days ago due to acute hypoxic respiratory failure secondary to community-acquired
pneumonia requiring intubation and mechanical ventilation. On the medical floor, he has new
symptoms of abdominal discomfort, nausea, and two episodes of vomiting containing a small
amount of blood over the past several hours. He has no prior medical problems other than COPD
and has never had similar symptoms. The patient smokes a pack of cigarettes daily and drank 12
cans of beer before presentation to the emergency department. Vital signs currently are normal.
Abdominal examination reveals mild epigastric tenderness, but no masses, rebound or guarding.
Which of the following is the most likely cause of this patient's hematemesis?

Choices:
1. Partial esophageal tear
2. Pancreatic autodigestion Injury
3. Gastric mucosal erosion
4. Dilated esophageal vessels
Answer: 3 - Gastric mucosal erosion
Explanations:
Acute erosive gastropathy Is characterized by the development of severe hemorrhagic
lesions after the exposure of gastric mucosa to various injurious agents or after a substantial
reduction in blood flow.
Presenting symptoms include abdominal discomfort, nausea, vomiting, and hematemesis.
Mechanical ventilation for 48-72 hours is a risk factor for stress-related gastritis and gastric
ulcer.
Mallory-Weiss tear is a partial tear in the esophagus that occur in the distal esophagus at the
gastroesophageal junction after repeat bouts of retching and vomiting. This patient’s
vomiting is not severe enough to cause an esophageal tear.

Go to the next page if you knew the correct answer, or click the link image(s) below to further
research the concepts in this question (if desired).

Research Concepts:
Curling Ulcer

We update eBooks quarterly and Apps daily based on user feedback. Please tap flag to
report any questions that need improvement.
Question 782: A 65-year-old woman with a past medical history of a cerebral vascular
accident two weeks ago is brought to the emergency department with shortness of breath and
fever. She is diagnosed with septic shock and the decision is made to intubate her. Which of the
following is the most appropriate paralytic for intubation?

Choices:
1. Succinylcholine
2. Ketamine
3. Propofol
4. Rocuronium
Answer: 4 - Rocuronium
Explanations:
Rocuronium is the correct answer. Depolarizing muscle agents, such as succinylcholine, are
contraindicated in cases of neurologic injury due to post-synaptic receptor up-regulation
that typically occurs within three to five days. A cerebral vascular accident two weeks ago
places the patient at risk for life-threatening hyperkalemia.
The risk of hyperkalemia is not associated with decreased potassium clearance however
attention should be given to those who have chronically elevated potassium levels such as
renal failure patients.
Ketamine and Propofol are commonly used induction agents for both intubation and
procedural sedation. They are not neuromuscular blocking drugs. The conjunctive use of
induction agents is vital to rapid sequence intubation to reduce the sympathetic reflexes,
improve intubating conditions, and avoid the unwarranted effect of paralyzing a conscious
patient.
The most well-known depolarizing neuromuscular blocking agent is succinylcholine. It is
the only such drug used clinically and is considered by many the drug of choice for
emergency department rapid sequence intubation. It provides the fastest optimal conditions
during intubation of critically ill patients. However, it has several contraindications that
need to be considered before administration. Advocates of rocuronium cite its lack of
contraindications and avoidance of depolarization in the middle of an intubation attempt.

Go to the next page if you knew the correct answer, or click the link image(s) below to further
research the concepts in this question (if desired).

Research Concepts:
Depolarizing Neuromuscular Blocking Drugs

We update eBooks quarterly and Apps daily based on user feedback. Please tap flag to
report any questions that need improvement.
Question 783: A 76-year-old woman presents to the emergency department with abdominal
pain that radiates to her back. On admission, her heart rate is 110/min, with blood pressure 87/62
mmHg. Her past medical history includes coronary artery disease and documented abdominal
aortic aneurysm. An ultrasound reveals free intraperitoneal fluid and abdominal aortic aneurysm
with a diameter of 6 cm. What is the best next step in the management of this patient?

Choices:
1. Exploratory laparotomy
2. Emergent CT scan
3. Cardiological evaluation for elective repair
4. Endovascular stent-graft
Answer: 1 - Exploratory laparotomy
Explanations:
A ruptured abdominal aortic aneurysm is an emergent situation. The risk of death from a
ruptured aortic aneurysm is as high as 90%. Patients should be divided into 2 categories;
stable and unstable (as in this case), according to hemodynamic factors, such as blood
pressure, hurt rate, etc. A prompt aortic repair should be performed.
Since abdominal aortic rupture is an emergent situation, diagnosis is made clinically or with
the support of ultrasound. Abdominal aortic aneurysm rupture most commonly presented
with a combination of the 3; hypotension, pulsatile mass, and abdominal/back pain.
Ultrasound may demonstrate free intraperitoneal fluids or abdominal aortic aneurysm
rupture.
In the case of an unstable patient with suspected abdominal aortic aneurysm rupture
stabilizing and immediate open surgery should be performed. In case of a stable patient, a
CT aortogram should be performed, to confirm the diagnosis and assesses suitability for
emergency endovascular stent-graft repair.
Even though before an elective repair of abdominal aortic aneurysm all patients should
overgo preoperative cardiological evaluation, in case of an unstable patient with a ruptured
abdominal aortic aneurysm, preoperative cardiological evaluation is not held. Even if a
patient carries a high risk for cardiological related complication, surgery should not be
delayed.

Go to the next page if you knew the correct answer, or click the link image(s) below to further
research the concepts in this question (if desired).

Research Concepts:
Abdominal Aortic Repair

We update eBooks quarterly and Apps daily based on user feedback. Please tap flag to
report any questions that need improvement.
Question 784: A 17-year-old boy admitted to the hospital complains of a new-onset chest
pain that started a few hours ago. It is associated with mild shortness of breath. He also
complains of fever and a cough. On further questioning, he reveals that the cough is associated
with yellow-colored sputum production. A detailed medical history reveals that he had met with
an accident and underwent nephrectomy a week ago. He has no known medical conditions. His
vital signs show blood pressure 100/70 mmHg, respiratory rate 22 breaths/min, heart rate 98
beats/min, temperature 102 F (38.8 C), and oxygen saturation of 88% on room air. A chest
examination performed reveals late inspiratory crackles on auscultation. A chest x-ray performed
shows an area of opacity in the left lung. What is the most appropriate next step in management?

Choices:
1. Prescribe piperacillin-tazobactam
2. Prescribe vancomycin
3. Prescribe vancomycin and levofloxacin
4. Prescribe linezolid
Answer: 1 - Prescribe piperacillin-tazobactam
Explanations:
Pneumonia that occurs 48 -72 hours after hospital admission is known as hospital-acquired
pneumonia (HAP) and usually presents with signs and symptoms of cough with sputum,
fever, and shortness of breath.
Patients with HAP without any risk factors for MRSA (methicillin-resistant staph aureus)
infection such as prior intravenous antibiotic use within three months, hospitalization in a
unit with 20% or above of methicillin-resistant Staphylococcus aureus isolates, or unknown
prevalence of MRSA, and low risk of mortality are prescribed antibiotics with activity
against MSSA (methicillin-sensitive staph aureus) like piperacillin-tazobactam.
A physical examination reveals dullness on percussion, an increase in vocal fremitus, and
crackles on auscultation. Other treatment options include cefepime, levofloxacin, imipenem,
or meropenem.
Vancomycin and linezolid are given in case of MRSA infection and should not be used in
MSSA infections.

Go to the next page if you knew the correct answer, or click the link image(s) below to further
research the concepts in this question (if desired).

Research Concepts:
Nosocomial Pneumonia

We update eBooks quarterly and Apps daily based on user feedback. Please tap flag to
report any questions that need improvement.
Question 785: A 60-year-old woman is ready to be discharged on the sixth postoperative
day following an uneventful total hip replacement. A physical exam shows localized necrosis of
skin at the sites of subcutaneous injection of enoxaparin on the abdominal wall. Labs show a fall
in the platelet count from 216000/microL (five days ago) to 64000/microL (today). Which of the
following is the next best step in the management of this patient?

Choices:
1. Reduce the dose of enoxaparin by half and administer two units of FFP
2. Switch enoxaparin to fondaparinux
3. Stop enoxaparin and transfuse two units of adult platelets
4. Hold enoxaparin only
Answer: 2 - Switch enoxaparin to fondaparinux
Explanations:
This patient’s history is suggestive of heparin-induced thrombocytopenia (HIT). HIT is an
adverse drug reaction to heparin.
The incidence of HIT is estimated at 1% with low-molecular-weight heparins (LMWH) and
about 5% with unfractionated heparins (UFH). HIT has been described with every route of
heparin administration.
Where clinical suspicion of HIT is intermediate to high, it is essential to stop UFH or
LMWH. There should be no delay in commencing anticoagulation with alternative agents
like bivalirudin or fondaparinux while awaiting confirmatory tests, as the risk of thrombosis
remains as high as 50% even after stopping heparin.
Adverse reactions are either non-immune-mediated (type I) or immune-mediated (type II).
The non-immune mediated reaction typically has an earlier onset and seldom leads to a drop
in the platelet count below 100000/microL. The immune-mediated reaction is clinically
more significant as it is associated with thrombosis. It occurs between 5 to 14 days post-
heparin exposure, known as ‘typical’ HIT.

Go to the next page if you knew the correct answer, or click the link image(s) below to further
research the concepts in this question (if desired).

Research Concepts:
Heparin Induced Thrombocytopenia

We update eBooks quarterly and Apps daily based on user feedback. Please tap flag to
report any questions that need improvement.
Question 786: A 65-year-old male with unknown past medical history is brought to the
emergency department by ambulance for chest pain that began 20 minutes ago. On arrival, the
patient is non-verbal but does groan upon sternal chest rub. His heart rate is 150 bpm, blood
pressure 65/48 mm Hg, and SpO2 88 %. His EKG shows a QRS width of 140 ms. What is the
next step in management?

Choices:
1. Obtain an EKG and place on a cardiac monitor
2. Start cardiopulmonary resuscitation
3. Obtain IV access and start aggressive fluid resuscitation
4. Proceed to intubate
Answer: 4 - Proceed to intubate
Explanations:
Providers should always remain vigilant when managing wide complex tachycardia as these
patients can quickly become unstable and experience sudden cardiovascular collapse.
It is critical to first evaluate both the patient’s ABCs (airway, breathing, circulation) and
hemodynamic status because it will guide your management in the emergency setting.
In this scenario, the patient is hemodynamically unstable and does not appear to be
protecting his airway; therefore, he should be intubated.
Obtaining an electrocardiogram or maintaining an intravenous line is not the best course of
action. The patient should be intubated.

Go to the next page if you knew the correct answer, or click the link image(s) below to further
research the concepts in this question (if desired).

Research Concepts:
Wide QRS Complex Tachycardia

We update eBooks quarterly and Apps daily based on user feedback. Please tap flag to
report any questions that need improvement.
Question 787: An 80-year-old woman is admitted to the hospital with progressively
worsening dyspnea of one-month duration. For one week, she has been sleeping on a recliner.
She states that she can hardly walk one block without getting short of breath. She also complains
of worsening lower extremity swelling. Currently, she feels depressed as she is unable to lead an
active life, which is a big change compared to a month ago. She has a 3-year history of ischemic
cardiomyopathy and takes aspirin, bisoprolol, digoxin, spironolactone, and furosemide. On
examination, the patient is slightly anxious and has cold peripheries. Bilateral lower extremity
pitting edema is noted up to the mid-shins. She is afebrile with a pulse rate of 110/min (regular),
blood pressure of 80/60mmhg, and respiratory rate of 30/min. Jugular venous distention is
present. Auscultation is significant for an S3 and bilateral basal rales. Laboratory studies are
significant for mildly deranged liver enzymes (AST- 160 U/L, ALT- 173 U/L), elevated
creatinine (3.1mg/dl), hyponatremia (130meq/l). Serum digoxin levels are low (0.2ng/ml). EKG
shows sinus tachycardia. Which of the following is the next best step in the management of this
patient?

Choices:
1. Enroll the patient for the transplant list
2. Increase the dose of digoxin
3. Start on dobutamine
4. Start her on milrinone
Answer: 3 - Start on dobutamine
Explanations:
The cardiogenic shock is defined by persistent symptomatic hypotension and end-organ
dysfunction. Symptoms suggesting this include worsening exertional dyspnea and
orthopnea. Cold peripheries (suggesting poor peripheral perfusion) with leg edema, S3, and
basal crepitations are some signs of shock. End-organ damage which is evident with
elevated liver enzymes and deranged renal functions can be noted in patients with
cardiogenic shock. Heart failure leading to cardiogenic shock is treated with ionotropic
agents like milrinone or dobutamine.
Dobutamine acts on beta1 receptors of the heart and induces the ionotropic effect. Even
though milrinone has a similar effect, it is renally excreted and could get accumulated in
patients with renal failure.
Beta-blockers are usually not recommended for patients in cardiogenic shock with volume
overload. This has a negative ionotropic effect and should be restarted once the patient tides
over this crisis situation and is being diuresed adequately.
The primary goal should be to improve cardiac function. Digoxin is known to reduce the
hospitalizations in patients with heart failure, but cannot be increased in patients with
deranged renal functions.

Go to the next page if you knew the correct answer, or click the link image(s) below to further
research the concepts in this question (if desired).

Research Concepts:
Cardiogenic Shock

We update eBooks quarterly and Apps daily based on user feedback. Please tap flag to
report any questions that need improvement.
Question 788: A 19-year-old woman presents to the clinic after having been found to have
dysarthria, ataxia, drooling, and Parkinsonism. Her husband reports no recent hospitalizations,
but she did see her obstetrician a few months ago for placement of an intrauterine device (IUD)
for emergency contraception as they were not ready to start a family. He explains that she has
been told she has a defective gene on chromosome 13 that was prevalent in her family. Which of
the following is the next best step in the management of this patient?

Choices:
1. Emergent plasmapheresis
2. Remove her intrauterine device (IUD)
3. Initiate trientine therapy
4. Genetic analysis
Answer: 2 - Remove her intrauterine device (IUD)
Explanations:
Patients with Wilson disease are very sensitive to increases in total body copper levels.
Wilson disease is an autosomal recessive disorder caused by a mutation in the ATP7B gene
on chromosome 13, leading to impaired copper excretion into bile, leading to excess copper
accumulation within hepatocytes and subsequently extrahepatic copper deposition.
The copper IUD is routinely used as an emergency contraception method.
Removal of sources of excess copper and subsequent chelation with a first-line agent such
as penicillamine should be pursued.

Go to the next page if you knew the correct answer, or click the link image(s) below to further
research the concepts in this question (if desired).

Research Concepts:
Copper Toxicity

We update eBooks quarterly and Apps daily based on user feedback. Please tap flag to
report any questions that need improvement.
Question 789: A 65-year-old patient in the intensive care unit (ICU) develops abdominal
pain and diarrhea. He was admitted to the ICU for acute infective exacerbation of chronic
obstructive pulmonary disease (COPD) 7 days back. He was started on intravenous ceftriaxone
and moxifloxacin for his exacerbation. His condition seemed to be improving; however, today,
he has developed abdominal pain and has had 6 episodes of loose stools since morning. He has
also spiked a fever and complains of crampy abdominal pain. Examination reveals a pulse 105
beats per minute, a temperature of 100 F, respiratory rate of 22 per minute, and blood pressure of
100/70 mmHg. His oxygen saturation is 93 % on room air, and he is not dyspneic. Abdominal
examination reveals tenderness over the left lower quadrant and increased bowel sounds. His
investigations show a total leucocyte count of 20,000 per microL, serum creatinine of 2.1
mmol/l, and a C-reactive protein of 85 mg/L. What is the appropriate treatment for the patient's
condition?

Choices:
1. Intravenous piperacillin-tazobactam
2. Intravenous hydrocortisone
3. Oral vancomycin
4. Oral loperamide
Answer: 3 - Oral vancomycin
Explanations:
This is an elderly patient who has been admitted to intensive care with acute exacerbation of
chronic obstructive airway disease. He has been on broad-spectrum antibiotics and has now
developed fever and diarrhea, making the diagnosis of antibiotic-associated Clostridium
difficile colitis likely. The treatment of choice for Clostridium difficile colitis is oral
vancomycin.
Development of diarrhea with greater than 3 loose stools in 24 hours in a patient who is at
risk (hospitalization, broad-spectrum antibiotic use, elderly) should raise suspicion for
Clostridium difficile colitis induced colitis. The management options include oral
vancomycin, oral metronidazole, or oral fidaxomicin.
The initial dose recommendation for vancomycin is 125 mg every 6 hours, with a duration
of treatment of 10 days. Intravenous vancomycin is not effective as it is not secreted in the
colon. Occasionally, patients may present as ileus and rectal vancomycin as a retention
enema can be administered.
Vancomycin can also be used for recurrence and is administered in a pulsed and tapered
regime. For prevention, unnecessary broad-spectrum antibiotic use should be avoided, and
shifting to narrow-spectrum agents should be instituted as early as possible.

Go to the next page if you knew the correct answer, or click the link image(s) below to further
research the concepts in this question (if desired).

Research Concepts:
Clostridium Difficile

We update eBooks quarterly and Apps daily based on user feedback. Please tap flag to
report any questions that need improvement.
Question 790: A 67-year-old man presents with generalized abdominal pain for one week.
He also complains of worsening edema in both of his lower limbs and gaining about 10 pounds
(4.5 kg) in the last month. His past medical history includes multiple myeloma and alcohol use
disorder. Vitals reveal oxygen saturation 97% on 2 liters oxygen, respiratory rate 16 per minute,
heart rate 95 beats per minute, blood pressure 105/70 mmHg, and temperature 37 C. On physical
examination, his abdomen is distended with 2+ edema in both lower limbs. On palpation, his
abdomen is tense and diffusely tender. Lung and heart sounds are normal. Laboratory studies
show WBC 2 x 10^9/L (4-10), hemoglobin 8.5 g/dL (13-17), platelets 65 x 10^9/L (150-400),
sodium 128 mmol/L (135-145), potassium 5.5 mmol/L (3.5-5.0, chloride 101 mmol/L (95-105),
blood urea nitrogen mg/dL 70 (8-21), creatinine 7.2 (baseline 0.9), calcium 1.0 mmol/L (1.03-
1.23), AST 55 U/L (5-30), ALT 34 U/L (5-31). What is the next best step in the management of
this patient?

Choices:
1. Kidney biopsy
2. Intravenous hydration
3. Kidney ultrasound
4. Bladder pressure measurement
Answer: 4 - Bladder pressure measurement
Explanations:
It is essential to rule out intraabdominal hypertension and abdominal compartment
syndrome in patients presenting with abdominal distension and signs of multiorgan failure.
The normal intra-abdominal pressures range between 0 to 5 mmHg, while pressures
between 12 to 20 mmHg are indicative of intra-abdominal hypertension (IAH). Abdominal
compartment syndrome (ACS) is defined as an intra-abdominal pressure of more than 20
mmHg along with signs of organ failure.
The increased abdominal pressure decreases the compliance of the lungs, leading to
hypoxemia and increased afterload due to a decrease in venous return. Eventually, this can
lead to multi-organ failure.
Although ultrasound is recommended, the bladder pressure should be done immediately
after the abdominal examination findings reveal a firm or tense abdomen in the setting of
decreased urine output or renal failure. If the patient has ascites, it needs to be drained, and
a surgical consult is needed in cases with acute abdomen or perforated viscus.

Go to the next page if you knew the correct answer, or click the link image(s) below to further
research the concepts in this question (if desired).

Research Concepts:
Abdominal Compartment Syndrome

We update eBooks quarterly and Apps daily based on user feedback. Please tap flag to
report any questions that need improvement.
Question 791: Which of the following is the best initial management of the patient with
diabetic ketoacidosis who becomes comatose after 12 hours of aggressive treatment?

Choices:
1. Hyperventilation
2. Stop insulin therapy
3. Stop K+ supplementation
4. Oxygen therapy
Answer: 1 - Hyperventilation
Explanations:
Hyperventilation reduces the blood flow through the brain by causing cerebral
vasoconstriction.
Stopping insulin therapy is not recommended.
Other modes of treatment would be mannitol to promote diuresis.
The most basic definition of cerebral edema is swelling of the brain. It is a relatively
common phenomenon with numerous etiologies. Cerebral edema categorizes into either
vasogenic, cellular, osmotic, and interstitial causes. It can arise from a variety of causes,
including head trauma, vascular ischemia, intracranial lesions, or obstructive hydrocephalus
resulting in interstitial edema. The consequences of cerebral edema can be devastating, even
fatal, if untreated. The explanation of the mechanism of injury arising from cerebral edema
comes via the Monroe-Kellie doctrine. The Monroe-Kellie doctrine states that space of the
cranial cavity is fixed in volume and contains fixed proportions of brain matter
(approximately1400 ml), blood (approximately 150 ml) and cerebrospinal fluid
(approximately 150 ml). Because of this fixed space, an increase in the volume of one of
these components must, therefore, result in the loss of another component in equal amounts.
In cerebral edema, the relative volume of brain tissue increases as the brain tissues swells
with edema. This increased relative brain volume decreases perfusion (blood) to the brain,
and the pressure can cause further damage to both the edematous and non-edematous brain.
Clinical presentation of cerebral edema is variable, ranging from asymptomatic to severe
autonomic dysregulation, coma, and death. Symptoms appear as the intracranial pressure
(ICP) rises above 20 cm H2O in most patients. Treatment for cerebral edema targets the
underlying cause and any life-threatening complications. Treatments include
hyperventilation, osmotherapy, diuretics, corticosteroids, and surgical decompression.

Go to the next page if you knew the correct answer, or click the link image(s) below to further
research the concepts in this question (if desired).

Research Concepts:
Cerebral Edema

We update eBooks quarterly and Apps daily based on user feedback. Please tap flag to
report any questions that need improvement.
Question 792: A 17-year-old male with a past medical history of asthma and depression is
found unconscious by a bystander and taken to an emergency department. He is minimally
responsive, so a detailed history cannot be obtained. On examination, his blood pressure is
100/70 mmHg, pulse 112/min, respiratory rate 12/min, and oxygen saturation 99% on room air.
His pupils are reactive to light, and his skin is flushed. Abdominal examination reveals decreased
bowel sounds and suprapubic fullness. An EKG shows QRS complex widening. Lab results,
including liver function tests, are unremarkable. His blood-alcohol level is less than 5%, and the
urine toxicology screen is negative. What is the best next step in management?

Choices:
1. Check an acetaminophen level
2. Observation
3. Administer IV sodium bicarbonate
4. Administer flumazenil
Answer: 3 - Administer IV sodium bicarbonate
Explanations:
Considering the patient is relatively unresponsive and the liver function tests are normal, an
acetaminophen level would be one of the tests to consider, but not necessarily the best next
step based on the presentation.
If the patient was stable and responsive, observation could be the next appropriate step. At
this time, an intervention is needed as the patient is minimally responsive.
The question stem indicates the patient presents with likely unknown intoxication, widening
of the QRS complex, reduced bowel sounds, and urinary retention which is indicative of
tricyclic antidepressant toxicity. The best next step would be administering IV sodium
bicarbonate. This would increase the sodium load and alkalinization, thereby being
cardioprotective.
Flumazenil is used in benzodiazepine intoxication. Patients with benzodiazepine
intoxication are sedated, and present with decreased respiratory rate but do not have specific
EKG findings as mentioned in the question stem.

Go to the next page if you knew the correct answer, or click the link image(s) below to further
research the concepts in this question (if desired).

Research Concepts:
Amitriptyline

We update eBooks quarterly and Apps daily based on user feedback. Please tap flag to
report any questions that need improvement.
Question 793: Pulmonary capillary wedge pressure increases in which cardiac disorder?
Choices:
1. Tricuspid stenosis
2. Right ventricle failure
3. Pulmonary hypertension
4. Mitral stenosis
Answer: 4 - Mitral stenosis
Explanations:
Left sided heart failure can increase PCWP.
The normal left ventricular diastolic pressure is 5 mm Hg. A pressure gradient across the
mitral valve of 20 mm Hg due to severe mitral stenosis will cause a left atrial pressure of
about 25 mm Hg. This left atrial pressure is transmitted to the pulmonary vasculature
resulting in pulmonary hypertension.
As left atrial pressure remains elevated, the left atrium will increase in size. As the left
atrium increases in size, there is a greater chance of developing atrial fibrillation. If atrial
fibrillation develops, the atrial kick is lost.
Thus, in severe mitral stenosis, the left ventricular filling is dependent on the atrial kick.
With loss of the atrial kick, there is a decrease in cardiac output and sudden development of
congestive heart failure.

Go to the next page if you knew the correct answer, or click the link image(s) below to further
research the concepts in this question (if desired).

Research Concepts:
Mitral Stenosis

We update eBooks quarterly and Apps daily based on user feedback. Please tap flag to
report any questions that need improvement.
Question 794: A 38-year-old woman with a past medical history of obesity and 10 pack-
year smoking is brought to the hospital with symptoms of hallucinations, disorganized behavior,
and catatonia. Her husband mentions that the patient complained of headache and fever 10 days
prior to admission. Physical examination is notable for new-onset tachycardia and fluctuating
blood pressure. Anti-NMDAR IgG antibodies are detected by indirect immunofluorescence
assay in the patient's serum. Which of the following is the next best step in the evaluation of this
patient?

Choices:
1. Transvaginal ultrasound
2. Chest x-ray
3. Blood culture
4. Urine toxicology
Answer: 1 - Transvaginal ultrasound
Explanations:
This scenario describes a case of Anti-NMDA receptor encephalitis.
The disease starts with a prodromal state mimicking common viral infections. But within
weeks to a few months (less than 3 months), complex neuropsychiatric features emerge
rapidly during the psychotic phase. The psychotic phase evolves into the unresponsive
phase characterized by mutism, decreased motor activity, and catatonia. Following the
unresponsive phase, a hyperkinetic phase with autonomic instability and prominent
movement disorders becomes evident.
Anti-NMDAR IgG antibodies, detected by indirect immunofluorescence assay in the serum
and the CSF, are diagnostic for the disease.
After confirmation of the diagnosis, a comprehensive evaluation should be done to detect
underlying malignancy. Transvaginal ultrasonography is the most crucial test for young
women presenting with the illness due to the high incidence of underlying ovarian teratoma.

Go to the next page if you knew the correct answer, or click the link image(s) below to further
research the concepts in this question (if desired).

Research Concepts:
Anti-NMDA Receptor Encephalitis

We update eBooks quarterly and Apps daily based on user feedback. Please tap flag to
report any questions that need improvement.
Question 795: A 30-year-old male athlete presents to the clinic with a history of syncope
and palpitations. He also reports a history of the loss of his father at a younger age. On cardiac
examination, he has a double pulse, and on auscultation, a systolic ejection murmur is heard,
which increases on Valsalva maneuver. The electrocardiogram (ECG) is suggestive of left
ventricular hypertrophy. What is the gold standard test to confirm the diagnosis in this patient?

Choices:
1. 2D echocardiography
2. Cardiac magnetic resonance imaging (MRI)
3. Cardiac catheterization
4. Radionuclide imaging
Answer: 2 - Cardiac magnetic resonance imaging (MRI)
Explanations:
Hypertrophic obstructive cardiomyopathy (HOCM) is an autosomal dominant condition
characterized by obstruction of the left ventricular outflow tract and asymmetric septal
hypertrophy. It is typically seen in young athletes who die of sudden cardiac death. The
evaluation includes ECG, 2D-echocardiography, cardiac MRI, radionuclide imaging, and
cardiac catheterization.
Cardiac MRI is established as a gold standard test in patients with HOCM. It not only helps
in anatomical evaluation and risk stratification but also diagnoses complications of HOCM,
such as aneurysm and fibrosis.
Though initial evaluation is always with 2D-echocardiography (which will diagnose 80% of
patients with HOCM), it still can miss HOCM in 20% of the patients. 2D-echocardiography
assesses ventricular function, septal hypotrophy, cardiac dimensions and helps in ruling out
other valvular heart diseases.
Cardiac catheterization and nuclear scanning may not be required in most cases. In selective
cases, it may be used to assess outflow tract obstruction, the severity of mitral valve
regurgitation, and the patency of the coronary arteries.

Go to the next page if you knew the correct answer, or click the link image(s) below to further
research the concepts in this question (if desired).

Research Concepts:
Pulsus Bisferiens

We update eBooks quarterly and Apps daily based on user feedback. Please tap flag to
report any questions that need improvement.
Question 796: A 65-year-old woman with a history of hypertension is brought to the clinic
for ongoing memory issues. The patient is accompanied by her husband, who reports that the
patient has had trouble with short-term memory for the past year. She is able to carry out her
activities of daily living but is now requiring assistance to walk and has to wear diapers because
she had a few episodes of incontinence. Upon examination, her Mini-Mental State Examination
score is 24/30, significant for 0/3 recall in 5 minutes. The neurological exam is significant for a
wide-based gait with short steps. A CT head without contrast shows dilated ventricles. Which of
the following is the next best step in the management of this patient?

Choices:
1. High volume lumbar puncture
2. Urinalysis
3. Thyroid function tests
4. MRI brain without contrast
Answer: 1 - High volume lumbar puncture
Explanations:
The patient's history and physical likely suggest normal pressure hydrocephalus. The next
step should be lumbar puncture with gait assessment before and after. Another more
effective method of evaluation is external lumbar drainage. A larger amount of CSF was
removed, and brain volume change was greater than with lumbar puncture.
Cognitive decline, magnetic gait, and urinary incontinence form the triad of normal-
pressure hydrocephalus.
CT head shows dilated ventricles. MRI brain will not help with the diagnosis or the
management.
Frequent incontinence and use of diapers predispose to urinary tract infections. However,
the patient does not complain of any symptoms suggestive of infection or fluctuating course
of confusion.

Go to the next page if you knew the correct answer, or click the link image(s) below to further
research the concepts in this question (if desired).

Research Concepts:
Differentiating Delirium Versus Dementia in the Elderly

We update eBooks quarterly and Apps daily based on user feedback. Please tap flag to
report any questions that need improvement.
Question 797: A 17-year-old college student is brought by his family to the emergency
department with complaints of fever, headache, and confusion. He has recently returned from
China, where he worked on a pig farm over the summer. He did not receive any particular
vaccination before the trip. Family remembers he was troubled by mosquito bites. Computed
tomography (CT) scan of the head shows bilateral thalamic hemorrhages. Cerebrospinal fluid
studies show increased lymphocytes, elevated protein, normal glucose, and raised opening
pressure. What is the most appropriate treatment for this condition?

Choices:
1. Doxycycline
2. Supportive care with intravenous fluids and antipyretics
3. Acyclovir
4. Penicillin
Answer: 2 - Supportive care with intravenous fluids and antipyretics
Explanations:
Japanese encephalitis is the most common form of preventable mosquito-borne encephalitis.
It is common in countries in South-East Asia and the Western Pacific.
Transmission is highest in rural areas, especially near pig farms or rice paddies with
standing water during the summer and rainy seasons.
There is an effective vaccine for prevention that is recommended for travelers to endemic
areas. However, there is no effective treatment beyond supportive care with IV fluids and
antipyretics.
Bilateral thalamic edema, lesions, or hemorrhage are common in Japanese encephalitis, and
their presence in an appropriate clinical context should suspect this disease.

Go to the next page if you knew the correct answer, or click the link image(s) below to further
research the concepts in this question (if desired).

Research Concepts:
Japanese Encephalitis

We update eBooks quarterly and Apps daily based on user feedback. Please tap flag to
report any questions that need improvement.
Question 798: A 90-year-old patient woman is transferred to the hospital from her nursing
home for evaluation of a declining neurological status. CT scan showed a collection of blood in
her left lateral rectus muscle. Which of the following conditions is most likely to be associated
with this finding?

Choices:
1. Factor V Leiden mutation
2. Atrial fibrillation
3. Alzheimer disease
4. Fanconi anemia
Answer: 2 - Atrial fibrillation
Explanations:
Spontaneous muscular hematomas have an increased incidence in patients who are on
anticoagulation therapy.
Patients with atrial fibrillation are commonly treated with anticoagulation therapy.
Spontaneous muscular hematomas almost exclusively occur in elderly patients on
anticoagulation therapy.
Factor V Leiden mutations lead to hypercoagulability, and would not increase the incidence
of spontaneous muscular hematomas.

Go to the next page if you knew the correct answer, or click the link image(s) below to further
research the concepts in this question (if desired).

Research Concepts:
Muscular Hematoma

We update eBooks quarterly and Apps daily based on user feedback. Please tap flag to
report any questions that need improvement.
Question 799: A 70-year-old man noted left side weakness when she woke up at 7:30 am,
and he presented to the emergency department at 10:00 am; he was completely fine when he
went to sleep at 11:00 pm the night before. On exam, he has left visual field cut, left
hemineglect, left facial palsy, and left side weakness that involves the upper limb more than the
lower limb. CT head shows right dense MCA sign and no evidence of hemorrhage, CTA head
and neck shows proximal (M1) right middle cerebral artery occlusion, and CT head perfusion
study shows a substantial region of salvageable hypo-perfused tissue, blood pressure is 135/85
mmHg, which of the following treatments is indicated?

Choices:
1. IV labetalol
2. Mechanical thrombectomy
3. IV heparin
4. IV recombinant tissue plasminogen activator (rtPA)
Answer: 2 - Mechanical thrombectomy
Explanations:
Embolic strokes may result in large vessel occlusion and cause severe strokes. Such patients
may benefit from mechanical thrombectomy, which can be done within the first 6 hours as
demonstrated in five different randomized controlled clinical trials including MR CLEAN,
ESCAPE, REVASCAT, SWIFT PRIME, and EXTEND-1A, and up to 24 hours from
symptoms onset in selected patients to save the viable brain tissues at risk as demonstrated
in the DAWN and DEFUSE 3 trials such as this patient with a substantial region of
salvageable hypo-perfused tissue at risk.
If endovascular therapy is contemplated, a noninvasive intracranial vascular study such as
CTA head and neck is strongly recommended but should not delay intravenous r-tPA for
patients who should be administered first. However, this patient presented more than 4.5
hours from symptoms onset and, therefore, not a candidate for IV r-tPA.
Data shows a systolic blood pressure target of less than 140 mm Hg was safe and associated
with better outcomes than the guideline target of 180 mm Hg. This patient systolic BP is
less than 140 mm Hg, and there is no need for IV labetalol.
IV heparin is not indicated for acute stroke management before mechanical thrombectomy.

Go to the next page if you knew the correct answer, or click the link image(s) below to further
research the concepts in this question (if desired).

Research Concepts:
Embolic Stroke

We update eBooks quarterly and Apps daily based on user feedback. Please tap flag to
report any questions that need improvement.
Question 800: A 16-year-old patient with status epilepticus following severe head injury is
being managed with paralyzing agents and kept on a ventilator. During clinical examination, the
treating provider notices synchronous rhythmic constriction and dilatation of both his pupils.
What is the next step in the management of the patient?

Choices:
1. Ocular ultrasound
2. CT head
3. MRI scan
4. Bedside EEG
Answer: 4 - Bedside EEG
Explanations:
The presence of synchronous rhythmic constriction and dilatation of the pupils is termed as
pupillary hippus.
The pupillary hippus signifies the presence of refractory nonconvulsive status epilepticus
and is a reliable bedside neurological sign in monitoring epilepsy in paralyzed patients.
When paralyzing agents are used in treating a patient with refractory status epilepticus, EEG
monitoring should be performed to assess the effectiveness of anticonvulsant therapy and to
rule out subclinical seizures.
CT, MRI scans, and ocular ultrasound only help in monitoring any changes in the
intracranial pressure and identifying probable causes for epilepsy.

Go to the next page if you knew the correct answer, or click the link image(s) below to further
research the concepts in this question (if desired).

Research Concepts:
Status Epilepticus

We update eBooks quarterly and Apps daily based on user feedback. Please tap flag to
report any questions that need improvement.
Section 9
Question 801: A 70-year-old African American male with a past medical history of
hypertension and diabetes presents to the emergency department complaining of severe
abdominal pain. The patient reports the pain started 2 hours ago and has been progressively
worsening. He describes the pain as throbbing, 9/10 in severity, and located in the left-sided
flank region. He denies any associated burning urination, nausea, vomiting, or any history of
urinary tract infection or renal stones. The patient has a 35 pack-year smoking history and drinks
three to four 20 ounce cans of beer daily but denies illicit drug use. On physical exam, the patient
appears to be diaphoretic, pale, and in severe distress due to pain. The abdominal exam is
significant for tenderness and ecchymosis. Vital signs show a blood pressure of 80/60 mmHg,
heart rate of 140 beats per minute, and SpO2 95% on room air. EKG shows sinus tachycardia.
Wide bore catheter access is obtained, and fluid resuscitation is started immediately. Basic labs,
including complete blood counts and basic metabolic panel, are sent. What is the next best step
in the management of this patient?

Choices:
1. CT angiography of thorax and abdomen
2. Departmental abdominal ultrasound
3. Exploratory laparotomy
4. Focused bedside abdominal ultrasound
Answer: 4 - Focused bedside abdominal ultrasound
Explanations:
The likely diagnosis in this patient is an abdominal aortic aneurysmal rupture, which is a
surgical emergency.
The next best step of management for this patient is to perform a bedside ultrasound of the
abdomen.
Bedside ultrasound should be done quickly to save precious time in this patient. Once the
diagnosis is confirmed, the patient should be immediately transferred to the operating room
for an emergent repair.
The patient has never been diagnosed with an abdominal aortic aneurysm in the past, so it is
important to perform a quick bedside ultrasound to confirm the diagnosis before transferring
the patient to the operating room. CT angiography is not indicated at this time since it takes
longer to perform and would delay surgery. Departmental abdominal ultrasound requires the
patient to fast, and it is time-consuming to transport the patient to the designated ultrasound
area.

Go to the next page if you knew the correct answer, or click the link image(s) below to further
research the concepts in this question (if desired).

Research Concepts:
Abdominal Aortic Aneurysm Rupture

We update eBooks quarterly and Apps daily based on user feedback. Please tap flag to
report any questions that need improvement.
Question 802: A 65-year-old male comes to the emergency department with the complaint
of sudden onset of weakness in his right arm and leg. He noticed these symptoms one hour ago.
Currently, he is unable to walk, and he has slurred speech. On examination, his blood pressure is
100/60 mmHg and pulse 98/min regular. The neurologist is consulted, and he recommends an
imaging study that shows compressed basal cisterns and midline shift. Which of the following is
the initial emergent treatment of his condition?

Choices:
1. Withdrawing cerebrospinal fluid via lumbar puncture
2. Mannitol
3. Acetazolamide
4. Phenobarbital coma
Answer: 2 - Mannitol
Explanations:
When intracranial hypertension is suspected, immediate intervention is necessary. Mannitol
is commonly used as a hyperosmolar agent and is usually given as a bolus of 0.25 to 1 g/kg
body weight. Serum osmolality should be kept less than 320 mOsm to avoid side effects of
therapy like renal failure, hypokalemia, and hypoosmolarity.
Mannitol works immediately, and a urinary catheter should be inserted to monitor urine
output. In the meantime, a neurosurgery consult should be obtained to determine the
benefits of a ventriculostomy.
If the patient is on a ventilator, hyperventilation can be performed. The low PCO2 can also
cause vasoconstriction of the vessels in the brain.
In addition to positioning the patient upright with a straight neck, initiate hyperosmolar
therapy.

Go to the next page if you knew the correct answer, or click the link image(s) below to further
research the concepts in this question (if desired).

Research Concepts:
Intracranial Hypertension

We update eBooks quarterly and Apps daily based on user feedback. Please tap flag to
report any questions that need improvement.
Question 803: A 16-year-old boy is brought to the emergency department after a motor
vehicle collision. He loses a lot of blood and requires a red blood cell (RBC) transfusion. After
the initiation of transfusion, he develops facial swelling and has difficulty breathing. On
examination, he has audible expiratory wheezing, laryngeal edema, and his blood pressure is
80/35 mmHg. He is emergently treated with epinephrine. On further questioning, he reports a
past medical history of recurrent otitis media, sinusitis, and lung infections. On exam, he is found
to have atopic dermatitis. Patients with this condition can be safely transfused with which RBCs?

Choices:
1. Washed RBCs
2. IgM deficient donor RBC
3. Frozen RBCs
4. Leukoreduced RBCs
Answer: 1 - Washed RBCs
Explanations:
IgA deficient individuals can develop an anaphylactic reaction to blood products even with
the first transfusion.
The suitable products for patients with selective IgA deficiency are those prepared from
other IgA-deficient individuals or saline-washed red blood cells or Deglycerolized,
previously frozen RBCs.
Patients with IgA deficiency who develop anaphylaxis during infusion or transfusion of
blood products will require laboratory testing to determine the presence of anti-IgA
antibodies.
Deglycerolization is equivalent to washing.

Go to the next page if you knew the correct answer, or click the link image(s) below to further
research the concepts in this question (if desired).

Research Concepts:
IgA Deficiency

We update eBooks quarterly and Apps daily based on user feedback. Please tap flag to
report any questions that need improvement.
Question 804: A 65-year-old man is admitted to the intensive care unit with septic shock.
Two liters of fluid has been infused, but blood pressure is still low. He requires central venous
access for vasopressor administration. He has a history of diabetes mellitus, hypertension, and
chronic renal failure. He is being dialyzed thrice per week through a right forearm arteriovenous
fistula. Which of the following locations is most appropriate for central line placement in this
individual?

Choices:
1. Left internal jugular vein
2. Right subclavian vein
3. Left femoral vein
4. Right internal jugular vein
Answer: 4 - Right internal jugular vein
Explanations:
The internal jugular vein is a common site for central line placement. The right internal
jugular vein has the straightest course to the right atrium. It is relatively easy to place a
central line in the right IJV with a high success rate with or without ultrasound. It is an
easily compressible site in the case of bleeding or accidental arterial puncture and it has
relatively low pneumothorax risk as compared to the subclavian vein. Due to ease of
placement and relatively fewer complications, right IJV is often the site chosen by
anesthesiologists for central line placement.
The left internal jugular has a smaller caliber than the right internal jugular vein. It has a
tortuous path to the right atrium. Because of the tortuous path, left internal jugular vein
canulation is associated with increased malposition. The incidence of arterial puncture is
higher with the left internal jugular vein as it is smaller in size and overlays the internal
carotid artery more often than the right internal jugular vein.
The presence of AV fistula in the right arm makes subclavian vein undesirable for central
line placement as it may cause interference with the outflow from the arm and potentially
may compromise the patient’s dialysis access.
The femoral veins carry a higher risk of thrombosis and infection.

Go to the next page if you knew the correct answer, or click the link image(s) below to further
research the concepts in this question (if desired).

Research Concepts:
Internal Jugular Vein Central Venous Access

We update eBooks quarterly and Apps daily based on user feedback. Please tap flag to
report any questions that need improvement.
Question 805: A 65-year-old male patient who just moved to the area and is setting up a
primary care facility presents to the hospital with complaints of low-grade fever, cough, and
shortness of breath. The patient's previous medical record shows that he was diagnosed with HIV
more than five years ago; however, the patient has been only intermittently compliant with the
treatment. The examination of the chest reveals bilateral crepitations. The patient is evaluated
with a chest x-ray, which shows bilateral lower lobe pneumonia. What is the minimum CD4
count required to start him on co-trimoxazole?

Choices:
1. 20
2. 40
3. 200
4. 500
Answer: 3 - 200
Explanations:
Pneumocystis Carinii Pneumonia (PCP), now referred to as Pneumocystis Jirovecii
Pneumonia is a fungal infection that most commonly affects the immunocompromised and,
in some cases, can be severely life-threatening. Typically, patients at risk are those with any
underlying disease states that alter host immunity such as those with cancer, HIV, transplant
recipients, or those taking immunosuppressive therapies and medications.
Individuals with HIV infection whose CD4 count falls below 200, and who are not on
Pneumocystis jiroveci (PCP) prophylaxis, are at increased risk for opportunistic infections.
Other patients prone to PCP include those with an immunosuppressive disorder like severe
combined immunodeficiency syndrome. Individuals who are malnourished are also
susceptible to PCP.
Finally, patients receiving long term immunosuppressive therapy following a transplant are
also prone to PCP.

Go to the next page if you knew the correct answer, or click the link image(s) below to further
research the concepts in this question (if desired).

Research Concepts:
Pneumocystis Jirovecii Pneumonia

We update eBooks quarterly and Apps daily based on user feedback. Please tap flag to
report any questions that need improvement.
Question 806: A 66-year-old woman presents to the emergency department with an onset
of dark-colored urine that began last night. Her past medical history is positive for hypertension,
hyperlipidemia, and osteoarthritis in both knees. Her chart indicates a prescription was given one
week ago for cephalexin following staph-induced cellulitis in her left axilla. What protein is most
likely to be found in the highest in concentration in her urine?

Choices:
1. Hemoglobin
2. Albumin
3. Myoglobin
4. Insulin
Answer: 1 - Hemoglobin
Explanations:
A potential side effect of cephalexin is hemolytic anemia, which results in the lysis of red
blood cells.
Red blood cells contain high concentrations of hemoglobin.
This can present as dark-colored urine.
Cephalexin can also cause nausea and interstitial nephritis.

Go to the next page if you knew the correct answer, or click the link image(s) below to further
research the concepts in this question (if desired).

Research Concepts:
Cephalexin

We update eBooks quarterly and Apps daily based on user feedback. Please tap flag to
report any questions that need improvement.
Question 807: A 17-year old patient has been brought to the emergency department. She is
minimally responsive and when the monitor is placed she has a very wide QRS with sinus
tachycardia. What class of medications has she likely ingested?

Choices:
1. Opioids
2. Tricyclic antidepressants
3. Analgesics
4. Beta blockers
Answer: 2 - Tricyclic antidepressants
Explanations:
Tricyclic antidepressants cause prolonged QRS complexes on ECG and this is an early sign
of toxicity.
Tricyclic antidepressants have a narrow therapeutic window, increasing the likelihood of
toxicity.
Sinus tachycardia is a very common feature of TCA toxicity and a wide QRS supports the
diagnosis.
The serum levels of tricyclic antidepressants do not correlate with the symptoms and one
must use the ECG and other blood work to support the diagnosis.

Go to the next page if you knew the correct answer, or click the link image(s) below to further
research the concepts in this question (if desired).

Research Concepts:
Tricyclic Antidepressant Toxicity

We update eBooks quarterly and Apps daily based on user feedback. Please tap flag to
report any questions that need improvement.
Question 808: A 29-year-old parturient patient at 38 weeks of gestation is undergoing
epidural catheter placement for labor and delivery pain management. Loss of resistance is
noticed, and the catheter is placed. A combination of lidocaine 2% and epinephrine 1:200,000 is
pushed through the catheter. The patient develops hypotension, respiratory depression, aphonia,
and inability to move arms and legs. What is the cause of the patient's current condition?

Choices:
1. Valvular heart disease
2. Familial dysautonomia
3. Drug toxicity
4. Medication wrong-route administration
Answer: 4 - Medication wrong-route administration
Explanations:
Heart valves abnormalities and heart function are very important considerations in pregnant
women undergoing neuraxial anesthesia due to the risk of sympathetic depression caused by
the anesthetics and associated complications. However, the patient's presentation is more
consistent with a total spinal (acute aphonia and high anesthetic level are not be associated
with a heart valve dysfunction).
Familial dysautonomia symptoms include the inability to produce tears, frequent lung
infections, difficulty maintaining the right temperature, bed-wetting, and poor balance.
The initial symptoms of local anesthetics toxicity include tinnitus, dizziness, circumoral
numbness, tongue paresthesias, and blurred vision.
The foramen magnum is the superior boundary of the epidural space. However, the spinal
space extends superiorly reaching the cranial nerves. Total spinal occurs when local
anesthetics reach the cranial subarachnoid space potentially manifesting symptoms such as
fixed dilated pupils, aphonia, and loss of consciousness. In this patient, the local anesthetic
was meant to be administered in the epidural space but was injected in the intrathecal space
instead.

Go to the next page if you knew the correct answer, or click the link image(s) below to further
research the concepts in this question (if desired).

Research Concepts:
Epidural

We update eBooks quarterly and Apps daily based on user feedback. Please tap flag to
report any questions that need improvement.
Question 809: A 59-year-old woman presents to the emergency department with a rapidly
worsening vision and an acute excruciating, debilitating headache, described as the worst in her
life. She denies having any focal neurologic deficits. Shortly after arrival, her blood pressure is
measured to be 190/90 mmHg, and she develops a generalized tonic-clonic seizure for one
minute. She loses consciousness and does not follow commands thereafter. Although no further
seizures are witnessed, she begins to deteriorate quickly, and her Glasgow Coma Scale (GCS)
drops to 7. She is intubated, and the intensivist is consulted for admission into the intensive care
unit. Which of the following is the next best step in the management of this patient?

Choices:
1. Control blood pressure emergently
2. Obtain an electroencephalogram (EEG) and start an antiepileptic drug
3. CT of the brain without IV contrast
4. MRI of the brain with and without IV contrast
Answer: 3 - CT of the brain without IV contrast
Explanations:
This individual’s presentation is concerning for a subarachnoid hemorrhage, possibly due to
an aneurysmal rupture. It is crucial to differentiate an emergent diagnosis, such as
subarachnoid hemorrhage, from posterior reversible encephalopathy syndrome (PRES),
especially since both diseases can present clinically in a similar way: visual disturbance,
seizures, acutely altered mentation, and headaches.
A CT of the head without IV contrast is obtained to help assess for intracranial hemorrhage,
including subarachnoid hemorrhage, and this is done as soon as the patient is stabilized. A
CT of the head without IV contrast can help to distinguish intracranial hemorrhage from
hypodense cerebral edema.
In PRES, cerebral edema is typically located in the parietal and occipital lobes, although
other regions can be involved. Sometimes, a CT of the head cannot detect cerebral edema,
and an MRI of the brain is recommended to observe the edema. The MRI, however, can
take a long time to obtain. In the clinical scenario above, assessing for acute intracranial
hemorrhage with a CT of the head is the next critical step.
This question also helps to illustrate the importance of relying on taking a good clinical
history. A subarachnoid hemorrhage can be distinguished clinically from PRES, as the
former is acute in origin, sudden, and associated with an immediately peaking excruciating
worst headache of life called thunderclap headache.

Go to the next page if you knew the correct answer, or click the link image(s) below to further
research the concepts in this question (if desired).

Research Concepts:
Posterior Reversible Encephalopathy Syndrome

We update eBooks quarterly and Apps daily based on user feedback. Please tap flag to
report any questions that need improvement.
Question 810: A 65-year-old male patient presents to the emergency department
complaining of muscle spasms and diarrhea. On examination, he has a dry mouth, tachycardia,
hypertension, hyperreflexia, diaphoresis, and dilated pupils. His friend says he takes “some
medicine for something”. What is likely to be in his medical history and medicine cabinet?

Choices:
1. Gout/Colchicine
2. Chronic Diarrhea/Lomotil (Diphenoxylate and Atropine)
3. Depression/amitriptyline
4. Schizophrenia/chlorpromazine
Answer: 4 - Schizophrenia/chlorpromazine
Explanations:
Colchicine (plant toxin/secondary metabolite) usually causes GI symptoms and CNS
changes like confusion, delirium, and hallucinations.
Lomotil has biphasic toxicity (anticholinergic and then opioid-like) and this presentation
may be consistent with the first phase. Hyperreflexia is not a usual feature of this overdose.
Tricyclic overdoses often cause tachycardia but hypotension is often present. Also, CNS
depression is common, not excitation.
Thorazine and other antidepressants can have extrapyramidal effects (a dopamine
antagonist) and in overdose cause a serotonin syndrome differentiated from other conditions
(like Neuroleptic Malignant Syndrome) by hyperreflexia. Basic treatment involves
abstinence from or removal of the drug (GI decontamination), benzodiazepines, and
supportive care.

Go to the next page if you knew the correct answer, or click the link image(s) below to further
research the concepts in this question (if desired).

Research Concepts:
Deadly Single Dose Agents

We update eBooks quarterly and Apps daily based on user feedback. Please tap flag to
report any questions that need improvement.
Question 811: A 65-year-old woman with hypertension, hyperlipidemia, and toxic
multinodular goiter undergoes total thyroidectomy. Morning bloodwork the day after surgery
reveals a calcium level of 6.2 mg/dL, phosphorus 2.0 mg/dL, albumin 4.1 mg/dL and PTH 72.0
pg/mL. An EKG is obtained, and her QTc is 480 ms. Which of the following is the best initial
therapy for this patient?

Choices:
1. Calcium carbonate 1000 mg by mouth daily starting right away
2. Calcium citrate 4000 mg by mouth once a day starting right away
3. Calcium chloride 1g IV STAT
4. Calcium gluconate 2g IV STAT followed by continuous infusion
Answer: 4 - Calcium gluconate 2g IV STAT followed by continuous infusion
Explanations:
Symptomatic or severe hypocalcemia, i.e. calcium level less than 7.6 mg/dL, requires IV
calcium treatment.
Calcium gluconate is less toxic if it extravasates into surrounding tissues.
Calcium chloride has higher elemental calcium content but requires central venous access
for administration.
Oral calcium should be initiated as soon as the patient can tolerate diet.

Go to the next page if you knew the correct answer, or click the link image(s) below to further
research the concepts in this question (if desired).

Research Concepts:
Hungry Bone Syndrome

We update eBooks quarterly and Apps daily based on user feedback. Please tap flag to
report any questions that need improvement.
Question 812: A 55-year-old man is brought in by emergency medical services (EMS) after
being involved in a motor vehicle accident. On examination, the patient responds inconsistently
and specifically to external stimuli, and he withdraws to painful stimuli. Which of the following
would also be expected in this category of the rancho los amigos scale (RLAS)?

Choices:
1. Responds more to his wife
2. Responds more to the medical staff
3. Ability to follow simple commands
4. Tremor, athetosis, and involuntary movements
Answer: 1 - Responds more to his wife
Explanations:
A patient with a rancho los amigos score of 3 demonstrates only localized responses to
external stimuli inconsistently. Voluntary motor testing will be unavailable for the therapist.
The patients will respond more to familiar people (friends and family) versus strangers.
When assessing motor control in these patients, only reflexes, rigidity, and tone can be
assessed.
Withdrawal secondary to pain would be a sensory reflex evaluation. The patient is unlikely
to be able to follow simple commands.

Go to the next page if you knew the correct answer, or click the link image(s) below to further
research the concepts in this question (if desired).

Research Concepts:
Ranchos Los Amigos

We update eBooks quarterly and Apps daily based on user feedback. Please tap flag to
report any questions that need improvement.
Question 813: A medical director of the adult intensive care unit (ICU) started using
angiotensin II one year back in the ICU. In the last one-year angiotensin II was administered to
50 patients. It was observed to work best in patients with a subset of acute respiratory distress
syndrome (ARDS) and septic shock. The director earmarks a fellow to study the potential
relationship between the finding. What is the reason for the improved efficacy of this medication
in ARDS and septic shock patients?

Choices:
1. Angiotensin II works best in patients with a high mortality rate
2. Reason for improved efficacy is unknown
3. Angiotensin II improves lung endothelium function
4. Procalcitonin is high in patients with ARDS and septic shock
Answer: 3 - Angiotensin II improves lung endothelium function
Explanations:
Angiotensin I is converted into angiotensin II by the angiotensin-converting enzyme, which
is produced by the lung endothelium. In acute respiratory distress syndrome (ARDS), there
is an injury of the endothelium leading to a decrease in the production of this enzyme,
resulting in potentially angiotensin II deficiency state.
In patients with ARDS and septic shock, angiotensin II seems to work better because of this
potential hypothesis of angiotensin II deficiency state.
Any other condition which impacts the lungs and causes vasoplegic shock would potentially
benefit from angiotensin II based on a similar mechanism.
Patients with ARDS and septic shock are certainly a very sick group of patients, and they
may have high procalcitonin. However, that is not the reason for the efficacy of this
medication in that group if patient.

Go to the next page if you knew the correct answer, or click the link image(s) below to further
research the concepts in this question (if desired).

Research Concepts:
Angiotensin II

We update eBooks quarterly and Apps daily based on user feedback. Please tap flag to
report any questions that need improvement.
Question 814: A 43-year-old man with a past medical history of end-stage renal disease is
brought to the emergency department for altered mental status. The patient’s initial vital signs
show heart rate 39/min, blood pressure 94/51 mmHg, respiratory rate 19/min, and oxygen
saturation 83% on room air. The patient withdraws his right arm when pinched, opens his eyes
spontaneously, and is speaking in clear sentences but is oriented only to self. The patient displays
an apneic pattern of breathing. An immediate decision is made to secure the patient’s airway. As
assessment of the patient’s airway shows an overbite of the jaw, a short neck, and morbid
obesity. While the patient is being pre-oxygenated, a point of care glucose shows a level of 94
mg/dL, and an electrocardiogram reveals peaked T-waves with widened QRS-complexes. The
patient is pre-oxygenated and achieves an oxygen saturation of 95%. After rapid sequence
intubation via direct laryngoscopy, the patient continues to desaturate and is now 79% despite
being mechanically ventilated. What is the most likely cause of this patient’s desaturation?

Choices:
1. Hyperkalemia
2. Pulmonary embolism
3. Esophageal intubation
4. Myocardial Infarction
Answer: 3 - Esophageal intubation
Explanations:
The patient who has a history of end-stage renal disease and presents to the emergency
department with altered mental status can have any number of reasons as to why he is
altered. However, once intubated, the most common complication of tracheal intubation of
difficult airways is esophageal intubation (choice 3). Esophageal intubation can cause the
desaturation of a patient despite mechanical ventilation.
The patient achieved adequate oxygenation prior to intubation via a bag-valve-mask, and
rapid desaturation after intubation is likely to be esophageal intubation. In morbidly obese
patients, auscultation is ineffective in determining breath sounds after mechanical
ventilation, and it is hard to see abdominal distention. Therefore it is safe to assume the
patient’s esophagus is intubated.
Esophageal intubation is common in patients with difficult airways. Intubations should be
confirmed with end-tidal carbon dioxide levels or color change monitors to ensure
successful cannulation of the trachea.
In patients with pulmonary embolism (choice 2), an increase in oxygen saturation would not
be expected; in addition, patients might be tachycardic and might complain of chest pain.
Patients with myocardial infarction (choice 4) can also cause a patient to desaturate.
However, the electrocardiogram shows evidence of hyperkalemia (choice 1), which will
cause bradycardia and cardiac arrest but does not typically cause rapid desaturation after
intubation.

Go to the next page if you knew the correct answer, or click the link image(s) below to further
research the concepts in this question (if desired).

Research Concepts:
Difficult Airway

We update eBooks quarterly and Apps daily based on user feedback. Please tap flag to
report any questions that need improvement.
Question 815: A 30-year-old primigravid female presents to the emergency department at
36 weeks of gestation with headache, vomiting, and right upper quadrant pain for the last three
hours. Her medical history is significant for gestational hypertension, which was being controlled
with labetalol. She has no other medical condition and has no known allergies. She is up-to-date
with her prenatal care and was tested GBS positive. Her body mass index is 28 kg/m2, and she
has gained about 16 kg during the pregnancy. Her temperature is 99 F, her blood pressure is
170/95 mmHg, and her pulse is 77/min. Physical examination a confused pregnant female with
right upper quadrant tenderness and reduced bowel sounds. Fetal heart rate tracing shows a
baseline of 150/min with moderate variability, no decelerations, and no accelerations. There is
moderate bilateral pedal edema. Complete blood count reveals a hemoglobin of 9 g/dl, platelet
count of 90,000/m3, and leukocyte count of 9000/m3. Liver function tests are deranged.
Coagulation studies are unremarkable. Proteinuria is also noted on urinalysis. Which of the
following correctly explains her presentation?

Choices:
1. Distended hepatic capsule
2. Obstruction of bile flow
3. Acute fatty liver
4. Liver capsule inflammation and adhesion
Answer: 1 - Distended hepatic capsule
Explanations:
The patient has presented with preeclampsia with severe features. Preeclampsia is
characterized by the triad of hypertension, proteinuria, and edema. Severe features include
right upper quadrant pain, elevated liver enzymes, thrombocytopenia, clonus, hyperactive
reflexes, etc.
Right upper quadrant pain is consistent with HELLP syndrome, one of the severe features
of preeclampsia. Further laboratory testing shows hemolytic anemia, thrombocytopenia, and
elevated liver enzymes.
Systemic inflammation and activation of the coagulation cascade, which leads to
microangiopathic hemolytic anemia, subsequently causes hepatocellular necrosis and
thrombi in portal circulation. This causes liver swelling, distension of liver capsule, and
right upper quadrant pain.
Acute fatty liver of pregnancy is also a complication of the third trimester or early
postpartum period. However, this setting is more consistent with HELLP syndrome. Option
four is consistent with pelvic inflammatory disease resulting in the right upper quadrant due
to Fitz-Hugh-Curtis syndrome.

Go to the next page if you knew the correct answer, or click the link image(s) below to further
research the concepts in this question (if desired).

Research Concepts:
Hypertension In Pregnancy

We update eBooks quarterly and Apps daily based on user feedback. Please tap flag to
report any questions that need improvement.
Question 816: A 37-year-old man with no past medical history is admitted to the trauma
intensive care unit following a car accident with polytrauma and traumatic brain injury. After
resuscitation and appropriate continued management, he has absent pupillary and corneal
reflexes, with intact gag and cough reflexes. The legal surrogate is told that he cannot be
declared brain dead but has little chance of meaningful neurological recovery. The surrogate
expresses that the patient would have wanted to 'die peacefully' and not be attached to artificial
life support but asks if organ donation is contradictory to comfort care?

Choices:
1. Contact the organ donation team for immediate organ harvesting
2. Inform the surrogate that organ donation is possible (donation after circulatory death) where
discontinuation of life support and comfort/end-of-life care can be provided in the operating
room prior to organ procurement
3. Continue life support indefinitely
4. Tell the surrogate that organ donation is not possible
Answer: 2 - Inform the surrogate that organ donation is possible (donation after circulatory
death) where discontinuation of life support and comfort/end-of-life care can be provided in the
operating room prior to organ procurement

Explanations:
'Termination of life support' involves several steps and can be conducted prior to organ
procurement in the appropriate clinical scenario.
Communication with legal surrogate must be detailed to explain the process
The patient's wishes should be respected, and attempt made to follow them as closely as
possible in the given clinical condition
Contacting the organ transplant team is not unethical and is part of the Donation after
Circulatory Death (DCD) process. Comfort measures are an essential component of that
process

Go to the next page if you knew the correct answer, or click the link image(s) below to further
research the concepts in this question (if desired).

Research Concepts:
Termination of Life Support

We update eBooks quarterly and Apps daily based on user feedback. Please tap flag to
report any questions that need improvement.
Question 817: A 70-year-old man is brought to the emergency department after being
found in an altered mental status, requiring endotracheal intubation in the field. Post-intubation
assessment reveals that the pulse-oximetry remains at 60%, and the capnography waveform is
flat, and the value is 0. What is the next best step in the airway management of this patient?

Choices:
1. Cricothyrotomy
2. Remove the endotracheal tube and repeat endotracheal intubation
3. King tube placement
4. Laryngeal mask airway
Answer: 2 - Remove the endotracheal tube and repeat endotracheal intubation
Explanations:
A flat capnography waveform with a value of zero and the patient remaining hypoxic would
indicate that the endotracheal tube is not in the trachea and is likely in the esophagus. The
tube would need to be removed, and repeat intubation would need to be performed to obtain
proper placement of the endotracheal tube.
Appropriate placement of the tube would be noted with a good (nonflat) capnography
waveform and a range value of 35-45 mmHg.
The gold standard for assessing the placement of an ET is direct visualization with the help
of a laryngoscope. Additional ways to assure proper confirmation of endotracheal tube
placement include carbon dioxide, capnography waveform, chest x-ray, ultrasound, and
clinical assessment. The AHA (American Heart Association) recommends continuous
waveform capnography besides clinical assessment as the most reliable method of
confirming and monitoring correct placement of an ET tube. Bedside mobile ultrasound is
another resource that some emergency departments have to confirm the position of the ET
tube. Many physicians frequently use a chest x-ray to assess the placement of the ET tube.
Clinically, abnormal tube placement can be diagnosed with absent breath sounds on the left
chest, if right mainstem intubation occurred, and no bilateral breath sounds bilaterally if an
esophageal intubation occurred. Additionally, with esophageal intubation air may be
auscultated in the mid-epigastric region upon ventilation administration. Lastly, low oxygen
saturation will be noted.

Go to the next page if you knew the correct answer, or click the link image(s) below to further
research the concepts in this question (if desired).

Research Concepts:
Airway Management

We update eBooks quarterly and Apps daily based on user feedback. Please tap flag to
report any questions that need improvement.
Question 818: A 72-year-old Caucasian male with a history of quadriparesis, post-
traumatic stress disorder (PTSD), diabetes, insomnia, and a chronic indwelling Foley catheter
presents for follow up with his primary care clinician after being discharged from the hospital
with sepsis secondary to pyelonephritis. He was told that he had an acute kidney injury while
hospitalized. His baseline glomerular filtration rate (GFR) is 46ml/minute. He takes baclofen for
muscle spasticity, prazosin for his PTSD, metformin for his diabetes, and diphenhydramine as
needed to help him sleep. Today, in the office, his GFR is 26ml/minute and his liver enzymes are
normal. He is concerned that he may need to stop medications given his decreased kidney
function. What is the preferred next management?

Choices:
1. Continue prazosin and diphenhydramine, then discontinue metformin
2. Continue prazosin, discontinue metformin and diphenhydramine
3. Continue metformin and diphenhydramine, then decrease the dose of prazosin
4. Continue diphenhydramine, baclofen, metformin, and prazosin at their respective current dose
Answer: 1 - Continue prazosin and diphenhydramine, then discontinue metformin
Explanations:
Prazosin and diphenhydramine are hepatically metabolized and require no adjustment of the
dose based on his GFR. Metformin is contraindicated with a GFR less than 30ml/min and
should be avoided with a GFR between 31-45ml/min.
Prazosin and diphenhydramine require no adjustment of dose based on GFR. Metformin is
contraindicated with a GFR less than 30ml/min.
Prazosin and Diphenhydramine require no adjustment of dose based on GFR. Metformin is
contraindicated with a GFR less than 30ml/min.
All medications should not be continued as metformin is currently contraindicated.
Baclofen has a risk of toxicity with renal impairment and dose should be decreased or
discontinued. Some clinicians recommend discontinuation of baclofen with a GFR less than
30ml/min. Prazosin and diphenhydramine may be continued at their current dose.

Go to the next page if you knew the correct answer, or click the link image(s) below to further
research the concepts in this question (if desired).

Research Concepts:
Prazosin

We update eBooks quarterly and Apps daily based on user feedback. Please tap flag to
report any questions that need improvement.
Question 819: A 48-years-old male with Marfan syndrome presented to the emergency
department after a motor vehicle collision. He was wearing a seat belt, and he decelerated rapidly
to save a pedestrian and swerved into a ditch. He complained of severe chest pain. Physical
examination reveals several bruises on the chest. His respiratory examination is unremarkable.
Which is most likely on a chest x-ray?

Choices:
1. Multiple right-sided rib fractures
2. A left pulmonary contusion
3. Hemomediastinum
4. Widening of the mediastinum
Answer: 4 - Widening of the mediastinum
Explanations:
Traumatic aortic injuries are usually deceleration injuries because of a huge differential
force to the fixed and mobile parts of the thoracic aorta. The most common sites of injury
are the following: aortic isthmus; just distal to the origin of the left subclavian artery as this
is the transition zone between the mobile ascending aorta and the relatively fixed
descending aorta; tethered site of the aorta at the ligamentum arteriosum, and the ascending
aorta, just proximal to the origin of the brachiocephalic vessels.
Patients with Marfan disease are more prone to these injuries. A chest x-ray suggestive of
thoracic aortic rupture may show loss of aortic knob, widening of the mediastinum,
deviation of nasogastric tube in the esophagus, an apical cap, or sternal fracture.
If there is a high clinical suspicion, then further diagnostic workup should be pursued,
which includes aortography, CT angiography, and transesophageal echocardiography.
A contrast-enhanced helical CT scan is the screening diagnostic investigation of choice for
aortic trauma.

Go to the next page if you knew the correct answer, or click the link image(s) below to further
research the concepts in this question (if desired).

Research Concepts:
Aortic Trauma

We update eBooks quarterly and Apps daily based on user feedback. Please tap flag to
report any questions that need improvement.
Question 820: A 43-year-old male is brought in to the emergency department with
concerns of epigastric pain with radiation to the back. He has a long history of daily alcohol
consumption and recently loss his job because of this. Labs are remarkable for a lipase level of
3,545 U/L. The patient is admitted to the regular medicine floor and started on aggressive fluid
resuscitation, pain control with morphine and he is kept NPO. On the second day of
hospitalization, the patient is found lethargic and hypotensive. Vital signs show blood pressure of
74/36 mmHg, which does not improve with 2L bolus of intravenous normal saline, hear rate of
114 bpm, respiratory rate of 14 breaths per minute and temperature of 97.6 F. What is the next
best step in management for this patient?

Choices:
1. Another bolus 2 L of intravenous normal saline
2. Initiate norepinephrine infusion
3. Hydrocortisone
4. Vancomycin and meropenem
Answer: 2 - Initiate norepinephrine infusion
Explanations:
Hypovolemic shock is a serious complication of hypovolemia. It refers to persistent low
arterial blood pressure after appropriate fluid resuscitation.
The etiology of hypovolemia is of utmost importance to properly treat and manage this
complication.
In the acute setting, the etiology may not be readily apparent. In this case, fluid resuscitation
and vasopressor support may be required to prevent further organ damage.
In this patient, pancreatitis appears to be the most likely etiology of hypovolemia and shock.

Go to the next page if you knew the correct answer, or click the link image(s) below to further
research the concepts in this question (if desired).

Research Concepts:
Hypovolemia

We update eBooks quarterly and Apps daily based on user feedback. Please tap flag to
report any questions that need improvement.
Question 821: A 35-year-old man comes into the hospital after being stranded on a top of a
mountain during the winter. The man was found by a search team after going missing for five
days. The patient's past medical history is unremarkable. On physical exam, the patient is
hypothermic and found to have frostbite on all his fingers and toes. The fingers and toes appear
dry, shrunken, and gangrenous. The decision is made to remove the frostbite fingers, but after the
patient is stabilized. The patient is admitted to the hospital for observation before surgery. On
day 4 in the hospital, the fingers and toes are found to be swollen with purulent discharge. Upon
palpation of the fingers and toes, crepitus is found. The provider suspects an infection of the
fingers and toes. What is the most likely microbe causing the infection?

Choices:
1. Gram-negative anaerobic bacteria
2. Gram-positive aerobic bacteria
3. Gram-positive anaerobic bacteria
4. Gram-negative aerobic bacteria
Answer: 3 - Gram-positive anaerobic bacteria
Explanations:
When extremities become necrotic and gangrenous, they may become infected with
Clostridium perfringens.
Frostbite is a form of dry gangrene. When gangrene becomes infected it becomes wet
gangrene.
Wet gangrene that has crepitus indicates a gas-producing organism. C. Perfringens
classically cause superinfections with crepitus. This microbe is a gram-positive spore-
forming anaerobic bacteria. It classically metabolizes carbohydrates into gas creating
crepitus along with infection.
Dry gangrene is a form of necrosis due to ischemia. Due to the prolonged cold
temperatures, the blood supply to the extremities is constricted. The body tries and redirect
the blood supply to the most vital organs. This protective mechanism can lead to organ
failure and limb loss.

Go to the next page if you knew the correct answer, or click the link image(s) below to further
research the concepts in this question (if desired).

Research Concepts:
Hypothermia

We update eBooks quarterly and Apps daily based on user feedback. Please tap flag to
report any questions that need improvement.
Question 822: A 16-year-old male patient presents to the hospital with complaints of
general malaise, weight loss, night sweats, and arthralgias for more than three weeks. He had a
holiday abroad where he got a tattoo, which later became itchy and red. On examination, he has a
high-grade fever, the heart rate is 52/min, and there are petechial hemorrhages in his conjunctiva
and oral mucosa. On auscultation, there are no cardiac murmurs, and the chest is clear. Urine
dipstick shows hematuria. His white cell count is 16000/microliter, and C-reactive protein is 140
mg/dl. An electrocardiogram shows that p waves are unrelated to QRS complexes. What is the
most likely diagnosis?

Choices:
1. Rheumatic fever
2. Infective endocarditis
3. Viral hemorrhagic fever
4. Pyelonephritis
Answer: 2 - Infective endocarditis
Explanations:
This patient appears to be suffering from infective endocarditis, which should be considered
first in patients with chronic fever, weight loss, malaise, and absence of murmurs.
The history of skin infection makes the staphylococcus infection the more likely cause. The
presence of a third-degree cardiac block shows the involvement of conducting tissue.
During a physical examination, attention should be paid to any signs of infection or skin
rashes, such as rheumatic fever, Lyme disease, and endocarditis, which cause heart blocks.
Conjunctival petechial hemorrhages are much more common than the classical features of
splinter hemorrhages, Osler nodes, and Janeway lesions.

Go to the next page if you knew the correct answer, or click the link image(s) below to further
research the concepts in this question (if desired).

Research Concepts:
Third-Degree Atrioventricular Block

We update eBooks quarterly and Apps daily based on user feedback. Please tap flag to
report any questions that need improvement.
Question 823: A 72-year-old woman with a past medical history significant for myasthenia
gravis, hypertension, hyperlipidemia, and acid reflux presents to the hospital with complaints of
generalized weakness and difficulty breathing. Her home medications include physostigmine,
lisinopril, atorvastatin, and omeprazole. She is diagnosed with myasthenia flare and treated with
plasmapheresis and albumin as the replacement fluid. After initiating plasmapheresis, she
develops flushing, bradycardia, and hypotension. Which of the following is most likely to have
triggered this event?

Choices:
1. Inadequate volume replacement
2. Hyperviscosity syndrome
3. Vasovagal reflex
4. Lisinopril
Answer: 4 - Lisinopril
Explanations:
The use of ACE inhibitors has been associated with atypical reactions during
plasmapheresis. Anaphylactic or atypical reactions including flushing, hypotension,
dyspnea, and bradycardia have been reported.
The reactions were worse when albumin was used as replacement fluid, and the patient had
ACE exposure in the last 24 hours.
A possible mechanism is increased kinin production.
It can be minimized by avoiding the use of an ACE inhibitor or by using a different
replacement fluid.

Go to the next page if you knew the correct answer, or click the link image(s) below to further
research the concepts in this question (if desired).

Research Concepts:
Angiotensin Converting Enzyme Inhibitors (ACEI)

We update eBooks quarterly and Apps daily based on user feedback. Please tap flag to
report any questions that need improvement.
Question 824: A 65-year-old man with a past medical history of hyperlipidemia, migraines,
and osteoarthritis presents to the hospital with new-onset atrial fibrillation. His heart rate
generally runs low between 40/min and 50/min. In an attempt to cardiovert the patient, a
transesophageal echocardiogram is performed without signs of thrombus, and electric
cardioversion is attempted with 300 Joules. The patient's atrial fibrillation is persistent, and he
fails electric cardioversion. The patient is advised about the long term effects of atrial fibrillation
and is interested in attempting antiarrhythmic therapy. Treatment with dofetilide is being
considered. Which of the following complications is most likely to occur in this patient with
treatment?

Choices:
1. Ventricular arrhythmia
2. Worsening bradycardia
3. Tachycardia
4. Right axis deviation
Answer: 1 - Ventricular arrhythmia
Explanations:
Dofetilide may induce or worsen ventricular dysrhythmias, producing life-threatening
polymorphic ventricular tachycardia.
Patients with a history of torsades de pointes, prolonged QTc (greater than 440 ms),
hypomagnesemia, or serum potassium below 4.0 mEq/L are at increased risk of developing
ventricular dysrhythmias when placed on dofetilide.
Patients are generally hospitalized when starting dofetilide to be able to actively monitor for
dosing related ventricular arrhythmias.
Dofetilide and sotalol are higher risk for torsades de pointes than amiodarone within the
class III antiarrhythmic medication.

Go to the next page if you knew the correct answer, or click the link image(s) below to further
research the concepts in this question (if desired).

Research Concepts:
Dofetilide

We update eBooks quarterly and Apps daily based on user feedback. Please tap flag to
report any questions that need improvement.
Question 825: A 49-year-old African American male with a past medical history of
hypertension and tobacco use presents to the emergency department complaining of left-sided
tearing chest pain radiating to his back. The patient states that he ran out of his hypertensive
medication approximately two weeks ago. The physical exam is pertinent for a patient in severe
distress with a grade IV/VI diastolic decrescendo murmur heard best at the left 3rd intercostal
space, which was not present at the previous outpatient visit one month ago. Vital signs are
notable for a heart rate of 106/min, blood pressure 205/115 mmHg in the left arm, and 199/105
mmHg in the right arm. A bedside transthoracic echocardiogram is performed, showing a left
ventricular ejection fraction of 55-60% and valvular heart diasease. Once hemodynamically
stabilized, what is the definitive treatment plan for this patient?

Choices:
1. Medical management of blood pressure with close observation for the next 24-48 hours
2. Surgical repair of ascending aortic dissection with transcatheter aortic valve replacement
3. Surgical repair of ascending aortic dissection with surgical aortic valve replacement
4. Surgical repair of ascending aortic dissection with aortic valve repair via replacement of
damaged aortic sinus
Answer: 4 - Surgical repair of ascending aortic dissection with aortic valve repair via
replacement of damaged aortic sinus

Explanations:
The patient presents with classic signs and symptoms of aortic dissection, noncompliant
with antihypertensives, and daily tobacco use place the patient at high risk for dissection.
Descending (type B) aortic dissections can be observed with medical management of blood
pressure if the patient remains hemodynamically stable, ascending (type A) aortic
dissections require emergent surgical repair.
The patient has evidence of moderate to severe aortic regurgitation on transthoracic echo,
often the treatment modality of choice would be valve replacement. Since this patient is at
low risk it could be argued open surgical repair would be the treatment of choice.
The patient does not have a previous echocardiogram to compare with, however, AR
murmur was not present one month ago, and there is evidence of dissection extending into
the L ventricle. It can be assumed the patient has a structurally normal aortic valve and the
regurgitation is secondary to the dissection. This is one of the few instances where repair of
disrupted sinuses is the treatment of choice rather than valve repair.

Go to the next page if you knew the correct answer, or click the link image(s) below to further
research the concepts in this question (if desired).

Research Concepts:
Aortic Valve Disease

We update eBooks quarterly and Apps daily based on user feedback. Please tap flag to
report any questions that need improvement.
Question 826: A 32-year-old man with a history of asthma, heavy alcohol use is being
treated for Rickettsia conorii infection in the hospital. On the 3rd day of treatment, his clinical
course begins to worsen, and he develops new-onset atrial fibrillation, worsening liver function
studies, thrombocytopenia, and spreading purpuric rash over his body. He dies on the 5th day
after a progressively worsening clinical course. Which of the following patient factors most
likely worsened the patient's clinical course?

Choices:
1. Age
2. Sex
3. History of asthma
4. History of alcohol use
Answer: 2 - Sex
Explanations:
Alcohol use disorder is known to cause worsening progression of Boutonneuse fever.
More severe manifestations of the disease can occur in select patient populations.
These include advanced age, immunocompromise, chronic alcoholism, glucose-6-phosphate
dehydrogenase [G6PD] deficiency, prior prescription of an inappropriate antibiotic, and
delay of treatment.
The other listed factors for this patient do not affect the clinical course.

Go to the next page if you knew the correct answer, or click the link image(s) below to further
research the concepts in this question (if desired).

Research Concepts:
Boutonneuse Fever

We update eBooks quarterly and Apps daily based on user feedback. Please tap flag to
report any questions that need improvement.
Question 827: A 16-year-old boy is intubated for adult respiratory distress syndrome
(ARDS) secondary to pancreatitis. He has a right radial arterial line placed. The patient is
currently on peak inspiratory pressure (PIP) 30 cm H2O, positive end-expiratory pressure
(PEEP) 10, and FiO2 80%. He has a SpO2 of 93% on the ventilator. Which of the following best
describes the primary indication for keeping an arterial line for this patient?

Choices:
1. Measurement of cerebral perfusion pressure
2. Separate access for infusion of medications which are not compatible together
3. Accurate measurement of oxygenation index
4. As an access point for cardiac catheterization
Answer: 3 - Accurate measurement of oxygenation index
Explanations:
In this patient who has hypoxemic respiratory failure, the arterial line would allow for the
assessment of the oxygenation index (OI). This is a measurement of the severity of lung
disease and is calculated by mean airway pressure/ partial pressure of oxygen in arterial
blood x FiO2.
The partial pressure of arterial blood requires an indwelling arterial catheter, especially
because the trend in the OI is more important than a single value.
Oxygenation index of more than 8 implies acute respiratory distress syndrome (ARDS),
which moderate and more than 16 implies severe ARDS.
This patient with pancreatitis would require closer monitoring of blood pressure and blood
gases and not of cerebral perfusion pressure. Medications should never be administered in
the arterial line. The question does not give any description of cardiac dysfunction, and
hence cardiac catheterization is not needed.

Go to the next page if you knew the correct answer, or click the link image(s) below to further
research the concepts in this question (if desired).

Research Concepts:
Arterial Lines

We update eBooks quarterly and Apps daily based on user feedback. Please tap flag to
report any questions that need improvement.
Question 828: An 86-year-old man is brought to the emergency department after a fall
reported by EMS. No history could be obtained from the patient. There is a visible contusion on
his forehead. His gaze appears to deviate to the left on the exam. What is the best initial step in
the management of this patient?

Choices:
1. Levetiracetam
2. Haloperidol
3. Phenobarbital
4. Aspirin
Answer: 1 - Levetiracetam
Explanations:
The deviated gaze with the history of the fall indicates that the patient likely had a traumatic
head injury with a seizure and is possibly in status epilepticus.
Levetiracetam is typically the anti-epileptic of choice in a traumatic brain injury in the
elderly.
Levetiracetam has a lower side effect profile when compared to other anti-seizure
medications in the elderly.
The patient is likely having a seizure and needs treatment for the seizure, such as
levetiracetam. An antipsychotic such as haloperidol is not indicated at this time. Aspirin is
initially contraindicated as there is a known traumatic head injury and giving aspirin could
make an intracranial bleed worse.

Go to the next page if you knew the correct answer, or click the link image(s) below to further
research the concepts in this question (if desired).

Research Concepts:
Geriatric Head Injury

We update eBooks quarterly and Apps daily based on user feedback. Please tap flag to
report any questions that need improvement.
Question 829: A 17-year-old male patient is being managed in an intensive care unit
following a severe head injury. His bodyweight is 55 kilograms. The patient has now developed
persistent hyponatremia with serum sodium below 120 mEq/l. His urine output is also lower than
25ml per hour. The patient also has gained weight in the past few days, as reported by the
attending clinician. What is the next step in the fluid management of the patient for managing his
hyponatremia?

Choices:
1. Administration of 500 ml of 0.9% NaCl
2. Administration of 500 ml of 3% NaCl
3. Fluid restriction and frequent re-evaluation
4. Administration oral fludrocortisone
Answer: 3 - Fluid restriction and frequent re-evaluation
Explanations:
The patient has clinical signs and symptoms suggestive of a syndrome of inappropriate
antidiuretic hormone (SIADH) secretion.
The patient has hyponatremia, with decreased urine output, and features suggestive of water
retention reflected from his weight gain. This is all compatible with the diagnosis of
SIADH.
The pivotal step in the management of SIADH is the fluid restriction in the patient. Urine
output in a normal adult patient is more than 0.5 ml/kg/hour.
Hypertonic saline is indicated in the management of cerebral salt wasting (CSW) wherein
the patient will have hyponatremia associated with characteristic polyuria.

Go to the next page if you knew the correct answer, or click the link image(s) below to further
research the concepts in this question (if desired).

Research Concepts:
Fluid Management

We update eBooks quarterly and Apps daily based on user feedback. Please tap flag to
report any questions that need improvement.
Question 830: A 50-year-old man is admitted to the ICU for ventilatory support after
sustaining a 45% total body surface area (TBSA) burn to his right side. On the 7th day of his
admission, he develops a temperature of 102.1°F (38.9°C) and has a blood pressure of 110/80
mmHg; however, no signs of end-organ dysfunction are present. Laboratory findings reveal a
white blood cell (WBC) count of 10,900 cells/mm3 and lactate levels of 1.1 mmol/L. He has a
Foley catheter in place. Urinalysis reveals 4 WBC/HPF, traces of RBC, a few bacteria, and
positive nitrites, but no leukocyte esterase, glucose, protein, ketones, or casts. Which of the
following is the next best step in the management of this patient?

Choices:
1. Start vancomycin and piperacillin/tazobactam for suspected sepsis
2. Start ceftriaxone for catheter-associated urinary tract infection (CAUTI)
3. Initiate sulfamethoxazole/trimethoprim for CAUTI
4. Change the Foley and take a new sample for urinalysis
Answer: 4 - Change the Foley and take a new sample for urinalysis
Explanations:
Approximately 75% of urinary tract infections (UTIs) acquired in the hospital are linked to
the use of a urinary catheter.
The Infectious Disease Society of America (IDSA) definition of catheter-associated urinary
tract infection (CAUTI) requires all of the following components: 1,000 colony-forming
units per ml or more of one bacterial species in the culture grown, signs and symptoms of a
UTI (e.g., fever, costovertebral angle tenderness, and hypotension), despite proper
evaluation, there is no other reason for these symptoms, the patient has a Foley's catheter in
place or catheter removed within less than 48 hours.
This patient has a Foley in position and is not showing any signs of systemic infection. A
new urine sample should be taken after the Foley catheter is replaced.
Prolonged use of the urinary catheter is by far the most significant risk factor for having a
catheter-associated UTI (CAUTI). As a result, catheters should only be used when
absolutely necessary and must be removed as soon as possible.

Go to the next page if you knew the correct answer, or click the link image(s) below to further
research the concepts in this question (if desired).

Research Concepts:
Urinary Tract Infection

We update eBooks quarterly and Apps daily based on user feedback. Please tap flag to
report any questions that need improvement.
Question 831: A 65-year-old male patient is brought by his caretaker with the chief
complaints of headache, nausea, vomiting, blurring of vision, abdominal pain, and drowsiness.
He has been having headaches since the winter started, but the frequency and intensity have
gotten worse. He lives in his house alone since his wife passed away last year. On examination,
his blood pressure is 110/70 mmHg, pulse 91 beats per minute and regular, respiratory rate 22
breaths per minute, and his Glasgow coma scale (GCS) is 11/15. Fundoscopy reveals bilateral
disc edema. During his time in the emergency department, his GCS declines further and oxygen
saturation starts to drop. He is immediately intubated and put on mechanical ventilation. Which
of the following should be the target for mechanical ventilation in this case?

Choices:
1. Set PaCO2 between 45 to 50 mmHg
2. Set PaCO2 between 35 to 40 mmHg
3. Set PaCO2 between 30 to 35 mmHg
4. Set PaCO2 less than 30 mmHg
Answer: 3 - Set PaCO2 between 30 to 35 mmHg
Explanations:
This is a case of raised intracranial pressure secondary to carbon monoxide poisoning.
Hypocapnia can reduce cerebral blood flow by 4% for each mmHg change in PaCO2.
Experts recommend a PaCO2 between 30 to 35.
This decreased PaCO2 can reduce intracranial pressure by vasoconstriction of blood
vessels. Hyperventilation can decrease cerebral oxygenation and may induce brain ischemia
in patients with traumatic brain injury and acute stroke.
The effects of hyperventilation last about 48-72 hours. Once hyperventilation is used, the
PaCO2 levels should return to normal gradually. The rebound vasodilatation can be
detrimental.
Below a PaCO2 of 30, cerebral blood flow reduction is minor. Low PaCO2 can induce
hypoxemia in brain tissues and so is not recommended. While some advocate normocarbia,
a short period of hypocarbia does work and is beneficial.

Go to the next page if you knew the correct answer, or click the link image(s) below to further
research the concepts in this question (if desired).

Research Concepts:
Intracranial Hypertension

We update eBooks quarterly and Apps daily based on user feedback. Please tap flag to
report any questions that need improvement.
Question 832: A 65-year-old woman with a past medical history of heart transplant due to
dilated cardiomyopathy presents with progressive dyspnea on exertion, orthopnea, and weight
loss. The patient underwent the transplant 8 months ago. For the past two months, her symptoms
have worsened gradually. Physical examination reveals pedal edema, bilateral crepitations at the
lung bases, and the presence of a new heart murmur. Which of the following will most likely be
present in this patient on further evaluation?

Choices:
1. Bradycardia
2. Asystole
3. Atrial flutter
4. Ventricular tachycardia
Answer: 3 - Atrial flutter
Explanations:
This patient most likely has a heart transplant rejection. A sign of cardiac allograft rejection
can be an atrial arrhythmia.
Due to vagal denervation post heart transplant, allograft recipients tend to have elevated
heart rate at baseline.
Besides atrial arrhythmias, sudden death also has been reported following a heart transplant.
These patients usually have no chest pain because the donor's heart is not innervated by the
recipient's nerves.
During an acute rejection, the patient may be completely asymptomatic or maybe in florid
heart failure.

Go to the next page if you knew the correct answer, or click the link image(s) below to further
research the concepts in this question (if desired).

Research Concepts:
Heart Transplantation Rejection

We update eBooks quarterly and Apps daily based on user feedback. Please tap flag to
report any questions that need improvement.
Question 833: A 50-year-old man with a history of diabetes, hypertension, and recurrent
alcoholic pancreatitis presents to the emergency department due to worsening shortness of breath
and abdominal discomfort for the past week. Vital signs include a temperature of 98.6 F, blood
pressure 145/90 mmHg, heart rate 115/min, respiratory rate 18/min, and oxygen saturation 92%
on 3 L/min oxygen by nasal cannula. Physical examination shows a dry oral mucosa, no
murmurs on the cardiac exam, absent breath sounds at the left hemithorax, and mild generalized
abdominal tenderness. A chest x-ray shows a large left pleural effusion occupying more than
90% of the left hemithorax. Which of the following is the next best step in the management of
this patient?

Choices:
1. Paracentesis
2. Thoracentesis
3. Magnetic resonance cholangiopancreatography
4. Echocardiogram
Answer: 2 - Thoracentesis
Explanations:
Thoracentesis is a procedure to evacuate pleural fluids such as that seen in pancreatic
pleural effusion and allows for further analysis of the fluid content.
A pleural fluid sample is tested for amylase, protein, albumin, glucose, lactate
dehydrogenase, cytology, gram stain, culture, cell count and differential, and adenosine
deaminase to help determine the etiology of the fluid accumulation.
Patients with pancreatic pleural effusion can present with pulmonary symptoms such as
dyspnea, cough, wheezing, or pleuritic chest pain.
Paracentesis is a procedure to evacuate fluids from the peritoneal cavity and allows for
further analysis of the fluid content.

Go to the next page if you knew the correct answer, or click the link image(s) below to further
research the concepts in this question (if desired).

Research Concepts:
Pancreatic Fistula

We update eBooks quarterly and Apps daily based on user feedback. Please tap flag to
report any questions that need improvement.
Question 834: A 65-year-old male patient is brought to the emergency department with
complaints of sudden onset of headache and inability to speak for three hours. On his way to the
hospital, he has vomited twice. His previous record reveals that he is suffering from diabetes and
hypertension. He has a metallic valve in place because he got mitral valve replacement done 6
years ago and since then he is on anticoagulants. Recently he consulted his family physician for a
fungal infection over his skin for which he received a course of antifungals. On examination,
blood pressure 180/110 mmHg, pulse 92/min regular, maintaining oxygen saturation on room
air, and respiratory rate of 21/min. Nuchal rigidity is positive. Fundoscopy reveals bilateral
papilledema. What needs to be administered to counteract the underlying pathology?

Choices:
1. Tenecteplase
2. Low molecular weight heparin
3. Vitamin K
4. Protamine sulfate
Answer: 3 - Vitamin K
Explanations:
This is a case of intracerebral hemorrhage leading to raised intracranial pressure.
The patient is on warfarin, and he has been given fluconazole which is an inhibitor of
cytochrome P450 C29, an enzyme that is responsible for the metabolism of warfarin. This
drug interaction leads to warfarin over-anticoagulation.
In the cases of raised intracranial pressure, the eyes should always be examined because the
pupils may become dilated and unresponsive to light.
The patient may also develop the Cushing reflex with elevations in blood pressure and
bradycardia.

Go to the next page if you knew the correct answer, or click the link image(s) below to further
research the concepts in this question (if desired).

Research Concepts:
Intracranial Hypertension

We update eBooks quarterly and Apps daily based on user feedback. Please tap flag to
report any questions that need improvement.
Question 835: A 65-year-old male patient with a history of emphysema presents to the
hospital complaining of sudden onset dyspnea and pleuritic type chest pain on the left side. He is
aa smoker of 25/day. On examination, he looks very restless, pale and sweaty. His blood
pressure is 140/85 mmHg, the pulse is 120/min regular, the respiratory rate is 32/min, and
oxygen saturation on room air is 88%. He has reduced chest expansion, hyper resonant note, and
decreased air entry on the left side. He is commenced on oxygen supplementation and analgesia.
What is the next best step in management?

Choices:
1. Urgent surgical intervention
2. Urgent CT scan of the chest
3. Chest drain insertion
4. Chest aspiration
Answer: 3 - Chest drain insertion
Explanations:
There are two types of pneumothorax: traumatic and atraumatic. The two subtypes of
atraumatic pneumothorax are primary and secondary. A primary spontaneous pneumothorax
(PSP) occurs automatically without a known eliciting event, while a secondary spontaneous
pneumothorax (SSP) occurs subsequent to an underlying pulmonary disease.
On examination, the following findings are noted respiratory discomfort, increased
respiratory rate, asymmetrical lung expansion, decreased tactile fremitus, hyper resonant
percussion note, decreased intensity of breath sounds or absent breath sounds.
In secondary spontaneous pneumothorax, if size/depth of pneumothorax is less than 1 cm
and no dyspnea then the patient is admitted, high flow oxygen is given and observation is
done for 24 hours.
If size/depth is between 1-2 cm, needle aspiration is done, then the residual size of
pneumothorax is seen, if the depth after the needle aspiration is less than 1 cm management
is done with oxygen inhalation and observation and in case of more than 2cm, tube
thoracostomy is done. In case of depth more than 2 cm or breathlessness, tube thoracostomy
is done.

Go to the next page if you knew the correct answer, or click the link image(s) below to further
research the concepts in this question (if desired).

Research Concepts:
Pneumothorax

We update eBooks quarterly and Apps daily based on user feedback. Please tap flag to
report any questions that need improvement.
Question 836: A 74-year-old woman was brought to the emergency department due to
constant chest pain accompanied by shortness of breath. She reported that the pain decreases in
intensity when she leans down. The patient has a history of hypercholesterolemia and diabetes
mellitus type 2. Her vital signs included temperature 38 C (100.4 F), blood pressure 110/75
mmHg, heart rate 95 beats/min, and respiratory rate 20/min. Cardiac auscultation revealed
friction rub, with no murmurs or added sounds. The patient was admitted and started on pain
medications. On day 2, her blood pressure is 95/64 mmHg. Her neck veins are elevated, and
cardiac auscultation reveals muffled S1 and S2. What findings are most likely to be found in this
patient's electrocardiogram?

Choices:
1. QRS complexes alternating in height
2. Prolonged QT interval
3. ST-segment elevation in leads V3-V6
4. Widened QRS complexes and loss of P wave
Answer: 1 - QRS complexes alternating in height
Explanations:
The patient was admitted with pericarditis. Pericardial effusion is a well-known
complication of pericarditis.
In patients with large pericardial effusion, ECG may show electrical alternans.
Electrical alternans refers to consecutive QRS complexes of varying amplitudes
corresponding to a swinging motion of the heart within the surrounding fluid.
ECG may show diffuse ST-segment elevation in patients with pericarditis. ST-segment
elevation in specific leads is a sign of localized myocardial injury.

Go to the next page if you knew the correct answer, or click the link image(s) below to further
research the concepts in this question (if desired).

Research Concepts:
Pericardial Effusion

We update eBooks quarterly and Apps daily based on user feedback. Please tap flag to
report any questions that need improvement.
Question 837: A 58-year-old male patient is brought to the hospital with altered mental
status after being found on the floor of his bathroom by his adult daughter. He lives alone, and
the last time she spoke to him was two days ago. He does not have any medical conditions that
she knows of and does not take any medications. She reports he drinks alcohol regularly. He was
well during their last conversation. On exam, his temperature is 38.5 C (101.3 F), the heart rate is
110/min regular, the blood pressure is 180/90 mmHg, the respiratory rate is 16, and O2
saturation is 98%, and he is agitated and anxious. EKG shows sinus tachycardia. Complete blood
count, complete metabolic panel, and urinalysis are all within normal limits. CT scan of the head,
brain, chest, abdomen, and pelvis is without pathology. What is the next most appropriate step in
the management of this patient?

Choices:
1. Administer IV crystalloid fluids at 20 mL/kg and start broad antibiotic coverage
2. Obtain TSH and administer beta-blockers after oral rehydration
3. Administer benzodiazepines for symptom management and seizure prophylaxis
4. Administer phenoxybenzamine for noncompetitive alpha-receptor antagonism
Answer: 3 - Administer benzodiazepines for symptom management and seizure prophylaxis
Explanations:
Intoxication and withdrawal from prescribed and recreationally consumed substances are
common causes of tachycardia.
Benzodiazepines are the first-line treatment for both alcohol withdrawal and for agitation
management in amphetamine intoxication.
Differentiating between sepsis and intoxication/withdrawal is a critical skill for clinicians.
Blood pressure, mental status, lab work, and infectious source can be important clues.
The early differential diagnoses for patients with sympathomimetic toxidrome are ingestion,
withdrawal, infection, pulmonary embolism, and hyperthyroidism.

Go to the next page if you knew the correct answer, or click the link image(s) below to further
research the concepts in this question (if desired).

Research Concepts:
Sinus Tachycardia

We update eBooks quarterly and Apps daily based on user feedback. Please tap flag to
report any questions that need improvement.
Question 838: A 36-year-old female with no known medical history is admitted to the ICU
with acute respiratory distress syndrome (ARDS) after a near-drowning event. The patient is
receiving volume cycled mechanical ventilation. On rounds, you note that the alarm for high
peak inspiratory pressure is going off. An inspiratory hold maneuver reveals elevated plateau
pressures as well. Which of the following complications is most likely to be seen in this patient?

Choices:
1. Bronchospasm
2. Barotrauma
3. Increased venous return
4. Increased mucus
Answer: 2 - Barotrauma
Explanations:
Patients with ARDS are at increased risk of developing barotrauma. Barotrauma describes
the manifestations of extra-alveolar air during mechanical ventilation.
Patients with ARDS have decreased compliance in the lung tissue. The decreased
compliance results in a significant elevation in both peak inspiratory pressures as well as
plateau pressures.
Plateau pressure is the pressure applied to the alveoli and other small airways during
ventilation. Elevation of plateau pressures may lead to rupture of alveoli units and result in
barotrauma.
The clinical presentation varies from absent symptoms with the subtle radiographic findings
of pulmonary interstitial emphysema to respiratory or cardiac distress due to a tension
pneumothorax. Other manifestations include pneumopericardium, pneumomediastinum,
pneumoperitoneum, and subcutaneous emphysema. An increase in alveolar pressure
mediates lung injury associated with barotrauma.

Go to the next page if you knew the correct answer, or click the link image(s) below to further
research the concepts in this question (if desired).

Research Concepts:
Barotrauma And Mechanical Ventilation

We update eBooks quarterly and Apps daily based on user feedback. Please tap flag to
report any questions that need improvement.
Question 839: A 72-year-old man with a past medical history of alcoholic liver disease,
diabetes, hypertension, chronic renal failure, and smoking presents to the hospital with
abdominal pain, grossly bloody diarrhea, hematemesis, and altered mental status. The patient's
history reveals an intake of wild mushrooms on the day before presentation. The patient is
unresponsive to standard medical and supportive therapy for his condition. Which of the
following is the strongest indication for the use of molecular adsorbent recirculating system in
this patient?

Choices:
1. Alcoholic liver disease
2. Amanita phalloides intoxication
3. Chronic renal failure
4. Age > 65 years
Answer: 2 - Amanita phalloides intoxication
Explanations:
Amanita phalloides, also known as death cap, is a deadly poisonous mushroom. It is
frequently mistaken for edible mushrooms and causes acute hepatic failure due to the
amatoxins.
The amanitin toxin is heat stable, remaining toxic whether eaten raw or cooked. The
mechanism of action of amatoxin is by inhibiting RNA polymerase, causing disruption of
transcription of mRNA. As a result, hepatocytes cannot synthesize key protein coding
genes, leading to the disintegration of nucleoli and pathologically centrilobular hepatic
necrosis.
The course of Amanita toxicity has three phases: The first stage does not begin until six to
12 hours after ingestion; often, foragers comment on how good the food made from
Amanita species tastes, and there are no signs of a problem for at least 6 hours. After this
silent phase, it is followed by the onset of nausea, abdominal cramps, profuse watery
diarrhea, and signs of dehydration. A physical exam may reveal dry mucosal membranes
and tachycardia, and given sufficient dehydration, hypotension. After the GI phase, the
second stage appears where the patient appears to recover transiently, and GI symptoms
resolve, but ongoing liver damage continues. This stage may last two to three days and is
characterized by rising liver function transaminase, bilirubin, the development of
coagulopathy, and eventually hepatic encephalopathy. In the third stage, both liver and renal
function become compromised. Hepatorenal syndrome and hepatic encephalopathy may
occur rapidly after laboratory signs of liver injury, and death can occur in three to seven
days.
Molecular adsorbent recirculating system has been used for the treatment of liver failure
from Amanita phalloides poisoning with positive results.

Go to the next page if you knew the correct answer, or click the link image(s) below to further
research the concepts in this question (if desired).

Research Concepts:
Molecular Absorbent Recirculating System

We update eBooks quarterly and Apps daily based on user feedback. Please tap flag to
report any questions that need improvement.
Question 840: A 68-year-old male patient was referred to the emergency department due to
severe bilateral lower limb coldness and pallor. His physical examination revealed the absence of
bilateral lower limb pulses from femoral to distal. Both limbs were severely cold. His vital signs
were blood pressure: 68/40 mmHg, pulse rate: 110 beats per minute, respiratory rate: 19/min,
and temperature: normal. The CT-angiography disclosed infrarenal aortic thrombosis. He
underwent the standard surgical procedure. Four hours later, his vital signs were blood pressure:
80/60 mmHg, pulse rate: 120 bpm, respiratory rate: 18/min, and normal temperature. His total
urine output for the last 4 hours was 30 mL. What is the best next step in management?

Choices:
1. Perform CT-angiography w/o contrast
2. Perform CT-angiography w/ contrast
3. Give a bolus of isotonic fluid
4. Start furosemide to increase urine output
Answer: 3 - Give a bolus of isotonic fluid
Explanations:
This patient most likely has oliguria secondary to hypovolemia.
Vascular surgery is a high-risk procedure and high volume blood loss should be anticipated.
Normal urine output is around 1-2 mL/kg/hour. The patient should be given adequate
isotonic fluids. If volume replacement does not remedy the situation, the other studies may
be of help.
Obtaining CT-angiography with contrast without prior adequate hydration might result in
renal shutdown and should be prevented. Administration of furosemide in lower limits of
blood pressure might result in aggravating hypotension.

Go to the next page if you knew the correct answer, or click the link image(s) below to further
research the concepts in this question (if desired).

Research Concepts:
Peripheral Vascular Bypass

We update eBooks quarterly and Apps daily based on user feedback. Please tap flag to
report any questions that need improvement.
Question 841: An 88-year-old man with mild dementia and metastatic pancreatic cancer is
admitted to the medical ward from his nursing home for abdominal pain and fever. Though
disoriented, he is pleasant and cooperative. He is found to have a urinary tract infection and
started on appropriate antibiotics and intravenous (IV) fluids. He shows clinical improvement for
his presenting complaints but gradually becomes hypoxic, which is treated with supplemental
oxygen. By the fourth day of hospitalization, he becomes acutely short of breath and confused.
Chest x-ray and arterial blood gas are ordered. His blood gas shows an elevated carbon dioxide
level. The chest x-ray shows pulmonary edema and volume overload. Unintentionally, his IV
fluid orders from admission had been continued for the entire hospital course to date. His
advanced directives preclude intubation and cardiopulmonary resuscitation (CPR). Which of the
following is the most appropriate management strategy for this patient?

Choices:
1. Reverse his code status and intubate
2. Comfort measures
3. Obtain vascular access and initiate extracorporeal carbon dioxide removal
4. Non-invasive ventilation and administer diuretics
Answer: 4 - Non-invasive ventilation and administer diuretics
Explanations:
Acute hypercarbia developing after several days of hospitalization may be iatrogenic. In this
case, the initial picture suggested sepsis. The patient appeared to be recovering, but then he
developed respiratory failure. As the chest x-ray shows pulmonary edema, diuretics should
be given to address the underlying cause.
The initial treatment for hypercarbic respiratory failure should be non-invasive ventilator
(NIV) support if no contraindications exist. In patients without treatment limitations, care
can be escalated to intubation and mechanical ventilation if NIV fails.
In a setting of volume overload, hypoxic respiratory failure usually develops before
hypercarbia. If pulmonary edema worsens to the point of impaired gas exchange and
respiratory distress, hypercarbia can result.
Patient autonomy should be respected, even though the hypercarbia is likely reversible.
Extracorporeal carbon dioxide removal is not widely available at hospitals in the United
States.

Go to the next page if you knew the correct answer, or click the link image(s) below to further
research the concepts in this question (if desired).

Research Concepts:
Hypercarbia

We update eBooks quarterly and Apps daily based on user feedback. Please tap flag to
report any questions that need improvement.
Question 842: An 82-year-old female with a history of heart failure with reduced ejection
fraction is admitted for evaluation of lightheadedness. The patient reports that she is feeling
lightheaded and generalized weak for the last 1 week. Lightheadedness is more pronounced
when the patient stands up from a sitting position. The patient feels thirsty and reports having a
weight loss of 4 pounds (2 kg) over the last 1 week. No nausea, vomiting, spinning sensation of
head, changes in hearing, tinnitus, or ear discharge is reported. The patient denied syncope,
palpitation, chest pain, and shortness of breath. Vitals include blood pressure 110/80 mmHg,
pulse 100 bpm, respiratory rate 20/min, pulse ox 93% on room air, temperature 97 °F.
Orthostatic vitals are positive. The patient appears dry with no leg edema and oral mucosa is dry.
The chest is clear to auscultation. S1 plus S2 plus no additional sound. No bilateral lower
extremity edema. No focal motor or sensory deficit. The patient is taking aspirin, carvedilol,
atorvastatin, lisinopril, furosemide. EKG shows normal sinus rhythm and no acute ischemia.
Echocardiogram performed showed an ejection fraction of 37% with no new changes compared
to the prior study performed within the last 6 months. Basic metabolic profiles show serum
creatinine of 1.7 mg/dL and basic urea nitrogen of 40. Baseline serum creatinine is 1.1 mg/dL.
Serum potassium was 2.9 milliequivalents per liter. What is the most common drug contributing
to the current clinical condition?

Choices:
1. Carvedilol
2. Furosemide
3. Atorvastatin
4. Lisinopril
Answer: 2 - Furosemide
Explanations:
The patient is on carvedilol, and no recent changes in medications have been made.
Although carvedilol like any other antihypertensive medication can lead to hypotension, but
for the patient in clinical scenario no recent changes in dose of carvedilol are made so it is
less likely for carvedilol to contribute to orthostasis and kidney injury.
Loop diuretics such as furosemide, bumetanide, torsemide, and ethacrynic acid tend to
cause hypovolemia and free water loss and can cause kidney injury due to dehydration. In
addition, the elderly patient population is more predisposed to these side effects and caution
should be practiced when prescribing diuretics along with close monitoring to observe for
development of any adverse effect is recommended. In the current patient, furosemide is
most likely the agent causing excessive diuresis leading to dehydration and acute kidney
injury. Due to hypovolemia and electrolyte loss, the patient is experiencing generalized
weakness and postural hypotension (orthostasis). The primary adverse effects of furosemide
include hypokalemia, hyponatremia, hypocalcemia, hypomagnesemia, hyperglycemia,
hyperuricemia, azotemia, and metabolic alkalosis. Monitor blood pressure, daily weight,
urine output, serum electrolytes, renal function (blood urea nitrogen and serum creatinine),
serum uric acid, and blood glucose levels in patients taking furosemide.
Statin therapy can lead to myalgias, muscle spasms, myopathy, arthralgias, fatigue,
hyperglycemia, abnormal hepatic function test. However, it is less likely to cause orthostatic
hypotension and kidney injury.
Lisinopril can cause dizziness, hypotension, increased serum creatinine, hyperkalemia, and
syncope. In the patient under discussion, furosemide is the most likely cause of orthostatic
hypotension and dehydration as evident from recent weight loss, elevated BUN, dry oral
mucosa, and other signs of dehydration. Lisinopril should be kept on hold till the kidney
injury is resolved, and then lisinopril should be resumed gradually at a lower dose and the
dose should be titrated according to the response and tolerance of the patient.

Go to the next page if you knew the correct answer, or click the link image(s) below to further
research the concepts in this question (if desired).

Research Concepts:
Furosemide

We update eBooks quarterly and Apps daily based on user feedback. Please tap flag to
report any questions that need improvement.
Question 843: A 47-year-old man is brought to the hospital after police found him
unconscious near a local bar. The patient regains consciousness shortly after arrival but appears
confused and disoriented. He is unkempt and smells of alcohol. Physical examination reveals
ataxic gait and difficulty moving his eyes horizontally. Horizontal nystagmus is noted. Which of
the following will occur if intravenous thiamine is not administered promptly?

Choices:
1. Lactic acidosis
2. Permanent memory loss
3. Paralysis
4. Liver failure
Answer: 2 - Permanent memory loss
Explanations:
Wernicke encephalopathy occurs early in the disease course and characteristically
demonstrates non-inflammatory brain lesions. It can present with ataxia, ophthalmoplegia,
punctate hemorrhages in the brain, altered mental status, and balance abnormalities.
If left untreated, Wernicke encephalopathy can eventually evolve to include Korsakoff
psychosis. Individuals will now present with delirium and permanent memory loss.
Treatment should commence emergently to prevent disease progression and irreversible
brain damage.
Empirically, WKS treatment is with at least 500 mg thiamine hydrochloride per 100 mL of
normal saline given over 30 minutes. This regimen should be repeated three times a day for
2 to 3 days. Thiamine administration should be before or alongside glucose.
WKS affects the central nervous system, which involves the brain and spinal cord. Its most
common cause is alcohol misuse, seen alongside poor nutrition, but can present in
individuals who are at risk for thiamin deficiency.

Go to the next page if you knew the correct answer, or click the link image(s) below to further
research the concepts in this question (if desired).

Research Concepts:
Vitamin B1 (Thiamine)

We update eBooks quarterly and Apps daily based on user feedback. Please tap flag to
report any questions that need improvement.
Question 844: A 60-year-old female with a history of hypertension, diabetes, chronic
obstructive pulmonary disease (COPD), and end-stage renal disease on hemodialysis presents for
a routine visit. She has no acute symptoms. She is concerned that one of her sisters died of a
myocardial infarction a few months ago and is worried about her risk of the same thing
happening to her. Her medications include metoprolol 12.5 mg twice a day insulin glargine 30
units at bedtime, and 15 units of insulin lispro with meals. Her vitals are all normal. She brings
with her an echocardiogram performed a month ago after one of her dialysis sessions that reveals
a region of wall motion abnormality in the left ventricle. What is the most likely explanation of
her echocardiogram finding?

Choices:
1. History of diabetes
2. History of hypertension
3. History of end-stage renal disease requiring hemodialysis
4. History of COPD
Answer: 3 - History of end-stage renal disease requiring hemodialysis
Explanations:
Diseased or ischemic myocardium, when reperfused, leads to stunning of the myocardium,
a phenomenon that is transient. Myocardium eventually regains its full contractile function.
Along with reperfusion, a few other causes of stunned myocardium include stress
cardiomyopathy (Takotsubo cardiomyopathy), neurogenic stunned myocardium, and LV
abnormalities associated with dialysis.
Patients who receive hemodialysis have an associated cardiovascular mortality and
morbidity burden. They are more prone to experiencing demand myocardial ischemia and
myocardial stunning.
In patients with end-stage renal disease, dialysis itself can induce left ventricular
abnormalities, a common feature of stunned myocardium.

Go to the next page if you knew the correct answer, or click the link image(s) below to further
research the concepts in this question (if desired).

Research Concepts:
Stunned Myocardium

We update eBooks quarterly and Apps daily based on user feedback. Please tap flag to
report any questions that need improvement.
Question 845: An 84-year-old woman presents with acute decompensated heart failure.
After an inadequate response to diuresis and systolic blood pressure values persisting over
150/98 mmHg, she is started on an infusion of a vasodilator that is known to cause lactic acidosis
if given for prolonged periods or at high doses. What is the other FDA-approved indication for
this drug?

Choices:
1. Hypertension during acute ischemic stroke
2. Hypertensive crisis
3. Arteriovenous shunting
4. Aortic coarctation
Answer: 2 - Hypertensive crisis
Explanations:
Nitroprusside is a potent vasodilator of both arteries and veins that acts quickly to lower
systemic vascular resistance. It must be used with extreme caution in patients with known
inadequate cerebral or coronary circulation.
Nitroprusside has labeled indications in the treatment of acute decompensated heart failure,
management of hypertensive crises, and controlled hypotension during surgery.
Off-label uses of nitroprusside include the management of hypertension in the setting of
acute ischemic stroke and medical management of acute mitral regurgitation in preparation
for surgical intervention.
Nitroprusside is contraindicated for the treatment of compensatory hypertension (aortic
coarctation or AV shunting).

Go to the next page if you knew the correct answer, or click the link image(s) below to further
research the concepts in this question (if desired).

Research Concepts:
Sodium Nitroprusside

We update eBooks quarterly and Apps daily based on user feedback. Please tap flag to
report any questions that need improvement.
Question 846: An otherwise healthy 16-year-old boy from Honduras presents to the
hospital with new-onset partial seizures. In terms of medical history, he reached his
developmental milestones appropriately and did well throughout the school. He now works as a
mechanic after having immigrated to the United States last year. He does not drink or smoke, and
he does not use any illicit or recreational drugs. His physical exam is non-focal, but brain MRI
reveals numerous ring-enhancing lesions. What is the best initial therapy for this patient?

Choices:
1. Ceftriaxone
2. Acyclovir
3. Albendazole and dexamethasone
4. Phenobarbital
Answer: 3 - Albendazole and dexamethasone
Explanations:
The likely diagnosis in this case is neurocysticercosis, the result of accidental ingestion of
eggs of Taenia solium (i.e., pork tapeworm). It is more common in Central and South
America due to ingestion of uncooked egg-infected pork.
Seizures are the most common presentation of this infectious disease, occurring in up to
70% of affected patients. In fact, in developing countries, it is the most common parasitic
disease of the nervous system, representing the chief etiology of acquired epilepsy in
endemic regions. Seizures can be generalized or partial.
Other common presentations include headache, stroke, and neuropsychiatric dysfunction.
Additionally, 10-30% of affected patients develop communicating hydrocephalus secondary
to inflammation and fibrosis of arachnoid villi and/or the meninges. Noncommunicating
hydrocephalus may also transpire in the setting of intraventricular cysts. Brain imaging
often reveals ring-enhancing lesions in multiple, perhaps bilateral locations. Single lesions
are also possible.
Treatments comprises anti-helminthic medications (e.g., albendazole) and steroids (e.g.,
dexamethasone) to suppress the inflammatory response induced by destruction of the live
cysticerci in the brain. In conclusion, specific regimens depend on the life cycle of the
parasite. First-line anti-seizure medications like phenytoin and carbamazepine are helpful in
controlling epileptic activity, and if the infection results in epilepsy in the chronic setting,
anticonvulsant therapy is directed in the same way as any other epilepsy syndrome.

Go to the next page if you knew the correct answer, or click the link image(s) below to further
research the concepts in this question (if desired).

Research Concepts:
Tapeworm

We update eBooks quarterly and Apps daily based on user feedback. Please tap flag to
report any questions that need improvement.
Question 847: A 17-year-old girl presents to the emergency department with complaints of
headache, body aches, fever, nausea, and vomiting for the past day. She is a known case of acute
lymphoblastic leukemia who is receiving maintenance chemotherapy. She is admitted to the
intensive care unit from the emergency department. Her temperature is 39.3 C, heart rate is 188
beats/min, respiratory rate is 56/min, blood pressure is 67/21 mmHg, and arterial oxygen
saturation is 93% on room air. She received a total of 80 mL/kg of crystalloid fluid resuscitation
in the emergency department and is now receiving another 20 mL/kg of intravenous fluid. Which
of the following is the next best step in the management of this patient?

Choices:
1. Dobutamine is the vasopressor of choice at this point in the therapy
2. Intravenous antibiotic therapy should be initiated within 1 hour of presentation
3. Lactate levels are late indicator so they should be checked at this point
4. Lumbar puncture should be performed before administration of antibiotics
Answer: 2 - Intravenous antibiotic therapy should be initiated within 1 hour of presentation
Explanations:
Initiation of broad-spectrum antibiotics within 1 hour of presentation has been consistently
associated with a reduction in mortality from sepsis syndromes.
The vasopressor of choice would depend on physical examination and assessment of warm
or vasodilatory shock versus cold or vasoconstrictive shock. Warm shock is characterized
by flash cap refill, bounding pulses, and warm extremities. Cold shock is characterized by
prolonged capillary refill, thready pulses, and cool extremities.
Lactate levels are an early indicator of inadequate oxygen delivery to end organs resulting
in anaerobic respiration and subsequent metabolic acidosis.
If a septic patient is hemodynamically stable, it may be reasonable to wait to obtain cultures
prior to the initiation of antimicrobial therapy. However, if a patient has progressed along
the continuum, that is from severe sepsis versus septic shock, practitioners should not wait
for diagnostic evaluations to be completed prior to administering life-saving therapeutics,
such as broad-spectrum antimicrobials, immediately.

Go to the next page if you knew the correct answer, or click the link image(s) below to further
research the concepts in this question (if desired).

Research Concepts:
Septic Shock

We update eBooks quarterly and Apps daily based on user feedback. Please tap flag to
report any questions that need improvement.
Question 848: A 65-year-old male presents to the emergency department with complaints
of chest pain. He has a history of diabetes, hypertension, and obesity. He developed the
symptoms 30 minutes ago. His 12-lead ECG reveals the presence of ST elevation in leads 2,3
and aVF. He is planned for thrombolysis and with alteplase. The patient is attached to a cardiac
monitor, and intravenous alteplase is started. The patient suddenly becomes unresponsive, and
his cardiac monitor shows the presence of disorganized, high-frequency ECG tracings without a
discernable p wave, QRS complex, or a T wave. CPR is instituted, and a defibrillator is attached.
The defibrillator is being charged to deliver a shock. Which patient factor would require the
defibrillator to be charged at a higher energy setting?

Choices:
1. Age of patient
2. History of obesity
3. ECG findings on arrival
4. Treatment with alteplase
Answer: 2 - History of obesity
Explanations:
This patient has presented to the emergency department with complaints of chest pain. His
ECG is consistent with the development of inferior ST-elevation myocardial infarction.
Unfortunately, the patient crashes during treatment and requires CPR. The rhythm strip is
consistent with the development of ventricular fibrillation, which requires prompt
defibrillation.
The energy select on the defibrillator is 120-200 joules biphasic and 360 joules monophasic.
The lowest energy settings to achieve a defibrillatory threshold should be used. The
presence of obesity and a thick chest wall required a selection of higher energy settings.
Individuals with thick chest walls have higher resistance and therefore, will require higher
energy selection to achieve defibrillatory threshold. Shaving chest hairs and the application
of conductive gel to skin surface increases conductivity.
Other factors that affect the selection of energy levels during defibrillation are the duration
of cardiac arrest, paddle size, previous defibrillatory shocks, and contact pressure. The aim
is to achieve adequate defibrillation using the lowest energy and thus prevent myocardial
injury.

Go to the next page if you knew the correct answer, or click the link image(s) below to further
research the concepts in this question (if desired).

Research Concepts:
Ventricular Fibrillation

We update eBooks quarterly and Apps daily based on user feedback. Please tap flag to
report any questions that need improvement.
Question 849: A 28-year-old adventure traveler presents to the hospital with headache,
back pain, fever, and jaundice after returning from backpacking through multiple countries.
Based on his presentation, yellow fever is considered. Which of the following pieces of historical
data makes the diagnosis of yellow least likely?

Choices:
1. Travel to a monkey rescue unit in sub-Saharan Africa
2. Vaccination to prevent yellow fever 10-years prior to presentation
3. Nosebleeds and black emesis
4. Self-limited course with a full recovery
Answer: 2 - Vaccination to prevent yellow fever 10-years prior to presentation
Explanations:
Since there is no effective therapy for yellow fever, the focus should be on prevention by
avoiding mosquito bites and vaccination to travelers to endemic areas. Vaccination confers
lifelong immunity within 30 days in 99% of patients.
Monkeys may be a reservoir for yellow fever and mosquitos serve as the vector that
transmits the disease to humans from other primates.
Patients with yellow fever may develop hepatic dysfunction leading to coagulopathy and
bleeding from the nose and mucous membranes.
Most cases of yellow fever are subclinical and have an excellent prognosis. About 15% of
symptomatic patients will develop severe disease.

Go to the next page if you knew the correct answer, or click the link image(s) below to further
research the concepts in this question (if desired).

Research Concepts:
Yellow Fever

We update eBooks quarterly and Apps daily based on user feedback. Please tap flag to
report any questions that need improvement.
Question 850: A 19-year-old male military recruit is lagging behind his squadmates during
a 12-mile ruck march on a hot and humid day. It is the culminating event for a day-long training
exercise. He is brought to the side of the march course by his squad leader, who forces him to
drink a 1 L canteen of water, his 5th canteen since starting the march. He continues on for an
additional 3 miles and then collapses. He is brought to the medical tent, where he is found to be
profoundly altered and combative. He is flushed, tachycardic, hypertensive, and normothermic,
with an otherwise unrevealing physical exam. Initial lab evaluation reveals sodium 128 mEq/L,
glucose 89 mg/dL, and otherwise normal electrolytes, LFTs, and blood counts. Which of the
following is the next best step in the management of this patient?

Choices:
1. Administration of 1 L IV normal saline now
2. Administration of 100 ml IV 3% saline now
3. Active cooling and reassessment of core temperature
4. Administration of oral sodium solution of 4 broth bouillon cubes in 125 ml of water now
Answer: 2 - Administration of 100 ml IV 3% saline now
Explanations:
Exercise-associated hyponatremia (EAH) must be correctly differentiated from other
exertional illnesses (such as heat exhaustion, heatstroke, and dehydration) prior to
treatment, as their treatments are often contradictory. Treatment is dictated by the presence
or absence of neurologic symptoms. The goal of treatment is the correction of serum
sodium and improvement in symptoms.
If a patient does not demonstrate neurologic symptoms, EAH is considered mild, and the
patient is eligible for oral fluid restriction. Patients restrict their total fluid intake to induce
osmotic ADH suppression and, therefore, free water excretion. Patients may also be given
concentrated oral sodium repletion. However, this may be poorly tolerated due to taste or
nausea and vomiting—proposed regimens include 100 mL of hypertonic saline, 3-4 broth
bouillon cubes in 125 mL of water, or a serving of salted pretzels. Oral hypertonic saline is
as effective as IV hypertonic saline in mild EAH, with one study demonstrating faster time
to recovery and shorter duration of hospital stay for athletes who receive oral treatment.
This patient is NOT eligible for oral treatment given his mental status.
If a patient demonstrates neurologic symptoms (altered mental status, seizure, coma), EAH
is considered severe. The patient should receive a bolus of 100 ml of IV hypertonic saline.
This can be repeated every 10 minutes for a total of 3 doses. Up to 600-1000 mL over 1
hour of hypertonic saline has been given without complication in case studies, though larger
volumes of sodium repletion increase the risk for rapid sodium correction and osmotic
demyelination. This patient MUST be treated with IV hypertonic saline given his mental
status.
IV isotonic fluids of any type or volume are not recommended in EAH as this will worsen
hyponatremia.

Go to the next page if you knew the correct answer, or click the link image(s) below to further
research the concepts in this question (if desired).

Research Concepts:
Exercise-Associated Hyponatremia

We update eBooks quarterly and Apps daily based on user feedback. Please tap flag to
report any questions that need improvement.
Question 851: A 42-year-old homeless woman is brought to the hospital after being found
unresponsive on the side of the road. She appears disheveled and unkempt. She has no known
medical records. Vital signs reveal a temperature of 39 C, blood pressure 108/68 mmHg, pulse
98/min, and respiratory rate 23/min. GCS is 12/15. Physical examination is significant for
multiple vesicles in the genital area, some of which appear to be crusted. Bilateral inguinal
lymphadenopathy is also seen. Lumbar puncture reveals white blood cells 250/microL (normal
8/microL) with lymphocytic predominance, protein 320 mg/dL (normal 15-45 mg/dL), and
glucose 50 mg/dL (normal 50-80 mg/dL). Which of the following is the most common side
effect of the best initial therapy for this patient?

Choices:
1. Elevated transaminases
2. Elevated creatinine
3. Leukopenia
4. Seizures
Answer: 2 - Elevated creatinine
Explanations:
The clinical vignette is most consistent with a diagnosis of herpes simplex encephalitis.
While HSV-1 often affects the perioral region and can be known to cause genital lesions,
HSV-2 is more commonly the consideration when patients present with genital lesions.
Aseptic meningitis occurs in 36% of women and 13% of men leading to the hospitalization
of a percentage of inflicted persons. During the prodrome of genital herpes and herpetic
eruption, affected individuals may experience more systemic symptoms such as headaches,
neck stiffness, and low-grade fever.
When these symptoms are associated with altered mentation, encephalitis is most likely.
Acyclovir is the most appropriate initial therapy for HSV encephalitis. One of the most
common adverse effects associated with acyclovir is crystalline nephropathy, which results
in transient renal impairment.
This risk can be reduced with adequate hydration and a slow infusion rate.

Go to the next page if you knew the correct answer, or click the link image(s) below to further
research the concepts in this question (if desired).

Research Concepts:
Herpes Simplex Type 2

We update eBooks quarterly and Apps daily based on user feedback. Please tap flag to
report any questions that need improvement.
Question 852: A 74-year-old male with a history of coronary artery disease, hypertension,
and paroxysmal atrial fibrillation presents with several days of mild chest discomfort and fatigue.
On arrival, his blood pressure is 135/88 mmHg, the pulse is 87/minute, and the respiratory rate is
20/minute. On physical exam, he appears tachypneic, and mile volume overload is noted. An
echocardiogram shows a reduced ejection fraction of 30% and alternating beat-to-beat left
ventricular arterial waveforms. What is the most common electrocardiogram finding for patients
with this condition?

Choices:
1. Atrial fibrillation
2. Sinus rhythm
3. Supraventricular tachycardia
4. Electrical alternans
Answer: 2 - Sinus rhythm
Explanations:
Sinus rhythm or sinus tachycardia is the most common ECG findings with pulsus alternans.
Do not confuse pulsus alternans with electrical alternans.
Pulsus alternans is identified as variable beat-to-beat ventricular arterial waveforms.
Pulsus alternans is most commonly associated with heart failure and considered a poor
prognostic indicator.
Electrical alternans is the ECG finding of variable R wave voltage. It is associated with
pericardial effusion.

Go to the next page if you knew the correct answer, or click the link image(s) below to further
research the concepts in this question (if desired).

Research Concepts:
Pulsus Alternans

We update eBooks quarterly and Apps daily based on user feedback. Please tap flag to
report any questions that need improvement.
Question 853: A 20-year-old male was extubated about 20 minutes ago. He had been
intubated two days prior for status epilepticus requiring emergency intubation with a 7.0 mm
endotracheal tube. He complains of a choking sensation in his throat, and his voice is muffled.
Physical examination reveals an anxious appearing patient with a respiratory rate of 28
breaths/min. He is hemodynamically stable, and oxygen saturation is 93% on mask oxygen at 5
L/min. There is a subtle, high-pitched sound noted on inspiration. Which of the following is a
risk factor for developing this condition?

Choices:
1. Intubation for more than six days
2. Male sex
3. Cuff pressure of 20 cmH2O
4. Undersized endotracheal tube
Answer: 1 - Intubation for more than six days
Explanations:
This 20-year-old male has post-extubation stridor, for which post-intubation laryngeal
edema is a major contributor. The 2017 American Thoracic Society and American College
of Chest Physicians guidelines recommend performing a bedside cuff leak test on all high-
risk patients to predict the chance of developing post-extubation stridor. Known risk factors
include duration of intubation greater than six days, female sex, traumatic intubation,
oversized endotracheal tubes, and reintubation after unplanned extubation.
The cuff leak test has varying sensitivity but an excellent specificity of more than 90% in
most studies evaluating the prediction of post-extubation stridor. For this reason, it has a
high negative predictive value in high-risk patients. A negative cuff leak test with a leak
volume of fewer than 110 ml or less than 24% of the delivered tidal volume predicts a
higher likelihood of post-extubation stridor. About half of patients with confirmed post-
extubation stridor require reintubation, and every reintubation event increases morbidity and
mortality.
Intravenous methylprednisolone 40 mg as a single dose may be given at least 4 hours prior
to extubation to help prevent the development of post-intubation laryngeal edema or stridor.
If the patient develops post-extubation stridor with or without the above measures (i.e., cuff
leak test in high-risk patients and prophylactic methylprednisolone), the treatment is
intravenous steroids for 24 to 48 hours. Some clinicians add adrenaline nebulization, though
the clinical benefit in adults is yet to be proved. Close monitoring is continued, and
reintubation should not be delayed beyond an hour if it is necessary.

Go to the next page if you knew the correct answer, or click the link image(s) below to further
research the concepts in this question (if desired).

Research Concepts:
Post Intubation Laryngeal Edema

We update eBooks quarterly and Apps daily based on user feedback. Please tap flag to
report any questions that need improvement.
Question 854: A 76-year-old man with chronic kidney disease presents to the emergency
department. He is found to have hyperkalemia with correlating abnormalities on his EKG. The
clinician recommends initiation of dialysis, but the patient refuses. His daughter, also the
patient's medical power of attorney, is at the bedside and says that he has refused medical
interventions in the past. The patient says that he doesn't want to die, but does not want to live on
machines. He states that he knows that without dialysis, he will likely die from the potassium
levels. He would rather go home to be with his family and "let nature take its natural course."
Who has the decision making power in this situation?

Choices:
1. The patient
2. The medical power of attorney
3. The medical provider
4. The hospital ethicist
Answer: 1 - The patient
Explanations:
The patient can communicate his refusal, understands the link between dialysis and his
potassium levels that he will likely die without treatment. His decision is in line with
previous decisions, and it does not appear to have altered cognition from substances or
delirium.
While the daughter has a power of attorney (POA), the patient has the capacity to make
decisions. A POA cannot make decisions in lieu of a patient with capacity.
The medical provider's input is important for a patient to understand a treatment plan, but a
paternalistic approach to medicine does not respect patient autonomy and could border on
assault.
There is no role for an ethicist in this scenario.

Go to the next page if you knew the correct answer, or click the link image(s) below to further
research the concepts in this question (if desired).

Research Concepts:
Competency and Capacity

We update eBooks quarterly and Apps daily based on user feedback. Please tap flag to
report any questions that need improvement.
Question 855: A 75-year-old woman with a past medical history of chronic kidney disease
3A, hypertension, COPD, and type 2 diabetes presents to the hospital with complaints of flu-like
symptoms for 4-6 days. She reports she tested positive for SARS-CoV-2 a day back at a drive-
through testing center. At admission, the patient is noted to be febrile (100.8 °F) and hypoxic
with SpO2 87% on 4 L/min nasal cannula oxygen. The initial laboratory workup at admission is
unremarkable. CT scan of the chest without contrast shows bilateral ground-glass opacities. The
patient is admitted to low-level monitoring floors, given remdesivir 200 mg at admission, and
scheduled to receive 100 mg daily for the next nine days. The next morning, the patient’s eGFR
is 27 mL/min compared to 45 mL/min at admission. Which of the following is the next best step
in the management of this patient?

Choices:
1. Continue remdesivir infusion as scheduled
2. Reduce the dose of schedule remdesivir infusion from 100 mg daily to 50mg daily
3. Add dexamethasone
4. Discontinue remdesivir
Answer: 4 - Discontinue remdesivir
Explanations:
The patient's presentation of flu-like symptoms, acute hypoxic respiratory failure with
positive SARS-CoV-2 PCR in the setting of COPD, is consistent with moderate to severe
COVID-19.
Given the patient's deterioration in eGFR compared to admission, remdesivir should be
discontinued immediately with close monitoring of renal function.
Remdesivir is contraindicated in adult patients with eGFR 30 mL/min.
Considering remdesivir is an investigational drug permitted for emergency use in
hospitalized patients with moderate to severe COVID-19, its safety profile has not been
evaluated. Currently, there are no recommendations regarding dosages based on renal
function.

Go to the next page if you knew the correct answer, or click the link image(s) below to further
research the concepts in this question (if desired).

Research Concepts:
Remdesivir

We update eBooks quarterly and Apps daily based on user feedback. Please tap flag to
report any questions that need improvement.
Question 856: A 72-year-old woman with a past medical history of heart failure with
reduced ejection fraction (HFrEF) and diabetes is admitted to the ICU for continued
hemodynamic monitoring after presenting to the emergency department with severe bradycardia
with a heart rate of 35/min and hypotension with a blood pressure of 86/35 mmHg. An initial set
of labs is shown below.
Patient value Reference range
Sodium 130 mEq/L 134-144 mEq/L
Potassium 6.7 mEq/L 3.6-5.0 mEq/L
Creatinine 2.5 mg/dL 0.8-1.4 mg/dL
BUN 58 mg/dL 5-20 mg/dL
Bicarbonate 9 mEq/L 21-28 mEq/L
Glucose 157 mg/dL 80–140 mg/dL
The patient has received atropine, calcium gluconate, insulin/dextrose, furosemide and high-dose
albuterol with improvement of her hyperkalemia and bradycardia. However, hypotension
persists, requiring the initiation of dobutamine. Despite initial interventions, the patient continues
to have bradycardia and refractory hyperkalemia and subsequently requires dialysis.
Discontinuation of which of the following patient's medications is most likely to improve her
clinical status?

Choices:
1. Atorvastatin
2. Ranolazine
3. Dapagliflozin
4. Methimazole
Answer: 2 - Ranolazine
Explanations:
Efficacy of standard ACLS management of bradycardia with atropine is unreliable in
BRASH syndrome as the bradycardia is not parasympathetically driven.
Medications that directly or indirectly decrease the calcium concentration or block the AV
node, increase the risk of developing BRASH syndrome.
Early recognition and aggressive treatment of this syndrome can decrease the likelihood of
requiring dialysis.
Complete cessation of these offending medications is often unrealistic as they are crucial in
the management of other comorbidities. However, it is important to review home
medications because understanding the patients increased risk will lead to a more rapid
diagnosis and treatment.

Go to the next page if you knew the correct answer, or click the link image(s) below to further
research the concepts in this question (if desired).

Research Concepts:
BRASH Syndrome

We update eBooks quarterly and Apps daily based on user feedback. Please tap flag to
report any questions that need improvement.
Question 857: A 69-year-old male is admitted to the intensive care unit with a hemorrhagic
stroke. On neurologic examination absence of brainstem reflexes is noted and the apnea test does
not reveal any spontaneous respiratory drive in the patient. What is the next best step in the
management of this patient?

Choices:
1. Pronounce brain stem death
2. Take consent for organ donation from the relatives
3. Stop supportive management in the patient
4. Repeat examination after 6 hours
Answer: 4 - Repeat examination after 6 hours
Explanations:
Diagnosing brain stem death requires complete examination and documentation from at
least two qualified physicians with a minimum interval of at least 6 hours.
The team advocated for the same include the medical superintendent of the hospital, the
treating physician with one neurologist or a neurosurgeon with no inclusions of any
members from organ donation or transplant team.
The relatives should be given the utmost respect and some time for bereavement from the
loss of their beloved before embarking upon the aspect of organ donation.
The supportive treatment needs to be continued until the relatives' consent of the
termination of the care. Supportive management is also justified for the preservation of
other organs during the probable aspect of organ donation.

Go to the next page if you knew the correct answer, or click the link image(s) below to further
research the concepts in this question (if desired).

Research Concepts:
Brainstem Death

We update eBooks quarterly and Apps daily based on user feedback. Please tap flag to
report any questions that need improvement.
Question 858: A 45-year-old man presents with progressive exertional dyspnea, fever, and
dry cough. He has a past medical history of HIV. Blood tests show CD4 count 150 cell/mm3
(500-1500). A chest x-ray reveals bilateral interstitial infiltrates. The patient is started on
trimethoprim-sulfamethoxazole but does not improve and shows signs of severe infection. Which
of the following is the most effective method of administration of the drug that should be
prescribed for this patient's new symptoms?

Choices:
1. Intramuscular
2. Nebulized
3. Intravenous
4. Oral
Answer: 2 - Nebulized
Explanations:
Pentamidine is an antiprotozoal drug for pneumocystis jiroveci pneumonia and other related
infections.
It can be administered IV, IM, orally, or as an inhaled dose.
It can be given therapeutically or prophylactically to HIV-positive patients.
A nebulized dose is more commonly used prophylactically and in treatment.

Go to the next page if you knew the correct answer, or click the link image(s) below to further
research the concepts in this question (if desired).

Research Concepts:
Pentamidine

We update eBooks quarterly and Apps daily based on user feedback. Please tap flag to
report any questions that need improvement.
Question 859: A 65-year-old male presents with complaints of watery diarrhea for three
days and decreased urine output for one day. He reports extreme lethargy and generalized
weakness. On examination, there is a pale looking male with signs of dehydration, his blood
pressure is 100/60 mmHg, pulse 88/min regular, respiratory rate 22/min, and decreased strength
in all four limbs. His arterial blood gases reveal pH 7.28, HCO3 15 mmol/L, and pCO2 33
mmHg. His serum sodium is 148 mEq/L, potassium 4 mEq/L, and chloride 126 mmol/L. Having
calculated his anion gap, how can one determine what system is leading to this acid-base
imbalance?

Choices:
1. By measuring calcium
2. By measuring albumin and correcting the gap
3. By calculating urinary anion gap
4. It cannot be determined
Answer: 3 - By calculating urinary anion gap
Explanations:
Determining exact etiology of a narrow anion gap hyperchloremic acidosis requires another
test, the urine anion gap. The urine anion gap is calculated. Urine anion gap = (Na + K) - Cl
where Na is urine sodium, K is urine potassium, and Cl is urine chloride.
The urine anion gap provides an estimate of urinary ammonium (NH4) excretion. The
normal renal response to metabolic acidosis is to increase acidic NH4 excretion renally.
A positive urine anion gap between 20 and 90 mEq/L is indicative of low or normal NH4
excretion, such as is seen in distal renal tubular acidosis. A negative urine anion gap
between -20 and -50 mEq/L is indicative of increased NH4 excretion.
This occurs in patients with metabolic acidosis generated by profuse watery diarrhea. A
urine anion gap approaching 0 is indeterminate.

Go to the next page if you knew the correct answer, or click the link image(s) below to further
research the concepts in this question (if desired).

Research Concepts:
Hyperchloremic Acidosis

We update eBooks quarterly and Apps daily based on user feedback. Please tap flag to
report any questions that need improvement.
Question 860: A previously healthy 37-year-old male develops acute chest pain associated
with dyspnea. He has no cardiac risk factors and denies illicit drug use. His condition
deteriorates, and he develops cardiogenic shock. Electrocardiogram demonstrates ST elevation in
the lateral leads and repolarization abnormalities. Cardiac enzymes are elevated. Select the most
important diagnostic test.

Choices:
1. CT of the chest
2. Coronary angiography
3. Endomyocardial biopsy and viral titers
4. Transesophageal echocardiogram
Answer: 2 - Coronary angiography
Explanations:
With a cardiogenic shock, coronary artery disease must be identified.
Even though the patient may have acute myocarditis presenting with pseudoinfarction,
myocardial biopsies do not provide guidance on treatment.
The patient should also have an echocardiogram to determine the extent of cardiac
dysfunction and also look for any valvular problem.
Surgery for cardiogenic shock carries very high mortality even in the best of hands. An intra
aortic balloon pump should be inserted before proceeding with any procedure or surgery.

Go to the next page if you knew the correct answer, or click the link image(s) below to further
research the concepts in this question (if desired).

Research Concepts:
Cardiogenic Shock

We update eBooks quarterly and Apps daily based on user feedback. Please tap flag to
report any questions that need improvement.
Question 861: A 65-year-old male with a history of coronary artery disease, hypertension,
chronic kidney disease stage 3, severe aortic valve stenosis is undergoing exploratory laparotomy
for suspected intestinal obstruction under general anesthesia. The surgeon is requesting paralysis
for the duration of the surgery. Which of the following paralytics should be avoided in this
patient given the patient's comorbidities?

Choices:
1. Rocuronium
2. Pancuronium
3. Vecuronium
4. Gantacurium
Answer: 2 - Pancuronium
Explanations:
Pancuronium is a nondepolarizing muscle relaxant used for endotracheal intubation and
general anesthesia, and if often associated with tachycardia.
Pancuronium is primarily excreted unchanged by the kidneys and may accumulate in
patients with kidney disease or injury. The liver does metabolize an insignificant amount. It
should not be used in patients with renal insufficiency.
Pancuronium may cause tachycardia and may increase myocardial oxygen demand, which
may be detrimental in a patient with coronary artery disease. The provider must also be
cognizant that tachycardia is not desired in patients with severe aortic valve stenosis as this
may precipitate arrhythmias and potentially sudden cardiac death.
Rocuronium, gantacurium, and vecuronium are not associated with tachycardia and are,
therefore, not contraindicated in this patient.

Go to the next page if you knew the correct answer, or click the link image(s) below to further
research the concepts in this question (if desired).

Research Concepts:
Nondepolarizing Paralytics

We update eBooks quarterly and Apps daily based on user feedback. Please tap flag to
report any questions that need improvement.
Question 862: A 66-year-old man with a past medical history of alcohol use disorder
underwent a right chronic subdural hematoma using a burr hole 10 days ago and discharged
home two days later. He now presents to the emergency department after suffering from a
seizure for the first time. The patient is drowsy. In addition to the seizure episode, he also
complains of headaches, fever, and chills. The initial exam shows right upper extremity
weakness. Which of the following is the most likely diagnosis?

Choices:
1. Epilepsy
2. Recurrent subdural hematoma
3. Subdural infectious collection
4. Alcohol intoxication
Answer: 3 - Subdural infectious collection
Explanations:
Infected subdural hematomas after surgical drainage may develop into an empyema.
Predisposing factors associated with the development of subdural empyema include prior
cranial surgery.
Drainage of a subdural hematoma can cause a subdural empyema. The patient can present
with a seizure, headaches, fever, and decreased level of consciousness.
Patients may be intoxicated from ethanol abuse and may develop seizures, but should not
develop a focal neurological deficit unless they have an intracranial pathology. The history
of headaches, fever, chills, and recent surgery points to an infected subdural hematoma.

Go to the next page if you knew the correct answer, or click the link image(s) below to further
research the concepts in this question (if desired).

Research Concepts:
Subdural Empyema

We update eBooks quarterly and Apps daily based on user feedback. Please tap flag to
report any questions that need improvement.
Question 863: A 65-year-old male smoker is diagnosed with community-acquired
pneumonia. He is febrile, respiratory rate 32/min, alert, and blood pressure 116/80 mmHg. Lab
tests show WBC at 15,000/microliter with a left shift. Serum electrolytes are normal, BUN 32
mg/dL and serum creatinine 1.5 mg/dL. Oxygen saturation on room air is 90 percent. Which of
the following is correct?

Choices:
1. Admit to inpatient service and treat with ceftriaxone and azithromycin
2. Discharge to home on levofloxacin
3. Admit and treat with vancomycin and gentamicin
4. Discharge to home on amoxicillin/clavulanate
Answer: 1 - Admit to inpatient service and treat with ceftriaxone and azithromycin
Explanations:
The CURB-65 criteria include age 65 years or older, confusion, urea > 19.6 mg/dL,
respiratory rate > 30/min, and blood pressure systolic 90 mmHg and diastolic 60 mmHg.
CURB-65 score estimates mortality based on the score achieved by using these measures.
The patient's age, respiratory rate, and BUN are elevated indicating giving a score of 3,
conferring a 30-day mortality of 9.2 percent.
A score of 2 or more requires inpatient treatment and he should be treated with a beta-
lactam and a macrolide or a respiratory quinolone.
Amoxicillin/clavulanate will not cover atypical pathogens.

Go to the next page if you knew the correct answer, or click the link image(s) below to further
research the concepts in this question (if desired).

Research Concepts:
Community-Acquired Pneumonia

We update eBooks quarterly and Apps daily based on user feedback. Please tap flag to
report any questions that need improvement.
Question 864: A 72-year-old man who suffered a large stroke two days ago is recovering in
the neurological ICU. He has a dense hemiplegia bilaterally but is otherwise stable. His medical
history and surgical history is significant only for poorly controlled hypertension. A nasogastric
tube is placed for feeds, and the clinician placing it insufflates air and auscultates under the left
costal margin to determine its position. When feeds are started, the patient desaturates, dropping
to a SaO2 of 88% on room air, and he becomes tachycardic to 140. What is the appropriate next
step in management?

Choices:
1. Stop the feeds and send a full electrolyte panel
2. Send the patient for an emergent CT pulmonary embolism protocol and begin therapeutic
heparin
3. Send the patient for an emergent CT scan of the head
4. Stop the feeds, withdraw the nasogastric tube, and consider bronchoscopy
Answer: 4 - Stop the feeds, withdraw the nasogastric tube, and consider bronchoscopy
Explanations:
Given that this patient is only two days out from his stroke, refeeding syndrome is an
unlikely diagnosis. The most likely diagnosis is the nasogastric tube was improperly placed,
and the patient had tube feeds placed directly into his lungs. Stopping the feeds is the right
option, but sending an electrolyte panel is unnecessary.
It is unlikely that this patient could develop a thrombus and then embolize it after only two
days of paralysis, although it is not impossible. Of course, with this presentation, a PE
should be considered. However, in the setting of a recent stroke, therapeutic heparin is
contraindicated.
While this patient is at risk for a recurrent stroke given his history of poorly controlled
hypertension, the proximity of his decomposition to the starting of tube feeds should raise
suspicion that the two are related. The placement of the tube was not confirmed via Xray
and only by auscultation, a known ineffective technique. This should put intrapulmonary
feeds as the highest diagnosis on the differential.
Intrapulmonary feeding is the most likely explanation for this patient's acute
decompensation after. Additionally, the improper technique used to identify if the tube was
properly placed should raise concern. The feeds should be immediately stopped and the tube
withdrawn. If only a small amount of feed was instilled, then observation may be
acceptable. If a large amount was given, then bronchoscopy should be considered to suction
out excess. Also, there should be no reluctance to begin antibiotics if the patient develops a
fever as these intrapulmonary feeds will predispose to pneumonia.

Go to the next page if you knew the correct answer, or click the link image(s) below to further
research the concepts in this question (if desired).

Research Concepts:
Nasogastric Tube

We update eBooks quarterly and Apps daily based on user feedback. Please tap flag to
report any questions that need improvement.
Question 865: A 65-year-old man with a history of human immunodeficiency virus (HIV)
infection, hypothyroidism, hepatitis C infection, and hypertension presents with a productive
cough and fever. His initial vital signs showed a temperature of 38.1 C (100.5 F), heart rate 111
bpm, oxygen saturation 88%, and blood pressure 103/54 mmHg. A chest radiograph reveals right
lower lobe opacity. His labs show a white blood cell count of 11,000 cells/mm3, creatinine of 1.8
mg/dL, troponin level of 0.4 ng/mL, and brain natriuretic peptide of 550 pg/ml. The patient
admits to nonadherence with his HIV treatment. His labs one month ago showed creatinine of
0.9 mg/dL, troponin level was 0.03 ng/mL, and brain natriuretic peptide 75 pg/ml. Bedside
echocardiogram shows reduced ejection fraction compared to the last transthoracic
echocardiogram he had nine months ago. His condition deteriorates in the emergency
department, he is requiring more oxygen and eventually gets intubated secondary to worsening
respiratory failure. What would most likely improve his cardiac function in the long term?

Choices:
1. Heparin infusion protocol
2. 40 mg of intravenous furosemide followed by 10mg/hr continuous infusion
3. 30 ml/kg/hr lactated ringers fluid resuscitation followed by broad-spectrum antibiotics
4. Continuous dobutamine infusion
Answer: 3 - 30 ml/kg/hr lactated ringers fluid resuscitation followed by broad-spectrum
antibiotics

Explanations:
Patients that are non-compliant with their HIV medications are at a higher risk to develop a
severe infection due to their immunocompromised status. This patient presented with sepsis
most likely secondary to pneumonia. He met severe sepsis criteria and had a new onset of
acute kidney injury and decreased cardiac function which is suggestive of cardiorenal
syndrome type 5.
Patients can develop type 5 cardiorenal syndrome in the setting of sepsis, systemic lupus
erythematosus (SLE), diabetes mellitus, decompensated cirrhosis, or amyloidosis; all of
these disorders can lead to disease in both the heart and the kidney.
The cornerstone treatment of type 5 cardiorenal syndrome is the treatment of underlying
causes. Early fluid resuscitation and antibiotics administration remain the gold standard
level of care for sepsis that would improve both heart and kidneys function.
The patient did not have active chest pain, and his elevated troponin might be secondary to
demanding ischemia due to sepsis. Starting heparin will not be necessary now. Moreover,
the patient needs to be fluid resuscitated now rather than to be started on a diuretic. He most
likely was intubated due to worsening infection and not due to worsening heart function.

Go to the next page if you knew the correct answer, or click the link image(s) below to further
research the concepts in this question (if desired).

Research Concepts:
Cardiorenal Syndrome

We update eBooks quarterly and Apps daily based on user feedback. Please tap flag to
report any questions that need improvement.
Question 866: A 40-year-old female is brought to the emergency department with a
gunshot wound to the abdomen. The patient is emergently intubated and was taken for an
exploratory laparotomy. She was found to have a small bowel injury with intraperitoneal
hemorrhage. The patient received six units of blood intraoperatively. After the procedure, she
was moved to the intensive care unit (ICU) for monitoring. The next day the patient undergoes
central line placement for total parenteral nutrition (TPN) administration. Which of the following
possible complication is most likely in this patient?

Choices:
1. Sepsis
2. Cholelitathsis
3. Hypocalcemia
4. Refeeding Syndrome
Answer: 1 - Sepsis
Explanations:
TPN has an increased risk of hyperglycemia, especially in the first 1-2 days, and is
associated with which would be a risk factor for sepsis in this patient. This association is
because hyperglycemia impairs leukocyte function and thus decreases the patient’s immune
function.
Cholelithiasis occurs weeks after the initiation of TPN.
Hypocalcemia ould be possible due to citrate in blood transfusions but unlikely as the TPN
contains calcium.
While TPN administration can be complicated by refeeding syndrome, which results in
electrolyte abnormality. Patients on TPN often have their electrolytes checked daily to
assess any changes.

Go to the next page if you knew the correct answer, or click the link image(s) below to further
research the concepts in this question (if desired).

Research Concepts:
Total Parenteral Nutrition

We update eBooks quarterly and Apps daily based on user feedback. Please tap flag to
report any questions that need improvement.
Question 867: A 17-year-old boy is brought to the emergency department by paramedics
with chest pain for 2 hours. He has associated diaphoresis and nausea. The patient is a university
student and was at a local in-house party when he suddenly fell to the ground because of the
debilitating chest pain. An accompanying friend says that he saw a packet of “white powder” in
his room the other day. His blood pressure is 160/94 mmHg, and the pulse is 144/min. Eye
examination reveals dilated pupils bilaterally. ECG shows ST-segment elevations in leads II, III,
and AVF. What is the drug of choice for the stabilization of this patient?

Choices:
1. Diazepam
2. Metoprolol
3. Citalopram
4. Heparin
Answer: 1 - Diazepam
Explanations:
A white powder that presents as dilated pupils, increased blood pressure, tachycardia, and
chest pain is most likely cocaine.
Cocaine increases the concentrations of catecholamines, such as epinephrine and
norepinephrine. Increased concentrations of catecholamines potentiate activation of the
alpha 1, beta 1, and beta 2 receptors.
In some patients, these increased concentrations can contribute to the onset of ventricular
fibrillation (V-fib) along with myocardial ischemia.
Diazepam reduces the sympathetic outflow by decreasing beta 1 and alpha 1 receptor
activity. Metoprolol is not used for the management of acute myocardial infarction due to
cocaine use.

Go to the next page if you knew the correct answer, or click the link image(s) below to further
research the concepts in this question (if desired).

Research Concepts:
Beta 1 Receptors

We update eBooks quarterly and Apps daily based on user feedback. Please tap flag to
report any questions that need improvement.
Question 868: A 70-year-old woman is readmitted to the ICU with respiratory failure
requiring intubation due to chronic obstructive pulmonary disease (COPD) exacerbation. She has
a history of COPD, type 2 diabetes, coronary artery disease (CAD), and a 7-day hospitalization
12 days ago. On the 4th day of her admission, she develops increased sputum production, a
temperature of 101.9°F (38.8°C), deteriorating oxygenation, and a recent infiltrate in the base of
the left lung. Which of the following empiric antibiotic regimen is most appropriate for this
patient?

Choices:
1. Azithromycin plus moxifloxacin
2. Linezolid plus tobramycin
3. Cefepime plus vancomycin
4. Ceftriaxone plus azithromycin
Answer: 3 - Cefepime plus vancomycin
Explanations:
Ventilator-associated pneumonia (VAP) is a term used to describe pneumonia (lung
infection) that develops in a patient who has been on mechanical ventilation for more than
48 hours.
Ventilator-associated pneumonia is suspected in the setting of fever, a change in
auscultation exam, a change in the chest x-ray, and an increasing requirement for
respiratory support.
Despite the fact that this patient has early-onset ventilator-associated pneumonia (VAP) for
the recent admission, a history of the previous hospitalization is a risk factor for multidrug-
resistant (MDR) organisms.
Initial broad-spectrum therapy with coverage of gram-negative bacilli, including
Pseudomonas aeruginosa and possibly methicillin-resistant S. aureus, is generally most
appropriate. Therefore, cefepime plus vancomycin should be given as empiric therapy in
such patients.

Go to the next page if you knew the correct answer, or click the link image(s) below to further
research the concepts in this question (if desired).

Research Concepts:
Ventilator-associated Pneumonia

We update eBooks quarterly and Apps daily based on user feedback. Please tap flag to
report any questions that need improvement.
Question 869: A 66-year-old woman is admitted to the intensive care unit for diagnosis of
septic shock due to right lower lobe pneumonia. She is given broad-spectrum antibiotics, and
fluid resuscitation is begun. Despite aggressive fluid resuscitation, she continues to be
hypotensive. A right subclavian line is attempted and inserted after 2 attempts. A few minutes
later, the patient started to have worsening shortness of breath and hypotension with increasing
pressor requirements. Which of the following is the next best step in the management of this
patient?

Choices:
1. Lung ultrasound
2. Broaden antibiotic coverage
3. tPA
4. CT angiogram of the chest
Answer: 1 - Lung ultrasound
Explanations:
This patient had a right-sided subclavian line placed after multiple attempts leading to
pneumothorax.
Pneumothorax is one of the common complications of subclavian line placement. It can
present as hypoxemia and shock.
Urgent decompression of the pleural space is needed with the placement of a chest tube.
Lung ultrasound can be performed quickly and efficiently at the bedside, which can show
absence of pleural sliding and a lung point.
Although a CT scan may be helpful, logistically, it can take time leading to delayed
diagnosis and potentially catastrophic complications.

Go to the next page if you knew the correct answer, or click the link image(s) below to further
research the concepts in this question (if desired).

Research Concepts:
Central Venous Access of The Subclavian Vein

We update eBooks quarterly and Apps daily based on user feedback. Please tap flag to
report any questions that need improvement.
Question 870: A 56-year-old man with hypertension presented to the emergency
department with central chest pain, radiating to the jaw for an hour. On arrival in the emergency
department, he was found to have a heart rate of 110 beats per minute, blood pressure of 190/120
mmHg, and the rest of the examination was unremarkable. A 12 leads electrocardiogram shows
sinus tachycardia with subtle ST depressions in inferior leads. What is the next step of
management?

Choices:
1. Intravenous esmolol
2. Intravenous hydralazine
3. Thrombolysis
4. Primary percutaneous coronary intervention
Answer: 1 - Intravenous esmolol
Explanations:
Hypertensive emergency is characterized by uncontrolled blood pressure (BP) lead to
progressive or impending end-organ dysfunction.
Cardiovascular end-organ damage may include myocardial ischemia/infarction, acute left
ventricular dysfunction, acute pulmonary edema, and/or aortic dissection.
The patient has presented with a hypertension emergency, resulting in acute coronary
syndrome. Beta-blockers are the preferred agents in this situation.
Thrombolytics are not recommended in non-ST elevation myocardial infarction (NSTEMI)
and are contraindicated in a hypertensive emergency.

Go to the next page if you knew the correct answer, or click the link image(s) below to further
research the concepts in this question (if desired).

Research Concepts:
Esmolol

We update eBooks quarterly and Apps daily based on user feedback. Please tap flag to
report any questions that need improvement.
Question 871: A 66-year-old male with a past medical history of chronic kidney disease,
hypertension, and diabetes presents to the emergency room with complaints of nausea, anorexia,
chest pain, and shortness of breath. His pain is sharp and gets worse with deep breathing and
coughing. His pain gets better when he leans forward. His vitals are suggestive of blood pressure
of 150/80 mmHg, pulse rate of 80 beats per minute, and temperature of 99.4 F. A pericardial
friction rub is present on auscultation. The patient has 2+ pitting edema in his lower extremities.
Laboratory data shows creatinine of 5.2 mg/dl, BUN of 110 mg/dl, and potassium of 4.8
mmol/L. His echocardiography shows a small amount of pericardial effusion. What is the best
treatment option for this patient's condition?

Choices:
1. Ibuprofen
2. Colchicine
3. Pericardiocentesis
4. Hemodialysis
Answer: 4 - Hemodialysis
Explanations:
Uremia is an important cause of pericarditis. It occurs due to the inflammation of the
pericardium due to toxic metabolites.
Most patient with uremic pericarditis responds to dialysis and have rapid improvement in
chest pain and pericardial effusion.
Pleuritic chest pain, pericardial friction rub, and fever are major signs and symptoms of
uremic pericarditis.
End-stage renal disease patients on renal replacement therapy can also develop uremic
pericarditis due to inadequate dialysis.

Go to the next page if you knew the correct answer, or click the link image(s) below to further
research the concepts in this question (if desired).

Research Concepts:
Fibrinous Pericarditis

We update eBooks quarterly and Apps daily based on user feedback. Please tap flag to
report any questions that need improvement.
Question 872: A 71-year-old woman is being evaluated in the intensive care unit. She was
diagnosed with advanced-stage lung cancer with bony and brain metastasis six months ago. The
patient was treated with aggressive chemotherapy and radiotherapy; however, she did not
respond to the treatment. The surgeon decides not to operate on the patient as the surgery will not
alleviate the suffering. The physician recommends terminating care and allowing the patient to
die. The patient has a GCS of 8/15. Which of the following is primarily responsible for deciding
on the physician's recommendation in this scenario?

Choices:
1. Ethics committee
2. Power of attorney
3. Physician
4. Patient
Answer: 2 - Power of attorney
Explanations:
When symptoms are no longer manageable, or patient suffering is too great, termination of
care must be considered.
Termination of care is not something decided on a whim; rather it is part of the initial end-
of-life discussions. When the treatment the healthcare team members are providing is
medically futile or prolonging patient suffering, terminating the care and allowing the
patient to die is necessary.
If the patient can participate in these decisions, it should be the patient’s decision when they
want to terminate care.
If the patient is developmentally or cognitively capable of making such decisions, this
responsibility falls to a power of attorney.

Go to the next page if you knew the correct answer, or click the link image(s) below to further
research the concepts in this question (if desired).

Research Concepts:
End of Life Care

We update eBooks quarterly and Apps daily based on user feedback. Please tap flag to
report any questions that need improvement.
Question 873: A 16-year-old girl is brought to the hospital with three weeks history of
confusion, unusual behavior, as well as difficulty in fluent speech and being forgetful. This was
preceded by flulike symptoms for about a week. During diagnostic workup, her CSF is found to
be positive for anti-NMDAR IgG antibodies. Which infectious disease in the preceding weeks is
most consistent with the most likely diagnosis?

Choices:
1. Herpes simplex encephalitis
2. Cat scratch disease
3. Staphylococcal skin infection
4. Streptococcal throat infection
Answer: 1 - Herpes simplex encephalitis
Explanations:
After HSV encephalitis, approximately one-third of patients develop autoimmune
encephalitis after 2-16 weeks of primary encephalitis.
Younger patients (less than 4 years) may develop autoimmune encephalitis in a shorter
interval than older patients.
The outcome of these autoimmune encephalitides is generally poor compared with the
overall outcome of anti-NMDAR encephalitis.
Cat scratch disease, staphylococcal skin infection, and streptococcal throat infection do not
have an association with anti-NMDAR encephalitis.

Go to the next page if you knew the correct answer, or click the link image(s) below to further
research the concepts in this question (if desired).

Research Concepts:
Anti-NMDA Receptor Encephalitis

We update eBooks quarterly and Apps daily based on user feedback. Please tap flag to
report any questions that need improvement.
Question 874: A 24-year-old male presents to the emergency department (ED) after an
opioid overdose. He required 0.6 mg of naloxone in the ED upon arrival, which he returned to
baseline. During the observation period, he becomes more somnolent with a decreased
respiratory rate and a falling oxygen saturation. He requires an additional 0.6 mg of naloxone
with improvement. Given his course, the clinician planned to admit him on a naloxone infusion.
What is the rate of the naloxone drip that should be ordered in this patient?

Choices:
1. 0.3 mg/hour
2. 0.4 mg/hour
3. 0.5 mg/hour
4. 0.6 mg/hour
Answer: 2 - 0.4 mg/hour
Explanations:
Naloxone is a competitive opioid antagonist that inhibits the mu-opioid receptor.
In a patient who requires multiple boluses of naloxone, an infusion can be started. The
initial dose per hour of naloxone is 2/3 of the bolus reversal dose of naloxone.
This patient required 0.6 mg of naloxone to have a reversal of his opioid toxicity. 2/3 of 0.6
is 0.4 mg.
Therefore, the initial infusion rate should be 0.4 mg/hour of naloxone via continuous
infusion.

Go to the next page if you knew the correct answer, or click the link image(s) below to further
research the concepts in this question (if desired).

Research Concepts:
Naloxone

We update eBooks quarterly and Apps daily based on user feedback. Please tap flag to
report any questions that need improvement.
Question 875: A 23-year-old male patient is admitted to the emergency department after a
motor vehicle collision. He has a blood pressure of 120/78 mm Hg, the respiration rate of 22
breaths per minute, a pulse of 123 beats per minute, and he is afebrile. He is immediately started
on 125 ml/hr of normal saline. Over the next 3 hours, he has a urine output of 10 cc/hr, 13 ml/hr,
and 18 ml/hr. His central venous pressure is 3 cm H2O, and his blood pressure remains the same.
The Foley catheter appears to be draining well. What is the next step in his management?

Choices:
1. Administer furosemide intravenously
2. Continue present fluid
3. Bolus patient with 500 mL of normal saline
4. Add dopamine to the drip
Answer: 3 - Bolus patient with 500 mL of normal saline
Explanations:
A low central venous pressure and urine output are suggestive of a lack of fluids.
The patient needs extra fluids until the central venous pressure is around 6 to 9 cm H2O.
Normal saline is the fluid of choice.
Pressure support is not needed.

Go to the next page if you knew the correct answer, or click the link image(s) below to further
research the concepts in this question (if desired).

Research Concepts:
Fluid Resuscitation

We update eBooks quarterly and Apps daily based on user feedback. Please tap flag to
report any questions that need improvement.
Question 876: A 65-year-old male patient with a past medical history of ulcerative colitis
and multiple hospitalizations for pancreatitis presents for a scheduled bowel resection. On
postoperative day two, nursing staff notice that he appears to be confused, and he complains of
anxiety, insomnia, nausea, and abdominal pain. His blood pressure is 166/90 mmHg, pulse
99/min, respirations 16/min, and temperature 100.0 F (37.8 C). The patient has an alanine
aminotransferase (ALT) of 209 IU/L and an aspartate aminotransferase (AST) of 422 IU/L. What
is the most concerning consequence of this patient’s condition if he does not receive treatment?

Choices:
1. Agitation
2. Hallucinations
3. Seizures
4. Elevated body temperatures
Answer: 3 - Seizures
Explanations:
This patient has a history of multiple hospitalizations for pancreatitis, and an AST:ALT
ratio of about 2:1, suggesting a history of alcohol abuse. More than 50% of those with a
history of alcohol abuse can exhibit alcohol withdrawal symptoms on discontinuing or
decreasing their alcohol use. However, only a few (3% to 5%) exhibit symptoms of severe
alcohol withdrawal with profound confusion, autonomic hyperactivity, and cardiovascular
collapse. This is defined as alcohol withdrawal delirium, more commonly known as
delirium tremens (DT).
The duration of the last drink becomes critical in recognizing the severity of symptoms. The
initial minor withdrawal symptoms are characterized by anxiety, insomnia, palpitations,
headache, and gastrointestinal symptoms. These symptoms usually occur as early as 6 hours
after cessation of alcohol use.
After 12 hours, minor withdrawal symptoms can progress to alcohol hallucinosis, a
condition characterized by visual hallucinations. It can typically resolve in 24 to 48 hours,
and may also be associated with auditory and tactile hallucinations.
One of the more serious complications of alcohol withdrawal is the potential for withdrawal
seizures. An alcohol withdrawal seizure is followed by alcohol hallucinosis, though it may
present as early as 6 hours before alcohol hallucinosis. It can present as the only withdrawal
symptoms but typically resolves 24 to 48 hours later. Seizures can recur, though rarely lead
to status epileptics. Uncharacteristic signs of seizure activity should warrant further workup.
If withdrawal symptoms remain untreated, this can typically lead to DT. DT is characterized
by visual hallucinations, profound confusion, tachycardia, hypertension, hyperthermia,
agitation, and diaphoresis.

Go to the next page if you knew the correct answer, or click the link image(s) below to further
research the concepts in this question (if desired).

Research Concepts:
Delirium Tremens

We update eBooks quarterly and Apps daily based on user feedback. Please tap flag to
report any questions that need improvement.
Question 877: Which is the most appropriate statement regarding patients with severe
burns?

Choices:
1. The addition of colloid-containing solution during the first 24 hours post-burn has shown to
decrease fluid requirements in critical burn patients
2. Use of the Parkland formula for fluid resuscitation is contraindicated in patients with
congestive heart failure
3. Lactated Ringer's is the resuscitation fluid of choice during the first 24 hours post-burn
4. The single best indicator of adequate fluid resuscitation in major burn patients is blood
pressure
Answer: 3 - Lactated Ringer's is the resuscitation fluid of choice during the first 24 hours
post-burn

Explanations:
Recent practice guidelines for burn shock resuscitation consider supplementation of colloid-
containing fluid after the first 24 hours post-burn, as it has shown to decrease overall fluid
requirements. The American Burn Association recommends a colloid infusion of 5%
albumin at the amount of (0.3-1.0)/16 mL/kg/% TBSA/hr after the initial 24-hour
crystalloid resuscitation.
There are no true contraindications to use of the Parkland formula. Patients with congestive
heart failure and renal failure must be adequately resuscitated, but also monitored carefully
for signs of fluid overload.
Lactated Ringer's is the most popular choice of crystalloid solution, as it effectively treats
both hypovolemia and extracellular sodium deficits caused by burn injury, and it is isotonic,
inexpensive, readily available, and easily stored.
The single best indicator of adequate fluid resuscitation in major burn patients is hourly
urine output. The fluid rate should be adjusted to maintain urine output between 0.5 and 1
mL/kg/hour in adults, and between 1.0 and 1.5 mL/kg/hour in children.

Go to the next page if you knew the correct answer, or click the link image(s) below to further
research the concepts in this question (if desired).

Research Concepts:
Parkland Formula

We update eBooks quarterly and Apps daily based on user feedback. Please tap flag to
report any questions that need improvement.
Question 878: A 16-year-old otherwise healthy female on no medications presents
complaining of swelling and is found to have a BP of 150/93, periorbital edema, 3+ peripheral
edema, normal cardiovascular exam and the following lab values: Creatinine 0.7 mg/dL, albumin
2.2 g/L, Urinalysis 3+ protein, 0 RBC/HPF, 0 WBC/HPF, 1+oval fat bodies. What should be the
first diagnostic test ordered?

Choices:
1. Renal ultrasound
2. Measurement of urine albumin
3. Serum protein electrophoresis
4. Test for anti-neutrophil cytoplasmic antibodies (ANCA)
Answer: 2 - Measurement of urine albumin
Explanations:
This is a case of nephrotic syndrome as indicated by low serum albumin resulting in
hypertension and edema.
Nephrotic range proteinuria is 3+ to 4+ protein by dipstick urinalysis or a urine albumin:
creatinine ratio over 3.5 mg/g and the nephrotic syndrome causes oval fat bodies to be
visible on microscopic analysis of urine specimens.
Autoimmune diseases such as systemic lupus can cause nephrotic syndrome but are
typically accompanied by other systemic signs and symptoms.
Nephrotic syndrome (NS) is a clinical syndrome defined by massive proteinuria (greater
than 40 mg/m2 per hour) responsible for hypoalbuminemia (less than 30 g/L), with resulting
hyperlipidemia, edema, and various complications. It is caused by increased permeability
through the damaged basement membrane in the renal glomerulus especially infectious or
thrombo-embolic. It is the result of an abnormality of glomerular permeability that may be
primary with a disease specific to the kidneys or secondary to congenital infections,
diabetes, systemic lupus erythematosus, neoplasia, or certain drug use.

Go to the next page if you knew the correct answer, or click the link image(s) below to further
research the concepts in this question (if desired).

Research Concepts:
Nephrotic Syndrome

We update eBooks quarterly and Apps daily based on user feedback. Please tap flag to
report any questions that need improvement.
Question 879: A 31-year-old man of Middle Eastern descent who recently returned from a
safari adventure in rural Kenya presents to the emergency department for fever, fatigue, and
abdominal pain. He is found to be febrile to 102.4 F with an unremarkable physical exam. Thick
and thin smears displayed intraerythrocytic trophozoite forms consistent with P. ovale infection.
Complete metabolic panel is unremarkable. He is admitted to the hospital and treated with
hydroxychloroquine and primaquine phosphate per the CDC protocol. On the second day of
treatment, the patient reports worsening of his abdominal pain and fatigue. He attempted to walk
to the bathroom and reports significant lightheadedness, palpitations, and weakness. His urine is
dark. His exam is significant for continued fever, tachycardia to 120/min, scleral icterus,
conjunctival pallor, right upper quadrant tenderness to palpation, and hepatosplenomegaly.
Which of the following is most likely to be found on further evaluation of this patient?

Choices:
1. Direct hyperbilirubinemia; sickled cells
2. Direct hyperbilirubinemia; bite cells and Heinz bodies
3. Indirect hyperbilirubinemia; bite cells and Heinz bodies
4. Indirect hyperbilirubinemia; sickled cells
Answer: 3 - Indirect hyperbilirubinemia; bite cells and Heinz bodies
Explanations:
P. ovale infection is treated with chloroquine or hydroxychloroquine with either primaquine
phosphate or tafenoquine.
Administration of primaquine may cause a hemolytic reaction in patients with G6PD
deficiency.
G6PD deficiency hemolytic reaction will show bite cells and Heinz bodies on a blood
smear.
Hemolytic reaction results in an indirect hyperbilirubinemia.

Go to the next page if you knew the correct answer, or click the link image(s) below to further
research the concepts in this question (if desired).

Research Concepts:
Malaria

We update eBooks quarterly and Apps daily based on user feedback. Please tap flag to
report any questions that need improvement.
Question 880: A 58-year-old man presents to the emergency department with complete
paralysis of the left side, new-onset facial drooping, and slurred speech. He has a history of end-
stage renal disease on hemodialysis, liver cirrhosis, history of gastrointestinal bleeding while on
warfarin, diabetes mellitus, hyperlipidemia, hypertension, atrial fibrillation, and coronary artery
disease. He has a sedentary lifestyle, eats fast food daily, and smokes 1 pack-per-day. However,
he did quit drinking over 2 years ago. CT head does not show any acute abnormality. He has an
NIH Stroke Scale (NIHSS) of 20. He receives a single dose of IV-tPA, which results in
significant symptom improvement. His weight is 120 kg, height is 5' 3", BP is 166/90 mmHg,
HbA1c 10.1%, hemoglobin 12 g/dL, AST 593 U/L, ALT 650 U/L, albumin 2.5 gm/dL, and INR
1.5. Which of the following best describes his prognosis?

Choices:
1. He is at increased risk of bleed, as he has a HAS-BLED score of 6 and a CHA2DS2-VASc
score of 5
2. He has an equal chance of bleed and clotting, as she has a HAS-BLED score of 5 and a
CHA2DS2-VASc score of 5
3. He is at increased risk of clots, as she has a HAS-BLED score of 5 and a CHA2DS2-VASc
score of 7
4. He has an equal chance of bleed and clotting, as she has a HAS-BLED score of 6 and a
CHA2DS2-VASc score of 6
Answer: 1 - He is at increased risk of bleed, as he has a HAS-BLED score of 6 and a
CHA2DS2-VASc score of 5

Explanations:
The risk versus benefit for assessment of initiating anticoagulation for patients with atrial
fibrillation can be calculated by using validated calculations.
CHA2DS2-VASc can be used to assess clot and stroke risk. MA patient has a CHA2DS2-
VASc score of 5 points. Age :0 (65), sex: 0 (male), CHF history: 0 (no), hypertension
history: 1 (yes), stroke/TIA/thromboembolism history: 2 (yes), vascular disease history: 1
(yes), diabetes history: 1 (yes).
HAS-BLED calculation can be used to assess bleeding risk. Patient's HAS-BLED score is 6
points. Hypertension: 1 (yes), renal disease: 1 (yes), liver disease: 1 (yes), stroke history: 1
(yes), prior major bleeding or predisposition to bleeding: 1 (yes), labile INR: 1 (yes), age
>65: 0 (no), medication usage predisposing to bleeding: 0 (no), alcohol use: 0 (no).
Comparing the patient's HAS-BLED and CHA2DS2-VASc, the patient is at increased risk
of bleed especially due to recent administration of IV-tPA and high NIHSS score.

Go to the next page if you knew the correct answer, or click the link image(s) below to further
research the concepts in this question (if desired).

Research Concepts:
Stroke Anticoagulation

We update eBooks quarterly and Apps daily based on user feedback. Please tap flag to
report any questions that need improvement.
Question 881: A 34-year-old diver suffered a thoracic fracture. The most common
complication is spinal shock. After ensuring airway, breathing, circulation, and resuscitation are
completed, what step should be taken to help prevent complications?

Choices:
1. Begin chemical deep vein thrombosis prophylaxis within the first 72 hours
2. Begin intermittent straight cath for volumes greater than 700 ml
3. Begin medications to slow bowel transit to prevent diarrhea
4. Begin skincare and turning at least every 6 hours
Answer: 1 - Begin chemical deep vein thrombosis prophylaxis within the first 72 hours
Explanations:
Lack of mobility puts those with spinal cord injured patients at a higher risk of developing a
deep vein thrombosis (DVT).
Clinically apparent DVT occurs in approximately 15% of patients with acute spinal cord
injury, and pulmonary embolism develops in approximately 5% of these patients.
The risk of venous thromboembolism rises sharply if treatment is delayed beyond 72 to 96
hours.
Deep vein thrombosis (DVT) is a common complication of traumatic spinal cord injury,
occurring in 50 to 100 percent of untreated patients.

Go to the next page if you knew the correct answer, or click the link image(s) below to further
research the concepts in this question (if desired).

Research Concepts:
Spinal Shock

We update eBooks quarterly and Apps daily based on user feedback. Please tap flag to
report any questions that need improvement.
Question 882: A 55-year-old is being evaluated after deteriorating in the intensive care
unit. He was admitted with aspiration pneumonia three weeks back and underwent elective
tracheostomy for anticipated prolonged ventilation. He was started on broad-spectrum antibiotics
and was demonstrating clinical and radiological signs of recovery. The patient’s condition
deteriorated this morning, and he developed extensive bleeding from the tracheostomy. The
nursing staff reported minor bleed while suctioning yesterday, but bleeding today has been
extensive. Tracheostomy suctioning shows bright red blood from the tracheostomy site. His
vitals are a blood pressure of 80/50 mmHg, a pulse of 135 beats per minute, and SpO2 80%.
Overinflating the tracheostomy cuff improves the bleeding. What is the definitive management in
this patient, given the underlying diagnosis?

Choices:
1. Bronchoscopy and cauterization
2. Percutaneous pulmonary artery thrombectomy
3. Arterial ligation and bypass grafting
4. Activated prothrombin complex concentrate transfusion
Answer: 3 - Arterial ligation and bypass grafting
Explanations:
This hospitalized patient has been in the intensive care unit for several weeks. He initially
improves on broad-spectrum antibiotics but deteriorates suddenly and develops bleeding
from tracheostomy. The previous history of minor bleed, the presence of fresh blood, and
improvement by over inflation of cuff suggests the development of trachea-innominate
artery fistula (TIAF).
TIAF is a potentially fatal disease if not diagnosed and managed timely. Definitive surgical
management includes either interposition grafting preserving flow in the innominate artery
or ligation of the artery and distal bypass grafting to ensure flow to the subclavian and
carotid artery.
Innominate artery ligation and bypass are generally preferred as it reduces the risk of
mortality and rebleeding. Catheterization and coiling of the fistula may be appropriate in
some conditions.
Prompt recognition and treatment are essential to reduce mortality in cases of TIAF.
Thrombectomy can be considered in pulmonary embolism. Cauterization and prothrombin
complex concentrate administration will be inappropriate in this patient.

Go to the next page if you knew the correct answer, or click the link image(s) below to further
research the concepts in this question (if desired).

Research Concepts:
Tracheo Innominate Artery Fistula

We update eBooks quarterly and Apps daily based on user feedback. Please tap flag to
report any questions that need improvement.
Question 883: What would you expect to see on prothrombin time (PT), partial
thromboplastin time (PTT), and bleeding time (BT) in a patient with vWF disease?

Choices:
1. Normal PT, prolonged PTT, prolonged BT
2. Normal PT, prolonged PTT, normal BT
3. Prolonged PT, prolonged PTT, prolonged BT
4. Prolonged PT, prolonged PTT, prolonged BT
Answer: 1 - Normal PT, prolonged PTT, prolonged BT
Explanations:
vWF disease will have normal PT, prolonged PTT, and prolonged BT. This is due to vWF
normally increasing the half-life of clotting factor VIII. vWF also binds to GpIb, and when
this does not occur, platelet dysfunction occurs, hence the prolonged BT.
vWF disease is the most common coagulopathy and often presents in the out-patient setting
with an adolescent female expressing concern about heavy menstruation.
Ristocetin agglutination assay induces platelet expression of GpIb. In vWF disease, you will
have impaired platelet aggregation due to vWF deficiency as vWF binds to GpIb.
Bernard-Soulier will have similar results on ristocetin agglutination assaying compared to
vWF disease. The difference between these two conditions in coagulation studies is that
vWF will also have prolonged PTT.

Go to the next page if you knew the correct answer, or click the link image(s) below to further
research the concepts in this question (if desired).

Research Concepts:
Von Willebrand Disease

We update eBooks quarterly and Apps daily based on user feedback. Please tap flag to
report any questions that need improvement.
Question 884: A 26-year-old male patient is being evaluated in the hospital for
progressively worsening kidney function. He was admitted five days ago and was diagnosed with
methicillin-resistant staphylococcus aureus infection. Intravenous vancomycin was immediately
started and has been adjusted as needed to maintain a level of 15 to 20 mcg/ml. His fever has
resolved, but his serum creatinine has gradually increased. His past medical history includes two
prior episodes of staphylococcal endocarditis treated with antibiotics. He has a history of
occasionally injecting heroin for the past four years. His only medication currently is
vancomycin. On physical examination, the temperature is 37.3 C (99.1 F), blood pressure is
110/70 mmHg, heart rate is 92/min, and respiratory rate is 18 breaths/min. His BMI is 22 kg/m2.
Cardiac examination reveals a soft diastolic murmur along the left sternal border and trace lower
extremity edema bilaterally. No skin rash or arthritis is seen. Labs show: C3 low, C4 normal,
creatinine 2.8 mg/dL from 1.5 mg/dL on admission, cryoglobulins negative, urinalysis- 3+ blood;
2+ protein; 30-40 erythrocytes per high power field (HPF); 10-15 leukocytes/HPF; and
erythrocyte casts. A transthoracic echocardiogram shows moderate aortic regurgitation without
vegetations. Renal ultrasound shows normal-sized, mildly echogenic kidneys. Doppler studies of
the renal arteries and veins were normal. What is the most appropriate management?

Choices:
1. Initiate glucocorticoids
2. Schedule a kidney biopsy
3. Switch vancomycin to daptomycin
4. Continue current therapy
Answer: 3 - Switch vancomycin to daptomycin
Explanations:
The differential diagnoses include infection-related glomerulonephritis, drug-induced
nephrotoxicity (from vancomycin), acute interstitial nephritis (AIN), and septic emboli.
Findings of nephritic urine sediment in a patient with an active infection in the presence of
therapeutic levels of vancomycin suggest infection-related glomerulonephritis rather than
other etiologies.
Glucocorticoids are not typically required for management of infection-related
glomerulonephritis as it typically improves with control of underlying infection. A kidney
biopsy is also not often required for accurate diagnosis in this setting.
Vancomycin-related tubular toxicity typically occurs after 7 to 10 days and usually has
bland urine sediment. Antibiotic-associated AIN is typically seen with beta-lactams and can
lead to leukocyte casts but not erythrocyte casts.

Go to the next page if you knew the correct answer, or click the link image(s) below to further
research the concepts in this question (if desired).

Research Concepts:
Acute Kidney Injury

We update eBooks quarterly and Apps daily based on user feedback. Please tap flag to
report any questions that need improvement.
Question 885: A 70-year-old woman is brought to the hospital with a fever. She had a
surgical procedure for the management of severe aortic stenosis 6 months ago. She is also noted
to have new nail infarcts. Peripheral blood cultures are positive for Streptococcus. A PICC line is
requested for long-term antibiotics. She takes warfarin daily. Which of the following is the
strongest contraindication for fluoroscopic PICC line placement in this patient?

Choices:
1. Raised INR
2. Age
3. Active infection
4. Mechanical valve surgery 6 months ago
Answer: 1 - Raised INR
Explanations:
Mechanical aortic valves require anticoagulation to a target INR of 2.5 (2.0 - 3.0).
Anticoagulation/raised INR >1.5 is a contraindication for PICC line placement, due to the
increased risk of hemorrhage.
Other contraindications to fluoroscopic PICC line placement include damage to the area of
skin to be cannulated, e.g., burns, trauma, contamination, and pregnancy (relative risk of
ionizing radiation to the fetus).
Following discussion with the local hematology department and using local trust policy
guidance, this patient should undergo briding low molecular weight heparin (LMWH)
treatment to facilitate a reduced INR, thus allowing the fluoroscopic PICC line placement
procedure on a scheduled, elective date.

Go to the next page if you knew the correct answer, or click the link image(s) below to further
research the concepts in this question (if desired).

Research Concepts:
Fluoroscopic PICC Line Placement

We update eBooks quarterly and Apps daily based on user feedback. Please tap flag to
report any questions that need improvement.
Question 886: A 46-year-old male patient with acute respiratory distress syndrome who is
mechanically ventilated (tidal volume 7 ml/kg, Continuous positive airway pressure (CPAP) 12.5
cm H2O, and a fraction of inspired oxygen (FIO2) 75% becomes hypoxic (SaO2 40%) and
hypotensive. Quick evaluation and physical examination of the patient is done, which shows
jugular venous distension and that the breath sounds are absent over the left lung field with right
tracheal deviation. Which of the following actions is most likely to be taken immediately in this
patient?

Choices:
1. Increase FIO2 to 100%, get a portable chest radiograph, and await results
2. Pull the endotracheal tube back 2 centimeters
3. Assess endotracheal tube position, increase FIO2 to 100%, suction the airway, obtain an
emergent chest radiograph, and consider emergent chest tube placement
4. Increase continuous positive airway pressure to 20 and start a dopamine drip
Answer: 3 - Assess endotracheal tube position, increase FIO2 to 100%, suction the airway,
obtain an emergent chest radiograph, and consider emergent chest tube placement

Explanations:
The most likely diagnosis is tension pneumothorax.
The practitioner should rule out intubation of the right mainstem bronchus, mucus plug
obstructing the left bronchus, and left hemothorax.
A left chest tube should be placed if oxygen saturation of arterial oxyhemoglobin and blood
pressure do not rapidly respond.
The patient will likely have rapid deterioration and cardiovascular compromise leading to
death if the tension pneumothorax is not recognized and corrected.

Go to the next page if you knew the correct answer, or click the link image(s) below to further
research the concepts in this question (if desired).

Research Concepts:
Tension Pneumothorax

We update eBooks quarterly and Apps daily based on user feedback. Please tap flag to
report any questions that need improvement.
Question 887: A 35-year-old male is rushed to the emergency department following a
motor vehicle collision. His presenting Glasgow Coma Scale score is 9. Urgent CT brain shows
diffuse cerebral edema with obliteration of all cisterns. Which of the following is the most
appropriate approach in the initial management of this patient's traumatic brain injury?

Choices:
1. Methylprednisolone
2. Hypertonic saline
3. Furosemide
4. Mannitol
Answer: 2 - Hypertonic saline
Explanations:
Hypertonic saline is the mainstay of medically managing traumatic intracranial
hypertension.
It has been shown to effectively reduce intracranial pressure and improve cerebral and
systemic perfusion.
It is safe, has a fast onset, sustained, and protracted action with minimal morbidities.
Mannitol has a risk of acute renal failure and rebound intracranial hypertension.

Go to the next page if you knew the correct answer, or click the link image(s) below to further
research the concepts in this question (if desired).

Research Concepts:
Cerebral Edema

We update eBooks quarterly and Apps daily based on user feedback. Please tap flag to
report any questions that need improvement.
Question 888: A 17-year-old male was brought to an emergency department after a
motorbike accident. On arrival, his blood pressure was 98/62 mmHg, pulse was 124 bpm,
respiratory rate was 11 breaths/min, and oxygen saturation was 84% at room air. His Glasgow
coma scale (GCS) score was 7/15. On physical examination, there is purplish discoloration of his
left tympanic membrane with bloody otorrhea. There was bruising of both eyes with tarsal plate
sparing and behind the left ear. Left ear drainage is also noted, and fluid is mixed with blood.
After initial management in the emergency department, a CT-scan of the head was taken, which
reveal pneumocephalus. What emergency intervention could have lead to this radiological
finding?

Choices:
1. Nasogastric tube intubation
2. Endotracheal tube intubation
3. Nasal intermittent positive pressure ventilation
4. Air embolization via a central line
Answer: 3 - Nasal intermittent positive pressure ventilation
Explanations:
Hemotympanum usually appears within hours of injury and maybe the earliest clinical
finding. Periorbital ecchymosis (raccoon eyes) is typically not present during the initial
evaluation and is delayed by 1 to 3 days. If bilateral, this finding is highly predictive of a
basilar skull fracture. Retroauricular or mastoid ecchymosis (Battle sign) is highly
predictive of basilar skull fracture.
Nasal intermittent positive pressure ventilation (NIPPV) should be avoided as it may induce
pneumocephalus.
Basilar skull fractures are usually due to significant trauma. A thorough trauma evaluation
with interventions to stabilize the airway, ventilation and circulatory issues is the priority.
Associated cervical spine injury is common, so attention to cervical spine immobilization,
particularly during airway management is necessary.
Nasogastric tubes and nasotracheal intubation should be avoided because of the risk for
inadvertent intracranial tube placement.

Go to the next page if you knew the correct answer, or click the link image(s) below to further
research the concepts in this question (if desired).

Research Concepts:
Basilar Skull Fractures

We update eBooks quarterly and Apps daily based on user feedback. Please tap flag to
report any questions that need improvement.
Question 889: A 42-year-old male was brought to an emergency department via ambulance
following an alleged physical assault. He is lying on a stretcher with a cervical collar around his
neck. At first glance, the health care provider noticed bruising to both eyes and behind the left
ear. Left ear drainage is noted, and fluid is mixed with blood. He is resting with eyes closed with
vomit in his mouth. His skin color is pale, and respirations are shallow. The Glasgow coma scale
(GCS) score is 6/15. The paramedic states that the patient vomited as they arrived in the
ambulance bay and became unconscious. The paramedic also relates that the patient was
unconscious on the scene, then became lucid and complained of a headache, and now is
unconscious again. What will be most likely finding on a CT-scan of the head?

Choices:
1. Oblique fracture of the left temporal bone along with a lentiform hyperdense area
2. Transverse fracture of the temporal bone along with crescent-shaped homogeneously
hyperdense area
3. Discontinuity of the frontal bone along with a an osseous defect in the cribriform plate
4. There will be no abnormal finding on a CT-scan
Answer: 1 - Oblique fracture of the left temporal bone along with a lentiform hyperdense
area

Explanations:
Hemotympanum: Fractures that involve the petrous ridge of the temporal bone will cause
blood to pool behind the tympanic membrane causing it to appear purple. This usually
appears within hours of injury and maybe the earliest clinical finding.
Basilar skull fractures are often associated with other CNS pathology like epidural
hematoma due to the weakness of the temporal bone and the close proximity of the middle
meningeal artery.
The epidural or extradural hematoma will show a lentiform hyperdense area on a CT-scan.
Patients with intracranial hemorrhage require emergent neurosurgical evaluation.
Otherwise, skull base fractures are often managed expectantly. Surgical management is
necessary for cases complicated by intracranial bleeding requiring decompression, vascular
injury, significant cranial nerve injury, or persistent cerebrospinal fluid leak.

Go to the next page if you knew the correct answer, or click the link image(s) below to further
research the concepts in this question (if desired).

Research Concepts:
Basilar Skull Fractures

We update eBooks quarterly and Apps daily based on user feedback. Please tap flag to
report any questions that need improvement.
Question 890: A 76-year-old woman presents to the emergency department with weakness,
fever, and confusion. She has a past medical history of chronic kidney disease requiring dialysis,
hypertension, and osteoarthritis. She has a pacemaker. On examination, she has a temperature of
39 C, and there is a new systolic ejection murmur noted at the right upper sternal border.
Conjunctival hemorrhages are noted on examination. Two blood cultures are positive for
viridans streptococci. Which of the following additional dermatologic finding is most likely to be
found in this patient?

Choices:
1. Warm, erythematous, painful plaques on the lower legs
2. Hemorrhagic, non-tender macules on the thenar eminences
3. Erythematous pustules on the chest
4. Scattered, brown patches with a waxy surface on the back
Answer: 2 - Hemorrhagic, non-tender macules on the thenar eminences
Explanations:
This patient’s presentation is infective endocarditis, which is associated with Janeway
lesions. Janeway lesions are typically described as non-tender, erythematous, hemorrhagic
macules commonly found on the palms and soles.
Janeway lesions are considered a vascular phenomenon, and minor criteria, of infective
endocarditis as per the Modified Duke criteria.
Janeway lesions may or may not culture positive for micro-organisms on biopsy.
The next step in treating this patient will be urgent intravenous antibiotics. An urgent
echocardiogram should also be ordered to confirm the diagnosis.

Go to the next page if you knew the correct answer, or click the link image(s) below to further
research the concepts in this question (if desired).

Research Concepts:
Osler Node and Janeway Lesions

We update eBooks quarterly and Apps daily based on user feedback. Please tap flag to
report any questions that need improvement.
Question 891: A 67-year-old man is brought to the hospital with difficulty breathing for 4
hours and not subsided by the repeated use of inhalers. 3 days ago, he had developed a mild fever
with a runny nose, which increased his frequency of coughing over time. The patient has a
history of smoking for the past 30 years, which he quit after the diagnosis of COPD about 2
years ago. He was started on medications that he claims he is not adherent to. He is given IV
hydrocortisone, noninvasive ventilation (NIV), and is transferred to the intensive care unit (ICU)
for continual management. Antibiotics are also given due to unrelenting fever. He mentions that
he is 'reluctant to be intubated and would rather die than having a tube down his throat.' His vital
signs show no improvement despite NIV, and a blood picture demonstrating rising respiratory
acidosis. Which of the following is the next best step in the management of this patient?

Choices:
1. Midazolam and invasive mechanical ventilation (IMV)
2. Conservative management
3. Extracorporeal carbon dioxide removal
4. Obtain consent for IMV from the patient's next of kin
Answer: 3 - Extracorporeal carbon dioxide removal
Explanations:
Extracorporeal carbon dioxide removal (ECCO2R) is a novel strategy used in patients with
non-invasive ventilation (NIV) failure despite 'ultra-protective' settings. Its use has
demonstrated a steady improvement of prognosis over the years and reduces the length of
hospital stay. It also allows for better lung recovery by reducing the work required by the
respiratory system for carbon dioxide removal. Of the possible strategies, invasive
mechanical ventilation would be discarded due to patient preference, whereas ECCO2R can
be considered.
ECCO2R can be best used for patients not desiring invasive mechanical ventilation or as to
bridge into IMV. The use of ECCO2R has demonstrated a 56% reduction in intubation
rates.
In conditions using IMV and ECCO2R as an adjunct to it, it is possible to allow earlier
extubation.
Sedating the patient with the intent to provide IMV despite his reluctance would be lawfully
unethical. The patient is having an acute exacerbation of COPD, which began the
downward spiral. Appropriate intervention is necessary before culminating in
decompensated respiratory failure. Although IMV is highly beneficial in this case and is a
valid option, it would be again unethical to move against the patient's choices with
negotiating with relatives.

Go to the next page if you knew the correct answer, or click the link image(s) below to further
research the concepts in this question (if desired).

Research Concepts:
Extracorporeal Carbon Dioxide Removal

We update eBooks quarterly and Apps daily based on user feedback. Please tap flag to
report any questions that need improvement.
Question 892: A 79-year-old woman with a past medical history of long-standing
hypertension, smoking, and coronary artery disease is brought to the emergency department by
the caregivers for sudden onset drooling, difficulty chewing, and dysphonia. She has an
abnormal expression of emotions and cries frequently. On examination, she has difficulty in
tongue protrusion and exaggerated facial reflexes. What is the most likely cause of these
symptoms in this patient?

Choices:
1. Progressive supranuclear palsy
2. Posterior fossa tumor
3. A previous episode of stroke
4. Bilateral thalamic infarction
Answer: 3 - A previous episode of stroke
Explanations:
Pseudobulbar palsy is due to an upper motor lesion caused by bilateral disturbance of the
corticobulbar tracts. The corticobulbar tracts exert supranuclear control over brainstem
motor nuclei and are involved in the muscular movement of the head and neck.
Consecutive lacunar strokes can involve both corticobulbar tracts and cause pseudobulbar
palsy. Complete history including any previous history of stroke is required. Pseudobulbar
palsy is one of the severe complications of cerebrovascular diseases.
Many pathological conditions can lead to pseudobulbar palsy. These include traumatic brain
injury, neoplasm, or metabolic abnormality, and neurological disorders, such as
amyotrophic lateral sclerosis and Parkinson disease. Other causes include post-stroke
pseudobulbar palsy, demyelinating conditions such as multiple sclerosis or congenital
malformations. Bilateral thalamic infarction is extremely rare.
Patients often have dysphagia, drooling, dysphonia, dysarthria. They have spontaneous or
inappropriate emotional outbursts. There is a brisk jaw jerk. Other facial reflexes also
become exaggerated. There is paresis of the tongue and facial muscles.

Go to the next page if you knew the correct answer, or click the link image(s) below to further
research the concepts in this question (if desired).

Research Concepts:
Pseudobulbar Palsy

We update eBooks quarterly and Apps daily based on user feedback. Please tap flag to
report any questions that need improvement.
Question 893: A 65-year-old man with a past medical history of hypertension and aortic
regurgitation who underwent surgical replacement of aortic valve 2 weeks ago presents to the
emergency department few days after being discharged with a chief complaint of fever, severe
body weakness and mild shortness of breath. His vital signs show blood pressure 112/64 mmHg,
pulse rate 98/min, temperature 102.6 F, and respiratory rate 22/min. On physical exam, he is ill-
appearing, in mild respiratory distress and has a grade IV systolic murmur. The patient is
admitted to the hospital, empiric antibiotic treatment is started, and blood cultures are obtained
before the initiation of treatment. Which of the following organisms is most likely to grow on the
patient's blood cultures?

Choices:
1. Streptococcus viridans
2. Enterococcus faecium
3. Staphylococcus aureus
4. Candida albicans
Answer: 3 - Staphylococcus aureus
Explanations:
Staphylococcus is the most common microorganism to cause prosthetic valve endocarditis
in the first 2 months after the valvular surgery, followed by streptococcus.
Staphylococcus induced endocarditis is associated with high morbidity and mortality.
Staphylococcus aureus endocarditis is associated with a higher need for surgical treatment
of endocarditis.
Enterococcus faecium and Candida albicans are less likely to cause prosthetic valve
endocarditis in the first 2 months after the valve replacement.

Go to the next page if you knew the correct answer, or click the link image(s) below to further
research the concepts in this question (if desired).

Research Concepts:
Prosthetic Valve Endocarditis

We update eBooks quarterly and Apps daily based on user feedback. Please tap flag to
report any questions that need improvement.
Question 894: A 58-year-old man with a past medical history of diabetes mellitus,
hypertension, and alcohol use disorder is admitted to the hospital with a grade 2 subarachnoid
hemorrhage (Hunt and Hess scale). 5 days post-endovascular treatment of the aneurysm, the
patient starts complaining of muscle cramps, nausea, and headache. His skin turgor is normal.
CT imaging of the brain does not show new findings. A current set of labs is shown below.
Patient value Reference range
Sodium 130 mEq/L 134-144 mEq/L
Potassium 4 mEq/L 3.6-5.0 mEq/L
Chloride 98 mEq/L 98-107 mEq/L
Bicarbonate 26 mEq/L 21-28 mEq/L
Treatment with fluid restriction in this patient is most likely to increase the risk of which of the
following complications?

Choices:
1. Vasospasm
2. Rebleeding
3. Seizures
4. Cerebral herniation
Answer: 1 - Vasospasm
Explanations:
This scenario describes a case of hyponatremia following subarachnoid hemorrhage (SAH).
Hyponatremia following SAH is due to increased secretion of anti-diuretic hormone (ADH)
in a majority of the cases. If the hyponatremia is due to cerebral salt wasting, there is clear
evidence of hypovolemia that presents in the form of hypotension or decreased skin turgor.
Hyponatremia, here, is best treated with hypertonic saline (3%). Some clinicians prefer
isotonic normal saline if they feel hyponatremia is due to CSW alone.
Fluid restriction leads to hypovolemia, which aggravates the risk of cerebral vasospasm.
Fluid restriction should be avoided.

Go to the next page if you knew the correct answer, or click the link image(s) below to further
research the concepts in this question (if desired).

Research Concepts:
Subarachnoid Hemorrhage

We update eBooks quarterly and Apps daily based on user feedback. Please tap flag to
report any questions that need improvement.
Question 895: An 87-year-old man is brought to the emergency department from a nursing
home with 1-day duration of diaphoresis, agitation, delirium, disorientation, hyperthermia,
nausea, loss of appetite, and tremor. The patient has a history of major depressive disorder,
hyperlipidemia, prostate cancer, and chemotherapy-induced nausea and vomiting (CINV) for
which he takes ondansetron. His medication list is not available. Which of the following clinical
criteria is most appropriate to use for the diagnosis of this patient?

Choices:
1. McDonald diagnostic criteria
2. SLICC classification criteria
3. Modified Duke criteria
4. Hunter criteria
Answer: 4 - Hunter criteria
Explanations:
Serotonin syndrome is associated with increased serotonergic neurotransmission. Many
drugs can precipitate serotonin toxicity, including but not limited to fentanyl, tramadol,
metoclopramide, ondansetron, lithium, buspirone, levodopa, SSRIs, SNRIs and linezolid.
The diagnosis of serotonin syndrome is made using the Hunter serotonin toxicity criteria.
The patient must be on a serotonergic drug and exhibit one of the following: spontaneous
clonus, inducible clonus with agitation or diaphoresis, ocular clonus with agitation or
diaphoresis, tremor with hyperreflexia, and hypertonia with temperature > 100.4 F and
ocular or inducible clonus.
Differential diagnosis includes neuroleptic malignant syndrome (4-14 days after the start of
a neuroleptic drug), malignant hyperthermia (during operation or early postoperative
period), anticholinergic toxicity, meningitis, and encephalitis.
Management involves discontinuation of all serotonergic drugs, supportive care,
benzodiazepines (agitation), and serotonin antagonists (e.g., cyproheptadine). Hyperthermic
patients (>105 F) will require administration of neuromuscular blocking agents (NMBA)
and endotracheal intubation.

Go to the next page if you knew the correct answer, or click the link image(s) below to further
research the concepts in this question (if desired).

Research Concepts:
Drug Interactions In Palliative Care

We update eBooks quarterly and Apps daily based on user feedback. Please tap flag to
report any questions that need improvement.
Question 896: A 65-year-old woman with sepsis secondary to pyelonephritis is admitted to
an intensive care unit. The lab report shows the growth of Proteus mirabilis carrying an inosine
monophosphate (IMP) carbapenemase on a nucleic acid amplification test (NAAT). The patient
is intubated and sedated. She has a persistent fever for which she was started on meropenem
empirically three days ago but her clinical status has worsened. Her complete blood count shows
a white cell count of 22,000. What is the most appropriate antibiotic regimen for this patient?

Choices:
1. Add ciprofloxacin to meropenem
2. Piperacillin/tazobactam + gentamicin
3. Colistin + ceftazidime-avibactam
4. Tigecycline + gentamicin
Answer: 2 - Piperacillin/tazobactam + gentamicin
Explanations:
Proteus mirabilis has intrinsic resistance to colistin. It is also resistant to tigecycline and
often to quinolones. Piperacillin/tazobactam and gentamicin should be considered.
Susceptibility testing should guide therapy.
Gentamicin is the best second-line treatment and rifampin can be used at third-line
treatment.
Using combination therapy of two or more antibiotics is associated with improved outcomes
in carbapenem-resistant infections.
Monotherapy with meropenem-voribactam has less efficacy than dual or triple therapy.
Combination therapy with colistin and ceftazidime-avibactam may be useful, but these are
not considered integral to an antibiotic regimen. Since the patient has had worsening of
symptoms on meropenem and the isolate has shown carbapenemase activity, therapy must
be switched.

Go to the next page if you knew the correct answer, or click the link image(s) below to further
research the concepts in this question (if desired).

Research Concepts:
Carbapenem Resistant Enterobacteriaceae

We update eBooks quarterly and Apps daily based on user feedback. Please tap flag to
report any questions that need improvement.
Question 897: A 42-year-old male patient was referred to the emergency department(ED)
following an episode of massive upper gastrointestinal bleeding. His vital signs on arrival to the
ED were blood pressure: 66/40 mmHg, pulse rate: 130 beats per minute, respiratory rate: 19/min,
and normal temperature. his past medical history is significant for 10-year cirrhosis due to
hepatitis C infection. His laboratory tests were WBC: 9800 cells/mm^3, Hb: 5 m/dl, platelet:
90000 cells/microliter. Four units packed cells were prepared for transfusion. Which one of the
following is among the next step management?

Choices:
1. Administration 1 gram intravenous ceftriaxone
2. Administration 4 units recombinant factor VII
3. Application of Sengstaken Blakemore tube in the ED
4. Request an emergent upper GI endoscopy
Answer: 1 - Administration 1 gram intravenous ceftriaxone
Explanations:
The management of upper GI bleeding initiates with resuscitation. Several endpoints have
been described to establish an adequate resuscitation, however, urine out of equal or more
than 0.5 cc/kg/hour in the adult population and lowering the lactate and base deficit level
might be utilized in the vast majority of institutes. The resuscitation process should initiate
with isotonic serum therapy and followed by blood products in a timely manner.
The management of upper GI bleeding due to esophageal varices should include
prophylactic antibiotic therapy to prevent the possibility of encephalopathy.
The efficacy of the resuscitation process should be evaluated frequently to prevent
overcorrection as it might disturb the formed clot and aggravate the bleeding.
The Sengstaken tube application demands ICU admission, induction of general anesthesia,
and orotracheal intubation. Moreover, it might be utilized for less than 36 hours. The tube
works with a tamponade mechanism and might result in severe necrosis during prolonged
use. Although several studies reviewed the possible beneficial role of recombinant factor
VII, it has not been established yet.

Go to the next page if you knew the correct answer, or click the link image(s) below to further
research the concepts in this question (if desired).

Research Concepts:
Esophageal Varices

We update eBooks quarterly and Apps daily based on user feedback. Please tap flag to
report any questions that need improvement.
Question 898: A 72-year-old man with a history of diastolic congestive heart failure is
brought to the hospital with fever, dry cough, anosmia, ageusia, and dyspnea for 4 days. His wife
had similar symptoms one week ago. On exam, he has a temperature of 100 F, heart rate 98/min,
respiratory rate 20/min, blood pressure 134/80 mmHg, and pulse oximetry of 95% on room air.
The respiratory exam shows diminished air entry bilaterally, and the rest of the exam is
unremarkable. His rapid SARS CoV-2 test is positive. His CT chest shows mild ground-glass
opacities bilaterally in both lungs. Which of the following is the next best step in the
management of this patient?

Choices:
1. IV bamlanimuvab only
2. IV bamlanimuvab and etesevimab
3. IV dexamethasone only
4. Oral azithromycin only
Answer: 2 - IV bamlanimuvab and etesevimab
Explanations:
Emergency use authorization for bamlanimuvab alone was revoked due to the increased
frequency of resistant variants.
Combination therapy of anti-SARS-CoV-2 antibodies such as bamlanimuvab plus
etesevimab or casirivimab plus imdevimab is given emergency use authorization for use in
patients with COVID-19 who are not hospitalized for COVID-19 and not requiring oxygen.
Corticosteroids such as dexamethasone are not indicated in patients with mild to moderate
COVID-19 who are not requiring oxygen.
Azithromycin and other antibacterial therapy are not recommended for patients with
COVID-19 who do not have evidence of concurrent or superimposed infection.

Go to the next page if you knew the correct answer, or click the link image(s) below to further
research the concepts in this question (if desired).

Research Concepts:
Best Practices For Administering Monoclonal Antibody Therapy For
Coronavirus (COVID-19)

We update eBooks quarterly and Apps daily based on user feedback. Please tap flag to
report any questions that need improvement.
Question 899: A workman is assigned to clean the outside entrance to an office complex
where a suspicious letter with a white powder in it had been discovered the day before. He wears
a paper facemask and latex gloves. Two of the cleaning fluids he is using are ammonia and
bleach. After mixing these two agents to save time, he begins pressure washing the sidewalk and
an enclosed alcove at the front entrance. After only a few minutes he begins to complain of
burning in his eyes and throat and develops shortness of breath. He is taken by ambulance to a
hospital, where the administration of albuterol via nebulization fails to improve his pulmonary
symptoms. Which of the following causes of this scenario is the most likely?

Choices:
1. There was anthrax in the letter and the patient is suffering from inhalational pulmonary
anthrax
2. The patient has occupational asthma due to the cleaning solution
3. The patient is experiencing a latex allergy to the gloves
4. The mixture of ammonia and hypochlorite generated chloramine gas, a highly water-soluble
pulmonary irritant.
Answer: 4 - The mixture of ammonia and hypochlorite generated chloramine gas, a highly
water-soluble pulmonary irritant.

Explanations:
Bleach (hypochlorite) and ammonia react generating a highly irritant gas causing immediate
pulmonary symptoms unresponsive to usual asthma medications.
The bleach decomposes to form hydrochloric acid. It reacts with ammonia to form toxic
chloramine fumes.
This reaction was used during WWI as a weapon.
Latex is more likely to cause full anaphylaxis. Occupational asthma should respond to
albuterol. Anthrax would take several days to show the onset of symptoms.

Go to the next page if you knew the correct answer, or click the link image(s) below to further
research the concepts in this question (if desired).

Research Concepts:
Chlorine Gas Toxicity

We update eBooks quarterly and Apps daily based on user feedback. Please tap flag to
report any questions that need improvement.
Question 900: A 45-year-old woman is admitted to the ICU with suspicion of severe acute
pancreatitis. The critical care team decides to asses the severity of the disease using the
APACHE-II scoring system instead of Ranson criteria. Which of the following factors best
explains why the use of Ranson criteria is less preferable in emergency settings?

Choices:
1. Fewer data available for Ranson criteria
2. Time required to fulfill the criteria
3. History of previous similar episodes
4. Etiology of pancreatitis
Answer: 2 - Time required to fulfill the criteria
Explanations:
Ranson criteria are used to predict the severity and mortality of acute pancreatitis. There are
eleven parameters for assessment.
Five parameters are assessed on admission, and the other six are assessed at 48 hours post
admission. The score and severity of acute pancreatitis cannot be determined completely
until 48 hours have passed since admission. This limits its utility in time-sensitive situations
like the emergency department
One point is given for each positive parameter for a maximum score of 11. A score of 3 or
greater predicts severe acute pancreatitis and possible mortality.
APACHE-II and the Bedside Index of Severity in Acute Pancreatitis (BISAP) are other
scoring systems that can be used at any time and is utilized by emergency medicine
physicians.

Go to the next page if you knew the correct answer, or click the link image(s) below to further
research the concepts in this question (if desired).

Research Concepts:
Ranson Criteria

We update eBooks quarterly and Apps daily based on user feedback. Please tap flag to
report any questions that need improvement.
Section 10
Question 901: A rapid response is called for a patient due to acute hypoxic hypercapnic
respiratory failure. The patient is a 67-year-old male with a past medical history of obesity
hypoventilation syndrome and chronic obstructive pulmonary disorder. The nurse reports that
over the past one hour, the patient has become increasingly lethargic, and his arterial blood gas
(ABG) analysis revealed the patient's pCO2 went from 65 mm Hg to 100 mm Hg. The patient
was prepared to intubate. Upon review of his airway, there is a Mallampati score of 4, and the
patient has a submental distance of 1 finger length. What would be the most critical step to
ensure successful intubation in this patient?

Choices:
1. Use a tracheal tube inducer
2. Extend lower cervical spine, and flex upper cervical spine
3. Flex lower cervical spine and extend upper cervical spine
4. Attempt a surgical airway
Answer: 2 - Extend lower cervical spine, and flex upper cervical spine
Explanations:
Positioning the patient correctly will help mitigate a difficult airway. Positioning the patient
with C6-C7 flexed and C1-C2 extended will allow the clinician the best chance of
visualizing the glottis in this situation. This position is called the "sniffing position."
The epiglottis serves as the primary landmark that will direct successful endotracheal
intubation. The origin of the epiglottis can be found at the base of the tongue. The valley
between the tongue and the epiglottis is called the vallecula.
At least two to three medical professionals are required to manage the airway for rapid
sequence induction. A clear delineation of roles is necessary for successful endotracheal
intubation. The lead clinician is responsible for directing the team, managing the patient's
airway, and intubating the patient. A nursing staff member is responsible for providing
induction and paralytic medications at the onset of the procedure.
At this point in time, a surgical airway is not necessary.

Go to the next page if you knew the correct answer, or click the link image(s) below to further
research the concepts in this question (if desired).

Research Concepts:
Tracheal Rapid Sequence Intubation

We update eBooks quarterly and Apps daily based on user feedback. Please tap flag to
report any questions that need improvement.
Question 902: A 65-year-old male patient is brought to the emergency department after an
episode of syncope. He reports no prodromal symptoms before the episode and no involuntary
loss of urine or tongue bite after the episode. The patient’s wife mentions that he did not
experience any generalized body movements during the episode. He mentions that he has not
experienced this before. He reports a history of episodic, substernal chest pain that lasts for 5 to
10 minutes each time and resolves with rest. He reports an upper respiratory tract infection a
week ago that resolved with two tablets of over-the-counter antipyretic. The patient has a history
of severe hypertension and gastroesophageal reflux disease, for which he takes lisinopril,
chlorthalidone, and famotidine. He does not smoke or drink alcohol. Vitals show a blood
pressure of 158/92 mm Hg, with no change noted on inspiration, a pulse of 72/min, respiratory
rate of 14/min, and a temperature of 98.6 F (37 C). Physical examination reveals distended neck
veins. Cardiac auscultation reveals distant heart sounds and an ejection systolic murmur heard on
the right second intercostal space. Pulmonary examination reveals clear lungs bilaterally.
Electrocardiogram reveals varying QRS amplitudes from beat to beat. A diagnosis is made, and
appropriate treatment is begun. What is the cause of the absence of a change in blood pressure on
inspiration?

Choices:
1. Valvular abnormality
2. Old age
3. History of angina
4. Medication side effect
Answer: 1 - Valvular abnormality
Explanations:
The most likely diagnosis is cardiac tamponade. The patient’s history of an upper
respiratory tract infection, current presentation, distant heart sounds, and distended neck
veins on examination and findings on the electrocardiogram indicate the diagnosis.
This patient also has aortic stenosis, as suggested by the patient’s history, and ejection
systolic murmur heard on the right second intercostal space.
Pulsus paradoxus refers to an exaggerated fall in a patient’s blood pressure during
inspiration by greater than 10 mm Hg and is frequently seen in cardiac tamponade.
Cardiac tamponade without pulsus paradoxus can be seen in aortic stenosis and
hypertension. Any coexisting disease that significantly elevates left ventricular diastolic
pressure would contribute to this finding. Old age, history of angina, and side effects of
medications that the patient takes do not contribute to this finding.

Go to the next page if you knew the correct answer, or click the link image(s) below to further
research the concepts in this question (if desired).

Research Concepts:
Pulsus Paradoxus

We update eBooks quarterly and Apps daily based on user feedback. Please tap flag to
report any questions that need improvement.
Question 903: A 25-year-old man presents to the hospital with fever, headache, and cough
for 5 days. His chest x-ray is suggestive of atypical pneumonia, and the microbiology lab
confirms the infection with gram-negative bacteria. Serology confirms the diagnosis and
doxycycline is used for treatment. The patient wishes to leave against medical advice, but he is
counseled about possible complications like osteomyelitis, endocarditis, and pseudotumors of
lungs and spleen if this infection is not treated properly. Working at which of the following
places is the strongest risk factor for acquiring this disease?

Choices:
1. Animal farm
2. Trekking industry
3. Fishing industry
4. Ship manufacturing factory
Answer: 1 - Animal farm
Explanations:
Q fever is usually seen among the workers of animal farms.
Chronic manifestations of Q fever include endocarditis, osteomyelitis, and pseudotumors of
lungs and spleen.
The diagnosis of Q fever is made with serology.
Doxycycline is the treatment of choice for Q fever.

Go to the next page if you knew the correct answer, or click the link image(s) below to further
research the concepts in this question (if desired).

Research Concepts:
Coxiella Burnetii

We update eBooks quarterly and Apps daily based on user feedback. Please tap flag to
report any questions that need improvement.
Question 904: A 35-year-old woman with a history of lupus nephritis who was on
tacrolimus presented to the emergency department with a history of sudden onset headache,
vomiting, disorientation, and seizure. On examination, the patient was disoriented, with a pulse
of 64/min, and blood pressure of 220/120 mmHg. Her pupils were equal in size and reacted
normally to light. Once she was stabilized, she complained of homonymous hemianopsia. What
should be the next best step in the evaluation of this patient?

Choices:
1. Blood tacrolimus level estimation
2. MRI brain
3. Complement assay
4. EEG
Answer: 2 - MRI brain
Explanations:
This patient has PRES (posterior reversible encephalopathy syndrome).
Tacrolimus is a risk factor for PRES.
MRI brain is the diagnostic test of choice for PRES.
Complement assay helps in the diagnosis of a lupus flare. EEG helps in the diagnosis of
seizures.

Go to the next page if you knew the correct answer, or click the link image(s) below to further
research the concepts in this question (if desired).

Research Concepts:
Cortical Blindness

We update eBooks quarterly and Apps daily based on user feedback. Please tap flag to
report any questions that need improvement.
Question 905: A 42-year-old woman presents to the hospital for abdominal pain and fever.
She was brought to the emergency department two weeks ago, following abdominal trauma. Her
blood pressure is 118/68 mmHg, temeperature 100.4 F, and the pulse is 122/min. CBC reveals a
WBC of 15300/microL. During her prior visit, she required embolization of the short gastric
arteries for internal bleeding. A CT is ordered to confirm the diagnosis. What is the most
appropriate treatment strategy for this patient?

Choices:
1. Pain control
2. IV antibiotics
3. Ultrasound-guided drainage and IV antibiotics
4. Observation
Answer: 3 - Ultrasound-guided drainage and IV antibiotics
Explanations:
This patient underwent embolization of the short gastric arteries which feed part of the
spleen leading to splenic infarction.
A rare complication of a splenic infarct is the formation of a splenic abscess.
A splenic abscess will require IR drainage and IV antibiotics.
Pain management and IV antibiotics are both indicated but not the mainstay of treatment.

Go to the next page if you knew the correct answer, or click the link image(s) below to further
research the concepts in this question (if desired).

Research Concepts:
Splenic Infarcts

We update eBooks quarterly and Apps daily based on user feedback. Please tap flag to
report any questions that need improvement.
Question 906: A 63-year-old man with metastatic lung cancer is being evaluated in the
ICU. He is intubated for acute respiratory failure. The patient appears in respiratory distress and
is tachypneic on the respirator. He is on a continuous dose of fentanyl via the intravenous route.
On examination of the ventilator, the inhaled tidal volume is 200 mL higher than the exhaled
tidal volume. The respiratory rate as seen on the ventilator is 40/min. Which of the following is
the next best step in the management of this patient?

Choices:
1. Increase the dose of fentanyl
2. Push 2-3 mL of air in the endotracheal tube balloon
3. Intravenous rocuronium
4. Intravenous midazolam
Answer: 2 - Push 2-3 mL of air in the endotracheal tube balloon
Explanations:
Intubated patients should be evaluated for ventilator-patient asynchrony to mitigate pain.
A discrepancy in the inhaled and exhaled tidal volume can depict an air leak, and air must
be instilled into the endotracheal tube balloon to abolish the leak.
Patients who are mechanically ventilated should have respiratory therapists performing
frequent adjustments to the ventilator, changing the ventilatory circuit, and suction.
Pulmonary medicine should be involved in abolishing ventilator-patient synchrony, which
can cause severe distress.

Go to the next page if you knew the correct answer, or click the link image(s) below to further
research the concepts in this question (if desired).

Research Concepts:
End Of Life Evaluation And Management Of Pain

We update eBooks quarterly and Apps daily based on user feedback. Please tap flag to
report any questions that need improvement.
Question 907: A 67-year-old man presents to the hospital with substernal chest pain. He
has a history of hypertension and ischemic heart disease. He recently underwent percutaneous
coronary intervention (PCI) of the right coronary artery for >90% stenosis. EKG shows ST-
depressions in leads II, III, and aVF. Intravascular ultrasound shows stent under-expansion.
Which of the following conditions is most likely to have caused this complication?

Choices:
1. Thrombus
2. Calcified plaque
3. Lipidic plaque
4. Spontaneous coronary artery dissection (SCAD)
Answer: 2 - Calcified plaque
Explanations:
Calcified plaque, when severe, can be associated with stent under-expansion, defined as
failure of achieving adequate stent expansion.
This can be mitigated with lesion preparation techniques before stent implantation which
includes cutting balloons, scoring balloons, high-pressure balloon inflations, atherectomy
(rotational, orbital, or laser) and intravascular lithotripsy (IVL).
Thrombus is not associated with stent under-expansion. However, thrombus may be
associated with complications including distal embolization and slow/no-reflow.
Lipidic plaque is not closely associated with stent under-expansion. Lipidic plaque however
at the site of a stent landing zone can be associated with edge dissections. Spontaneous
coronary artery dissection (SCAD) is not associated with stent under-expansion and can
often be managed medically without stent implantation.

Go to the next page if you knew the correct answer, or click the link image(s) below to further
research the concepts in this question (if desired).

Research Concepts:
Calcified Plaque

We update eBooks quarterly and Apps daily based on user feedback. Please tap flag to
report any questions that need improvement.
Question 908: A 40-year-old patient with HIV/AIDS has not been on any therapy for 18
months due to economic hardship. Four months ago, he was treated successfully for avium
complex (MAC). For the past four days, he has had a fever, dry cough, and progressive dyspnea.
Vital signs show a temperature of 38.9 C, blood pressure of 88/58 mmHg, heart rate of 140/min,
and a respiration rate of 40 breaths per minute. Oxygen saturation is 80% on room air. The neck
shows bilateral lymphadenopathy, the lungs show bilateral rales, and the abdomen is non-tender.
Oxygen by a non-rebreather mask does not improve the condition of the patient. Which of the
following is appropriate management for this patient?

Choices:
1. Bronchoalveolar lavage for direct immunofluorescence
2. Rapid sequence intubation and intravenous fluids
3. Complete blood count, CD4+ count, viral load, and chest x-ray
4. Sputum culture, complete blood count, and chest x-ray
Answer: 2 - Rapid sequence intubation and intravenous fluids
Explanations:
A further clinical evaluation must wait as the condition of the patient is worsening.
The patient has impending respiratory failure most likely due to Pneumocystis jiroveci
pneumonia.
Arterial blood gas analysis will aid the provider in determining if steroids are required for a
patient with Pneumocystis pneumonia (PCP.)
Chest radiography may also be useful for cardiac presentations or for those patients who
need evaluation for pulmonary infections. If chest radiography does not reveal an obvious
pulmonary process and there is still concern for lung pathology computed tomography of
the chest may be a consideration for further evaluation.

Go to the next page if you knew the correct answer, or click the link image(s) below to further
research the concepts in this question (if desired).

Research Concepts:
Acquired Immune Deficiency Syndrome

We update eBooks quarterly and Apps daily based on user feedback. Please tap flag to
report any questions that need improvement.
Question 909: A 35-year-old man presents to the emergency department with complaints of
rigidity and difficulty walking. On examination, the patient has fine tremors and a shuffling gait.
On a fundoscopic examination of the patient, disc edema and hyperemia are seen. The patient
works at a factory that manufactures house paint. He says that he mistakenly consumed a liquid
that looked and smelled like alcohol six days ago. Which toxic metabolite of the liquid consumed
is most likely responsible for the patient's symptoms?

Choices:
1. Glycolaldehyde
2. Formic acid
3. Glycolic acid
4. Oxalic acid
Answer: 2 - Formic acid
Explanations:
Methanol is broken down by ADH into formaldehyde and subsequently converted to formic
acid by the action of ALDH.
Formic acid accumulates in the putamen of the brain and causes parkinsonian symptoms.
Formic acid also causes ocular toxicity, which can be seen on fundoscopy.
Glycoaldehyde is a breakdown product of ethylene glycol when acted upon by ADH in the
liver. Glycolic acid and oxalic acid are the final metabolites of ethylene glycol. Both oxalic
acid and glycolic acid are the end products of ethylene glycol metabolism. And the history
and exam findings of the patient, it's evident that he is a case of methanol poisoning.

Go to the next page if you knew the correct answer, or click the link image(s) below to further
research the concepts in this question (if desired).

Research Concepts:
Ethanol

We update eBooks quarterly and Apps daily based on user feedback. Please tap flag to
report any questions that need improvement.
Question 910: A 22-year-old woman has just returned from a backpacking trip across
Southeast Asia. She had taken loperamide for the last week for symptoms of traveler's diarrhea.
She now presents to the emergency department with severe abdominal pain, abdominal
distention, intractable vomiting, and an inability to pass gas. Which of the following is the most
likely underlying pathophysiologic process for the patient's symptoms?

Choices:
1. Delayed gastric emptying causing toxic megacolon
2. Mesenteric vein thrombosis causing ischemia
3. Parasite infection causing intestinal obstruction
4. Constipation from dehydration
Answer: 1 - Delayed gastric emptying causing toxic megacolon
Explanations:
The most common side effects of loperamide are dry mouth, abdominal cramps, and nausea.
Rare gastrointestinal side effects include toxic megacolon.
Toxic megacolon is a known but rare complication of loperamide.
The patient is unlikely to have mesenteric ischemia given her young age and lack of risk
factors for vascular disease. Parasitic infections, while possible, are more likely to cause
crampy abdominal pain and diarrhea.

Go to the next page if you knew the correct answer, or click the link image(s) below to further
research the concepts in this question (if desired).

Research Concepts:
Loperamide

We update eBooks quarterly and Apps daily based on user feedback. Please tap flag to
report any questions that need improvement.
Question 911: A 65-year-old man living in rural Ghana presents to the municipal hospital
with intermittent fever for three days, severe fatigue, shortness of breath, and decreased
urination. He reports a history of multiple bouts of P. falciparum malaria throughout his life,
including three episodes in the last year diagnosed at his local hospital. He has been treated
intermittently with quinine sulfate over the years. In the emergency department, his temperature
is 104.5 F, blood pressure is 150/92 mmHg, heart rate is 124 bpm, and respiratory rate is 22
breaths/min with an O2 saturation of 99%. His exam is significant for scleral icterus,
conjunctival pallor, jaundice, a flow murmur on cardiac auscultation, and diffuse abdominal
tenderness. CBC is significant for a hemoglobin of 5.4 and a platelet count of 112,000. His
creatinine is 3.0, potassium 5.8, bilirubin 4.6, and direct bilirubin is less than 0.2. Urine collected
via straight catheter is dark; urinalysis is pending. The blood smear is significant for ring forms
of P. falciparum and fragmented red blood cells. The pathophysiology occurring in this patient
will most likely lead to which of the following?

Choices:
1. Hemoglobinuria and subsequent renal failure
2. Septic shock
3. Pulmonary edema and subsequent acute respiratory distress syndrome
4. Fatty liver infiltration
Answer: 1 - Hemoglobinuria and subsequent renal failure
Explanations:
Blackwater fever is severe anemia with hemoglobinuria and renal failure in the context of
massive intravascular hemolysis; this patient is displaying signs of Blackwater fever via
evidence of hemolysis and renal failure, given his decreased urine output, dark urine
concerning for hemoglobinuria, elevated creatinine, indirect hyperbilirubinemia, and
profound anemia. Blackwater fever is most often seen in the setting of repeat P. falciparum
infections treated with chronic quinine; it is rare and thought to be associated with G6PD
deficiency.
Western Africa has high rates of P. falciparum malaria, which has the highest morbidity and
mortality of the Plasmodia species.
Severe anemia in malaria stems from TNF-alpha-mediated mechanisms involving both
increased destruction and decreased the production of erythrocytes including cell lysis as
parasites replicate and exit erythrocytes; splenic removal and autoimmune lysis of immune
marked erythrocytes; poor iron incorporation into new heme molecules; and bone marrow
suppression during severe infection leading to decreased production.
In patients with malaria, complete blood count reveals thrombocytopenia in 60-70% of all
cases and varying degrees of anemia in 29% of adults and 78% of children.

Go to the next page if you knew the correct answer, or click the link image(s) below to further
research the concepts in this question (if desired).

Research Concepts:
Malaria

We update eBooks quarterly and Apps daily based on user feedback. Please tap flag to
report any questions that need improvement.
Question 912: A 54-year-old lady presents to the emergency department with complaints of
abdominal discomfort and vomiting of 2 days duration. On examination, she has a heart rate of
120 beats/min, blood pressure of 100/60 mmHg, and peripheries are cold and clammy. The
abdominal examination reveals the right hypochondriac tenderness. CT abdomen was done,
which revealed a perforated duodenum with a subdiaphragmatic collection. She was taken up for
exploratory laparotomy, intraoperatively she required multiple vasopressors and she was shifted
to ICU past Durham's patch closure of the perforation. Over the next 4 days, she was weaned off
vasopressors support and mechanical ventilation. However, on a postoperative day 7, she had a
new fever spike (102 F) and hypotension (80/60 mm Hg) in spite of fluid resuscitation and hence
started on vasopressor support. Blood cultures revealed branching yeast. What is the medication
of choice?

Choices:
1. Fluconazole
2. Amphotericin B
3. Anidulafungin
4. Flucytosine
Answer: 3 - Anidulafungin
Explanations:
Echinocandins (eg. anidulafungin) is the antifungal of choice in critically ill patients with
fungal septicemia.
Candida is a commensal in the upper gastrointestinal tract but may result in life-threatening
sepsis in immunocompromised patients.
The duration of therapy is 14 days after the last negative blood culture. It is recommended
to send blood cultures 48 hours after starting therapy.
Once the patient has stabilized azoles, or other antifungals can be initiated according to
culture sensitivity.

Go to the next page if you knew the correct answer, or click the link image(s) below to further
research the concepts in this question (if desired).

Research Concepts:
Antifungal Antibiotics

We update eBooks quarterly and Apps daily based on user feedback. Please tap flag to
report any questions that need improvement.
Question 913: A 17-year-old male patient has been brought to the emergency department
with acute confusion. The patient was at a house party, and his friends described that he was
drinking alcohol and might have taken other drugs. He also has a history of panic disorder and
takes medications for it. They are unable to provide any further information on the patient’s
medical history. His blood pressure is 134/82 mmHg, heart rate is 116 beats per minute,
respiratory rate is 18 breaths/min, and the temperature is 37.2 C (98.9F). He is lethargic, oriented
to time and name, but not to place. His physical examination reveals dry mucus membranes,
pupils are 4 mm, equal in size, reactive to light, and there is conjunctival hyperemia. His
cardiovascular and respiratory examination is normal. What is the most appropriate next step in
the management of this patient?

Choices:
1. Administer fomepizole
2. Administer naloxone
3. Supportive management
4. Gastric lavage
Answer: 3 - Supportive management
Explanations:
Tetrahydrocannabinol (THC) is one of the main cannabinoids derived from the cannabis
plant that can produce euphoria through the binding of CB1 receptors in the central nervous
system (CNS).
Marijuana intoxication presents with dry mouth, conjunctival injection or hyperemia, and
tachycardia.
THC use can very rarely induce seizures in those with risk factors or a past history of
seizures.
Treatment for marijuana intoxication is supportive management.

Go to the next page if you knew the correct answer, or click the link image(s) below to further
research the concepts in this question (if desired).

Research Concepts:
Tetrahydrocannabinol (THC)

We update eBooks quarterly and Apps daily based on user feedback. Please tap flag to
report any questions that need improvement.
Question 914: An 82-year-old man is brought to the emergency department by his daughter
who describes that the patient has recently been weak, lethargic, confused, and nauseous. He was
recently prescribed sertraline three weeks ago. Her vital signs are within normal limits. Which of
the following is the best initial step in the evaluation of this patient?

Choices:
1. Serum potassium
2. Serum sodium
3. TSH
4. Serum lipase
Answer: 2 - Serum sodium
Explanations:
This patient presents with weakness, confusion, nausea, and lethargy in the context of
recently starting an SSRI, suggesting syndrome of inappropriate antidiuretic hormone
(SIADH). Syndrome of inappropriate antidiuretic hormone ADH release is a condition
defined by the unsuppressed release of antidiuretic hormone (ADH) from the pituitary gland
or nonpituitary sources or its continued action on vasopressin receptors. SIADH is
characterized by impaired water excretion leading to hyponatremia with hypervolemia or
euvolemia.
A number of drugs that are associated with SIADH act by enhancing the release or effect of
ADH. The most common drugs include carbamazepine, oxcarbazepine, chlorpropamide,
cyclophosphamide, and selective serotonin reuptake inhibitors (SSRIs). Carbamazepine and
oxcarbazepine act in part by increasing the sensitivity to ADH. Chlorpropamide increases
the number of V2 receptors in collecting tubules. As high-dose intravenous
cyclophosphamide is given with a fluid load to prevent hemorrhagic cystitis, SIADH in
such patients is a particular problem, leading to potentially fatal hyponatremia. SSRIs cause
SIADH by an unknown mechanism, but people above 65 years of age are more at risk.
"Ecstasy" (methylenedioxymethamphetamine)\ is particularly associated with the direct
release of ADH (It also stimulates thirst, which further worsens hyponatremia). Less
commonly, non-steroidal anti-inflammatory drugs (NSAIDs), opiates, interferons,
methotrexate, vincristine, vinblastine, ciprofloxacin, haloperidol, and high dose imatinib
have been linked with SIADH.
Other causes of SIADH include stroke, hemorrhage, infection, trauma, mental illness,
psychosis, surgery, pneumonia, asthma, atelectasis, acute respiratory failure, pneumothorax,
hypopituitarism, and hypothyroidism, exogenous hormone administration, and HIV
infection.
Clinical manifestations of SIADH can be due to hyponatremia and decreased ECF
osmolality, which causes water to move into the cells causing cerebral edema. Signs and
symptoms depend upon the rate and severity of hyponatremia and the degree of cerebral
edema. The earliest clinical manifestations of acute hyponatremia include nausea and
malaise, which may be seen when the serum sodium concentration falls below 125 to 130
mEq/L (normal 135 to 145mEq/L). Vomiting is an ominous sign for patients with acute
hyponatremia. With a more severe and acute fall in sodium concentration, headache,
lethargy, obtundation, and eventually, seizures can occur. Coma and respiratory arrest can
occur if the serum sodium level falls below 115 to 120 mEq/L. Acute hyponatremic
encephalopathy may be reversible, but permanent neurologic damage or death can occur,
particularly in premenopausal women.

Go to the next page if you knew the correct answer, or click the link image(s) below to further
research the concepts in this question (if desired).

Research Concepts:
Syndrome of Inappropriate Antidiuretic Hormone Secretion

We update eBooks quarterly and Apps daily based on user feedback. Please tap flag to
report any questions that need improvement.
Question 915: A 45-year-old man with a history of hypertension, chronic bronchitis, and
diabetes presents to the emergency department with generalized fatigue, malaise for the last six
months. He reports a low-grade intermittent fever, anorexia, unintentional weight loss of 20 lbs
with abdominal fullness. He is monogamous and lives with his girlfriend. He denies any
substance abuse and has no pets. He is compliant with his medications with an HbA1c of 6.5%.
For the past two weeks, he has had a fever, chills, night sweats, myalgia, and chest pain. On
clinical examination, vitals are within normal limits with hepatosplenomegaly and clubbing of
fingers. White cell count is 13,000/microL, and platelet count is 110,00/microL. Blood urea
nitrogen is 36 mg/dL, creatinine 1.2 mg/dL, aspartate aminotransferase 68 IU/L, alanine
aminotransferase 72 IU/L, alkaline phosphatase 130 IU/L, and total bilirubin 1.6 mg/dL. Urine
analysis is normal. ESR is 100 mm/g and CRP 35 mg/L. A chest X-ray reveals no acute
abnormalities. Blood cultures reveal no growth. A transesophageal echocardiogram reveals
normal cardiac function. An immunofluorescence based phase I IgG antibody titer is 1:1200.
What is the best initial therapy for this patient?

Choices:
1. Doxycycline and hydroxychloroquine for 18 months
2. Doxycycline and ciprofloxacin for 18 months
3. Doxycycline and rifampin for 18 months
4. Doxycycline and hydroxychloroquine for 24 months
Answer: 1 - Doxycycline and hydroxychloroquine for 18 months
Explanations:
Q fever endocarditis therapeutic regimen includes doxycycline and hydroxychloroquine for
18 months in native valve endocarditis and 24 months for prosthetic valve endocarditis.
Valve surgery is a must in prosthetic valve involvement.
Endocarditis is a typical presentation of chronic Q fever affecting aberrant valves and
prosthetic valves due to a dysfunctional immune response.
A fourfold decrease in IgG antibody levels implies treatment completion and an antibiotics
halt.
In cases of intolerance or contraindication, ciprofloxacin or rifampin can substitute
hydroxychloroquine.

Go to the next page if you knew the correct answer, or click the link image(s) below to further
research the concepts in this question (if desired).

Research Concepts:
Q Fever

We update eBooks quarterly and Apps daily based on user feedback. Please tap flag to
report any questions that need improvement.
Question 916: A 65-year-old woman with alcoholic cirrhosis is admitted to the hospital for
the evaluation of frequent dark stools. She reports having a history of esophageal varices that
have been repaired. Vitals appear to be stable. Examination shows scleral icterus, mild puffiness
of the face, mild ascites, and pedal pitting edema. She is coherent but appears to have impaired
attention and intermittent jerks of the right arm, left arm, and right foot asynchronously. The
patient undergoes a neurophysiological test to differentiate the intermittent jerks from seizures.
What are the characteristic waveforms that may be seen in this patient?

Choices:
1. 3 Hz spike and biphasic waves
2. 3 Hz spike and wave
3. Triphasic waves
4. Generalized paroxysmal fast activity
Answer: 3 - Triphasic waves
Explanations:
The patient most likely has myoclonus secondary to alcoholic cirrhosis.
electroencephalogram (EEG) is the best test to determine that these symptoms are typically
not associated with any abnormal epileptogenic discharges.
The commonly associated EEG findings in hepatic encephalopathy are diffuse slowing with
abundant waveforms with a triphasic morphology and an anterior to posterior phase lag
associated with arousal.
3 Hz spike and wave patterns are associated with primary generalized epilepsies like
juvenile myoclonic epilepsy and absence seizures.
Generalized paroxysmal fast activity is commonly associated with Lennox–Gastaut (LGS)
or late LGS (LLGS) syndrome related to tonic-axial seizures, medication intractability, and
poor prognosis, including mental deterioration. These are also associated with generalized
epilepsies.

Go to the next page if you knew the correct answer, or click the link image(s) below to further
research the concepts in this question (if desired).

Research Concepts:
Electroencephalogram

We update eBooks quarterly and Apps daily based on user feedback. Please tap flag to
report any questions that need improvement.
Question 917: A 67-year-old man is admitted to the intensive care unit for pneumonia and
sepsis. He has a radial artery catheter in place, from which blood has been drawn for labs. Labs
show WBC count 10200/microL, hemoglobin/ hematocrit 10.1 g/dL /29%, serum sodium 139
mEq/L, serum potassium 4.5 mEq/L, CO2 19 mg/dL, BUN 14 mg/dL, INR 1.1 and activated
partial thromboplastin time 105 seconds. Which of the patient's labs is most likely to be
erroneous?

Choices:
1. Hemoglobin/ hematocrit
2. Serum sodium level
3. INR
4. Activated partial thromboplastin time
Answer: 4 - Activated partial thromboplastin time
Explanations:
Arterial lines usually have heparin-containing solutions running through them. This can
affect the extrinsic pathway of coagulation and cause alteration in activated partial
thromboplastin time. A common reason for coagulation profile error on drawing blood from
the arterial line may be due to inadequate waste (of initially withdrawn blood), which can
contain significant residual heparin.
Heparin is strongly acidic in nature, sometimes can cause falsely low bicarbonate because
of neutralization if there is too much residual heparin in the arterial line.
Instead of heparin-containing solutions, normal saline can be continuously infused through
the arterial lines. There has been no difference shown in the incidence of thrombosis with
the use of heparin vs. nonheparinized solutions.
In patients where there are concerns for bleeding or systemic exposure to heparin with
preexisting derangement of coagulation profile, heparin-containing solutions can be
avoided. Any solution being used should be infused at 1-3 mL/hour (depending on the age
of the patient) through arterial lines to prevent clotting in the catheter.

Go to the next page if you knew the correct answer, or click the link image(s) below to further
research the concepts in this question (if desired).

Research Concepts:
Arterial Lines

We update eBooks quarterly and Apps daily based on user feedback. Please tap flag to
report any questions that need improvement.
Question 918: A 30-year-old man who is a gymnast by profession presents with a three-
day history of pain in his nape after he hit it against a pole. He says the pain radiates along his
right arm and forearm. On examination, there is reduced power in right biceps (4/5), and the
biceps tendon reflex is diminished. A very well known provocative test for the assessment of the
cervical spine is performed. Which of the following statements is true regarding the use of this
test in the evaluation of cervical radiculopathy?

Choices:
1. It has a relatively high potential for false-positive results
2. It has a relatively high potential for false-negative results
3. It is best used as a solitary screening test for cervical radiculopathy
4. It is contraindicated in patients presenting with cervical radiculopathy
Answer: 2 - It has a relatively high potential for false-negative results
Explanations:
The Spurling test has high specificity but low sensitivity. This means it has a relatively high
potential for false-negative results.
The low sensitivity of the Spurling test means that in the evaluation of cervical
radiculopathy, it is not an adequate solitary screening test and is best used in conjunction
with other physical exam tests, in addition to detailed patient history.
Cervical radiculopathy is the primary indication to perform the Spurling test.
The high sensitivity of the Spurling test means that a positive result is highly indicative of
cervical nerve root compression, and further diagnostic testing is indicated.

Go to the next page if you knew the correct answer, or click the link image(s) below to further
research the concepts in this question (if desired).

Research Concepts:
Spurling Test

We update eBooks quarterly and Apps daily based on user feedback. Please tap flag to
report any questions that need improvement.
Question 919: A 45-year-old African American man presents with a two-hour history of
swelling of his tongue and lips and difficulty breathing. His past medical history includes type 2
diabetes mellitus and hypertension. His regular medications include metformin, and he was
recently started on captopril six weeks ago. The patient is immediately intubated, and all
medications stopped. What substance affected by captopril is responsible for this disease
process?

Choices:
1. C1 esterase
2. Potassium
3. Bradykinin
4. Angiotensin II
Answer: 3 - Bradykinin
Explanations:
Angiotensin-converting (ACE) inhibitors inhibit the conversion of angiotensinogen to
angiotensin I. Overall, ACE inhibitors reduce the levels of angiotensin II, and aldosterone
while increasing the levels of renin and bradykinin. The main use of ACE inhibitors is to
prevent remodeling and reduce cardiovascular mortality in heart failure, reduce proteinuria,
and prevent the progression of diabetic nephropathy.
Angiotensin II, which is decreased by captopril, causes vasoconstriction of precapillary
arterioles and postcapillary venules, inhibits the reuptake of norepinephrine, and causes the
release of catecholamines from the adrenal medulla. It also reduces the excretion of sodium
and water, stimulates synthesis and release of aldosterone, and causes hypertrophy of
vascular smooth muscle cells and cardiac myocytes.
Some side effects include paroxysmal cough (1% to 10%), angioedema which can affect
any region, hypotension, and hyperkalemia. In terms of angioedema, it can occur in any
region such as the intestine, but angioedema of the tongue, glottis, or larynx can obstruct the
airway. The prevalence of angioedema is higher in the African-American population.
Treatment for angioedema involving the airways involves immediate stabilization with an
endotracheal tube until the swelling resolves.
Many pharmacological agents like diphenhydramine, methylprednisolone, epinephrine, or
bradykinin blocking agents have been tried as treatment without definitive results.

Go to the next page if you knew the correct answer, or click the link image(s) below to further
research the concepts in this question (if desired).

Research Concepts:
Captopril

We update eBooks quarterly and Apps daily based on user feedback. Please tap flag to
report any questions that need improvement.
Question 920: A 56-year-old female patient with a malignant pleural effusion (MPE) due
to metastatic breast cancer has been admitted for pleurodesis. A thoracostomy tube is placed, and
3.3 liters is rapidly evacuated from the right chest. The patient immediately begins coughing
violently and complaining of pleuritic chest pain. The chest x-ray shows unilateral haziness, and
fluffy infiltrates. Which of the following would prevent this condition from occurring?

Choices:
1. Thoracostomy in the operating room
2. Slow removal of fluid with intermittent clamping of the tube
3. Fluid administration at the same time that fluid is withdrawn from the chest
4. A maximum removal of 750 ml per day
Answer: 2 - Slow removal of fluid with intermittent clamping of the tube
Explanations:
Re-expansion pulmonary edema can be a lethal condition if not recognized.
It is safer to transfer these patients to a monitored bed. Pain control should be adequate.
These patients are frail, to begin with, and do not tolerate any more stress.
Removal of pleural fluid by thoracostomy should be done with intermittent clamping.

Go to the next page if you knew the correct answer, or click the link image(s) below to further
research the concepts in this question (if desired).

Research Concepts:
Chest Tube

We update eBooks quarterly and Apps daily based on user feedback. Please tap flag to
report any questions that need improvement.
Question 921: A 65-year-old man with a history of prostate cancer presents to the
emergency department with severe tetany and numbness around his mouth and tips of fingers.
Examination reveals a GCS of 15/15, with no focal neurological deficit. Computed tomography
rules out metastasis, intracranial bleeding, and ischemia. His blood tests are unremarkable except
for a calcium (ionized) level of 2.5 mg/dL (reference range: 4.4 mg/dL - 5.2 mg/dL). He does not
take calcium supplements, nor vitamin D. He has just been started on intravenous zoledronic acid
for bone metastases. Which of the following associated conditions is most likely present in this
patient?

Choices:
1. Hypoparathyroidism
2. Hypothyroidism
3. Sarcoidosis
4. Peripheral neuropathy
Answer: 1 - Hypoparathyroidism
Explanations:
Bisphosphonates infusion can precipitate hypocalcemia.
Hypoparathyroidism, hypomagnesemia, and vitamin D deficiency are associated with
hypocalcemia.
Extra caution needs to be taken while starting bisphosphonates in these patients.
Sarcoidosis is associated with hypercalcemia, not hypocalcemia. Hypothyroidism and
peripheral neuropathy are not usually associated with hypo/hypercalcemia.

Go to the next page if you knew the correct answer, or click the link image(s) below to further
research the concepts in this question (if desired).

Research Concepts:
Bisphosphonate Toxicity

We update eBooks quarterly and Apps daily based on user feedback. Please tap flag to
report any questions that need improvement.
Question 922: A 26-year-old male presents following intentional ingestion of
approximately 30 grams of copper sulfate 4 hours before admission. On admission, he complains
of epigastric and right upper quadrant pain, severe nausea, and melaena. There are no other
bleeding manifestations. His urine output is reduced and noted to be dark in color. He does not
complain of shortness of breath but does have chest pain. What is the most appropriate next step
in the management of this patient?

Choices:
1. Induce emesis and initiate penicillamine therapy
2. Induce emesis and initiate plasmapheresis therapy
3. Initiate antiemetics to avoid emesis and initiate trientine therapy
4. Initiate antiemetics to avoid emesis and initiate penicillamine therapy
Answer: 4 - Initiate antiemetics to avoid emesis and initiate penicillamine therapy
Explanations:
Emesis must be avoided in cases of copper ingestion as repeated exposure of the esophagus
to this corrosive agent may inflict further damage on the mucosa.
Symptom management and augmenting excretion, thereby reducing the total body copper,
are the mainstays of treatment in acute copper toxicity.
Antiemetics should be readily available for this patient to avoid possible sequela related to
corrosion and loss of integrity of the esophageal mucosa.
Penicillamine is the first-line therapy in patients with copper toxicity and trientine should
only be used as a second-line agent.

Go to the next page if you knew the correct answer, or click the link image(s) below to further
research the concepts in this question (if desired).

Research Concepts:
Copper Toxicity

We update eBooks quarterly and Apps daily based on user feedback. Please tap flag to
report any questions that need improvement.
Question 923: A 70-year-old male has a known history of alcohol-related cirrhosis,
coronary artery disease, hypertension, hyperlipidemia, diabetes mellitus type 2, and obesity is
brought to the emergency department. He also has an alcohol use disorder. He developed leg
edema and ascites over the past two months. Yesterday he was found confused in the park. In the
emergency department, he had altered mental status, scleral icterus, tense abdomen, and leg
edema. His blood pressure was 85/55 mmHg, heart rate 95 bpm, and temperature 38.6 C. He was
admitted to the hospital for further management. His blood work showed anemia, low platelets,
impaired kidney function, and increased INR 2.5. He had increased liver enzymes and total
bilirubin 4.5 mg/dL. Abdominal ultrasound showed a nodular appearance of the liver,
splenomegaly, and large ascites. A paracentesis was completed, and peritoneal fluid analysis
showed increased neutrophil count (>250 cells/µL), and the culture is pending. What is the best
next step in management?

Choices:
1. Initiate a liver transplant evaluation
2. Initiate hemodialysis
3. Start vitamin K subcutaneously
4. Start broad-spectrum antibiotic therapy
Answer: 4 - Start broad-spectrum antibiotic therapy
Explanations:
This patient has acute on chronic liver failure (ACLF) type C, given that he has
decompensated cirrhosis over a period of 2 months before the current clinical presentation.
He had ascites for two months, and at the time of this presentation to the ED, he was febrile
and confused. The peritoneal fluid analysis confirmed spontaneous bacterial peritonitis
(SBP), which likely exacerbated the hepatic encephalopathy. The best next step in the
management is to start a broad-spectrum antibiotic therapy.
ACLF type-C is an acute worsening of liver functions in decompensated cirrhosis. Specific
treatment is needed when the etiology is known, in this case, SBP. The broad-spectrum
antibiotic regimen can be adjusted pending culture and sensitivity results, and the treatment
should be continued for five days unless unusual organisms are found.
ACLF requires supportive care and management of complications, including renal
dysfunction, coagulopathy, and sepsis, in this case.
Hemodialysis is not indicated yet, and the patient has alcohol use disorder despite having
decompensated cirrhosis. Therefore initiation of a liver transplant evaluation is inaccurate at
this time. It is alright to give vitamin K, but the initiation of broad-spectrum antibiotic
therapy is the best next step.

Go to the next page if you knew the correct answer, or click the link image(s) below to further
research the concepts in this question (if desired).

Research Concepts:
Acute On Chronic Liver Failure

We update eBooks quarterly and Apps daily based on user feedback. Please tap flag to
report any questions that need improvement.
Question 924: A 17-year-old female presents to the clinic for headache, nausea, dizziness,
and photophobia that has persisted for 5 weeks after a fall during a basketball game at school.
She reports that she fell and hit her head on the floor but did not experience loss of
consciousness. She reports that she felt “dazed” and took a few minutes before returning to play.
Shortly after returning to the game, she began to experience a headache and dizziness and
returned to the sideline. She has not tried to play basketball or run since that time. She did not
seek medical attention after the incident. Her sports trainer told her to see a healthcare provider,
and she was diagnosed with having sustained a concussion. She endorses a history of a previous
concussion that occurred 6 months before this injury, but states she “was back to normal by the
end of the week.” Her initial evaluation in the clinic today reveals 12 positive symptoms on
SCAT5 (Sport Concussion Assessment Tool fifth Edition) and is VOMS (Vestibular/Ocular-
Motor Screening) positive for concussion. What would be the best recommendation at this time?

Choices:
1. Return to play
2. Obtain a CT head
3. Obtain brain MRI, continue to rest, and take NSAIDs for headache
4. Obtain brain MRI, exercise or activity as tolerated, and re-evaluate in 1 month
Answer: 4 - Obtain brain MRI, exercise or activity as tolerated, and re-evaluate in 1 month
Explanations:
A patient who is symptomatic at 1-month post-injury should obtain a further evaluation.
Head CT is recommended in the emergent setting, but for patients that remain symptomatic
for great than 1 month, an MRI is recommended.
At 1 month post-injury, patients should not continue the use of NSAIDs for pain or
headache but may use NSAIDs within the first week of the injury.
Delayed cognitive and physical rest has not shown benefit, and improved outcomes have
been noted in patients who return to normal activity as soon as tolerated.
Patients will require close follow-up to assess for the further need for cervical physical
therapy, vestibular therapy, or vision therapy. This patient is at high-risk for delayed
recovery and postconcussive syndrome due to female gender, age, and prior history of
concussion just 6 months before the most recent injury.

Go to the next page if you knew the correct answer, or click the link image(s) below to further
research the concepts in this question (if desired).

Research Concepts:
Postconcussive Syndrome

We update eBooks quarterly and Apps daily based on user feedback. Please tap flag to
report any questions that need improvement.
Question 925: A 34-year-old female admitted to an inpatient unit. She was diagnosed with
primary syphilis a few days ago, and she is started on IV medications. Two hours later, she
complains of feeling feverish and headache. Examination reveals a temperature of 100.2 F, blood
pressure of 135/80 mmHg, and a pulse rate of 68/minute. A careful examination of the patient's
body reveals a rash, previously absent. There is no history of drug allergies in the past. Her past
medical history is unremarkable. What is the most likely cause of the patient's current condition?

Choices:
1. Drug side effect
2. Allergic reaction
3. Tissue invasion by spirochetes
4. Toxin and cytokine release secondary to lysis of spirochetes
Answer: 4 - Toxin and cytokine release secondary to lysis of spirochetes
Explanations:
Herxheimer reaction occurs because of toxins and cytokines released during the lysis of
spirochetes following antibiotic treatment.
Herxheimer reactions show an increase in inflammatory cytokines during the period of
exacerbation, including interleukin 6, interleukin 8, and tumor necrosis factor-alpha.
The release of toxins and cytokines results in body aches, fevers, rashes, nausea, and
vomiting, along with other symptoms.
A careful history taking is essential, covering any known drug allergies.

Go to the next page if you knew the correct answer, or click the link image(s) below to further
research the concepts in this question (if desired).

Research Concepts:
Jarisch Herxheimer Reaction

We update eBooks quarterly and Apps daily based on user feedback. Please tap flag to
report any questions that need improvement.
Question 926: A 71-year-old man is being evaluated in the ICU for profound weakness,
after being admitted for three weeks. Electrodiagnostic studies are performed at the bedside.
Which of the following muscles is most likely to show early recruitment of small-amplitude
motor unit action potentials on needle EMG in this patient?

Choices:
1. Abductor pollicis brevis
2. Tibialis anterior
3. Diaphragm
4. First dorsal interossei
Answer: 3 - Diaphragm
Explanations:
In critical illness myopathy, proximal muscles more than distal muscles, including the
diaphragm muscle, will show early recruitment of small-amplitude motor unit action
potentials on needle EMG.
Patients with profound weakness following a prolonged ICU admission or intubation should
be evaluated for critical illness myopathy.
Although it is technically challenging in the ICU, a needle EMG of the diaphragm muscle
may reveal abnormal spontaneous potentials with MUAPs of small amplitude, shorter
duration, and early recruitment.
To accurately diagnose CIM, EMG studies should be performed in at least three extremities,
comparing both distal and proximal muscles, and ideally, evaluation of the diaphragm
should be performed as well.

Go to the next page if you knew the correct answer, or click the link image(s) below to further
research the concepts in this question (if desired).

Research Concepts:
Electrodiagnostic Evaluation Of Critical Illness Myopathy

We update eBooks quarterly and Apps daily based on user feedback. Please tap flag to
report any questions that need improvement.
Question 927: A 65-year-old patient presents to the emergency department with complaints
of loose, watery stools for the past 2 days. He has had 6-7 episodes per day associated with
crampy abdominal pain and fever. His past medical history includes hospitalization 6 months
back for a bout of pneumonia during which he had developed a prolonged bout of diarrhea. On
examination, he is pyrexial with a temperature of 100 F, blood pressure of 90/50 mmHg, a pulse
of 110 beats per minute, and a respiratory rate of 20 breaths per minute. His abdominal
examination shows tenderness in his left lower quadrant. His investigations revealed a white
blood cell count of 28,000 per microL, CRP of 50 mg/l, serum creatine of 1.3mg/dl, and albumin
of 3.0 g/l. His stool assay is positive for glutamate hydrogenase and toxin B. He is started on
appropriate therapy. Addition of which of the following agents is associated with a reduced
incidence of relapse in this patient?

Choices:
1. Infliximab
2. Prednisolone
3. Bezlotoxumab
4. Ciclosporin
Answer: 3 - Bezlotoxumab
Explanations:
This is an elderly patient with a history of hospitalization and likely a previous episode of
Clostridium difficile associated diarrhea, who has presented with severe diarrhea. The
diagnosis is a relapse of Clostridium difficile colitis. Bezlotoxumab has been approved by
the FDA to be used as an adjunct to reduced the likely hood of relapse in high-risk patients.
High-risk patients are those with advanced age, greater than 65 years, previous episodes of
Clostridium difficile colitis, or with severe disease. MODIFY 1 and MODIFY 2 trials
demonstrated the superiority of bezlotoxumab in preventing recurrence while compared to
placebo.
It is a human monoclonal antibody that is directed against clostridial exotoxin B. It is no
used as an isolated treatment but rather in combination with the standard of care.
Pulsed vancomycin, followed by tapering, can also be used to prevent a recurrence.
Infliximab and ciclosporin are immunosuppressive agents used in the treatment of
inflammatory bowel disease.

Go to the next page if you knew the correct answer, or click the link image(s) below to further
research the concepts in this question (if desired).

Research Concepts:
Clostridium Difficile

We update eBooks quarterly and Apps daily based on user feedback. Please tap flag to
report any questions that need improvement.
Question 928: A 75-year-old woman presents to the emergency department with significant
shortness of breath and sputum production. Her past medical history is significant for depression,
for which she takes fluoxetine. Initial vital signs are significant for a heart rate of 101/min and
temperature of 38.2 C. She is producing copious amounts of sputum during the encounter.
Culture results reveal mecA positive gram-positive cocci. What is the best initial therapy for this
patient?

Choices:
1. Linezolid
2. Vancomycin
3. Ceftriaxone
4. Nafcillin
Answer: 2 - Vancomycin
Explanations:
This patient most likely has Staphylococcal pneumonia.
mecA positivity indicates MRSA (methicillin-resistance staphylococcus aureus).
Vancomycin is, therefore, the correct answer as it covers MRSA and does not have a
contraindication with SSRIs.
Linezolid covers MRSA but should not be used if the patient is on an SSRI. Ceftriaxone can
be used with SSRIs, but it does not cover MRSA, for which this patient is infected as she is
mecA positive. Nafcillin covers MSSA but not MRSA.

Go to the next page if you knew the correct answer, or click the link image(s) below to further
research the concepts in this question (if desired).

Research Concepts:
Staphylococcal Pneumonia

We update eBooks quarterly and Apps daily based on user feedback. Please tap flag to
report any questions that need improvement.
Question 929: A 38-year-old man with sickle cell anemia presents to the hospital with a
fever, borderline hypotension, and left flank pain. Urosepsis is suspected as the source of this
patient's early septic shock. She has no peripheral venous access, and there is no intraosseous
drill available in the emergency department. The decision is made to proceed with an internal
jugular central venous catheter. A superficial cervical plexus block is planned. Which of the
following complications is most likely to occur in this patient?

Choices:
1. Malignant hyperthermia
2. Facial nerve paralysis
3. Torticollis
4. Puncture of the external jugular vein
Answer: 4 - Puncture of the external jugular vein
Explanations:
Puncture of the external jugular vein and carotid artery is possible if the needle tip is not
kept under direct visualization at all times.
To prevent this, it is also highly recommended to not advance the needle more than 3 cm in
depth.
Inadvertent deep injection or excessive local anesthetic volume can also cause dysfunction
of the phrenic nerve, recurrent laryngeal nerve, deep cervical plexus, and brachial plexus.
Malignant hyperthermia is a complication of general anesthesia. Accidentally blocking the
accessory nerve can cause sternocleidomastoid and trapezius muscle weakness.

Go to the next page if you knew the correct answer, or click the link image(s) below to further
research the concepts in this question (if desired).

Research Concepts:
Cervical Plexus Block

We update eBooks quarterly and Apps daily based on user feedback. Please tap flag to
report any questions that need improvement.
Question 930: A 65-year-old male presents with his daughter for fever and increased
drowsiness. His daughter states that he has been having high-grade fever and chills for the past
week. For the last two days, her father has become progressively more confused and difficult to
arouse. He has a history of diabetes mellitus and benign prostatic hyperplasia. His medications
include metformin, glibenclamide, dapagliflozin, multivitamin, and tamsulosin. His examination
reveals a blood pressure of 90/40 mmHg, a pulse of 125 beats per minute, respiratory rate of
12/min, a temperature of 102 F (38.9 C), and spO2 of 88%. His physical examination reveals
suprapubic and right flank discomfort. His investigations reveal a WBC count of 17,000 per
microL, hemoglobin of 11 gm/dl, platelet count of 120,000 per microL, and CRP of 75 mg/l.
Ultrasound demonstrates mild right renal pelvis dilatation and the presence of a well-defined 7
cm hypoechoic collection in the right perinephric space around the lower pole of the kidney. The
lesion has increased peripheral vascularity. His post-void residual urine volume is 175 mL. He is
started on broad-spectrum antibiotics, intravenous fluids, and vasopressors. Which additional
treatment modality should be used next in this patient?

Choices:
1. Bladder irrigation
2. Cystoscopy with double J stenting
3. Percutaneous drainage of the perinephric collection
4. Nephrectomy
Answer: 3 - Percutaneous drainage of the perinephric collection
Explanations:
This patient has presented with signs of fever and altered sensorium. His examination
reveals the presence of suprapubic and costovertebral angle tenderness, indicating the
presence of an underlying urinary tract infection. His investigations reveal leukocytosis and
an elevated CRP. His ultrasound demonstrates a perinephric collection of fluid, indicating
the development of a perinephric abscess.
Urosepsis requires intensive care and early goal-directed therapy. Institution of antibiotics,
fluids, and pressors should be initiated as appropriate. The primary and critical treatment is
drainage of the abscess.
Image-guided percutaneous drainage should be performed for this perinephric collection.
Drainage will ensure a response to antibiotics and lead to a reduction of the microbial load
and bacteremia. A pigtailed catheter can be left in situ to ensure complete resolution.
This patient is currently hemodynamically unstable and cannot safely tolerate a major
surgical procedure. Double J stents are useful in the case of ureteric obstruction, but that is
not the case here as there is no hydronephrosis. A Foley catheter should be placed in this
individual for his urinary retention, but bladder irrigation would not be helpful.

Go to the next page if you knew the correct answer, or click the link image(s) below to further
research the concepts in this question (if desired).

Research Concepts:
Urosepsis

We update eBooks quarterly and Apps daily based on user feedback. Please tap flag to
report any questions that need improvement.
Question 931: Targeted Temperature Management (TTM) is a recommendation following
cardiac arrest if a return of spontaneous circulation occurs and the patient remains comatose.
What is the target temperature range?

Choices:
1. 35 to 36 degrees Celsius
2. 32 to 36 degrees Celsius
3. 28 to 32 degrees Celsius
4. 24 to 28 degrees Celsius
Answer: 2 - 32 to 36 degrees Celsius
Explanations:
The target temperature, according to AHA, is 32-36 degrees Celsius.
Targeted temperature management (TTM) is useful when the return of spontaneous
circulation (ROSC) has occurred, and the patient remains in an unresponsive/comatose state
according to the American Heart Association 2015 update.
In adults, TTM is maintained for twenty-four hours.
The routine cooling of patients in a prehospital setting after the return of spontaneous
circulation (ROSC) is no longer a recommendation as of the 2015 AHA update.

Go to the next page if you knew the correct answer, or click the link image(s) below to further
research the concepts in this question (if desired).

Research Concepts:
Anoxic Encephalopathy

We update eBooks quarterly and Apps daily based on user feedback. Please tap flag to
report any questions that need improvement.
Question 932: A 65-year-old woman presents to the emergency department with a
headache and dyspnea that have been worsening in the last several hours. Further questioning
reveals that she has new-onset pain and swelling of her hands, cold, painful fingers with cold
exposure. On examination she appears anxious, beak-like nose, thickening of fingers skin and
bilateral crackles. Her vital signs are pertinent for blood pressure 240/110 mmHg, heart rate
90/minute, afebrile. Laboratory investigations show white blood cell 3.7 x10^9/L, hemoglobin
10 g/dL, platelets 460 x10^9/L, blood urea nitrogen 52 mg/dL, creatinine 3.4 mg/dL, positive
anti-nuclear antibody (ANA) at titre 1:320 in speckled pattern, anti-ribonucleic acid (RNA)
polymerase antibody 30 units/mL (20 units/mL), presence of anti scleroderma-70 antibody 2 U
(1 U), normal complements. Which of the following is the most appropriate treatment initial
treatment for her blood pressure?

Choices:
1. Oral hydralazine
2. Labetalol infusion
3. Intravenous enalaprilat
4. Oral hydrochlorthiazide
Answer: 3 - Intravenous enalaprilat
Explanations:
Systemic sclerosis (SSc) is a multisystem autoimmune disorder that can manifest as diffuse
cutaneous or limited cutaneous variant. Scleroderma renal crisis (SRC) complicates the
course of 10% of patients of systemic sclerosis. It is a life-threatening complication
characterized by endothelial dysfunction, renin-angiotensin-aldosterone system (RAAS)
activation, that presents with the abrupt onset of severe hypertension, rapidly progressive
renal failure, and microangiopathic hemolytic anemia.
Most cases of scleroderma renal crisis occur in patients with diffuse scleroderma (10% to
25%) as compared to limited cutaneous variant (1% to 2%). The disease usually occurs
early in the course of scleroderma, up to 75% of cases of scleroderma renal crisis
developing within the first four years from the diagnosis.
This patient is presenting with hypertensive emergency, renal failure, anemia, serology
pointing towards scleroderma.
As RAAS activation plays central role in this condition, angiotensin inhibitors have been
used and found to reduce mortality rate from 85% to 24%.

Go to the next page if you knew the correct answer, or click the link image(s) below to further
research the concepts in this question (if desired).

Research Concepts:
Scleroderma And Renal Crisis

We update eBooks quarterly and Apps daily based on user feedback. Please tap flag to
report any questions that need improvement.
Question 933: A 91-year-old woman is admitted to the hospital with a cough and fever.
She was recently diagnosed with stage 3 small cell lung cancer. A chest x-ray is suggestive of
right lower lobe pneumonia. The patient is intubated, transferred to the intensive care unit, and
treated with appropriate antibiotics and fluids. After two days, the patient's condition does not
improve, and the family expresses their decision to extubate the patient terminally. After a
detailed discussion and appropriate documentation, terminal extubation is planned. The patient is
extubated, but she does not regain consciousness. She continues to breathe at 10 breaths per
minute, saturating at 70%. Her family is at the bedside. Which of the following is the next best
step in the management of this patient?

Choices:
1. Repeat chest x-ray
2. High flow oxygen
3. Turn off monitor alarms
4. IV morphine
Answer: 3 - Turn off monitor alarms
Explanations:
The clinical scenario describes a patient needing end of life care after terminal extubation.
The patient is breathing at 10 breaths per minute and saturating at 70% after being
extubated. Her condition is highly suggestive of the patient passing away in the next hour or
so.
As the end of life approaches for a patient, the alarms monitoring the patient's vital signs are
likely to start blaring.
To decrease the family's anxiety of watching their loved one die, it is appropriate to turn off
the alarms.
Repeating the chest x-ray serves no purpose as the patient is likely to pass away soon. High
flow oxygen is considered too aggressive a therapy for end-of-life care. The patient is not
conscious and likely not in any distress. There is no indication for IV morphine for
palliation.

Go to the next page if you knew the correct answer, or click the link image(s) below to further
research the concepts in this question (if desired).

Research Concepts:
End of Life Care

We update eBooks quarterly and Apps daily based on user feedback. Please tap flag to
report any questions that need improvement.
Question 934: A 65-year-old male with a past medical history of paroxysmal atrial
fibrillation, ischemic cardiomyopathy, severe left ventricular dysfunction, and COPD presents
with symptoms of the increasing burden of heart palpitations and fatigue for one week. On
physical exam, he is well-nourished and in no acute distress. His breath sounds are diminished
but clear, and he is noted to have an irregular rhythm at a rapid rate. Implantable loop recorder
interrogation reveals atrial fibrillation with a rapid ventricular rate for the past one week. He has
a normal renal function, and EKG reveals a QT interval of 414 msec. He was treated with
amiodarone for the maintenance of sinus rhythm in the past but was discontinued due to
abnormal pulmonary function testing. What would be the best course of action?

Choices:
1. Start flecainide, and if persistent atrial fibrillation at the next outpatient visit, proceed to
cardioversion
2. Admit to hospital, and start propafenone awaiting pharmacologic conversion
3. Rechallenge amiodarone short term with plans for discontinuation after pharmacologic
conversion
4. Admit to hospital, and start dofetilide, and await pharmacologic conversion
Answer: 4 - Admit to hospital, and start dofetilide, and await pharmacologic conversion
Explanations:
Admitting to the hospital is important in the setting dosing dofetilide as it is proarrhythmic
upon initiation and can cause dangerous ventricular arrhythmias including polymorphic
ventricular tachycardia.
Initiating dofetilide is important in this situation as he has symptomatic atrial fibrillation
with significant previous history with medication failure.
Dofetilide is a class III antiarrhythmic used for the maintenance of sinus rhythm in
persistent atrial fibrillation and is useful for application in this patient.
Dofetilide is safe in patients with underlying lung disease while amiodarone is
contraindicated given its potential side-effect of pulmonary toxicity. Dofetilide is safe in
patients with underlying structural heart disease. On the contrary, Flecainide and
propafenone are contraindicated with underlying heart disease.

Go to the next page if you knew the correct answer, or click the link image(s) below to further
research the concepts in this question (if desired).

Research Concepts:
Dofetilide

We update eBooks quarterly and Apps daily based on user feedback. Please tap flag to
report any questions that need improvement.
Question 935: A 65-year-old man presents with a one-week history of central chest pain.
He also complains of shortness of breath that has been worsening over the same period. His past
medical history includes hypertension, hyperlipidemia, lymphoma in remission, and diabetes
type 2 with a recent HbA1c of 9.2%. He currently takes aspirin, atorvastatin, amlodipine,
hydrochlorothiazide, and insulin. He completed a course of anthracycline-based chemotherapy
for his lymphoma six months ago. On examination, his vital signs are all within normal limits.
There are bilateral decreased breath sounds on auscultation of the lungs, and the cardiovascular
exam is unremarkable. On a chest x-ray, there is bilateral vascular congestion and reduced
aeration throughout the lung fields. Which of the following findings most supports a diagnosis of
cardiotoxicity secondary to his chemotherapy?

Choices:
1. Elevated cardiac troponin levels
2. T-wave inversions in V1, V2, and V3 leads
3. Decreased left ventricular ejection fraction
4. Prolonged QRS complex in leads V1 and V2
Answer: 3 - Decreased left ventricular ejection fraction
Explanations:
Due to the cardiotoxic profile of adriamycin, baseline left ventricular ejection fraction
(LVEF) should be done before starting adriamycin and should be monitored routinely using
an echocardiogram. An LVEF drop of more than 10% from baseline is considered to
indicate cardiotoxicity from adriamycin, and immediate intervention is required.
An alternative to echocardiogram monitoring is radionuclide ventriculography, such as
multi-gated acquisition angiogram (MUGA) or equilibrium radionuclide angiogram
(ERNA) scans, although these are less frequently used.
Regular echocardiogram monitoring in patients with current or recent use of adriamycin is
essential.
Troponin levels and ECG abnormalities are not specific for adriamycin toxicity.

Go to the next page if you knew the correct answer, or click the link image(s) below to further
research the concepts in this question (if desired).

Research Concepts:
Anthracycline Medications (Doxorubicin)

We update eBooks quarterly and Apps daily based on user feedback. Please tap flag to
report any questions that need improvement.
Question 936: A retrospective study is studying the sensitivity of organisms to ampicillin
and its extended coverage combination ampicillin-sulbactam. Patients with positive blood
cultures who were given the antibiotics during hospitalization were followed up with blood
cultures to document the eradication of the bacteria. Patients infected with which bacteria are
most likely to have positive blood cultures after treatment with ampicillin but negative blood
culture with ampicillin-sulbactam?

Choices:
1. Listeria monocytogenes
2. Escherichia coli
3. Acinetobacter baumannii
4. Streptococcus pneumoniae
Answer: 3 - Acinetobacter baumannii
Explanations:
The combination of ampicillin-sulbactam acts synergistically to cover strains of bacteria
resistant to ampicillin, thus providing broader coverage than ampicillin alone.
Ampicillin-sulbactam is used to treat skin and subcutaneous infections caused by beta-
lactamase producing strains of Staphylococcus aureus, Klebsiella species, including K.
pneumoniae, Escherichia coli, Proteus mirabilis, Bacteroides fragilis, Acinetobacter
calcoaceticus, Acinetobacter baumannii, and Enterobacter species.
Ampicillin-sulbactam is used to treat intra-abdominal infections caused by beta-lactamase
producing strains of Escherichia coli, Klebsiella species, including K. pneumoniae,
Enterobacter species, and Bacteroides species, including B. fragilis.
Ampicillin-sulbactam is used to treat gynecological infections caused by beta-lactamase
producing strains of Bacteroides species, including B. fragilis and Escherichia coli.

Go to the next page if you knew the correct answer, or click the link image(s) below to further
research the concepts in this question (if desired).

Research Concepts:
Ampicillin/Sulbactam

We update eBooks quarterly and Apps daily based on user feedback. Please tap flag to
report any questions that need improvement.
Question 937: An 82-year-old woman with a history of congestive heart failure and chronic
obstructive pulmonary disease is brought to the hospital with acute onset left dense hemiplegia.
She remains lucid and communicative and expresses her wishes to ‘die at home’ and agrees with
her advance directives for ‘do not resuscitate’ and ‘do not intubate.’ Which of the following is
the next most appropriate course of action?

Choices:
1. Discharge the patient home
2. Continue active intensive care management
3. Recommend funeral arrangements to the family
4. End-of-life care assessment
Answer: 4 - End-of-life care assessment
Explanations:
The hospice team can help end-of-life patients, who are expected to live for several days or
more to ‘die at home.’
The hospice care team and multidisciplinary support (including intensivist, nursing staff,
primary care physician, palliative care team, social worker, physical therapist, etc.) for
providing home hospice care.
The hospice team can provide the resources and help that a family needs to take care of a
terminal patient at home.
It is unwise to send the patient home without making arrangements first.

Go to the next page if you knew the correct answer, or click the link image(s) below to further
research the concepts in this question (if desired).

Research Concepts:
Termination of Life Support

We update eBooks quarterly and Apps daily based on user feedback. Please tap flag to
report any questions that need improvement.
Question 938: A 29-year-old woman with a history of generalized anxiety disorder presents
to the hospital with acute onset of shortness of breath. Labs are unremarkable with a creatinine of
0.8 mg/dL. She is started on enoxaparin for VTE prophylaxis while she is evaluated for the cause
of her symptoms. If she begins to bleed excessively, what is the mechanism of action of the most
appropriate reversal agent used for enoxaparin?

Choices:
1. Humanized monoclonal antibody
2. Competitive binding to factor Xa
3. A cationic peptide that binds to low molecular weight heparin forming an ionic complex to
neutralize the effect
4. Administration of an essential cofactor for the gamma-carboxylase enzymes
Answer: 3 - A cationic peptide that binds to low molecular weight heparin forming an ionic
complex to neutralize the effect

Explanations:
Protamine sulfate is a cationic peptide that binds to low molecular weight heparin, thereby
forming an ionic complex with no anticoagulant activity.
Protamine neutralizes about 60% of low molecular weight heparin anticoagulant activity.
The recommended dose is 1 mg protamine sulfate per 1 mg of enoxaparin or 100 anti-
Factor-Xa units of dalteparin.
The use of protamine sulfate correlates with fewer bleeding complications post
percutaneous coronary intervention (PCI).

Go to the next page if you knew the correct answer, or click the link image(s) below to further
research the concepts in this question (if desired).

Research Concepts:
Enoxaparin

We update eBooks quarterly and Apps daily based on user feedback. Please tap flag to
report any questions that need improvement.
Question 939: A 68-year-old man with severe lion pain is found to have calculi in his right
ureter. On further evaluation, he is found to have hydronephrosis and acute kidney injury. He
undergoes percutaneous nephrostomy tube placement by to relieve his obstruction. Immediately
afterward, he suffers septic shock. He is started on broad-spectrum antibiotics covering gram-
negative bacteria after a culture is sent. He is started on norepinephrine and vasopressin at
maximal doses. However, he still remains in shock and consideration for angiotensin II is made.
Which of the following is the most appropriate intervention before starting angiotensin II therapy
in this patient?

Choices:
1. Serum lactate level
2. Serum prolactin level
3. Serum troponin level
4. IV 30 mL/kg normal saline
Answer: 4 - IV 30 mL/kg normal saline
Explanations:
In the ATHOS-3 trial, all patients who were enrolled and received angiotensin II were
adequately resuscitated, with CVP of 8 to 12 cmH2O, and had a central venous catheter and
arterial line for monitoring.
Starting angiotensin II without adequately resuscitating a patient does not make sense
physiologically, as well as you don't want to vasoconstrict without intravascular volume
repletion.
The only guidance for this medication that we have in our clinical practice is from the
clinical trial ATHOS-3. Thus, to ensure the best result, if there is no contraindication, the
conditions of the trial should be replicated.
Obtaining lactic acid, procalcitonin or troponin is not a prerequisite for starting any
vasopressor, let alone angiotensin II.

Go to the next page if you knew the correct answer, or click the link image(s) below to further
research the concepts in this question (if desired).

Research Concepts:
Angiotensin II

We update eBooks quarterly and Apps daily based on user feedback. Please tap flag to
report any questions that need improvement.
Question 940: A 67-year-old man with a past medical history of hypertension, alcohol use,
depression, gout, and benign prostatic hyperplasia is admitted to the intensive care unit with a
grade 3 subarachnoid hemorrhage (Modified Fischer scale). A cerebral aneurysm is identified on
angiography, and endovascular coiling is done to reduce the risk of further bleeding. 7 days post-
treatment of the aneurysm, the patient starts complaining of nausea and headache. CT imaging of
the brain does not show new findings. There is no dizziness on standing. Capillary refill time is
normal. The patient's vitals show no new increase in the heart rate or fall of blood pressure. A
current set of labs is shown below.
Patient value Reference range
Sodium 130 mEq/L 134-144 mEq/L
Potassium 4.2 mEq/L 3.6-5.0 mEq/L
Chloride 100 mEq/L 98-107 mEq/L
Bicarbonate 24 mEq/L 21-28 mEq/L
Which of the following is the best next step in the management of this patient?

Choices:
1. 3% sodium chloride solution
2. 0.9% sodium chloride solution
3. 0.9% sodium chloride with 5% dextrose solution
4. Total fluid restriction
Answer: 1 - 3% sodium chloride solution
Explanations:
This scenario describes a case of hyponatremia following subarachnoid hemorrhage (SAH)
due to increased secretion of anti-diuretic hormone (ADH).
Hyponatremia following SAH is either due to increased secretion of ADH (majority) or due
to cerebral salt wasting. If the hyponatremia is due to increased ADH, the patient is
euvolemic. If the hyponatremia is due to cerebral salt wasting, there is clear evidence of
hypovolemia.
Hyponatremia, here, is best treated with hypertonic saline (3% sodium chloride).
Some clinicians prefer isotonic normal saline (0.9% sodium chloride) if they feel
hyponatremia is due to CSW alone. Fluid restriction leads to hypovolemia, which
aggravates the risk of cerebral vasospasm. Fluid restriction should be avoided.

Go to the next page if you knew the correct answer, or click the link image(s) below to further
research the concepts in this question (if desired).

Research Concepts:
Subarachnoid Hemorrhage

We update eBooks quarterly and Apps daily based on user feedback. Please tap flag to
report any questions that need improvement.
Question 941: A patient presents with a blood pressure of 70/40 mmHg, a pulse of 135
bpm, a pulmonary capillary wedge pressure of 27 mmHg, a CVP of 11 mmHg, and a systemic
vascular resistance of 2000 dynes sec cm. The hemodynamic profile of the patient suggests
which of the following shock states?

Choices:
1. Septic shock
2. Hypovolemic shock
3. Neurogenic shock
4. Cardiogenic shock
Answer: 4 - Cardiogenic shock
Explanations:
Invasive monitoring is useful in helping to rule out other causes of shock.
Cardiogenic shock usually has an elevated wedge pressure and a cardiac index of less than
2.2 liters/min.
The presence of large V waves on the wedge pressure tracing indicates the presence of
severe mitral regurgitation.
A step up of oxygen saturation between the right atrium and right ventricle is diagnostic of a
ventricular septal rupture.

Go to the next page if you knew the correct answer, or click the link image(s) below to further
research the concepts in this question (if desired).

Research Concepts:
Cardiogenic Shock

We update eBooks quarterly and Apps daily based on user feedback. Please tap flag to
report any questions that need improvement.
Question 942: A 26-year-old previously well male with no comorbidities is admitted to the
intensive care unit following a traumatic head injury. He is currently on day 4 post endotracheal
intubation and initiation of mechanical ventilation. He has been receiving enteral feeds directly
into the small intestine. He is now noted to have a temperature of 39C, a spontaneous respiratory
rate of 30/minute, a blood pressure of 96/64 mmHg, a heart rate of 122 beats/minute, oxygen
saturation of 92% in room air and finger-prick blood glucose of 6 mmol/L. A complete blood
count reveals a hemoglobin of 15.1g/dL and a white blood cell count of 15 000 cells/mm3.
Purulent secretions are noted to be present in the endotracheal tube. He has been receiving
enteral feeds directly into the small intestine. The culture result of an endotracheal aspirate
specimen is pending. He is commenced on empiric antimicrobial therapy and intravascular fluid
volume expansion is escalated. Which of the following represents the most likely mechanism of
introduction of the micro-organisms typically implicated in the etiology of the patient’s
condition?

Choices:
1. Ileus
2. Urinary catheterization
3. Prone positioning
4. Sedation break
Answer: 1 - Ileus
Explanations:
The patient in this scenario has a likely diagnosis of ventilator-associated pneumonia
(VAP). VAP is defined as the development of pneumonia more than 48 hours after the
initiation of mechanical ventilation.
The diagnosis of ventilator-associated pneumonia (VAP) is based on a positive semi-
quantitative culture result from a specimen obtained via a non-invasive sampling technique
(i.e. endotracheal aspiration) and the presence of clinical criteria (such as pyrexia and
leucocytosis [white blood cell count > 11 000 cells/mm3]), as per the American Thoracic
Society and Infectious Diseases Society of America guideline for the management of VAP,
jointly published in 2016.
Aerobic gram-negative bacteria (AGNB) are typically implicated in the etiology of VAP.
According to the gastropulmonary hypothesis, gastric colonization with AGNB occurs,
followed by colonization and infection of the respiratory tract via gastric reflux and
aspiration, which may manifest as ventilator-associated pneumonia. As per the hypothesis,
AGNB are introduced into the stomach exogenously (for example, through contaminated
nasogastric tube feeds) or endogenously (via reflux of intestinal content). Ileus enables the
reflux of intestinal content into the stomach. The presence of bile acids in the refluxed
intestinal content causes an increase in gastric pH, which promotes aerobic Gram-negative
bacterial colonization and growth in the stomach.
The pH of enteric feeds is between 6 and 7. Although not a part of routine, standard
practice, enteral feeds may be administered directly into the small intestine to prevent the
effect of an increased gastric pH from enteral feeds. An increased gastric pH promotes
aerobic Gram-negative bacterial colonization and growth in the stomach in the pathogenesis
of ventilator-associated pneumonia.

Go to the next page if you knew the correct answer, or click the link image(s) below to further
research the concepts in this question (if desired).

Research Concepts:
Ventilator Complications

We update eBooks quarterly and Apps daily based on user feedback. Please tap flag to
report any questions that need improvement.
Question 943: Which of the following adjustment should be made to the regimen of an
elderly white female with history of congestive heart failure with an ejection fraction of 30% on
an ACE inhibitor, a beta-blocker, and a loop diuretic with a well controlled blood pressure but
who is experiencing orthopnea and is found to have pulmonary rales and pitting edema
peripherally?

Choices:
1. Start warfarin
2. Start nitrates
3. Stop beta-blockers
4. Start spironolactone
Answer: 4 - Start spironolactone
Explanations:
Spironolactone is beneficial for patients with Class III (limitation in normal activities) or
class IV CHF (dyspnea at rest) and an ejection fraction 35%.
Beta-blockers have been shown to decrease mortality and are indicated for all stable CHF
patients but are not indicated in acute congestive heart failure.
Warfarin is indicated in patients with atrial fibrillation that are at a high risk of developing a
stroke.
Nitrates can help in an acute congestive heart failure exacerbation by decreasing afterload
and allowing the heart to pump against less resistance.

Go to the next page if you knew the correct answer, or click the link image(s) below to further
research the concepts in this question (if desired).

Research Concepts:
Congestive Heart Failure

We update eBooks quarterly and Apps daily based on user feedback. Please tap flag to
report any questions that need improvement.
Question 944: A 60-year-old woman with a history of leukemia is brought to the
emergency department with altered mentation. On physical examination, her blood pressure is
80/40 mmHg, her pulse rate is 110/min, and she has a low-grade temperature. She is disoriented
and lethargic. She has recently been prescribed sunitinib for leukemia treatment. An initial set of
investigations is shown below.
Reference
Patient results
range
Hemoglobin 9 g/dL 11.5-16 g/dL
4100-
WBC count 9840/microL
10900/microL
150000-
Platelets 297000/microL
400000/microL
Creatinine 1.09 mg/dL 0.8-1.4 mg/dL
134-144
Sodium 126 mEq/L
mEq/L
Potassium 5.2 mEq/L 3.6-5.0 mEq/L
Chloride 93 mEq/L 98-107 mEq/L
Bicarbonate 21 mEq/L 21-28 mEq/L
Urinalysis showed cloudy urine, with a trace of blood, increased WBC, and leukocyte esterase
consistent with a urinary tract infection. A chest radiography, CT of the abdomen and pelvis, and
CT brain do not reveal any abnormalities. Blood and urine cultures are sent. The patient is given
3 liters of intravenous fluid resuscitation along with broad-spectrum antibiotics. However, her
hypotension persists, so vasopressors are then initiated. Which of the following cause is
contributing to the persistent hypotension?

Choices:
1. Severe sepsis from urinary tract infection
2. Sunitinib induced pituitary failure
3. Sunitinib induced adrenal insufficiency
4. Anemia
Answer: 3 - Sunitinib induced adrenal insufficiency
Explanations:
The patient has an adrenal crisis precipitated by sunitinib use. Patients on tyrosine kinase
inhibitors should be educated that they can precipitate adrenal insufficiency. The patient has
hypotension with tachycardia, hyponatremia, anemia, and hyperkalemia with confusion,
confirming the diagnosis of adrenal crisis. Emergency treatment of suspected adrenal crisis
is immediate administration of iv hydrocortisone, aggressive IV fluid resuscitation, and
appropriate antibiotics.
The dose of intravenous hydrocortisone should be 100 mg (stress dose ) intravenously or
intramuscularly (IV/IM) as an initial bolus followed by 100 to 300 mg daily after that for
another 2-3 days of boluses every 6 hours or as continuous infusion until complete
recovery. These individuals also require aggressive fluid and vasopressors. A thorough
search should be done for the cause, and empiric antibiotics are recommended.
Close monitoring in the intensive care unit is required. At discharge, the patient should be
given a medical bracelet that alerts medical personnel about adrenal insufficiency. Adrenal
crises can lead to severe refractory shock and death if not promptly recognized.
Even though the patient has severe sepsis from urinary tract infection, treating underlying
sunitinib-induced adrenal insufficiency with intravenous steroids, along with IV fluid
resuscitation, and antibiotics will help improve hypotension. Sunitinib does not cause
pituitary failure.

Go to the next page if you knew the correct answer, or click the link image(s) below to further
research the concepts in this question (if desired).

Research Concepts:
Adrenal Crisis

We update eBooks quarterly and Apps daily based on user feedback. Please tap flag to
report any questions that need improvement.
Question 945: A 65-year-old patient with a left-sided malignant middle cerebral artery
infarction was being managed in the neurology intensive care unit (ICU) by placing an
intraventricular ICP monitoring catheter. The neurosurgeon was asked for consultation following
the appearance of ICP waves rising 50 mm (normal value of 5-15 mm of Hg) above baseline and
lasting for 5 to 20 minutes. Which of the following is the most appropriate next management
plan for the patient?

Choices:
1. keeping the patient in the ventilator and sedated
2. Asking for mannitol infusion
3. Hypothermia
4. Decompressive hemicraniectomy
Answer: 4 - Decompressive hemicraniectomy
Explanations:
The given clinical scenario is typical of a raised ICP following malignant middle cerebral
artery infarction owing to the presence of Lundberg A waves in the ICP waveform.
Decompressive hemicraniectomy (DHC) is advisable for the life-saving procedures for the
same.
Timely performed DHC with durotomy allows for the immediate release of the raised ICP
and is effective in reducing mortality as well as high dependency among these patients
compared to medical management.
Other tiers of medical management such as barbiturate coma, mannitol infusion,
hypothermia have not been validated for reducing mortality compared to surgical
management.

Go to the next page if you knew the correct answer, or click the link image(s) below to further
research the concepts in this question (if desired).

Research Concepts:
Intracranial Pressure Monitoring

We update eBooks quarterly and Apps daily based on user feedback. Please tap flag to
report any questions that need improvement.
Question 946: A 46-year-old man presents for an elective liver transplant. He has a past
medical history of end-stage chronic kidney disease. Blood tests show creatinine 3.4 mg/dL (0.8-
1.3), bilirubin 7.3 mg/dL (0.1-1.2), international nationalized ratio (INR) 2.3 (1.1 or below), and
sodium 125 mEq/L (135-145). A non-depolarizing neuromuscular blocking drug is administered
to facilitate endotracheal intubation and provide skeletal muscle relaxation during surgery. How
is this drug metabolized?

Choices:
1. Non-enzymatic Hoffman elimination
2. Diffusion from the neuromuscular junction
3. Redistribution into adipose tissue
4. Via cholinesterases located in the pulmonary endothelium
Answer: 1 - Non-enzymatic Hoffman elimination
Explanations:
Atracurium is a neuromuscular blocking agent of the benzylisoquinolinium class.
It undergoes Hoffman's elimination. This is a nonenzymatic process that is independent of
renal or hepatic function.
Hoffman elimination is a temperature and pH-dependent process that can be slowed by
hypothermia and acidosis.
Neuromuscular blocking agents of the Benzylisoquinolinium class are preferred in critically
ill patients as their metabolism is not affected by renal or hepatic dysfunction.

Go to the next page if you knew the correct answer, or click the link image(s) below to further
research the concepts in this question (if desired).

Research Concepts:
Atracurium

We update eBooks quarterly and Apps daily based on user feedback. Please tap flag to
report any questions that need improvement.
Question 947: A 34-year-old woman with a history of epilepsy is brought to the emergency
department after several generalized seizures. There is no history of trauma or any sign of injury.
Convulsive movements stopped after benzodiazepine administration, but she remains
unresponsive 30 minutes after apparent seizure cessation. Which of the following is the next best
step in the management of this patient?

Choices:
1. CT head
2. MRI head
3. Cranial angiography
4. EEG
Answer: 4 - EEG
Explanations:
Persistent unresponsiveness after generalized convulsive status epilepticus may indicate the
transformation of the seizures to a type of nonconvulsive generalized status epilepticus. The
highly abnormal electrical activity of the brain continues but may no longer be reflected
with motor movements. An EEG is necessary to detect continuing seizures.
Generalized convulsive status epilepticus may evolve or change into a state with persistent
coma or unresponsiveness without convulsions. Terms used to describe this include subtle
status epilepticus or transformed status epilepticus. This is a continuing emergency and
should be detected by EEG and treated.
Continuous EEG monitoring is becoming used more frequently in ICU settings. Unexpected
seizures on EEG are detected surprisingly often.
CT or MRI are commonly performed after stabilization of patients with status epilepticus,
but for patients with a history of epilepsy, the yield of new findings with neuroimaging is
low.

Go to the next page if you knew the correct answer, or click the link image(s) below to further
research the concepts in this question (if desired).

Research Concepts:
Coma

We update eBooks quarterly and Apps daily based on user feedback. Please tap flag to
report any questions that need improvement.
Question 948: A 20-year-old male missionary worker returned from a mission in Indonesia
one week ago. Upon return, he started to have intermittent fever, headache, and malaise. He took
over-the-counter acetaminophen 300 mg every 6 hours daily. His symptoms worsened over five
days. In the emergency department, he has a high-grade fever, hypotension, and somnolence
with brisk deep tendon reflexes and hypertonia. CBC reveals severe anemia and
thrombocytopenia. A malaria Rapid Diagnostic Test (RTD) is negative. Lumbar puncture is
unremarkable. He then develops a sudden onset of limb jerking for approximately 60 seconds
and goes unresponsive afterward. MRI brain of this patient would most likely reveal which of the
following?

Choices:
1. Ring enhancing lesion in the temporal lobe
2. Significantly increased brain volume
3. Elliptical collection of subdural fluid
4. Bilateral ventriculomegaly
Answer: 2 - Significantly increased brain volume
Explanations:
In a patient returning from a malaria-endemic region with intermittent fever and neurologic
symptoms, cerebral malaria should be high on the differential.
In patients with suspected cerebral malaria, meningitis must be ruled out with a lumbar
puncture.
Malaria RTDs currently can only detect P. falciparum. In a patient returning from Southeast
Asia, P. vivax infection is likely and will not be detected with RTD.
MRI brain in patients with cerebral malaria will likely reveal significantly increased brain
volume due to vascular congestion, impaired perfusion, endothelial activation, blood-brain
barrier changes, and cerebral edema.

Go to the next page if you knew the correct answer, or click the link image(s) below to further
research the concepts in this question (if desired).

Research Concepts:
Malaria

We update eBooks quarterly and Apps daily based on user feedback. Please tap flag to
report any questions that need improvement.
Question 949: A 65-year-old man is brought to the emergency department with complaints
of new-onset confusion, hyperventilation, and vomiting that started five hours ago in a nursing
home. The patient has a past medical history of mild dementia, high blood pressure, and chronic
back pain. His medications include lisinopril, aspirin, amlodipine, and ginseng. The nurse
accompanying the patient is unsure if he has been compliant with his medication. Currently, the
patient is afebrile and has a blood pressure of 145/90 mm Hg. Physical examination reveals a
patient who is not oriented to time, place, or person. There is no sign of dehydration on the exam.
Arterial blood gas results include a pH of 7.41, Pa CO2 of 24 mm Hg, bicarbonate level of 15
mEq/L, sodium level of 143 mEq/L, chloride levels of 100 mEq/L and PaO2 of 98 mm Hg. What
is the best next step in the management of this patient's condition?

Choices:
1. Measurement of serum thiamine
2. Measurement of serum ethylene glycol
3. Measurement of serum salicylate levels
4. Measurement of serum ketones
Answer: 3 - Measurement of serum salicylate levels
Explanations:
The drug intoxication most likely cause of mixed acid-base disturbance of respiratory
alkalosis, and anion gap metabolic acidosis is a salicylate (aspirin).
Aspirin (salicylate) directly stimulates the medullary respiratory center to cause tachypnea
and respiratory alkalosis. In addition, aspirin causes an anion gap metabolic acidosis due to
increased production and decreased renal elimination of organic acids (eg, lactic acid,
ketoacids).
Although thiamine deficiency can cause elevation of lactate levels in the setting of beriberi,
it does not cause respiratory alkalosis.
Serum osmolality should be measured in suspected cases with anion gap acidosis, given
ingestion of certain substances such as ethylene glycol or methanol can cause metabolic
acidosis and an osmolal gap. However, these substances do not cause respiratory alkalosis
and would not explain the mixed acid-base disturbance, as in this case.

Go to the next page if you knew the correct answer, or click the link image(s) below to further
research the concepts in this question (if desired).

Research Concepts:
High Anion Gap Metabolic Acidosis

We update eBooks quarterly and Apps daily based on user feedback. Please tap flag to
report any questions that need improvement.
Question 950: A patient undergoes abdominal surgery for a splenic rupture and receives
many units of transfused blood. Post-surgery he complains of numbness around his mouth and a
carpopedal spasm. An electrocardiogram reveals a prolonged QT interval. Which of the
following would be the best intravenous treatment for this patient?

Choices:
1. Bicarbonate
2. Potassium
3. Calcium
4. D50
Answer: 3 - Calcium
Explanations:
Chvostek sign occurs when tapping the skin over the facial nerve causes ipsilateral
contraction of facial muscles. This is a suggestive sign of hypocalcemia.
Trousseau's sign occurs when the blood pressure cuff is inflated on the arm and causes
flexion of the wrist and extension of interphalangeal joints. The thumb usually adducts.
Trousseau's sign is more specific for hypocalcemia.
Other abnormalities that may occur in hypocalcemia include athetosis, dystonic spasm,
Parkinson disease, and hemiballismus.
Coarse hair, brittle nails, and dry skin are other features of long-standing hypocalcemia.

Go to the next page if you knew the correct answer, or click the link image(s) below to further
research the concepts in this question (if desired).

Research Concepts:
Hypocalcemia

We update eBooks quarterly and Apps daily based on user feedback. Please tap flag to
report any questions that need improvement.
Question 951: A 72-year-old man is brought to the hospital with confusion. He has normal
vital signs except for a respiratory rate of 32/min. Initial labs show 136 mEq/L, potassium 2.6
mEq/L, bicarbonate 14 mEq/L, chloride 112 mEq/L, BUN 17 mg/dL, creatinine 0.9 mg/dL,
glucose 89 mg/dL, calcium 9.8 mg/dL, and urine pH 7.6. Arterial blood gas shows pO2 91
mmHg, pCO2 30 mmHg, and pH 7.26. Which of the following tests is most likely to reveal the
underlying diagnosis in this patient?

Choices:
1. Acid load test
2. Bicarbonate infusion test
3. Serum aldosterone level
4. Urine culture
Answer: 1 - Acid load test
Explanations:
The patient has a hyperchloremic non-anion gap metabolic acidosis with appropriate
respiratory compensation. It can be caused by gastrointestinal bicarbonate losses from
diarrhea, proximal renal tubular acidosis (RTA), hypokalemic distal RTA, type I, and
hyperkalemic distal RTA, type IV.
The fact that the patient has an elevated urine pH in the setting of acidosis points to a distal
RTA because the normal response would be a lowering of urine pH in the setting of
metabolic acidosis.
The acid load test is the specific test for infusing acid into the blood with 100 mg/kg of
ammonium chloride and checks urine pH hourly and plasma bicarbonate at a 3-hour
interval. A healthy person will be able to excrete acid and will decrease urine pH. Those
with distal RTA cannot excrete acid, and urine pH will remain basic despite increasingly
acidic serum. Plasma bicarbonate should drop below 21 mEq/L unless the patient vomits (in
which case the test should be repeated with antiemetic). If urine pH remains greater than 5.5
despite plasma bicarbonate of 21 mEq/L, the diagnosis of type 1 RTA is confirmed.
Treatment for distal renal tubular acidosis is with oral sodium bicarbonate, sodium citrate,
or potassium citrate.

Go to the next page if you knew the correct answer, or click the link image(s) below to further
research the concepts in this question (if desired).

Research Concepts:
Renal Tubular Acidosis

We update eBooks quarterly and Apps daily based on user feedback. Please tap flag to
report any questions that need improvement.
Question 952: A 65-year-old female patient presents with chest pain. The pain is described
as tearing and radiates to her back. Her blood pressure is 160/110 mmHg, her pulse rate is 92
beats/min, and her temperature is 98.6 F (37 C). She is suffering from hypertension,
dyslipidemia, obesity, and she has a 40 pack-years smoking history. On examination, the right
lower limb has all pulses palpable; however, on the left lower limb, there is a weak femoral
pulse, and all other pulses are absent. CT scan shows the displacement of intimal calcification.
What is the most common site of tears in this condition in general?

Choices:
1. Just distal to the left subclavian artery
2. Within a few centimeters of the proximal ascending aorta
3. At the proximal aortic arch
4. Distal to the descending aorta
Answer: 2 - Within a few centimeters of the proximal ascending aorta
Explanations:
Aortic dissection is due to the separation of the layers of the aortic wall. A tear in the
intimal layer results in the progression of the dissection (either proximal or retrograde)
chiefly due to the entry of blood between the intima and media.
Cystic medial degeneration plays a role in the pathology of aneurysmal formation and
dissection in the ascending aorta. Type 1 dissection involves the entire aorta, type 2
involves only the ascending aorta, and type 3 involves the descending aorta.
Other factors related to dissection in the ascending aorta are hypertension, cannulation
during cardiopulmonary bypass, Marfan syndrome, pregnancy, retrograde dissection, and a
bicuspid aortic valve.
The majority of dissections (95%) occur in two places. The most common site is within
several centimeters of the atrioventricular valve, and the second most common is just
beyond the ligamentum arteriosum in the descending aorta. The tear is usually transverse
and can progress in either a retrograde or an antegrade fashion. In the patient in the
scenario, the dissection is distal to these two sites, most likely in the distal aorta.

Go to the next page if you knew the correct answer, or click the link image(s) below to further
research the concepts in this question (if desired).

Research Concepts:
Aortic Dissection

We update eBooks quarterly and Apps daily based on user feedback. Please tap flag to
report any questions that need improvement.
Question 953: A 37-year-old woman who is 25 weeks pregnant (gravida 4, para 3) presents
to the emergency department with a two-hour history of vomiting. Her husband states that she
has never experienced such symptoms before. A clinical evaluation is significant for a blood
pressure of 160/110 mmHg and bilateral lower extremity edema. Laboratory investigations are
normal except for urinalysis (proteinuria 3+). The patient is now being treated and is
administered digoxin immune Fab. A few hours later, the patient started to clinically improve
with a reduction in blood pressure and severity of abdominal pain. Which of the following
possible mechanisms best explains the clinical utility of digoxin immune Fab in this patient?

Choices:
1. Upregulates tumor necrosis factor-alpha release
2. Binds to placental sodium-potassium ATPase pumps
3. Binds to vascular endothelial surface adhesion molecules
4. Binds endogenous compounds such as bufadienolide
Answer: 4 - Binds endogenous compounds such as bufadienolide
Explanations:
Lower extremity edema, elevated blood pressure, vomiting, and proteinuria in a pregnant
female are definitive clinical signs of pre-eclampsia. Pre-eclampsia is known to occur due to
incomplete trophoblastic invasion by the placenta, and it is considered an exaggerated
inflammatory condition compounded by endothelial dysfunction.
Several other biologically active factors are also implicated in its pathogenesis. One such
factor found in the maternal serum of pre-eclamptic patients is the endogenous digoxin-like
factors (EDLFs). The endogenous digoxin-like factor binds to and inhibits mesenteric artery
Na/K ATPase pumps, resulting in the accumulation of intracellular calcium leading to
vasoconstriction and maternal hypertension. These factors are also known to preserve
maternal renal function, lower maternal pulmonary edema, and the risk of intraventricular
hemorrhage in infants. Examples of EDLFs include cardenolide, ouabain, bufadienolide,
and marinobufagenin.
Digoxin immune Fab are anti-digoxin antibodies that can bind and neutralize endogenous
digoxin-like factors. More specifically, it reverses the Na/K ATPase pump inhibition caused
by EDLFs, resulting in an improvement in maternal symptoms and hypertension.
Digoxin immune Fab also reduces TNF-alpha-induced endothelial surface adhesion
molecules such as ICAM, VCAM, and E-selectin. It also inhibits TNF-alpha mediated
downregulation of Na/K ATPase pumps.

Go to the next page if you knew the correct answer, or click the link image(s) below to further
research the concepts in this question (if desired).

Research Concepts:
Digoxin Immune Fab

We update eBooks quarterly and Apps daily based on user feedback. Please tap flag to
report any questions that need improvement.
Question 954: A 42-year-old female presents to the emergency department with
breathlessness and an altered state. Her blood pressure is 90/58 mmHg, heart rate 121/min,
respiratory rate 33/min, temperature 99 F, O2 saturation of 67% on room air. She is started on
norepinephrine, intubated, and taken to the intensive care unit. She has a past medical history of
diabetes mellitus type 2, which is well controlled on metformin. Complete blood count and basic
metabolic panel are unremarkable. Chest X-ray shows cardiac silhouette within normal limits,
bilateral reticular, and alveolar infiltrates. Arterial blood gases are ordered, which demonstrates
pH 7.3, paCo2 55 mmHg, PaO2 90 mmHg while on FiO2 of 60%. What is the most appropriate
treatment?

Choices:
1. Tidal volume 10 ml/kg of predicted body weight, plateau pressure >40 cm H20
2. Tidal volume 12 ml/kg of predicted body weight, plateau pressure >30 cm H20
3. Tidal volume 6 ml/kg of predicted body weight, plateau pressure 30 cm H20
4. Tidal volume 2 ml/kg of predicted body weight, plateau pressure 30 cm H20
Answer: 3 - Tidal volume 6 ml/kg of predicted body weight, plateau pressure 30 cm H20
Explanations:
The patient has acute respiratory distress syndrome based on chest X-ray with bilateral
infiltrates present and PaO2/FiO2 ratio less than 300 based on the Berlin criteria.
Acute respiratory distress syndrome treatment is based on low tidal volume, approximately
6 ml/kg of predicted body weight, and a goal of maintaining plateau pressure 30 cm H20.
Acute respiratory distress syndrome can be diagnosed as mild, moderate, severe based on
the PaO2/FiO2 ratio. A ratio of less than 100 is severe, 100-200 is moderate, 200-300 is
mild.
In addition to identifying acute respiratory distress syndrome quickly and adjusting the vent
appropriately to improve mortality, these patients also benefit from early proning.

Go to the next page if you knew the correct answer, or click the link image(s) below to further
research the concepts in this question (if desired).

Research Concepts:
Fraction of Inspired Oxygen

We update eBooks quarterly and Apps daily based on user feedback. Please tap flag to
report any questions that need improvement.
Question 955: A 35-year-old man with a history of obesity is undergoing an elective
herniorrhaphy. Anesthesia is successfully induced. Right after giving a muscle relaxant, positive
pressure ventilation by face mask fails to provide an adequate seal. His oxygen saturation of the
patient begins to fall. What is the next best step in the management of this patient?

Choices:
1. Ask for a large plastic adhesive, cut a hole into it and place over the patient's mouth in an
attempt to make a seal
2. Rush to intubation as mask ventilation is failing
3. Attempt to find a larger mask in order to make a better seal
4. Place a second-generation laryngeal mask airway (LMA)
Answer: 4 - Place a second-generation laryngeal mask airway (LMA)
Explanations:
After induction of anesthesia, including a muscle relaxant, maintaining pre-oxygenation and
some ventilation is of paramount importance to avoid hypoxia. Especially in larger
individuals, redundant tissue may make positive pressure ventilation difficult. Once the face
mask fails to provide an adequate seal, an LMA is a good choice to provide a seal further
down the airway.
The LMA is designed to create a seal around the larynx in order to allow positive pressure
ventilation up to a certain pressure, which can vary between patients and the design of the
LMA. Once the patient's saturation begins to fall, swift and decisive action such as inserting
an LMA will help to prevent further hypoxia.
In addition to bypassing facial features, missing teeth, and full beards, LMAs can also be
useful to slide past redundant pharyngeal tissues which might obstruct the upper airway.
LMA's have different designs and features, such as silicone cuffs, gastric access ports, non-
inflatable gel cuffs. 2nd generation LMAs often provide higher seal pressure to allow for
higher positive pressure ventilation pressures and are preferred in a rescue situation. LMA's,
preferably 2nd generation, should be immediately available during induction of anesthesia if
airway rescue becomes necessary.

Go to the next page if you knew the correct answer, or click the link image(s) below to further
research the concepts in this question (if desired).

Research Concepts:
Laryngeal Mask Airway

We update eBooks quarterly and Apps daily based on user feedback. Please tap flag to
report any questions that need improvement.
Question 956: A 50-year-old female presents with daily generalized headaches for a
month. She reports the headaches are worst at night and in the morning before work. She
recently separated from her husband of 25 years and moved into an older house. She admits she
has been depressed and was started on an SSRI after her pet cat died in the basement. Her
neurologist gave her sumatriptan for the headaches. The patient is brought to the emergency
department by paramedics, unresponsive and flushed. What is the diagnosis?

Choices:
1. Serotonin syndrome
2. Lead poisoning
3. Carbon monoxide poisoning
4. Drug overdose
Answer: 3 - Carbon monoxide poisoning
Explanations:
Carbon monoxide from an old furnace could cause this scenario.
The cat died of chronic exposure, while the patient had headaches while at home.
Carbon monoxide binds to hemoglobin causing progressive cerebral anoxia.
Other symptoms associated with carbon monoxide are dizziness and flu-like symptoms
while patients with poisoning have flushing and pink skin.

Go to the next page if you knew the correct answer, or click the link image(s) below to further
research the concepts in this question (if desired).

Research Concepts:
Carbon Monoxide Toxicity

We update eBooks quarterly and Apps daily based on user feedback. Please tap flag to
report any questions that need improvement.
Question 957: A 56-year-old male farmer presents to the emergency department with a
chief complaint of chest pain, shortness of breath, and generalized fatigue. He reports that he was
feeding the animals when he had a sudden onset of symptoms. On exam, the patient is awake,
alert with mild respiratory distress. His vital signs are heart rate 110/min, blood pressure 127/63
mmHg, SpO2 84% on 4L nasal cannula, and respiratory rate 12/min. The nurse reports his blood
appears darker than normal. Which testing modality is most likely to reveal the diagnosis?

Choices:
1. Co-oximetry
2. Arterial blood gas
3. Venous blood gas
4. Chest x-ray
Answer: 1 - Co-oximetry
Explanations:
This patient has induced methemoglobinemia as a result of exposure to nitrogen dioxide.
The patient will have apparent hypoxemia, but ABG will show a normal partial pressure of
oxygen.
Co-oximetry will show a methemoglobin level greater than 15%.
Chocolate brown blood is a hallmark of methemoglobinemia.

Go to the next page if you knew the correct answer, or click the link image(s) below to further
research the concepts in this question (if desired).

Research Concepts:
Nitrogen Dioxide Toxicity

We update eBooks quarterly and Apps daily based on user feedback. Please tap flag to
report any questions that need improvement.
Question 958: A 61-year-old woman with a history of hypertension, type 2 diabetes,
paroxysmal atrial fibrillation, and obesity is brought to the hospital with severe 9/10 diffuse
abdominal pain that started suddenly 2 hours ago. She denies diarrhea/constipation. Her last
bowel movement was normal yesterday. Her pulse is irregular 108-115/min, blood pressure
135/70 mmHg, and respiratory rate 22/min. On physical exam, the abdomen is soft with mild
diffuse tenderness with no signs of peritonitis. WBC count is 12000/microL, hemoglobin 9.5
g/dL, lactate 2.2 mg/dL, and creatinine 2.4 mg/dL. Non-contrast abdominal CT scan shows mild
small bowel loops distention and thickening. She has never had abdominal surgery. The last
colonoscopy 5 years ago was normal. Which of the following is the next best step in the
management of this patient?

Choices:
1. Exploratory laparotomy
2. Diagnostic laparoscopy
3. Colonoscopy
4. CT angiography
Answer: 4 - CT angiography
Explanations:
This patient most likely has mesenteric ischemia. The most common presenting symptom is
abdominal pain. The patient’s symptoms and presentation may help determine the etiology
of the ischemia. An arterial embolism usually causes sudden, severe, periumbilical pain and
is associated with nausea and vomiting. Thrombotic mesenteric arterial occlusion is usually
associated with pain that is worse after eating. Patients with mesenteric venous thrombosis
usually have slower-onset, “waxing and waning” abdominal pain.
The physical exam may be normal. In fact, patients present with pain that is out of
proportion to the initial physical exam. There may be mild distension present, but peritoneal
signs only begin to show when transmural bowel infarction and necrosis develops.
CT angiography is the best non-invasive test for suspected mesenteric ischemia.
Exploratory laparotomy is indicated for peritonitis or bowel gangrene. Diagnostic
laparoscopy is reserved for patients with unclear diagnoses once all noninvasive tests are
done and still inconclusive.

Go to the next page if you knew the correct answer, or click the link image(s) below to further
research the concepts in this question (if desired).

Research Concepts:
Mesenteric Artery Ischemia

We update eBooks quarterly and Apps daily based on user feedback. Please tap flag to
report any questions that need improvement.
Question 959: A 65-year-old female with a lung malignancy presents to the emergency
department from her oncologist for a thoracentesis. She has shortness of breath and cough. Her
temperature is 101.4 F, blood pressure is 92/60 mmHg, respiratory rate is 28 breaths/min, and
her heart rate is 122/min. On physical examination, there are decreased breath sounds at the right
lung base. An ultrasound is performed, confirming a right-sided pleural effusion. While
performing thoracentesis, the patient's respiratory rate increases to 40/min. Which of the
following clinical course is expected during the procedure?

Choices:
1. The patient collapses and immediate resuscitation is needed.
2. The patient begins to improve clinically, oxygen saturation is unchanged, and he complains of
an abnormal sensation in the chest.
3. The patient begins to cough, oxygen saturation decreases, and he complains of a strange
sensation in the thoracic cavity as lung re-expands.
4. The patient remains asymptomatic and oxygen saturation remains unchanged.
Answer: 3 - The patient begins to cough, oxygen saturation decreases, and he complains of a
strange sensation in the thoracic cavity as lung re-expands.

Explanations:
As lungs reexpand, patients often will begin coughing and complain of a strange sensation
in the chest.
Oxygen saturation often decreases acutely secondary to ventilation-perfusion mismatch.
Blood is shunted away from the collapsed lung, following the procedure, the lung will have
increase ventilation but takes time for perfusion to occur.
Lung re-expansion is often uncomfortable for patients.
Imaging eliminates the concern for damage to nearby organs or other causes of chest pain.

Go to the next page if you knew the correct answer, or click the link image(s) below to further
research the concepts in this question (if desired).

Research Concepts:
Thoracentesis

We update eBooks quarterly and Apps daily based on user feedback. Please tap flag to
report any questions that need improvement.
Question 960: A 67-year-old man is with stage 3 prostate cancer is referred for cerebral
spinal fluid (CSF) testing. Previous investigations identified a lesion on the lower cervical cord
at C6 to C7 and multiple lumbar lesions from L2 to L5. The lesions seem intradural, and because
the primary tumor was prostate, diagnostic CSF has been requested. A lumbar puncture is
contraindicated due to the tumor invasion. Where is the best place to obtain a CSF sample in this
patient?

Choices:
1. Above the arch of C1
2. Between C2 and C3
3. Upper thoracic area, between T1 and T2
4. Lower thoracic area, between T11 and T12
Answer: 1 - Above the arch of C1
Explanations:
Suboccipital puncture is performed between occiput and C1 with a midline or lateral
approach.
Suboccipital puncture is indicated when there is a complete obstruction documented or for
the definition of the superior margin in a complete block.
Suboccipital puncture is contraindicated in patients with craniocervical deformities.
Below C2, this procedure is too risky because of the anatomy of the cervical and thoracic
spinal cord, which occupies most of the intradural compartment.

Go to the next page if you knew the correct answer, or click the link image(s) below to further
research the concepts in this question (if desired).

Research Concepts:
Suboccipital Puncture

We update eBooks quarterly and Apps daily based on user feedback. Please tap flag to
report any questions that need improvement.
Question 961: A 43-year-old male patient came into the emergency room with a decreased
level of consciousness and an acute episode of falling down. His vital signs were blood pressure:
65/40 mmHg, pulse rate: 130 beats per minute, respiratory rate: 24 /min, and temperature: 39 C.
He obeyed the commands and hardly replied to the questions. Following intravenous hydration
with 1000 cc of Ringer's lactate serum, his vital signs were blood pressure: 60/40 mmHg, pulse
rate: 120 bpm, respiratory rate: 23/min, and temperature: 38.9 C. His focused abdominal
ultrasonography for trauma (FAST) was positive for severe free fluid in Morrison's pouch and
pelvic area. His only past medical history is for depression and chronic pain syndrome. He
cannot remember the names of her medications. He is induced with general anesthesia and
started prophylactically on a vasopressor infusion. Shortly after beginning the infusion, he has a
much greater-than-expected elevation in blood pressure that remains much longer than expected
after turning off the infusion. What is most likely the mechanism of this event?

Choices:
1. Decreased clearance of norepinephrine from the synaptic space
2. Decreased pain receptors leading to hyperalgesia
3. Increased V2 activity at precapillary arterioles
4. The infusion was incorrectly mixed with a dextrose-containing solution
Answer: 1 - Decreased clearance of norepinephrine from the synaptic space
Explanations:
The effects seen here are most likely to reduced norepinephrine uptake from the synaptic
cleft.
Remember that the termination of norepinephrine’s effects is primarily via reuptake into the
presynaptic neuron.
Certain drugs, such as many antidepressants, will block the reuptake of norepinephrine,
both endogenous and exogenous administration.
Care is necessary when using norepinephrine concomitantly with monoamine oxidase
inhibitors or amitriptyline and imipramine-type antidepressants. The combination of any of
these drugs can lead to severe, prolonged hypertension.

Go to the next page if you knew the correct answer, or click the link image(s) below to further
research the concepts in this question (if desired).

Research Concepts:
Norepinephrine

We update eBooks quarterly and Apps daily based on user feedback. Please tap flag to
report any questions that need improvement.
Question 962: A 66-year-old man with a past medical history of alcohol use disorder is
brought to the hospital after a witnessed seizure. Ten days ago, he underwent a procedure for a
right chronic subdural hematoma using a burr hole and was discharged home two days later. The
patient is somnolent. The patient also reports headaches, fever, and chills for the past two days.
His temperature is 39 C. Physical examination reveals left upper extremity weakness. Which of
the following risk factors is most likely responsible for the patient’s condition?

Choices:
1. Epilepsy
2. Recent surgical procedure
3. Skull fracture
4. Alcohol use disorder
Answer: 2 - Recent surgical procedure
Explanations:
Predisposing factors associated with the development of subdural empyema include prior
cranial surgery, head trauma with open skull fractures or penetrating injury, infected
hematoma or subdural effusion, and not properly treated ear and sinus infections.
Infected subdural hematomas after surgical drainage may develop into empyema.
Up to 20% of the subdural empyemas occur following head trauma or cranial surgical
procedures.
Seizures can occur in subdural empyemas but are not predisposing factors.

Go to the next page if you knew the correct answer, or click the link image(s) below to further
research the concepts in this question (if desired).

Research Concepts:
Subdural Empyema

We update eBooks quarterly and Apps daily based on user feedback. Please tap flag to
report any questions that need improvement.
Question 963: A 16-year-old male presented to the emergency department following a
high-speed motor vehicle collision. The patient fractured both femurs and developed a tension
pneumothorax in his right lung. Chest intubation was done to relieve the pneumothorax. He was
transferred to the intensive care unit (ICU) and was kept on noninvasive ventilation (NIV).
Intravenous fluids were administered through a central percutaneous catheter, passed into the
medial cubital vein. At the time of discharge, while removing the catheter, the tip breaks off, and
the removed catheter is 1 cm shorter than the length inserted. What is the next best step?

Choices:
1. Ambulate the patient
2. Perform vagal maneuvers
3. Place a tourniquet above the insertion site
4. Place the patient in Fowler's position
Answer: 3 - Place a tourniquet above the insertion site
Explanations:
A tourniquet should be applied above the insertion site.
The patient should be placed on bedrest, monitored, and kept calm. The patient may be
asymptomatic or may have cyanosis, dyspnea, chest pain, hypotension, tachycardia,
increased central venous pressure (CVP), loss of consciousness, arrhythmias, or a
pulmonary thrombus.
X-ray confirmation of the catheter fragment is performed, and surgical removal may be
necessary.
The same treatment is indicated for a peripheral catheter.

Go to the next page if you knew the correct answer, or click the link image(s) below to further
research the concepts in this question (if desired).

Research Concepts:
Percutaneous Central Catheter

We update eBooks quarterly and Apps daily based on user feedback. Please tap flag to
report any questions that need improvement.
Question 964: A 65-year-old male comes to the clinician with fevers, malaise, weakness,
and shortness of breath for the past few days. Upon physical examination, there is a systolic
murmur along the left lower sternal border. There are tender nodules on the fingers. The
fundoscopy shows retinal hemorrhages. The diagnosis is made, and the patient is moved to the
critical care unit. Patients with which of the following conditions could have benefitted from the
antibiotic prophylaxis for the pathology seen in this patient?

Choices:
1. Previous pacemaker placement
2. Previous coronary artery bypass graft
3. Previous episode of bacterial endocarditis
4. Mitral valve prolapse without regurgitation
Answer: 3 - Previous episode of bacterial endocarditis
Explanations:
Indications for subacute bacterial endocarditis (SBE) prophylaxis include previous
endocarditis, prosthetic cardiac valve, congenital cardiac defects (except for those
specified), hypertrophic cardiomyopathy, previous valvular surgery, and rheumatic valvular
dysfunction.
Indications for SBE prophylaxis include surgically constructed systemic or pulmonic shunts
or conduits. Dental or surgical indications for subacute bacterial endocarditis prophylaxis
include procedures likely to cause mucosal bleeding, tonsillectomy/adenoidectomy, surgery
involving the respiratory or gastrointestinal mucosa, and rigid bronchoscopy.
Prophylaxis against endocarditis is not recommended in patients who are at risk of infective
endocarditis for other non-dental procedures, for example, TEE,
esophagogastroduodenoscopy, colonoscopy, or cystoscopy, in the absence of active
infection.
Currently, there is no indication for dental, gastrointestinal, or genitourinary procedural
prophylaxis for patients with implantable cardiovascular devices. However, prophylaxis
with an anti-staphylococcal antibiotic is indicated at the time of cardiovascular device
implantation and any subsequent manipulation of the surgically created device pocket.

Go to the next page if you knew the correct answer, or click the link image(s) below to further
research the concepts in this question (if desired).

Research Concepts:
Subacute Bacterial Endocarditis Prophylaxis

We update eBooks quarterly and Apps daily based on user feedback. Please tap flag to
report any questions that need improvement.
Question 965: A 17-year-old obese male patient involved in a motor vehicle accident had
to be emergently intubated in the field by the EMS personnel. Intubation was reported as difficult
because of his body habitus, and it required four attempts to successfully place the endotracheal
tube. The patient suffered a small subdural hemorrhage, which was conservatively managed. On
day 3, spontaneous awakening trial and spontaneous breathing trial (SBT) were done. The
patient was awake, following commands, and passed his SBT. What is the next important step
prior to extubating the patient?

Choices:
1. Obtain CT head prior to extubation
2. Perform cuff leak test
3. Administer intravenous lorazepam
4. Perform SBT for 6 hours
Answer: 2 - Perform cuff leak test
Explanations:
Cuff leak test should be routinely done prior to extubation, especially in someone who has
risk factors for laryngeal edema including difficult intubation, multiple intubation attempts,
airway injury, prolonged mechanical ventilation, etc.
Cuff leak test is performed by deflating the cuff of the endotracheal tube and checking for
an air leak around the tube.
A patient who has an absent cuff leak should be evaluated for airway or laryngeal edema
prior to extubation.
These patients with absent cuff leak should be treated with intravenous steroids for a couple
of days to treat the laryngeal edema prior to extubation to reduce the risk of stridor and
post-extubation respiratory failure.

Go to the next page if you knew the correct answer, or click the link image(s) below to further
research the concepts in this question (if desired).

Research Concepts:
Ventilator Weaning

We update eBooks quarterly and Apps daily based on user feedback. Please tap flag to
report any questions that need improvement.
Question 966: A 65-year-old woman is brought to the emergency department with an
intermittent loss of consciousness. ECG reveals prolongation of the QT interval. The patient
recently started antifungal therapy for a complicated skin infection. Her current medications are
omeprazole, pimozide, acetaminophen, and ketoconazole. A drug interaction between which of
the following medications most likely caused this patient presentation?

Choices:
1. Omeprazole and acetaminophen
2. Pimozide and acetaminophen
3. Ketoconazole and omeprazole
4. Ketoconazole and pimozide
Answer: 4 - Ketoconazole and pimozide
Explanations:
Ketoconazole can inhibit the metabolism of pimozide.
High plasma concentrations of pimozide can cause cardiac arrhythmias.
Pimozide is used to treat Tourette syndrome.
Ketoconazole is an antifungal.

Go to the next page if you knew the correct answer, or click the link image(s) below to further
research the concepts in this question (if desired).

Research Concepts:
Drug Metabolism

We update eBooks quarterly and Apps daily based on user feedback. Please tap flag to
report any questions that need improvement.
Question 967: A 30-year-old previously healthy patient is transferred to your intensive care
unit (ICU) for further management. He was diagnosed with influenza three days prior and
presented to another hospital with shortness of breath and developed acute respiratory failure
requiring intubation and mechanical ventilation. His oxygenation has continued to deteriorate,
and he has developed a requirement for significant ventilator settings. His pulse oximetry is only
reading 82% on FiO2 of 1 and a mean airway pressure of 30. Due to the patient's acuity and
potential for needing further support, he is transferred to a tertiary care center. On arrival, his
oxygenation index is 40, and his P: F ratio is 60. He is not requiring vasopressor support. He is at
a center that provides extracorporeal membrane oxygenation (ECMO) support, and it is
anticipated that this patient will need it. Of the following options, what is the best-standardized
approach?

Choices:
1. Alert your ICU team, surgical team, and ECMO team that this patient will likely need veno-
arterial ECMO.
2. Alert your ICU team, surgical team, and ECMO team that this patient will likely need veno-
venous ECMO.
3. Alert your surgical team and ICU team that this patient will need veno-arterial ECMO.
4. Continue to manage the patient with the current ventilator support and alert the ECMO team if
he were to deteriorate clinically.
Answer: 2 - Alert your ICU team, surgical team, and ECMO team that this patient will likely
need veno-venous ECMO.

Explanations:
Veno-venous ECMO support is an appropriate mode of ECMO support in this patient as his
cardiac function appears to be intact, as shown by his hemodynamic profile. Veno-venous
ECMO requires cannulation via the vena cavae with a double lumen cannula. This cannula
provides the ability to remove venous return and send it to the ECMO circuit for
oxygenation and carbon dioxide removal and sent back to the patient via the same cannula
in the second lumen directed towards the right side of the patient's heart. This mode requires
that the patient's cardiac function is intact.
Veno-venous ECMO is a mode that is appropriate for patients who have isolated respiratory
pathology. Through simulation-based training, checklists are often formed to assess patient
eligibility.
The patient in the scenario has acute respiratory distress syndrome (ARDS) that is being
managed with ventilatory settings that may be causing worsening baro- and volu-trauma to
his lungs. Continuing to manage the patient as he is maybe detrimental and cause further
damage.
When a patient may need ECMO support, it should be an approach that involves all teams
that would be involved in placing the patient on ECMO, management of the patient
afterward, and preparing and managing the ECMO circuit through the patient's course.

Go to the next page if you knew the correct answer, or click the link image(s) below to further
research the concepts in this question (if desired).

Research Concepts:
Extracorporeal Membrane Oxygenation Simulation

We update eBooks quarterly and Apps daily based on user feedback. Please tap flag to
report any questions that need improvement.
Question 968: At what interval is cardioversion most effective in patients with atrial
fibrillation (AF)?

Choices:
1. >14 days after onset of AF
2. >21 days after onset of AF
3. 7 days after onset of AF
4. >7 days after onset of AF
Answer: 3 - 7 days after onset of AF
Explanations:
During AF, the atria contract chaotically and irregularly when compared to the ventricles.
Cardioversion is most successful if attempted 7 days after onset of AF.
It should be done emergently for those with symptomatic (hypotension, heart failure or
angina) AF.
Because of the risk of thromboembolism, obtaining an echo is highly recommended before
cardioverting a patient. If a blood clot is seen in the left atrium, the patient should be
anticoagulated before the procedure.

Go to the next page if you knew the correct answer, or click the link image(s) below to further
research the concepts in this question (if desired).

Research Concepts:
Atrial Fibrillation

We update eBooks quarterly and Apps daily based on user feedback. Please tap flag to
report any questions that need improvement.
Question 969: A 68-year-old man with a history of major depressive disorder,
osteoarthritis, and type 2 diabetes mellitus is brought to the emergency department by his wife
due to attempted suicide. She says she is not sure what medication he ingested. His current
medications include lisinopril, fluoxetine, and over the counter naproxen as needed. He appears
drowsy and has sinus tachycardia; however, all other vitals are normal. On further investigation,
he is noted to have severe metabolic acidosis, and his BUN/Creatinine has increased significantly
compared to previous records. What is the next best step in the management of this patient?

Choices:
1. Supportive treatment only
2. Hemodialysis
3. Activated charcoal
4. Continuous monitoring only until the patient stabilizes
Answer: 2 - Hemodialysis
Explanations:
This patient presents to the emergency department with a possible overdose of naproxen.
Naproxen overdose is common due to its over the counter availability, but the overdose is
usually mild in severity, and serious adverse effects from overdose are rare.
There is no available antidote for naproxen overdose, however, monitoring of vital signs
and supportive care is recommended.
However, ingestion of large amounts of naproxen can lead to severe toxicity causing
seizures and metabolic acidosis, which can potentially cause renal failure. Therefore,
although hemodialysis is not generally recommended, in specific situations, it can correct
acid-base disturbances and provide additional support to those with renal impairment. The
role of activated charcoal is uncertain due to time constraints and unclear benefits.

Go to the next page if you knew the correct answer, or click the link image(s) below to further
research the concepts in this question (if desired).

Research Concepts:
Naproxen

We update eBooks quarterly and Apps daily based on user feedback. Please tap flag to
report any questions that need improvement.
Question 970: After coronary angioplasty, a patient develops decreased pedal pulses, livido
reticularis, abdominal pain, and confusion. Laboratories show BUN 120 mg/dL, creatinine 5.0
mg/dL, and phosphate 9.2 mg/dL. Urinalysis has 8 to 10 WBC/HPF, 5 10 to 15 RBC/HPF, and
few hyaline casts. Select the most likely diagnosis.

Choices:
1. Acute tubular necrosis
2. Cholesterol embolism
3. Renal artery thrombosis
4. Acute tubular necrosis secondary to hypotension
Answer: 2 - Cholesterol embolism
Explanations:
Disruptions of plaques in the aorta or renal arteries during left heart catheterization can
dislodge cholesterol.
This can affect peripheral vasculature, bowel, pancreas, and kidneys.
The urinalysis can show amorphous sediment or hyaline casts but, with ATN, granular casts
would be more likely.
Eosinophiluria and decreased complement levels are possible.

Go to the next page if you knew the correct answer, or click the link image(s) below to further
research the concepts in this question (if desired).

Research Concepts:
Cholesterol Emboli

We update eBooks quarterly and Apps daily based on user feedback. Please tap flag to
report any questions that need improvement.
Question 971: A 30-year-old female underwent a popliteal fossa nerve block in preparation
for a complex ankle repair with ultrasound guidance to facilitate placement of 35 mL of 0.25%
bupivacaine. Five minutes after administration of the nerve block, the patient experiences
confusion followed by rapid cardiovascular collapse and asystole on the electrocardiogram. Lipid
emulsion is not available from the pharmacy due to a nationwide shortage. Standard advanced
cardiac life support measures are ineffective after 20 minutes. Which of the following measures
should be considered for this patient?

Choices:
1. Emergent plasmapheresis
2. Emergent dialysis
3. Emergency placement of a left ventricular assist device
4. Emergent cardiopulmonary bypass
Answer: 4 - Emergent cardiopulmonary bypass
Explanations:
Prior to lipid emulsion being a treatment of choice for local anesthetic systemic toxicity, a
cardiopulmonary bypass in order to support the patient's cardiovascular status was used as a
life-saving maneuver.
Bupivacaine is a long-acting local anesthetic, so prolonged support may be needed in cases
of profound toxicity.
Because access to cardiopulmonary bypass takes time and the expertise of surgical
colleagues and support staff, this therapy needs to be considered early so that preparations
can be in place. Smaller hospitals may not have access to the equipment or staff needed.
A left ventricular assist device would take too long to place and is not appropriate therapy
for this situation. Due to particle size, dialysis or plasmapheresis would not be effective at
removing bupivacaine from the circulation and are not appropriate treatments.

Go to the next page if you knew the correct answer, or click the link image(s) below to further
research the concepts in this question (if desired).

Research Concepts:
Bupivacaine

We update eBooks quarterly and Apps daily based on user feedback. Please tap flag to
report any questions that need improvement.
Question 972: A 45-year-old male has a blood transfusion after being involved in a motor
vehicle collision. Forty minutes after, he reports chills, discomfort at the infusion site, and severe
flank pain. His vital signs show oxygen saturation of 98% on room air, respiratory rate of 21
breaths per minute, heart rate 125 beats per minute, blood pressure 135/83 mmHg, and
temperature 102.7 F (39.2 C). The transfusion is stopped, but he has no relief of the symptoms.
His catheter bag becomes red, and he begins to bleed around his intravenous catheter site. Which
of the following is the most likely cause of this reaction?

Choices:
1. ABO incompatibility
2. Febrile non-hemolytic transfusion reaction
3. Reaction to cytokines stored in the transfused blood
4. IgA deficiency
Answer: 1 - ABO incompatibility
Explanations:
Acute hemolytic transfusion reaction (AHTR) typically develops within an hour after the
transfusion is started. It is a medical emergency that is usually caused by ABO
incompatibility; symptoms include fever, chills, hemoglobinuria, flank pain, and discomfort
at the infusion site.
This may advance to renal failure due to immune complex deposition or disseminated
intravascular coagulation (DIC) (e.g., bleeding from the intravenous site in this patient).
The management includes the immediate cessation of transfusion while maintaining
intravenous access for fluids (normal saline) and supportive care (diuretics such as
mannitol).
Massive blood transfusions can often cause hypothermia, hypocalcemia, and hyperkalemia.

Go to the next page if you knew the correct answer, or click the link image(s) below to further
research the concepts in this question (if desired).

Research Concepts:
Hemolytic Transfusion Reaction

We update eBooks quarterly and Apps daily based on user feedback. Please tap flag to
report any questions that need improvement.
Question 973: A 31-year-old marathon runner presented with a complaint of fatigue for the
past day. Her blood pressure was measured which turned out to be 200/130 mmHg. She had no
previous memory of ever measuring her blood pressure. Her medical and family histories were
unremarkable. Her other vital signs were normal and physical examination was unremarkable.
The targeted blood pressure in the next 24hours is 130/80 mmHg. What would be the most
appropriate way of normalizing her blood pressure?

Choices:
1. 5 to 15% in the 1st hour and 10 to 20% over the next 24 hours.
2. 10 to 20% in the 1st hour and 10 to 20% over the next 24 hours.
3. 10 to 20% in the 1st hour and 5 to 15% over the next 24 hours.
4. 5 to 15% in the 1st hour and 5 to 15% over the next 24 hours.
Answer: 3 - 10 to 20% in the 1st hour and 5 to 15% over the next 24 hours.
Explanations:
Malignant hypertension is extremely high blood pressure that develops rapidly and causes
some type of organ damage. About 1% of people who have a history of high blood pressure
develop this life-threatening condition.
It is not recommended to decrease blood pressure too fast or too much, as ischemic damage
can occur in vascular territories that have become habituated with the elevated level of
blood pressure.
For the majority of hypertensive emergencies, mean arterial pressure (MAP) should be
reduced by approximately 10 to 20% within the first hour and by another 5% to 15% over
the next 24 hours.
This often results in a target BP of less than 180/120 mm Hg for the first hour and less than
160/110 mm Hg for the next 24 hours, but rarely less than 130/80 mm Hg during that time
frame.

Go to the next page if you knew the correct answer, or click the link image(s) below to further
research the concepts in this question (if desired).

Research Concepts:
Malignant Hypertension

We update eBooks quarterly and Apps daily based on user feedback. Please tap flag to
report any questions that need improvement.
Question 974: A 55-year-old man was recently diagnosed with hypertension. He was
started on hydrochlorothiazide and amiloride. The coadministration of thiazide with amiloride is
done because amiloride does which of the following?

Choices:
1. Potentiates the effect of antidiuretic hormone
2. Has anticholinergic action
3. Blocks the epithelial sodium channel in the kidney
4. Inhibits carbonic anhydrase in the proximal convoluted tubule
Answer: 3 - Blocks the epithelial sodium channel in the kidney
Explanations:
Both amiloride and triamterene work by inhibiting the epithelial sodium channels (ENaC) in
the distal nephron. Amiloride is co-administered with thiazide to prevent thiazide-induced
hypokalemia.
Normally the sodium moves down its electrochemical gradient to enter the tubular cells
through the ENaCs. This gradient is created by the basolateral membrane Na/K ATPase.
Reabsorption of sodium is associated with depolarization of the apical membrane which
creates a lumen-negative transepithelial potential difference. This potential difference
enhances potassium secretion through the apical potassium channels and subsequently
potassium excretion
Amiloride selectively inhibits ENaCs resulting in a decrease in hyperpolarization of the
apical membrane and subsequently decrease in potassium, hydrogen, calcium, and
magnesium secretion
It should be noted that other potassium-sparing diuretics, spironolactone, and eplerenone,
act via a different mechanism. These drugs are aldosterone antagonists and they are helpful
in cases of secondary hyperaldosteronism in edematous states like congestive heart failure

Go to the next page if you knew the correct answer, or click the link image(s) below to further
research the concepts in this question (if desired).

Research Concepts:
Amiloride

We update eBooks quarterly and Apps daily based on user feedback. Please tap flag to
report any questions that need improvement.
Question 975: A 48-year-old man with severe obesity and a history of IV drug use end-
stage renal disease on dialysis is brought to the hospital with shortness of breath. Shortly after
arrival, the patient suffers a cardiac arrest. IV access cannot be obtained. What is the most
appropriate access to obtain at this time?

Choices:
1. IV access using ultrasound
2. A central line via the patient's femoral vein
3. An intraosseous line
4. An external jugular IV access
Answer: 3 - An intraosseous line
Explanations:
In a critically ill patient, IO access can provide quick access when IV access cannot be
obtained promptly.
Patients with a history of IV drug use, obesity, hemodialysis, or hypotensive will be more
difficult to obtain IV access and will likely take longer to obtain a peripheral IV even with
the guidance of ultrasound.
When these patients are critically ill, do not delay care by attempting ultrasound-guided IV
multiple times. It might be more prudent to switch to another form of access.
While there are no absolute contraindications to the use of ultrasound-guided IV access,
there are relative contra-indications such as multiple failed attempts, infections, or injuries
to the site, or when time is critical.

Go to the next page if you knew the correct answer, or click the link image(s) below to further
research the concepts in this question (if desired).

Research Concepts:
Ultrasound Intravascular Access

We update eBooks quarterly and Apps daily based on user feedback. Please tap flag to
report any questions that need improvement.
Question 976: An 84-year-old man with a past medical history of hypertension,
hyperlipidemia, benign prostatic hyperplasia, and bladder cancer has been admitted to the
intensive care unit with acute hypoxemic respiratory failure in the setting of sepsis due to a
urinary tract infection. The patient has been intubated for four days and is unable to be weaned
off the vent. The ICU team orders a nerve conduction study and electromyography to rule out
critical illness neuropathy or myopathy. Which of the following is the most appropriate reason to
avoid performing this study on this patient?

Choices:
1. Subcutaneous heparin use
2. Cellulitis of the patient's leg
3. Too early in the disease process to find electrodiagnostic evidence
4. Too much electrical interference in the ICU setting
Answer: 3 - Too early in the disease process to find electrodiagnostic evidence
Explanations:
Wallerian degeneration (axonal denervation) may take at least seven days or longer before
becoming evident in an electrodiagnostic study.
This patient has only been septic and intubated for four days and therefore it is unlikely that
the study will provide any clinical use at this point in the admission.
If the patient remains ventilated for longer, NCS/EMG should be performed to assess
critical illness neuropathy/polyneuropathy.
While caution should be taken in performing a needle study on patients who are on blood
thinners, one of the only true contraindications to an EMG study would be in a patient with
a severe bleeding disorder or who has an uncontrolled coagulation profile such a
supratherapeutic INR while on warfarin. Subcutaneous heparin is not the reason to postpone
this exam at this time. EMGs should not be performed directly over an area of soft tissue
infection such as cellulitis. However, a complete study could still be performed using the
other 3 limbs and would still provide enough electrodiagnostic information. In this case, the
cellulitis does not prevent the healthcare provider from performing the exam at this time,
but the patient has only been in the ICU for four days, and therefore an EMG at this time
will likely not show significant results. Although it is true that the electrical interference in
the ICU setting may create a suboptimal environment to perform electrodiagnostic studies,
the test can still be performed. Some strategies to improve the quality of the study include
ensuring electrodes properly adhere, using the notch filter on the machine, turning off
unnecessary machines or monitors, and unplugging the bed.

Go to the next page if you knew the correct answer, or click the link image(s) below to further
research the concepts in this question (if desired).

Research Concepts:
Electrodiagnostic Evaluation Of Critical Illness Neuropathy

We update eBooks quarterly and Apps daily based on user feedback. Please tap flag to
report any questions that need improvement.
Question 977: A 42-year-old male, who has been in the medical intensive care unit for 2
weeks due to acute respiratory distress syndrome (ARDS) has been failing his spontaneous
breathing trials (SBTs) for 4 straight days. Before the start of SBT, he is awake and alert and
hemodynamically stable. Fifteen minutes into his SBT, his tidal volumes are in the range of 100-
150 mL with a respiratory rate of 35/minute. He becomes lethargic and is minimally responsive
now. He is switched back to full ventilator support. Arterial blood gas obtained at this time
shows a pH of 7.2, pCO2 60 mmHg, and pO2 100 mmHg. Chest x-ray shows low lung volumes
with otherwise normal parenchyma. Ultrasound of the diaphragm shows bilateral diaphragm
thickness of 10 mm with thickening to 11 mm on deep inspiration. What is the next best step in
management?

Choices:
1. Proceed with a diaphragmatic pacer
2. Surgical diaphragmatic plication
3. Discuss tracheostomy and transfer to long term acute care hospital (LTACH)/rehabilitation
facility
4. Continue with daily SBT trials
Answer: 3 - Discuss tracheostomy and transfer to long term acute care hospital
(LTACH)/rehabilitation facility

Explanations:
The patient has bilateral diaphragmatic atrophy and weakness from prolonged mechanical
ventilation, as evidenced by a thinned diaphragm and reduced fractional shortening on
maximal inspiration. Critical illness polyneuropathy (CIP) is a common complication of
critical illness affecting the motor and sensory neurons. Muscle involvement causing loss of
muscle mass and eventual weakness has been referred to as critical illness myopathy.
These patients have worse outcomes with prolonged weaning, higher hospital length of
stays, and dependency on mechanical ventilation.
There is no current evidence of using diaphragmatic pacing for these patients. Trials
examining their role are currently undergoing. The diaphragm strength and function would
usually recover over time with intensive physical therapy and nutrition. Therefore,
tracheostomy and transfer to LTACH/rehabilitation center is a reasonable approach.
Diaphragm ultrasound, if used early in the weaning phase, can help identify patients who
are at risk of weaning or extubation failure.

Go to the next page if you knew the correct answer, or click the link image(s) below to further
research the concepts in this question (if desired).

Research Concepts:
Diaphragm Disorders

We update eBooks quarterly and Apps daily based on user feedback. Please tap flag to
report any questions that need improvement.
Question 978: A 65-year-old man comes to the emergency department with a complaint of
tongue deviation. He has a history of diabetes mellitus for ten years, but he is not compliant with
the medications. He has smoked 30 cigarettes a day for the past 25 years. On examination, his
blood pressure is 160/95 mm Hg, pulse is 85 bpm, and respiratory rate is 20/min. On
neurological examination, there is a decrease in the power of the muscle on the left side of the
body. Decreased position and vibratory sense in the left side of the body is seen, but other
sensations are normal. The tongue deviates to the right, and while the rest of the cranial nerve
examination is unremarkable. Which imaging modality should be used to visualize the lesion?

Choices:
1. Computerized tomography (CT) scan
2. Magnetic resonance imaging (MRI) of the brain
3. Ultrasonogram (USG) of the brain
4. Computerized tomography (CT) angiogram of the brain
Answer: 2 - Magnetic resonance imaging (MRI) of the brain
Explanations:
Contralateral hemiplegia, ipsilateral hypoglossal dysfunction, and contralateral decrease
proprioception is the diagnostic triad of medial medullary syndrome or Dejerine syndrome.
For the diagnosis of medial medullary syndrome, one can do both CT scan and MRI. MRI
can visualize the brain lesion better than a CT scan because it is difficult to visualize
posterior cranial fossa structures by CT scan, which may be obscure by the bony structures.
The patient tongue deviates to the left, and there is a weakness in the right side of the body.
So the diagnosis is medial medullary syndrome or Dejerine syndrome. The patient has two
risk factors (diabetes mellitus and smoking) of atherosclerosis. These risk factors are among
the most common risk factors for Dejerine syndrome.
The Dejerine syndrome is most commonly caused by atherothrombotic occlusion of
paramedian branches of the anterior spinal artery or vertebral artery or basilar artery.
Common risk factors are dyslipidemia, hypertension, diabetes, and smoking. Other risk
factors are atrial fibrillation, atrial septal defect, migraine, and Takayasu arteritis. Dissection
of the vertebral artery can also cause medial medullary syndrome, especially among young
patients.

Go to the next page if you knew the correct answer, or click the link image(s) below to further
research the concepts in this question (if desired).

Research Concepts:
Medial Medullary Syndrome

We update eBooks quarterly and Apps daily based on user feedback. Please tap flag to
report any questions that need improvement.
Question 979: A 60-year old male presents to the emergency department (ED) with fatigue
and weakness. His blood pressure is 76/62 mmHg, and his heart rate is 49 beats per minute. EKG
indicates PR prolongation. He states that his only medication is verapamil. A urine toxicology
screen is negative. He continues to be hypotensive and bradycardic despite IV bolus of normal
saline and 3 atropine doses. Which of the following is an appropriate next step?

Choices:
1. Intraarterial calcium gluconate infusion delivered over 3 hours
2. Intravenous calcium gluconate infusion and recheck calcium level
3. Intravenous pralidoxime bolus administered twice
4. Increase verapamil dosage and monitor for symptomatic improvement
Answer: 2 - Intravenous calcium gluconate infusion and recheck calcium level
Explanations:
Hypotension, bradycardia, PR prolongation on EKG are characteristics of calcium channel
blocker (CCB) toxicity, likely from verapamil. Calcium gluconate is part of the treatment
for patients with persistent shock despite fluid resuscitation and atropine in patients with
CCB toxicity.
CCB toxicity should be treated with intravenous (not intraarterial) calcium gluconate.
Dosing is a bolus of 0.6 mL/kg of 10% calcium gluconate solution, followed by a
continuous infusion of 0.6-1.5 mL/kg/hr.
Throughout the CCB toxicity treatment, ionized calcium levels should be monitored to
achieve a goal of calcium level two times normal.
Pralidoxime is administered for organophosphate poisoning. It has no role in the treatment
of CCB toxicity. Verapamil dosage should not be increased in this patient who in shock
likely due to toxicity from this medication. Intraarterial calcium might be utilized during
moderate to severe burn.

Go to the next page if you knew the correct answer, or click the link image(s) below to further
research the concepts in this question (if desired).

Research Concepts:
Calcium Gluconate

We update eBooks quarterly and Apps daily based on user feedback. Please tap flag to
report any questions that need improvement.
Question 980: A 53-year-old man is brought to the emergency department with collapse
and seizure. The patient was on a long walk with his wife and had missed lunch. The patient was
well until 6 weeks ago. For the last 6 weeks, the patient has been complaining of occasional
sudden onset sweating, tremors, blurred vision, and mental fog. These episodes usually occurred
after exertion or delaying his meal. His past medical history includes hypertension, for which he
takes amlodipine. His family history includes ischemic heart disease and type 2 diabetes.
Physical examination is unremarkable. Point-of-care blood glucose is 40 mg/dL. Insulin, C-
peptide, and beta-hydroxybutyrate are ordered. Which of the following is the next best step in the
evaluation of this patient?

Choices:
1. Thyroid function test
2. 72-hour fast
3. Liver function tests
4. Serum glucose by laboratory
Answer: 4 - Serum glucose by laboratory
Explanations:
The patient has a low blood glucose level on fingerprick testing.
However, it is important to remember that a glucometer's blood glucose reading can
occasionally give false and inaccurate readings.
Therefore, any abnormal blood glucose readings of a glucometer should be confirmed with
a serum glucose measurement. Furthermore, C-peptide and insulin levels can only be
accurately interpreted with the serum glucose.
Thyroid function would be part of the panel. There have been case reports of hypoglycemia
in patients with severe hypothyroidism or hyperthyroidism. 72-hour fast is an important test
to undertake for diagnosing hypoglycemia and its cause. This test may need to be arranged
at a later date. A liver function test would also form part of the panel.

Go to the next page if you knew the correct answer, or click the link image(s) below to further
research the concepts in this question (if desired).

Research Concepts:
Non-diabetic Hypoglycemia

We update eBooks quarterly and Apps daily based on user feedback. Please tap flag to
report any questions that need improvement.
Question 981: A 40-year-old woman with severe persistent asthma presents to the
emergency department (ED). She is well known to the intensive care unit, having required
intubation with mechanical ventilation in the past. On initial evaluation, the patient is
maintaining adequate oxygen saturation; however, she is tachypneic while on non-invasive
positive pressure ventilation, and her chest examination reveals diffuse wheezing. In the ED, the
patient is intubated, and her transport to the ICU is to be expedited, following portable chest
radiography. After arriving at the ICU, the respiratory therapist notes elevated peak inspiratory
pressures, unequal chest wall expansion, and unilateral wheezing. Attempts to view the post-
intubation radiograph are unsuccessful as the file is corrupted. Bedside thoracic sonography
reveals the presence of lung sliding in the right lung field. While lung sliding is absent in the left
lung field, rhythmic pleural movements in concert with the cardiac cycle are noted. Suddenly, the
patient develops hypoxia. A normal waveform is seen on the pulse oximeter. Which of the
following is the next best step in the management of this patient?

Choices:
1. Reposition the pulse oximeter
2. Increase the PEEP
3. Perform an emergent left-sided needle thoracostomy
4. Retract the ETT
Answer: 4 - Retract the ETT
Explanations:
Right main bronchus intubation requires prompt identification and treatment with retraction
of the ETT to a suitable position (=2 cm) above the tracheal carina.
In this patient, elevated peak inspiratory pressures, unequal chest wall expansion, and
unilateral wheezing are highly suggestive of right main bronchus intubation. As mentioned
in the article, airflow is required to produce wheezing. The absence of wheezing in the left
lung field is consistent with the absence of airflow.
It is imperative that proper placement of the endotracheal tube be established as soon as
possible after its insertion. Clinical examination should always be performed; however, it is
insufficient for the determination of ETT placement. Quantitative capnography is a simple,
but accurate method, for confirming the placement of the ETT in the airway (rather than the
esophagus) and should always be performed. This method, however, cannot determine the
depth of the ETT in the trachea, which is best established with a chest radiograph.
The findings of bedside thoracic sonography effectively rule out pneumothorax, making
emergent needle thoracostomy an incorrect answer choice. Additionally, while increasing
PEEP is a solution to hypoxia, it is inappropriate in this clinical scenario, as doing so could
potentiate barotrauma, given the already elevated peak inspiratory pressures. Lastly, the
repositioning of the pulse oximeter would be futile, given the normal waveform on the pulse
oximeter, making this an incorrect answer choice.

Go to the next page if you knew the correct answer, or click the link image(s) below to further
research the concepts in this question (if desired).

Research Concepts:
Wheezing

We update eBooks quarterly and Apps daily based on user feedback. Please tap flag to
report any questions that need improvement.
Question 982: An 85-year-old male is brought to the emergency department after being
assaulted by a group of teenagers in a pub. He was stabbed in the left chest and the abdomen. His
blood pressure is 80/50 mm Hg, and his pulse rate is 114/min. A chest x-ray reveals a left-sided
tension pneumothorax, and an emergent ultrasound shows a puncture in the spleen. A left-sided
chest tube insertion is performed to relieve the tension pneumothorax. A laparotomy is planned
to repair the damaged spleen. A central venous line is passed in the internal jugular vein to
administer fluids. During central venous access of this vein, the needle is directed towards what
at a 45-degree angle?

Choices:
1. Hip
2. Posterior spine
3. Ipsilateral nipple
4. Contralateral nipple
Answer: 3 - Ipsilateral nipple
Explanations:
During central venous access of the internal jugular vein, the finder needle is angled at a 45-
degree angle towards the patient's ipsilateral nipple.
The catheter is inserted into the internal jugular vein between the clavicular heads of the
sternocleidomastoid, at the apex of the triangle formed by the clavicle and the heads of the
muscles.
Ultrasound guidance reduces failure rate, number of attempts, misplacement, and
subsequent line sepsis.
The bottom line is that one must know the anatomy of the jugular vein. It is lateral to the
carotid artery. Complications occur when the line is inserted by physicians who have little
experience.

Go to the next page if you knew the correct answer, or click the link image(s) below to further
research the concepts in this question (if desired).

Research Concepts:
Internal Jugular Vein Central Venous Access

We update eBooks quarterly and Apps daily based on user feedback. Please tap flag to
report any questions that need improvement.
Question 983: A 66-year-old female presented to the hospital with a complaint of an
episode of blood in the urine that appeared an hour ago. She also complains of pain in her left
flank. On further questioning, she reveals that she has had a burning sensation with urination for
the past two days. A detailed medical history reveals that she was diagnosed as a case of diabetes
mellitus twelve years ago, and her blood sugar levels are well controlled. Her vital signs show
blood pressure 120/80 mm Hg, heart rate 110/min, respiratory rate 16/min, and temperature
102.2 F (39 C). A physical examination performed reveals left flank tenderness on palpation. A
urinalysis performed reveals 15 white blood cells per cubic millimeter of centrifuged urine. What
is the complication associated with her condition that causes necrotizing infection?

Choices:
1. Xanthogranulomatous pyelonephritis
2. Emphysematous pyelonephritis
3. Corticomedullary abscess
4. Acute lobar nephroma
Answer: 2 - Emphysematous pyelonephritis
Explanations:
The clinical scenario is most consistent with complicated acute pyelonephritis as the patient
is above 65. Acute pyelonephritis is an ascending infection of the urinary tract.
Emphysematous pyelonephritis is a severe necrotizing infection that occurs as a
complication of acute pyelonephritis of the kidney.
It is usually caused by Escherichia coli or Klebsiella pneumonia and occurs mostly in
females. It is associated with diabetes mellitus.
Corticomedullary abscess, acute lobar nephroma, xanthogranulomatous pyelonephritis are
complications of acute pyelonephritis as well but are non-necrotizing complications.

Go to the next page if you knew the correct answer, or click the link image(s) below to further
research the concepts in this question (if desired).

Research Concepts:
Acute Pyelonephritis

We update eBooks quarterly and Apps daily based on user feedback. Please tap flag to
report any questions that need improvement.
Question 984: A 30-year-old woman presents to the emergency department with severe
breathing difficulty. She has a history of asthma. Chest auscultation reveals bilateral wheeze, and
she is unable to complete a sentence. Treatment is commenced with back-to-back nebulizations
with salbutamol and ipratropium bromide; IV hydrocortisone is also given. She is confused and
exhausted. Rapid sequence induction and intubation with a size 8.0 endotracheal tube is
performed. However, it is impossible to ventilate the patient, and the saturations drop to 70% on
100% oxygen. Which of the following is the next best step in the management of this patient?

Choices:
1. Check the position and patency of the endotracheal tube
2. Needle decompression
3. Administer further salbutamol and ipratropium nebulizers
4. Magnesium sulfate
Answer: 1 - Check the position and patency of the endotracheal tube
Explanations:
If it is impossible to bag a patient after intubation, you must ensure your endotracheal tube's
patency and position. This question is really more about testing your ability to apply
common sense in a stressful situation rather than acute severe asthma management.
Severe bronchospasm may make ventilation very difficult, and only very low tidal volumes
may be achievable, but a complete obstruction to any gas flow is unlikely.
Gas trapping makes over-distension a hazard, and there is a high risk of barotraumas and
pneumothoraces. If the patient is truly impossible to ventilate, you should disconnect the
circuit and manually decompress the chest before further ventilation attempts.
Generally, ventilation strategies should include low tidal volumes and a low respiratory rate
with prolonged expiratory times. The use of PEEP is controversial.

Go to the next page if you knew the correct answer, or click the link image(s) below to further
research the concepts in this question (if desired).

Research Concepts:
Status Asthmaticus

We update eBooks quarterly and Apps daily based on user feedback. Please tap flag to
report any questions that need improvement.
Question 985: A 58-year-old male with a history of hypertrophic cardiomyopathy presents
with orthopnea, dyspnea at rest, and 15 lb (7 kg) weight gain. He has been managed previously
on metoprolol and verapamil. An echocardiogram reveals reduced ejection fraction and left
ventricular chamber enlargement. Which management strategy is best for this patient?

Choices:
1. Cardioversion
2. Septal ablation
3. Add disopyramide
4. Discontinue verapamil and add diuretic and ace inhibitor
Answer: 4 - Discontinue verapamil and add diuretic and ace inhibitor
Explanations:
A small percentage of patients with hypertrophic cardiomyopathy (HCM) develop systolic
dysfunction and a clinical picture consistent with dilated cardiomyopathy.
Verapamil should be discontinued as it can reduce output and more conventional heart
failure medical optimization should be implemented.
Cardioversion is appropriate for managing atrial fibrillation as a complication of HCM.
Septal ablation is considered in cases of severe symptomatic obstructive HCM.

Go to the next page if you knew the correct answer, or click the link image(s) below to further
research the concepts in this question (if desired).

Research Concepts:
Hypertrophic Cardiomyopathy

We update eBooks quarterly and Apps daily based on user feedback. Please tap flag to
report any questions that need improvement.
Question 986: A 61-year-old man is brought to the emergency department for abrupt loss
of consciousness after a fall at his home that happened 2 hours ago. His past medical history is
notable for atrial fibrillation. He has otherwise been well without additional medical problems.
His medications include warfarin and metoprolol. Currently, his temperature is 98.6, his blood
pressure is 125/75 mm Hg, his pulse rate is 77/min and irregular, and his respiration rate is
17/min. The patient is obtunded without localizing neurologic findings. The cardiac examination
reveals an irregularly irregular rhythm. The remainder of the examination is unremarkable. A
head CT scan shows a large subdural hematoma. Lab values indicate a hemoglobin level of 12.8
g/dL, platelet count of 283,000/µL, and INR of 3.0. Intravenous vitamin K is administered, and
plans are made for emergent neurosurgery. Which of the following is the next most appropriate
step in managing this patient?

Choices:
1. Fresh frozen plasma
2. Factor IX complex
3. Cryoprecipitate
4. Idarucizumab
Answer: 2 - Factor IX complex
Explanations:
Factor IX complex, known as prothrombin complex concentrate (PCC), should be used to
reverse the effects of warfarin anticoagulation in patients experiencing severe bleeding and
those requiring urgent surgery.
Although vitamin K alone can be effective in reversing the effect of warfarin, its hemostatic
effect can take several hours.
In urgent situations, simultaneous replacement of the vitamin K–dependent coagulation
factors is necessary.
Cryoprecipitate would be indicated to treat severe hypofibrinogenemia, usually arising as a
consequence of disseminated intravascular coagulation (DIC) or severe liver disease.
Idarucizumab is a monoclonal antibody that binds the non–vitamin K antagonist oral
anticoagulant dabigatran and causes a rapid reduction in available dabigatran in the body for
up to 24 hours. Idarucizumab will not reverse warfarin anticoagulation.

Go to the next page if you knew the correct answer, or click the link image(s) below to further
research the concepts in this question (if desired).

Research Concepts:
Prothrombin Complex Concentrate

We update eBooks quarterly and Apps daily based on user feedback. Please tap flag to
report any questions that need improvement.
Question 987: A 68-year-old male with a past medical history of hyperlipidemia,
hypertension, and benign prostatic hypertrophy is admitted to the hospital with septic shock. He
is treated with intravenous fluids, piperacillin-tazobactam, and norepinephrine. On day 2 of
admission, he complains of abdominal pain in the left lower quadrant. The pain started
approximately 2 hours ago and 50 minutes before the evaluation, he had an episode of
hematochezia. His home medications include atorvastatin, low-dose aspirin, and chlorthalidone.
On physical examination today, the patient is alert and oriented. Vital signs reveal temperature
38.7 C (101.6 F), blood pressure 100/60 mmHg, pulse rate 85/min, respiratory rate 19/min, and
oxygen saturation 94% on ambient air. Abdominal examination reveals tenderness over the left
side of the abdomen without guarding. Which of the following is the most appropriate next step
in the management of this patient?

Choices:
1. CT angiography
2. MR angiography
3. Plain radiography
4. Sigmoidoscopy
Answer: 1 - CT angiography
Explanations:
Patients with acute colonic ischemia commonly present with sudden onset cramping
abdominal pain, which usually involves the left side. The pain can be accompanied by an
urgent desire for a bowel movement. Instead of the periumbilical pain characteristic of
small intestine ischemia, patients with colonic ischemia usually feel the pain laterally.
In hemodynamically stable patients with clinical features of acute colonic ischemia, a
definitive diagnosis requires advanced abdominal imaging. The initial test of choice is
computed tomographic (CT) angiography of the abdomen.
The CT scan should be obtained without oral contrast, as the presence of contrast within the
bowel can obscure the mesenteric vessels and bowel wall enhancement.
Colonoscopy or sigmoidoscopy with biopsy is then performed to confirm the diagnosis.
This should be performed within 48 hours of symptom onset, as delayed evaluation will
decrease the biopsy yield.

Go to the next page if you knew the correct answer, or click the link image(s) below to further
research the concepts in this question (if desired).

Research Concepts:
Bowel Ischemia

We update eBooks quarterly and Apps daily based on user feedback. Please tap flag to
report any questions that need improvement.
Question 988: A 48-year-old woman is referred to the hospital from her primary care
provider’s office for the management of high blood pressure, which is unresponsive to therapy.
She has a history of neurofibromatosis type 1, though without any neurological deficits. She has
multiple café-au-lait spots on her body. She is found to have a blood pressure of 168/128 mmHg.
Her 24-hour urine metanephrines and vanillyl mandelic acid (VMA) come back elevated. Her
abdominal CT shows a 2cm adrenal mass. It is decided that the patient needs surgery; however,
she needs to have her blood pressure under control. What is the best medication for use prior to
surgery?

Choices:
1. Clonidine
2. Propranolol
3. Captopril
4. Phenoxybenzamine
Answer: 4 - Phenoxybenzamine
Explanations:
This patient likely has a pheochromocytoma. The drug of choice in this patient is
phenoxybenzamine, which is a nonspecific, irreversible alpha antagonist.
It is used before and during surgery to lower blood pressure in patients with
pheochromocytoma.
It is important to treat with an alpha-blocker prior to a beta-blocker as beta-blocker therapy
alone will lead to unopposed alpha stimulation, resulting in a hypertensive crisis similar to
that seen with cocaine intoxication.
It is also used off-label to treat thyrotoxicosis.

Go to the next page if you knew the correct answer, or click the link image(s) below to further
research the concepts in this question (if desired).

Research Concepts:
Chromaffin Cell Cancer

We update eBooks quarterly and Apps daily based on user feedback. Please tap flag to
report any questions that need improvement.
Question 989: A 30-year-old female is brought to the emergency in an altered mental state.
She had a seizure en route to the hospital. Her sister states she found her unconscious with an
empty bottle of unknown medicine. Vital signs are pulse rate 108/min, blood pressure 96/54 mm
Hg, respiratory rate 23/min, and temperature 101 F (38.3°C). Physical examination reveals hot
and dry skin. ECG shows sinus tachycardia with QT prolongation. The patient starts seizing
again. What is the next step in the management of this patient?

Choices:
1. Phenytoin
2. Haloperidol
3. Etomidate
4. Benzodiazepines
Answer: 4 - Benzodiazepines
Explanations:
Tricyclic antidepressants impose their therapeutic effects by inhibiting presynaptic reuptake
of norepinephrine and serotonin in the central nervous system (CNS). This effect in the
CNS can cause seizures. Seizures secondary to tricyclic antidepressant toxicity are usually
self-limited.
Seizures usually respond to benzodiazepines, but in cases of refractory seizures, prompt
administration of anticonvulsants, such as phenobarbital or propofol, or even general
anesthesia, should be considered.
The seizure activity is secondary to blockage of sodium channels.
As with all cases of poisoning, proper management of airway, breathing, and circulation is
critical in cases of TCA poisoning.

Go to the next page if you knew the correct answer, or click the link image(s) below to further
research the concepts in this question (if desired).

Research Concepts:
Tricyclic Antidepressant Toxicity

We update eBooks quarterly and Apps daily based on user feedback. Please tap flag to
report any questions that need improvement.
Question 990: A 70-year-old woman with a past medical history of diabetes mellitus,
poorly controlled hypertension, and alcohol use disorder presents to the emergency department
with 2 days of worsening abdominal pain, intractable nausea, and vomiting for 24 hours
progressing to coffee ground vomitus. The abdominal pain is described as sharp, epigastric, and
non-radiating. It is worsened by oral intake and does not appear to have any alleviating factors.
On examination, the patient has epigastric tenderness and a normal digital rectal exam. Vital
signs show a heart rate of 110/min and blood pressure of 90/65 mmHg. An initial set of
investigations is shown below.
Patient value Reference range
Hemoglobin 8 g/dL 13.2-17.5 g/dL
4100-
WBC count 16000/microL
10900/microL
Lactic acid 25.2 mg/dL 4.5-19.8 mg/dL
Serum creatinine 2.21 mg/dL 0.6-1.2 mg/dL
ALT 121 IU/L 10-35 IU/L
AST 175 IU/L 35 IU/L
ALP 64 IU/L 44-147 IU/L
Amylase 30 IU/L 30-125 IU/L
Total bilirubin 1.2 mg/dL 0.3-1.0 mg/dL
A CT of the abdomen and pelvis without contrast is unremarkable. Which of the following is the
next best step in the management of this patient?

Choices:
1. Ultrasound of upper abdomen
2. Magnetic resonance cholangiopancreatography
3. Upper gastroesophageal endoscopy
4. CT angiogram
Answer: 3 - Upper gastroesophageal endoscopy
Explanations:
Upper gastroesophageal endoscopy provides direct visualization and is the only test that can
diagnose acute esophageal necrosis.
Commonly reported co-morbidities associated with acute esophageal necrosis include
diabetes (38%), hypertension (37%), and alcohol use disorder (25%).
Upper gastrointestinal bleeding, vomiting, epigastric pain, and tenderness with
hemodynamic compromise are common presenting features for acute esophageal necrosis.
The main clinical findings, in this case, are signs of septic shock in the form of hypotension
and leukocytosis. Liver function tests do not show an obstructive picture, and amylase is
normal; hence ultrasound and MRCP are unlikely to provide the diagnosis. CT angiogram is
reserved for lower gastrointestinal bleeding and is therefore not indicated.

Go to the next page if you knew the correct answer, or click the link image(s) below to further
research the concepts in this question (if desired).

Research Concepts:
Esophageal Necrosis

We update eBooks quarterly and Apps daily based on user feedback. Please tap flag to
report any questions that need improvement.
Question 991: A 66-year-old man with a history of type 2 diabetes mellitus, hypertension,
and diabetic neuropathy is brought to the hospital with fever, chills, cough, shortness of breath
nausea and vomiting. He currently takes lisinopril, insulin glargine 15 units twice a day, and
gabapentin 600 mg three times a day. Vital signs show temperature 101 F, heart rate 110/min,
blood pressure 80/60 mmHg, and respiratory rate 24/min. Physical exam shows the patient being
extremely lethargic, tachycardic, and has left lower lobe crackles on chest auscultation. Chest x-
ray shows left lower lobe consolidation. CT head shows no acute findings. The patient is
diagnosed with pneumonia, with septic shock. Labs show WBC count 22,000/microL, creatinine
4.8 mg/dL, creatinine clearance less than 15 mL/min, potassium 3.8 mEq/L. An arterial blood
gas is within normal limits. He is started on IV fluids, antibiotics and epinephrine, and blood
cultures are drawn. Which of the following is the next best step in the management of this
patient?

Choices:
1. Discontinue gabapentin
2. Hemodialysis
3. IV sodium bicarbonate
4. IV potassium chloride
Answer: 1 - Discontinue gabapentin
Explanations:
Gabapentin is primarily excreted through the renal path, with no active metabolites.
It needs to be stopped when the patient is extremely lethargic with severe worsening of
creatinine clearance.
Dosage adjustment is necessary for patients with renal impairment.
Hemodialysis needs to be considered in usual indications for renal failure, not if the patient
is able to make urine as the patient can clear gabapentin slowly even with creatine clearance
around 15 mL/min.

Go to the next page if you knew the correct answer, or click the link image(s) below to further
research the concepts in this question (if desired).

Research Concepts:
Gabapentin

We update eBooks quarterly and Apps daily based on user feedback. Please tap flag to
report any questions that need improvement.
Question 992: A 24-year-old man presents to the hospital with vomiting, dyspnea,
palpitations, and hematochezia. His friend said that he ate some seeds while they were out
hiking. Physical examination is significant for inspiratory crackles bilaterally. His blood pressure
is measured to be 84/49 mmHg and his heart rate is 110/min. Despite aggressive management,
the patient passes away. Which of the following cellular processes is most likely affected by the
toxin responsible for the patient's demise?

Choices:
1. Transcription
2. Translation
3. Oxidative phosphorylation
4. Glycolysis
Answer: 2 - Translation
Explanations:
Pulmonary edema, gastrointestinal bleeding, vomiting, and diarrhea are signs of ricin
toxicity.
Ricin inhibits ribosomes which would prevent the translation of proteins.
Ricin has no antidote, supportive care must be given.
Ricin is found in castor beans. Chewing these beans will release ricin.

Go to the next page if you knew the correct answer, or click the link image(s) below to further
research the concepts in this question (if desired).

Research Concepts:
Castor Oil

We update eBooks quarterly and Apps daily based on user feedback. Please tap flag to
report any questions that need improvement.
Question 993: A 17-year-old male presents to the emergency department following a
syncopal episode in which he lost consciousness during football practice. He denies any injury.
He does not smoke or take alcohol or any illicit drugs. Family history reveals the sudden death of
his father at the age of 40 due to cardiac failure. Temperature is 37 C (98.6 F), the pulse is
84/min, blood pressure is 130/80 mm Hg, and respirations are 16/min. Physical examination
reveals a bifid carotid pulse with a brisk upstroke. Chest auscultation reveals a fourth heart sound
and systolic ejection murmur without carotid radiation that decreases in intensity while squatting
and increases with Valsalva. Which of the following is the best management of this patient?

Choices:
1. ECG should be done and if normal, sports activities can be resumed
2. Ambulatory Holter monitoring should be done
3. An ECG and an echocardiogram should be done and the patient should refrain from sports
pending the results
4. A head CT scan should be done
Answer: 3 - An ECG and an echocardiogram should be done and the patient should refrain
from sports pending the results

Explanations:
This patient likely has hypertrophic cardiomyopathy (HCM). HCM occurs due to the
mutations of genes encoding cardiac contractile proteins. It leads to left ventricular
hypertrophy and subsequently left ventricle outflow tract (LVOT) obstruction and diastolic
dysfunction.
In HCM, syncope is caused by inadequate cardiac output with exertion. This is due to the
hypertrophy of the left ventricle that causes obstruction of outflow (LVOT). The murmur of
hypertrophic obstructive cardiomyopathy increases with Valsalva and standing and
decreases with squatting and handgrip.
HCM can lead to acute LVOT obstruction, arrhythmia, and myocardial ischemia, especially
during exertion. All these can cause sudden cardiac death as well as heart failure.
Therefore, HCM needs to be ruled out with ECG and echocardiogram before the patient
may resume physical exercise due to the risk of sudden cardiac death. ECG alone cannot
rule out HCM. The patient should refrain from participating in sports until the test results
are available.

Go to the next page if you knew the correct answer, or click the link image(s) below to further
research the concepts in this question (if desired).

Research Concepts:
Hypertrophic Obstructive Cardiomyopathy

We update eBooks quarterly and Apps daily based on user feedback. Please tap flag to
report any questions that need improvement.
Question 994: A 64-year-old patient has been admitted to the intensive care unit after
major abdominal surgery and develops signs of shock and chest discomfort. An
electrocardiogram shows that the QRS duration is 140 ms with a heart rate of 130 beats per
minute. Which of the following is the most appropriate initial pharmacological treatment?

Choices:
1. Digoxin
2. Adenosine
3. Diltiazem
4. Lidocaine
Answer: 4 - Lidocaine
Explanations:
Lidocaine is a class IB antiarrhythmic and can be used to treat ventricular
tachydysrhythmias.
In normal cardiac myocytes, lidocaine shortens the action potential without affecting
conduction velocity, but in ischemia, it prolongs the action potential, and the conduction
velocity is reduced.
Lidocaine is not indicated for supraventricular arrhythmias.
Lidocaine may have a particular role in ventricular tachycardias that are refractory to other
treatments.

Go to the next page if you knew the correct answer, or click the link image(s) below to further
research the concepts in this question (if desired).

Research Concepts:
Lidocaine

We update eBooks quarterly and Apps daily based on user feedback. Please tap flag to
report any questions that need improvement.
Question 995: A 68-year-old man with a mechanical mitral valve and a history of atrial
fibrillation and hypertension presents to the emergency department for sudden onset slurred
speech. His current medications include warfarin and amlodipine. According to the patient, he
had a mild fever with chills over the last couple of days. On initial evaluation, his National
Institutes of Health Stroke Scale (NIHSS) score is 1. His blood pressure is 118/89 mmHg,
temperature 38.6 C, pulse rate 85 bpm, and respiratory rate 12 bpm. Physical examination shows
mild muscle weakness in the left arm but is otherwise unremarkable. Transoesophageal cardiac
echocardiography reveals a 4x4 mm mass on the atrial side of the mitral valve. Staphylococcus
aureus is found in three sets of blood cultures. What is the next step for a definitive diagnosis?

Choices:
1. CT brain with contrast
2. MRA
3. Diffusion-weighted MRI
4. Carotid doppler sonography
Answer: 3 - Diffusion-weighted MRI
Explanations:
The patient is suffering from acute ischemic stroke with infective endocarditis.
Staphylococcus aureus is the most common causative pathogen for embolism in IE.
Urgent brain imaging is mandatory in all patients with sudden neurologic deficits or acute
stroke. Noncontrast brain CT, rather than a contrast CT, is the image of choice to exclude or
confirm hemorrhage. Recent studies show MRI-DWI (MRI with diffusion-weighted
imaging) can be used as the imaging modality of choice for suspected acute ischemic or
hemorrhagic stroke in those patients without MRI contraindications.
However, a CT scan is more widely available than MRI in institutes that are not stroke
centers.
Noncontrast brain CT, rather than a contrast CT, is the image of choice to exclude or
confirm hemorrhage. MR angiography (MRA) identify vascular stenosis or occlusion.
Carotid duplex ultrasound is a noninvasive examination to evaluate extracranial
atherosclerotic disease.

Go to the next page if you knew the correct answer, or click the link image(s) below to further
research the concepts in this question (if desired).

Research Concepts:
Neurological Sequelae Of Endocarditis

We update eBooks quarterly and Apps daily based on user feedback. Please tap flag to
report any questions that need improvement.
Question 996: A 76-year-old male with a past medical history of coronary artery disease,
chronic obstructive pulmonary disease (COPD), hypertension, and type 2 diabetes mellitus was
admitted to the ICU for ventilatory support due to COPD exacerbation. He was intubated and
mechanically ventilated for hypoxemic hypercapnic respiratory failure the day prior. The patient
is sufficiently sedated with propofol and fentanyl drips, but it is noticed that he developed an
acute rise in his peak pressures on the ventilator to 30 mm Hg from 19 mm Hg. His plateau
pressure has remained unchanged at 22 mm Hg. Which one of the following is the potential
cause for the increase in his peak pressure?

Choices:
1. Pneumothorax
2. Pleural effusion
3. Mucus plug
4. Acute respiratory distress syndrome
Answer: 3 - Mucus plug
Explanations:
The peak pressure represents the resistance of the respiratory system from the ventilator
tubing down to the segmental bronchi. Any factor that affects the resistance of the tubes will
cause a change to the peak pressure. Examples including mucus plugging, bronchospasm,
patient biting down on the endotracheal tube, and blood clots.
A pleural effusion will cause plateau pressures to rise. Plateau pressures are representative
of the compliance of the respiratory system (chest wall, lung, abdomen). A decrease in the
compliance of the system will cause plateau pressures to rise.
A pneumothorax will cause plateau pressures to rise. Plateau pressures are representative of
the compliance of the respiratory system (chest wall, lung, abdomen). A decrease in the
compliance of the system will cause plateau pressures to rise.
Acute respiratory distress syndrome will cause plateau pressures to rise. Plateau pressures
are representative of the compliance of the respiratory system (chest wall, lung, abdomen).
A decrease in the compliance of the system will cause plateau pressures to rise.

Go to the next page if you knew the correct answer, or click the link image(s) below to further
research the concepts in this question (if desired).

Research Concepts:
Positive Pressure Ventilation

We update eBooks quarterly and Apps daily based on user feedback. Please tap flag to
report any questions that need improvement.
Question 997: A patient initially notices double vision after eating supper. Shortly
thereafter, she starts to experience weakness in arms and legs, difficulty with swallowing,
extreme trouble focusing her vision, and some minor breathing difficulties. 911 is called and she
is brought to the ER. The clinician notices bilateral drooping of the eyelids and dilated fixed
pupils. Her respiratory status has become more fragile. Which of the following is the antibiotic
of choice?

Choices:
1. Chloramphenicol
2. Clindamycin
3. Penicillin
4. Gentamicin
Answer: 3 - Penicillin
Explanations:
High-dose intravenous penicillin therapy is the antibiotic of choice for wound botulism.
Chloramphenicol and clindamycin are alternate choices.
Botulinum inhibits acetylcholine release, thus blocks motor neuron to muscle cell
communication. Typically, muscle weakness and paralysis begins in the face and spreads
peripherally to the upper and lower extremities. Diaphragmatic paralysis is life threatening.
Diagnosis is made by establishing the presence of botulinum toxin or bacteria in the
stomach, intestinal contents, vomit, feces or blood. Tests used include enzyme-linked
immunosorbent assays (ELISAs), electrochemiluminescent (ECL) tests, and mouse
bioassay.

Go to the next page if you knew the correct answer, or click the link image(s) below to further
research the concepts in this question (if desired).

Research Concepts:
Botulism

We update eBooks quarterly and Apps daily based on user feedback. Please tap flag to
report any questions that need improvement.
Question 998: After an injury, when do the symptoms of acute respiratory distress
syndrome usually begin?

Choices:
1. 1 hour
2. Within 24 hours
3. Within 5 days
4. 1 week
Answer: 2 - Within 24 hours
Explanations:
The symptoms of acute respiratory distress syndrome are very rapid.
Most people have some clinical and x-ray features within 24 hours.
In fact, the majority of patients will have hypoxia within 6 hours and x-ray features within
24 to 48 hours.
Initial symptoms are non-specific and include tachypnea, tachycardia, hypotension, and
altered mentation. Additional symptoms will be present depending on the precipitating
cause of the lung injury.

Go to the next page if you knew the correct answer, or click the link image(s) below to further
research the concepts in this question (if desired).

Research Concepts:
Acute Respiratory Distress Syndrome

We update eBooks quarterly and Apps daily based on user feedback. Please tap flag to
report any questions that need improvement.
Question 999: A 42-year-old man was diagnosed with acromegaly and recently underwent
transsphenoidal resection of a pituitary adenoma. What is the next most accurate step to assess if
the surgery was successful?

Choices:
1. MRI pituitary immediately post op
2. IGF-1 level immediately post-surgery
3. GH level immediately post-surgery
4. GH level after OGTT 4 months after surgery
Answer: 4 - GH level after OGTT 4 months after surgery
Explanations:
To assess for remission for acromegaly after surgery, IGF-1 level should be done. The
timing is important. Immediate post-surgery IGF-1 levels are elevated due to their long half-
life and can take at least 3 months to normalize.
In addition to checking IGF-1 levels, one can check a GH level after 75 g of glucose being
administered. Some argue that checking GH levels after a normal IGF-1 level may not add
additional information; however, up to 30% of patients can have discordant results.
A normal IGF-1 level and GH suppressed to less than 1 ng/mL after OGTT is indicative of
remission. These levels should be checked at least annually as relapse can occur many years
after remission.
Even though repeat imaging should be done after surgery, it should be done a minimum of 3
months after the surgery as it can take that long for the sterile compressed sponge and fat
packing to be resorbed and not affect image interpretation assessing for disease recurrence.

Go to the next page if you knew the correct answer, or click the link image(s) below to further
research the concepts in this question (if desired).

Research Concepts:
Acromegaly

We update eBooks quarterly and Apps daily based on user feedback. Please tap flag to
report any questions that need improvement.
Question 1000: A 65-year-old homeless woman is brought to the emergency department
with complaints of abdominal pain and decreased urination. She is afebrile, has blood pressure
180/120vmmHg, pulse rate 100/min, and respiratory rate 20/min. Physical examination is
significant for IV drug use tracings in her upper extremities and nonhealing ulcers in lower
extremities just above the medial malleolus. Her initial labs showed potassium 6.2 mEq/L,
bicarbonate 14 mEq/L, BUN/Cr 40/5.6 mg/dL, and urinalysis is significant for WBC, RBC, and
RBC casts. Further labs show a high HCV viral load, AST/ALT 80/100 IU/L, negative ANA,
normal C3, low C4, and a low titer of cryoglobulins. Which of the following is the best initial
therapy for this patient?

Choices:
1. Ribavirin and PEG- interferon-alpha
2. IV immunoglobulins
3. Corticosteroids and sofosbuvir
4. Plasmapheresis
Answer: 4 - Plasmapheresis
Explanations:
Plasmapheresis is used in life-threatening situations of mixed cryoglobulinemia.
Plasmapheresis aids in the fast removal of circulating immune complexes thereby removing
the major insult to the organs.
Renal failure is involved with increased mortality in patients with mixed cryoglobulinemia
and therefore a fast-acting treatment is required.
Ribavirin and interferon-alpha used to be the treatment strategy for hepatitis C prior to the
era of direct-acting antivirals (like sofosbuvir). They were helpful in mild cases of mixed
cryoglobulinemia previously as they helped in antigen clearance (viral clearance).
Corticosteroids are used in mild cases of mixed cryoglobulinemia but are not of much use in
serious complications such as renal failure. IV immunoglobulins are contraindicated in
mixed cryoglobulinemia as they cause precipitation of immune complexes leading to
multiorgan failure (PMID- 27870771). Antiviral agents take several doses to show effect
and hence not useful in our situation. Antiviral therapy needs to be started in this patient
after plasmapheresis when she is in a more stable situation.

Go to the next page if you knew the correct answer, or click the link image(s) below to further
research the concepts in this question (if desired).

Research Concepts:
Cryoglobulinemic Vasculitis

We update eBooks quarterly and Apps daily based on user feedback. Please tap flag to
report any questions that need improvement.

You might also like